You are on page 1of 359

.

F:j

1) La efectividad corresponde a: . a) La cap.acidad de una intervencin ~e lograr resultados favorables medidos en aos de vida ganados, stados por calidad La capacidad de una intervencin de lograr resultados favorables a menor costo e) La capacidad de una intervencin de lograr resultados favorables en condiciones ideales, controladas d) La capacidad de una intervencin de lograr resultados favorables en condiciones similares a las reales e) La capacidad de una intervencin de lograr resultados favorables cuando otras intervenciones han fallado previamente 2) Un paciente de 32 aos est convencido de que su mujer lo engaa con un amante. Ella lo niega . rotundamente y dice que su esposo est volvindose loco con ese tema. Por esto, l la sigue e incluso ha contratado un detective para que la e!;pfe, sin lograr ninguna Informacin. A pesar de esto l insiste en el engao de su mujer, ya que ha visto manchas en las sbanas de la casa, que con toda seguridad son producto de la infidelidad. No ha presentado alucinaciones y el resto de su examen psiquitrico es normal, al igual que el examen neurolgico. El diagnstico ms probable es: a) Esquizofrenia b) Depresin psictica e) Trastoroo.-ebsesivo.. cor.npulsivo ~(Trastorno delirante crnico t) Trastorne-l:>ipol&J 3) Cul de las siguientes afirmaciones es FALSA? , 5) A los pacientes pertenecientes al tramo 13 de FONASA no se les exige copago ..j . e) Los pacientes pertenecientes al tramo A de FONASA reciben atencin gratuita d) Los pacientes del tramo 8 de FONASA no pueden optar al sistema de libre eleccin / e) A los pacientes del tramo C de FONASA se les exige copago
~,Los pacientes pertenecientes al-tramo .Qde-FONASA pueden optar al sistema de libre eleccin

yr

4) Las inmunizaciones o vacunas son m-edidas de: a) Proteccin de salud i(Prevencin primaria .. 'e) Prevencin de daos d) Prevencin secundaria e) Prevencin terciaria 5) Una mujer de 54 aos dej de ir al trabajo por miedo a que la asaltaran en el camino o que sufriera un accidente de trnsito. Cada vez sale menos de la casa y mantiene todo con llave, porque teme que puedan entrar a robar a la casa. Esto le Lla producido una menor Interaccin con sus familiares y amigos. El examen fsico es normal. El diagnstico ms probable es: a) Agorafobia b) Trastorno de pnico e) Depresin mayor gfTrastorno de ansiedad generalizada ~)Trastorno de personalidad paranoide 6) Se evalu la utilidad de una nueva prueba de sangre oculta en deposiciones, como pesquisa de cncer de colon . Para esto se realizan tomas a 200 pacientes con sospecha clfnica de cncer de colon se com ar con el old estndar: la colonosco la, obtenindose los si uientes resultados: Colonoscopa (+) Colonos~opia (-) Prueba(+) Prueba (-) El valor predictivo negativo de la nueva pr!Jeba es: a) 33/40 33

- b) 143/160
e) 3~/50

/~) 7/143

9p143/150

---,..

'

\~

::-.

. 7) En una poblacin de 3.000 personas hay 1.000 que fuman. Si3 sabe que la tasa de Incidencia de cncer de vejiga, a lo largo _del tiempo es de 5% en los pacientes no fumadores y de 1O% en los pacientes fumadores. Si ninguno de los pacientes de la poblacin fumara, Qu porcentaje de los casos totales se prevendra? a)25% b) 33% e) 50% d}66% e)75% 8) Una paciente de 23 aos corta el gas y la luz de la casa antes de salir, porque si no 'lo hace tiene la sensacin de que puede explotar. En algunas ocasiones se ha regres~~o de la playa a cortar el gas y la luz, ya que la persona que debra hacerlo, lo olvid. Ella sabe que es exagerado y ridfculo, pero no p~erlo le causa ~ucha ansi~dad y malestar. El diagnstico ms probable es: .~;;,atlrastorno obsestvo compulstvo ~/b) Trastorno de ansiedad generalizada e) Trastorno .de estrs postraumtico d) Trastorno delirante crnico e) Trastorno adaptativo 9) A qu tipo de estudio corresponde el siguiente enunciado? Se identifica un grupo de 200 pacientes diagnosticados de cncer de estmago y se aleatorizan a recibir tratamiento quirrgico exclslvo o tratamiento quirrgico, asociado a quimioterapia postoperatoria.Se comparan las tasas de mortalidad y recafdas. a) Casos y controles b) Cohprtes e). Trn.sversal d) Ensayo clinico controlado ,e) Ensayo ~e campo 1O) La demencia frontotemporal se caracteriza fundamentalmente por: a) Slntomas extrpiramidales b) Alucinaciones visuales e) Caldas frecuentes d} Afasia de expresin

1 [?)~sinhibicin

11) Cul de los siguientes es un efecto adverso de los antidepresivos.tricfclicos? a) Hipertensin arterial b) Oi~rrea _9.)-aja de peso /d} Glaucoma e) Sialorrea 12) Qu entidad se encarga de administrar los recursos econmicos del sistema de salud pblico Chileno? a} FONASA b)CENABAST ~) SeJVicio de salud d) Consultorio e} Seremi 13) Una paciente de 33 aos presenta un cuadro de 4 semanas de evolucin de tristeza e insomnio. No identifica una caL:~sante clara, pero tiene baja autoestima y se ':onslder~ Intil y no querida. Le cuesta mucho concentrarse en su trabajo y no se junta con amigos, ni reali~a actividades recreativas, ya que no le hacen sentir mejor. La conducta ms adecuada es: a} Solicitar un TAC de cerebro y un ECG y decidir conducta segn "'allazgos ,;t(Solicitar ex~menes generales e iniciar un antldepresivo IRSS . ' e} Iniciar un antlconvulsivante d) Iniciar antipsicticos y antidepreslvos e) .lnicjar psicoterapia. sin necesidad de frmacos, con controles mensuales

14) Cmo se llama a aquel trastorno caracterizado por fingir una patologa, bajo control voluntario, con el fin de ganar atencin y cuidados? a) Disociativo b) Conversivo y fF'aclicio / d) Sornatomorfo e) Simulacin 15) Si se quiere descartar una patologa E;e requiere un test con: a) Validez interna - - bj.Aiia sensibilidad /c) Alto valor predictivo positivo d) Bajo precio y fcil de implementar e) Alta especificidad
16) Cul de las siguientes asociaciones entre patologa y clnica es incorrecta? a) Disquinesia tarda - Movimientos reptantes de la lengua b) Balismo- Movimientos bruscos de los grupos musculares proximales e) Acatisia -Malestar al pe~manc(r quieto, que se alivia con cambios frecuentes de posicin d) Distona- Disminucin del tono muscular y reflejos osteotendneos e) Parknsonsmo- Temblor de reposo

'

17) Usted desea comparar los niveles pl1smticos de vitamina 812, en relacin al hematocrito, ambos como variables continuas, en una pobla<:in de 200 pacientes. La mejor prueba para determinar la significancia estadstica es: a) La "t" de Student b) El anlisis de varianza e) El anlisis estratificado d) La "r" de Pearson e) El Chi cuadrado
18) Una mujer de 23 aos presenta relaciones interpersonales muy Intensas e inestables . Adems es muy impulsiva en las compras y tambin en el consumo de algunas sustancias. Sus familiares refieren que responde con Ira y agresividad cuando se siente agredida, lo que suele ser muy frecuente. El diagnstico ms probable es: a) Trastorno bipolar b) Trastorno de personalidad paranoide p}.Trastorno de personalidad limtrofe d) Trastorno de adaptacin e) Trastorno esquizoafectivo

19) Usted cuenta con la siguiente informacin: Poblacin total: 1.000 habitantes, muertes totales: 14, muertes por influenza: 2; casos de influenza: 140 Calcule la tasa de letalidad de la influema a) 140/1 .000 -...._ - .. b)-14/1.000 ;;>211 .ooo ~,9/21'140 e) 2/'14
20) Un paciente consume una sustanci;, presentando palpitaciones, mareos y temblor. Al examen fsico est plido, sudoroso, con taquicardia e hipertensin importante, con midriasis y fiebre hasta 38oc, sin presentar signos focales. Qu sustancia es ms probable de ser la causante de este cuadro? ~)"" Rivastigmina o) Alcohol cv~prnerol .g) cocana "e) lmipramina

,.,,,

4J..

RUT

Al\

,oo
NOMBRE:

ooo
00

2
3
4

00 00 000 000
000

o ooo ooo o
000 .000
1

000 000

t:J1!!J.
4\
~
~
~

RUT:

00 000 00 000 00 000 00 000 .

7
8

00 000

FECHA:

o 000 o 000 o 000 o 000 o 000 o oo.o o. o


.

~~

CURSO:
.R E S P U E STA S

4-.
~

E
31

ABCOE

,;a.
/'11111!~

, 00<)00
2
3 4

S
6

7
B 9

10

00000 00000 00000 00()00 00000 00000 00000 00000


00000 00000 00000

00000

32
33 34

35
36

37
lB 39

~o
41

00000 61 00000 00000 6~ 00000 00000 63 00000 00000 64 00000 00000 65 00000 00000 66 00000 00000 67 00000 00.000 . 68 00000 00000 69 00000 00000 70 00000 00000 00000 00000
71

""""
~

~.
~

~
~

. 'r.

,....
~

11

..

',

12
13

~2
43

7.2
73

14
15

ooooo, ooooo
00000
" 45 4

00.000 00000 00000

74
75

16

17
18

19 20

00000 0000.0 00000 00000


00000

46

47
48

00000.

00000 00000 00000

76
n
78

49

so
51

00000 00000 00000 00000 00000.


00000 OOQOO 000.00 00000 00000

79 80

00000 00000 00000 00000 00000 00000 00000 00000 00000

ooooo

~
~
~

~
~-

/Wf!<.,
~

21
22 23

00000
00000 00000 00000 00000 00000 00000 00000 00000

81

~ ~

00000

52
53
54

82
83
84

4\.
All\

24
25

55
56

85
86

26

27
.

57
58
59

87
88
89

~8
29 30

60

00000 00000

90

00000 00000 00000 00000

00000 00000 00000

~ ~
~

~
~
~

4'1

.
.

LA CAUSA MAS

f'\.

IN.FECTOl.Gft~

1. TBC {localizacin) f....)\ .r-~ _ 2. Meningitis en RN -~ ...t \ " 7 -~lo~ ..;\

FRECUE~TE

~=:

,\-4':)..C:....:. .. , l. .. s\
; ,

'\... .:.

Primera parte

3. Meningitis Bacteriana en nios ~-t.-.Jf'-c..<--'='~.,._-::;; 4. Me~ingitls Bacte;i_a~ en adultos \_;-t-..-l ~--c:......<.t -~..:.

~-

..... - : .

S. Meningitis Bacteriana en adultos mayoreS\ c.i.. ...~><\t....: - (, ... 6. Meningitis en TEC abierto o postneuroquirrgica J. ....., .......} J 7. Encefalitis viral . "- (t..--~ -~ f"-l 1

8. Absceso cerebral
9. NAC en RN .r ~
~

Dr. Guillermo Guevarc Aliaga

10. NAC en nios r-'-'...; .........-.<.. ....:.:.~;.._ e ..... ~.

,...

LA CAUSA MAS
11. NACen adultos ~v -.-o.co c ...., ' Q.. ... ' 12. NAC atpica rv ... '- .:::. r? t (_. -t'--.. . . t" -.... 13. NAC aspirativa 14. NACen inmunodeprimidos fi'-J 1'-co'-.: c. 15. Neumonra intrahospitalaria 16.Empiema 17. Derrame pleural P'-"---t--~ ... -......~...; ,.., 18. Absceso pulmonar 19. Celulitis J e;'- .v ~-

LA CAUSA MAS
~

FRECUENTE

21. Sinusitis crnica 22. Laringitis aguda


1.!"-

FRECUENTE
f (_,_._. f-C~ .f~ --.

.;<_

23. Amigdalitis aguda S IC ,:,) 24. Resfrro ...._.:. ~~...>{ 25. Bronquio litis .il ~l26.SBOR 27. Celulitis preseptal P----<:... .. --...:..<.. .!-..~-=

.Q...-.~( "-: .

r
\ ,J\'r...:. (

28. Celulitis orbitaria 29. Erisipela ~ y; A


30. Linfangitts -~~ .r.:) .

20. Sinusitis aguda ,.... ~...... .--"C .. ,:.c..---::


~.--: \.-.

LA CAUSA MAS
31.1mptigo .1. ?'J.-...~ .1 .f.Lc:-a,.J" .. 32.1mptigo buloso J ..-.... .....;.-~~:.r 33. Ectima
~-(.,. ~-r

FRECUENTE

34. Ectima gangrenoso ~ Q..~ ....1... <:-- -<!7..--. 35. Foliculitis .r '\t~ ., .r r. '-r ~ / 36. Foliculitis del sauna 37. Furnculo 38. Antrax o carbunclo cutneo . 39. Sndrome de piel escaldada J'. e.'-'->-.40. Shock txico (. ' _-., ,-...
j

41. Fasceftis necrotisante ~ (!.' &. 42. Miositis bacteriana 43. Atritis sptica S -u -...A~- r 44. Artritis sptica en adolescentes cj:':. -- -=-c-.:-J- -~ ; 45. Osteomielitis aguda ~ . .._. . .,..---lt~ 46. Oste~mi~l~tis crnica : _ l. e .:. 47. Pie d1abet1co ,...._.._.,,L )" fu
J

LA CAUSA MAS FRECUENTE

J-o:. -<~----~

48. Tia capitis 49. Tiiia corporis ~r. fJ.,. G 1:-:. .-J
SO. Onicomicosls

,.,. -,

CASOS CLNICOS
lOO.Paciente con ICC por estenosis mltral, presenta fiebre alta y gran compromiso del estado general, de 2 dfas de evolucin. Al-examen RR2T con soplo sistlico y diastlko. Hemoc~ltivos posftivqs para ... .A-...... ~ S. aureus e.~&.t"j v(V"- 0J. . A- ~ P;, a~....... . 101.Pacien~e de 33 aos, con cuadro de fiebre y , 1, malestar de 1 semana de evolucin. Al examen soplo sistlico y 3 petequias en ~ID .C.,.-b-t..'c.f:.'- .... 102.Mujer hospitalizada por ICC, manejada con diurticos y va~odilatadores endovenosos,con buena evolucin inicial. Presenta fiebre, sin foco claro. Se controlan hemocultivos que resultan positivos para S. coagulasa negativa sensible

CASOS CLNICOS
103.Paciente de 4 aos, con fiebre y dolor cervical. Al examen se ob ses:va adenopatf~d~ ~,5 ~~.d~loJ_ro~~ '- '~ j ~--- ,.... ... en relaci n a 1musculo ECM ~ ~ -\ \.- ~' 104.P,..aGiEt_nte en QT por leucemia. Presenta neutrfilos .. ...-. -~~~asociados a fiebre hasta 38,8C "'-"-'-~--~~..y'(:-~.~ ) ...~ ~ lOS.Paciente con promiscuidad sexual consulta por :.:\=i .1.-i.{./l. ~ odinofagia1 fiebre y un exantema mac~lar rosado . ( --~:. ~ generalizado. Al examen slo destaca'adenopatas 1.~ ! cervicales bilaterales y faringe eritematosa :..) ( ~ 106.VIH con cefalea~ sognos focales y TAC con lesiones que -captan contraste en anillo

d ,. __

.l

'J?= ~, v~-<.f\.-c:.

...

. 1

.,..

~-

i.. (, l. .> _). __ '---:.

CASOS CLNICOS

CASOS CLNICOS

107.VIH consulta por cefalea y compromiso de 113.Hombre viene llegando de Ecuador. Presenta fiebre alta y mialgias generalizadas, con algunas conciencia. Signos menngeos esbozados. LCR con escasas clulas de predominio mononuclear .. petequias en extremidades superiores. Al tomar la presin aparecen petequias en la zona de presi~n 108.VIH con prdida importante de la agudeza visual del manguito. t"'-e-~ d..... ...:.:: ~-mayor a Izquierda, de una semana de evoluci' 114.Nio de 45 aos, febril, de aspecto txico. Al 109.VIH con lesin cutnea de crecimiento en 3 examen destaca edema.palpebral bilateral y meses. Se observa ndulo de 2 cm, violco rinQrrea p~m:l.l~~~~ -~bund~o~e. 110.VIH con diarrea acuosa, prolongada(,~,...:_~~ J '-'-0 {..e; f 111.VIH con disfagia. Al examen se aprecia algorra r,C,.....u _ abundante en toda la cavidad oral y faringe c.: t,_J--. 112.VIH con disfagia. EDA muestra lceras esofgicas
).!.>

j;._.,\'-:..-. .

l{t.:. -~-:r.:.

ce
\~..(.} ~-:-.. \.;--40~
(..._,_- .:-..-tt_cl. .... .:_\

- L... .:_~ . .e (:.rct.._L'-'\...ll_t


,.. \' ~- <..--'\0 i/ \ 1

51. Escaras Infectadas (agentes) 52. Eritema infeccioso 53. Exantema sbito 54. Escarlatina 55. Sd. De pie mano boca 56. Complicaciones de varicela en nios 57. Complicaciones de varicela en adultos 58.1TU baja 59.1TU alta 60. Prostatiti$ aguda

LA CAUSA MAS FRECUENTE

61. PIP (salpingitis, endometritis) 62. Vaglnosis bacteriana 63.leucorrea 64.ETS 65. Uretritis (3 causas) 66. Unfogranuloma venreo 67. Condiloma acuminado 68.Chancro 69. Aftas recurrentes 70. Mononucleosis Infecciosa

LA CAUSA MAS FRECUENTE

71. Sndrome mononuclesico (otras 5 causas) 72. Hepatitis aguda viral 73. Diarrea aguda 74. Disenteria 75. Diarrea por antibiticos 76. Diarrea crnica 77. Fibre tifoidea 78. Araazo de gato 79. Abseceso heptico 80. Sd de Loeffer

LA CAUSA MAS FRECUENTE

81. larva migrante visceral 82. Neurociticercosis 83. Hidatidosis 84. Quiste hidat(dico {sitio anatmico) Tb 2da causa 85. Distomatosis 86. Enfermedad de Chagas 87. Candidiasis (sitio anatmico) 88. Infeccin intraamnltlca 89. Corioamnionitis 90. Infecciones congnitas (TORCH)

LA CAUSA MAS FRECUENTE

LA CAUSA MAS FRECUENTE 91. Sepsls neo natal


92. Endocarditis aguda 93. Endocarditis subaguda 94. Sepsls asociada a catter intravascular 95. Absceso y flegmn de cuello y piso de la boca 96. Adenitis supurada 97.Adenitis supurada, asociada a enfermedad
perio~ontaJ

LA CAUSA MAS
lOl.Primoinfeccin VIH (manifestacin) 102.Convulslones en VIH 103.Leucomalada ~ultifocal progresiva 104.Lfnfoma cerebral primario lOS.Corlorretinitis en VIH 106.Sarcoma de Kaposi 107 .Cncer de Cuello uterino 108.Diarrea crnica en VIH (tb 2da.y 3ra} 109.CompromisosNC en VIH (5 causas) llO.Esofagitis en VIH (3 causas)

FRECUENTE

98. Neutropenia febril (agente etiolgico) 99. Fiebre de origen desconocido lOO.Minife~taciones de VIH

~.

'

111.Malaria (agente y vector) 112:oengue (agente y vector) 113.Fiebre amarilla

LA CAUSA MAS FRECUENTE

EXAMEN(~s) MS

IMPORTANTE
1. [tg TBC 'pulmonar 2. Dg TBC pleural (estudio de derrame pleural mononuclear) 3. Meningitis 4. Encefalitis herptica S. Absceso cerebraJ 6. Orientan a Hanta 7. Dg de Hanta
8. NAC

9. NAC por Pneumocystls carinii

EXAMEN(es) MS IMPORTANTE
10. NAC en inmunodeprlmidos 11. Neumonfa intrahospitalaria 12. Derrame pleural paraneumnico 13. Absceso pulmonar 14. Celulitis 15. Diferenciar celulitis prese.ptal de orbitaria 16. Evaluar gravedad de celulitis preseptal u orbitaria 17. Erisipela, Linfangitis, Imptigo, Imptigo bu loso 18. Diferenciar Ectima, Ectlma gangrenoso, Foliculitis, Furnculo

EXAMEN(es) MS IMPORTANTE
19. Antrax o Carbunclo cutneo . 20. D~t Sindrome de piel escaldada 21. Shock txico , 22. Sospecha de Fascertis necrotisante 23. Sospecha de Atritis sptl_ca 24. Os.teomlelltis primera imagen 25. Osteomielitis mejor Imagen 26. Evaluar conducta ante Pie diabtico 27.DgTia 28. Dg Sarampin

EXAMEN(es) MS IMPORTANTE
29. Dg Eritema infeccioso (PVB19) 30. Dg Exantema s~bito 31. Dg Escarlatina 32. Dg Sd. De pie mano boca 33. Dg Rubeola . 34. Dg Parotlditis 35. Dg Varicela 36. Dg Herpes zster 37.DgiTU 3.8. Dg Prostatitis aguda

EXAME.N(es) fVIS IMPORTANTE


39. Dg PIP (salpingitis, endometritis) . 40.sospecha de ATO 41. So!;pecha de ATO roto 42. Diferenclar entre las distintas vaglnosis 43. Dg Uretritis gonoccica en hombres 44. Dg Uretritis gonoccica en mujeres 4S . Sospecha de sifllis 46. Confirmar s(filis 47. Sospecha de Neurosfilis 48. Dg Herpes genital

EXAMEN(es) MS I-MPORTANTE
49. Dg Primoinfeccin herptica (ginglvoestomatitis) SO. Dg Encefalitis herptica 51. Imagen en sospecha de encefalitis herptica 52. Dg Sfndrome mononuclesico por CMV 53. Dg Sindrome mononuclesico por toxoplasma 54. Dg Mononucleosis infecciosa 55. Dg Hepatitis A 56. Dg Hepatitis B aguda 57. Dg hepatitis B cronica 58. Sospecha Hepatitis C

EXAMEN(es) MS IMPORTANTE
59. Confirmadn Hepatitis C 60. Evaluar etiologfa de Diarrea por antibiticos 61. Dg Peritonitis bacteriana espontnea 62. Og Fibre tifoidea 63. Og Brucelosis 64. Og leptospirosis 65. g Bartonelosis (Araazo de gato) 66. Dg Amebiasls Intestinal 67. Dg de parasitosis intestinal 68. Mejor imagen para Neurociticercosis

Jo.

J:o.

::'.

""

F.,

EXAMEN(es) MS IMPORTANTE
69. Dg Hidatidosis pulmonar 70. Dg Hidatidosis heptica 71. Dg Enfermedad de Chagas (TripasonomJasis) 72. Sospecha de Aspergilosis invasora 73. Dg de micosis invasora (mucor y aspergilus) 74.1nfecci6n lntraamnitica 75. Corioamnionitis 76. Endocarditis aguda 77. Endocarditis subaguda 78. Sepsis asociada a catter intravascular

EXAMEN(es) MS IMPORTANTE
79. Absceso y flegmn de cuello y piso de la boca 80. Dg Adenitis supurada 81.1magen para evaluar infeccin grave de cuello 82. Screening VIH 83. Confirmar VIH 84. Dg Toxoplasmosis cerebral 85. Dg Leucomalacia multifocal progrsiva 86. Dg Meningitis por criptococo 87. Coriorretinitis por CMV 88. Dg Sarcoma de Kaposi

EXAMEN(es) MS IMPORTANTE
89. Dg Criptosporidiasis 90. Dg lsosporiasis 91. Microsporidiasis 92. Evaluar Disfagia en VIH 93. Sospecha de Malaria 94. Evaluar contactos de VHB 95. Evaluar contactos de VIH 96. Evaluar contactos de TBC bacilfera

TRATAMIENTO
1. TBC pulmonar 2. TBC pleural 3 .. TBC menngea 4. Meningitis Bacteriana en RN S. M_eningitis Bacteriana en nios 6. Meningitis Bacteriana en adultos 7. Meningitis Bacteriana en adultos mayores 8. Meningitis bacteriana en embarazo 9. Meningoencefalitis viral 10. Encefalitis herptica

TRATAMIENTO
11. Absceso cerebral 12.Hanta 13.NACATS 1 14. NAC ATS 11 15. NAC ATS 111 16. NAC ATS IV 17. NAC atfpica 18. NAC aspirativa 19. NACen inmunodeprlmldos 20. NAC por Pneumocystis carinii

TRATAMIENTO
21. Neumonfa intrahospitalaria 22. Empiema 23. Derrame pleural paran~umnico simple 24. Absceso pu'lmonar 25. !lulltis preseptal 26. Cf!lulitis orbitaria 27. Ct!lulltts 28. Erisipela .29. Liofangltis 30. Imptigo

TRATAMIENTO
3l.lmptigo buloso 32.Ectima 33. Ectima gangrenoso 34. Foliculitis 35. Foliculitis del sauna 36. Furnculo 37. Antrax o Carbunclo cutneo 38. srndrome de piel escaldada 39. Shock txico estreptoccico 40. Shock txico estafiloccico

TRATAMIENTO
41. Fasceftls necrotisante 42. Miosltis sptica . 43. Atritis sptica 44. O~teomielltis aguda 45. Osteomielitis crnica 46. Pin diabtico 47.1nfecciones por SAMR 48. Amigdalitis pultcea 49. Amigdalitis pultcea en alrgicos SO. Escarlatina farngea

...

TRATAMIENTO
51. Escarlatina quirrgica 52. Herpes zster 53. Neuralgia postherptica 54. Tl capitis . 55. Tia corporis 56. Onicomicosis 57. Escaras simples Escaras Infectadas . 59. Sarampin 60. Eritema infeccioso (PVB19)

TRATAMIENTO
.61. Exmtema sbito 62. Sd. De pie mano boca 63. Rulbeola 64. Parotiditis 65. Varicela 66. Herpes zster 67. Varicela en Embarazo 68.1TU' baja 69.1TU alta 70.1TU alta en urolitiasfs

sa.

TRATAMIENTO
71. Bacteriuria asintomtica 72. Prostatitis aguda 73. PIP {salpingitis, endometritis) 74.ATO 75.ATO roto 76. Vaginosis bacteriana 77. Tricomoniasfs vaginal 78. Cndldiasis vaginal 79. ETS (prevencin} 80. Uretritis gonocdca

TRATAMIENTO
81. Gonorrea en embarazo 82. Uretritis no gonoccica 83. Linfogranuloma venreo 84. Condiloma acumlnado 85. Sffilis primaria 86. Sffilts secundaria 87. S(fills terciaria 88. NeurosffiJis 8~. Herpes genital 90. Primoinfeccin herptica (gingivoestomatitis)

TRATAMIENTO
91. Sfndrome mononuclesico por CMV 92. Sfndrome mononuclestco por toxoplasma ."93. Mononucleosis Infecciosa 94. Hepatitis A 95. Hepatitis B 96. Hepatitis C 97. Diarrea aguda 98. Dlsenterfa aFuda 99. Diarrea por 'clostrldium dificil e 100.Diarre por clostridium dificile que no responde

TRATAMIENTO
101.Diarrea por clostridium que no responde denuevo 102.Peritonitis bacteriana espontnea 103.Abdomen agudo 104.Fibre tifoidea 10S.Brucelosls 106.leptospirosis l07 .Bartonelosis (Araazo de gato) 108.Ameblasis intestinal 109.Abseceso heptico amebiano 110.Ascariasis

TRATAMIENTO
111.Triquinosis 112.larva migrante visceral 113.Teniasis: niclosamida y praziquantel 114.Neurociticercosis 11S.Qulste hid~tfdico 116.Distomatosis
117.~~1fer.medad

TRATAMIENTO
121.Candidiasis orofarfngea 122.Aspergilosis 123.Mcormicosis 124.Rabia 12S.Ttanos 126.1nfeccin intraamnitica 127.Corioamnionltis 128.Sepsis neonatal 129.Endocarditls aguda 130.Endocarditis subagud~

de Chagas (Tripasonomlasis): . Benznidazol, Nifurtlmox 118.Pediculosis 119.Sarna 120.Candidlasis cutnea

r '-

TRATAMIENTO
131.Sepsls asociada a catter intravascular 132.Absceso de cuello y piso de la boca 133.Fiegmn de cuello y piso de la boca 134.Adenitis supurada 135.Neutropenia febril 136.VIH 137.VIH en el embarazo , 138.Toxoplasmosis cerebral 139.Leucomalacia multifocal progresiva 140.linfoma cerebarl pmao

TRATAMIENTo
l41.tlemencia subaguda por VIH 142.1\Jleningitl$ por crlptococo 143.Corlorretlnitls por CMV 144.Sarcoma de Kaposi 145.Cptosporldiasis 146.1sosporlasis 147.Microsporidiasls 148.E:sofagitis por cndida 149.E:sofagftis por CMV 1SO.Malaria

TRATAMIENTO
151.Dengue 152.Fiebre amarilla 153.Profilaxis en contactos de Meningitis por Hib 154.Profilaxis en contactos de meningitis por meningococo 155.Profilaxis en contactos de VHB 156.Profilaxis en contactos de VIH 157.Profilaxis en contactos de pacientes de alto riesgo deVIH 158.Profilaxis en contactos de TBC bacilrfera

INFECTOLOGA Segu.nda parte

. Dr. Guillermo Guevara Aliaga

8. Mf!nlngitis con coccea gram positiva 1\Jt 0 ~~i_tc e 1. Paciente con fiebre y baja de peso, de 1 mes, ~-'!'!\ . .asociado a expectoracin hemoptolca ~ 9. Mtmingitis con coccea gram negativa v--:_(,. '""'~/;_ GC:(c 2. Baciloscopfas mensuales 1(+), 2(+), 3(+), 4(+)fo..4~ ti 0 10. Mnningitis con bacilo gram positivo L{,.) . H. r'~(..._.. ~ 0'-"..;."'-f' ... , .r 3. Baciloscopfas mensuales 1(+}, 2(+), 3(-), 4(-j, 5(+) ' 11. MEmingitis con bacilo gra~ ne~ativo ~ -Cl'-Li j ~ \: l:-~ ~ ( 4. Baclloscopfas mensuales 1(+), 2(-), 3(-), 4(+), 5(+)~:} L-11 ~r~ . 12. Ni\o con cefalea y fiebre, signo~ menfngeos , .. ..:. : ~} .-, 5. Paciente con fiebre, compromiso de conciencia, dudosos. Estudio de LCR: 10~ ce lulas, 80% MN, f\.1\. '-.~ ~l protenas elevadas discretamente, glucosa. normal ' ' ..... ~ signos menngeos y p~rlisis..de algunos nervios f-_Cll]omotor@S. ~'-i.. "'N.\ t~..... ~ \\t{ 13. Adolescente con cefalea, vmitos, fiebre y luego 6. Clfnlca de meningitis. Estudio de LCR: 1000 clulas, . . . "cc1nducta e}Straa~~~~ algunD_h_~.r~~ -~~.~u~'!~!~~- y. 80% MN, protenas altas, glucosa baja r.N.r....:.-."'1 l+ \ \ ~ A...- fi_~:a~~~~~~!!_~ls!~~-' . . . . .-t"-.-,l\_.~tl....l~ti-... i \-\.!.. \~, J \ '.{7. Cllnica de meningitis. Estudio de.LCR: turbio 1300 clulas, 20~. MN, protenas altas, gluc~-~:!.. )\....\. '- '\;.~,:~..,,.a,-,.......,,.........__,_ _ _ _ _ _ _ _ _ _ _ ,........_" _____, 4 --;\ .. \ . \ ~~ Ir" "":.. .6- -v.... \ ~--- _ r. , ':k .e u-::- .L.-~~ 1 "\.:.: ,. . ' ' \\ - lll.L \ 1 , ' -,:.!~ ,.. (\. t l,k _':~...!J / :J.", ~

CASOS CLNICOS

CASOS CLNICOS

.{

'

.l

(.

'"-

1"

-:.\

'.

{~, ~

(~ ..

-~

\
-" "\'

''l.tl {f.

})...{./- ~~(_,.\,~'_,N
,.

t'

".;\ (

i
\

-~ ._._..' ' \\

1
'

'

CASOS CLNICOS
14. Paciente de 45 aos, cursando cuadro respil".atorio alto que evoluciona a distress respiratorio. El hemograma muestra Hcto:~~j6 Blancos:_~~-~~~~L Plaquetast.zJhO \) ~- f.J -~ \~U. C"'l~15. Hombre 69 aos, con diabetes, con tos, fiebre y expectoracin abundante, asociado a confusin mental. Satura 91% a Fi02 ambiental ~"\..,. ~~ 16. Estudiante de medicina, previamente sano, c:on CEG, tos y expectoracin mucopurulenta. Al examen crepitaciones en base izquierda. En buenas condiciones, eupnelco.

CASOS CLNICOS
17. Mujer de 70 aos, hipertensa, con clinlca de neumonia, con FR:19x;, FC:97)<', t 0 :38,2C, ,_ PA:140/88, Satura 89% a Fi02 ambiental y 95% con oxigeno N t-"\C ~-,r. 18. Paciente de 4S aos, diabtico mal controlado con fiebre, tos proc!uctiva, disnea de reposo y finalmente compromiso de concle~~ta. Al examen { :- t crpltos en b'ase de"recha y camp-pulmonar Izquierdo. FR:a!ic~, saturacin 02: ~_!~%a Fi02:50% 19. Nio de 8 aos, con cuadro caracterizado por tos con expectoracin y rinorrea de 8 dias de evolucin. En los ltimos 2 dias ha incrementado la tos, agregndose ex~ectoracin m~copurulenta y

. r:-\--

\ ~;

11"\J ':\... i

'-

l~\

, ..- .

. ,,\ f-.J"} ' \

1
~ fj
i,

J )~r:.~,

(~ Paden~~~~o~ V~~!~!~~~ta ~r
20.

CASOS CLNICOS
:r

disnea, tos y expectoracin. Al examen: MP(+), --- crpitos finos escaso en ambas bases. Sat02: 90%. R~Tx: Infiltrado intersticial bilateral. N t:-' 1-~ ~ "-.: i 21. Paciente operado de apendicitis, evoluciona c:on tos y expectoracin mucopurulenta, a la QJ!e-lu.qo se agrega disnea. RxTx: condensacin LID~~\~::-22. Exudado pleural con ~~~_t~N, ADA: 52 \'"\~c.

25. Obesa de SS aos, con. micQ.S.ls. pJ~O_!,!Jr. Consulta por aumento de v~lumen y eritema en pierna derecha. Al examen se aprecia placa eritematosa, ~ \:t 'r--o... d~.s~~~, indurada y dolorosa a la palpacin, de L..\_' . 1-<"~ bo;des difrciles de definir. t:~J-d.J..JJ-...;;;.:.._ ~u_( ... te\.. .._ e., :~t. 26. Nio con sinusitis, evoluciona con eritema ):: 1(\A. periocular derecho. AV, oculomotilidad y RFM normales t:A.Lu.....\...:.,.._f f' f~,-e~ ~h\. 23. Exudado pleural coh.~_!.% PM~_, lacta!~~.?, P~_: .!~~~ '=.. . ~".,:_'- . 27. Nia con sinusitJ~d~'lQ.!uciona confiebr~.Y.~!J!~~..!.. pertocular.tzqul~rd~. 'Ai";xaieil;:p-,optosts .leva, 24. Derrame pleural con 90% PMN, lactato 2, pH: 7,32 limitacin de los movimientos oculares y L_ li.J..it._;.....,----.J. dis;ilicici"e.a-~y~--~:- ------ ,..__ ,. \') .:;_.,..;....;._._ .J ~ \, (_ : LLC!._, Lo.J..:.--lL"" 1-. C:.r;) . ~-

1.:: ~~\v.;

. . '\

. i J' t., (_ t L.~ Ll~ ~

,_ c.'-_l_L...

CASOS CLNICOS
28. Hombre de 45 aos, con lesin eritematosa brillante en la zona malar derecha, de S cms de!

CASOS CLINICOS
32. Hombre de 25 aos, C9J1!Qiiculitis a repeticin en cuello, consulta por dolor:,l;.ii[Uii-eto-cie___ ..v~i~~.j.~n..~n-~o..na.;d'Cai posterior. Al examen se aprecia..JU.i!lJJ'Ias Inflamatoria, fluctuante, con ----., . \ . ptlst'!las confluent~.._m_m~t..ruJJg1!.f.P.Ii~ldosf 1 . ~ .. k ..c- \ G, pDosos. fq_.--:.._~~o~ G (...(,..____~..J -....<..--',j...Y-'-l L; ~).--'..;-o v..33. Nio de 1 ao, comienza con fiebre, eritema ~~ periocular y e~J?li.i"Ms;apareaenao-- . -~-

dim~~!'-~~.~.9.r.!l~~:~tile~os, as~ciado~;
ad~f1.QPtJa~.preauricy_lares,

ce~~~al_~~ P.9~!!!l9.-:~$.Jpsllaterales. <: ,..,,\ ~

retroaurlculares y

r-t\....;...__

29. Nia con erisipela, presenta adems cordn _ ... 1 : eritematoso que asciende por el muslo. L~.1"\.--r _._.:;) t'i-f 30. Prescolar con aparicin de lesiones periorales v mejillas. Al examen se aprecian ~ostras, claras, poc:_Q_.5J~herentes en zona perioral. infr~!!LY ~~-

~~-~~~-~~J.!~~s . \ ,-. ~

\' ..l "\-' e}<--:.

..

gener'!_l~~~~a~.Y...~-~~~~!-.f.e~~es.

rpi"d~!ien!i.P_lis.lJlli!LeJLP-Iiegues ~les, cub~ta~~s,'!_~lar!!~..'t.P.O~~!.~~~L9.~~-~-~~-~-~-

31. Paciente cursando imptigo, se produ.~e..lcera central en la. lesin rinci al ~~- c:~. . --z;'..._
:;_.: : .\. ,\..' \ (-,.i_..LL'-f,._.l.__'.f
1

-...--.. --~

J ~~l,

( Ji,J..._J:.

-E~,\ C..-f~..J.: .. u'---Y ..

.t

... ( _ ~ _,
tt

, .

''

J_. ~ )
-\~

__ .... .... . . G~- r.y d~- : L...


.
~

. -

'-""--"

"\. :"'\ -~ t

1"-- -- .

._j'

,:\.f-.('-\,1.'-{..._ (/'-..

(._c.o.-~

J..

. ------- ... - ..

-t.'-. e \, ...(,'--

'\,lt.........

. >--". ~
' 1

- '..!...

37. f.Jio de 4 aos, con claudicacin de la marcha y )' por S. aureaus. Evoluciona con eritema _L {, fiebre. Al examen eritema y aumento de volu{llen l generalizado, fiebre Intensa e hipotensiO- ,~~ 1 .,...., en zona pretlbial derecha.+l.G.'-'--'.!L-~1.f sostenida a pesar de suero folslolgico. -------.:..:=::: 38.1\lio de S aos con lesin en cu~ro cabelluc::lo, 35. Hombre 65 aos, con micosis e hlperqueratosis caracterizada por placa alopcicc,.descamativa,..de plantar e interortejo. Consulta por fiebre y dolor ...- ~~ 4 cms con pelos quebrados. J ., /\:e:.\ t ;. f~ -(K J'J ~ en EEII. Al examen placa eritematosa en musl_o ....~\..:~39. Estudiante de veterinaria con lesin pruriginosa en t ~'!.~ ~\1-.\ :-~ derecho: muy dolorosa, con vesf~~~~(i.<J:u.--~-kl~ el muslo izquierdo.+;~ 1-~( 1':-~."'\Sr--........ \"-'0 V\ hemorragicas de gran tamaiio. fr~ Al examen se observa lesin de 6 cm, con borde-...., ..-., 36. Un pacient~ de 56 aos consulta por intenso dolor neto p~ul~-~~!!losq(frclforme,_~~~~~ndencla a~ ~ / en rodilla derecha, CEG y fiebre. Al examen se la fU racin central. -== --- - aprecia rodilla derecha eritematosa, caliente Y 40. Paciente postrado con lesin sacra negra, de 4 cm, f'- ._: ~ aumentada de volumen 'FV'~\ f ~ ~~ S!Ca, con m{nimo eritema circundante. ~ 1 \.. \. r_..Ju,~..-t.A-1 -. ~-- '-~ \ <:. r:. ... ~ -r t{ ,.... : ) _:\ .: .~\c. t t.-\- ( ~~~. ~~1~~~~~~ -\ 1,...._+( ~-""-....,o .1 fY.... -,\~ -..,.., l ~..-.e e (' , ~ . ~

34.

Adoles~!~6~~. 5,~~~!~~~ltlstr.....Q . aguJ


. --- .

r--------------------\!-...~"\~~~ ...l

.\

i .} \ (..\.:

""""'

CASOS CLNICOS

1-< .. ,.~

tc-:-

( ' .Ji ~ :. ., ,..,)'-~.


~l.-.~

\-( (

t,~

. . '-'...)1--.-d"' f. ( _,,. ~' (.

1:, ):..

' _('-:. '-

.,,fL.t. -+ ('.AJC . . \ L ~------~~~~~~~~---------------------n(!~ ~-------------------------------------------,


c.-\j\_

~ : eL. t-:::nt. -1'(f

V"

-:~., ....

CASOS CLNICOS

\....../i

:..,r J

CASOS CLNICOS
44. Nfo de 7 meses con fiebre hasta 39,5C, se ve de buen aspecto general. la fiebre cede --.}(:.. ,} !<... espontneamente y se agrega exantema _)!'";)" --::.o.. """ macu.lopapular en tronco. J .L7-. 1 :j-<-r' ...':J; (_::-.-,J~ 45. Pc1ciente cursando amigdalitis, presenta exantema ,)~-.c:r~ generalizado, en piel de gallina, con petequias 4 ~. .. 4'1 lineales en pliegues cubitales~.S.e.()~~@D'- -c;J.~m~s ~ ~--ev\..l~ le'l!~~-~m..fmtU!a. blanca 46. Nio cursando varicela en 7 dfa, con buena evolucin, presenta nuevamente fiebre y compromiso del estado general, con aparicin de nuevas lesiones cutneas papulares, asociadas a lengua enrojecida y lf,neas de pastia.
r

41. Paciente postrado, con lesin gltea ne~a de S i' cms, con escaso exuda~o purulento y_ ha~g t eritematoso de 4 cms. ~') .~ ~ 'C ~"1- ~\ r:!;.e :.. .:;.\~..... 42. Nio de 4 aos, no vacunado, con cuadro caracterizado por fiebre alta, gran compromiso del estado general, tos, conjuntivitis y luego apar!ci~ de exantema macular rojo Intenso ~(}vi\"-'~ 43. Nio de S aos, con fiebre y eritema en ambas mejillas. Se agrega exantema macular en encaje en tronco y extremidades . .( ... ~ \..... . . ',,....:.-\-eL V{"~ ~ .. j ~ lcl .
1'\.::'

1 e_(

v(~'--~c~~ (~

CASOS CLNICOS

CASOS CLNICOS

so. Nlilo de 6 aos, con fiebre leve y aumento de 47. Nio de 8 aos, con odinofagla Intensa v CEG. Al ; volumen parot(deo derecho. Evoluciona luego col') ~ examen se of?servan vesculas, algunas ulce~das .,.J ~ \1'1{'\..-i.~'J l-C aumento de volumen y dolor testicular. 1 en paladar blando y 3 vesfculas en planta :yt.rt izquierda. ~ L\.,- !'-\.\...~~ -'-.~C:....<I- -~ t;::,:.- 51. Adolescente de 15 aos, consulta por fiebre, mialgias y exantema muy pruriginosos. Al examen 48.lactante de 15 meses, con fiebre e irritabilidad, se ;aprecia paciente febrU, en buenas condiciones, _ asociado a reticencia para alimentarse. la madre con exantema papulo-pustular que compromete cree que le duele tragar. El examen ffsico muestra . , '- faringe eritematosa,_ con vesfculas en paladar ~\e 1 .e-) cabeza, tronco y extremidades. Se observan costras en cu~ro cabelludo y escasas vesfcias en blando, algunas ulceradas, sin otras alteraciones troco -J~'~ 49. Nia no vacunada, con fiebre leve y CEG. 52. Hombre de 47 aos, con ;!olor en tronco. Al exmen . Evoluciona con exantema macular generalizado, rosado vadenopatfas cervicales posteriores. se aprecia placa eritematosa lnframamaria derecha, que se extiende hasta el dorso, con \'lA~ l .. t.~J~----(~ . .

(r::...-.

pt'-

.....

~-----~--~--------------~----~--~~~ ~ ~' :J- 'v'- 1' .l.\~,.r,. ' , ..._. l. ~,_...,_ 1~!-. ~-... ..,i ' J....~

-\ ~~ \-.. ~ .. l\

y....,,\ \.

~ ~.,

r.~

53. Nio con Vqrlcela, en 7mo dfa de sntomas. ' ~(..l.~-,.~ Consulta por eritema confluente y aumento d~ volumen de algunas de las lesiones en EEII. J\1 examen, febril, con placa eritematosa en Ell)0 , con 2 vesrculas hemorrgicas sobre el eritema.

CASOS CLNICOS _r;,s:-~5----~- -lt.

54. Paciente embarazada de 28 semanas, evoluciona... jYJ'- con varicela. Al tercer da presenta tos y .., ,4~ compromiso respiratorio importante. ?~\, SS. Hombre con fiebre y disuria e Jnte~o dolor u la palpacin prosttica (..~(~~'r\ / l,;:f.l,..o . / {; 56. Mujer de 22 aos con d~lor abdominal, fiebr, y l.eucorrea. Al examen~-;; anexlafizqierchi'muy d-;;iorosa V dolor a la palpct-m---- _..:.~ - ....... .

57. la misma paciente anterlo! evoluciona co~ aumento del dolor abdommal y signos per1toneales ~ }....,;., 58. Mujer con leucorrea de mal olor, griscea de pH 1 L .J..!.._; alcalino. Al examen no se aprecian signos 0 uv"-~ '!-"' inflamatorios e_n la mucosa vagin~l 59. Mujer con leucorrea verde abundante e intensa disuria. Al examen vagina muy nflamada con

CASOS CLNICOS .f\~~ '?-'f:lo

petequlas~L-0"...._.,._~~~-

60. Mujer de 33 aos con leucorrea y prurito vulvar,. /}\\ li con secrecin blanquecina grumosa. c.. l'-""'-cJ.._.l,..,~ 61. Paciente de 35 aos, consulta por disulj~ .i~t~.nsa, asociado a secr~~~-r:J uret.~l de 2 dfas ..de evolucin. Al examen se observa secrecin uretral purulenta.

1 '-

CASOS CLNICOS
62. Recl~n nacido, con retardo del crecimiento. Es evaluado por letargia e hipotonfa. Al examen presenta perforacin del tabique nasal y epistaxis Intermitente...~~.~ ~.J. -> tfi'J C 63. Mujer 18 aos, consulta por fiebre, adenopatfas inguinales bilaterales y ulceraciones confluentes y dolorosas en vulva. l-l-0'(fl.1 __ 64. Hombre 19 aos, consulta por dolor y lesiones en ..~ pene. Al examen: adenopatfas inguinales y vesfculas en glande y f;;~e_~J:l~ .~:~_pene. ~~-} 65. Paciente de 45 aos con mltiples lesiones polipofdeas, Indoloras en surco balanoprepucial y cercanas al meato urinario. Cxl'-IJ.f3r:rrA..

CASOS CLNICOS

....

(!.

Ol!i;\
\
~

66. Hombre de 22 aos, con disuria y secrecin uretral, (~ ' ,J-' dfas despu~s de relacin sex~a] sin proteccin. El . '' } J Gram de secrecin uretral no muestra bacterias.(..';-. 67. Paciente de 25 aos, consulta por lesin en pene. Al examen se aprecia lcera de 6mm, no dolorosa, : . 1 limpia, de bordes netos en el glande. (\--e--(..v<) l ,\ ,_. . l. "',\

r:::

,n . ). . .

68. Paciente de 44 aos, con pareja sexual nueva hace 2 meses. Consulta por astenia y fiebre intermitente, de varios dfas de evolucin. Se agreg exantema maculopapular generalizado, con compromiso de palmas y plantas.

. J
i;
'1

L::_' \ \.. j 1

-\ O. ....(....

r:.rt,"J r-.1-c (1A


. 1.''., ,
t ;._

',

'.-e

. J\

\.,~- ''l

: ..\( ,., J:.

.:;~l..~-{

l ( \~ \ ~ \ '-

CASOS CLNICOS

CASOS CLNICOS

69. Paciente de 16 aos, con odinofagia, aftas, fiebre y 72. Paciente de 16 aos, con fiebre, odinofagia y ': C. ,. ~-:; . , compromiso del estado general importante. A.l poliadenopatias cervicales. Amfgdalas con gran \ ..... examen lceras bucales en lengua, paladar blando exudado blanco adherente.. Se inicia amoxicilina, y labio Inferior, algunas confluentes, asoclada:s a .1 presentando rash maculopapular. mltiples adenopatfas cervicales sensibles ~\ t-"' {\.. ) . 73. Paciente de S aos, con fiebre intense, gran 70. Adolescente de 16 aos confiere de 7 dfas de ,:.:/ compromlse del estado general, odinofagia intense evolucin, con odtnofagia y a~enop~tas cerv\~t~Y . .P .-- ~\ y poliadenopatfas ce~icales y bazo palpa!;lle. En Al exame~ escaso exudado ~migdalino. El - fv'_,~\5~ ')-"!,; ~\ sus exmens destaca hemograma con leucocitosis hemograma muestra....liniQ&Itosts con algunos : Al'de 16.000, con 80% de linfocitos, 15% de ellos :r..... IJ.!!!.~~~-~.c:!~~!!plcos. ~~ lgM \!CA rersulta negatl"a atfpicos . . \LA .. ,-. J; . ) v-JJ L- , ~ \ , 71. Embarazada con sfndrome mononuclesico con ('1\J~..r. lgM toxoplasma(+) e lgG toxoplasma()

. ,---..~ ;,_.~] ~~~

"' - ..-~
-

V"' . .

')

l.e..-7p-J.oJ i\.~' ,'CJJ'


J

~ ... _,

-JC:.7.J

e,

CASOS CLNICOS
74. Paciente de 26 aos, con fiebre y compromiso del estado general y dolor leve en HD. Se agrega lcter!cia. Al exame~ des~:~a hepatomegalla

cAsos CLNICOS
.

81. ~aclente en tratamiento erradipa~_or ~eH P.Y.Iori, evoluciona con diarrea':\ ci.:..:i- (:'~ ..o~; l:j_:l\_\~ . . f.~ ~~ 82. oH~ por OH, ingresa con encefalopata hept~c~ y sen~-~le .f..\~~-~.l-\.b '-) . , \J t \ a:;citis. Paracentesis diagnstica demuestra 380 7S~_H~s J\~{'-~, H~~!:~~:'"}, HBc Ac lgM(-), HBc Ac lgG(+) ~::: ~~,__,. ~- clulas de predomino PMN ('! C?::.' :;:.~ 76. HB~__Ag(-), H@s Ac(+), HBc Ac lgM(:-), ti_~ e; .Ac:lgG(::) '-' . .. ~8~. Piiciente con fiebre alta y CEG. de S das de 11. HBs Ag(+), HBs Ac(-}, HBc Ac lgM(-), HBc Ac lgG(+) .~... i.A. J. ~:'Y e\/olucin, se agrega ictericia.y.dismln!:l..99_1!.~~!78. HBs Ai(+), HBs Ac(-), HBc Ac lgM(+), HBc Ac lgG(-) V:_/.j. .. \ -: \~ / nl~~t4.~-~-m:fen.Gia. Se habra baado e~ aguas _ . . \ j; e!;tancadas. Al examen se observa lcteracia leve, cc~njuntivitls hemorralca b.ilateral_y -f~ez ':f'e. 79. Paciente de S ano~ con dasenterra. Evoluciona con 1 ' - -------rr..:.::Dz;:::::~-:-.-- - .... --palidez y anuria . .\..~ \ J ~~~~-~~~~~~~a _V...L~)-vt \f~-). ;<i \ _. f 80. Paciente tomando clindamicina por absceso \\ ( 84. Pclciente con fiebre y adenopatras cervicales. lgM pulmonar, presenta diarrea bclrtonella (+) .
') \

1 h.:..~_,.. . . _ ... ..._.~ .1


('.\__..~ J 1 X
1

"i:'

'\JI

"~ q:.:~./,.) ..... t_tl. ~-~y:_:.~ ..


.. :- "-
t:-

-U''-'-'-----.

~ 1....

LL~

..

1}

9 ..

<(_)

c"t-

CASOS CLNICOS
8S. Paciente de 6 aos. La mama muy preocupada , .._.) porque habfan gusanos en sus deposiciones .::j .... 86.-Nio de 4 aos con prurito anal importante(:).\_ 0 '~"

CASOS CLlNICOS

90. Paciente de 4S aos, con insuficiencia cardiaca de redente diagnstic~, pofmic;caraio;atfa-dllatada:- . Se agreg~ dJ~i!g~a, po~~~ ~~~.!!ea~~~a ED:4fque (!\:....! O ' / ' ~ muestra dilatacin im~~rt:!~_~e. d~. e~~~-87. Paciente del sur con fiebre mialgias gen.eralizadas, es1lfgico. ... - ... _. mayor en oculomotores. En h_emo;ra_ma se ~_,o . ----.-,-91~ Paciente con lesiones Intensamente pruriginosas observa Importante eosinofiha ) ren tronco, manos y genitales. Al examen se ven 88. Paciente del sur. Pr;;eiita tos-ion salida de . .L,- >) signos de grata] e, ppulas y escoriaciones. se .. ""' abundante Uquido trasp~J~~te ~or boca, e.~ \ ..r-0 r ' observan adem;s_vesiculas..p.~. Familiares ~ . ..~ .f relacin a dicha tos. J ~..-..e~.\; }~-. L con sfntomas similares -J (f- , -e-,_ ..e \.: .J (.-( .._ '" .=;.__rt~

c.1 './'\

(>,JL

~t./

CASOS CLNICOS
93. Una mujer de 40 aos, diabtica en tratamiento con insulina, lngres~ por fiebre, dolor facial, cefalea, disminucin del nivel de concienda y enrojecimiento nasal con lesi?n ne;ruzca en fosa nasal derecha. jh.Jvt_.IJ( \""'-. c ~ }\ J ... 94. Mujer con RPO de 33 semanas. Se realiza amniocentesis que muestra leucocitos:60, \-,-~c'..L. glucosa:lO, gram sin bacterias. ' .. t' e> " 95. Embarazada de 33 semanas de gestacin, con fiebre, taquicardia y leucorrea esca~a. Se constata DU de 2 contracciones en 10 minutos. A la especuloscora se _observa que la leucorrea sale por OCE. .
, \..e

CASOS CLNICOS
96. RN con bajo peso al nacimiento. Al examen hipotnico y con perforacin del tabique nasal, con 'escasa hemorragia.I \. F-\. '- .--'\ 97. RN con bajo peso al nacimiento, ... ,., 1 hepatoesplenomegalia, microcefalia y <. \ .... , calc:lflcaciones cerebrales en la ecografa cerebral 98. RN con leucocorla bilateral, soplo sistlico con frrnito y microcefalia f2/0 L;...A~_./.-~ 99. RN con hipotonra y rechazo alimentario. Al examen ictrico, hipotrmico.

,.r

-~}

\~'-~-~--k0
~~.._/)';'

a> ..

,, ., f. . .
1
i .\ '

.
:

.-:-
~)

--~J.----____,- .' 1\
.i .. , (. ; ./-'

....

''\.e~,

'1

':

1. . -, ~-

~ ,-. '

1) Pac(nte de .ae~ cesarizada-atrteTior, cursando embarazo de 36 semanas, presenta BP le 1o h~~-~--d4!..QY.QIJJ.cjn. No se registran contraccionesufennas ni modificaciones e---lalesy-el registro basal no estrn.tieslr..cmjj_lillizador. La conducta ms adecuada es:
Dejar evolucior espotneamente, sin necesidad de frmacos b) Dejar evolucionar, t?aje-se~eftl:lr-a-aRUbJtica...}laj.o...cor.tiGeiEies-f)ar-a-asel:Jrar-la-maetlr-aeiA-fetal )1 ndicar reposo, antibiticos, corticoidt3S e interrumpir cuando se cumplan 37 semanas de gestacin ~- \ ~3 ~ d}1nducir el parto inmediatamente con misoprostol . t-J..o '~ ~O 7 ? -.~)Realizar cesrea (_).Ll~.r ~&...~C~

//

--

2) En un test de tolerancia a las contracciones, las desaceleraciones precoces son

.)("

~b)_Compresin

:~~~=i~:~~l cordn umbilical


de la cabeza fetal e) Hipoxi fetal d) Hipo~ia uterina e} Cofupresin abdominal

. ..

~/)

-~

-----

(~/""K

~~-k. ..
'-.,(.) \

(~\-

_ ____--

Cul de las siguientes opciones, es la causa ms frecuente de infertilidad?


.. ndometriosis ) novulacin crnica e) Factor cervical d~ !)~lores orrrionales ~actor asculino

7\~'N~
'-->\,.~ .

cvi;\ _;
p~-.~L

.;u.J

~permenorrea
b) Hipome orrea ~o enorrea d) Por enorrea e) enorrea

4) La' alteracin menstrual que se observa con mayor frecuencia en pacientes hipotirodeas
. .

Que mo71trar la biopsia endomertrlal de una paciente cursando un

., ~~dometno normal . \ J ~ Reac~in_.de Ari~s Stella ~~ CU--a..n / \-P L'1 " e) Vellostdades cortales .Jl--t., e~ .._._,~ ~ d) At~ofia endometrial __, 1....o c9-< ] ~L _ '\....~....:...P e) Eritrocitos nucleados y normales P ~J~~d' -~~~

"e YA,

~arazo tuba-;:?>

.
C.-{;

v-:--- - .

A.,"-L'iv-o v---- cj
~

r ,

~ -"Y~/ 0
c.1

~;~J.(.. eL

:L

) La principal ventaja del preservativo, frente a otros mtodos anticonceptivos es que: } Es. ms efectivo en la prevencin de!l embarazo
b) No tiene efectos adversos ~)__~iene menor precio ~isminuye el riesgo de contraer enf13rmedades de transmisin sexual ) Es de especial utilidad en las relaciones sexuales, practicadas durante el embarazo

~--.,-...'t:-) J'{-0---G--

..

7) Una paciente a$htt9mtica_s.e_reali~a Papanjc..Q@_y, que muestra. atiRas e..P-itell~.l~--~e if!e.iano grado. La especuloscopa es normal. La conducta ms adecuada es:
- ---- --

..~ ./:. ealizar nuevo Papnrcolau en un Cto ms . ~ ) Realizar nuevo Papanicolau en 3 meses e} Repetir el Papanicolau inmediatamemte --1=tl) Derivar a colposcopia y biopsia /e} Realizar cono cervical

.. -''

. 8) Una pac;Jnte de 24 aos, primpara, presenta importante llJ.etrorragia, luego del parto de . un embarazo gemelar. Al examen se aprecia paciente ~quicrdlca. normotensa y se palpa_el fc:~~erino entre el ombligo y la apfisis xifoides. El diagnstico ms probable es: ~ ~ ..)lf!9ercia uterina ' 0 ~ " _ C--L-,(C ~ , aceracin del canal del parto V~ ~ o IJ...Ov . ... ) otura uterina , "\ \ ~ ~ d) Hematoma del canal del parto J 1/V\... ~~ e) Alumbramiento incompleto . -z,) ~co.._,._<:J '; . 4111 ~---._ ~~ J ol)J-.Ly~ 9) Paciente ~~~ presenta.esca.pes de pequeas cantidades de orina, altoSer. rerse q_ ~ al hacer cierto ejercicios, desde hace un mes. No presenta otros sntomas, sin embargo su problema ha comenzado a afectar sus actividades sociales y laborales. La cc;>nducta lnipiaL ms adecuada es: . . {) ~ ..a) Iniciar alfa agonistas adrenrgicos t,/('0 C..A..A..JL- . ".::::>lb) Solicitar urocultivo ... Q ~_/'l_U (..)._:U / re) Indicar eje 6icios de fortalecimiento del piso pelviano C"' r#erivar resolucin quirrgica e> ()...~ ('l r ~ l 3 e) Solici r ecografla renovesical, perfil uretral y cistoscopra

.-

t--p

._.,.;. .

1)

...

1\ e 2 '-' \ - .

1O) ~ a mujer purpera de 7 dfas inicia dolor y .eritema e111 la mama izquierda, asociados a f tire hasta 38C. Al examen fsico se aprecia mama Izquierda aumentada de volumen, con ~ atema de la piel, dolorosa a la palpacin y de consistencia. ligeramente aumentada, de ~ 1'-1-.JAl~ anera difusa. No se palpan ndulo~..fluctuante!S. La conducta ms adecuada es: ~ a) Suspender lcretanc1a materna e IniCiar 81ilibiticos con cobertura para estafilococo ureo J._. b) Suspem:ter-lactancia materny.ealizar drenaje quirrgico E! iniciar antibiticos con cobertura para $-v-rC\-<. ~ es~filococo a ureo <h.. -}..re~ ~ /YY Mantener lactancia mat~na, mejorar la tcnica,kfe lactancia e iniciar antibiticos con cobertura _( ..,_ 1 P" para estafilococo ureo// ~ U-Jo~~ . . l oLl\ ~ 19 d) SuspEmder1a1acfsmCia materna e iniciar antibiticos con cobertura para bacterias Gram ,.. negativas .' e) Mantener jPcancia materna, mejorar la tcnica de lactancia e iniciar antibiticos con cobertura para bacterias Gram negativas 11) JJta:ujer de 18 aos cursa embarazo 8 semanas. Consulta por cuadro de . 9'trorragia, fiebre, ictericia, dolor abdominal y gran corrtJlromjso del examen general. Al \_/"examen se aprecia paciente febril hasta 39C, ictrica, hi(!otensa. con salida de liqyjdo purulento por orificio cervical externo. Se sospechal!. ~~niobras abortivas. El agente microbiolgico, que con mayor probabilidad ~s el causaiieae este cuadro es: .

~~ Stap'Jjlococcus aureus ~~r~~~~;tc~~~~yogenes e)

1-~ft\c ~~~(N ~ ~1-s~.:.::ro--, ~~ ;) \~~ c>l'laf


<....1.0 -J. -

provu e:~

d) ~5teroides fragillis ~CJostridium perfringens

.
.

Q.cv olJ..A;\4

\o~

Una mujer de 40 aos, sin antecedentes de importancia, multjpara de 3, esterilizada diante ligadura de trompas, consulta por hipermenorrea importante. Al examen fsico se stata tero aumentado de tamao, de <!_~ntornos irreg.utams. Se solici.tan exmenes donde destaca anemia ferropmca y ~:~na ecograffa transvaginal con demostracin de un mioma submucoso de 5 cm y otro mural de4 cm. El tratamiento ms adecuado para esta paciente es: a) Iniciar anticonceptivos orales combinados b) Iniciar tamoxifeno e) Iniciar agonistas de GnRH d) Realizar tumorectomfa endoscpica .....e~ealizar histerectom(a transvaginal

13) Primigesta de ~_umaRas-ini&ia4r-abajo-de-parto: La dinmica uterina es 3 contracciones en 10 minutos y se palpa cuello uterino borrado con dilataGiR-dOm y membranas ~ /~ integras. Sin embargo se constatan lesion~s vesiculares en labio maY..Qr:.derecho, consis~entes con ..he~pes genital.~cti'~~-ca conduc~~ mas adecuada es: e ~ Realizar cesrea, Sin necesidad aelratamlentos adiCionales b) Dejar evolucionar el parto vaginal, sin necesidad de tratamientos adicionales ~ ~ e) Iniciar aciclovir endovenoso y dejar evolucionar el parto vaginal . pfOejar evolucionar el parto vaginal y cid ministrar aciclovir endovenoso al recin nacido ~--e) Iniciar aciclovir endovenoso a la maclre, realiz~r cesrea y administrar aciclovir endovenoso al recin nacido

'\-vi-

. ro/?0._. '.

.y . . J? \jJ' \\

.f'it'\

14) . paciente consulta por leucorrea de mal olor, el que se acenta durante la me!Jsfruacin. A la especuloscopa se aprecia leucorre~ griscea de olor a pescaao podriCIO;) n signos lnfl~.matorios en~~c!lsa vaginal. El agente etiolgico ms probable es: ~ ,._, ,-Gardnere!la vaginalis ~ . ' C.:. :~ e~ . J) _L.~ >'-" ) Trichol)lna vagina lis -? ~ ~.-:- J, \. rq e} Cn~ia albicans ~ ~Y~ r~ ~ ~ )'(-!J:J~ d) N~isseria gonorreae -~" e) ,lamydia trachomatis ~ V"

Un~

de~s

v -

, _,.._y

15) La causa ms frecuente de abort':> es: a) ~ompetencia cervical _.../ff..Aiteraciones genticas embrionarias . 'e} Sfndrome antifosfolrpidos d) Alteraciones hormonales e) Malformaciones uterinas

4\
Ai\
1$\

parte de estudio de un 16) Una paciente de cuadro de dolor abdominal. La ecograffa demuestra preJtf;!~e un tumor anexial derecho de 6 cm de dimetro, de aspecto qu~~tico, con numerosas increscenclas. No se observa li~~do libre peritoneal. La con'il'etarffils adecuada, de entre las siguientes, es: arpbservar y realizar nueva ecograffa en 6 meses /b}'lniciar anticonceptivos orales, mantenidos, durante 3 meses y repetir la ecogratra 9) Realizar puncin con aguja fina y dec;idir ~jo segn hallazgos i;f) Realizar tumorectomra ovrica~ con hiop~ ntraoperatoria y decidir manejo segn hallazgos -t:!:~ ~ealizar ooforectomra ms anexectomra der c~as, por vra laparoscpica, con estudio de biopsia
.ap1da

~ii~se reali2~a ec~e

-t

\.'V'-l_A_}.{_G~ --~~__:.:9-:-:. ~.

17) El concepto sinusorragia se refiere a: a) Menstruaciones irregulares b) Men2tr Ciones abundantes y dolorc,sas C.~~g do genital fuera del perrada menstrual ~~~.2 rada genital durante o despus de una relacin sexual ' e} S ngrado vulvar, que no se origina en la vagina ) Una paciente m.uiDparajfe 2, cur~;ando embarazo de 33 semanas fue diagnosticada de Cliabetes gest~9onal mediante test de tolerancia a l-glcosa, a las 28 semanas de gestacin. Se inici dieta, sin embar!}O persiste con glicemias postprandiales cercanas a 150 mg/dl. La conducta ms adecuada es: a) Proseguir con dieta y ejercicios por 4 semanas ms y controlar b) Iniciar metformina e) ~iar glibenclamida ,.arlniciar insulina e} Iniciar una asociacin de glibenclamlda e insulina

(lA

ff:l

~
~'
~

fiA
'"

~ -'

. 19) Paciente d~~ aos, con menstruaciones previas irregulares, present~raso mens~rual de 6 semanas por lo que se realiza testtle embarazo, ue resylfi positiY.o. Presenta dolor ablom1"a , sm me rorrag1a. e so 1c1 a ecogra 1a ransvaginal que no muestra ~aco embri9riario intrauterino. Slo se ob~.e.~.a endometrio e~gr.Q&ado, sin lquido libre - - _ 1~) ) peri oneal. Se controlan niveles de gonadotrofia corT~nic~ (~~~ .) c~yo resul~ado eS1.o~~.: D ~Ull La conducta ms adecuada es: ~~
Solicitar nuevos niveles de HCG y ecograffa transvaginal e~n 2 dfas b) Indicar metotrexato )~'-; e) Resolver q~jrrgicamente, mediante laparoscopra d) Resolver,.qirrgicamente, mediante laparotomra e) Dejar v'olucin espontnea, sin necesidad de nuevos controles ni tratamientos

19'
/l!f>

:>\ ~

20) na paciente de 25 aos, cursando emb~razo de&~~ presenta cultivo vaginal sitivo para StreptocQccus betahemolitico, grupo B (S. agalactlae). La conducta ms adecuada es:
a) Administrar una dosis de penicilina benzatina intramuscular inmediatamente b) Iniciar ampicilina oral desde las 36 semanas hasta el parto r~ministrar ampicilina endovenosa durante el trabajo de parto /d) Administrar penicilina endovenosa al recin nacido e) Realizar cesrea 21) Paciente de 35 aos, cursando embarazo de 36 semanas, consulta por disminucin de

X:

los movimientos fetale~. Se realiza perfil blofsico que es informado como 8/10, con liquido amnitico 0/2. La conducta ms adecuada es:

. a) Tranquilizar a la madre y dejar que evolucione espontneamente . ffiealizar nu.e.vo perfil bioff~ico en 2 dfa~ y decidir manejo segn h~llazgos / :~ ~olicitar ecodoppler umbilical y decidir manejo, segn hallazgos ' ~ ~) Interrumpir el parto por vra vaginal ~) Reali:.~r cesrea de urgencia inmediatamente .

,QA.

A
,

-~ causa ms frecuente de retraso del crecimiento intrauterino, de tipo asimtrico es:


.

Constittrcitma1- enopatlas ecci9nes intrautennas (TORCA}~ rrJ)Stl}iiencia placentaria ~ficiencia de insulina fetal
"(a ecogr~fra obsttrica del primer trimestre b) La ecograffa ginecolgica del segundo trimestre e) La ecograffa ginecolgica del tercer trimestre d) El ecpdoppler de arterias uterinas e) E~_.eodoppler umbilical

) pr examen ms importante para determ. inarMpronstico de un embarazo gemelar es:

EI perodo frtil se extiende desde:

n dia antes hasta un dfa despus de la ovulacin C_!!lco dfas antes hasta cinco dfas despus de la ovulacin ~..e1nco dias antes hasta el d[a de la ovulacin d) Diez dia antes hasta tres di as despus de la ovulacin e) Siete dias antes hasta tres dias despus de la ovulacin

25) Cul de las siguient-es opciones orienta mejor a una

ominal crnico? Dolor abdominal hipogstrico ) Dolor ~bdominal que no se acompaa de alteraciones del trnsito intestinal _,......G-Y'Dolor/bdominal cfcljco, relacionado al ciclo menstrual --:- d) Dolsfr abdominal que cede con la eliminacin de gases ~, e) D ror abdominal, asociado a cefalea y dispareunia superficial
) Antigripal

causa.~~ de dolor

~ ~

~
~

/ An!ivaricelatosa_.-~-~
.../
l~

b) Antineumoccica-~ e) Anti hepatitis B _ _.. . d!fo,ntitetnica..

) Cul de las -

~iguientes

vacunas est contraindicada durante el embarazo?


r-

~ '-J \.0..0

27) Paciente ~e2s aoa)consulta por cijsmeRorrea intensa, asoci~~.l_ ~-~ispare.YJ]ia_pr.ofnda

~
~

~
~

e
~

y di.s~~, d~ evolucin, c1ue ha ido en aurnento_en_el.ltimo tiempo. No refiere leucorrea. ~examen ginecolgico se constata doloV!..!~ ~ovilizacin cervical y a la P-!e~-~. an~l. Tambin se constata cierto grado de Inmovilidad d!~..~'!~!~. ~!erina. El diagnosliCO ms probable es: .. _- a) Cncer de endometrio )-Proceso inflamatorio pelviano dometr' i$ d) Miomatosis e) N~pplasia intraepitelial cervical

e
~

~ ~

~
(ii\

e
(!FA

28) Una adolescente de 17 aos, sin antecedentes de importancia, es trafda por su madre ya que an na.,. resenta menstruacin. l\lo presenta otros sfntomas y su desarrollo puberal es normal. Al exame gmeco og1co se cbserva m_men peoradci'y el resto del examen fsico no muestra alteraciones. La conducta ms adecuada es: a) Iniciar anticonceptivos orales b) Solicitar ecograffa transvaginal e) Solicitar TSH, hemograma, rndice de andrgenos libres. prolactina, FSH, LH, cariograma, ostenediona y 17-hidroxiprogesterona olicitar prueba de progesterona . e) lntciar pulsos de agonistas de GnRH
29) Una paciente de 35 aos, cursando

.ry

i _/
.)

JI/~

~
~ ~

le~uruLentaAiexTmen se constata FC: 130x'. PA;.i1JU6.0.,.-t:3s,~. La

embarazo..d~, con.lllta por fiebre y .?>

e ~ e

e
~

especulosCQpra demuestra salida de exudado pury)ellto_pQr_eLo.riffcio cervical externo. En sus e)Cmenes de sangre destaca ..ieiiccicltosishaS@ 18.000 y elevaci~-~-~-~..~~_f_CR."ia co~ucta ms adecuada, de entre la!; siguientes, es: , / Cj) ~dministrar corticoides, antibiticos Hndovenosos de amplio espectro e interrumpir el embarazo l' /al alcanzar la madurez pulmonar . b) Solicitar amniocentesis, iniciar antibiticos y corticoides y decidir manejo segn estudio del Hquido amnitico e) Realizar cesrea de urgencia, sin necesidad de antibiticos d) Solicitar uroooltivo;-radlograffa-tle-t-rax:-::.:,;:_he=m:...::o=cu_:l~tiv:.:.:o=-s=-=y=,d=ec=:.;i:-;d:-ir__ m_a_n_e-;-jo-s-:-eg-u-;-.n---:-re--s~u:-::lt-:-a-;do-:-s--

~d~-~~~tra~ _a~~ibi~-~~~S_f;!ll_t!~!l0.$.Q$ ..e.JIJ!errump_ir_~l pa!"!g _ppr.una .vla~xpedita

e
~

r'

e e e.

3 ) Una paciente purpera de 7 dfas inicia metrorragia de moderada cuanta. La causa ms ----robable es:
a} Inercia uterina . ~ ~r. b} Laceracio11.es del canal del parto e) Endom7tritis puerperal .---...::, .va e:~ )11 Perststencia de restos ovulares (e) Dr naje espontneo de hematoma del canal del parto -------

ul de las

es ,una causa clsica de ioma uterino subseroso ~ ~ (>re~. 1~7Z.. .

s~

-;:;;:e~eno~

Plipo endometrial +r-e N-ce; ~ Uso d~l=ltiGenceptlVos 8) cancercte endometrio 5> ~ 1 e) srndt oil le a---e-ovario pollqurgtico

..

32 na mujer de~~~ Q.bua, con antecedente de ovario pollqylstjco durante superndo f' rtil, presenta metrorragia escasa. A la especuloscopla :se _constata_gY...I.iLS.angre--pr:Gv-iene. de la cavidad endometrlal. El tacto vaginal es normal, allgual que la ecografa transvaginal. La conducta ms adecu'ada es: \? A.... . ~dicar anticonceptivos orales en altas dosis '< \ \ ~~ ~olicitar biopsia endometrial e) lnicjaf estrgenos locales d) S_p'licitar Papanicolau, colposcopia y biopsia cervical bservar y controlar en 7 dfas, decidiendo manejo segn evolucin

\-\-<J \-.)

~
~

Son

Metrorragta ce-causa no prectsada ~ b) Cncer de mama ...,.. . e) Cncer de endometrio d)~patopatras activas / Cncer de cuello uterino

c~indicaciones pai'a la terapia hormonal de reemplazo(E;cE~


--

~
~
~

X .A

34) Una paciente de 24 aos, consulta por dolor abdominal. Al examen se constata paciente subfebril. con dolor a la palpacin hipogstrica, palp~n anexial derecha zacin . .:.:. . .~JGa-1. No hay signos de irritacin peritoneal. El diagnc s 1co ms probable es: a) Endometrisis . .Proce}c{jnf!amatoria plvieo"' ~<. \:x(_
e) Apep icitis aguda d~~sceso tuboovrico roto ,

~
~

~
~

) mbarazo tubario

J:

S) Paciente embarazo de 28 semanas,.asilll!!mtica.._se ealiza urocultivo, como parte de su control prenatal, el c::ual resulta positivo para Escherichiacoli sensible. La conducta ms adecuada es;:

m~ursando

_..

~
~

~
~
~
~

a} No reqlifciniueves-ur:eel:tltivas-AiaamiAistr.ar-af.ltibitices,..dada ausencia ae-sffitGmas b)jldrlJiistrar -eiproflexacino .araliD_!l~!ida~~~~rocultiygs d~ntr-el . .,....-e) A ministrar cefadroxilo oral y realizar urocult1vos peridicc,s .... d) am inistrar-eeftFia~naeAEievenesa-y-Feali~N:I rocultivos pendJcos e Realizar urocultivos peridicos sin necesidad de tratamiento antibitico

~
~

36) Cul es la causa ms frecuente de


a) Hidrosalpinx

tumor_an~~ --------------- - - - - j las pacientes;~-~d-ad~~~~\ -:-- ------

~
~

Ad) Quistes de origen funcional Adenoma


e) Cncer de ovario

b)~ceso tuboovrico

""'
~

.~

~ ~ ~
~/

~
~

. A

37) ~~Jer d~.1!!os, cons~lta por re_glas irregulares, las que presenta cad!...2 a 3 meses. Al examen se aprecia -ebesa,-eon-birsutL$...mo leve, acne y piel grasa. La causa ms probable de su'alteracin menstrual es: j Hipotiroidismo rsrndrome de ovario poliqufstico ) Hipogonadismo hipergonadotrpico d), Hiperplasi suprarrenal e) Tumor productor de andrgenos

y
ro

'!!'
~

~
~

~
~

38) Una de gestacin, asintomtlca, art.!~.~-' q_~Q!~Oil ~lliLQ~as..in. El resto del examen ffsico es no~mal. Se controla <:. ).' / protei!!_~..alitativa-que-resulta-lev.emente...p.o..sitiva ..La conducta ms adecuada es: / ) aflnjQiar diurticos, asociados a iECA . . . -~ A . j}ffidicar dieta y ejercicios t3ii l lb ,.{J (. {- r, ~Indicar semihospitalizacin de 6 horas, controlando la presin arterial en varias ocasiones ~ ../()-' ~ )~ ..r ci) Iniciar alfametildopa y solicitar exmEmes que determinen la gravedad del cuadro e) Administrar corticoides e interrumpir el embarazo

paci~pri~ta, d~manas

presen~ ~
'f .

~
~

39) Pacient~.d SS aos, present"'j(!ntorragi!)A la especuloscopfa se observa una lesin d~ .~ -~~~ una pequea ulceracin. El Papanicolau resulta normal. La conducta ms ad~cu aaes: --~ a) ,.etir Papanicolau en 6 meses , ~/.b: erivar a colposcopia y biopsia ~e} Realizar cono bipsico ,d) Realizar histerectomfa simple e) Realizar...histerectomfa radical

....-----.....

fumadora, multlpam de 2, cursando embarazo. de 32 seman s s ta..R.or orragiay-delt)i=.:a~inal. Al exiiifisico se aprecia metroJ:149 casa oscura se a litera hipertnico. El monitorec fetal demu~$J[.Y.ariabilidad-disminuida~ desaceleraciones vciriables complejas. El diagnstico ms probable es: ,,, a) _placenta previa -+ ~.V\ -\ ,_x._('--t .)._A._4 ......-_':.tt,.=oesprendimiento prematuro de placenta normoinserta -!J : ""- --...... <f L/ ~ ..-r..- e} Rotura uterina d) Rotura de .va.sa previa e) Acreti~,tn placentario
,.

ru(~~ciente

~
m

'~ -

~
a

e
~

41)_ul de le}~ siguientes medidas est coJltraindicada en una Raciente.con rotura_, t1r..m..~l.trr_.d.,_in~mb~,_ que inicia dinmica uterina ~ las 30 seman~~ d_e g_e~!~~i~!'? __ //~) Administrr corticoides ____ .. ------ ---------- --- --- .---.- .. ~'-l;l' ~ b) Adrninis(rar antibiticos ~ r-N"-C--- t-o L.QJL ~rnin{str.ar-.tocolfticos \\-' ~a) Rea(;zar amniocentesis e) ~alizar especuioscopla

-f' ()

~ fl!\ ....

r'
f'.

~{) Una mujer, sin an edentes mrbidos, cursando embarazo ~emanas ~resenta_.. \./hemogl b" O mg/dl n hemograma control. La conducta ms adecuada es: a} Controlar con hemograma en un mes ... ~ ~~atar con fierro oral . . . J. )\./'~/ e} Descartar hemorragia oculta h cJVd} Administrar cido flico e) Solicitar cintica de fierro

!\f\

(fA

LX-~

.- n

~'- l..D-.

?
.

~-t ..Ju. l.-1r'V"'.

O CC-<.~ u <-..c"Q

(_

B re c..~"

~~

Y=' ~-.Jl..oL c.__p_


!'...rO

.-. _

~
~
~
~ ~

43,) Cul de las siguientes presentaciones tiene opcin de parto vaginal? . ; Nasoiliaca izquierda anterior . . .r .. o-J' (L.~'r) b) Mentopq_bjca . e en o1acra ') d)_l)lasosacra ~Acrmioiliaca derecha posterior

~
~

44)/Una paciente de 22 aos, cursando embarazo de trnlino- inic.i~jo de paJ19. El ~ re~istro estresante muestra.desaceler!~ines_tardas e_relacin l8_0%'de las ~ \ ~' n~racciones uter~nas~ La frecuencla cardf~a basal del feto es 130 lpm y la variabilidad es.,. \ ormal. La conducta ms adecuada es: . . 1 -(~ ~ealizar un taqto vaginal para deterr11inar la vra de interrupcin inmediata ' A~ ~ ~ ... b) Solicitar ec9doppler umbilical ~ 1 e) Realizar perfil biotrsico ) ~v~ d) Esper51r"'evolucin espontnea . ~ e) Administrar corticoides endovenosos. antibiticos y tocolfticos

'

r .

ntensas ..d!!..~~in.ar-1. que muchas veces le es imposUi_f.~_@lcanzar.ll~gar..al.b.aflg..,mojan..d9JU ropa con gra~_des cantidades de orina. No presenta escapes con la tos ni con el Y.ls.alva. No pr~s~9ta otrs sntomas y el examen fisico_Jlo aporta tnayor inforlifaci-~ ~1-dagnsti~-~m~'Prob~ble es: _,.. . r ..-?~Yincontiriencia urinaria de urgencia ..._,... .. 6) lnfec.cln urinaria e) CisJitis intersticial d) ll)ontinencia urinaria por hipoestrogenismo ncontinencia urinaria de esfuerzo ) Una paciente de 55 aos, a~jntamftica, se rej\liza-m~u:pografa,.como.par:te_de...su.equeo de salud, la q~!..~.:! ~t:t19.IID!lda com_Q_~~a~~)L.a conducta ms adecuada es: ~ ~L~epetir.Ja. mamgrafa en 6 meses -- ~o licitar ecograf(a mamaria e) _l)olicitar resonancia magntica nuclear ~) Derivar para realizar biopsia estereotxica . e) Derivar para resolucin quirrgica inmediata 47) Paciente d!-~34 aos, cursando e~~-~~~~~<!!-.t?.!!!l~111..as, con hlpermesis ~ravidica, consulta por metrorragia. Al exam~n f1s1co se palpa el ':1!~-~~. ~.la -Jtur.a_del..p,ubls_y__~l ~~~o vaginal /l)opre.s~~ta alteraciones cervicales. ~1 diagnstico d~ s<?specha es: ~ -{L..

4~laciente dehecho ms frecuentes en i.Y!Ymo tiempc,. Refiere que siente ganas muy 50 aos, consulta porque sufre de escapas._!!!_orint_,_c:les!;ii!E.I!P.!.~I!I~Rs, 6s que se han

~
~

r?'r

""'
J_A-rc,l'-"(..

~~~
~ola completa ~
) e) Embarazo ectpico

------ ..... .

.1\~ t . '
-----~

---------~---
t-'--1., ~

~1--r-o rv-c~

-,

""'1

1
1_~~-..-..~:~d-N c.p'
--- C.~-:fvo V""-rG ~
t.~ k_:_~~

4-tb OJ--""

-t1,~=-

' !''
48) Paciente..Cte 25 aos, primigesta, cursando embarazo de 30 semanas ingresa por intenso dolor epi.sstrico y gran compromisc> del estado general. Al examen se aprecia paciente en regular:,es condiciones generales, ictrica con frecuencia cardaca de 1251pm, presin rt 1de 150/100 mm Hg. la palpacin abdominal es muy dolorosa, especialmente en el ondrio derecho. Se solicita proteinuria cualitativa que resulta +++ El diagnstico ms p able es: . ) Eclamsia .... boHa Ei lrquido amnitico dro e de H ELLP ) tasia intraheptica del embara:!o e) Hl ado graso agudo del embarazo

..

"

~-

~-

!':

~
a)

~ ~

~Lanariz

!.a frente

) El punto de referencia en la pres,entacin de frente es:

"1: .
\

j__

__
-,

v~

~l

e) El mentn d)EIZregm .. e) El Occ cio

~~.....

J --? ~~ n

\ 50}...l.,., ul de las siguientes alternativas no es un factor de riesgo para el cncer de \.__...efJI.metrio? )'('Uso de anticonceptivos qrales . b) Hipertensin arterial~,./ e) Diabetes __..-~. d) Obesidad _ ...e) srndrome de ovario poliqufstico

C.'

' J ' ' - -..__.. .....

f'

~~~~

f!'

~ ~
2

8'7o
@. 8J1~
na mujer est en trabajo de parto, detsde hace algunas horas. Presenta 2 contracciones cada diez inutos, y el tacto vagina demuestra cu,ello uterino completamente borrado, con dilatacin de 10 cm y se palpa la cabeza fetal en presentacl en es as -2. Se administra la anestesia epidural, se inicia aceleraci n con oxt ocma, logrand contracciones o membranas, dando salida a Uquido amntico claro. Se reevala luego de 2 horas sin presentar cambios. La conducta ms adecuada a) Dejar evolucin espontnea ) ~ ...J \ _ (;/":- b) Realizarforceps ~ "' J-"J ~ - e) Suspend fel goteo oxitsico '"\,.. ~1 .F ~ <} ~ume ar el goteo oxitsico \ ~ ~ \'""'",/) Re zar cesrea \O{}J'-'~ {11.)

~ ~

~ ~

es:

f\: r .

\.f

1\

l (AJ

J:.P

(!!'

3 Cul de los siguientes es un si_gno de asfixia neonatal? a)DIP1 E~ b) pH de sangre de cordn de 7,2 t:_ ':.\- 1o ")
_. e) Poliglobulia / ~~~no de Babinsky y-veco espeso en el liquido amnitico a paciente de 55 - . . srntomc1s climatrjcos, acude a control ginecolgico, detectndose a pequea esin solevantada de 1 crn en la zona posterior del cuello uterino. Su ltimo PAP se lo realiz hace meses y ue m armadocomo normal. La conducta ms adecuada es: a) Solicitar nuevo PAPen 6 meses b)~olicitar nuevo,PAP inmediatamente ~ ~erivar a col(oscopia y biopsia d) Realiza nizacin cervical e) reali r histerectomfa total

r1' -

~
~

~
~

4)

~
~

e ~ e ...
\

~ ')

e
~

n lactante puede levantar la cabeza entarse. Qu edad tiene? a) 1 a 2 meses .)7f3 a 4 meses ,.. . e) 5 a 6 mese d) 7 a 8 me s e)9a10 eses
5)

)f

mirar a su alrededor, sin embargo an no es capaz de

6)

e
~

a paciente de 28 aos, obesa, cursando embarazo de 8 semanas, se realiza glicemia de ayyno_ e resulta en ~g/d:.t:No presenta sntomas y el examen ffslco no aporta mayor informacin. La d cta ms adecuada es: ,/ ntener los controles rutinarios d~Jarazo, sin necesidad de estudios ni tratamientos adicionales ) citar nueva_glicemia de ayuno e) Solicitar tes tolerancia a la glucosa oral d) Iniciar di a y ejercicios y controlar con nueva glicemia de ayunos en 2 semanas e) Inicia sulina

7) n lactante de~ previamente nano, presenta rinorrea. tos y luego dificultad para respirar. Al xamen fsico se aprecia ~qr ripnelco, ccn retraccin intercostal y se ausculta mltiples sibilancias. inspiratorias y espiratorias. El agente causal ms probable es: ' a) Neumococo b) ~reptococo betahemolitico grupo 8 _.r;YVii'us respiratorio sincicial d) Vir~s parainfluenza e) Virus influenza

")

~
~
Una mujer acude a control obsttrico, con L.m E!mbarazo avar~o. que no ha sido controlado . revlamente por mdicos y que no se ha realizado exmenes de sangre ni ecografas. Recuerda el da de su ltima menstruacin, que fue hace 32 semanas y 5 dfa, y refiere que previamente ss reglas eran reqylares. Tambin refiere que hace 13 semanas y 3 das inici la percepcin de los movimientos ~ es. La mejor forma de determinar la edad gestacional en e!;ta paciente es: Q..)echa de la iJinia regla t .A _ ll\ J \. b) Ecografia opsttrica, con fetometrla . ~ ~ ~ d- sr' , e) Fecha de 'nicio de la percepcin de los movimientos fetales. ~ ./\ d) Altura erina e) Nive s plasmticos de gonadotrofina corinica humana . 9) nio d(; por general de 3 semanas de evolucin, asociado a fiebre in rmltente ~a, que en el ltimo tle~po se ha hecho diaria Y: gue res o e slo arcialmente al ome al oderada. No se o de parace mol. Al examen ffsico se ~precia plido y se pal a es encuentran ot~s alteraciones. El diagn~tlco ms probable es: a Mononuc e ts tn ecctosa x ...b~ _I;!Pfoma ~;_uce a aguda ' d) Meni itis viral e) Tu r del sistema nervioso central na paciente cursando embarazo de me anas; la que se confirma con la positlvldad del test de cristaacl6.AI examen fsico se palpa ~.fe o en presentacin ceflica, no hay dinmica uterina ni modificaciones cervicales y las pruebas de onitrizac;in fetal son normales. La conducta ms adecuada_ es: a) Administrar antibiticas y ser:tiGoide~a y lllaRteRer YRa ceREh:tete e(pectattte b)~dmil)iStrar Gorticnides endover:aosos y aAliblticos i# tnterrumpir~ler:Rbarazo eR 48 horas con mlsoprostol 7 fRe izar amniocentesis y decidir manejo segn hallazgos --tJ) 1 ucir el parto con misoprostol \{) ~ ~ O e ealizar cesrea ' 11) Una mujer por leucorrea y prurito vulvu. Al examen se observa leucorrea bl a ~rumo~a, con ~ignos inflamatorios en la pared vaginal. El agente causal ms probable es: ndtda albtcans . . ~ 1-o \e__,b) Trichomona Vaginalis f'J ~ e) Gardner~lla vaginalis f 11 ,.,._.c::. .v- '~ o--'--. d) Neissria gonorreae 0 u ~~ e) 1: ponema pallidum

~
411)
~

~
~
~ ~

~consulta

malest:~r

,-,
~

~
~
~
~.

de~ e_rlmige~ta,

~Hfl:& ~

~
~

..-,
~
~

de~consulta

-~

~
4\'t

,;' 1Jtr-a causa ms frecuente de anemia en los nios es: ~~ ~~rropnica b) Talasemias e) De enfermedades crnicas d) Aut inmune e) P dficit de folatos ) ~ul de .los.siguien.tes hallazgos _es consl~erado como normal en un recin nacido? ahda conlJn de liqutdo por el ombligo olsa se sanguinea .-/ . e) Pete as d) Le ocoria ~ ~ (O:h ~~~ e) otona
4) El feto grande para la edad gestacional se define como aquel que:

~
~

a) Pesa ms de 4.000 g b) Pesa ms de 4.500 g e) Presenta un peso fetal mayor al percentil 90 para su talla ~resenta un peso fetal mayor al percentil 90 para su edad gestacional e) Presenta una relacin perimetro abdominal/ perimetro ceflico mayor a 1

,..,

' '
15) Un lactante menor presenta infecciones respiratorias recurrente~, que han respondido adecuadamente el tratamiento antiiirotrco. Al examen fsico destaca auscultacin pulmonar con esc@sos crpitos bibasales y auscultacin cardaca con un soplo continuo. El diagnstico ms ~probable es: ~' a) Coartacin artica ~ b) Trasposicin de grandes vasos ~e). Ductus arterioso ersistente

P'

~ ~

~~~~~:~:~:6~-:~!:~=~~~~~------

.. . --- - - - - - -- - - - - - -

--~-'--

--....

!'

16) Una muj r de 34 ~s consulta por clismenorrea marcada, que ha aumentado en los ltimos meses.
Se solicit na ecl)grafa que demuestra un tumor ovrico izquierdo de 4 cm de dimetro, guistico, con conten o en vidrio esmerilado. El diagr.1stico ms probable es: a) a . te folicular b ncer de ovario ~~ ~erpo lte~- _.JJf~ndomet efna e) Terato nio de 6 aos, presenta desde hace ugos meses. diarrea y baja de peso. asociado a piel seca, itis y calda del cabello. Refiere que presenta dlarrea acuosa, con gotas de grasa, sin presencia de a oposiciones. La etiologra ms probable es: '- a) Enfermedad infl~rnatoria intestinal b~ ~dasis _..., ~a1a2bs F&n intestinal d) lnvagi cin intestinal e) Est osis pilrica hipertrfica

~ ,..

~ ~

p>..

'

Un mujer de 35 aos, diagnosticada de hipertensin arterial crnica, en tratamiento con idroclorotiazida, con adecuados controles, presenta atraso menstrua.!, por lo que se realiza un test de mbarazo que resulta positivo. La conducta ms adecuada para el tratamiento de su hipertensin rt rial es: / Suspender .la.hidroclorotiazida e iniciar alfametil-dopa '-.../ ) Mante~r'r hidroclorotia~ida, sin neceei~aef efe etros meelioameAtes e) SusP, der la hidrocloritiazida e Iniciar enalapril d) S ender la hidroclorotazida e iniciar sulfate ele magnesio e) olicitar proteinuria de 24 horas para deGidir el hipotensor ms adecuado

19) Cul de las siguientes coondicionet; es .. recin nacido de trmino? a) Slmfrome de aspiracin de meconio ~ b) yardiopatfas congnitas ..-;C]Hipocalcemia d) Poliglobulia e) Cefalohematoma

m~ el recin nacido pretrmino que en el

><

~. N~~
~

20) Una paciente cursando un embara_3P~ semanas, presenta un retraso del creci~mento intrauterino, con feto creciendo en Rrce~Qu afirmacin es FALSA? _,/" ~ ~ ~) En caso de no haber complicaciones debe mterrumpirse el embarazo a las 37 semanas , ~ 'f_.Q_, \.V ~:~~~- ~ be solici r de arterias uterinas cada 1 a 2 semanas ......-.J y.~l.J._<-A-0 ~ r- _p .. La presencia de oligohidramnios es un criterio de interrupcin del embarazo_. ~: . ~,. ~ d) Deben realizarse 3 veces a la semana un perfil bioflsico o bien un registro basal no estresante - ~ yc () e) Deb. e indicarse reposo en decbito lateral izquierdo ~ ~91 ?( .. r,.;?/('0~~~~.)~~ . ~ ~\...J' t' 21) Una paciente presenta ciclos menstruales d 4\ as de duracin en cada menstruacin presenta ~ ~t' sangrado muy escaso, menor a 20 ce y de slo 2 s e duracin. Se puede afirmar que presenta: _, a) Oligomenorrea

5/

e
1111'

~ ~

-~=~ ft'1:olimenorrea e hipomenorrea


.

e) Oligomenorrea e hipermenorrea

22) Un ciente de 6 aos presen~ fiebre, cefalea y signos me~. por lo que se solicita una pune n lumbar que da salida a liquido purulento, con 100.0000eucocitos por mm3, de marcado pre ominio palimorfonuclear, glucosa baja y protenas elevadas. La tincin de Gram muestra e ceas gram negatiVas. El agente causal ms probable es: eningococo ~ Q., . b) Neumococo W (.A c.J....-..6 CJ-a.f"'r- ~ e) Haemophilus influenza bo. e:..iJl...o V['~' d) Staphilococo-- 00\:::'t):.::C::k.~ ~ ~ 4:?: : e) Listeria ~~ <=jfo.,- -\23) La ausa ~s frecunte de constipacin en la edad peditrica es: a) E ermedad de Hirschsprung b olipos juveniles ..-))Zffermedades inflamatorias intestinales ~~los hbitos -e) Giardasis

5,.

24) Un recin nacido de tnnino, Ictericia,- desde gue actualmente llega hasta los muslos. Al nacer,peso 3.600 g y actualmente pesa 3.650 g, alimentndose exclusivamente con leche materna. No presenta otros sfntoma!S y el examen fisico no aporta mayor informacin. El diagnstico ms probable es: ~) Ictericia fisiolgica b) Ictericia hemoltica por incompatibilidad de grupo clsico ~~ 1_91Micia hemoltica por incompatibilidad de grupo Rh >'r:~!ericia poj-leche materna-? \"\~ 0-- ~ ~'~ e) Hipotiroi smo congnito 25) U paciente presenta alll!norrea secundaria a una falla oyrjca prematura de origen autoinmune. La rueba de progesterona y la prueba de estrgenos ms progesterona resultarn respectivamente: .-\Positiva y positiva b) Positiva egativa ~egati\l y positiva d) Neg va y negativa e) No odrn ser realizadas 26 Un recin nacido de 36 semanas de gestacin, presenta dificultad res~iratoria~e~ ~e nacer. Se icia oxigeno y se realiza radiografia de trax que muestra bflQS pnlmoares prnmtrumtes, aumento de la trama vascular pulmonar y leve derrame pleural bilateral. El diagnstico ms probable es: a) Enfermedad de membrana hialina b) Cardiopatia congnita e) Hipertensin pulmonar persistente dt,Smdrome de aspiracin meconial veJ Taquipnea transitoria

~ d~ ~~santa

hace~J.,

27) Una paciente de 34 aos, cursando embarazo deJ1 semanas inicia metrorragia,asoclada a dolor _ /hipogstrico. Se realiza ecograffa transvaginal que demuestra saco gestacional inCJemne, con . ~ visualizacin de latidos cardiofetales. El tacto vaginal muestra, cuello completamente borrado, con cr-d~in de 3 cm. La conducta ms adecuada es_: . ~~'\ _.,l , ~t~ ~ ~nviar a domjpilt indicando reposo absoluto ~ S\.rv CJ'-".' "'b) Realizar c~rlaje de emergencia ~ .-<:-qL-0 e) Adminisjrr progestgen~s y reevaluar en 48 horas ,. 0 ~A. <f \ ~d) HospiJlizar, administrar analgsicos y tocoliticos ~ U---r, .~ 0 "" "n . . ~ / 1 ~ Re H: - ~ ~ ~() G\r'v'

.-,

~
~ ~
~ ~
.

Qu examen NO pediria al inicio del estudio de una pareja que consulta por infertilidad de 18 eses de evolucin? a} Ecografa transvaginal b) Espermiograma ~ e) Seguimiento foliular ~ d) Test postcoital rAisterosalpingo~rafia ._..., :L.~

c.

('
El punto de reparo de la p~~regma es: ) La fontanela posterior ....>.> ~) La fontanela anterior e) La frente d) La nariz~~ e) El mentn ~ro , .2
~-~......,.. .,.!."30)6.Qu ' dieament&~sr.de..eleG.Gin,.p ara~a,.profilax~deJnfe(l(;iGfleS-idel.t-ra&to..udaario,..~n"un.nio -- --

~ ~

r'

fA

~
\

(A

lujo vesicoureteralleve? itrofurantoina b) Eritromicina e) Ciprofloxacino d) Gentamicina e) Amoxici_ljna +cido clavulnico


31)

,..

U~aciente de 6 aos pres~nta edema facial y de extremidades inferiQtes. Al examen fsico

~ (!'

_...

pre enta signos vitales normales para edad. Se solicita estudio de orina com eta ue demuestra mdros creos,' sin hemtauna 'm leuco,:ID!rfa, con proteinuria de 241ioras 2; La creatinina pTasmtica y los electrolitos plasmticol3 son normales. E agnos co mas pro able es: 1 a) Sndrome hemolitico urmico x . . ,., .f . ._ .t'\~ ~ b) Enfermedad-de Berger ~ )~'-J--u e) Glomerul t'efritis aguda postestreptoecica r- J d R' n liquistico ,.. 1,..._?.-(1):.{ 1 r~ edad por cambios minimos ~ _ : -.n . !\ f"\
~ '(.):l 'J'~ ~':)

sil

,.,

\JI/""'

Desde qu edad deben J!.lic~ar la im~ta de alimentos slidos los nios alimentados con lactancia aterna exclusiva? /a)~ meses ~ C/'

~arneses

e) 9 meses d) 10 91eses e) 1 ,.meses ) En cul de las siguientes pacientes aconsejarra el uso de anticonceptivos por vfa transdrmica en ugar de vfa oral? ..a) Mujer con cncer de endometrio b) Mujer con a ecedente familiar de cnce!r de mama ~r co upus eritematosos sistmico ~! ~~~~{:r n hipertrigliceridemia_,. e) Muj purpera, dando lactancia La enfermedad de Kawasaki suele cc>mplicarse con: Meningitis asptica b) Trombosis venosa e) Artritis deformante d)j.Jlleltis Aneurismas coronarios

A
A

...... ~.}'('a endometriosis es una causa de dsmenorrea y de Infertilidad. Suele diagnosticarse mediante: ~ ~a clfnlca b) Ecografla transvaginal e) Biopsia de endometrio d) laparoscopja ) Ca-125

, / . ~)~a mirada y presenta sonrisa social -~ Fr

36) Qu opcin caracteri2a mejer a un nio de 10 meses de edad?


f-'o-

e sienta por sf mismo, vocaliza (ag) y es capaz de tomar objetos y pasarlos de una mano a la otra e) Sef~per s1 misma dice ceFea Ele 10 1,alab1as y se mUestra lhdlfeieute a la presencta de extranos ~~-Se para por si mismo sin caminar entiende el concepto de "no" y llora frente a Jos extraos ....../' e) Carnina sin apeye y eliee celta de 10 parm:mm-

y-'(a. ~
. q

\L-~'

,.,
~

~
~
~

:~~~--.~)--~~~~~~~~-~~~--~------~~-----------~ ...
8) L "nducta ms adecuad ante un lactante cursando una diarrea aguda por rotavirus, con de adratacin severa (mayor al10% del peso corporal), es: ,... ... () C'\ Hidratar con suero fisiolgico por va endovenosa l~ :a:ls d1arreca, segn Plan A el d} Iniciar tibitiGes y sales de 1 lid atacron oral e) lnici sales de rehidPataci oJal e Indica vaet:Jnacin contra el rotavirus

..-,

~~ :~~::~
39

~= ~~:~~=~~~

::: ~==~::: :~~5ll

,.,
~
~
~~

os corticoides prenatales han aumentado considerableme!nte la sobrevida de los recin nacidos aturos. Reducen la Incidencia de todo lo citado a continuacin, EXCEPTO: sis neonatal } ermedad de membrana hialina / e} Hemorragia periventricular / d} Muerte neonatal precoz / / e} Enterocolitis necrotizante /

'~
~

,. primera causa de insuficiencia renal aguda, en la edad peditrica, es: alforinaciones nefrourolgicas b) Glomrulanefritis C.~ ~dro hemolitico urmico _JJf:,rerr al e) N rosis tubular aguda
o

4
~

~
~

) La etiologa de las a) Re

b) ~presin
~Hipoxia fetal

esaceleraciones tardas en el registro estresante, es: res1 n de la cabeza e 1 del cord6R I::IHibilical

~
~

~
~

d) Flujo reverso en distole de las arterias umbilicales e) Aumento del retomo venoso fetal, producto de la compresin de la placenta

V
'\.
o

~:::Jea:~. BPT y pouoora1


e) La va e na DT --

42) Segn el programa nacional de vacunaciones, al cumplir un ao, los nios deben recibir: ra) Las vacunas pollo oral

0'2!

(\~

._..d) Las vacunas DPT, polio orai,~HB

(_. \.-(_ 1\'1...

~ 'Pf'\

,..,
~

43) U /mujer de 34 aos, cursando embarazo.de 35 semanas presenta poliaquiuria, sin disuria, que .la hac r al bao cerca de 15 veces al_da. Se-solicita sedimento de orina que r.esulta normal y urocultivo q li.es sita Aeg?tttvo. La 'conducta ms adecuada es: Iniciar ce dfxilo en una dosis diaria b) Solicit serologa contra mycoplasma e) sor ar urodinamia c;t) inuir el consumo de agua ranquilizar a la paciente, envindola explicando la normalidad de su condicin
o

~
~
~

~
~

44) Cul de las siguientes mujeres tiene mayor riesgo de PllUiftntar cncer de endometrio+ a} Mujer con promiscuidad sexual y antecedente de procesos inflamatorios plvicos tratados -<> ~ujer obesa. con diabetes mellitus 2 e hipertensin arterial e) Mujer postmenopusica, con uso de terapia de reemplazo hormonal por 1O aos d} Mujer con endometriosis e} Mujer c6 mtolilatoss uterina sintomtica

~
~

-v,.

,. . \ ~)./\ . t\0
1"'"'

~
~

~ ~

~
~

S-'
~

~
~

e
~ ~
~

45) D nte la primera semana de vida los recin nacidos pueden presentar una baja de peso de m o: a 00 gramos ) 100 gramos e) ~ del peso de nacimiento ~!1 O% del peso de nacimiento e) 20% del peso de n eitfieto
--..... " ----- - - - - - ... -

-~ ~.

__ , - ...

---~~---~-~~~~~---~--~--~.-~-!!"'.-........ , ...._-. ... -~- ----.-.-..-...-...-..-- - . -

-- ~-----.-- 9---- -~---~, . . ' - .. -- . __ - - .. ,

~
(A

~ ~
~
($'
47) Son causas de oligohid~ios, EXCEPTO: a) Insuficiencia placentaria / . . . .. :. ~ b) ~tura prema ra de membranas /\ ..~ \........:... c_9....,-o~-- ~Diabetes itus gestacional ~ ~vd) Precia a e) Malf aciones renales /
J

e
/

~
~~

f:'
~
~

1primer signo de Vello pbico b) Menarquia ci.l(otn mamario ~ Adrenarquia _ e) Aceleracin del crecimiento

desarrol'~~n las nias es: ~-------

e}
/'

~ ~-

-O

y,
.Y

-{)

CT

~ ~ '-.

>(
,

.....-

~ y- Uf.\
~ \)-~

~ <tl~u

J .,.

~ , rutirario sin poder identificar un foc;o infeccioso, por lo que se realiza arnnjocentesls que demuestra ' '2,'- 31 leucocitos por mm3; glucosa de :Z5 mgldl. A la tincin de Gram no se visualizan bacterias y el 'I , 1 _,g~') 'Cu,ltivo de liquido amnitico resulta negathcp. La conducta ms adecuada es: ~"-~"":'""

:i; 38,2C, sin presentar otros srntomau ni otras alteraciones en el examen flsico. Se realiza estudio
) Descartar el foco utenno y continuar el estudio de la fiebre
l'~ rt
n1c1ar an 1 10 eo y mantener una conducta expectante ~mini rar antibitiCOS y COrticoideB e interrumpir el embarazo en 48 horas el) lnte mplr el embarazo bajo cobertura antibitica e) Re izar cesrea de urgencia
JY
l...

49) Una mujer de lO-aos, cursando embarazo

de~' presenta fiebre persistente, hasta

0\ . )

~ C..O 0'-",C/

~CA

\.-, ~

l{).\)

u\

J.

50 Un nio llora y hace berrinches cuando es llevado al jardln Infantil, quedando muy ansioso y rando por largo rato luego que su madre se va, siendo dificil de consolar~ La madre suele irse muy ansiosa y preocupada del estado dEt su hijo. El diagnstico ms probable es: a} Trastorne eJ:"osicio11ista desafia 1te .... o , b) Trastorno de conducta r ~ ..-Lt ~ o ~\ e) D~resin infantil .t~ , 9,...0. ~ ~ \./--;y".JIYfrastorno por ansiedad de separacin -1 -t(p \. CA/.... er--c94~ ~ e) Tra orno por dficit atencional .') ~ -" 0 s anticoncptivos orales con microdosis de estrgenos se car~a11,.por: resentar myor incidencia de goteo (spotting) que los de macrodosis b) Present mayor riesgo de cncer de mama que los de macrodosis e) Prese ar mayor riesgo de trombosis venosa profunda que los de macrodosis d) Ten una mayor efectividad en la prevencin de embarazo que los de macrodosis e) S inyectables 51

ov--J-...L.t. o-~- \....A

V:

~ El tratamiento de eleccin de una laringitis obstructiva aguda moderada es: a) Salbutamol inhalado b) Salbutamol y corticoides inhalados r e) Adrenalina racmica inhalada y corticoides inhalados ....-cfJAdrenalina racmica inhalada y corticoides orales e) Corticoides inhalados y corticoides orales

~
~
53) Un recin nacido d 4 dfas de vid. , nacido preJQOFlturo a las 33 semanas, presenta desde algunas hora dificultad para regular a emperatura corporal y debilidad para alimenta!Jie. Al examen presenta tono muscular disminuido y debilidad del reflejo de succin. El diagnstico ms probable es: a) ParaiTsis cerebral Hipoglicemia o e) Hi ocalcemia ~~resSe sis --?tf>
'

~
~ 7

~
-

--,.-~~m~~;;,nrman-IIPBIBai'ROrt~---...------~~- . ------.~-------~_....~---- ...--~~--- 0 -0


54) Una ciente en edad frtil, Se{ualmente activa, presenta dc~sde hace una semana dolor hi o trico, asociado a dispareuma profunda. Al tacto vaginal se constata dolor a la movilizacin cerv. a y a la palpacin anexlal. El diagnstico ms probable E!s: a ncer de cuello uterino ) Neoplasia intraepitelial ~~ ~cer de endometrio ~~~cesas i amatorio plvico e) Endome osis 55) L cardiopatia congnita ms frecuente es: a) omunicacin interauricular
Y?~menoval
~~~unicacin interventricular

a) Dueto arterioso persistente e) Coartacin artica


1

o~,

56) Mujer de 3-aos, C.!,Sarizada antEmyr, cursando embarazo deTS9 sefR&Ras, con ,polihidramnio~, de causa no precisada, presen~ sahda de abundante liquido ciare p'ru:.los genitfles. No presenta ~ / dinmica uterina yola especloscopfa demuestra salida de liquido transparente por el orificio cervical . ~ extern. Se p~lpa el feto en podlica y no hay modificaciones cervicales. El registro b a s a l @ 2 o estresante es re~ctivo y el perfil bioffsico es 10/10. La conducta ms adecuada es: ~) Administra;,.atibiticos y corticoides y mantener una conducta expectante b) Administrar corticoides endovenosos y antibiticos e interrumpir ~1 embarazo en 48 horas ~mi istrar corticoides, antibiticos y tocollticos por vla endovepllSa d) 1 , ~

(YfQ
rs?- -

ov\
cP

e)

'S'J.

7) Respecto a la hepatitis A, es verdadero que: a) No existe vacu~ncin e b) Set~vlavertical . -"-1' "\ e) SJ..Eiiagr.astico de certeza se realiza con la deteccin d~l antlgeno de superficie ~~- / arEl tityripo de protrombina es un muy buen indicador de la severidad de la enfermedad "V' e) sysre presentarse c~n ictericia y elevacin ~e las tr~~minasas cercaJI11fa""~-aanldmo~b~le~dee-tclo~s~u~n:a!tl11o~r~ers-s-19ntfloftFffl:w:ta~lleess- (\:

~~~

(~ v-0~

\-.

-s;>

~-

~J~~ ~

A~ax' ~
~

J:

Una mujer de 3.~oaos se realiza una mamografia, que es lnformada como Birrads O. La conducta ms adecuada eos: ~--) Repeti~la afuografia en un ao b) Repetir 1 mamografla en 6 meses ~eali~ r ecografia mamaria d) So "Citar TAC mamario e) rivar para biopsia por puncin

f/)

59) Un nio d~ presenta fiebre hasta 39! y decain:al&flto, asociado a la aparicin de mltiples ulceraciones ~vidad oral, comprometiend~ el paladar blil..DJ:Io paladar duro. lengua. labios y encias. Adem~ s: palpan varia~. a.(:lenopatfal cervicales sensl~s. El diagnstico ms probable es: a} Herpangina ~ o.'-1i l-6'\ ~ c:&:z . b) Mononuecleosis infecciosa ~rimoinfeccin herptica

~a

wariRgoamlgdalitjs por anaernhjos

60) Un ujer se realiza un Papanicolaou, que es informado como muestra NO satisfactoria. La con cta ms adecuada es: a) ontrolar PAP anualmente )J) ~icitarft~tudio de virus papiloma humano :_wRepeyrPAP de inmediato d) De ~ar a colposcopia e) rivar a conizacin

61) En promedio, Cunto mide un nio de 12 meses de edad?


a) 37 cm b) 50 cm ~ c}J>2'"cm
~75cm e) 90 cm

~ ~ ~

~
~

~ lnter e)

62) Una paciente cursando embarazo de..:3..6..semanas de gestacin, consulta por disminucin de la perce~cin de los movimientos fetales. Sie solicita registro basal no estresante que es informado como no reactivo, luego de 40 mmatos de monitorizacin. La conducta ms adecuada es: a) Enviara domicilio ~ender el registro por 60 minutos ms ~. - e) Solicitar ografia-Doppler de arterias umbilicales

----:::::?"<
pnes

-vO

mpir el embarazo por la vla ms e>,pedita

.~ ~

I'...:I:" A. , rJ

el...__

-t0 Or ';'c}-o ~ '-1..._---.--.-v

~,

11e .-)~'-{ \

t.~")~~

.-k

f:'
~

~
~

(.Qu alternativa es adecuada para alimentar a un lactante de 3 meses de edftd, cuya madre ya no e continuar amamantndolo? eche purita al7,5%, ms azcar al2,5%, ms aceite al1,5% b) Leche purita al 7,5%, ms azcar~,, mas cereal al 3% e) Leche purit~.l 7 ,5%, ms azcar al 5%;-tfls ce1 eal al 1,5% d) Lec~hpu~(a al10%, ms azcar al2,5%, ms aceite al1,5% e) Leche rita al10%, ms azcar al5%, rns cereal al3% . nio de 7 aos presenta fiebre, decaimiento y un exantema polimorfo, generalizado, con las, vesfculas. pstulas yOCCostras en la cabe21a, tronco y extremidades. El agente causal ms p bable es: _ 11 tt . _; ) Streptococcus \J c..;..._r b) Enterovirus e) Virus herpes 6 d) Parvovirus 819 ~irus varicela zoster
64) p

CJ.J'-

v-

~
-

J A

..
\

65) Una paciente de 34 aos, cursando su tercer embarazo, de 30 semao..as, inicia metrorragia. asociado a p_ol.or abdominal intenso. Al examen ffsicQ.se palpa el ~:itero dolorosa e hipertnjco. El monitoreo fetal demuestra variabilidad disminuida, sin otras alteraciones. La conducta ms adecuada es: ___.., ~drnini~trar e fuoides y mantener una conducta expectant.e o O~ \ . b) Administra orticoides e Inducir el parto :m 48 horas, con misoprostol ~\\"~ e) Realizar rueba de trabajo de parto ~ ..;!,_) Solici r un perfil biofisico y decidir conducta segn hallazgos . ci-""" ~) 1 nt m r <pedifa

., ?'

Cul de las siguientes es una~cin de hospitalizacin n un nio que sufre un golpe en la beza? . a) Hematoma del cuero cabelludo de 4 cm ">r-. b) Amnesia del momento del golpe e) Prdida de consciencia de segundos de duracin, inmediatamente despus del golpe ~~useas ~uspecha de maltrato infantil .

~
~

~
~
) Los miomas que con mayor frecuencia presentan sntoma~; son: a) lntramurales ...... b) Subserosos ) ediculados Submuc:>sos e) Capsados 68) a

~
~

mujer d'!, 24 .aos, con ntecedefs de promiscuidad :sBU:al, 1nlc1a1 do1or abdominal, asociado
ia. Su ltima menstruacin fue hace 2 meses. Al examen fsico se constata

-,
~
~
~

~-...... m~m.:.:..H~g, FR: 16x', con dolor a la palpacin hipoglistrica, sin signos de irritacin peritoneal. Se

FC: 110x'.-PA:

realiza tes Cle embarazo que resulta positivo y se realiza una ecografa transvaginal que no visualiza saco gestacional. Se administran analgsicos y cristaloides. La conducta ms adecuada es: ~ a) Solicitar beta HCG plasmtica y administrar cristaloides endovenosos b) Controlar en ~a horas con nueva ecografa transvaginal h.. e) Iniciar metq.trexato , 1 '"' ~ealizar tgrado uterino '<.., so er quirrgicamente de inmediat n recin nacido de 5 das de vida presenta l@rias pstulas pegue~s, con base eritematosa en la ra, el tronco y las extremidades. Se aprecia de buen aspecto y el resto del examen fsico es normal. / E_!jliadnstico ms probable es: ~ Eritema t,xico b) Varice! ,1eonatal e) Herpe neonatal d) Sfil' congnita e) E ntema sbito Los principales beneficios de la terapia de reemplazo hormonal son: ) P vencin del envejecimient{l prematuFe y GQRb:ol de las sinto1+1a8 "asomotQr:es elimatricos ntrol de los sintoma~...v-S_Q_JTIO~ climatricos y prevencirJ.Jj~ !a osteoporosis ......,/ ) Prevencin del envejecimiento premaltmr y de la osteopo~- d) Control de los sfntomas vasomotores, prevencin de eventos cardiovasculares y prevencin de la osteoporosis e) Control de los sfntomas vasomotores, prevencin del envejecimfento prematuro y.prevencin de eventos
cardiova~culares

~
~

,..,
~
~

~ ~

~
~
~
~

~
~
~

~
~
~
~

~
~
~

Alli)
~

~
~
~

""

~ ~
~

..,

~
Prueba 111 CIRUG(A, SALUD PBLICA, PSIQUIATFt(A Y ESPECIALIDADES 1) Un nio de.!J!os presenta retraso_de.llengnaje y parece no escuchar bien cuando .!ie.le...habla. Actyalmente dice cerca ~!_.100 palabras, con mltiples errores en la ronunc cin, mayores a los esperados para su edad. A la otoscopia se observan tfmpanos os bilaterales. La conducta ms ecuada es: ----- - ~ pbservar evolucin .rbfSolicitar impedanciometrfa / e) Iniciar antibiticos d) lndic!3r"audfonos e) D~rivar para cirugfa de implante coclear

~ ~ ~

~ ~ ~

A
J

~) Los sesgos de seleccin

2.l'I aumento del tamao muestral es til para disminuir:

~ ~

b) Los sesgos de informacin ~tk:os sesgos por variable contundente ~ ~1 error aleatorio o tipo 1 ....e) Los sesgos de medicin 3) Una mujer de 24 aos se separa de su marido, con quien llevaba slo 2 aos de c~da, evolucionando con t~eza, falta de mijyacln, ~bilidad emocional e iteis de muerte.;Ha dejado de ir al gimnasio y de juntarse con sus amigas, porque se siente ~ver_gonzada de su situacin y ya no le genera bienestar el relacionarse con otros. Adems ha tJaJado considerablemente..su.r:endimiento en el trabajo y refiere que le es muy dificil concentrarse. Los sintomas se han vuelto especialmente intensos en el ltimo mes~ porro que decide com~UJtar. El diagnstico ms probable es: 1

(!'
~ ~ ~
~

\
....

~
~

: ~~~~:n;;:~-:~str~-s ~~~traumtico.>.:
) oslimia rastorno adaptativo epresin mayor

- -

~ ~ ' ..r-

_cvr r

,.
~ ~
~

x.. ~)

4) Cul de las siguientes alteraciones nspera encontrar en un paciente con cataratas? Alteracin del rojo pupilar b) Defecto pupilar aferente ;,( e) Midriasis d)-Anisocoria f) Leucoma 5) Un paciente presenta un tumor testicular derecho, que es confirmado con ecograffa testicular, por lo

.~/ que se decide realizar una orquiectomlariari['"la biopsia quir9J.g~ Informada como semi noma
~

e'
~
~ ~
~
\

puro y los marcadores tumorales demuestran alfa-fetoprotei.negativa; beta-HCG levemente. positiva. Adems, el TAC abdo~inal no muestra adenopatias retroperitoneales y la radiografia de 'trax es normal. La conducta ms adecuada es: ~) Realizar radi_9trapia retroperitoneal t=> b) Realizar q~iinioterapia . e) Realizarlnfadenectomia lumboartica (LALA) -~ d) Realizr orquiectomia bilateral e) So;;pechar teratoma maligno y reestudiar la pieza quirrgica -<: tiroidectomra total se clasifica como una herida de tipo: mpia b) L1mpia contaminada e) Contaminada d) Sucia e) Superficial

~
~ ~

'-.

!"
~

f!'
~

e
~

~
~

~
~
7) Un paciente de 35 aos consulta por aparicin de una lesin dolorosa en el muslo derecho, de rpido crecimiento en algunas horas. Al examen fsico se observa una placa de 15 x 1Ocm con centro ~ertico, rodeado de un rea viOlcea y por fuera una zona eritematos~. El diagnstico ms probable es: a) Pioderma gangrenoso clima gangrenoso asceitis necrotizante ...<. oxocelismo
e)~ueemdeesis ~

~
~
~

/ ""-

~
~
~
~

"

8) Un paciente cae de una bicicleta, cayendo sobre la extremidad superior izquierda, resultando con

un .-fractura supracondllea de hmero. Adems presenta anestesia de la cara palmar de los dedos m Clio, ndice pulgar, asociada a imposibilidad de flectarlo8:1Qu estructura se encuentra afectada ayor probabilidad? NeiVio mediano b) NeJVio cubitfll e) NeNio radial d) NeJVio musculocutneo e) NeJVio axilar

~
~

~
~

9) La probabilidad de enfermar en un perfodo de tiempo, se denomina:

) Tasa de ataque Tasa de incidencia ) Tasa de letalidad Prfasa de prevalencia e) Tasa de trasmisibilidad

~
~

1O) Un paciente de 25 aos vive con sus padres y trabaja espordicamente en diversos trabajos, sin embargo nunca ha'podido conseguir un trabajo estable, ya ue.e1 considerado como Ineficiente y Jo... -marcidam . :is , no tiene amlaos y en m re has ... extrao por sus com"Dal'ieros e o~asi~n~s- a a ...~es r u ma 1 DQs_y_br.uj~ue lo pers1gu~n y ponen a la gente contr!J!?EI diagnostico msprobable es: -.. a) Trastorno ,2tfPersonalidad paranoide b) Trastoll)P i::le personalidad esquizoide c))"rasto no de personalidad esquizotrpico __;tfTra orno delirante crnico

~
~
~

~
~
~

) Un paciente ingresa a cirugfa de urgencia por una heri~~netrante abdominal. Qu medida estarfa MENOS Indicada para el manejo an.estsico de este paciente? . "':::J a) Administrac n de oxigeno al 100% b~ducci . on drogas en.dovenosas ~~ ~entila on con amb d) Rela'acin con succinilcolina e) lnt acin ototraqueal La visin tubular es caracter[stica de: ) Compresin del quiasma ptico ~laucoma de ngulo abierto e) ~PFeAEiimiento de 1etina d) Degeneracin macular relacionada con la edad e) Edema macular
13) Cul de las siguientes situaciones .NO debe ser manejada de manera quirrgica? ~ / a) Insuficiencia mitral severa, secundaria a endocarditis bacteriana aguda, por estafilococo dorado .x...__ b) Estenosis artica, asociada a angina crnica y sincopes de esfuerzo _.. A Insuficiencia artica, asociada a marcada disnea y disminucin de la capacidad funcional / d) Estenosis mitral severa, con elevado gradiente transvalvular y rea de abertura valvular de 0,6 cm 1 ~ Endocarditis bacteriana subaguda, sobre vlvula artica bicspide

"'
~
~

"'
~
~
~

~
~

~
~ ~
~
~

St vt.< en . o ~\~~...-0 -\-\...!h..

~
~

.r.

"
~
~
~

~~

e
~
~

14) Una nueva prueba de deteccin de fenilcetonuria presenta un valor predictivo positivo d ~alor predictivo negativo de 100%. Con e!Jta informacin se puede afirmar que: ---a) La sensibilidad ser de 100% . 'b) La especificidad ser de 100% : ~ un paciente presenta un resultado positivo se confirma la enfermedad ~? ~o existirn faJsos positivos e) La prevalef1ci de la enfermedad es ms baja de lo esperado
/

s~>y un

f'
~

~ ~ ~
~ ~ fA

15) Un IJP.fu"bre de 25 aos es considerado como antiptico y petulante por sus compaeros de univep;1dad. Es muy envidioso y muchas veces se burla de quienes obtienen malas calificaciones 1 ndo_los de estpidos. Explica que sus compaeros lo aislan porque se sienten envidiosos de su igencia. El diagnstico ms probable' es: pisodio maniaco b) Jrastorno delirante crnico _.,ft}Trastorno de personalidad narcisista d) Trastorno de personalidad antisocial e) Trastorn.o de conducta ,. 16)

tra rse con:

L~ili-opatia obstructiva no complicada, secundaria a una hiperplasia benigna de la prstata, suele

~ ~ ~
(A

a) irugia Sonda Fo,ey e) Alfa-agonistas d) Beta-agonistas 1 el_~lf _Jltagonistas


17) n paciente presenta vrtigo desde hace una hora, lo que le difi~'-.llta) . ar a, sufriendo varias e das. Al examen ffsico se aprecia r'\lSla~ espontneo, ~ultiafrecciona que se mantiene nstante al solicitarle que fije la mirada en un punto, Al cerrar los o os presenta tendencia a caer acia cualquier lado, la marcha es descoordinada y presenta gran torpeza a los movimientos de las extremidades.,La conducta ms adecuada es: . r-.F a) Iniciar antiyertiginosos orales de manera horaria por 7 dlas 1(" J"-L~c)U b) lniciar~o.1coides segn horario y antlvertiginosos segn stntomas L~ e) Solicita examen de octavo par e iniciar tratamiento sintomtico con flunarizina d~~dic~ manibras de reposicin y antivertiginosos ;-: Hospitalizar y solicitar resonancia magntica de_cerebro

f!!!'

. \_ t. ...
~

~ ~ ~
~
~ (!!'

o-1'<-..r-->:::

.J'

~ 19) U

18 n paciente de 30 aos presenta dolclr lumbar muy intenso que inici al levantar un mueble y que 1 1mpide la mayorfa de los movimientos de la columna lumbar. Al examen fsico se observa muy dolorido, con signo$ vitales normales y limitacin de la flexin y rotacin de la columna lumbar, debido al dolor. ELexamen neurolgico os normal. La conducta ms adecuada es: a) Solicitar radiogrfia de columna lumbar b) SolicitarTAC8e columna lumbar e) Solicitar RMN de columna lumbar d) Indicar re6oso absoluto y relajantes mus:ulares .~) lnd~rcarINEs y reposo relativo . aciente presenta olvidos frecuentes y repeticin de algunas ideas, asociados a dific tad para realiZartus tareas habitual es. En el examen mental invierte ~_ru:les-eon normalidad, y ma tiene la memoria a largo plazo, sin embargo presenta varias falla_!.!.l2 las pruebas d~...~~ia . r iente. El examen neurolgico no presenta signos focales y slo de~t.~.. ~~ertatorpeza motsa, sm ignos cerebelosos. El diagnstico ms probable es: .... _, __ _ a) Envejecimiento normal b) Depresin .e) 'Enfermedad de Alzheimer d) Delirium e) Demencia por cuerpos de Lewy

de~65 ~~~

~
~
Una paciente de 45 aos fava la loza con un nuevo detergente, presentando a las 6 horas prurito, ritema y dolor de las manos. Al examen se observa edema, eritema, microvesfculas y grietas en a91bas manos. El diagnstico ms probable es: ..} Dermatitis de contacto alrgica . -0 ... ~ 0.. r ~ V'-.:~~ b) Dermatitis de contacto irritativa ~ ~ ~ c!J--o ~ ,.....__ -~ "-~ . c. e) Urticaria aguda u, ([)_0 4 , f ~~.,::f d) Dermatitis atpica e) Si,ndrome de Steven Johnson 2

~
~

r ."
.

~
~ ~

,.

1) Un paciente de 20 aos cae a nivel, apoyando la mano en e:Ktensln. Presenta dolor leve, manteniendo los m~iinientos de la mano, sin embargo al examen ffsico presenta Importante dolor a la alpacln de la tapaquera anatmica. El diagnstico ms probable es: ) Tendinitis d~puervein b) Fractura d rapecio . . -~Fractur~ e trapezoide .:2~CP-~\&Jt d) Fractu de trapezoide e) Frac ra de la apfisis estiloides del radio 22) e seleccionan 1.000 pacientes diagnosticados de insuficie-ncia cardaca severa, de los cuales 500 s n tratados con enalapril, atenolol, furosemida y espironolactona, mientras que a los otros 500, dems de recibir esos medicamentos, se les Implanta un marc:apasos con reslncronizacin. Se evala la _mejor[a de fas sintomas, la _ocurrencia de arrlmlli!s. y las sobrevidas promedios en am6os grupos. A qu tipo de estudio corresponde? ------.-------, a) pe prevalepcia NltEnsayo /c;:llhico . ~-- =:t (_ -\ j\1\ ~\.(.. l)'eohoijes '<. \' d) Cas9s y controles _ / e) E sayo de ~mpo ...\) q~_r\.K. ro..~ 1 ~<..fiJ ~~d-o c9-P P'-e\r-lA...{:_'-1') El litio suele ser el estabilizador del nimo de eleccin para la mantencin de los pacientes con rastorno bipolar, excepto en el embarazo, por ser teratognicc. La Intoxicacin por litio suele manifestarse como: a) Midriasis, palpitaciones e hipotonla b~iosis, temblor e hipertonfa ~ ~emblor, slntomas gastrointestinales y confusin d) Anuria, confusin y depresin respiratoria e) Tetania, depresin respiratoria y taquicardia 24) Una asesora del hogar presenta una amputacin de la falange distal del dedo medio izquierdo, mientras cocinaba en la casa en que trabaja desde hace 3 ao~;. Tiene contrato vigente y est afiliada a una AFP y a FONASA, en categorfa C, sin embargo no est afiliada a la Asociacin Chilena de Seguridad ni a la Mutual de Seguridad. Los costos del tratamiento y rehabilitacin, deberian ser financiados por: ...._/ a) FONASA ~ b) La Asoclaci9n 'Chilena de Seguridad o la Mutual de Seguridad t~aAFP .,. ~) El em~tedor
~~tr ~~~

,..,
~
~
~
~ ~ ~

~
~ ~

~
~
~

~
~

~
~

,.,
~

.._,

~
~
~

na paciente de 28 aos presenta desde hace 6 meses plac:a ritematosas con descamacin fina, li era mente pruriginosas en el C...Y.!!:_5) cabelludo y en las cejas, recidivantes, con per o actividad. El diagnstico ms probable es: , a) Urticaria crnica b) Eccema numular e) Dermatitis de contacto _...d)"ermatitis sebC?rreica e) Psoriasis

~
~

'.

/
n

./\. "
~'-

!!"

el glaucoma agudo, usted espera encontrar todo lo siguiente, EXCEPTO: umento de lflonicidid del ojo comprometido iosis que no responde a la luz < 'e) Vmitos// .,... d) Ojo roj profundo ""' e) Dolo
27~na paciente de 65 aos consulta por fiebre y dolor en el hemiabdomen inferior, mayor a la

"

/!"

iz ierda. Al examen se aprecia dolor a la palpacin abdominal, especialmente en la fosa iliaca .quierda, asociado a resistencia muscular. Qu medida NO tomarra frente a esta paciente? a) lndicqr rgimen liviano ~ S-k. ~::l.J. ~O""\ tA. cstJ ~ DlA t ;-#:' r b) Soli :ftar TAC de abdomen y pelvis/ 1 -- e) lni ar antibiticos de amplio espectro..,,., Jf alizar colonoscoia de urgencia e) dministrar analgsicos 8) Una paciente consulta por nerviosismo, palpitaciones. diarrea y baja de peso. Ha presentado en 2 oportunidades palpitaciones intensas, ilsocladas a disnea y sensacin de m.uerte. Al examen ffsico destaca piel fina y hmeda, temblor fine, de extremidades Inferiores y se palpa pulso lrre~lar. El . diagnstico ms probable es: a) Trastorno de pnico b} Trastorno de ansiedad generalizada e) Trastorno conversivo Jf}flpertiroidismo e) Trastorno por somatizacin 29) Se desea comparar el valor del hemato,c.rito de dos grupos. El primero est compuesto por 100 pacientes insuficientes renales, tratado!~ co hierro endovenoso y el segundo est formado por 150 /pacientes con insuflcienGia-w:&Ra~dos con eritropoyetina subcutnea. La mejor prueba para ~ de~rminar l,"""fillJnificancia estadfsti~ es: ,B'{Chi cuadrado "t" de Student .. e) Anlisis de varianza d) "r" de pearson e) Prueoa de distribucin normal

f'

r-~

~ ~

r'
~
~
~
~

.A

<

~
~ ~

f'
~

n paciente de 70 aos, sin antecedentes ni hbitos de importancia, consulta por vmitos randiales asociados a baja de pe~~: Refiere que vomita luego de .c~da comida y qu~ ?ste cuqgro a ido empeorando progres vamen e, nsde hace 5 mese~ Al examen ffsiCo ..se aprecia pahdo X enfi.ruluecido. La conducta ms adecuada es: a) Iniciar prokinticos b) Iniciar metronidazol e} Solicitar TAC de abdomen y pelvis d) Solicitar ecografla abdominal ~olicitar endoscopa digestiva alta
~s

ft
~

r'
~

31) Un paciente de 60 aos, obrero de la construccin, fumador habitual, consumidor de alcohol, es traido al servicio de urgencias, ya que s3 presenta muy_agitado,. con alucinacjone~isuales de . .J V' s~oientes y lagartijas que lo atacan yjrP.aJl.RQL~~~erpo. Al examen fsico se aprecia agitacin ~ I/)!~\. 0 psicomotora y marcado temblor. La conducta ms adecuada es: .. -1 ~/ . ...j . , / a) Adminstrar haloperidol10 mg intramuscular, solicitar exmenes generales y un TAC de cerebro'-') ~-.ll.__).r->yJ /\ b) Administrar haldol 1 mg endovenoso e Iniciar estudio de patologla infecciosa _ ___./ \ ,-~ ~dministrar suero glucosado y tiamina endovenosa / _, d) Solicitar exmenes generales e iniciar clozapina oral ->)e) Administrar diazepam endovenoso y solicitar exmenes generales

f'
~

~
~

32) El tratamiento de un tumor renal slido de 5 cm en un paciente previamente sano, suele ser: a) Nefrectomia radical ms quimioterapia postoperatoria b) Nefrectomfa radical ms radioterapia poBtoperatoria {'-JO V- G OJ ~) Nefrectomia radical rtl) Nefrectomla parcial ms quimioterapia p:>stoperatoria e} Nefrectomia parcial ms radioterapia poBtoperatoria

r<S_

'{-A-::\-

~.,. . '"

c::r""

(:"

33) Un paciente sufre atropello, con un i ortante golpe en el pecho. Evoluciona con dificultad respiratoria, que requiere de oxf eno 130 para alcanzar sat~iiCIOhes arteriales normales. A~ examen se est ~o, no.rmo enso, con yugulares normales y trax slmtrll:tr.Secmscultan itost~laterales, may~res eef campo pulmonar izquier~~ 1:1 diagnstico ms probable es: emo rax ~ ~ b} NeumatQrax.a-leflsiA ~- e) Neumotrax ogrmoteRsiva . ~) Contusin gulmonar ,

~
~
~

e) Tapo/.lnto cardiaco
34) Un aciente de 35 aos presenta una lesin pruriginosa en lla zona posterior del cuello desde hace
algu os meses. Al examen se aprecia una placa deJ cm de dimetro, eritematosa, liguenificada, con d carnacin fipa, algunas costras Y. signos de grataje. El diagnstico ms probable es: Dermatitis j:l contacto
e)~~><

~
~ ~

~iquenj.mple
d=)=rFrQO e). . i&- )('

) Cul de las siguient-es valvulopat(as es especialmente riegosa, desde el punto de vista en el m~nejo anestsico de un paciente? ~ ~~tenosis artica .. b} Estenosis mitral e) lnsu~iencia tricuspidea d) ln~ciencia artica ) 1 suficiencia mitra!
c~iovascular,

"" ,-,
~
~

"'")
~

Qu institucin representa al ministerio de salud en las dintintas regiones del pafs? ENABAST ONASA" e) Municipalidad _dYSE MI . e) Se icio de Salud 37. Un paciente de 25 aos, mientras jugaba ftbol, sufre varo t=orzado de la rodilla derecha, olucionando con intenso dolor. Al examen presenta dolor muy intenso a la palpacin de la cara ~teral de la rodJIIa derecha y al repr?ducir ~l.var~. La lll&Riobra1-de Appley !!..!'~ativa.L~I igua!_ce:!~_los .. __ s1goos de cajn y los bostezos. El diagnstico mas probable e!J: a} Lesin meniscal aguda b_}endin_!Ji6 de la pata de ganso ~s~uJ,Rce de rodilla ... _. ... ~d} Ro}J:I_ra de ligamento colateral r--> ~" e) ura de rodilla

~
~

~
~

~
~
}

) Una mujer refiere q~e presenta rinorrea y ob~tuccir:a nasal-diaria, de-predominio nocturno y que se exacerba cuando limpia la casa. Tambin presenta estornudlos frecuentes y prurito nasal. Qu tratamientos rec~rnendaria para la patologfa descrita? 0 a) AntihistaJ11inic;;,;;s H2 orales y corticoides orales. h 0 r--"{ b) Desconge~tinantes orales y corticoides tpicos ) . e) A!}tihistlJlifnicos H2 tpicos y corticoides orales .- / ~z.n1ihit~fnicos H1 orales y corticoldes tpicos ~).q. \~Yl ~/~ e) D ngestionantes tpicos y cortlcoides orales el, \'

iJ '\.

pacientes con trastorno obsesivo compulsivo, que no es adecuadamente .tratado, suelen mplicarse con: /a)..fog'orafobia
~Depresin

e} Esquizofrenia d) Trastorno bipolar .e) Trastorno de angustia

r
40} U aciente de 30 aos presenta un aumento de voll;l!!len en el antebrazo izqui!Ulo-deA..cm, de di tro, no doloroso, de muy lento crecimiento, en 5 aos. Al examen fsico se pal~g_ 1 iel, de consistencia blanda, bordes bien definidos y regulares. La piel se moviliza fcilmente sobre icho tumor y n9.- presenta alteraciones macroscpicas. El diagnstico ms probable es: . a) Celulitis /, b) Sarcoma d partes blandas e) ~elano nodular ..d)Lipo e) Qui e sebceo

"
~

"
f:
~

t:'

"
f!'
~ (!!'
~

n paciente de 23 aos presenta m1~ulas hiP-opigmentadas en las mano$. cerca de los ojos y de-laca, que apar:ecieron.l'@~e y:~ eses y qu!_ han ido aumentando de tamao. No presentan dolor, p~ito ni signos inflamatorios. El diagm)s'fco ms probable es: J.l)'Vitiligo .. b) Pitiriasis alba e) Pitiriasi~ersicolor d) Der itis de contacto e) Ps asis gutatta
41

'!'
~

Cul de las siguientes ) Cuerpo extr~o tarsal . ><' b) Contusin cular .... .P)-Tlrida ppebral transfixiante d) Dacri istitis aguda e) He orragia subconjuntival

alteracione~~e derivacin al oftalmlogo?


?'
~ ~

~ ~ ~ fA
~
~

43 Una paciente de 1~~ presenta ba.ja de peso, diarrea y amenorrea desde hace algunos m~s. s muy delgada, pesando ;40 kiloaramos., sin embargo ella se considera obesa y por eso vomit-. despus de ser obligada a comer. La madre la sorprendi usando lactulosa y sibutramina. El ~gnstico ms probable es: /a) Anorexia nervosa ~ ~~__, o \f-..jrlo \-....:._,U b) Enfermedad celiaca e) Esquizofrenia d) Trastorno obsesivo compulsivo e) Bulimia '? 9 r d--t ~

~ ~
~ ~

e cuello uterino, sin embargo SEt sabe que las mujeres que presentan actividad sexual con parejas tienen ms incidencia de cncer de cuello uterino y que tambin suelen ocupar rconceptivos orales con mayor frecuoncia que aquellas con pareja nica o sin actividad sexual. Con a informacin, se puede afirmar que: -- - No se puede determinar si existe relacin entre el uso de anticonceptivos orales y el cncer de cuello rino . b) El efecto carcinogniav;tlos anticonceptivos orales sera mayor en aquellas pacientes que mantienen actividad sexual con va~parejas _p1 La actividad sexual con varias parejas actuaria como variable de confusin d) El uso de anticonceptivos orales no se mlacionara con la aparicin de cncer de cuello uterino, ya que ste .... y es causado por el virus papiloma humano e) Aquella.s mujeres que mantienen varias parejas sexuales no deben ocupar anticonceptivos orales, ya que aumenJafan an ms el riesgo de presentc1r cncer de cuello uterino X"
vari~

-;.-i 44) U~e ~. lo pretende determinar la relacin entre el uso de anticonceptivos orales y la aparicin de
cnce

45);{~ causa ms frecuente de fiebre, durante las 24 horas siguientes a una ciruga, es:
a.(lnfeccin de la herida operatoria

fi"
~ f!'

~ ~telectasias
d) Secundaria los medicamentos e) Flebitis

A<) Neumonfa

~ ~

e ,..

"
~

(!!'

~
.

46) Un Qiente consulta por dolor ocular intenso, especialmente al estar expuesto a la luz, 19 qi:J~ ~e impid ~brir el ojo. Al examen se observa ojo rojo profundo, sin secrecin. Las pupilas son1socricas y lo reflejos fotomotor y de acomodacin se encuentran conservados. El diagnstico ms probable

,.,
~

e .
) t1veltis ag(ia b) hjs;$ociclitis ..... ...craueratitis agda " d) Glatle~a:de-ngulo abie1 W e) ~nJlJOtiVitis por aeent:JViflS 47

~
~
~

Qu alteracin NO esperarla encontrar en un paciente con trastorno por,omatizacin? ) Prurito farlngeo ~ ~rconvulsiqnes "atipicas" e) Disfagia .. d) Dolorjl6dominal e) Dis na 48 Cul de las siguientes afirmaciones es CORRECTA? La incontinencia urinaria de urgencias se caracteriza por prdidas de pequeas cantidades de orina -< b) La incontinencia urinaria de esfuerzos presenta contracciones no inhibidas en la cistomanometrla < e) Ante un escape continuo de orina, a modo de goteo persistente, debe sospecharse una incontinencia "(,. urinaria de esfuerzos d)-La incontinencia urinaria de urgencias suele ser de_ resolucin quirrgica "' /) La vejiga neurognica suele tratarse con autocateterismo limpio

,._,
~

~
~
~
~

~ ~

49) Una paciente de 17 aos ingiere 100 comprimidos de loratadina hice 4 horas. Al examen fsico sus

~ signos vitales son normales y no presenta otras alteraciones. La conducta ms ade(:uada es: ; -\-t-Jo e ~~ a) Realizar lavado gstrico ~ --4 ''\. ~u \ b) Enviar a domicilio, tranquilizando a los familiares, ya que la loratetdina es un medicamente relativamente ~

'?,<...

/~~~iar a domic!' prevenir a los familiares de esconder los medicamentos ms peligrosos e Indicar control
- J
ambulatorio en ~1 policlinico de salud mental . d) Observar ppf 6 horas y en caso de no presentar empeoramiento enviar a domicilio con antidepresivos inhibidore~ ..p la recaptura de serotonina y control en el pollclinico de salud mental ~) Ho~pitl(zar, mantener en observacin, indicando evaluacin psiquitrica

""l
.

V'!

hombre de 65 aos presenta varias lesiones en la frente y en dorso de las manos, indoloras, no ginosas, solevantadas, speras al tacto, y con fina descamacin, de lento crecimiento. El nstico ms probable es: ../l~~ueratosls seborreicas ~uerat9sis actfnicas e) Lntigs solares d) Car fnomas espinocelulares e) O matitis seborreica Un paciente de 35 aos presenta rotacin e inversin f~rzada del pie izquierdo, al tropezar en una scalera, evolucionando con intenso dolor e im.posihilidad de =~poyar el-pie. Al examen se observa gran edema y equim:!del tobillo izquierdo, asociado!? .a..int.anso dolor a la Q!!pacin de la zona.Y~-especialmente cle]S:-malofos. El diagnstico ms probable es: a) Rotura del tendn aquiliano b) Esguince de tobillo grado 1 e) Esguince de tobillo grado 11 d) Rotura de la sindesmosis peroneo-tibial ,...e) Fractura de tobillo

~
~ ~

"'
~

) \o:_, ~.+o r
'{\1\Ci... ,

s-

uov~\ '"f

.S

\'-.."--

sv~\....a,

~. ~
52) Una paciente de 30 aos presenta un cuadro de palpitaciones, disnea, opresin precordial, parestesias y marcada angustia, de instalacin relativamente rpida, mientras estaba en el metro. Luego de esa vez ha presentado 3 episodios similares, s~oder Identificar un desencadenante claro. Est muy preocupada por lo que le ha pasado, porque cree que poda sufrtr un infarto en alguno de . esos eventos, razn por la cual evita ir al metro Y. sale siempre acompaada. El diagnstico ms " \ / ' probable es: . ~ a) Trastorno de ansiedad generalizada " .t __ . J.. ~ ~' ~)Jr9storoo de angustia con agorafobia --'l N'-L~ 1 t~ .fJv('J--'--Q..f L.-V'\.A...\l t\./e-Z.Q.. ,/q "!rastomo de pnico sin agorafobia d) Fobia social e) Fobia especifica

~ ~ ~
~

. /

Q V\

~ ~ ~ ~

53) Un ~aiente de 60 aos consulta pctr intenso dolor abdominal, epigstricoLirradiado a dorso. de 6 horas de evolucin, asociado a vmitos alimentarios marc e neral.

e
~ ~
~

~ ~ ~ ~ ~

PA: 0/50 mm" g, t 0 {37,5C auscultacin cardiaca con ritmo regularen 2 tonos y uscu tacin pu monar con e .difusos bilaterales. En sus exmenes de sangre presenta creatinin~: 1.t7 mg/dl, BUN: 30 mg/dl, N'~(Eql!, K: 4,7 mE.qfl, Cl: 97 mEq/1, Ca: 7,8 mg!d_l, __ .. ___ _ hematocrtto: 32%, glbulos blancos.:dl' or mm3, plaquetas: 400.000 por mm3, GOT: 230 Ul/1,> billrrubina: 2,1 mg/dl, pH: 7,3, Pa02: 58 mmHg, PaC02: 40 mmHg. La radiograffa de trax demuestra edema puTmonar y el electrocardiograma slo muestra alteraciones tnespeciflcas de la repolarizacin. El diagnstico ms probable es: a) Diseccin artica b} AccideAte-vascofat-meseotrico e) ~racin de lcera gstrica ;:trPancreattus aguda e) Infarto agudo al miocardio

E~~iirci~na c=~d res:]a y ten ene a a a 1potensln Al examen flslco presenta pulso de
1~

.X

. /

~ ~ ~

e
~

~
~

) A

54) Cul de l.as siguientes asociaciones entre patologra y cUnlca es)_ncorrecta? a) Disquine~ia tardla- Movimientos reptantes de la lengua " ....; _5l _....;;b) BalismoL Movimientos reptantes de grupos musculares pequeos de la zona distal de las extremidades -' \fJJ ru ..~. .>-~ e) ~catiJ;l~ - Malestar al permanecer quieto, que se alivia con cambios frecuentes de posicin-~ ji(Di 6nla- Aumento del tono de un grupo de msculos . \ e)R rkinsonismo-Temblordereposo~ oJ:L~~ -1 \1~'-.\ ~-~~~~ .~ ) Respecto a los estudios de casos y controles es FALSO gue: a) Son ml!.y. 'tiles p~ra estudi9atologias poco frecuentes ......,/ b) Sue~eser de baJo costo . e) Per 1ten calcular el odds ratio , P iten determinar causalidad . / rmiten determinar el pronstico de una patologia

Cul de las siguientes ncer de pu!mn ncer de clon "" ) ncer de . ~sfago><.... d) Cncer;:t estmago e) Hep~carcinoma

p~tologfas ti~e

mayor sobrevida?

ff'
~ ~
~

57) 'ul de los siguientes enunciados es ms representativo de una amigdalitis pultcea co Jicada? -- Fiebre de 40C, asociada a compromiso del estado general b) Presencia de mltiples adenoptlas cervicales, s~les a la palpacin --'\ ~olor muy intenso, que impide la apertura bucal d) Nuseas y vmitos alimentarios en varias oportunidades e) "Vos de papa caliente" y ronquidos en el sueo

X"

e
~

'!'
~

,...
~

~
~ ~
58 n paciente de 54 ao.s, hlpertenso, portador de una arritmia completa por fibrilacin auricular. esenta, de manera sbita:dolor e1a extremidad izquier~c;oclada a parestesias. Al examen fisico e aprecia palidez y ausencia de pulso en dicha extremidad. la conducta ms adecuada es: a) Iniciara anticoaglacin con hepanna y sohcitar eco-doppler de e>dremidades inferiores b) Solicitar pletismografla de volumen de extremidades inferiores ""><' e) Realizar un bypass femoro-femoral d) Administrar heparina endovenosa y realizar angiografia femoral ' e) Solicitar Holter de arritmias e iniciar antiagregantes plaquetarios
~

~
~

f7

,.,
~
~
~

59) Calcule el Odds Ratio del consumo de cigarrillo para el desarrollo de cncer de vejiga, con la siguiente informacin (Valor aproximado):
Cncer de Ve"i a Fumadores No fumadores TOTAL Sanos TOTAL

100

100

(.

50
150
~.,.

300 400

200

a)0,17 b) 2,oo -6) 2,67


d) 3,50

09

P..
;.

_,...
(.,

/
Q .

~~

o;> ks~ ,.,_ p:1 -~ ,

350 550
\

~
~ ~
~

,;>y> 6,~0 /

y s despierta temprano, para pr_oseguir con sus actividades.-habiendG-incurddn en gllllldes_gtu!!.l~51~ d.h o. El se _ve muy molesto con su sit~acin y refiere que sus familiares no confian en l y por e~ " dejan}~.Q.Quir adelante con el negocio que est iniclando,..en el que obtendr gra~tilidades. Habla mucno y pasa fcilmente de un tema a otro. El diagnstico ms probable es: --~--a) TraslOfhO delirante crnico x b) l;squjzofrenia . /. ....e(frast'orno bipolar en fase maniaca ._,.., d) De resin psictica e) Tr. storno adaptativo Una paciente de 24 aos presenta. dolor lumbar, irradiado a la vulva, muy Intenso, desde hace 1 ora. Ha presentado varios episodios devmitos y a la observacin se aprecia muy.Jn.QY!eta. EL diagn~ltico de sospecha es: a) Col~ stitls 'Sgada b) Coreetncolitiasis - lico renal d) ielonefritis-aguda , e lnfarto-mesent~..

.frritL~~. Trabaja hasta avanzadas horas de la noche en un nuevo pro.yecto que ha decidido ~..D'lP.[eDdar .

so> ulpaciente de 32 aos es trardo por sus familiares, ya que presenta ideas extraas v est muy

~
~

~
~

~
~
~
~

~
~

r-----

2) Cul de los sjgjentes parmetros es ms adecuado para medir la t!roductlvldad de un ndice de ocuyacin de camas Nmero d~pacientes hospitalizados e Promedi oe dlas de estadla d) ecurs s prop os genera os e) Nivel e endeudamiento

h~ -

~ ~
~ ~

~
~

63) paciente de 20 aos presenta autoestima muy baja y no se relaciona con otras personas, p que se considera inferior y teme ser rechazado. Por esto evita los trabajos en grupo y no acude a estas ni otras actividades donde deba intera~tuar con tos dems. Refiere que le gustarla tener amigos, pero su timidez se lo dificulta. El diagnstico ms probable es: a) Depresin b) Distimia ~~ ~ia social ~~~~~torno de personalidad evitativo e) Slndrome de Asperger

~
~

~
~
~

~
~

~
~

~
~

64) Un .la'ciente de 35 aC?s presenta otal ia intensa, asociada a otorrea. Al examen ffsico se observa a lcf;~ritema d~ducto aud~~xternq,' con abun ante otorrea, sin compromiso del tfmpano. onducta mas adecuada es: ciar amoxicilina oral y analgsicos ciar amox~.eiina ms c~do clavulnico oral y analgsicos iciar cip~ofloxacino en gotas ticas y analgsicos d).Oer" teffiRela:iJ:gologlf!, de manera urgente e) Ho~ "mcial::an~lgslcos y amin,Jglic6sidos endovenosos 5 Cmo se comprah los medicamentos en los consultorios? u.-"7ri(-~--CQffioran a disUtns p:eveeEier-es,seutiA-Su.precin, caUdad-y-garantias_. -. Se compran al mismo laboratorio os compra la.ENABAST y luego los asigna al consultorio d) Los compra Servicio de Salud y luego los asigna al consultorio e) Se c:zop fi al proveedor ms barato y c:on menores tiempos de_ ~!2~ega
/

--'\

"'

" "
~ ~ (/!'
~

66) Una aciente de 35 aos presenta una fa~ctura de te'mua:.derec.fld, durante un accidente de trnsito. Evolu91ona con importante dolor, que da a da se hace ms intenso. Al examen ffsico presenta ftante dolor a la movilizacin de los artejos y seaprecia adelgazamiento de la piel del pie derecho minucin de los fanreos. La sensibilidad y los pulsos distales son normales. El diagnstico ms able es: a) Retraso de la consolidacin . b) Pseudoartrosis (no unin) ~~ J.Pdtome compartimenta! ~~~trofia simptico refleja e) Retraccin isqumica de Volkmann 67) Una paciente de 54 aos, fumadora, presenta hematuria en 2 ocasiones. sin otros..sfntomas. Al examen ffsico no se aprecian alteracion~s. Se solicita un sedimento de orina que muestra ms de 100 glbulo.s ro os por campo, sin dismoia. El hemograma, la creatinina y los electrolitos plasmticos son normales. La con ucta ms adecuada es: '"u. /1(}. ... ;>J(J._) a) Solieitar-perftl uretral .} ~) ~ __v/v .--. b) Solicitar uretrocistografia retrgad endoscpica ~- r--J~ f ~ ~ -o ~"\~e_, e) Sollcttar ANA, ANCA y complemento ~\-e Q....--' "" d) Solicitar ecografla renal y vesical ...... dJ're) Solicitar TAC de abdomen y pelvis - e~S ~e \.9 (J.--' 68) Se realiza un test de tolerancia a la ~1lucosa a 500 personas, diagnosticadas de obesidad y a 500 personas, con peso normal, determinando la cantidad de pacientes diabticos e intolerantes a la glucosa en cada uno de los grupos. A qu tipo de estudio crresponde? a) Casos y controles "-. b) E sayo clinlco ohortes ansversal ) colgico

~ ~ ~
~

(/'-.

p
~

,r

y/

~
~ ~
~
~

~ ~
~
~

69) Un nio de 5 aos es diagnosticado de trastorno oposicionista desafiante. Qu situacin NO le parece"corde con este diagnstico? a) Ynabietas cuando no se hace lo que quiere __,.' . _9.) Buenas relaciones con sus compaeros de juegos ~ " \..._/e) Inteligencia normal _.. _2-1{obos de p~qeas cantidades de dinero a sus padres e) Actitud ir_reverente al recibtr una orden_ /

.A !;Jbitualmente son de origen maligno


1}-YFrecuentemente producen dolorx

7~ R7specto a los ndulos pulmonares solitarios es cierto que:

~e) El quiste hidatidico pulmonar se diagnostica mediante ecografa de trax -~

~
~

d) Las radiografas previas no suelen ser de mucha utilidad ><" e) Los ndulos parahiliares se biopsian mediante puncin percutnea, guiada por imagen

----------

f'
~

~ ~

lf\. ...... "''--'

....

~ ~
~
~
/

,/

Pruebafiepaso No 7 NEU,LOG(A + SALUD PBLICA

/.,.... de los siguientes hallazgos cUnic:os NO apoyarfa el diagnstico der::-----,,, un Cul polineuropatfa)m
---

" "'
~ ~ ~

aciente con debilidad progresiva en miembros inferiores?: ) Fasciculaciones. b).)\~rofia musc~l~r- . ~...eJ Signo de Bab1sk1. d) Hipo o agefexia. e) Hipo~Flfa.

2)~a mujer de 60 aos con antecedentes de ja~ueca e HTA en trata.miento, acude al-servicio de

~ ~

~ ~ ~

Ji

ur encias por haber sufrido una cefaleaJntensa instaurada de manera hnsca tras haber realizado un fuerzo fsico. En la exploracin se aprecia fotofobia. rigidez de nuca y una tensin arterial elevada, iendo norm-1 el resto de la explo~=aGin-ltew:olglca. El diagnstico ms probable es: a) Hematoma cerebral intraparenquimatoso hipertensivo. ~<: b) Tumor cerebral. ~ e) Crisis d,ejaqueca. ~ tfRotur de aneurisma sacular intracraneal. ;.e) Cefiilea producida por una subida de la T.A . ../

"'

(!A
~ ~

.{cul de los hallazgos es muy frecuente en la enfermedada de Parkinson idioptica?: . Demencia al inicio ) Mioclonas. . Blefaroespasmo. d) ~troc_9111s. / ,....efPrdjda de movimientos asociados en la marcha. de acude a la consulta por furu:,za, en iembros superiores. La exploracin neurolgica evidel')_cia_p_[dida.d.e_fyerza. atrofia y sciculaci~s en varios grupos musculares de miembros superi~s, asr c_o~_Q..-l!!E'~J-~di~-de uerz.a ~!!-!~~""~s::m~~~-~~-a~~~ore!>. ~os ..r~trejos os.teotendinosos. son.viY.as..v:mm..t.Oc::os_y__kl senst~thda-d es nor~. El estuaro electrc,fistologlco demuestra signos de dene~~~~~-1} ~-~--n:_l_ultip,@j mscuros-co-coauccin nerviosa sensitiva normal. El diagnstico ms probable es: . .~'"=-~_...a)"tsclerosis lateral amiotrfica. 0...~ J_ \.!)' ' b) S~ndrome.mias!nico tipo ~aton-~ambert \. Q.~J \f . 'nd' ~ "' e) M1opatia t1rotxtca con fasc1culactones. O' d) Espondilosis cervical. .-re) Atrofia muscular espinal de comienzo tardo. 4)

sigu~entes

(A
~
~
~

~:rmo 7~os,qe

p~Jdida_de

fun~~~.~~~-~-~~1:'-~~ p_~Qximal,

J.

0.........r..

fA
~

~
~

~
~

~
~

5) Una p'ciente de 22 aos consulta por presentar desde una semana antes ptosis palpebral izquierda, sin dpfor, con diploplaen al mirada lateral izqu!erda. En la exploracin ffsica se comprueba la ex~tencia de tiapfosis-izquier'la, Liria paresia de la abduccin del ojo izquierdo, con unas pupilas i 6cricas y. normorreactivas de la luz. Qu enfermedad es ms probable que padezca la paciente?: ) Una neuritis ptica izquierda en relacin con Esclerosis Mltiple. =::}\!.lry sin~rone de Horner. /01 Una Miastenia gravis. ,/ d) Una 'Parlisis dellll par izquierdo. e) un'a miopatia hipertiroidea con afectacin de la musculatura extraocular.

fA
~

V~)

~
~
(!~!'

Paracetamol b) Vitamina C e) Ergotamina d) Sumatriptn / } Flunarizina

~Cul de los siguientes medicamentos es til para prevenir la recurrencla de jaquecas?


~{r:,fi---.~

r~P~-t

"
f.B'
~

"
~

"

7) Un paciente presenta de manera sbita hemlparesia izquierda, ~1sociada a compromiso de los pares craneanos V, VI y VIl derechos. A qu nivel se encuentra la lesin? orteza cerebral psula int~rna ~ c)Tlamo ~ Troncoencfalo --:~ Nervios

V'

8) U ~ciente de 34 aos sufre un accidente de trnsito, con traumatismo cervical y seccin medular a ltura de C5. Qu alteracin serfa MENOS probable de encontrar? arlisis diafragmtica ~-vVU!) C....2.- e_") C A ) lnContinencia urinaria .._....... ~ Li l e) TetrapJejia d) Ane;;tesia de las 4 extremidades .,e) HJ,Ptensin arterial J' ~ (\.)U..JNJ~CO / 9}'Un paci~nte de 68 aos presenta un cuadro de meses de evolucin de olvidos frecuentes y onversaciones repetitivas, al que se le ha agregado en las ltimas semanas dificultad para realizar sus actividades cotidianas, como salir a comprar o llamar por telfono. El examen neurolgico no presenta alteraciones motoras ni sensitivas. El diagnstico ms probable es: ~) Demencia senil b) Delirium e) Demencia por cuerpos de Lewy d) Depresin e) Demencia vascular

c....l?

10) Un r:tio de 10 aos presenta un cuadro de movimientos involuntarios de la extremidad inferior izqu!erla, que luego se extienden a la extremidad superior izquierda, siendo seguidos de lenguaje in 6herente y finalmente una convulsin tnico-clnica. Qu etijologfa es la ms probable entre las lanteadas a continuacin?
~

a)pilepsia lnlcoclrllca

..,

b)_,-Epi1eps1a m1oclmca Lesin focal . d) ..Encefalitis-'lirat___ e) Grisis-e~twulsivafarmacolgica -- .... _


11) Estl usted evaluan.do una prueba diagnstica y finalmente

s~ encuentra en la siguiente tabla:

Enfermedad

SI 8Q
r

NO 100

~-ru_e_b_a_N_e---t~iv~a------~~2-0--~~-80-0----~
/

!l ~:-\ 1
/

J>5

A partir da ella y redondeando decimales, seale cul de las siguientes firmaciones es. verdadera: a) El v~lr predictivo positivo es del 56% y el valor predictivo negativo del13%. ~lyalor predictivo positivo es del 44% y el valor predictivo negativo del 98%. e) ~1 valor predictivo positivo es del 80% y el valor predictivo negativo del 90%. d)1El valor predictivo positivo es del 80% y el valor predictivo negativo del 11 %. ~ El valor predictivo positivo es del 1O% y el valor predictivo negativo del 20%. 2} Suponga que en un ensayo clinico hay diferencias entre lo~ grupos en algunos factores pronsticos. Afecta esto a la interpretacin de los resultados?: a) No, gracias a la asignacin aleatoria ("randomizacin"}. ' Hara necesario utilizar alguna tcnica estadlstica que permita tener en cuenta las diferencias entre ellos en los factores pronsticos. e) Invalida el estudio, ya que la asignacin aleatoria no ha conseguido su objetivo de obtener grupos homogneos. d) Invalida el estudio, ya que el nmero de pacientes incluidos es im;uficiente para obtener grupos . homogneos. e) Indica la necesidad de reanudar la Inclusin de pacientes en el eBtudio, hasta conseguir equilibrar el pronstico de los grupos.

Dr.ANDRSTORREALBAli
Mdlco CiruJano
Ru\.:15.936.726-~

er
~

~
~ ~

~ ~
~

1 ) Uno de los principales inconvenientes de los estudios de casos y controles es: Que a menudo requieren muestras de gn:m tamao. )rSu mayor susceptibilidad a incurrir en seHgos respecto a otros estudios epidemiolgicos. e) Que no son adesuado.s_p~tudiaumf~rmedade.SJ:ar.as. d) Que-sueleR-ser-eams-y-Eie..JaFga-dl:tFSein. e} Que no permiten deterroinat:Jaadds..tatio.

14) Un nio de 5 aos pesa 20 kg. Segn las tablas de Tanner-Whitehouse, este valor corresponde aproximadamente al percentil 75 de la distribucin del peso en nios de dicha edad. Qu significa esta expresin?: a} Et15% de los nios de 5 !Q.s.._p.esan.apr~ximadameRte-2Q-kg-. b) El oitlo-pesa-un-TSO/oms--que-etpromedio-cle-les-AiAes--de-st 1 edad. ~ 75% de los nios de 5 aos pesan menos de 20 kg . .~1 nio tiene un peso que es un 75% superior a la mediana del peso de Jos nios de su edad. e) EI-75Wo-t:le-les-Aios-de-5-aes~esaA,-Cam.o_mr:aimGr2-Q-kg.

El

15) Suponga que en la fase de diseo se calcula que para tener suficiente poder estadfstico en el estudio de cierta intervencin teraputica mediante un ensayo clnico es necesario Incluir 100 sujetos. f!!' .: . _.. Sin embargo el ensayo se realiza con slo 50 pacientes y el resultado es estadsticamente significativo . . (P < 0,001). Suponga que en todo lo dems el estudio parece correcto. En. vista del pequeo nmero de ~ .. ..(?Jacientes incluidos, son crebles los re!;ultados?: ) No, ya que en los ensayos demasiado pequeos los errores sistemticos son inevitables. \('~: ... b) No, ya que hay muchas posibilidades de que las diferencias encontradas sean un resultado falso positivo. ~ , ~) Si, ya que una vez alcanzado un resultado concluyente no importa lo probable o improbable que fuese el (legar a l antes de hacer el estudio. f!' d) No, ya que la probabilidad de encontrar diferencias, si stas existen, es baja cuando el estudio es pequeo . .._ e) Si, sier,npre y cuando estos resultados se utilicen exclusivamente para unirlos a Jos de otros estudios " peque~os sobre la misma cuestin y hacer un meta-anlisis o una revisin.

. G .

'\? .

~ ~ fA

~
~
~

~1
~-

.'Un estudio compara dos tratamiento~; antibiticos (frmaco A y frmaco 8), para las neumonfas S 111. Se administra cada antibitico a '100 pacientes distintos y se ve la tasa de mejora. El frmaco presenta una tasa de mejoria de 98% y el frmaco 8 de 90%, sin embargo el tratamiento completo n el frmaco A cuesta $100.000 y con e'l frmaco B cuesta $20.000. Se calcula el dinero adicional que y que Invertir, al utilizar el frmaco A, EID lugar del frmaco B, para que un paciente adicional mejore. qu tipo de estudio corre$P.Onde7 . 1......0 ( \' ) ~ Costo-eficacia .rl e~~. ~ .. .r~. + CA._, ..., b) Costo-utilidad~ ~ '-'" ~- .-l e) Costo-b.em~fi. ~ ~ r G..A. tAV' ~ l d) Minimizacin de costos e) Efectividad

"'

,..
f!"

17) Para determinar la relevancia de una ~nfermedad infecciosa grave conviene ocupar: a) Mortalidad _gneral .~----------- / ' ~ortalidad especifica t;J (C) st.---1.-.-d_. . ~~~ ---'f) Jncid~tlcia 1 /d) Pr valencia e) sa de ataque

....J.

~
~
~

"
~
~

8) Se calcula el IMC de 80 pacientes diabticos y de 150 pacientes sanos. Qu prueba estadstica es la ms adecuada para determinar si existen diferencias significativas entre ambos grupos? f i t de Student b) Chi cuadrado e) r de Pearson d) Anlisis de Varianza e) Riesgo atribuible

9-X

,.,
~
(I1A

;x

"" a) Realizar autopslas-de-fallecimientus-trapmticosy-suicidio~ \'J'--t.c)...A \J..J b) 9scalizar a las ISAPREs .~ \-.J (Jf'b \ ..-.....~ ~~' ~ontrolar el corre.cto funcionamiento de Jos laboratorios pblicos y privados c.. G ~dministr.ar-los..tt.tlrsos-de. la salud pblioa (~)Entregar el..aporte-per cpi@ ---

19) EIISP tiene como una de sus funcionr~s:

L-Q c e---l

N (\ ~ t'\-.-rl

~--- ..

" '!'

~
2 J.Jn hospital tiene un fndice mensual de ocupacin de camas. del 90%. Esto quiere decir que:
.. Se 'Ocupa el 90% de los das-cama disponibles en el mes b) Hay dlas...ao..que..el-RGspitakurtier-te-eamas-EIIspeRibles ct~l.Jlospital siempre tiene algunas camas disponibles . ,4f~durante el mes hay un 10% de camas que no fueron ~!ilizadas , , / / e) Lbs-paeientestteri~lfllrnralta-rotacin _.. --

~
~

~
~

~ ~
~ ~ ~

~ ~

~
~

~
~
~ ~

~
~ ~

.e;

..,
Al)
~

~
~

~ ~
~ ~
~

~
~

~
~

1) Una gestante de 28 semanas se realiza test de tolerancia a la glucosa, como parte de un control del embarazo, que muestra glicemia de ayunos.de 100 mg/dl y de 155 mg/dl, a las 2 horas despus de recibir 75 g de glucosa oral. Se encuentra asintomtica y el examen fisico slo muestra los cambios nonnales del mbarazo. La conducta ms adecuada es: --?a) Solicitar nuevo test de tolerancia a la glucosa oral con 100 g de glucosa b) Solicitar hemoglobina glicosilada ~ . ..S.. . --::---. ,,~-lnicia~a_y...ejeFelcios :==->);_. i~~--v-<. <...-~~ d)1ntclar med1das generales y metformina e) Iniciar medidas generales e insulinotempia

.A

'A

1/1!\

dej~92 mmHg en un control ginecolgico. Se controla la prE!Sin arterial en varias oportunidades durante 6 horas, constatndose varios valores mayores de 140/90. La conducta ms adecuada es . -)j.-\:--) ~ a) Controlar frecuentemente la presin arterial por 48 horas ms y decidir manejo segn resultados / <.. 0 r ~ SCar protemuna de 24 hora]) <<o v . , e) Solicitar hemograma, pruebas heptica:;, LDH, tiempos de coagulacin, clear~nce de .creatinina, proteinuria ~ ... . \. ~. de 24 horas, electrolitos plasmticos ~--.;_~O\ e\.~ e,'--{, ~V d) Indicar dieta hiposdica y ejercicios ~; ~~~'.A.~ d ft~Jdlcar dieta, ejercicios y alfametildopa > ~ -~ 4, 9 ::,_,.:;;:- ~

2) Una mujer de 36 aos, primigesta de 30 semanas, presenta presin arterial

3) Una mujer de 25 aftos, cursando embarazo de 8 semanas de amenorrea, confirmado co est~--Q..v'"-v'-J.:.C deteccin de beta-HCG urinaria, consulta por dolor h~ROQ;.irl~QJotenso y metrorragia. Evoluciona con ~~ c . IJ.potlmias.y-or-to.sM~ Al examen flsico se apreia plida, con frialdad de extremidades, FC: 124x' y PA: 80/50 mmHg. El diagnstico de scspecha es: a) Desprendimiento de placenta normo inserta ~ ntO ~-.-sU. -\. u r'\= r )""Cti---. \ .r~ b) Rotura uterina r ~ e) Aborto incompleto l_. ., e ~ (.) ~~ v-Q' d) Embarazo molar t~ o--~.o ~ ' ., '--"- -~) Embarazo ectpico roto ~ ~ ~ ,,.....J--of (.J.J 1'-"'

---~e;)~-

r- .. " .

u t

4) Cul de las siguientes alternativas NO es una indicacin de administrar inmunoglobulina anti-0 a

1 f' 1.L.

--->l~rueba de Coombs indirecto positiva v' ~ r-G!> :S.' (


~~mniocentesis

una paciente embarazada..Bh negativo? . e) Postparto ~ -r t ~'\? 1 t"> "k d) Aborto completo,/' e) 28 semanas de gestacin

...rl..

-->)~ J--~oy...) .. ..La,


_.e-

-----

e 1 f.:::.<.:.-0 ~c..... N-

t\
\

o J

\e~

\ 1 .'"'"'' J.,-U.J

r~ . ~~

~!A

~ f!!!!!'
~

('A
~
~
~
~
~

5) Una paciente de 22 aos, cursando embarazo de 29 semanas presenta salida de liqui.do claro, con .o 0 olor a cloro, por genitales, desde hace algunos das. No presenta otros sntomas. La especuloscopia.../> ~ es normal y el test de cristalizacin presenta formacin de .. hojas de helecho". La ecograffa obsttrica .\......:o-~.. ~'-demuestra un feto en ceflica, con oligoamnios, sin otras alteraciones. El registro basal no estresante "\ C-e es tranquilizador. La conducta ms adecuada es: /0~~~-r/'-_ a) Interrumpir el embarazo por la via ms expedita ~~c.. "'= b) Inducir el parto con misoprostol ~dministrar corticoides y antibiticos a la madre, manteniendo una conducta expectante respecto al parto Y d} Administrar corticoides y antibiticos e interrumpir el embarazo en 48 horas JV .Q."-'v'~ \i e} Realizar cerclaje de emergencia e iniciar antibiticos

"S----- ,

.f'

-e~~~" .
-~~ J_.
..,:;..<--~ ......
'. _ :'

c_Y"C/

(ta
~

,
~

6) Una mujer de 25 aftas consulta porquet present un aborto espontneo a las 8 semanas de su primer embarazo. Actualmente se encuentra usando anticonceptivos orales, pero desea un nuevo embarazo. La conducta ms adecuada es: a) Solicitar ecografia transvaginal y decidir manejo segn hallazgos . ~) Solicitar anticuerpos anticardiolipinas, progesterona plasmtica de fin de ciclo, ecografia transvaginal, ./prueba de Hegar y decidir manejo segn hallazgos \....,/" 4r.Stlspendel auticonee:tj)Jes-y admtmstrnrprogesterona en la segndcnnitad-del-eiete.menstr.uaL_........ " ... d) Realizar seguimiento folicular y induccin de la ovulacin con clomifeno -----?elTranquilizarla e indicarle CllJe puede intentar un nuevo embarazo sin necesidad de estudio '-

- "-~ ""_ v.:\-S

vu'- <
?yo

V"'.
'.......A

'~; \.

~A.f

~ ~""\-
\

~ c....,._......L~
_&.,

"'

\ r-... ~~ (A_.';;> \....1 . .

l -

~~e~ o 1
.
,,~.

~
~

5 )

c::i._..l~.

fi'A_____

(A

D
7) El pynt de reparo de la presentacin de cara es: -a) fantanela anterior b)",..Ojos -~e) Nariz d) 3oca tfMentn
8) /-L:a elevacin de la alfafetop~o~ a) Sndrome de Down .

~9GWS-del-t~:~ee-Reu~=a~ ~~
\!JO
\.
/

-Y

es un marcador de:

(.) \;:\ L;i


___

e) Trisomia 18 .d) Hernia diafragmtica congnita e) Restriccin del crecimiento intrauterino

,\d- ~

~~'t?~

1 -~-

~~

~-~

~--~

..R._

\.

?)

9) Paciente multpara de dos en trabajo de parto, cuyo monitoreo muestra una FC f~tal .!?~ ~in, buena variabilidad y presencia de aceleraciones. La rotura artificial de membranas da salida a un lquido con meconio fluido, amarrillo verdoso. Cul es la ccnducta obsttrica a se:;;::guir? ~~" ,a) Interrupcin inmediata del embarazo mediante cesrea ,.J.) . b) Reanimacin fetal in tero con reevaluacin en 20 minutos .~ ... .J. e) Reanimacin fetal in tero seguida de interrupcin del embarazo ~ , ~J.J . ~ Mantener una conducta expec~perroitiu.uS-GGAtine el traeajo..d~ ~' \.- . A Fnterrupc1on InMediata por la vla ms expedita /.

b
.

b-:
o

- ,./

1 O) A las 34 semanas de gestacin un feto pesa aproximadamente:


a) 500 a 1.000 g

-z. 2

~2.000g2.500g ?'-\ . e) 2.500 g 3.000 g -?:, '-e

yKsoo a 2.ooo 9

b) 1 000 a 1.500 g -z..-.1S'

j)

:> ~

,'v~ .p-o_ ./___.-- o o ~~ ., , ~ . -_y- -~G , - '2,..A,:,.

,t'')

~y............-'
..

. .. a

~~~

;;---

&!0 _~

11) Una paciente con preclamsla severa presenta cefalea intenna y luego una convulsin tnlcoclnica
autglimitada. L~ conducta ms adecuada es:
/.ar..Jiministr~r sulfato de magnesio. corticoides endovenosos e inducir el parto con misoprostol

_,...,- ,b);fniciar {$Uifato-da.magr:lesi9 estabilizar a la madre e interrumpir e! embarazo / e)" Admi.nistrar lorazepam y realizar cesrea de urgencia d) Aqninistrar sulfato de magnesio y solicitar tomografia axial computada de cerebro e) Realizar cesrea inmediatamente, sin uso de medicamentos / .

-~) Aborto cqnpleto, embarazo ectpico y uso de progestgenos exgenos


/

~~La biopsi} ndometrial, con reaccin de Arias Stella se obs;erva en:

b) Embar~io molar, embarazo ectpico y uso de estrgenos exgenos e) Emb~r~~o molar, climaterio y uso de progestgenos exgenos d) Ab rlo completo, climaterio y uso de estrgenos exgenos barazo normal, anovulacin crnica y uso de anticonceptivos combinados

na gestante de 32 semanas consulta por metrorragia y contracciones uterinas dolorosas. Al en fsico destaca tero hipertnico. El diagnstico ms probable es: sprendimiento previo de placenta normoinserta . ' ) acenta previa e) Rotura uterina d) Rotura de vasa previa e) Acretismo placentario

.\

-~

14) Uns:aniujer de 25 aos, cursando embarazo de 9 semanas inicia metrorragia abundante. La (.,/ especllloscopfa demuestra salida de sangre y restos ovulares por el OCE, la ecogratra transvaginal - . vi~liza el saco gestacional de contenido irregular y el tacto vaginal constata dilatacin cervical de 2 / _ . . -:., . \.... ~El resto del examen fisico es normal. La conducta ms adecuada es: ~ ' ,.; Q } Solicitar niveles de HCG ~ . (~ g ~- NC7 ~Lp ,.b}dministrar misoprostol intravaginal \ _,o . (U ~ J r . ( ~ ..J'o-.J"-\... ' el ~ )._., ~ e) Re aliJar ce relaje de emergencia \~.\ .....v QJ . / ....../') . 0 t:J d) lnj:liar reposo y dejar evolucin espontnea .!\"\'V ..._ .if ) p>Realizar legrado uterino

,V

cJt'

'

15) Una multpara de 34 aos, cursando uu cuarto embarazo, de 40 semanas inicia dinmica uterina. Al ,-e.L-DC.>J? examen se, constatan 3 contracciones en 10 minutos y el tacto vaginal muestra un cuello completanente borrado, con 4 cm de dilatacin y membranas ntegras. Se instala monitoreo fetal que ...-:>.> -- muestfa desaceleraciones en relacin a la mayora de las contracciones uterinas, sin decalage, en jo con stas. La conducta ms adecuada es: ~----e~ ejar evolucin espontnea ~,y<- C..t . ;.; b) Administrar oxigeno, poner-en decbito lctteral izquierdo e indicar anestesia epidural e) Iniciar aceleracin occitcica ) C.....v-v---.. / d} Realizar rotura artificial de membranas y observar el lquido amnitico 0 (l- Q_ -~ c...-v~ e) Realizar cesrea de urgencia o---"-"~

f> .. _ __----

-s

'

~
~

16) La corrtphcacin ms frecuente de la cesrea es: a)-H'morragia intraperitoneal ~) Metrorragia e} Dehiscencia de la herida ~ d} Atelectasia~ pulmonares ~}-lnfecci!Jcfe la herida operatoria -~ '-...>'
.......
/'

LJ...A-v'"""

,~-

'-"o.~ '-re

-..

~(y~~+ c._S.<,~

~~en?te~ol

~
~ ~

17) _,ul de l~s siguien~es !rmacos,e,s un ~ocoltico? ~ \ f .- ,()__A {'-...:.:~<;;:;) ~b) Occ1toc1na '~-(;.) ~ ' 10 e) Alfametildopa d) MetiJergpnovina e) Prostaglandina

xu \..

e
~
('A .

~
~

18) Cul es el mejor parmetro para estimar la edad gestacional en un embarazo avanzado, sin controles previos? a) Altura uterina c. ~~Lo { G\.-- o ~, o ~ ~'l p.,e_..o _, 3 ~z:: b) Inicio de la percepcin de movimientos fetales ~cha de la lti~a regla ponfiable y segur-.. -;:> " " .:::<-( \f~c.)~ ~'-"~ ~. /)J)tcografia ~ s.~-- _ J' 1 cLL~!~, e) Niveles plasmticos de beta-HCG

..

.-=,--\. ( -

19) El una ecografia del primer trimestre de un embarazo gemelar se observa el signo lambda. Qu _ --L tipo de embarazo gemelar es? , ~ ~C)f-...:.

.j(

~ ~,.;~.. <.:.J.-..\_

e"'
~ ~
~

a) Qiggt!co .

~n~~lgtiCO

/ ,

.--<\\

---

~(

\...Jl~

0> \~"-'>o_{(

Monocorial e) Monoamnitico
2~

.,

~ (._pCJ0 .._

~ --~~.

primera causa de muerte materna on Chile es:

~
~

.-'} Slndrome hipertensivo del embarazo

J~

(_(v

~ ~
~ ~ ("'
~ ~

b} Hemorragia puerperal e} Aborto sptico d) Enfermedades crnicas descompensadas por el embarazo e} Embolia de lquido amnitico

V
-~

7 ..:::_; ~
1\

y c.c1o {.. . L,O

'"'

, <._, c.:_ 0"1- LJ\.) ~~ ........P ~

\ .~

RUT

o
NOMBRE:

,_oo
2

00

3
4

00 00 00

'

5
6

RUT:

7
8

FECHA:

00

00 000 000 o 00 000 000 o 00 000 000 o


000

00

000 000 000 000 000

000

000 o 000 o 000 O 000 o 000 o

ooo

000 O.

CURSO: RESPUESTAS A B C D E
1 A 31 B C O E 61

2 3

ooc,oo
00000 00000 00000 00000

ooc,oo'

.00000

32 33

4
S 6

34
35 36

7 . 8
9 10

OOC>OO OOC>OO 00()00 00()00

37
38
39 40

00000 67 00000. 68 00000 00000 69 00000 00000 70 00000

00000 00000 00000 00000 00000

6~
63

00000

64
65 66

00000 00000 00000 00000 00000 00000

r-

..

, 00()00 12 00()00
13

41

42
43

00000 00000

71

72
73

14

15
16

17
18

19
20

00()00 00C)00
00C)00

00000 QQQ00 00()00

ooooo \ ooooo
44
<4

00000

00000
QQQQQ

74

00000

75

00000

ooooo.

00.000 00000 00000

46

76

47
48

49
50

00000 00000. 00000


00000

n
78

79
80

00000 00000 00000 00000 00000

21 22 23 24 25 26

00000 00000

00000 00()00 00000

51 52
53
54

55
56

27

~8
29
30

00000 00000 00000 00000

00000

57 58
59

.. oooop ooooo

00000 81 00000 82 00000. 83 00000 84 00000 85 OO.OO 86 00000 . 87 00000 88 00000 89

00000 00000 00000 00000 00000 00000

~
~

00000 00000 00000

""'~
~

~ ~

1)Paciente de 35 aos, ce~ada ~~r, cun?ando embarazo d~.3~-~~!TI-~as, presenta ~PO de. . 10 horas de evolcion. NtJ"Se registran contra_cciones uterinas ni modicaciones tervicales~y-el-reglstro-basatno estl'esante es tranq~Uiz.a.~or. La conducta ms adecuada es:
} a) Dejar evolucionar espontneamente!, siri necesidad de frmacos b) Dejar evolucionar, bajo cobertura antibitica y bajo corticoides para asegurar la maduracin fetal e) Indicar reposo, antibiticos, corticoicles e interrumpir cuando se cumplan 37 semanas de gestacin . , d~nducir el parto inmediatamente con misoprostol -")' Realizar cesrea ~

.
1

/ 1 2) En un test de tolerancia a las conttracciones, las


producidas por:
a) C.ompresin del cordn umbilical _..b}-tompresin de la cabeza fetal _.,:>) H~oxia fetal d) bt1poxia uterina ~)tompresin abdominal
1

C~saceleraciones precoce~ sqn


'-.

1
.

~- ~

~
~
~

/ 3) Cul de las siguientes opciones, es la causa ms frecuente de infertilidad? 1 a) En.dometriosis b) Anovulacin crnica e) Factor cervical d1fac;r'es hormonales ,;;J Fattor masculino

<. ,!ta alteracin menstrual que se observa cOn mayor frecuencia en pacientes hipotiroideas
'-.. /es:
".../ a) Hipermenorrea b)...Hipomenorrea , : :.. l~Oli~omenorrea ,...-: d) P61imenorrea e )~~menorrea

e
~

/ 5) Que mostrar la biopsia endomertrial de una paciente cursando un embarazo tubario?

{!'
~
~

a) Endometrio normal . _b) Reaccin de Arias Stella - e} Vellosfdades coriales d) Atrpfia endometrial e) Eritrocitos nucleados y normales

Jli{a principal ventaja del preservativo, frente a otros mtodos anticonceptivos es que:

e!' t9'
~
~

a) Es ms efectivo en la prevencin del embarazo b) No tiene efectos adversos e) ..:p.ene menor precio <.dfDisf9thuye el riesgo de contraer enfermedades de transmisin sexual ... ' e) Es ,de especial utilidad en las relaciones sexuales, practicadas durante el embarazo

V,? Realizar nuevo Papanicolau en un ailo ms


b) Realizar nuevo Papanicolau en 3 meses e) Repetir el Papanicolau inmediatamente drerivar a colposcopia y biopsia . .:. :~) Realizar cono cervical

7) yna paciente asin~9mtica se realiza Papanicolau, que muestra atipf!lS ~piteliales de m~.cJiano gra..fto. La speculoscopia es normat La conducta ms adecuada es:

~
~

(lA

f;. _ __

8) Una .. ~aciente de 24 aos, primipara, presenta imp_ortaote..metrorragia,.-luego-del-parto de un ep{barazo gemelar. Al examen se aprecia .PJt(!ie.nte-taquicrdica, .. normotensa y se. palpa el fo1J.9-0 uterino entre el ombligo '!.. ~~ a.p~f~s.!~ ~i!~ides. El dliagnstico ms probable es: ' .. a~mercia uterina . . ~) Laceracin del canal del parto . e) Rotura .uterina. . .. . d) Hematoma del canal del parto e) Alumbramiento Incompleto 9) paciente de 65 aos, presenta escapes de pequeas cantidades de orina, al toser, reirse o al/hacer cierto ejercicios, desde hace un mes. No presenta otros sin~omas, sin embargo su p'roblema ha comenzado a afectar sus actividades sociales y laborales. La conducta inicial /ms adecuada es: / a) _!piciar alfa agonistas adrenrgicos / ~olicitar urocultivo \ . d) 'indicar ejercicios de fortalecimiento del piso pelviano d) Derivar a resolucin quirrgica e) Solicitar ecograffa renovesical, perfil uretral y cistoscopfa 10) Una mujer purpera de 7 dias inicia dolor y eritema en la mama izquierda, asociados a fiebre hasta 38C. Al examen fsico se aprecia mama .izquierda aumentada de volumen, con eritema de la piel, dolorosa a la palpacin y de cpr}sfst~rl~ia ligeramente aumentada, de manera difusa. No se palpan ndulos ni zonas -f~s. La conducta ms adecuada es: a)...Sspender lactancia materna e iniciar antibiticos con cobertura para estafilococo ureo '.. ,))fSspender lactancia materna, realizar drenaje quirrgico e iniciar antibiticos con cobertura para ~ "'. estafilococo a u reo ~&-Mantener lactancia materna, mejorar la tcnica de lactanc:ia e iniciar antibiticos con cobertura para estafilococo ureo d) Suspender la lactancia materna e iniciar antibiticos con eobertura para bacterias Gram . negativas e) Mantner lactancia materna, mejorar la tcnica de lactancia e iniciar antibiticos con cobertura para Qcterias Gram negativas
/

11

11)na mujer de 18 i.i\os cursa embarazo de 8 semanas. Consulta por cuadro de metrorragia, fiebre, ictericia, dolor abdominal y gran COinpromiso del examen general. Al -...._____,xamen se aprecia paciente febril hasta 39C, .i_~trica, hipotensa, con salida de liquido purulento por orificio cervical externo. Se sosi)ectiari maniobras abortivas. El agente microbiolgico, que con mayor probabilidad es el c~usante. de este cuadro es: a) Escherichia coli b) Streptococcus- pyogenes e) Staphilococcu.s aureus d)J3cteroide~ .fragillis e) Clostridiu~.. perfringens . . 12) Una 1)1ujer de 40 aos, sin antecedentes de importancia, multipara de 3, esterilizada media?.t ligadura de trompas, consulta por hipermenorrea importante. Al examen fsico se cons}ata tero aumentado de tamao, de contorn~s irregulares. Se solicitan exmer:aes do2de destaca anemia ferropnlca y una ecografa transvaglnal con demostracin de un ~ioma s_~!>.nJ.~coso de 5 cm y-otro.muraLde...4..c.m. El tratamiento ms adecuado para esta \. _,paciente es: '....../a) Iniciar anticonceptivos orales combinados b) Iniciar tamoxifeno e) Iniciar agonistas de GnRH d) Rea.!.izar tumorectomla endoscpica ~};Realizar histerectomra transvaginal
/

~.

~
13)/Primigesta de 39 semanas inicia trabajo de parto. La dinmica uterina es 3 contracciones e11/10 minutos y se palpa cuello uterino borrado con dilatacin de 2 ~m y membranas ir-tegras. Sin embargo se constatan losiones vesiculares en labio mayor derecho, 9p$istentes con herpes genital activo. La conducta ms adecuada e$: / ....a) Realizar cesrea, sin necesidad de tratamientos adicionales b) Dejar evolucionar el parto vagina.t sjn 11eqesidaq d~ tr?Jta~ie.ntos adiqionales e) Iniciar aciclovir endovenoso y dejar evolucionar el parto vaginal d} Dejar evolucionar el parto vaginal y administrar aciclovir endovenoso al recin nacido e) Iniciar aciclovir endovenoso a la madre, realizar cesrea y administrar aciclovir endovenoso al recin nacido
14) Una paciente de 23 aos consulta por leucorrea de mal olor, el que se acenta durante la menstruacin. A la especuloscopia se~ aprecia leucorrea griscea de olor a pescado podrido, . "'si~.. signos inflamatorios en la mucosa vaginal. El agente etiolgico ms probable es: Gardnerella vaginalis
b)) TCrichd~dmon ab.vaginalis e n 1 a a 1cans d) N.eisseria gonorreae e) .~lamydia trachomatis

(f'

~ ~ ~

f'
~
~

\.~

~ ~.- \ ----/ ..
~ (tiA

.. fiy

1S) La causa ms frecuente de aborto es:


...a)JJlcompetencia cervical \ / !.bfAiteraciones genticas embrionarias \,_//'e) Sfndrpme antifosfolrpidos , .. d) Alteraciones hormonales e} M9lformaciones uterinas

~ ~

~
~

~
~

~
~.

(A
~

16)'Una paciente de 55 aos se realiz;! ecografa transvaginal como parte de estudio de un c;uadro de dolor abdominal. La ecografa demuestra presencia de un tumor anexial derecho ,.de 6 cm de dimetro, de aspecto qustico, con numerosas increscencias. No se observa lquido libre peritoneal. La conducta rns adecuada, de entre las siguientes, es: a) Observar y realizar nueva ecograffa en 6 meses b) Iniciar anticonceptivos orales, mantenidos, durante 3 meses y repetir la ecograffa e) Realizar puncin con aguja ffna y decidir manejo segn hallazgos d) Realizar tumorectomra ovrica, con biopsia intraoperatoria y decidir manejo segn hallazgos e.)~ealizar ooforectomfa ms anexectornra derechas, por vra laparoscpica, con estudio de biopsia ~pida
\.

~
~

'

~
~

17) El concepto sinusorragia se refiere a: /) Menstruaciones irregulares .,__ b) Menstruaciones abundantes y dolorosas e) Sangrado genital fuera del perfodo menstrual ..~rsangrado genital durante o despus de una relacin sexual _,/e) Sangrado vulvar, que no se origina en la vagina
18) Un~~aciente multipara de 2, cursando embarazo de 33 semanas fue diagnosticada de diab!!tes gestacional mediante test de tolerancia a la glucosa, a las 28 semanas de gestacin. Se inici dieta, sin embargo persiste con glicemias postprandiales cercanas a ... - 1SO mg/dl. La conducta ms adecuada es: a) Proseguir con dieta y ejercicios por 4 semanas m~ y controlar b) Iniciar metformina e) l[liciar glibenclamida

~
~

~
~

~
~

Atniciar insulina

. /e) Iniciar una asociacin de glibenclamicla e insulina

~
~
~

~ ~
~

'-.

"-.../_..:-'~)Sohcltar nuevos mveles de HCG ttY.,....

y .ecografa transvag1nal e!n 2 das ..- b) Indicar metotrexato / e) Resolver quirrgicamente, mediante laparoscopra ~f) Resolver quirrgicamente, mediante.laparotomra _.e) Dejar evolucin espontnea, sin necesidad de nuevos controles ni tratamientos
i

19) Paci"f~nte de 20 aos, con men~truaciones previas irreguJares, p_resenta atraso menst1ual de 6 ~manas por lo que se realiza test de embarazo, que resulta positivo. Presenta dolor abd9ininal, sin metrorragia. Se solicita ecografa ~pthsvaninal que no muestra saco embrionario intrauterin~. Slo se observa endorrietrio en1arosado, sin lquido libre pritoneal. Se controlan niveles de gonadotrofina corlnica (~CG,) cuyo resultado es 1.000 /~9J~~ conducta ~s adecuada $5: .
.

'

20) Una paciente de 25 aos, cursando embarazo de 35 semanas, presenta cultivo vaginal positivo para Streptococcus betahemoltico, grupo B (S. agalactiae). La conducta ms adec~ada es:
a) Administrar una dosis de penicilina benzatina intramuscular inmediatamente b}ni~iar ampicilina oral desde las 36 semanas hasta el parto ~~ministrar ampicilina endovenosa durante el trabajo de parto d) ~dministrar penicilina er.~dovenosa al recin nacido e) ,.Realizar cesrea

)~s movimientos fetales. Se realiza perfil biofsico que eSi informado como 8/10, con lquido

t1) Paciente de 35 aos, cursando embarazo de 36 semanas, consulta por disminucin .de

\
\ ../

amnitico 0/2.' La conducta ms adecuada es: a) Tranquilizar a la madre y dejar que evolucione espontneamente b) Reali~ar nuevo perfil bioflsico en 2 dfas y decidir manejo s;egn hallazgos e) Solicitar ecodoppler umbilical y decidir manejo, segn hallazgos
d) ~pt~rrumpir el parto por vfa vaginal
);r~ealizar cesrea de urgencia inmediatamente

22) La causa ms. frecuente de retraso del crecimiento intrauterino, de tipo asimtrico es: .. a) Constitucional ..... b} Genopatras .", c)J,nfecciones intrauterinas (TORCH) kq) "1 nsuficiencia placentaria 7 e) Deficiencia de insulina fetal
}~lJil examen ms importante para detrminar el pron!stico de un embarazo gemelar es: ' . ?J-La ecograffa obsttrica del primer trimestre _.. b) La ecograffa ginecolgica del segundo trimestre e) La ecografla ginecolgica del tercer trimestre d) El ecodoppler de arterias uterinas e) ~1 ecodoppler umbilical

~4) El perodo frtil se extiende desde:

-~/- b) 9Jnpo dfas antes hasta cinco dlas despus de la ovulacin


_c}. . .Cinco dfas antes hasta el dla de la ovulacin

.....-'a) Un dfa antes hasta un dfa despus de la ovulacin

. ::---;:>(f) Diez dfa antes hasta tres dlas despus de la ovulacin


../ e) Siete dfas antes hasta tres dlas despus de la ovulacin

~
25) ,Cul de las siguientes opcionc~s orienta mejor a una causa ginecolgica de dolor a~dominal crnico?
., a) Dolor abdominal hipogstrico Dplor abdominal que no se acompaa de alteraciones del trnsito intestinal .., c)..-OoJor abdominal cfclico, relacionado al ciclo menstrual .4) Dolor abdominal que cede con la eliminacin de gases ..-- e) 091r abdominal, asociado a cefalea y dispareunia superficial

\. /b)

~ ~
,

26J' Cul de las siguientes vacuna:3 est contraindicada durante el embarazo?


a} Antigripal ., __ _,,6) Antineumoccica e) Anti hepatitis B .__. d) Antitetnica ._!3fAn~ivaricelatosa
J' /

..

~ ~

~
~
~

27) Paciente de 25 aos, consulta por dismenorrea intensa, asociada a dispareunia profunda y ~isquexia, de 1 ao de evolucin, que ha ido en aumento en el ltimo tiempo. refiere l~ucorrea. Al examen ginecolgico :se constata dolor a la movilizacin cervical y a la -palpacin anexial. Tambin se cons;tata cierto grado de inmovilidad del cuello uterino. El diagnstico ms probable es:

No

e
f!"'
~
(&'
...... ...._.~

f:t>

a) Cncer de endometrio b) yrQ~eso inflamatorio pelviano -~~dometri.osis ....J),..Mtomatosts e,)" Neoplasia intraepitelial cervical

~ ~

.28) Una adolescente de 17 aos, sin1 antecedentes de importancia, es trada por su madre ya / que an no presenta menstruacin. No presenta otros sntomas y su desarrollo puberal es normal. Al examen ginecolgico se observa himen perfo'rado y el resto del examen fsico no muestra alteraciones. La conducta ms adecuada es:

f'
~
~

a) Iniciar/anticonceptivos orales b) Sol!itar ecografla transvaginal e) S~Jicitar TSH, hemograma, lndice de~ andrgenos libres, prolactina, FSH, LH, cariograma, a,X)(;ifostenediona y 17-hidroxiprogesterona ;il}- Solicitar prueba de progesterona _/.- ,e) Iniciar pulsos de agonistas de GnRH
.~;,

....

~
~

~ ~

29) Una paciente de 35 aos, cursando em~arazo .C:~A~tsemanas, consulta por fiebre y leucorrea purulen~~- Al examen se constata-r=c: 130x', PA: 110/60, t 0 : 38,3C. La especulosc.pfa demuestra salida dEt exudado purulento por el orificio cervical externo. En sus exmenes de sangre, destaca leucocit~sts h~~t~ 1.8.00.0 y elevacin de la PCR. La siguientes, es: - .. conducta ms adecuada,'CJeentre

las

~ ~
~

~
~
~ ~
~
/
,, ,

;'

a) Administrar corticoides, antibiticos 3ndovenosos de amplio espectro e interrumpir el embarazo al alcanzar la madurez pulmonar b) Solicitar amniocentesis, iniciar antibiticos y corticoides y decidir manejo segn estudio del lfquido amnitico e} Realizar cesrea de urgencia, sin necesidad de antibiticos d) Solicitar urocultivo, radiogratra de trax, hemocultivos y decidir manejo segn resultados ~)';,.Administrar antibiticos endovenoso:s e interrumpir el parto por una vla expedita

~ ~ ~

f"'

) J:

El punto de reparo de la presentacin de cara es: ) Fontanela anterior b) Ojos e) Nariz

~-

. )~~n
8) La elevacin de la alfafetoprotena es un marcador de:

::. . f Sindrome de Down

~ ff!'

~fect~s del ~L-> e nsom1a d) Hernia diafragmtica congnita e) Restriccin del crecimiento intrauterino

~ ~

~
,

~.) Paciente multipara de dos en trabajo de parto, cuyo monitoreo muestra una FC fetal basal de 130/min, buena variabilidad y presem:ia de aceleraciones. La rotura artificial de membranas da salida a un liquido con meconio fluido, amarrillo verdoso. Cul es la conducta obsttrica a seguir? a) Interrupcin inmediata del embarazo mediante cesrea b} Reanimacin fetal in tero con reevaluacin en 20 minutos e) ~al)imacin fetal in tero seguida de- interrupcin del embarazo -~;tol~litener una conducta expectante y permitir que contine el trabajo de parto e) Interrupcin inmediata por la va ms expedita
_. 10) A las 34 semanas de gestacin un feto pesa aproximadamente:

,/ a) 500 a 1.000 g
b) 1.000 a 1.500 g e} 1.500 a 2.000 g j.ft.OQO g 2.500 g r~ e) 2.500 g 3.000 g

t!'
~

'-../

~ ~

~
~

~ ~

.11) Una paciente con precJamsia severa presenta cefalea Intensa y luego una convulsin tnlcoclnlca : autolimltada. La conducta ms adecUi~da es: ''-.._.../' a) ~dministrar sulfato de magnesio, corticoides endovenosos e inducir el parto con misoprostol . .;;btlniciar sulfato de magnesio estabilizar a la madre e interrumpir el. embarazo e) A<:;lmir:~istr-ar-lorazepam y realizar cesrea de urgencia d) Adm.lnistrar: sulfato de magne-slaysolicitar tomografiaaxial computad~!' de cerebro e)'.Reaiizar cesrea inmediatamente, sin. uso de-medicamentos~
.......

~
~

12) ~ biop~-~~-~~d~~-~~ial, con reaccin de Arias Stella se 1/ _p)'i~~borto completo, embarazo ectpico y uso de progestgenosobserva en: ... exgenos_.
b) EmbJJazo.molar; -embar.azo-eetpico l' uso de estrgenos exgenos e) EmJ):r.azo-molar; climaterio y. uso..~~ progestgenos exgenos d) A~orto completo, climaterio y uso de e strgenos exgenos e) _Embarazo normal, anovulacin crnica y uso de anticonceptivos combinados

~
~
.

/1,3) Una gestante de 32 semar1as cons uIta por metrorragia y contracciones uterinas dolorosas. Al
examen ffsico destaca tero hipertnico. El diagnstico ms probable es: .a}. Dsprendimiento previo de placenta normoinserta -:6) Placenta previa e) Rotura uterina d) Rotura de vasa previa e) Acretismo placentario

~
~
~

~
~

f'!A
~

'!;
~
~
~

1) Una gestante de 28 semanas se realiza test de tolerancia a la glucosa, como parte de un control del emba~o, que muestra glicemia de ayunos de 100 mg/dl y de 155 mg/dl, a las 2 horas despus de reci~ir 75 g de glucosa oral. Se encuentra asintomtica y el examen fisico slo muestra los cambios n~ales del embarazo. La conducta ms adecuada es: a) Solicitar nuevo test de tolerancia a la glucosa oral con 100 g de gluc-osa ~) S_9licitar hemoglobina glicosilada ~)'1niciar dieta y ejercicios d) lnciar medidas generales y metformina e). . lniciar medidas generales e insulinoterapia
'/

,./ 2) Una mujer de 36 aos, primigesta cle..30...smnanas, presenta preE;in arterial de 144/92 mmHg en un ' control ginecolgico. Se controla la presin arterial en varias opo11unldades dui-aiiles horas, ' constatndose varios valores mayores de 140/90. La conducta ms adecuada es ~teontrolar frecuentemente la presin arterial por 48 horas ms y decidir manejo segn resultados ~ ~olicitar proteinuria de 24 horas e) Solicitar hemograma, pruebas hepticas, LDH, tiempos de coagulacin, clearence de creatinina, proteinUiia de 24 horas, electrolitos plasmticos d) Indicar dieta hiposdica y ejercicios e) Indicar dieta, ejercicios y alfametildopa
) Una mujer de 25 aos, cursando embarazo de 8 semanas de amenorrea, confirmado con test de eteccin de beta-HCG urinaria, consulta por dolor hipogstrico Intenso y metrorragia. Evoluciona con potimlas y ortostatismo. Al examen fsico se aprecia plida, con frialdad de extremidades, FC: 124x' PA: 80/50 mmHg. El diagnstico-de sospecha es: . a) Desprndimlento de placenta normo inserta b) Rotlfra uterina e) )\borto incompleto mbarazo molar mbarazo ectpico roto Cul de las siguientes alternativas NO es una mdacaclon de admmistrar mmunoglobuhna anta-O a . .. ,.afPrueba de Coombs indirecto positiv"--~ b) Amnio9eritesis .. e) Postprto ..... d) Aprto completo........e),28 semanas de gestacin
u~paciente embarazadaJ~~-~!~ativo?

1
/5) Una paciente de 22 afios, cursando embarazo de 29 semanas presenta salida de lquido claro, con , j olor a cloro, por genitales, desde hace algunos dias. No presenta otros sfntomas. La especuloscopia . C') ~Vr- es normal y el test de cristalizacin presenta formacin de "hojas de helecho". La ecografia obsttrica 1 .~ r.J demuestra un feto en ceflica, con oligoamnios, sin otras alteraciones. El registro basal no estresante ~,t.- . \f es tranquilizador. La conducta ms adecuda e,s: . " L\\.. a) Interrumpir el embarazo por la via ms expedit-.~-:./ "-. .: b) Inducir el parto con misoprostol A-!Administrar corticoides y antibiticos a 1~ madre, manteniendo una .conducta expectante respecto al parto j \; ":"\ 'l) Administrar corticoides y antibiticos e interrumpir el embarazo en 48 horas "' , \Y e) Rea\i:ar cerclaje de emergencia e iniciar antibiticos . ~
,.

1 ~\.\f? V~ v \-

U"'"-.)'

V
/

6) un'a mujer de 25 aos consulta porque present un aborto espontneo a las 8 semanas de su primer e~barazo. Actualmente se encuentra usando anticonceptivos orales, pero desea un nuevo embarazo. L conducta ms adecuada es: Solicitar ecografa transvaginal y decidir manejo segn hallazgos b) Solicitar anticuerpos anticardiolipinas, progesterona pla~mtica de fin de ciclo, ecografia transvaginal, prueba de Hegar y de~idir manejo segn hallazgos e) Suspender anticonceptivos y administrar progesterona en la segunda mitad del ciclo menstrual c;!)~e.alizar seguimiento folicular y .induccin de la ovulaci~n con clomifeno )~trnquilizarla e indicarle que puede intentar un nuevo embarazo sin necesidad de estudio

')

"-...J
,..

------- - - -~\ 14) Uha mujer de 25 aos, cursando ~mbaraz~de-9-sern.@_r~as 'inicia metrorragia abundante. La esl?culoscopfa demuestra salida de sangre y restos ovul~ por el OCE, la ecografa transvaginal vi~ualiza el saco gestacional de contenido irregular y el tacto vaginal constata dilatacin cervical de 2 El resto del examen ffsico es normal. La conducta ms adecuada es: /a) Solicitar niveles de HCG b) Administrar misoprostol intravaginal e) Realizpr cerclaje de emergencia d) \ndigr reposo y dejar evolucin espontnea /e( Rjalizar legrado uterino

s:m.

19). Una multpara de 34 aos, cursando uu cuarto embarazo, de 40 semanas inicia dinmica uterina. Al

v
~
18\

~xamen se constatan 3 contracciones en 10 minutos y el tacto vaginal muestra un cuello :completamente borrado, con-4-cm_d.Q.giJ.ntc\n y membranas inteQras. Se instala monitoreo fetal que de"'lasco-traccic;"'"nes uterinas, sin decalage, en / muestra de~celerciones en relacin a la

_:fb~i~rc~;o~~=:~~~~~~~;ta~lll~;

aaecaaa es:- - ------------- . . .

mayarTa.

b) Ad~inistrar oxigeno, poner en decbito lateral izquierdo e indicar anestesia epidural e) lnjiar aceleracin occitcica . d} ~ealizar rotura artificial de membranas y observar el liquido amnitico !Realizar cesrea de urgencia
~
~

6) La complicacin ms frecuente de la cesrea es: ) Hem9h-agia intraperitoneal b) ['Jlet~orragia ,,afbt;tiscencia de la herida ~telectasias pulmonares nfeccin de la herida operatoria Fenoterol . b) Occitocina e) Alf~metildopa d) rv1etilergonovina e) Prostaglandina \

~Cul de los siguientes frmacos es un tocolftico?

""-

.J:9)

Cul es el mejor parmetro para estimar la edad gestacional en un embarazo avanzado, sin controles previos? a) Altura uterina }!li.~jo de la percepcin de movimientos fetales Fecha de la ltima regla confiable y segura d)_.,Ecografa ef Niveles plasmticos de beta-HCG

/s)

El una ecografla del primer trimestre de un embarazo gemelar se observa el signo lambda. Qu

tipo de embarazo gemelar es? a) Dicigtico b). Monocigtico .....~ B,itorial -:.. d).Monocorial ~) Monoamnitico // ,.._/ 20)J-a primera causa de muerte materna on Chile es: d~~indrome hipertensivo del embarazo / ..:'.b) Hemorragia puerperal e) Aborto sptico d) Enfermedades crnicas descompensadas por el embarazo e) Embolia de liquido amnitico

RUT

o
NOMBRE:

2
3
4

00 000 000. o ,-oo 000 000 o

RUT:

7
B

FECHA:

9
K

00 000 000 00 000 000 00 000 000 00 000 000

00 000 000 00 000 000 00 000 000

00 000 000 O

o o o o o O. o
O E
61
A B C O E

CURSO:

RESPUESTAS
A B

e:

E
31
32

, OOC>OO 2 OOC>OO
3 4

5
6

7
B
9

.
" t=
:

10

00()00 00()00 00()00 00000 00()00 00()00 00()00 00()00

00000 00000

6~
63
64

00000 00000

33
34

35 36
37
38 39

00000 00000 00000

65

00000 00000 00000


00000 00000 00000 00000 00000 00.000 00000 00000

40

00000 66 00000 67 00000. 68 00000 69 00000 70 00000 00000 00000


71

11

12
13

00000

41

14
15

00()00 00()00 43 oo<)OO "' 44 ooooo 00<)00 45 00000


42

72
73

74
75

16

17
18 ' 19
20

00000
00()00 00()00
00000

46

00<)00 00()00

47
48 49

00000 00000.

00000
00000

76
77
78 79
BO

OQQ,OO 00000

00000

00000

ooooo

soooooo
51

00000 00000 00000 00000

~ ~ ~

21

00000

Bl

22 23
24

ooooo
00000

00000 00000

52 53
54 55

25 26

00000

00000 00000
4 BS

ooooo.

00000

82 83

ooooo

00000

~ ~ ~ ~
~ ~ ~

00000 27 00000 ~8 00000 29 00000 30 00000

56

57
58

59
60

000.00 00000 00000 00000 00000


f

86

87
88

89
90

00000 00000 00000 00000 00000

~
~

~ ~

!'

30) Un~ paciente purpera de 7 dias inicia metrorragia de moderada cuantia. La causa ms prob~llle es: a) Inercia uterina b) ,~ac7raciones del canal del parto c)"~~qometritis puerperal \, /JifPefsistencia de restos ovulares '-~/e) .drenaje espontneo de hematoma del canal del parto
(

~
'

~ ~
~

. 31) Cul de las ~jguientes es una .'a) Mioma uteriQ~subserosq);-~--'\ ',,_/ _bf'Plipo endometrial ...... e) Uso~~. anticonceptivo.s d) Cn.cer:.de-endometrio :.. .. ..-: e) Sl~r.ome.de ovario poliqurstico

c~~$a clsic' de hipermenorre~:?


' ----... -- :...... ____ -

...... -

32) ~na mujer de 62 afios, obesa, con antecedente de ovario poliqustico durante su perodo
f~l, presenta metrorragia e~casa. A la especuloscopfa se constata que la sangre proviene d~. la cavidad endometrial. El tacto vaginal es normal, al igual que la ecografia transvaginal.

f:!'
~

~
~

fA
~

):a conducta ms adecuada es: ..._, a)Jntlrtar anticonceptivos orales en altas dosis ... d!>)'Solicitar biopsia endometri~l ::):..;e) Iniciar estrgenos locales .. / d):Solicitar Papanicolau, <?Oiposcopia y biopsia cervical ~)Observar y controlar en 7 dlas, decidiendo manejo segn evolucin
33) Son con_traindicaciones p~ra la t~ara__pia hormonal de reemplazo, EXCEPTO: a) Metrorragia de causa no precisada\... / b Cncer de mama '- o Cncer de endometrio , /J) tfepatopatras activas \..,/ Jl?} Cncer de cuello uterino
. .
1

fA
~
~-

.l

(1$'
~

r.

'v

~
(fi'

"

34) Una paciente de 24 aos, consulta por dolor abdominal. Al examen se constata paciente subfebril, con dolor a la palpacin hipogstrica! palpacin anexial derecha y movilizacin cervical. No hay signos de irritacin perito11eal. El diagnstico ms probable es: a) J;ndometriosis : A?fProceso inflamatorio plvico ,_>'e)' Apendicitis aguda dj Absceso tuboovrico roto t1> Embarazo tubario

~
~

/35) Paciente multpara de 33 aos, cursando embarazo de 28 semanas, asintomtica, se ,'--) realiza urocultivo, como parte 'de su c~ontrol prenatal, el cual resulta positivo para Escherichia coli sensible. La conducta ms adecuada es: a} No realizar nuevos urocultivos ni administrar antibiticos, dada ausencia de srntomas ~)Administrar ciprofloxpcino--uralsin necesidad de urocultivos de control J~}-1(drninistrar cefadroxilo oral y realizar urocultivos peridicos --"Jd) Administrar ceftriaxona endovenosa y realizar urocultivos peridicos 1e) Realizar urocultivos peridicos sin necesidad de tratamiento antibitico 36) Cul es la causa ms frecuente de tumor anexial en las pacientes en edad frtil? a) Hidrosalpinx b) Absceso tuboovrico . ~feuistes de origen funcional .~- d) Adenoma e) Cncer de ovario

~ \ ) ~ '~

~
~

"'

37),.11ujer de 18 aos, consulta por reglas irregulares, las que presenta cada 2 a 3 meses. Al eJSmen se aprecia obesa, con hirsutismo leve, acn y piel grasa. La causa ms probable de s. alteracin menstrual es: /~} Hipotiroidismo w$rndrome de ovario poliqurstico /e} Hipogonadismo hipergonadotrpico d} Hi~2rplasi suprarrenal e} Ttynor productor de andrgenos
1

''una pacj~nt~. P.rimigesta, de 30 seman. as de g.estacini, asintomtica, presenta presin erial de 1~0/90 en una ocasin. El resto del ex~men fsiico es normal. Se controla oteinuria cualitativa que resulta levement~ ..Positiva. La conducta ms adecuada es: Iniciar diurticos, asociados a iECA b} !J:)dicar dieta y ejercicios .-::C)1ndicar semihospitalizacin de 6 horas, controlando la presin arterial en varias ocasiones ./ d) Iniciar alfametildopa y solicitar exmenes que determinen la gravedad del cuadro e) . dministrar corticoides e interrumpir el embarazo 3 ) Paciente de 55 aos, presenta genitorragia. A la especuloscopfa se observa una lesin e 2 cm, con una pequea ulceracin. El Papanicolau resulta normal. La conducta ms adecuada es: \"a}_,~epetir Papanicolau en 6 meses ,t.i}'Uerivar a colposcopia y biopsia .Yc) Relizar cono bipsico d) R~alizar histerectomra simple e) Realizar histerectomfa radical

.
\

'V

40) Una paciente fumadora, multpara de 2, cursando embarazo de 32 semanas consulta por metrorragia y dolor abdominal. Al examen fisico se aprecia metrorragia escasa, oscura y se palpa tero hipertnico. El monitoreo fetal demuestra variabilidad disminuida y desaceleraciones variables complejas. El diagnstico mis probable es: a) ~acenta previa ~esprendimiento prematuro de placenta normoinserta e) ~otura uterina . d)._~Rotura de vasa previa Acretismo placentario

T)

'.\,_ j

41) Cul de las siguientes medidas est contraindicada en una paciente con rotura prem~tura de membranas, que inicia dinmica uterina a las 30 semanas de gestacin? a) Administrar corticoides b) Administrar antibiticos -e.l.Administrar tocolfticos dYRealizar amniocentesis ~) Realizar especuloscopla

....

/ 42) Una mujer, sin antecedentes mrbidos, cursando entbarazo de 28 semanas presenta hemoglobina de 1 O mg/dl en hemograma control. La conducta ms adecuada es: a) Controlar con hemograma en un mes '~b]4Tatar con fierro oral e) Descartar hemorragia oculta d) Administrar cido flico e) Solicitar cintica de fierro

~
43) ql de las siguientes presentaciones tiene opcin de part~ vaginal?

~ ~
.

) N 'soiliaca izquierda anterior


" ,_.
entopbica en tosa era "'-.../' d) Nasosacra e) Acromioiliaca derecha posterior

.j

~ ~

e
~ ~
~

44)

r~gistro

.\ ~:>ntrcy>Ciones uterinas. La frecuenc~la cardiaca basal del feto es 130 lpm y la variabilidad es ".../no..!JTf'al. La conducta ms adecuada es: ,,_.....- va]Realizar un tacto vaginal para determinar la vra de interrupcin inmediata b) Solicitar ecodoppler umbilical e) Realizar perfil biotrsico d) Esperar evolucin espontnea e) A~ministrar corticoides endovenosos, antibiticos y tocollti~os 45(Paciente de 50 aos, consulta pt>rque sufre de escapes de orina, desde hace 6 meses, Is que se han hecho ms frecuenlE!S en el ltimo tiempo. Refiere que siente ganas muy \ Jntensas de orinar y que muchas ve~Ces le es imposible alcanzar a llegar al bao, mojando su ' ropa con grandes cantidades de orina. No presenta escapes con la tos ni con el Valsalva. No presenta otros sntomas y el examen fsico no aporta mayor informacin. El diagnstico s probable es: ) Incontinencia urinaria de urgencia ) Infeccin urinaria e) Cistitis intersticial d) Incontinencia urinaria por hipoestro9enismo e) Incontinencia urinaria de esfuerzo

yru; paciente demuestra desace:leraciones tardastrmino iniciaaltrabajo de parto. El 22 aos, cursando embarazo de estresante en relacin 80/o de las

~
("

~
~

~ (!A

~
~

~
~

e
~
~
~

46) Una paciente de 55 aos, asintorntica, se rea.Jka.Jll.amograf(a, como parte de su chequeo de salud, la que es informada como 1 Eflrrads 4.. La conducta ms adecuada es: a) Repetir la mamograffa en 6 meses _./ .. b) Solicitar ecogratra mamaria e) So,icitar resonancia magntica nuclear ~rivar para realizar biopsia estereotxica e~erivar para resolucin quirrgica inmediata

\"'~ ..

./ ~

i
'. ,_._. i

~~~Paciente de 34 aos, cursando etnbarazo de 8 semanas, con hipermesis gravdica, consulta por metrorragia. Al examen fsico se palpa el tero a la altura del pubis y el tacto

"'.

~
~

./ vaginal no presenta alteraciones cervicales. El diagnstico de sospecha es: / a) Aborto retenido ~ b) Aborto incompleto ) la parcial . ola completa mbarazo ectpico

--.

~ ~
~

(ii';

~
('!A'

V
.

a);Eclamsia ~bolia de Hquido amnitico ~rndrome de HELLP : ClfC"olestasia intraheptica del embarazo e) Hfgado graso agudo del embarazo

48) Paciente de 25 aos, primigesta, cursando embarazo de 30 semanas ingresa por intenso dolor epigstrico y gran comprom~e-del.~stado general. Al examen se aprecia paciente en .regulares condiciones generales, ctric~on frecuencia cardiaca de 125 lpm, presin .arterial de 150/100 mmHg. La palp inabdominal es muy dolorosa, especialmente en el / hipocondrio derecho. Se solicita proteinuria cualitativa que resulta +++ El diagnstico ms probable es:

9j

~---,.

.......\

49) El punto de referencia en la presentacin d~'frente ~s: p~ ~La fre~te ,,._ . ./ .


~nanz

e) El mentn d) El Bregma e) El Occipucio

O) Cul de las, siguientes alternativaf~~ ~~ un factor da riesgo para el cncer de


~

ometrio?

Uso de anti(fonceptivo~orales Hipertens_!P'n arte9av-' e) Diabetes / d) Obesidad e) Slndrome de ovario poliqufstico

r;
RUT

~
o
NOMBRE:

e!'

00 OO 000 o ,.00 000 000 o 2 00 000 000 O


3
4

~
RUT:

~ ~

S
6

~
~

7
8

00 00 00 00 00 00

~ ~

FECHA:

000 000 000 000 000 000 00 000

000 o 000 o 000 o 000 o 000 o 000 o 000 O.

CURSO: RESPUESTAS A
1

~ ~

B C

O E
31

B C

O E
61

A
6

O E

~ ~ ~
~

00000

2
3

00000

~ (!A

~
~
r

.
l,

00000 00000 S 00000 6 00000 7 00000 B 00000 9 00000 10 00000


11 12 13 14 15 16 17 18

00000 00000 33 00000 34 00000 35 00000 36 00000 37 00000


32
38

00000 00000 63 00000 64 00000 65 00000 66 00000 67 00000


~
69

00000. 68 00000

39

40

00000 00000. 00.000 00000 00000

70

00000 00000

fA
~
~

.r.

.
':

00000 00000 00000

41 42 43 44

71

72

~
~

00000 00000

ooooo \ ooooo
45 46

00000 00000
00000 00000 00000

~ ~

19 20

00000 00000 47 00000 48 00000. 49 00000 so 00000


51 52

00.000 00000 73 00000 74 00000 75 00000 76 00000 n 00000 iB 00000 g 00000 80 00000
81 82 83 84 85 86 87 88
89 90

~ ~ ~ ~ ~
~

21

22
23
24 25 26

00000

27
?B
29
JD

~
~

00000 53 00000 54 00000 55 00000 56 00000 57 00000. SB 00000 59 00000

00000 00000 00000. 00000 00000


000.00

SO

00000 00000 00000 00000

00000 00000 00000 00000 00000 00000 00000 00000 00000 00000

~
~

e
~

.
EUNACOM 2009

/)>- Al\

!'
SEGUNDA PARTE 1) Un paciente est convencido de ~aclecer un cncer a pesar de contar con numerosos exmenes demuestran lo contrario. El (liagn<'>stico ms probable es: " .. rastorno disociativo b) Trastorno psictico residual _:"'e) Traslerru sotnatomerfo v' fd) Depr~sln t:r:tayor. ;>Trastorno conversivo /~

~ ~

f'

t:

t'
~ ~ ~ ~

j
J
j

.Lf

2) Una mujer consulta por prur~o vulvar de una semana de evolucin y leucorrea. A la especuloscopfa se observa abundante leucorrea grumosa, con signos Inflamatorios en la pared ~~inal. El agente etiolgico ms probable es: . ~andida albicans b) Escherichia coli e) Gardnerella vaginalis d) Trichomona vaginalis e) Clamydia trachomatis

f!'
~

f!!'
~

3) Un paciente de 27 aos presenta un cuadro de cefalea, coriza y tos de 7 dfas de evolucin. Desde -'yer se agrega diSei"Yflebre hasta-'31~AI examen ffsico presenta FC: 80x', PA: 120/80 mmHg, / F~~ y se aus~ltan creprtos escast s ifusos en ambos pulmone~. La radiografa de trax demuestra un patror- alveolo-lhterstlelail bilateral, mayor en las bases. El agente causal ms probable es: a) Streptococcus pneumoniae b) Streptococcus pyogenes ~ycoplasma pneumoniae d) $taphilococcus aureus e).Ni rus influenza

(!'
~

~ ~
~ ~

; El mejor tipo de estudio para determinar la asociacin entre un factor de oco frecuente es: "' ohorte aso control e) Ensayo clinico --d) Transversal e) 0o91e ciego

riesgo._v-Una~enfermedad

~ ~

~ (!i!'
~

5) .~ recin nacido d ~ alimentado con pecho materno exclusivo, inicia ictericia. ~ociado a cel na yac Ha .. AI examen fsico preserita"icte~i~ia hasta los muslos y hepatomegalia. El dragns co m s pro a e es: a) Ictericia por lactancia materna b) Ictericia fisiolgica e) Ictericia hemoltica tarda, por incompatibilidad de grupo ~~tresia biliar primaria. e) Sindrome de Gilbert 6) U~. paciente de ~s sufre dos sim:opes en relacin al esfuerzo. Al examen ffsico presenta un pulso amplio, regular, a 72 lem, Coi1 pre1Jio arterial normal. Se ausculta un soplocStSfc)lico eyectivo' ~m~yor parae.stemal izquierdo. Se solicita un electrocardiograma que muestra onda T negativa lgunas derivaciones precordiales. El diagnstico ms probable es: Miocardiopatia hipertrfica b) Miocardiopatia dilatada e) Estenosis artica d) Estenosis mitra! e) Angina inestable

e
~

~
~

e
~ ~
~

cuada es: . Hospitalizar para manejo metablico niciar hipoglicemiantes orales ~blicitar nueva glicemia de ayuno .. d) Solicitar test de tolerancia a la glucosa e) Solicitar hemoglobina glicosilada

~~embarazada de 7 semanas presenta una gllcemla de ayunos ~La conducta ms


----

Jf

40 semana de estaci ngresa en traba~ de parto, en dilatacin co~ple~~~n feto e ',s inas - dinmica uterina de 4 en 1O r~utos y membranas rotas. Luego de ras no se produce ese so y se palpa feto en posacaon occapito llfaca derecha transversa. La nducta ms a~ecuada es: Dejar evolucin espontnea Realizar extraccin con forceps e) Realizar rotacin manual hasta posicin OP d) Iniciar aceleracin oxitcica ~ea~!~r cesrea ,----------._ ___

. . 8)u na pacaente embarazada

~~Ulesivas en una oportunidad. Refiere adems historia de paUidez y molestias abdominales de 1 e;P:~~=:a~~:::~~~~=~~~:,~;d~u:~:o~ea~~~~n~~-~t~.:~~~=e~~3d~:~~~~:~'!'~\~~~~~s e
es deevolucin. El examen "fsico muestra un pacle.pte plido. con abdomen blando, depreslble y el tacto rectal demuestra una ampolla rectal sin deposiciones, sin hallar otras alteraciones. Cul es el diagnstico mas probable 1 Vlvulo de colon sigmoides ncer de colon ) Obstruccin intestinal por bridas d) Sindrome de Ogilvie e) Divert~.culitis 1O)

ex~en se constata t~oriaco y no se auscultan latidC's cardlofetales. Se realiza ecografa


csttrica que confirma un bito fetal y el tacto vaginal demuustra metrorragia escasa, de color scuro. El diagnstico ms probable es: ) Placenta previa .x b) Rotura de vasa previa Y* otura uterina Desprendimiento prematuro de placenta normoinserta ) Ell)bolia de liquido amnitico
'

y~ mujer cursand~~nas ~tacin inicia contracciones uterinas muy dolorosas. Al

J:

~----....._,.

Una mujer embarazada d 33 semanas, con antecedente de enfermedad reumti~en su infancia s la ci en fisaco se aprecia dasnelca, con resenta disnea y ortopnea d frecuencia tar atidos por manut resin arterial de 90/60 mmHg, sudorosa. E~ diagnstico ms probab e es: a) Taquicardia paroxstica supraventricular b) Enfermedad del nodo sinusal e) Embarazo normal ~ibrjlcin auricular ~ quicardia ventricular 1

La conducta ms adecuada ante un hematoma del tabique nasal es: bservacin b) Antibioterapia de amplio espectro c::~ompresin con apsitos ~enaje inmediato /) Taponamiento nasal anterior

~
13) e,~iente de 50 aos, con tos y expoctoractin de exinien fsico presenu:,pR: ~ PA:T.roJSW'FC: 9x, to fobase izquierda, co!!_ roncofonfa. "Se"!RStrnita una radio

~ ~
f'

...d~volucin, asociado a disnea. Al


,

' . Se auscultan crpitos localizados en a de trax que demuestraua- --- --- - -

~f~~~~=~~~~n: en el lbulo .nfenor Izquierdo. El antlmic~o_de_elecchSn ~s: __ _


b} Cefuroxima '?1teftriaxona

~ ~ (f'

~~moxicilina

e} Metronidazol 14) .l.o que luego de recibir su vacuna de los 2 mes!s, presenta una convulsin. Cul de los

;;p~nentes de la vacuna es el ms p1-obable. responsable?

f!!'

r'
~
~
~

e
~ ~ ~

ertusis . Hepatitis 8 ~ F\ e) Polio d} Difteria e) Tt~!los 1 15) Un hombre de 76 aos presenta varios sincopes en las s poroso, hipopertundido, tiene una frecuencia car taca en a tJ _ _ mm g. e auscu ta un primer ruido cardia ~s probable es: ~ ~loqueo AV comple,u--" b) Enfermedad del odo sinusa e) S irttooe-\tast~:taP-__.~ d) Sn91;eme coroflarie egYdo e) Fib~~ricular
1

'

~
~

16)'6n hombre de 75 aos presenta aumento de volumen escrotal, de 6 cm de dimetro, indoloro y consistencia quistica. El diagnstico mn probable es:
) EpidJdjmjtJs-

~
~
~

b) Varicocele e} Cn~r de testculo J'Hiqrbcele e1rnia inguina.e.scrotaL 7) Un anciano de 78 aos presenta sim:opes al incorporarse. Hace un ao se realiz una artroplastia e cade~ est postrado hace 4 mes~s. Cul es la causa ms probable de este cuadro? ~ ) Sincope cardiognico / ..btfromboembolismo pulmonar V' e) Epilepsia focal . d) Accidente vascular enceflico e} Enf~_rynedad ~oronaria de las articulaciones interfalnqicas de 1~ manos. Presenta rigidez rh~~Sminutos de duracin. Se solicita radiografa de manos que emuestril.!!ismlnucin del espacio interartjculgr, c~unos o=epfitos. El iagnstico mas probable es: ) Artritis reumatoide b) Artritis por cristales .rtrosis ~) Lup~s eritematoso e) Artritis viral parlisis facial bilateral es:

~
~
~

~
~

18)?n paciente consulta por J:)Q!Q_r y~1ento de volumen

~ ~
~

J:

~ ~
~

~
~

La causa ms frecuente de una ~paresia flcida, con arreflexia risis miastnica ~olirradiculoneuritis ,V . e} Infarto de troncoencfalo d) Lesin medular alta e) Mononeuritis mltiple

~
~ ~

ft
~

A _

. /

,~\'enlafaxina
e) Omeprazol

21) Una paciente est en tratamiento con cirpofloxacino por una plelonefritis ~gy_da por E ...cali sensible, sin embargo permanece febril ycon disii.la,-a pesar de llevar 5 dfas de tratarniento_:' antibitico. Adems usa crnicamente o~prazol. salbutamQ.b_ aspirina,. ven~~faxin~i:c~l~~cuns .. vitamina D, por diversas patologfas. Cul de los frmacos efs el causante de esta situacin, con mayor probabilidad? a) Salbutamol b) Aspirina _,e) Calcio 1Vitamina D

22) El xmen ms costoefectivo para evaluar la patologa del manguito rotador es: a) R iografia cogratra )..Gintigrafia d) Tomografia axial computada e) Resonancia magntica nuclear

~onsulta por tos con expectoracin m u cap milenta, aso"'ciada a fiellre-hasta 38,~~C y dolor torcico dereclio, que aumenta con.Ja.JnJ~I examen presenta FC: 9Jbc!., PA: 1~mmHg, frecuencia respiratoria: @ y se ausculta disminucin del murmullo pulmonar e9 la ba..se derecha, asociada a crepitaciones en la misma zona. Se solicita radlografla de trax que depfestra una condensacin en el lbulo inferior.-~~~~~~~p_'?i!~~ a un derra~e pleural moderad9. : concta ms adecuada es: . ) Iniciar cliodamieiAa oFBI y realizar nueva radiografa de trax eri480ras ) Iniciar levofJoxacino y controlar evolucin clinica ~ospitalizar y realizar estudio de liquido pleural d) Enviar a domicilio con amoxicilina por 7 dias e) Iniciar ~moxicilina ms ~cido clavulnico y realizar toracocentesis en 48 horas
23) Un paciente de 24) n'~aciente de (l""~, con antecedente de fi~bre reumtica en la infancia, consulta poulisnea d esfuerzos, ortopnea y di$nea paroxrstica nocturna de 2 meses de evolucin. Al examen flsico resenta rubor tacaal, aumento de la Intensidad del primer ruido, con un ruido de tonalidad alta a inicio del aaastole,seguido de un sopo e ona 1 a aJa, ruco. 1diagnstico ms probable es: a) Estenosis artica - b)'~stenosls tricuspidea _ ~~~ ~ ( ~stenosis mitral "' l /7"f'o.-/\\)o.._ , --\ ..._ . d) Insuficiencia mitral vv .. \' \ ! _ -- r : e) Insuficiencia artica . V)~~........... '. \

25) Un recin nacido datrmioo al nacer. Es evaluado a pesaJ lo mismo que al nacer. Es alimentado con lactancia materna exclusiva. La conducta ms adecuada es: . (1B)..Mantener lactancia materna y reevaluar en 3 di as ~Mantener lactancia materna y reevaluar en 7 dlas L~j e) Mantener lactancia materna e iniciar suplemento col) frmula d) Suspender lactacia materna e rnrcrar alhentaeiA-eeA-fffftUia de inicio e) Suspender lactancia materRa e iRieiar alimentacin paFaeRtera~

pes~amos

IQ!...~

d J.

~
26) Una paciente de 33 a9$1)resenta debilidad muscular y amenorrea de 6 meses. Al examen ffslco destacan presin arterial~ 160/1 oolnmHg, pltora facial, obesidad troncular y estrias abdominales violceas. La conducta adecuada para proseguir el estudio es: a) TAC de de glandulas suprarrenales b) RMN de silla turca J l - --n:._ .. ,..)/( CAJ~ '-J~ ) -J .)"'._Cortisol basal Jqt-):ortisol post dexametasona ~ ACTH basal

f'
;

f'

f'

--

m;

~ (f'

~.

. .

J'
J

~ 27) Una mujer de'~con m_!!lo!'lutrorragia se realiza ec..o.grafa.transvaginal-que-demu~stra-a


miomas submucosos y un endometno heterogneo y de grosor aumentadp. La conducta mas adecuada es: A ~ :-t.e>~ /~) Realizar miomectomia ~

'f

~
~

~ealizar histerectomfa :I!J. ~~iciar anticonceptivos combinados orales d) Iniciar progestgenos solos por va transdrmica ,r(Solicitar biopsia endometrial -

-f._.

~
~
~

,.. ."

28) _Mujer de 35 aos, ce.$.@!!_zada antericlr, cursando un nuevo embarazo de trmino, inicia trabajo dEt p~rto y 2 horas despus evOliiCiO'mron intenso dolor aodommal y metrorra la escasa, asociada a /hipotensin. 1mom oreo e a muestra ra 1car 1a e a sos en a. iagnstico ms probable es: a) Desprendimiento prematuro de placenta normoinserta b) Rotura de vasa previa e) Embolia de liquido amnitico d}glacenta previa BRotur uterina
~~-

(!'
~
~
~

29) n lactante presenta aumento de volumen escrotal, con prueba de transiluminacin positiv . El d,gnstico ms probable es: Varicocele ~orsin testicular

4 ~idrocele l) Orquitis

e) Her~_ia inguinal indirecta

~
~ ~
_

~
~

J:

30)-n paciente ha dejado de salir de su casa, ha descuidado el orden de la misma y tambin su spec~personal. Es m4_1' unido a la religin y lee la Biblia a daario. Escucha voces que comentan odas sus actos y_q.ue_lo insultan. El dlagnsbco mas probable es: a) Depresin psictica . b) Trastorno delirante crnico ~squizofrenia paranoide d) Trastorno adaptativo e) Episodio psictico reactivo

~
~

~
~

pacie~te de 3~ consulta por placas eritematosas que aparecen en los codos, tronco cuero cabelludo, que aparecieron hace 4 meses, con p o os e remisin y exacerbacin y que eo /e(ttfro tiempo han aumentado en tamaOfCantidad. El dagnostico ms probable es:
31) U,na

/a) Q@r.i8ttns=seeetTetCa

~ ~
~

.(e) L~en plano

s ~soriasis

b) ~asis rosada e) DermatitiSde contacto

~
~

J;

2) Un nio de 4 aos pr enta inapetem:ia y prurito anal y oral. La madre detecta un gusanito de 5 mm en la zona perianal. El iratam ento e eleccin es: a}Jinidazol e) Prazicuantel d) Clindamieina e) Cotrimoxazol

~~ebendazol

~
~ ~ (/1'
~

(lA
~

33)

1?/~efropata diabtica Inicial se defin~ como:

tcroalbuminuria, asociada a elevacin discreta de la creatitina plasmtica acroalbuminuria, asociada a elevacin discreta de la creatinina plasmtica . tcroalbtaminuria 1 COI=l creatini~ia normal d) Macroalbuminuria, con creatininemta normal e) Prbieinuria-&igflifieetiv.a.,..con.g_~ormal

34) consulta por y sinusorragla. A la se observa una 'sln.exofjtica de 2x3 cm en el cuello uterino. Tiene uh PAP hace 3 meses que result normal. La~onducta ms adecuada es: Realizar nuevo PAP . ) Solicitat=-AisterosGepia ) Solicitar..eco.grafla transvagirlal "jJfExtlrpar la lesin y enviarla a biopsia e) DeFivar e c1 iote1 apia-

Una,pa~iente

~casa

e~peculoscopra

\. J X

36) Un paciente presenta de forma aguda tos y disnea intensafs, que luego se recuperan el gran medida. Al examen fsico se constata asimetrfa del murmullo pulmonar. El diagnstico ms probable es: a) E''ioOlraj:iUtmonar ~ Cuerpo extrao bronquial ~eumotrax espontneo d) T~Jwulsi"a e) BrQp~dtis aglda
37)'Un nio de ~~ cursando u Jarre evoluciona con P.!!fu!!Z progresiva, asociado a oliguri~. Berna generalizado. Al examen f(slco se eserita e-tctertctcn:nrmet y mucosas. El CliagnOsttco ms probable es: a) Slndrome hepatorrena b) H atitis aguda grave drome hemoltico urmico d) Glomerulonefritis postestreptoccica e) Anemii;l hemolltica autoinmune 38) consulta por odinofaqia, fiebre y varias adenopatfas cervicales. doldrosas. Al examen fislco se encuehtra 'febril, con faringe eritematosa, con escaso exudado 1 b Se palpa esplenomeg_aJia leve. El hemograma muestra linfocltosis con algunas atiplas. e realiza seroi?J~fas en busca ;;e_l ~gente c~usal, .con los siglJiientes reslfiidos;_ ~ lgM VCA: negativa; lgG ve os v -; ~v .o ~ '" 1 oxo v ; lgG toxoplasma: positiva lg CMV: positivalgG CMV: negativa n s probable es: feccin por VIH feccin por citomegalovirus e) lnfeccTOl'fpor toxoplasma d) lnfeccin.par-llifys Ebstein.Baar e) Infeccin etm1anta-porto)(eplasm4f...ldr..us Ebstein Baar
/

U.r{~aclente d~

~. A
39) EH~xamen de ms alto rendimientc para diagnosticar el asma es: a) P,fueba cutnea para alrgenos respirJtorios nmunoglobullna E especifica Velocidad mxima de flujo espiratorio (PEF) -E> !]adiografa de trax . ~spi~9fnetria basal y postbroncodilatador /
/

!l'

~ (!P'

J:

40) uia paciente de 43 aos con un ndulo mamario de 3 cm en la mama derecha. Se realiza una mafnografia que demuestra un ndulo denso, con todos sus bordes espiculados y con ~arias micro 'lcificaciones agrupadas. El diagnostico mas probable es: } Displasia maA=taria ...
}RwoadeA~

"'
~

"or philloides Cflf de mama ~ e) cM/ fo.patfa fiba:oqulstica

)T

(~1 tratamiento de eleccin de la depresin severa, durante el embarazo es:


lmipramina Venlafaxina . e) Amitriptilina d) Litio JYFiuoxetina .

~ (A

"'

~-\
(!'
~

~
~

Ji

~ydocolitiasis

42) Un P,\lCiente con d~lor abdominal d& .24 horas de evol ci , mufinten~oclado a lctericla;l Los exmenes de laboratorio demuestran hematocrito: 2o/o leucoc1tos:{t;Sl_ mm3, plaquetas: 340.0O x mm3, bilirrubina total' 5,9 rf'gldl, bilirrubina d recta: ~' GOT: 197 Ul/1, ~PJ~ 88 Ul/1, GGT: ~Q.O Ul/l,rosfatasas alcalinas: 326 u111. albmina: 4,1 y protrombTha: 936. La ecografa abdominal uestra hgado de ecogenicidad conse1vada, vesfcula biliar de paredes norm~ sin...presencia de_ lculos y la va biliar intra y extraheptica de calibre normal. En''hagnostlco mas probable es: ) -seteclsllbs agu'8a ~ ) Colangitis aguda -}- . , Qn ~V e) Pancr~atitis aguda \.~o(_j'J~ d) !;lepatitis aguda

(!'

A 3 ) Una paciente de cursa con d.@.rrea aguda, con deshidratacin severa. Se solicitan electrolitos plasmticos que demuestran ~ K:3,"9; Cl: 88. ta conducta 1r11em1 ms adecuada es ,.... ~dm'1strar: ~ -o olucin de NaCI al 0,9% l-f..-~ ~ ... f" ) Solucin de NaCI al 3% e) Solucin de NaGI al 3% GOn 2 gr-amos de...I!CI ~ . d) Soluci_!J.Qiucosaea al5%, con 2 gramos~~ ~laCI y 1 sremo de J<eJ ~ e) Sol~~in d NaCI al1,5%

60~,

-c..:.:r .:;-- --

P'
:
~

~
(A
~

J:xtracciones dentales, hipermenorrea yoguimosis frecuentes. El examen fisico no aporta ms nformacin. El diagnstico ms probable es: a) Hemefitiab) Prpura trombocitopnico inmune ~~isfuncin plaquetaria por uso de aspirina ~ ~nfermedad de von Willebrand e) ~ator V de eeyden .

4~)1Dna paciente de 17 aos presenta eqY!mosis cutneas desde la infancia, alveolorraqia en las

fiA
(!A
~ . ,
~

Una paciente control, consulta por una herida en el cuarto orteio /derecho, de 1 centmetro de dimetro ma~, dolorosa y de bordes irre ulares, eritematosos. Al -~ ~xamen fsico se aprecia disminucin. La radiogra 1a e p1e no reve a compromiso seo. El ~miento antibiOtico de elecclii es: iprofloxacino + clindamicina . etronidazol e) Cefazolina d) Meropenem + Vancomicina e) Azitromicina

46;

diabt~on ~~~

~
~ ~
~

./!"'.____

/~ ms probable encontra.!/ri este paciente? er J


. Alteracin del rojo pupilar ) Alteracin de la prueba de Hirschberg e) Alteracin de la oculomotilidad d) Alteracin de la tonometria digital e) Alteracin del reflejo pupilar

/46) Un nio de 9 meses presentt,~leucocoria en el ojo derecho. Qu alteracin en el examen fsico

0vYv-r

':~~

47) Un hombre de 44 aos insiste que su jefe quiere perjudicarlo y que le asigna tareas y actividades con el fin de hacerle dao, humillarlo y tratarlo mal. Se evala el ambiente laboral de la empresa en que trabaja y parece ser bueno. La familia del paciente niega los malos tratos de parte del jefe. El diagnstico ms probable es: !rastorno de personalidad paranolde (_ """"N rastorno psictico breve e) Esquizofrenia d) Trastorno adaptativo ~e) Trastorna:deJi[ante crnico

/ S

<.

_ /

48) Usted desea aplicar un plan de pesquisa de cncer de cuello uterino, con toma de Papanlcolaou en una poblacin de 10.000 mujeres, con cobertura: 75%, concentracin: 1. Una matrona tiene rendimiento de 5/h. Cuntas horas matronas se necesitan? a) 750 .? o
~~O

\J._ -"-~ --z.'{ r..'S .. ~


~

b 7.500 2.000 e) 1.000

\.0~~~
.....

,,v~o

o-~

. . ~,?

r.

~
~

~ ~

~
~

J:

50)/n hombre de 30 aos sufi un accidente de trnsito)lat! 2 meses, en el que result Ileso. Ahora .fpieza con recuerdo desagradables y recurrentes de lo sucE~dido. e siente muy angustiado y con un sentimiento disfrico, que le cuesta describir. El diagnstico ms probable es: a) Trastorno por estrs agudo b) Trastorno adaptativo ansioso e) Depresin ~a~trno obsesivo compulsivo ?''7 astorno por estrs postraumtico

~
~
~

~ ~
~

1 1) Un paciente de 68 aos presenta insomnio, tristeza, nimo ba o desconcentraci


recuentes. Su examen ffsico resulta no diagnstico ms probable a) Demencia de Alzheimer epresin ) potiroidismo d) Demencia frontotemporal e} Slndro!T'e confusional agudo

olvidos 1mental de Folstein de 5/30. El

~
~
~

~
~
~

52) Vl'i~ paciente de 38 aos, cursando con una pielonefritis por Esch~richia coli multisensible inicia trpfamiento antibitico, con adecuada cobertura, sin embargo 5 d(as despus permanece febril, con asuria y dolor lum r. La conducta ms adecuada es: ambaar e tratamiento antibitico por uno de mayor cobertura ~ b) Solicitar una tomografla axial computada e) ~n-tener el tratamiento antibitico y reevaluar en 2 di as _,...ar~olicitar ecografia e) Solicitar nuevo urocultivo y decidir segn antibiograma

~
~
~

'f

..,
~
~
~

...

~
53) Una mujer de 50 aos presenta aste!nia y prur.ito de 3 meses de eyolycin. El examen fislco no resylta normal. Las prueb~s he~ticas demu@.stran bilirrybjna total 2.3~bilirrub!na con~ugada: 2,0.. mg/dl, fosfatasas alcahna:t,!t_?.f Ul/1, GOT: 102 Ul/1, GPT: 112 Ul/1, GG 1/I . EI dragnstico ms tobable es: ) Hepatitis autoinmune ) Esteatohepatitis no alcohlica e) l-jiQ.ru:P.atatir:Gielismo pi h11a1 io ... d) Hepatitis viral crnica \--~)Cirrosis biliar primaria 54) Una paciente cursando diarrea y vmitos, de 2 das de evolucin, presenta d~terioro del egtado general y oliguria. En sus exmenes de labor~estaca nitrgeno ureico: 50 mg/dl, Crea6ninemu~.;....5,6 mg/dl, potasio plasmtico~q/1, sodio urinario: 40 mEq/1. EL diagnstico ms ""P!Y,bable es: ~ nsufrciencia renal aguda prerrenal .. b} Sfndrome hemoltico urmico e) Necrosi uda Obstruccin urinaria e) Insuficiencia renal crnica 55) Pac,.ente de~ presenta dificultad res iratoria, cianosis ta ul nea im ortante. Al examen ffsico ..Se constata I;R: 32x', PA: 100/60, FC: 1l\'~us gases arteriales demuestran : 7,3 Pa02: 57 mmt-ig, PaC02: 47mmHg;1Se inicia oxigeno a 35% evolucionando con sopor profundo. La conducta 's adecuada es: ... _ _ Disminuir la Fi02 de oxigeno Y _,.--.:r~ . .~ ~-r Iniciar antibiticos y corticoides endovenosos . L Aumentar la Fi02 de oxigeno .. d) Realizar apoyo ventilatorio no invasivo, Gon Bi-PAP ~ubar y conectar a ventilado[ mecnico 56) HoJllbre de ~consulta por un cuadro de una semana de evolucin de cefalea. fiebre, od.inofagia intensa y malestar general. En su examen trsico destaca palidez y esplenomegalia de 2 e~ 'bajo el reborde costal. Las amfgdalas se encuentran hipermicas, sin exudado. Se sohcrta un emog~aque demuestra hematocrito:leucocitos~ neutrfilos: 31%, linfocitos: 22%, s: lasto 41 , plaquetas: 18.000. El diagn.stico ms prob "!. Leuc mla aguda b) \.eucemia crnica ~ e) Infeccin por virus Ebstein Baar d) Mielofibrosis e) Linfoma 57) ~n mujer de 26 aos, P.!il]ligesta, cursando embarazo de 37 semanas, consulta ppr mareos y d~:cque aparecen al recostarse y que se han hecho muy frecuentes durante los ltimos dfas. La u ta ms adecuada es: ) Solicitar evaluacin por neurlogo b) Solicitar Holter de arritmias e) Solicitar perfil biofsico ~~terru_rripir el embarazo t:d~9r a la paciente sobre la normalidad de estos sntomas
/

~ ~ ~
~

Ji

~ ~ ~ ~
~ ~

~
~

~ ~
~
~

..

~
~ ~
..

~
~

~
~

~
~
~

~ ~
~

58) n hombre de 62 aos despierta por dolor precordial er o de 2 horas de evoluci Intenso, O mmHg, con RR2Tsin q e persiste hastarahl:rni". Al examen frsrco se presenta F : 110 ', PA opios y murmu__!!9 pulmonar simtrico, =:;.in otros ruidos. El electrocardiograma demuestra su~desnivel del segmento ST en las dE!rivacrones 011, 0111 y AVF. La primera toma de enzimas \..5:rnTacas resalt negativa. El diagnOstico mas probable es:. ~Infarto agudo al miocardio ._..,.-

~ ~
~

b) Angina inestable
e) Diseccin artica. d) Tromboembolismo pulmonar e) Fibrilacin auricular

~
~

~
~

""}

~
~

~
59) Un paqiente de 66 aqs presenta desde hace 4 meses disminucin del calibre del chorro miccional asociado ~ nicturia y poliaquiuria, con algun grado de d1sur1a de esfuerzo. Los sfntomas se han Id intenSH1cando progresivamente. El examen fsico 'ho aporta mayor informacin. El diagn co ms probable es: ~a) lnfe cin urinaria..< erpla'sia benigna de la prstata v ancer de prstata Estenosis uretral e) Vejiga neurognica 60) Un paciente presenta de forma brusca im,eosibilidad para evocar palabras. No es capaz de nomb.!'lr objetos, slo puede decir algunas palabras cortas, con mucha dificultad, sin embargo repite con m~r~acll1-.d. Presenta ~ola mlmma dltlqultad d~ la cornprens16n. El resto del examen fisico no apo ayor Informacin. El diagnstico mas probable es: 1 farto silviano derecho m. a JZ uierdo s Infarto postrolndico iz ueir . d) a e troncoe

~
~

~
~
~
~

~
~
~

~
~

e)~e--

61) Un paciente de 71 aos presenta Se solicitan TSH, niveles de plasmticos de vitamina 812 y folatos, ferrltina y ferremia todas normales. La conducta ms adecuada es: ~ a) lnietar l:lieFFe, felates y vitamiRa 812 po1 via o1al b) Iniciar vitamina 812 intramuscYiaF~niciar estudio de talasemia V' d) Solicitar estudio morfolgico de mdula sea --e) Jr@::ia1_ 11 ritraJ:Ieyetiaa
.

~~g-rpgea~i~

---

~
~

~
~
~

/1'

62)Un paciente de 60 aos consulta por una lesin en el-ala nasa, solevantada, con una ulceractQn_ c{entral. con borde perlado y una telangiectasia adyacente. El diagnstico ms probable es: a) Queratosis actrnica b) Granuloma telangiectsico e) Melanoma d) gprci~~mia espinocelular e-Y-Carcinoma basocelular
/

~
~

~
~
~
~

~ ~~~ma

63(Una paciente de 3~s consulta por tos recurrente, de algunas semanas de evolucin, Efspecialmente n~ y que en elultimo tiempo aparece adems con la actividad fsica se / ~~paa de J!!!n!a. Tiene antecedente de rini~~s alrgica. Ef 1agnos co m s probable es: b) Coqueluche e) Tuberculosis d) Enfermedad pulmonar obstructiva crnica e) Bronquitis crnica ral

~ ~
~

~
~
~

(~ ;=:::::=mhamzo
J~Tasa de incidencia

64) na mujer, cursando embarazo de 33 semanas, consulta por cefalea, tinitus malestar i portante. Al e~amen fsico presenfi"FC: 108x', PA:~m g, e ema de extremidades nferiores y e~altacin de los refleJos osteoiedneos~nstico ms probable es: a) Preclamsi~ moderada ~reclamsa severa ~ i(:: ..

~
~ ~

sylcul es el mejor parmetro para evaluar la Importancia de una enfennedad crnica?


~evalencia
e) Tasa de mortalidad general d) Tasa de mortalidad especifica e) Tasa de morbilidad

~
~
~

~
~

..,

~
66) Una paciente con antecedente de a1tritis reumatoide en tratamiento con prednisona 15 mg/da }l. metotrexata en una dosis semanal. 'COnsulta por dolor en la ingle derecha. muy jntensorde-1-das-de-. evolucin, que le impldeJama.mha. Al ElXamen ffsico se--.Qrecia dolor con los movimientos de la cadera, especialmente con la abduccin. s siiCita racliografja ~e:aderas ~ue muestran..J alteraCiones de la cabeza femoral derecha, compatible con necrtrs1s avascuab Cul es la causa ms probable de la necrosis avascular ~n esta paciente? .. a) la artritis reumatoide .1 b) la prednjsona ...JfEI metotrexato d) Traumtica e) ldioptica
1

~ ~

~ (B'
~

~ ~
~
~
... /

67} Un paciente portador de ua" enfermedad obstructiva crnlc , ingresa durante una exacerbacin~~ taqui,pReiGo..y....can gran disnea. Se sohcs an gases a er1a es que muestran pf:I~,;)P02: 55 mmHg,. PC02: 49mmHg. Se inicia oxigeno alf.2a~, evolu<!_ioQan~~ c~.!l..~!.~erioro de la funcin respiratori~. Al exarten ~e ye sudoroso. vigil, con c1anc,sis y se controlan nuevos gases con pH: 7,26, Pa02: 46 mmHg, PaC02: 67 mmHg. La conducta ms adecuada es: ) a) Suspender el oxigeno 'V L (0'> ~ b) Administrar hidroc9rtisona endovenosa ~ '\ e) Realizar nebulizaciones con salbutamol, cada 20 minutos, por 3 veces \ \ d) Realiz f apo o ventilator" 'nvaii"O n ~ r y conectar a ventilador mecnico

~ ("\
~
~

~::q:::::s son ms e~~':_s~ra el trat_!llle~t~ ~~-~s ~~~~~! e-~-~~ u-~~-a.~a ~bS.~c~~~-~


./

~ Betabloqueantes
69) U

~
~

e) Anticolinrgicos d) ntiandrognicos e) Diuztico

~
~

~
~

~~eva temblor fino de extr.emidades y_ taquicardia. ''---' a) Anorexia neosap<" b) Depresin


e) l:umnr bjpnfisjarjo

~;fitlada a AQI:IIiqsi&ma, m.alrelldlmienl:<> escolar y dificultad para dorr(llr. Al examen fislco se----

.---., a nia d~e!'consulta por un cuadro de pJ~l~peso de algunas semanas de evolucin.

d) ~mocitoma
~"Hipertiroldismo '

~
~ ~

~
~

70) Hombre de 24 aos, con historia de un mes de malestar abdominal feb a.ocasieAal-: asociado a diarrea frecuente, que en el.ltim~JJ~fi:!~O se a ec o sanguinolenta. Al examen fsico presenta palid~dad abdo inaJ y aument de fos ruidos hidroareos. En sus exmenes '-..-./. ,. destaca he. atocrito:., 2~ Hb: _ ,1 , blantos::SI3.1lHlliEI diagnstico ms probable es: /' a) CoHtis-par-as-itaria ~ :7

ee'

,_
f!'lt'.

b) CeHtis-vfl:af . e) Co~itis hacteiana_ . olitis .ulero n}e edad de Crihn


del rendimien asistencial de mdicos que trabajan en un programa de ) alud Q!!..!len_c~ria-eS:~~a relaclOefre el total de horas asistenciales y el total de horas trabajadas . ~~1 nmero de pacientes atendido~ por hora de trabajO-.../ e) La relacin entre los das trabajados y los das faborales l!s( d) El total de pacientes atendidos en el ao e) La relacin entre el total de horas trabajadas y el total de horas disponibles t(

'Z:t(el

~ indic~

l~s

r-

,...
fG\
(C\

f*"
~

(6.
(lit\

('t'
~

72) U paciente de 67 aos

present~ e 3 dfas de duracin, autollmltada. No present otro sntomas y el examen fsico resulta normal. Cual es el examen de rminar la ~tioi'09iifdel sangri@P { a omografia axila computada ) Hemograma y pruebas de coagulacin e) Enema baritado
e) Test de sangre oculta en deposiciones
/./

~olonoscopla

73) Un;r'inujer se realiza un PAP que es informado como satis;factorl~, ~In presencia de clulas .
~sicas ni atipla&, con cambios inflamatorios inesP'ecfflc:os. La conducta ms adecuada es: licitar nuevo PAPen un ao licitar nuevo PAP en 6 meses e) Tratar la inflamacin y repetir el PAP d) Realizar cono fro ) r,~ e) Derivar a patologla cervical '--=-- ,..\:'<- 1

ficio cervical externo,_con t~.! ~e cdsta~cin posit~. El tacto vaginal muestra un ~ue1.,9.. 4.~nno lado, dut~n posicin posterior. El ~er gemercrest en presentacin podJica y el segundo en lica, y el pertal baofasaco de amt?9.~. ~eto.s. r~~ilr.anquillzad.or. La conducta ms adecuada es: a) Inducir el part~sprost.QI b) Administr11""'6sr:ticoide~ y anfibj6tjcos e interrumpir el parto en 2~ ~alizar cesrea d) Rea1ttar atlllliocemesis e) Ad inistrat antibiticos corticoidss-y..toc:GUticss y maRteRei=-GeRdueta expeetaRte Un nio dei-aos presenta <c!iludicacln de la marcha y fiebre has~ Al examen fsico no U@Ye la extremidad inferior derecha y presenta dolor a la m_ovilizacln de la cadera d~r.e~ha. El resto del examen fsico no aporta mayor informacin. El diagnstico 'is.probabf-es: a) Enfermedad de Perthes b) Sinovitis transitoria ~rtritis sptica d) Epifisiolisis de la cabeza femoral e) Osteomielitis

74) yna mujer de 33 aos, cursando embarazo bicorial biamnitic presenta rlla de lquido claro porgenital~s. A la espe. culoscop~a ap~~~i.a .escaso lquido saliendo por_ el.

g~lar

'e

d~.emanas,

4ciente que trabaja en un juzgado fue tomado como rehn durante una fuga_ de reos. Co sulta 2 das despus por sentirse muy angustiado, con re-cuerdos muy desagradables de lo ontecido. Refiere que no ha podido llorar y que no quiere volver a su trabajo. El diagnstico ms probable es: a) Trastorno adaptativo ansioso btJrastorno por estrs postraumtico ~rastorno por estrs agudo , d) Depresin e) Trastorno dispciativo
76)
.. "'

77) Un ,w:,ti~nte de 74 aos presenta~~stenla, dolor cervical y dorsal alto, con dificultad para lncg~ me. El examen flslco no apOrta mayor inform~) En sus exmenes se constata

h
~

tocrito -~~~ancos y plaquetas normales, Vljs:- !nm711; CK normales, factor reumatrdeo (NCA (-), f'N~sitivo en dllucaon 1/40. El diagnstic m s probab~
.

un

b) ~~~!!i~eitis 5js de Wegener d) Polimiositis polimialgia reumtica


e)

r rz., .-e .
1

~
78) 'n nio de 1...1 ao,s presenta rinorrea u enr jecimiento periocular derech~, co~.~~~~tE~Ii~lili~~aR~~ pr bable es: Celulitis preseptal ~ Celulitis orbitaria e) Pansinusitis d) Mucormicosis e) Dacriocistitis aguda 79) Un paciente de~ chnsulta por dolor abdominal. asociado a distensin abdominal .. pro..9feSiva, sin eliminacin de deposiciones en las ltimas horas. Al examen fsico presentaun abdo.fu;n distendido, doloroso difuso, con aumento de los ruf!~ hidroareos. Se solicita radiografa de bdomen simple que muestra dilatac~a~sas in!e~_~i!!a!!~ E?n!llV!!~ sin sencia de gas en el colon. El diagnstico mas probable es: olitis pseudomembranosa . b) Peoracin de lcera pptica e) Cncer de colon ;W'Ubstruccin intestinal re) Colitis ulc~rosa

~ (!"

~~eali1ar extraGciA-eeA-fereepS
d) Realizar..ammohwsiJl

80) Una paciente embarazada a trmino ngresa en trabajo de part~. Al examen presenta dinmica. uterina ct@.-3 CU en 1O minutos.;''clilatacin completa, feto en posicin OllA, ~ cm por sobre las espinas iliacas, coh m~branas rotas y lquido amniotico claro. El monltoreo fetal muestra desaceleraciones vat:ismles. con variabilidad normal. ca co ttducta mas aaectiada-es-: -~ - .. - - - -...__ __ ....) @>~:~rz~~~~~~~~:r el parto momtoriiado .. - - --\ -~ ~~~----- ~ .
__.,.-

~
~
~
(!!A

e) Reat~r toclisis de urgencia

~c..~.<--- L cY-orJ?

.. ~

~
~

J
.. .

~
~ (0\
~

81)tJn recin nacido presenta ci4lnas.i_gue no responde a oxigeno al100%. Al examen fsico se 6serva cianosis generalizada, con examen pulmonar normal y no se ausculta soplos. El diagnstico ms probable es: _ ~' ~ )CaJnuoic.acin..interaurlcular ~ jc:Y- ~~ ~ ~ b) Cotm:&Aieaciu inteventricolarc) CeaFtaeiiHlftiead) Ouctvs arterioso persistente ~Tr~risposicin de grandes va.sos

-r

..../

2) Cul es el mejor examen para seguir la evolucin del derrame pericrdico, durante una

~ocar~jografla

pericarditis aguda? . b) Examen clnico e) Eleqt{ocardiograma d) T~e de trax adiografia de trax

'

""

~ (!A

~
~

f!!A

f!'
~
(4\

Un nio d~ consulta por un cuadra...de dolor abdomiRal, artralgias de rodillas, aparicin de rash en los m~sl~: y P.iernas y hematuria en una oportunidad. Al examen fisico se aprecia el rash cqn lesiones pal ab es, que no blanquean con la compresin. En sus exmenes destaca hematuri~ con 15% de dismorf!.a. El diagnstico m!; probable es: a) Leucemia b) Meningococcemia ~rpura de Schoenlein Henoch d) ~klmen1loReffitis postinfeecjo'ga e) SlOOt=etne liemolitico 01 mico

~
~
~

~
~

~
84) Una paciente con diagnstico de diabetes mellitus 2 f1~ce 3 mes.es, en tratamiento con dieta bien llevada, persiste con glicemlas eleva;tas. La fiemogloblna glic:osilad ~esulta en 8,3%. Cul es el . trataiJ)lenta-que-Et be indicarse? : Metfonmna X /~)..Metformina 1Glibenclamida
/<._~) ~benclamid
~

,..,
~
~

d) Insulina e) Mejorar el cumplimiento diettico y controlar en 3meses


f

~
~

85) Una paciente dia=sa ~o 2, con control irregular, presenta dolor perianal y fiebre. Al examen ,2 , con presin arterial y pulso nonnale15. Se aprecia una zona aumentada fsico est febril has de volume11, eritematosa en la regin perlapal y el tacto rectal resulta myy doloroso, palpndose una zona abombada dolor Se solicitan exmenes donde destaca un hemograma con hematocrito: 0 , leucocitos~plaquetas: 189.000. La conducta ms adecuada e : ) l iciar antibioterapia ~ .. ealizar drenaje inmediato e) Derivar para resolucin quirrgica_ A \e~? o) Realizar baos de asiento e) Solicitar colonoscopia ;

~ ~

~
~

~
~
~

,Evoluciona con d~potencaa funcional del codo, asociado a leve aumento de volumen. El / diagnstico ms probable es: / a) Fractura supracondflea ..~ b) Luxacin de codo e) J;.ractura de olcranon .,.,.afFractura de cpula radial e) Fractura de epitrclea

8~) ~a mujer de bs~ufre cafda apoyndose en la mano derecha, con ~1 codo en extensin.

87) Un nl.o de 2 aos presenta un exantema popular generalizado, con compromiso de palmas y planta~; 'Refiere antecedente de cuaafo feDrll de 5 tilas de evo~Presenta InyeCCIn conJuntaval .biit ral y la boca est eritematoso, con lengua en fresa. El diagnstico ms probable es: a) antema sbito Mononucleosis infecciosa e) Infeccin por citomegalovirus

,?1 ~nfermedad de Kawasaki


. 88) La auscultacin de un tercer ruido (R3l en un p~ciente cursando con un infarto agudo al miocardio, es sugerente de.: a) Rotura del msculo papilar-o b) Infarto ~~~:~~=ioo~ e) tnfrt e de1 echcr d) Insuficiencia car{liaca -e) Blequ~o MI =-

~~scarlatina

lk

-{~uJ de los siguientes agentes se ha demostrado capaz d~ir la mortalidad y la aparici~


un nuevo infarto, cuando se administra a pacientes que han sufrido un mfarto de miocardio f=

lif8dipino.
~~tab!oqeantes. /
e .
b)\1erapam1IO. c)Nitroglicerin.

~~)Fasciculaciones. b).~trofia muscular

90 Cul de los siguientes hallazgos clinicos NO apoyarla el diagnstico de polineuropatfa en un p ciente con debilidad progresiva en miembros inferiores?:

~igno de Babinsk1. d)Hipo o arreflexia. e)Hipotonta

\o 0

lJ'v-=r-
\.

\ ~-

f""" ~ ~

_ .~ ~ L-') ~v-- ...~ ,..,. _ ~i--[<--<J

'=>

. ~

ov----f-9-u.)l-~
e(
:S'~--~.

~
.-

n&CJ
'-\
'{
EUNACOM 2009

~r~~
~mpo~ara

!'

PRIMt:RA PARTE

('

r'
!'

J,
f
e)

es la mes:lida prevenir la insuficiencia renal aguda secundaria al uso de dio de contraste. durante una omografa? --N-acetyl cistena Suero fisiolgico e) Bicarbonato d) Diltiazem e) Acido ascrbico

~ul

---~':}

~t N\.
(\1\

\. d \J

l'\.J

, sCp

X:

\,~ ~

t/J..\'s(o
(
~\l1

~x

2) Una paciente de consulta porque en las ltimas 2 semanas ha presentado pujo y tenesmo rectal, asociado a dep se ones diarreicas, con moco y sangre. El diagnstico ms probable es: Ce.tis pseudol 11e1 11btanosa _ ~ _ .L.....l1=:. .~ b olitis utce ----- O L ~' ... o 1t1s por Shiguella.-------d)~~~

/ =

c9n .
/

. .....

~
~

~
"

~ ~
~

mujer consulta por leucorre~e mal oiQf, que en la especuloscopa se aorecia de color y cuyo PA es5}JiJ agente etiolgico rlls probable es: Gandida albicans ' ) Escherichia coli ~ardnerella vaginalis d) Trichomona vaginalis e) Clamydia trachomatis
r~o, espumosa

'Una

4) Un nio

~
~

J
/"-

~lo. No ~~~~ habla. La conduela ms adecuada es:

d~!f'hresenta de manera brusca un episodio de agitacin, llevndose las manos al

~
~

a) Realizar compresiones cardiacas XDar golpes interescapulares -e e) Realizar maniobra de Heimlich d)Tomarlo de Jos pies y sacodlrro e) Dar insuflaciones boca a boca
5) Un .reCin nacido~no, nacido_J?or cesrea inicia taquipnea y quejido, luego de 20 minutos
'

~ ~
~

~
~

de pcer. Al exal' fsico se ausculta disminucin del murmullo pulmonar en las bases, sin otros iCfos y la. auscultacin cardfaca es norinal. Se solicita radiografra de trax, que resulta normal. El agnstico ms probable es: ) Hipertensin pulmonar persitente b} Enfermedad de membrana hialina e) Aspiracin meconial ~aquiP':l.ea transitoria e) Neurt;~onia neonatal 6)
p~sentado

~
~
~

~
/

r)g}-~a'K
d) ~dnpatia x

tres episodios de epistaxis severas por la misma fosa nasal en el ltimo mes. El ms probable es. ) Desviacin septal / ~giofibroma nasal ..
agnstico

~~~~;~nte de .k:::: con obstruccin nasal demcba permanente desde hace algunos meses. Ha

~ ~
~ fl:\
~

e) S,J.ousitis srRica

(\

~
~
(S'

7) Qu parmetro permite determinar con mayor precisin la severidad de una enfermedad obstructiva crnica? ) La capacidad vital forzada (CVF) ~) El volumen de espiracin forzada en el pmer minuto (VEF1) \) ~La relacin entre el VEF1 y CVF Id) El flujo de espiracin forzada entre el25% y el 75% de la CVF (FEF 25-75) e) la capacidad de reserva inspiratoria

81\

~
(lA

.
.

}Jv.lb f\lorQft\

,.,
.,.--'<_.~-4-u

\f>r'~r'u~
~r

~ . . ')

8) Un pac1ente de 6.4.!f!gs consulta por astenia. Ha presentado 4 infecciones bafer anas en lpmos 3 mees. Al examen~se aprecia ,plido, sin otras a teraciones. Tiene hematocrito 9% plaquetas: 160.000, blanco~ creatinina d~ y proteinuria de 4 gramos/24 horas.} El estu 10 radiolgico demuestra osteoporosis generalizada. El diagnstico ms probable es: "' a) Mieloma mltiple J)~ ~~ b) Mielofibrosis ~-r lperparatiroJ ismo primario ~ ~ Glomerulopata membranosa ~ e) Anemia de enfermedades crnicas

~~~-<--~1

'"~ \

,_,
,-,
~

L,c;C..~~

~
~
~

~
~ ~
~

. A
p

9) Un ~io de 4 aos presenta un Guadro febril de dlas de evo.lu.G.i9n, asociado a odinofagh

y_

aparipin de un exantema eritematoso generalizado. Al examen fisico se observa ojo rojo bilateral, labJs con costras, eritematoso y se palpan 2 adenopatias cervicales, la mayor de 3 cm. El gnstico ms probable es: nfeccin por adenovirus Infeccin por virus Ebstein Baar e) Escarlatina )iiYEnfermedad de Kawasaki e) Sarampin

~
~

1 O) Una paciente ~;os ica presenta elevacin d.!Jas transaminasas en exmenes de cheq_ueo, con GO . O y G. : 92 Ull . La bilirrublna, GGT y fosfatasas alcalinas son normales. Presenta ferritina de 360 ng/inl (VN: -150 1 Al examen fsico destaca hepatomegalia de 3 cm bajo el reborde costal y o moderada a sevet La ecografa abdominal demuestra aumento dela ecogenicidad heptica. El diagnstico mas pro able es:
c?~macromafosis "'""t>~)depatitis viral

9) steatohepatitis no alcohlica

l) HePatitis autoiTmlne-e) Cirrosi~ heptica

11) La.incidencia de EPOC en los paciente que fuman es 40 por 100.000 y en los pacientes que no fumn, es 2 por 1 00.000. Calcule el riesgo relativo: a} 0,05 \ ~) O l ,_. /,_b)2 o 4-g. 0.0~ O \ . '"" l -t- t{O
o ""'

~8 e) 40

---,;~ .....-;_.

--(a:_/-

f -

... \ ....

12) Una paciente de 33 aos consulta por !JIU'l{lo~isiJ!p y d:~l. Al examen se palpa bocio difuso, doloroso a la a resenta TSH: 0_1 y 14: ~'! __ cg/dl. La conducta ms adecuada para el'tra am1ento de esta patologa es: ~ ~~ a) Iniciar levotiroxina (levo-T4) .-K.'<""' ~ p b) Iniciar antitiroideos ~---~(;9." e) Administrar yodo radiactivo ~ d) In. iciar triyodotironina (T3) ..~.di~f. AINs ~
o

lO::>.~
~

~
~

;J:. '

13) ,Yfa paciente de 34 aos, presenta por primera vez presenta un cuadro de rpida instalacin de {lg!_or precordial, disnea, palpitaciones, sydoracjn, dificultad para tragar y parestesias en las xtrem&daaes superiores. El cuadro cede espontneamente luego de 20 minutos. Los exmenes de aboratono exclUyen patologa orgnig. El diagnstico ms probable es: a) Trastrno de pnico b) Hipocondra c},Jrastomo adaptativo ansioso ,ti) Crisis de pnico e) Trastorno de estrs agudo

'

~
14) Un recin nacido d~3 dias de vida, eon antecedente de asfixia severa durante el arto, presenta vmitos, distensin abd m 1 elhihacin de sangre e 1 RQS.!Q!~tnes. Al examen fisico est irr a e, con distensin abdominal y timpanicidad a a percusin abdominal. El diagnstico ms pr96able es:

"

Malmtacin-intest~l V~~ Atresia duodenal " ~nterocolitis necrotizante

1!

/d) Es~rHca del piloto


e) 1 nvt:Jginaci6n-irttestifl8115) ,n nio ingiere una pila de reloj, evolucionando inicialmente con tos y sialorrea, que luego ceden espontneamente. Se so1icita radiograffa que muestra la pila en el tercio inferior del esfago. La onducta ms adecuada es: . ) Observar evolucin, sin necesidad de estudi_s> ..a-nienos que presente sintomas ) Buscar la pila en las deposiciones hast9 qu aparezca e) Administrar liquidas por vi a oral y _confrorar con radiografa en 1 hora d) Realizar control con radiografian 6 horas ~xtraccin inmediata

~ ~ ~

A
A
X

~ ~
~
~

~ ~
~

16) Ul) paciente con diabetes. tipo 2 de la~ata, tratada e~ que cumple de forma irregular... ~r~nta hemoglobinas glicosiladas de 1 , o y creatinina d~g/dl. La conducta ms adecuada es rctar: Clorpropamida b) Tolbutamida ...L {_.A .. e) Glibenclamida ms metformina '< ~~~ S L.u~-.... d) Metformina x pnsulina

,.

(lA
~
~

#
/) / b) -./ e) d)
.

17) Qu alteracin del liquido cefalorraquideo aparece ms tempranamente y es una meningitis bacteriana aguda+ c. a) Proteinas aumentadas b) Clulas mayores 50G-per-mm~ X' re. dom.inlo de p'lilll.Qrfon_!:l_g_'qre.s-ismi~ucin de la glucosa urbidad ,,

IJ)~

s.

~
~-

Debilidad de instalacin sbita de un hemicuerpo < . Debilidad proximal, asociada a elevacin de la creatinfosfoquinasa (CK) Debilid~d distal asimtrica que mejora con el ejercicio Debili.dad proximal y distal simtricas, sin compromiso ocular }kAs~Iia y debilidad muscular de curso fluctuante

1~rlo ms caracterstico de la miastenia gravls es:

~
~

f"A

f!A
~
~

1gyLa aleatorizacin, en un ensayo clnico sirve para: Aumentar la precisin del estudio Mejorar la potencia del estudio e) Evitar sesgos de informacin ~vitar s~~gos por variable contundente e) Con~-~gl.Jir que ambos grupos sean del mismo tamao

~
~
~

20) J,Ji paciente de 6~ cursando neumona,~ratamiento con.s;:eftrajxona, presenta un derrame pletJral con protena~ ~ di, LDH: 558 Ul/1, pH:~_.&Jglicemia: 40 mg/dl. La tincin de Gram no sualiza bacterias. La conducta ms adec:uada es: ~ ~'"' .. / a?'1nstalar toracostoma con trampa de agua ~ \ .... ~ /b) Cambiar la ceftriaxona por clindamicina endovenosa e) Iniciar ceftriaxona endovenosa, asociada a metronidazol y controlar con radiografla en 48 horas d) Realizar toracocentesis evacuadora e) Administrar antibiticos intrapleurales

t!"
~
~

("'

(!"'
~

22) Un nio d 7 aru-~f'H'E~flta edema generalizado v palidez. Presenta albuminemia de 210 g/dl, ~oteinuria d y micrhematuria. El diagnstico ms probable es: .:>Sndrome nell'f{.IIAA----.,fif Sndrome nefrtico e) Sndrome hemoltico urmico d) Enfermedad de Berger e) Sndrome hepatorrenal 23) Una mujer de 26 aos dice tener una relacin amorosa con un famoso cantante y que ste le declar su amor a travs de una cancin. Ha intentado contactarlo en varias oportunidades a travs de cartas e incluso visitndolo en su casa, sin !3mbargo nunca ha logrado encontrarse con l, ya que los guardias se lo impiden. El diagnstico ms probable es: a} Esquizofrenia ?b) Trastorno delirante crnico e) Trastorno ps1ctico reactivo d) Trastorno adaptativo ,e} Trastorno esquizoafectivo 24) Un .niio de 11 aos con mal rendimiento escolar y peleas frecuentes con sus compaeros de clases. Tiene antecedentes de haber maltratado algunos anima1es. No parece sentir culpa y s~onsabiliza a otros por sus actos. El diagnstico ms probable es: Trastorno oposicionista desafiante . P)'Trastorno de conducta disocial e) Tr~terno peF efieit ateReionat d) Depresin infantil e} Esquizofrenia

.,...-..

25) Mujer primigesta, cursando embarazo acude a control. obsttrico, dinmica uteri.n. El tacto vaginal demuestra un cuello cerrado, duro, posterior, sin borramiento. La conducta ms. adecuada es: ~)Realizar cesrea .._/ b} Observar evolucin espontnea e) Realizar amnicentesis d) Realizar test de tolerancia a las contracciones ~~ducir el parto con misoprostol
26) ~na paciente de 33 aos, usuaria de DIU, con antecedente de una gastrectomta parcial, consulta p~rr'astenia y debilidad. Al examen ffsic~staca ~Je.lidez impo1tante, sill otras alteraciones. Se .sblicita hemograma que muestra Hcto@. Hb: 8, VCM: 66, H(:M: 26, plaquetas: 255.000, Blancos: //7.500. El diagnstico ms probable es: ~ .._/ ~Anemia ferropnica p~ ~ . ~)Anemia por enfermedades crnicas e) Anemia por dficit de vitamina B12 d} Talasmia e) AP,.Iasia medular
~)

de~

si~

2j:/Qu antidepresivo est contraindicado en la Moclobem1 mitriptilina .


e)~

c-....__/b)~~ e Cit.olnnr,o,n-

'
40 aQs presenta rigidez matinal de una hora y artralgias de a~iculacione~ angicas, muecas y tobillos, desde hace 6 semanas. Al examen fis1co se aprecta aumento de volumen y signos inflamaft~rios en las articulaciones descritas. El diagnstico ms blees: ritis reumatoide ritis por cristales e) Artrosis d) Lupus eritematoso e) Artritis viral .
meta~

~
28) U

hombr~

'~'

~ ~

un tos. odjnofagia y rjnorrea de 10 das de evolucin, a Jo que se ha agregado en las ltimas horaH fiebre hasta 38.5C y disnea. El examen ftsico demuestra crepitaciones difusas, mayore a bal~e izquierda. La radiografa de trax demuestra un infiltrado intersti quial, yor en la base izquierda. El tratamiento de eleccin es: a) Amoxicilina Eritromicina c. r\--P ~'O\. <--t evofloxacino ~ "'----7 \J r~LCeftriaxona ~ Clindamicina

29) un nio de

~resenta cuadro~

fl!;

~ ~

30) Una paciente de 30 m transvaginal que mu~~dometri ae~:J.::;::::::::!m~,;..s.l:i-=n:.;.s:.:.a=c:.;.o:..:g::.e=-s...::ta::,s...c~ion;;.;ai71::::y--una qustia de 3 cm en el ovario derecho. La gona atropina corinica es de ~~~--r...;;;;..;~.;.;;.;.~~~ adeuaaa es: a(Administrar metotrexato ) Solicitar nueva ecografa transvaginal en 2 das ~ ~olicitar nueva gonadotropjna corinica E!O 2 das/ d) Realizar laparoscopa e) Realizar biopsia endometrial
31) UIJ usa ibuprofeno como tratamiento de cefalea crnica, inicia dolor e i strico urente, recurrente. intenso, gue en al un ocasiones lo des ierta orla noche. El e amen 1s1co no aporta mayor informacin. La conducta ms a ecuada es: /"'') Agregar sucra!fato al trataR:Jit~Rto cos:1 ibYJ.tt'Ofem> ~ .._.,. ~iciar trtamiento sintomtico y solicitar E~studio endoscpico ...e) Iniciar omeprazol y reevaluar en 2 semanas d) Prohibir el uso de Al N Es e iniciar otro tipo de analgsicos e) Erf-adicar--l:l.elico.baeterpyron

~. co~

~
~
~
~

p~ciente de~e

~
~

~
~

~
~

~
~

'...

e) Agregar un proquintico d) Iniciar lactulosa ~Solicitar ecografa abdominal 33) Un paciente se molesta fcilmente con las crticas, reaccionando con ira y agresividad. Es muy celoso y se involucra fcHmente es discusiones. El trastorno de personalidad que padece este paciente es: t! a) Narcisista b) Antisocial e) Evitativo d) Histrinico :2,_Paranoide

@ ...o.tic.i1ar.~~!?.!:!.r:>.~~2~.

32) Paciente con antecedente' de sfndrome de intestino irritable de 10 aos de evolucin, consulta por cambio de su hbito intestinal de algunas semanas de evolucin, con tendencia a la constipacin y que no ha cedido, a pesar de haber agregado fibra en la dieta. El examen ffsico resulta normal. La conducta ms adecuada es: a) SolicitarTAC de abdomen y pelvis

de~

~
~
\ i

/ {

(!l'
(9'
4'1
'

~
~

~
~
~ ~

~
~
34) Un paciente de 77 aos presenta dolor abdominal intenso, fiebre e ictericid, asociados a hieotensin seyer~tos .exmenes de sangre demuestran elevacin marcada de los glbulos bl n~os y otros armetros inflamatorios la ecograffa abdominal muestra colehbasas con aalatacin de la '! a iliar intraheptica y del coldoco. El tratamiento de eleccan es: ~ a O scomprtmir la va biliar por va percutnea .. _ escompresin biliar endoscpica e) Drenaje quirgico de la via biliar y colecistectomla laparoscpica diferida C'\ IJ d) Colecistectomla con coldocostomla precoz ~~ ' e) Antibioterapia endovenosa

,.,
~
~

'R .

35) l:Jna paciente cursando embarazo de 7,semanas presenta dt>lor abdominal la ecografa transvaginal no visualiza gestacin antrauterana y la subunidad beta de gonadotrofina rinica humana resulta 3400 Ul/1. El diagnstico ms probable es Aborto completo. Aborto incompleto .AR"Embarazo ectpico ~d} Oprendjmjenta de 3laee11ld e) Emhara&g ffiolar

y~rragia ese~

~
~
~

,._,
~

~
~

~
~

ea 1zar rotura artificial de membranas e) AumQntar al eioele la velocidad del..g.Q!eo de oxitocina 37) Ur:t nio de 4 aos con historia de fati abilidad resenta crpitos escasos en la auscu pulnmar. En la auscultacin cardiaca se oye un soplo sistlico eyec avo, mas intenso en la zona J?lfae,.sternal izquierda, asociado a desdoblarrile~to fijo del segundo ruido. El diagnstico ms probable es: a) Ductus arterioso persistente P15) Comunicacin interaurtcular e) Comunicacin interventricular <.... 1 ~
d) OesRaein aFlica e) T.~n6o arterioso

~ ~ ~

~ ~
~ ~

3~} 'Una mujer de 33 aos consulta por al ttaciones 1 ole ancla al calor. debilidad diarrea @ )tolerancia al calor. Al examen presen boci~e tamao moderado, con frmito, exoftalmo / bilateral, edema pretibial, con piel fina y hmeda. El diagnstico ms probable es: __./ a) Enfermedad de Hashimoto b) Bocj multinodular txico e) Tir.oiditis de Riedel dYftnfermedad de Basedow Graves ~}-'nroiditis subaguda

~
~
~

~
~
~
~

~9) Una mujer de 23 aos se ha alejado de su familia y otros conocidos, ha dejado de asistir a su

trabajo y ha descuidado su aspecto personal. Adems .escucha voces que le obligan a realizar actos. que ella no desea hacer. Su padre refiere que siempre fue retraida y si bien logr terminar su carrera, de economfa, nunca ha logrado mantener un trabajo estable. El diagnstico ms probable es: a) Depresin psictica storno delirante crnico quizofrenia d) Ps1cosis reactiva e) Trastorno esquizoafectivo

~
~
~

~
~
~
~

"""\
~

. --. -- --- -- ----, 40) Una paciente de 53 aos presenta unbj_opsia endometrial informada como hiperplasia

endometrial atpica de alto grado y com~Ja. El tratamiento de elecc1on es: a)fib~ervacin b Tamoxifeno sterectomla ) orm<DtlG\era~ia eeA estr6ge1 os slos e) Hormonoterapia con estrgenos y prog.estgenos 23 present desde hace 6 dias ojo rojo derecho, con S!,Crecin serosa y sensacin de areriIIa. Hace 1as os sintomas se extienden al ojo izquierdo. El diagnstico mas probable es: ~Ce'hjuntivitis viral 'b),Conjuntivitis bacteriana ~ Conjuntivitis alrgica d) Uveitis e) Epiescleritis
41) Una .mujer d

y con antecedente de cncer de mama tratado con tamoxifeno, pres~ta ~~rl!!lla de'3 dlas de evolucin. La ecografa transvaglnal demuestra u.t-e'19_ometrio de 12 mm, e ecogenicidad heterogenea, sin presencia de miomas. El ,d_i~gn"stico ms. P.r9.bable es. ((~,Cncer de endometrio)
42) Una paciente de-Os, con 15 aos de amenorrea
1)}'At~triat---

~denomiosis

1'"c)'E-Rdemetritise)-EndametilOsis
43) Un paciente de

S0,
r a 80 latidos or minutos, con QRS

n osto. Se .
~sina

encuer~y;;.~todinQJicame.nte establ~. El tratamien o farmacolgico de eleccin es:


.

Coa- presenta umi~~e

effipamHo e) Amiodarona d) Digqxina "'e) Propanolol


44) Una

p51c~nte de 22 aos presenta artritis probable'es: -) Art tfs reumatofdea


~d-de-Wegener-

fe1TlaiiOS'Y sndrome nefrtic9? El diagnstico ms


.

rmedaG-EJe-Qood-easflse us eritematoso sistmico e)'Enfermedad de Berger


45) t}r'la paciente de 24 aos, cursando un episodio deP,resivo moderado, inicia tratamiento con fluxetina 40 mg al dfa, con buena respuE!sta. A las 3 semanas de tratamiento la paciente presenta

d(sminuiOn de las necesidades de sueo, verborrea e irritabilidad. La conducta ms adecuada es: //a} Cambiar la fluo~tina por sertrallna =...,/' b} ~ntener la dosis de fluoxetina y derivar a especialista r=. ,4suspender la fluoxetina y derivar a especialista d) Disminuir la dosis de fluoxetina y controlar en 2 semanas e) Agregar una benzodiazepina al tratamiento

'y{
-~

/@1
~

46) Paciente de 7taos de edad, que en los ltimos meses presenta de manera progresiva algunas alteraciones cognitivas y de la memoria mciente, asociadas a torpeza motora, sin otras alteraciones

fi?:\
~

~ ~

. . en el exrmeT1slco general nr neurolgico. El diagnstico mas probable es: .. / .. ,~ Enfermedad de Parkinson ~~ Demencia de Alzheimer .. ~emencialrontotemporal d) Proceso expansivo intracraneano e} Depresin

fiJ\
~

~
~

47) <;ul es el tratamiento de eleccin del tras_torno obsesivo c:ompulsivo? a) Bnzodiacepinas b~ticonvulsivantes 9: ntidepresivos ~C"\ (_ \ ../d) Litio \ '-' e) Clorpromazina
48) Una f!1Ujer de 1~ presenta un ex!!ltema petequial que compromete las extremidades infe~s. Presenta hematoguezia en una oportunadad, art~de grandes articulaciones asimtricas y el a lisis de orina demuestra presencia de microhematuria y proteinuria. El diagnstico ms pro able es: ne n IS posttnfecciosa } Sindrome hemoltico urmico
~~~~~~~~~~

Prpura de Shoenlein Henoch

e) M.eningoeeeeeu Ji a
49) Una paciente, cursando embarazo de 7 semanas por amenn.uea. se realiza ecografa transvaginal que no visualiza gestacin intrauterina y HCG es 900 Ul/1. La conducta ms adecuada es: a) Solicitar laparoscopa b) Iniciar metotrexato _,- - --...._ e Solicitar n a ecografa y beta-HC.G en 2 ~ d) Realizar legrado u enno ;wcontrolar -con ecografia y HCG en 7 das

presenta asociada a ardor, reaccin 50) Una una a~nopata preauricular derecha. El diagnstico ms probable es: ~njuntivitis viral byonjuntivitis bacteriana .~ Conjuntivitis alrgica d) Queratitis aguda e) Glaucoma agudo

,P~iente

Q~bilater..al,

tarsa~ presencia de
~ ....., <

51) Un paciente de 57 aos presenta dolru" anal intenso durantt:! la defecacin, seguido de rectorragia escasa: El diagnstico ms probable es: a) }tisceso perianal b) Colitis ulcerosa .._/) Hemorroides d) Ffstut~fperianal ~ Fisura perianal 52) La.primera causa de hemorragia subaracnoidea espontne~es: a) 1-!ipertensin arteril . . ~v- ngiopata amiloidea ~ Malformacin arteriovenosa ~otura de aneurismas saculares e) Meningitis viral

~ Otorrea purulenta

53) Cul de las siguientes alteraciones se puede encontrar en una ---

y~l~
54) Un nio de 7 aos presenta le~nes papulares. pruriginosas especialmente en los pliegues cubitales y fosas poplteas. Su madre es asmtica. El diagnstico ms probable es: afPrrigo b) D~rmatitis de contacto alrgica S).-Pftiriasis alba __./'d) Pitiriasis versicolor ~Etermatitis atpica

/#~~a

f'

~
55) Un paciente cursando ~ncreati'~, evoluciona con_pliguria y desorientacin. Al examen fls~resenta F~, P~SO g. Se solicitan exmenes que demuestran e ti'ninr~ BUN:j3_0_; Na: 138; K;' La.conducta inicial ms adecuada es: dministrar suero glucos~a al JO%, CO~IIiuS de (qaCI dministrar suero glucosado al 5% dministrar suero glucosado al 5%.,-eeA-:2-gfemos de NaCI y 1 91 atitO de ~01 kdministrar suero fisiolgico e) Administrar suero glucosado al1 0%, 2 gramos de KCI_

("

fA '!'

Ci:103.

f'A
~ ~
~

J:

~ ft'
~
~

56) ~n paciente de aos, present infl':!!_nza hace 15 dias. Ac~ualmente evoluciona con fiebre, tQS_ con.expectoracin pu enta y disnea. Al examen ns1co presenta FC: 1171pm, PA: 101/58 mmtig, FR: p , con epi c1o s monares difusas. Se realiza radiografla de trax que muestra ~ acidade~ multifocales, que com.~me~mbos pulmones, algunas de ellas con imgenes de avitacin. El agente causal ms probable es: ) Anaerobios b) Streptoccus pneumoniae e) Klebsiella pneumoniae ~taphilococcus aureus e) Mycoplasma pneumoniae

~
~

57) Un nio de 4 aos consume una su'-tancia desconocida, evolucionando con agitacin ps. omo bucal midriasis, iJeo intestinal, taquicardia, sudoracin y confusin. El cuadro clnico corresponde a un s n rome: a) Narctico . b) Qpiceif e)... Muscarnico

f'
~
~
~

~dre~.~rgico ..

~Anticolinrgico

~8)ft~ . paciente de Q s presenta temblor ceflico y de las e)temidades superiores,

~
~

pecialmente al tomar alguna postura o al tomar un objeto4 El resto del examen neurolglco es ormal. El tratamiento ms adecuado es: Sertralina \ f\ y._r:- r _- ~J 1 b)Prolopa ~~ . ~ e) Diazepam ~ ~ropanolol e} Carbamacepina 59) unpaciente presenta dolor en la cara lateral del codo) mayor con la actividad, especialmente con los ovimlentos habituales de pronosupinacin. Al examen fsico presenta dolor a la palpacin de la na, sin otras alteraciones. El diagnsth::o ms probable es: ) Artrosis de codo
~picondilitis

(!fA

~
~

~
~

.. e) Epitrocleltis d) Tendinitis tricipital e) Atrapamiento del nervio cutneo antebraquiallateral 60)):1na paciente de 30 aos, consulta por dolor abdominal bajo. Se realiza ecografa transvaginal e muestra tero normal y lquido libre ~~n la cavidad peritoneal. La beta-HCG es de 7.000 Ul/1. La nducta ms adecuada es: .t~ ~olicitar laparoscopia ,,.. ~ '-r=-' ~ b) Iniciar metotrexato ~-iG 7' e) Solicitar nueva ecografia y beta-HCG en ~~ a 3 das G d) Realizar legrado uterino e) A~ministrar toclisis de urgencia

~
~ ~ ~
~

A
J

U'

~:1) Qu patologa se asocia con mayor frecuencia a cncer de testculo?


~)

f!A
~

~ ~ ~

Varicocele b) Hidrocele e) Testculo en ascensor ~~~drome de Down / f vrrptorqufdea

~
~

~
~

~
~
62) Una nia de aos es la ms ba a de su curso. Actualmente su talla est en el percentil 5 de la curva ~ crecimiento para su e a Al nacer pes 3.300 gramos midi 47 cm. Su examen fisico r~eu ta normal. Cul es la conducta ms adecudapara cornmuar el estuCITO de su talla? a) aJuar la talla de los padres p~olicitar radiografa de carpo, para calcular la edad sea ~ ~~onfeccionar una curva de crecimiento d) Solicitar cariograma e) Solicitar pruebas de estimulacin de hormona d~.credmlento

~
~

~
~

~
~

,-,
~
64) Una paciente de 2 W , con antecedente de parto prematuro a las 34 semapas, cursa un embarazo actual de..._33' semanas, consultapor contrac~jones ut~, algunas de ellas dolorosas. Se comprueba que presenta dlttrnica uterina de 3 contracciones en 10 minutos, la que persaste luego de un tiempo razonable de observaCin. La conducta ms adecuada es: a Hacer un ;. edicin de longitud cervical por ecografa ~ e) Realizar test de fibronectina d) Realizar un perfil biofsico e) Realizar amniocentesis
6p) Cul de los siguiente hallazgos ecogrficos tiene un peor pronstico en una paciente /mbarazada, con diagnsticoae restriccaon del creciQliento a]:rauterino? / a) Feto creciendo entre los percentiles 5 y 10'para la edad gestacional -::__/ b) Perimetro ceflico en el percentil5 para la edad gestaclonal c)pusencia de movimientos respiratorios d} Oligohidramnios e)Perlmetro abdominal mayor al permetro ceflico

Aj
~

..,
~

~
~

~
~

~
~

. 66) Cul diagnstico nsaonn UI'J nio d ' G que presenta los siguientes indicadores antropometracos: IPE: +2DS, T: +1 D ITE: +1 OS? / . . a) Desnutricin

~s.el

~
~

.$!6

~set":~obrepes

(f \

tl

u '

~ ~

d 1 ad e) Obesidad mrbida 67) La fuerza de asociacin entre un factor de riesgo y una patologra, en un estudio de casos y controles, se estima mediante: . a) Riesgo relativo -....,( b) Likelihood ratio ( ~iesgo atribuible ;(1 . dds Ratio ~) L~ "t" de student Una mujer dEL11-aiios, cursando embarazo en e imestre presenta cefalea. seguida de unfl onvulsin tnico clnica generalizq a. resenta presin arterial de 160/100, edemas y proteinuria ). El dignstico ms probable es: -a) Preclamsia . b) Hemorragia cerebral e) Epilepsia ;({ Eclamsia /e) Slndrome de HELLP

~
~
~
~

~
~
~

~
~

4\')

11'

!'A
69) Una mujer de 36 aos, con una Pf!reJ~ sexua_l nueva consulta J:.!Or dolo! genita~ y aparicin varias lesio.nes ulceradas, dolorosas en Jos labios vagmales y adenopatias ingumales bilaterales. El diagnstico ms probahte es: nfermedad de Behcet erpes genital ~ ) ondilomas acuminados d) Linfogranuloma venreo e) Sfilis primaria
70) para el diagnstico de preclamsia, durante un embarazo, se necesita la presencia de hipertensin al}rial, asociad~ .., )Exaltacin neurolgica ) Edema e) Elevacin de la creatinina plasmtica ~oteinuria significativa ( e) Alteracin de las transaminasas

~ ~

~
~
~

~
(C\

fi'A

~
(ft:l

,_
~
("~'

7H Una paciente acude a control 3 semanas luego de !Jna pancreatitis aguda, en buenas _.c-ondiciones. Se realiza una ecografa abdominal que muestra una lesin gustica de 1O cm de dim;:tro y paredes lisas. compatible con un pseudoquiste pancretico. La conducta ms adecuada es: 1i~ Drenaje por puncin Marsupializacin quirrgica al duodeno e) Remocin quirrgica del pseudoquiste d) Iniciar antibiticos f e) _9.bJiery~r. ~a evolucin

,.

'6f

. -">
V

(!'b
~

;e trmino, de ictericia hasta los 72} musls. Es alimentado con leche materna exclusiva. Su grupo sanguineo es O Rh(+} y el de la madre esLA~h{-). EJ hemograma es normal, la bilirrubina total es 12,2mg/dl y Ja bihrrubma directa es 1,3 mg/dl. El diagnstico ms probable es: Hemlisis por incompatibilidad de grupo clsico b) Hemlisis por incompatibilidad Rh e) Ictericia por lactancia materna exclusiva d~epatitis neonatal ~ Ictericia fisiolgica

Ur~ -reci~ ~ac~~o

~~hijo m~nicia

~
~
~
~

~
~

~
~

~
~

~
~

Refiere adems un cuadro de~ e un mes de evol n. Al examen fsico se recia p~ destacan algunas a~~~rv1cales ie~ emograma con hematocrito: 4% hemoglobin~dl, blancos:@Si'm 11 , infocitps: 8~, neutrfilos: 12% y plaquetas: 8.000. El d?gnstlco ms proba~le es: .._,Q _, _ .,...__ .,_/ ~_knfermedad de Hodgk10 " '"/ --~v-~ ~ .. ' bfleucemia aguda Llv~ ~ /e) Mononucleosis infecciosa ~~o ~~ d) lnfeccif!. por citomegalovirus ~ p-~ ' e) Let:toe:r:fia linftica crnica ~ / \~. 74) yl Jactante de das. de vida presenta vmitos postprandiales explosivos, que se han ido nsificando, asociados a constipacin. Se....ve enflaquecido, pero hambriento. El diagnstico ms obable es: Hernia hiatal ~tenosis hipertrfica del ploro ~ Invaginacin intestinal d) Atresia duodenal e) Reflujo gastroesofgico

73)Un~ftode~ons~~~~~~~~~~~~~~~~~~~~~~~~

1' .

ao

<""'
~

~
~

e
~

~
~

.:...:: 1

, .d

J
. /

75) Una paciente cte 43 aos, presenta desde hace 4 meses disnea de esfuerzos, ortopnea, disnea paroxistica nocturna. edema de extremidades jnferiO@s. Los sintomas han ido empeorando gradualmente y ;?. la actualidad presenta disnea al realizar pequeos esfuerzos. Al examen fsico se aprecia edema de extremidades inferiores, hepatomegaha de 3 cm balo el reborde costal, se palpa desplazamiento del choque de la punta al ~~~~ ~~~tercostal, a la altura de la lnea axilar media y_ se ausculta un soplo holositlico mltt;il. El tratamiento ms adecuado para i nireftralamiento es =i ,...a) nalapril, digoxina y furosemi~ . ~ b) Hi lafna, esptrVI olaetal"'a {iigexil"la

\.C..'-

~
~-

LFw

~
~

da r;gnrgibt;in

~
nt\'\

~Jk:o ca,::,;:{!~::;JI!'~i== anlcoagulante


Enalapril, carvedilol, espironolactona y furosemida

76) El tratamiento erradicador d~ Helicobacter pylori en los pacientes con lcera pptlca tiene como
finalidad: a) Aumentar la velocidad de cicatrizacin (\liD Prevenir las recurrencias 'a} Disminuir la intensidad de las hemorragias Pd) Prevenir el cncer gstrico e) Disminuir los slntomas
77) Una mujer embarazada de trmino est en tra~jo de parto, con contracciones uterinas de 4 en 1 O

t\

minutos, 6 entmetros de dita'ta"Cfn, membranas ntegras y feto en presentacin de vrtice, en espina -2. uego e oras e evolucin no hay cambios en la progresin del parto. El registro f~~ es ranquilizador. La conducta ms adecud~ es: ~g~acer una cesrea por prueba de parto fracasada - b) Iniciar aceleracin oxitcica e) Administrar misoprostol d) Re r rgtl:,u:a artificial de membranas e Realizar frceps

78) La medida ms importante p a Q


a) Antibiticos locales b) Cubrjr con apsit& ~ntibiticos sistmicos ,.

la~cin de un.a herida traumtic

es:

><

d).--AsSO quJrly:gieaetVacunacin andtetnica


79) Un paciente sufre un golpe en el hombro derecho, presentmdo el signo de la charretera. El

diagr{stico ms probable es: a}"'Fractura subcapital /6) Disyuncin acromioclavicular ' - . / gluxacin anterior d) Fractura de clavicula e) Luxacin posterior 80) Un d ~ac con antecedente de insuficiencia 1:ardiaca crnica, secundaria a una , r-==: ,__ cardiopata hipertensiva, de larga data, en tratamiento con enalapril, es iro ol --iroclorotrazda, aspirina y atorvastatina, consultii por aumen o e su sintomatologa y progresin e su d1snea, que actualmente aparece ante minimos esfuerzos. El examen fsico demuestra FC: ~.regular, PA: ')J0/~0, crpitaciones pulmonares escasas blbasales, y un RR2T sin soplos. La ,conducta ms adecua a para 'mejQrar el pronstico en este p~1C1ente es: a) Cambiar el enalapril por un inhibidor del receptor de angiotensina 2 ...-b) Iniciar digoxina e) Iniciar propanolol .. ....____ d) Jpiciar furosemida Alniciar carvedilol
---~"'

~~ciente ~s,

e~

~
81)

f!t'

el!'

~ ~

ametro. La conducta ms adecuada es: ontrolar en atencin primaria, ya que cierra sola / ) erivar a resolucin quirrgica a los 6 meses de edad '.../ e) Derivar a resolucin quirrgica al ao de edad d) Comprimirla con una faja e) Resolver quirrgicamente a la brevedad

~~nio de 3 meses de edad presenta una_bernia u~ con anillo palpable de un centmetro

82) Un paciente sufre un accidente de tr.nsito, resultando con fractura de pelvis. Evoluciona con
imposibilidad de orinar y globo vesical. La conducta ms adecuada es: a) Instalar senda ~JeiBtC1l b} Instalar sonda Foley e) Instalar cistostomla ea IZar Soli~_itar ecografia renal y vesical 83)/iQu parmetro es el ms importante para manejar la prevencin flcrovasculares en la diabetes mellitus? resin arterial menor a 130/80 Realizar ejercicio peridico ) Hemoglobia-glicosilada f!lenor .a . 8_% d) Su~pender E'!t tabaquismo e) ~ntene glucosuria negatilJa
'
/

""

K.,

(!!'
~ ~

'

las complicaciones micro y

ft'
~
~

~
~

~
~ ~ ~
~
. . _ /,

4) Se calculan los casos nuevos de uns1 enfermedad infecci~ a lo~o. El parmetro descrito es: a) Prevalencia b) 'Iasa de morbilidad general ,. e) Incidencia acumulada d) Densidad de incidencia e) Ta?a de morbilidad especifica

~te de 33 aos consulta por aparicin de varias eeulas de 3 a 4 mm en e~


bi1ica_~~o

dolorosas. El diagnstico ms probable es:

7'.

Verrugas
Pitiriasis rosada g rttata~:}filis seGYRdaria ~ol~co cut 1lagioso

e)

~oriasis

~ ~
~

~
~

. A
'

86) ,Un paciente de 70 aos pierde sbitamente la visin del ojo derecho. Al examen se aprecia tdida del rojo pupilar de dicho ojo. Eiaial)nostlcu 111s J3Fobaele es:

,.C.atarata-

"Hemorragia vitrea Glaucoma agudo d) Trombosis de la arteria central de la retn:. e} Degeperacin macular relacionada con ls1 edad

~ ~

87) Una mujer de 2~os presen.ta.un.ndul.Qmamario.c.ie\t de consistenci~omosa, fcil de m9~i~~~-de b~!des lisos. Es indoloro c1la palpacin. El diagnstico ms probable es: 71 Cncer oe mama ~b) Tumor philloides .
~ibro9denoma

"'

/ d) PaP,iloma mamario e) Qufste mamario

~
~

~ ~
~
~
~

Realizar Papanicolaou --~ Medir el antgeno prosttico---' e) Tomar la presin arterial en pacientes sanos prt="luoracin del agua potable

~Cul de las siguientes acciones corresponde a una medida de prevencin primaria-o/

e) "reffiaf-mamogffifia...--,

~
Prueba Global nmero 1

1) La primera medida a tomar ante una hlperkalemia severa es:


a) Administrar suero fisiolgico endovenm; '1?) Administrar furosemida endovenosa e) Administrar adrenalina endovenosa d) Administrar calcio endovenoso e) Administrar amiodarona endovenosa 2) Un paciente de 58 aos, fumador, con antecedente de flbrosls pulmonar, secundarla al uso de nitrofurantoina durante la Infancia, pre!;enta un cuadro de tos, con expectoracin y compromiso del estado general. Al examen fislco se obnerva orientado, bien perfundldo, decafdo, fe ril hasta 38,5C, mHg y con frecuencia cardiaca de 93x', frecuencia respiratoria de 19x', presin arterial de 08/6 satura 95% con FI02 ambiental. La ausc;ultacln pulmonar demuestra crpitos bllatera es, mucho ms Intensos en la base pulmonar Izquierda. El examen cardiaco es normal. Se solicita radiograffa de trax que muestra una condensacin en el lbulo Inferior izquiefdO y los signos de flbrosls pulmonar en el resto de los campos pulmonares, similares a los que presentaba en radlogratra previas. La conducta ms adecuada es: a) Administrar penicilina benzatina intramuscular y enviar a domicilio b) Iniciar amoxicilina oral y controlar de forma ambulatoria e) Iniciar amoxicilina + cido clavulnico y controlar de forma ambulatoria 4 } Hospitalizar en sala e iniciar ceftriaxona endovenosa-.<: e) Hospitalizar en UCI e iniciar ceftriaxona ms claritromicina 3) Una paciente de 33 afios consulta por dismenorrea moderada, que responde parcialmente al uso de cido mefenmlco. Se realiza una ecog1afia transvaginal que demuestra un t!!!!lor anexial quistlco, de 5,~- cm de dimetro, con paredes lisas y sin lncrecenclas El diagnstico ms probable es: a) Cncer de ovario b) Teratoma quistico ...G> Endometrioma -.../' :?'d) Cuerpo lteo e) Quiste folicular 4) La insercin velamentosa del cordn se asocia a: a) rotura prematura de membranas. b) hemorragia fetal antes del parto. e) torsin del cordn umbilical. d) malformaciones fetales. . . \--:-~ ~)malformaciones uterinas. 5) Las Infecciones en pacientes con n!!ttropenla febril son producidas principalmente por: a) Bacilos Gram negativos anaerobios facultativos --:> VJ ~ ,..... --.Q. b) Anaerobios ,- ~~ C..'V~ ~ ' e) Staphylococcus aureus meticilino resistEmtes (SARM) d) Aspergillus niger e) Streptococcus pneumoniae 6) La causa ms comn de muerte perlnatal en Chile es: a) Hipoglicemia b) Infeccin e) Malformaciones congnitas d} Traumatismos eyPrematurez 7) Nlujer de 50 aos, consulta por aumento de volumen doloroso de las .!tllculacloneslntru:falng~s distales. Al examen ffslco se compruebu que el aumento de volumen es de consistencia dura. Cul es el diagnstico ms probable? a) Artropatfa psoritica. b) Artritis reumatoidea. e) Artritis gotosa. d) Artritis reactiva. A Artrosis.

e'
~

~
~

~
~

~
~

~
~

e"'
~

e:"
~

(lA
~

~
~ ~

~ ~
~

~
~
~

~
~

~
~

~
~

~ ~
~
8) Las complicaciones cardiovasculares postoperatorias son ms frecuentes en pacientes con antecedentes de: a) hipertensin arterial b) insuficiencia cardiaca e) edad mayor de 65 aos d) fibrilacin auricular e) angina de pecho 9) En un paciente con una placa blanquecina retlculada adherente en mucosa yugal, la primera posibilidad diagnstica es: a) Cncer oral -b) Candidiasis e) Liquen plano d) Estomatitis aftosa recurrente e) Linea alba

'"'>
~

,._,
~

1O) Un paciente de 65 aos, con antecedente de reflujo gastroesofglco, en tratamiento Irregular con
omeprazol, consulta por exacerbacin Importante de los slntomllS, que Incluso lo despiertan por la noche, a pesar de haber aumentado la dosis de omeprazol a 40 rngldfa. El examen ffslco no presenta alteraciones. La conducta ms adecuada es: a) Mantener el omeprazol y agregar y bloqueador H2 b) Solicitar manometria esofgica e) Realizar tratamiento erradicdor de Helicobacter pylori d) Solicitar pHmetrla de 24 horas .,e>r Solicitar endoscopia digestiva alta 11) Cul ~~1-.s..alguientes-eonductas es ms adecuada frente al diagnstico cllnlco y ecogrflco de tumor tsticular? ------- _ __.;:._ _ _ _ ___:;.____.,;::;...___ a)SOilcltar c1nt1grafia osea b) Realizar orquectomia por via inguinal ~a) Realizar biopsia testicular por puncin d) Realizar exploracin quirrgica testicular por vla escrotal e) Solicitar tomografia axial computada de retroperitoneo

~
~ ~
~

\ -P .
~
c.P Oj fi\
..9
-

~<JC

~ \j. .,

""
~

)p

~
f\JD

\en ( l \"~

(tA....-v\

12) Paciente hombre de 5 aos, autovalente, con antecedentes. de hjpefteAslR-SfteFial-diabetes.Jipo 2 y p a leve de me e ace 3 a aos. Hace una semana lo notan aptico y con comportamiento extrao Hace tres dfas, sall de compras y un vecino debi traerlo a casa pues no recordaba el camino. Desde hace dos dlas habla Incoherencias, se agita y presenta Insomnio. El diagnstico ms probable es: , ~a) demencia por enfermedad de Alzheimer b) demencia multiinfarto e) sindrome confusional agudo d) depresin enmascarada e) psicosis aguda 13) Debe sospecharse hlpotlroidismo congnito en todo recin nacido que presente: a) Irritabilidad b) bajo peso para la edad gestacional e) ictericia prolongada ../ .Ai) temblor fina. e} llanto frecuente 14) Un paciente de 65 aos presenta una hemiplejia Izquierda de Instalacin brusca, con una tomograffa axial computada de cerebro normal a Jos 90 min de Instalado el dficit. La conducta ms adec.u.ad.!_es: a) iniciar tratamiento con vasOCiiraracfores-r:eb~~ b) administrar antiagregantes endovenosos ... ,...-e) bajar la presin arterial si est elevada. d) realizar angiografla cerebral. e) iniciar protocolo de trombolisis endovenosa. __....

w
3-<.

~._()_,'Lt ~

"\.9-") o ~ c-~(.s_J~

"
~

(!'

~
~
15) Primigesta de 24 anos, embarazo de 36 semanas de evolucin fisiolgica, consulta por cefalea y tinltus. Al examen presenta presin arterial de 170/100 mmHg, edema generalizado y reflejos osteotendfneos exaltados. Exmenes det laboratorio: proteinuria (+++), hematocrito 27%, plaqu~s 35.000/mm3. El diagnstico ms probable es: a) preeclampsia moderada. b) hipertensin arterial transitoria. e) hipertensin arterial crnica. _...dtljreeclampsia severa. e) hipertensin arterial crnica ms preeclampsia sobreagregada. 16) Hombre de 54 aos que consulta en Servicio de Urgencia por dolor torcico de d.ps..boras-deduracin. Ingresa a la Unidad Coronaria donde se demuestra un Infarto de miocardio en cara fnferolateral (CPK 900 U/1 para un normal hasta de 250 U/1), sin .orida!_ El ECG evolutivo demuestra Q. inversin de .Qnda..:t Se mantiene hemod!lnmicamente stable, sin Insuficiencia cardiaca, durante toda la hospitalizacin. Una semana ms. tarde, se practica un test de esfuerzo y el paciente no presenta angina ni nuevos cambios al EGG. Cul de las siguientes drogas es la ms til para seguir tr~tando a este paciente? ,..S) Acido acetilsalicllico ._;,;. 0<.- ~e}~ \'l. a-J~ (forO r~~ o.. b} Beta bloqueadores e) lnhibidores de la enzima de conversin-? tf'... ~-* C&.r ~~ ~ d) Anticoagulantes orales e) lsosorbide oral 17) Con el objetivo de conocer los facto.-.~s de riesgo asociados al desarrollo de cncer vesical, se estudi a un grupo de enfermos de sexo masculino entre 40 y 50 anos de un rea determinada. Se seleccion un grupo de hombres sanos de la misma edad y de la misma rea. Se estudiaron los factores a los cuales estuvieron expuestc>s ambos grupos en los ltimos 3 aos. Cmo se denomina este tipo de estudio? a} De cohortes paralelas. _.b}'De casos y controles. e) Experimental. d) Cuasi experimental. e) Retrospectivo modificado. 18) Gestante de 38 semanas que Ingresa con trabajo de parto. Durante el periodo de dilatacin presenta cuadro de dolor brusco. A la exploracin usted objetiva metrorragia escasa y aumento del tono uterino a la palpacin abd,ominal qu~sulta muy doloroso. Cul serf~ su diagnstico?: ___a}'Rotura uterina b) Placenta previa. e) Desprendimiento de placenta. d) Corioamnionitis hemorrgica. e) Rotura de vasos previos. 19) Cul de los siguientes cuadros clinleos se asocia con mayor frecuencia a trastornos de la conducta alimentaria? a) Trastorno depresivo mayor. ,t:r}irastorno por angustia generalizada. e) Trastorno por somatlzacin. d) Trastorno disociativo. e) Trastorno delirante. 20) En un paciente cirrtico con ascitis a tensin. Cul es el parmetro que mejor predice los resultados del tratamiento y evolucin de la ascitis? a) Medicin del perfmetro abdominal b) Concentracin del sodio plasmtico e) j;xcrecin de sodio en orina de 24 horas .dfProtrombinemia e) Medicin del volumen urinario de 24 hora!;

~
~

~ ('A
~

~
~

~
~
~

~
~

~ ~
~ ~
~ ~ ~

~
~
~

~
~

~
~

~
~ ~
~

~
~
~

~
~

~
~

~ ~

~---21) Cul de los siguientes factores est ms fuertemente asocl1do (parto prematur~)
a) Hbito tabquico b) Diabetes gestacional e) Preeclampsia en embarazo anterior ~rto prematuro previo e) Cesrea anterior --

~
~

o ~

3-.. l.e~
~

22) El frmaco antihipertensivo de eleccin para un diabtico es: a} Hidroclorotiazida --b) Nitrendipino e) Hidralazina ~titenolol ?1 Enalapril 23) El primer parmetro que se altera en un paciente en shock hlpovolmico es: a} el estado de conciencia. ~~..,presin arterial. ,..,..ere pulso. d) la perfusin tisular. e) la diuresis 24) En un paciente de 38 aos, con antecedentes de sifilis tratada hace diez aos se encuentra un FT AABS positivo y un VDRL negativo. Cul es la conducta ms adecuada? a) No tratar b) Tratar como sffilis secundaria tardla. e) Tratar como sffilis terciaria. d} Descartar compromiso neurolgico con puncin lumbar. ~ontrolar con exmenes en 3 meses. 25} Cules de las siguientes alteraciones metablicas es ms probable que se presenten con el uso . . .\ . prolongado de diurticos de aS'ag -=> f=-> ~ Lv-.: ~ .~ v LC -. 1 J-... ~ ~_,..._,_, a) Hiperkalemia, hiperuricemia, acidosis metablica L ) ' b) Hipokalemia, hipouricemia, alcalosis metablica 1 e) Hipokalemia, hlpouricemia, acidosis metablica d) Hiperkalemla, hiperuricemia, alcalosis metablica _pYHipokalemia, hiperuricemia, alcalosis metablica. 26) La conducta ms apropiada ante el hallazgo de un soplo continuo llll VI, de la base, Irradiado al dorso y pulsos-saltones en un lactante .d~s es: a} explicar a los padres que se trata de un soplo Inocente ~)1hdicar evaluacin cardiolgica, para confirmar un ductus arterioso persistente e). controlar peridicamente, ya que es probable que desaparezca d) realizar un hemograma para confirmar anemia e) hospitalizar por sospecha de endocarditis bacteriana 27) Un paciente de 45 aos consulta por un cuadro de Instalacin lenta, en varios meses, de parestesias en el 4 y so dedos de la mano derecha, asociados a debilidad para flectar dichos dedos. El resto del examen no demuestra alteraciones. El diagnstico de sospecha es: a) Dedos en gatillo b) Sindrome de tnel carpiano _y}-Atrapamiento del nervio cubital d) Tendinltls de Quervein e) Lesin del nervio radial

~
~

~
~

~
~
~

e-"

f"

("

fV p l ~(_ (3f)
~

.br'-.) J

c.. -o

fA (!'

~(I~~'<V

28) Paciente de 25 aftas atendido en el s rvicio de urgencia por una fractura de pierna derecha. Se

f:'
~

f"
~
~
~ ~ ~

indic yeso bota larga y reposo con pie a en alto en su domicilio. El paciente vuelve a consultar a las 12 horas por dolor intenso y progresivo e la pierna afectada. Cul es el diagnstico ms probable? a) Desplazamiento de la fractura b) Trombosis venosa profunda e)J:ompresin neurolgica A Sindrome compartimenta! e) Necrosis muscular eumonias recurrentes, tos crnica productiva. expectoracin if"r,qne-empeora en ls maftanas y con el decbito. Al examen se aprecia compromiso del e tado general, acropaquia y crepitaciones en ambas bases pulmonares. El diagnstico ms probabl es: a) CJlcer pulmonar. ~..tUberculosis pulmonar. e) bronquiectasias. . d) bronquitis crnica. e) enfisema pulmonar. '

29) Paciente

m.u.cQ.ourulenta~:(nalmente""

d~2~~o~ con historia de

~ ~
~ ~ ~

J'~ ,P

~~

""

~ (!'

30) Paciente de 26 aftos consulta por pre entar, desde la adolescencia, miedo intenso y persistente a hablar en pblico, por temor a ser mal e aluado por los dems. A causa de ello no asiste a dar su examen de ttulo. Se descarta pato logia o mtica concomitante. Cul es el diagnstico ms probable? a) Trastorno por ansiedad generalizada .. L, b)FobiaSocial. -~ \fV\\:.e...d,_..o di.... C/l~ ~ u~ " 0 _. ~0'-";h;z~ e) Agorafobia. d) _!;obia especifica, tipo ambiental. .e)Fobia especifica, tipo situacional. 31) Se realiza un estudio para evaluar la alidez del diagnstico clfnlco en 1.000 pacientes fallecidos por infarto agudo al miocardio; se utiliza como estndar de oro los resultados encontrados en autopsia. "/ Diagnstico cUnico Hallazgo de a topsia Sin infarto Total l~fart~.2 J lnfarto .. 80 240. \.! __. 720)-.:J'~ 40 No infarto 760 1000 Total 200 800 .. :.: De acuerdo a estos datos, el valor predic ivo positivo del examen clfnlco fu.e: .- --a)20% Al~ .11 ~ ~G,o' _l. b) 33% -\-O J. .t V'./u-v o:....Jl-0 ' G e) 67% ""2....'-\. O ~ s,Y ~~ ~~~ ~~ ~

~ ~
~
~

1... 1) ,

@g)J\-

"
'!'
~

f'l',
~ ~

32) Cul de las siguientes situaciones e ms sugerente de

~
~
~ ~

primaria? ) A1perpgmentacin cutnea asociada a hi okalemia severa . b) Oebilidad-mt;JsGular-asociada-a-hiperglice ia_se~ra- e) Hipotensin sostenida en presencia de hi erkalemia d) Hipotensin sostenida er-t-presencia-de-hi natremiae) Debilidad musctJiar-asociada-a-hipernatr-e ia-e-hiperkalemia

unl~n-~~-;~~;~~-c~~-~~~rarr~~al-~~~)
,
t\-\roQ( t"'

~ UC'Vl(\ 'lr'-t\1~0\. 1 ''-

V . f:l ~ ~, w- ~ -

~
~

~
~
~

33) Una 'nia de 6 aos tose con frecuenc a durante un control de rutina. La madre refiere que tose de noche, cuando hace ejercicio y siempre ue tiene una infeccin de las vfas areas superiores. No hay otros datos anamnsticos relevantes y el examen trsico es normal. Adems de una radiograffa de trax 9u examen es el ms adecuado parad terminar l~causa_!!_f!_!!_U t~s? .-:-> Pruebas de alergia. b) Radiograffa de esfago-estmago-duode o e) Broncoscopfa. d) Espirometrra. e) P~ueba cutnea para tuberculosis.

~
~
~
;;,

~
~

..,
~
~

34) El hombro doloroso se debe con mayor frecuencia a: a) artritis glenohumeral. ffirtrosis glenohumeral. e) uncoartrosis. .... ..L d)tendinitisybursitis. .....==, ~~~ (Gl~ e) gota aguda 35) Cul es la conducta ms adecuada frente a una paciente d 40 aos, asintomtlca, que presenta un Papanicolau Informado como lesin de alto grado? a) Realizar hlsterectomfa b) Repetir el Papanicolau en 3 meses e) Realizar legrado blpsico fraccionado d) Realizar biopsia cervical en los cuatro cuadrantes ~ealizar colposcopfa y biopsia 36) El diagnstico de hepatitis aguda por virus B se confirma e n la deteccin serolglca de HBsAg de: e;, ~ a) Anti HBc total b) Anti HBc lgM ..crf!Be Ag -Dr---lAi'- _-.\ , 1\ 2,1 ;:> e d) DNA-VHB .... C.") 0J \ e) Anti HBs

~
~
~

~ ~ ~

~
~

;y

'1 N~-'> ~V~~.

~
~
~
~

37) Paciente asintomtica de 49 af\os, multfpara de 5, esteriliza a qulrrglcamente, presenta menstruaciones regulares. Al examen ginecolgico presenta tero aumentado de tamao qulvalente a embarazo de 10 semans, compatible con lelomloma. En control efectuado hace un ao tero presentaba Iguales ~ caracteristlcas. La conducta ms adecua;;a es: . c_.,.9 \o--- C/1, a) controlar cada 6 meses u.)..JO ('..J _.,T~,_P b) indicar agonista de hormona liberadora de gonadotrofina ~ --::.. \f' e/ ~ : e) iniciar terapia con progestgenos . d)realizar histerectomfa e) iniciar terapia hormonal combinada 38) Paciente de 75 aos, que presenta desde hace tres meses de esfuerzos progresiva, actualmente de pequeftos esfuerzos. de piel y mucosas, con tinte sublctrlco de las escleras. FC 11 hallazgos. Hemograma: GR 1.200.000 x mm3; hematocrito 11% plaquetas 54.000 x mm3; reticulocltos 0,4%. Frmula leucoclta osls ++ Caracteres moolglcos: anlsocitosis.++, Ferremia 200 pg/dl Capacidad total de fijacin de Fe 300 pg/dl El diagnstico ms probable es una anemia: a) f_,-ropriva por sangramiento digestivo crnico )\> )>)-por sfndrome mlelodisplsico ~'()) c).por enfermedad inflamatoria crnica \f ~ d) megaloblstica - c<L\. ~~.<" \~ e) hemolftica autoinmune ' ompromlso del estado general, disnea xamen ffslco: Intensa palidez cetrina /mln, discreto edema pretlblal, sin otros ~g/dl; leucocitos 2.300 x mm3; la normal. ullocitosis +. OH 630

~
~

~
~

~
~
~

~
~

unt. (G

'O,L\J'-..

39) En una esquizofrenia incipiente sin agitacin, el tratamlent farmacolgico de eleccin es: a) Medicacin ansioHtica b) Antidepresivo tricfclico e) lnhibidor selectivo de la recaptacin qe serotonina. ~~ ~~rolptico. de depsito -:->:> -~eurolptico oral ~~

~~
~

CL1l

1.

--

~
~

4\)

~
~

~
~
~

..,

f" ,..
40) Un nio de 10 aos presenta un cua ro de 2 dias de evolucin de malestar general y nuseas, evolucionando con dolot;,.abdomiRal-y-v mnos:er.l 4 epoftUAklades. Algunas horas, previo a la consulta, se agrega desorientacin y lu go sopor. Al examen fsico se aprecia deshidratado, en Glasgow 13, con respiraciones profund s, su pulso es regular a 1391pm y su presin arterial es 90/60. El resto del examen fisico no aporta ma or informacin. La saturacin de oxigeno por pulsometrfa es 98%. La conducta inicial ms adecuada s:
a)j.Riciar hidratacin con sales de rehidrat cin, por vla oral

fA
~ f'
~

.)1 ~dministrar 20 cc/Kg de suero fisiolgiclpor vla perifrica y reevaluar en 1 hora


e) Solicitar hemoglucotest e instalar va ve osa perifrica d) Administrar suero fisiolgico e hidrocorti ona por via endovenosa y solicitar hemograma, electrolitos plasmticos, glicemia y catcemia e) Solicitar hemograma, electrolitos plasmticos, glicemia, calcemia y TAC de cerebro y decidir conducta segn hallazgos \

"
~
~ ~

~ ~

,.,
~ ~
~

41) La ecografia para cuantificar la gestacin: a) 5 y 10 ~1y14 e) 15 y 19 d) 20 y 24 e) 25 y 29

tran~lucencia nucal del feto se debe efectuar entre las semanas de


l
1

42) Cul es el diagnstico ms probablf de un paE!._ente abdominal bajo y dolor severo al defeca ? a) Fistula extraesfinteriana compleja. b) Fluxin hemorroidal aguda. 1 e) Absceso perianal. d) Cncer rectal. e) Absceso petvirrectal.

feb~ con signos de Jrritacln vesical, dolor

~ ~

~
~

43) Un nio de 7 anos presenta un cuadr de edem.a..palpebral y hematuria en una ocasin. Al examen ffsico est hl~ertenso, con edema facial, escrotal de extremidades Inferiores. El sedimento de orina demuestra hematuria con 11% de dismo la. Al interrogar a la madre, refiere que el nio habria sufrido una faringitis hace un par de semanas. ~diagnstico ms probble es:
a) Enfermedad de Berger b) Nefrosis lipoidea e) Glcpnerutonefritis mesangiocapilar ~}-6fomerulonefritis aguda postestreptocci a e) Slndrome hemolftico urmlco
1

~
~

~
~
~ ~

~
~
~

fA
~

44) En una hemorragia vitrea e. 1elementd semlolgico ms importante para el diagnstico se relaciona con:. ' 1 ~lejofotomotor - \._1 ~~,'""\ / b) campo visual por confrontacin \....iC\ ----~ P \" e) proyeccin luminosa ' d) rojo pupilar e) visin con agujero estenopeico 1 45) Un paciente hospitalizado presenta 1Cts siguientes resultados de gases arteriales pH: 7.04 p02: 82 mmHg pC02: 38 mmHg .HC03: 14 mEq/1 Est situacin describe una acidosis: - --a) respiratoria y metablica b) respiratoria con la compensacin metabl ca esperada e) metablica y alcalosis respiratoria d) metablica con la compensacin respirat ia esperada 1 _...e(metabtica no compensada

+ 30

~~-;.(el\

1G

'"'

'30.
A c.
\'J~

"'
~
~ ~

~~- p.J"d

.:=Jl 2... r
<..u
J

un..
t.~ev~)

a
'.\

._p' ;:;-l

( ,, . ._ - (

1\ ,....

,.

~
~

~
46) Una paciente de 67 aos, fumadora de 25 paquetes ao, con ulta por disfonfa de 4 semanas de evolucin, asociada a tos lrrltatlva ocasional. El examen ffsico e normal. La conducta ms adecuada es:
a) Indicar suspensin del tabaco e iniciar AINEs y antibiticos b) Solicitar una radiograffa de trax e) Solicitar una tomograffa axial computada de cuello d) Indicar suspensin del tabaco y reevaluar en 4 semanas e) Solicitar una nasofibroscopla

~
~
~

~
~ ~

47) Paciente de 49 aos, multfpara de 5, con antecedente de hls erectomfa vaginal por mlomatosls hace un mes. Relata escape de orina continuo desde la segund semana postoperatoria. Cul es el: diagnstico ms probable?
a) Dano esfinteriano iatrognico b) Urge incontinencia 9)"Fstula vesicovaginal d) Fistula vesicointestinal e) Incontinencia urinaria mixta

~
~
~

l ()

~ ~

48) Lactante de 1 ao, presenta desde los 3 meses un episodio 1mes de sfndrome bronquial obstructivo. Adems tose en la noche o con el llanto. Ninguno e los episodios ha requerido consulta d9"gencla. Antecedente de atopia en la madre. Cul es la ter pla farmacolgica de eleccin?

,a'f Broncodilatadores de accin corta durante las crisis


b) Broncodilatadores de accin corta a permanencia e) BroncodilatadQre~ de. acci~n corta y corticoides in~alados. d) Broncodilatadores de accin cofta y corticoides orales durante la e) Broncodilatadores de accin prolongada

~l~ e-~
~

~
~

49) El principal determinante del pronstico a largo plazo de u paciente diabtico e hlpertenso que present hace un mes un Infarto del miocardio de pared antero eptal es:
a) los hallazgos de la coronariograffa. IK'el estado de la funcin ventricular. ~ t'c) el grado de control de sus factores de riesgo. d) la presencia de angina. e) la presencia de extraslstolfa ventricular.

f-12\.1 1
l. ' t .k- ./ \ "" 0
_ ...

~
~

~
~
~

50) En urgencia se hospitaliza a un paciente con un trastorno polar en tratamiento con litio que presenta una Insuficiencia cardiaca congestiva. Su tratamiento con litio se mantiene sin cambios, sin embargo, a los tres dlas la lltemla se eleva a 1,4 meq/Lt. Este e mblo puede deberse a: .~ t
a) la administracin conjunta de un cardiotnico b) uso de inhibidors de la lECA .o}~ administracin de diurticos d) hipotiroidismo e) administracin de benzodiacepinas ... \.p..

LA..-.. a

~,~

OJ'""

o 1cB - t , L..

~.:/""\ ~ .h. 1 , r ........... vw\.. l ~ 'I.Y'-"- \../f '\

f) t }'-ft J : ~~ 0 1.. vre.'


'
e--

~ ()..__~~(\

~ Broncodilatadores en aerosol a permanencia

51) Carpintero de 62 aos, ex fumador de 20 cigarrillos diarios Consulta por presentar tos crnica con expectoracin mucosa y disnea progresiva, actualmente de m dlanos a pequeos esfuerzos. Examen pulmonar: sibllancias bilaterales escasas y disminucin globa del murmullo pulmonar. Espirometria: CVF% Terico: 99 (Basal) 112 (Post Broncodllatador) VEF1% Terico: 54 (Basal) 56 (Post Broncodllatador) VEF1/ CVF %: 44 (Basal) 43 (Post Broncodllatador) Cul de las siguientes Indicaciones es la ms apropiada? a) Broncodilatador en aerosol, expectorantes y profilaxis peridica on antibiticos . f1 _ f. _ , r ~ -...J_.. r"\ .J b) Antibiticos y broncodilatadores slo durante las descompensac ones v 1 N v '-"~ "-J ~
d) Corticoides por vi a inhalatoria a permanencia e) Antibiticos y expectorantes slo durante las descompensacion s

C.

V~

,(CA..

P'

!\

!'.

A.

ll\,

,. ""'

52) Mujer de 19 aos, en el quinto dla d en fosa ilaca derecha y ha tenido en cu examen hay resistencia muscular en la normal. Temperatura axilar es de 38C era posibilidad diagnstica es: pendicitis aguda. ) olecistitis aguda e) proceso inflamatorio pelviano. d) salmonellosis intestinal. e) pielonefritis aguda.

menstruacin. Desde hace 24 horas est con dolor continuo tro oportunidades deposiciones liquidas sin sangre. En el ona del dolor y esbozo de Blumberg. Examen ginecolgico rectal de 39.5C. El recuento de blancos es de 22.0001mm3. La

53) Una mujer de 24 aos, consulta por rtralgias de manos y fiebre ocasional. El examen flsico no aporta mayor informacin y los exmen s de laboratorio arrojan llnfopenia y protelnuria. El diagnstico ms probable es: ' a) Arteritis de la temporal 4(upus eritematoso sistmico e) Leucemia aguda d) Artritis viral ,-e) Vasculitis de vaso pequelio 54) Un escolar con hepatitis A se debe ospitalizar si: a) las transaminasas se mantienen elevad s por ms de un mes ~~llirrubinemia es mayor de 12 mg/dl ....)"! vi \lempo de protombina es igual o men d) presenta anorexia persistente e) presenta intenso prurito
.__.'>)

~~J._.,.~ rcA._U_.-6

P~~~~~

r,

~ ~

{tf\

,_
('t'l

55) Cul de las siguientes alteraciones ioqufmicas se observa en un paciente con hipotiroidismo pril!)8rio? .aYflipertrigliceridemia b} Hipernatremia e) Hipercolesterolemia d) Hipocalciuria e} Hipercalcemia 56) Una paciente de 40 aos, cursando e barazo de 33 semanas, multlpara de 3, doble cesarizada_ . '"""? ~ anterior, presenta rotura de membranas, iniciando ttaffiijO(I parto espontneo a la hora despus. Al __ ~ examen presenta 2.contracclones en 10 inutos y!i_ene 2 cm de dilatacin cervical, con un . borramiento de 80%. El registro basal n estresante resulta normal. La conducta ms adecuada es: .L-e V\.,. ......~ministrar corticoides y antibiticos y d ar evolucin espontnea b) Administrar corticoides y antibiticos e in errumpir el embarazo en 48 horas ~.f\-.- L-..__P .. e) Realizar cesrea ~ d) Administrar misoprostol por vla vaginal n _ r . -~ \),.-).,..... e) Administrar corticoides, antibiticos y toe llticos, manteniendo una conducta expectante 1~ ~ c_, Cl

,._
{ir~!\

(ti\
(ffl\
~

~
~

~ ~
~

57) La mejor estrategia de implementacl n y renovacin de los equipos mdicos de un hospital, de acuerdo a las especificaciones tcnicas e sus profesionales es: a) comprar el equipamiento ms moderno d sponible. b) comprarlos de acuerdo a los precios y ga antias que ofrecen los proveedores. e) renovarlos anualmente para evitar prdid de garantlas y obsolescencia. d) externalizar todos los servicios que requi ran de equipamiento. e) comprar al mismo proveedor, garantizan o asl compatibilidad tcnica.

~('Vl

ta-o
~

~
~

f1i\!ll\
~

58) Un paciente de 3 aflo,.de s xo mas ulino, consulta por presentar cefalea hemlcranea desde hace tres dfas ~ .~ esta de a co ol. Los sintomas se repiten cada tres a cuatro horas. Presenta morrea unilateral, lagrimeo y iosis. La principal sospecha diagnstica es: a} Migraa Oaqueca) clsica b) Neuralgia de 1ra rama del trigmino e) Trastorno de ansiedad d)~fteritis de la temporal .!f Cefalea en racimos (cluster headache)

,.,.
~
~

59) Paciente de 30 aos, multfpara de tres, cursa embarazo de 2 semanas. Se pesquisa gllcemla en : ayy.pas de 108 mg/dl. Cul es la conducta ms adecuada? .....arRepetir glicemia en ayunas b} Realizar glicemla postprandial e) Realizar prueba de tolerancia a la glucosa d) Iniciar tratamiento con metformlna e) Restringir ingesta de hidratos de carbono 60) Una paciente de 49 aos, presenta elevacin persistente de las transamlnasas, con valores cercanos a 120 mU/ml, con blllrrublna, GGT y fosfatasas alcali s normales. Refiere que toma una copa de vino con el almuerzo y que nunca se emborracha. Se s licitan exmenes que demuestran ANA(), ANCA(), ASMA(), VHBsAc(-), VHBsAg(+), lgM antlcor -}, VHC-Ac(-). El diagnstico ms probable es: a) Hepatitis autoinmune ~ ~ b) Hepatopatra por consumo de alcohol e) ~teatohepatitis no alcohlica .-dfHepatitis 8 crnica e) Hepatitis B aguda, sobre hepatopatfa crnica

e_,

61) Preescolar de 4 aos consulta por fiebre de 4 d(as de evolu In. Refiere dolor seo en parte Inferior del fmur derecho que le hace claudicar. En el examen sico destacan palidez, esplenomegalla, petequias y equimosis aisladas en el tronco. ul es el diagnstico ms probable? a) Prpura trombocitopnico inmune b) Lupus eritematoso sistmico e) Artritis reumatoidea juvenil ,..d}teucemia aguda e) Artritis sptica 62) Paciente de 22 aos, sufre calda sobre la mano con el cod en extensin. Presenta dolor, leve : aumento de volumen e Impotencia funcional del codo, con imp slbllldad de pronosuplnar en forma activa.EI diagnstico ms probable es una fractura: a) de epitrclea. b) marginal de capitelum. e) supracondflea d) de cpula radial. e) de olcranon. 63) La conducta ms adecuada frente a un nio asintomtlco, on pie plano, ~uyo arco plantar aparece a~~lperextender el primer ortejo, es:
~~bservar

b) Indicar uso de plantillas e) Indicar ejercicios de fortalecimiento de la musculatura plantar d) Solicitar TAC de pie e) Resolver quirrgicamente
6~

~~

El momento ms adecuado para reducir una fractura nasal esplazada es: ~ en el curso de las primeras 24 horas. J b) en el curso de la segunda semana e) en el curso de los primeros tres meses. d) en cualquier momento {el tiempo no es un factor importante). e) ninguno, es preferible esperar que consolide para rinoseptoplast a. 65) Paciente de 37 aftos, hlpertenso de diagnstico reciente. A tualmente con PA de 140/100 mmHg~ Exmenes de laboratorio: orina con ~y sin albuminuria, lectrolltos lasm s Na O mE /1; 4JlmEq/l y Cl100 mEg/1, bicarbonato 25 mEQ/1, glicemla 90 m fdl, creatlnlnemla 0.8 mg/dl, calcemla 9.0 mg/dl y uricemia 5.0 mg/dl. Cul es el diagnstico ms probable?. a) Slndrome de Cushing b) Aldosteronlsmo primario ..~pertensin esencial d) Nefritis crnica e) Estenosis arterial renal bilateral

66) Cul es la frmula lctea artificial n leche Purita fortificada (LPF) recomendada para un lactante de -meses? a) LPF 5 %, azcar 2.5 %, cereal 3 % b) LPF 7.5 %, azcar 5%, aceite 5% e) LPF 7.5 %, azcar 2.5 %, cereal ~- ~ d) LPF 10%, azcar2.5 o/o,.. 3% -~ LPF 10 %, azcar 5 %, cereal 2.5 %

ereai

67) Un paciente adulto presenta esplen egalia de 12 cm bajo el reborde costal, baja de peso, leucocitos de 100.000 mm3, no presenta anemia ni trombopenla. El diagnstico ms probable es: a) leucemia mietoide crnica b) l~cemia aguda linfoblstica ..o)1eucemia aguda mieloblstica "d) mieloma mltiple e) mielodisplasia 68) Si en un programa de salud se aume ta el rendimiento del instrumento al doble, sin variar los dems parmetros: a) la cobertura del programa aumenta al do le b} el nmero de actividades aumenta al do le e) el nmero de instrumentos necesarios a menta al doble d) la concentracin de actividades disminuy a la mitad e) el nmero de instrumentos necesarios di minuye a la mitad 69) Mujer de 46 aos, hipotiroidea entra miento, consulta por compromiso del estado general y fatiga de cuatro meses de evolucin. Sin antec dentes personales ni familiares de hepatopatla. No consume drogas ni alcohol, ni ha recibido transfu iones. El examen fisico es normal. Laboratorio: transamlnasas cinco veces obre el valor normal; fQfatasas alcallnas..y_. gamm~glutamlltranspeptida~a al dobl& d i valor normal.~ Bilirrubina srica normal. Serologia negativa para virus Cle-hepatitlsey-c:----- ... - - .......... _ ~ -- -Cul es el diagnstico ms probable? a) Esteatosis heptica b) Ci~r.Q.sis biliar pmaria ._ -..._ e) Esteatohepatitis no alcohlic~ ~atitis -autoinmune-- - - e) Cirrosis criptognica 70) El agente etiolgico ms frecuente en las neumontas de pacientes con bronquitis crnica es: a) Haemophilus influenzae b) Legionella pneumophila e) Staphylococcus aureus ..Pt'Streptococcus pneumoniae e) Moraxella (Branhamella) catharralis

,.,

'11!\

(8'1

,1t'\

rfl'\

,.
~ ~

~ (!A
~
(?~!\'

(fA
(f!A
(fA.
(A'

~
(1!lh

($'\

~
('#1

<*'
tff/!!'t
~

~
Prueba 180 preguntas nmero 2 Sexto ao medicina SEGUNDA PARTE Importante:
Esta prueba ser corregida por el lector automtico. por lo que debe preocuparse por contestar adecuadamente la hoja de respuesta. Adems. aquel que sea sorprendido copiando reprobar de inmediato.

t\

e\

<"'

1) Un paciente consulta a aricin de un exantema macular, con fina descamacin~ que compromete la a ronco m ades, con compromiso de las palmas y las plantas. El diagnstico ms probable es:
a} Slndrome de pie-mano-boca b)_Eilidasis-rosaEia

d~

pow~~

~ritematoso

<R"""PseFia&is~)Sifilis

('$\

~~

te'\

2l Un paciente d~~presenta un cuadro de tos, con rinorrea y odlnofagia, de 7 dias de evolucin,


al que luego se agrega ~isnea y fiebre hata 38,5C. Se solicita radiograffa de trax que demuestra un patrn_intersticial, con engrosamiento peribronquial. El tratamiento de eleccin es:

f"'A
~

(01

a) Ceftriaxona b) Amoxicilina ~~enicilina benzatina .JaJ vlaritromicina e) Clindamicina

~ ~

~
(!~"

3) Una paciente primigesta ingresa en .,trabajo de parto, en fase activ~. Est en buenas condiciones y se palpa un fet<Jen presentacin de tronco. La dinmica uterjna es 2 CU/mjn y tiene 6 cm de dilatacin cervical, con membranas rotas. Las pruebas de bienestar fetal son normales. La conducta ms adecuada es:
a) Iniciar aceleracin occitcia b) Administrar fenoterol endovenoso c}~ealizar frceps ;(11 Realizar cesrea e) Realizar prueba de trabajo de parto

~ ~
~

~
~

~Trastorno de angustia generalizada


b) Trastorno de pnico e) Agorafobia d) Fobia social e) Trastorno obsesivo compulsivo

4) Una !l'Ujer de 45 aos es muy ansiosa y se preocupa frecuentemente de qe le pueda pasar algo malo a ella o a sus familiares. Por esto evita salir de la casa y no le gusta que sus hijos manejen o salgan por miedo a que sufran un accidente. Esto ha limitado considerablemente sus actividades. El diagnstico ms probable es:

('A

.... ,_
~

~
fl!A
~

5) Un paciente de 58 aos, diagnq~lo dWo~, evoluciona co11 fiebre tos y expectoracin y marcado .umento de su disnea. Al examen ftslco est muy taquipneico, sudoroso, febril, desorientado y P-Q!l cianosis er ora. asa uracin arterial de oxgeno, m~dida por pulsometrfa es de 80%. Cul de las siguientes medidas est MENOS ind1icada? 'i eno or mascarilla de recwanaao;para' conseguir saturaciones arteriales sobre 90%
b) Salbutamol in a a o e} Corticoides endovenosos d) Atibiticos de al)'lplio espectro, e~dovenosos v--e) Toma de gases arteriales .> ...

(lt't
~

~
~

~
~

6) Calcule la prevalencia de VIH, al FINAL del perodo, con la s;iguiente informacin: Poblacin al inicio del periodo: 1.000.000 . Poblacin al final del periodo: 1.000.000 Nmero de pacientes diagnosticados de VIH a principio del pEtriodo:~ Nmero de pacientes que contraen VIH durante el perfodo~Q. Nmero de paciente con VIH eue muereu durante el perfodo."'l-80' Porcentaje de los pacientes VIH que muer&R a causa del Vll:l!-,:909/o a) 90/5.000

b) 90 /1.000.000 e) 5.090 /1.000.000 d 5.100 /1.0 0.000 . 5.200 /1.000.000


7) Un paciente sufre un ~ideRte de bnsito, resultando con:fractura de. pelvis. sin desplazamiento. Evoluciona con imposibilidad de orinar y dolor abdominal bajo. Al examen fisico se aprecia uretrorragia y spalpa un globo vesical. La conducta MENOS adecuada es: a) Administrar suero fisiolgio endovenoso b) Administrar AINEs dlnstalar sonda Foley d) Solicitar uretrocistografa retrgada e) Realizar cistostomla 8) Cul de las siguientes vacunas puede inmunocomprometido? ~}Polio oral b) "Friurica .:..e) Virus hepatitis A d) Varicela e)'"Bea~er

administrada a un paciente severamente.

9) El examen de mayor utilidad ante la sospecha clinica de sfndrome de Cushing es: ~ ..PYtortisollibre urinario de 24 horas b) Cortisol plasmtico matinal e) Cortisol y ACTH plasmticos d) Costisol basal y post-ACTH e) ACTH plasmtica matinal y nocturna

1 O) A qu tipo de herida corresponde la amigdalectomia? a) Limpia /bfLimpia contaminada e) Contaminada d) Sucia e) Infectada

11) Una mujer d@1 consulta por aRarJcln de equimosjs, P-e1lequias generalizadas y epistaxis en 3
oportunidades. Su examen fsico no aporta mayor info acin y se soliclta.ft'exmenes generales, donde destaca un hemograma con hemoglobi 1, hematocrito: 46%, blancos; 8.400 x mm3, con frmula diferencial normal plaquetas: io.tfOO x mm~. El tratamiento de la patologfa descrita es: a) Quirrgico . b) Hidroxiurea e) ~spirlna --ci}Corticoides e) Quimioterapia combinada 12) Un nio d~a presentado durante el l~mQ..afto 3 crisis convulsivas tmco-clnici!s en relacin a ep.l!_odios febriles y 2 crisis sin desencadenante claro. Las crisis han sido autolimitadas y no muestra alteraciones en el examen flslco. la conducta ms adecuada es: ~ r a especialista, por sospecha de e ile sia b) lniciarfen1 o na e) Indicar AINEs y benzodlacepinas durante los episodios febrile~; ~Solicitar TAC de cerebro e) Observar evolucin

~ ~
13) Usted evala un paciente de 35 aos, as_intomtico, que acude a chequeo mdico. Su examen f(sico y sus exmenes de rutina son normales, sin embargo en su historia detecta que fuma entre 20 y 30 cigarrillos al d[a. Al preguntarle sobre su hbito tabquico refiere que no tiene intencin alguna de dejar de fumar. La conducta ms adecuada es: ~Explicarle que por su edad puede seguir fumando sin mayor inconveniente /'DJ Aconsejarle brevemente que deje de fumar e) Indicarle parches de nicotina d) Iniciar bupropion e) Solicitar espirometria
14) Un hombre de 6.9.!illos consulta poi' dolor en la ingle izquierda, especialmente al caminar. Al examen fsico se aprecia dolor a los movimientos extremos de la e , sin otras altel'aciones. Se solicita radiogra a e caderas que muestra disminucin del espa~io articular, con esclerosis subcondral. El diagnstico ms probable es: ft} A1trosis lb) Necrosis avascular de la cabeza femowl e) Enfermedad de Paget d) Distrofia simptico-refleja e) Cncer

!!'

!liiZ\

(iA

(fA

~
(P'

15) Un paciente de 5 aos, cursando una diarrea aguda, presenta-Oliguria, asociada a elevacin de la creatininemia y marcada disminucin de la excrecin urinaria de sodio. El diagnstico ms probable es: )lflnsuficiencia renal prerrenal b) Acidosis tubular e) Sndrome hemolitico urmico d) Necrosis tubular aguda e) Glomerulonefritis aguda 16) Un paciente de 20 aos consulta por ojo rojo bilateral, asociado a prurito ocular. Al examen fsico se aprecia ojo rojo superficial, con reacdn tal'$21 papilar. El diagnstico ms probable es: a) Conjuntivitis viral ~~ t '-V b) Conjuntivitis bacteriana y}tonjuntivitis alrgica d) Chalazin e) Blefaritis

aJ-"

f!ii'
~

~ f!!'
~

~
~
~

17) Una mujer cursando un embarazo d~t..Z semanas de amenorrea, diagnosticado mediante un test de orina, consulta por dolor abdominal, metrorragia y ortostastism~ Al examen frsico est plida, hipotensa y taquicrdica. La ecograffa transvaglnal demuestra ausencia de gestacin intrauterina, con visualizacin de un tumor anexial izquierdo de 3 cm de dimetro y presencia de lquido libre en la cavidad peritoneal. La conducta ms ad1~cuada es: a) Administrar suero fisiolgico e iniciar metotrexato ~Administrar suero fisiolgico y realizar cirugia de urgencia e) Administrar suero fisiolgico y realizar le!)rado uterino de inmediato d) Administrar suero fisiolgico y solicitar subunidad beta plasmtica e) Administrar suero fisiolgico y solicitar nueva ecografa en 48 horas 18) Un paciente presenta un cuadro de..baja de peso compatible con una malabsorcin Intestinal. confhmada con ~st de Sydn. Se realizct endoscopfa digestiva alta y se toman biopsias du'ode.n_a~es.,_. que muestran atrofia de las vellosidades, alargamientQ_!:Ie _las crip_ms~o inflLtr.adQ... li~Itado-en-1-a-mucosa intestinal. El diagnstico ms probable es:,.- .. !JY Enfermedad celiaca b) Enfermedad de CrOhn e) Colitis ulcerosa d) Giardiasis intestinal e) Sindrome de inmunodeficiencia adquirid1 (SIDA)

fP'
~ (ii'

~
~

~ ($"
~

~
(!'!t

~
~

20) Un nio de~ aos presenta claudicacin de la mamha, asociada~ y marcado decaimiento. Al examen se aprecia dolor a la movilizacin de la cadera izquierda. El diagnstico ms probable es: a) Sinovitis transitoria ~Artritis sptica e) Artritis juvenil idioptica d) Enfermedad de Perthes e) Epifislolisls de la cabeza femoral

21) Un adulto mayor de 76 aos presenta desde hace unos meses apatfa e ideas incoherentes, asociadas JnCGRtiR&Reia-udnaRl. Adems ha presentado va"""i1Ias caldas sin resultar con lesiones serias. Al examen fsico se aprecia una marcha insegura y torpeza motora. El diagnstico ms probable es: a) Demencia senil b) Demencia vascular e) Delirium d}pemencia por cuerpos de Lewy ~Hidrocefalia normotensiva

~ 22) Es causa frecuente dhr_m_e_n-r-rf;0a) Cncer de endometrio ~Hiperprolacti.nemia .,., e) Cncer de cuello uterino:.<' d) Cncer de ovario x e) Miomatosis uterina .,.
23) Una paciente de aos presenta gtorrea y otalgia recurrente en el oldo Izquierdo, de algunos meses de evolucin, asociados a.ensacin de odo tapado. A la otoscopia se observa una perforacin timpnica redonda, de dimetro jgual a un tercio del dimetro del tmpano.. El diagnstico ms probable es: a} Tuberculosis b) Miringltis e) Otitis fngica ~titis media crnica e} Otitis externa complicada 24) Un paciente de 43 aos consulta por @!fagia alta y sensacin de presin facial. Refiere que en algunas oportunidades siente prurito vparestes1as en las extremidades inferiores. Adems est en tratamiento con amitriptilina y trimebutino por fibromialgia y colon irritable. Su examen ffsico es normal y cuenta con una TAC de cavidades paranasales y con una endoscop(a digestiva alta, ambas normales. El diagnstico ms probable es: a) Sindrome paraneoplsico ><" b} Trastorno por hipocondria -xA Trastorno por somatizacin d) Trastorno por conversin e) Trastorno por disociacin

'O

e
25) Un paciente sufre una fractura de trazo, evolucionando con imposibilidad de extender la mueca. Qu estructura fue comprometida? a) Nervio msculocutneo b) Nervio circunflejo ffiervio radial - d) Nervio cubital e) Nervio mediano

t'

fA

26) Una mujer de 19 aos, cursando embarazo de tnnino, ingresa en trabajo de parto, en fase activa y el registro fetal muestra frecuencia cardiaca fetal de 105x', con variabilidad minima y presencia de desaceleraciones varjables. La conducta ms adecuada es: a) Solicitar amniocentesis ,>rnterrumpir el embarazo por la via ms expedita __... e) Solicitar perfil biofsico d) Administrar toco lticos e) Observar evolucin y dejar evolucin e:;pontnea
27) Un paciente de 60 aos consulta pcr disnea. En el examen frsico destaca abolicin del murmullo pulmonar en la base derecha, asociada a matidez y egofonfa. Se solicita radiOgrafa de trax que confirma la presencia da. un derrame pleural moderado, el que se punciona, dando salida a u~i~jdo .transparente, amarillento, con proteina:s de 1 g!dl, LDJid_e 18UIII, clul~ de 30 pos:. mm3, con~- de m~onucleares y 30% de polimorfonucleares y pH ~L diagnstico ms probable, de entre los mencronados, es:

($\

a) EmJ;Iiema
b) Tuberculosis pleural e) Cncer d) lnsufiGiencta cardiaca

f!l'
;,

(fA

~
~

e) plirrame paran e' 'mnico simple28) Un paciente de 47 aos consulta por lm otencia sexual, astenia dificultad ara ver. Al examen fsico se aprecia prida..d.el campo visual externo de ambos o os. 1diagnstico ms probable es: a) Enfermedad de Basedow graves b) Enfermedad de Cushing e) Panhipopituitarismo ~umor hipofisiario e) Apopleja hipofisiaria 29) Una e!_imigesta de ~2 semctnas presanta m~trorragia ab.unda.otfi, de co.~o_r..~oj~_~rillan!~~i~... ~tr._~~ sfnto_m~s. No presenta ~i~mi~~_!!terina y el registro fetal es tranquilizador. El diagnstico ms

(fA
~

("
~

(*1
~
~

probable es: ------------------------- - a) Embarazo molar b) Placenta previa e) Desprendimiento prematuro {fe -placenta normoinserta d) Rotura-de-vasapr-evia e) Liberacin del tapn mucoso

fit'
~
~

30) Un paciente de . 56-aos presenta un cuadro de un ~Q.Q_d_~_E!..V9.1Uc.i..n._q~_qQJQrb_c;lo.minl reQ1U:r.ente, de tipo clico, asociado a c.onsti.Q-.~J9n :~ diarrea en algunas oportunidades. Adems ha presentado deposiciones con .r:e.stos .d_~-~-ngre. Su ~xame-si.co no aporta-mayor Informacin y cuenta con exmenes generales, donde destaca la presencia de anemia ferropnica. El diagnstico ms probable es: .. ----- - ---.. -- ---a) Sndrome de intestino irritable b) Colitis infecciosa e) Enfermedad de Crohn d) Enfermedad diverticular ,e~ Cncer de colon

fl'
~

~
~
~ ~

~
~

(f!'t

~
($'

-~---

31) Un nio de 4 aos consulta por fiebre, cefalea y marcado decaimiento. Al examen ffsico destacan signos meningeos, por lo que se realiza una puncin lumbar, que da salida a un Uquido - cefalorraquide~, con 1.100 clulas por mm3, con 90o/o dE~ polimorfonucleares, glucosa de 20 mg/dl y proteinas de 70mg/dl. La tincin de Gram muestra presencia de cocces Gram positivas en diplos y en cadenas cortas. El agente etiolgico ms probable es: a) Meningococo b) Estafilococo ureo ~eumococo d) Listeria monocytogenes e) Haemophilus influenzae 32) Un paciente de ~ consulta por aparicin de una leslc)n en la cara lateral de la lengua, consistente en una placa blanquecina, de disposicin reticula1r, indolora. El diagnstico ms probable es: a) Cncer b) Leucoplaquia e) _9ndidiasis _..,..Ef~lquen plano e) Sndrome de Steven Jotinson ;33) El agente causal ms frecuente de diarrea en los lactantes es: '"<..a) Shiguella .X b) Escherichia coli ~ e) Rotavirus ~ d) Campilobacter jejuni ~ e) Enterovirus 34) Cul de las siguientes alternativas a) Hematuria ....... _/ b) Elevacin de la creatinina e) Hipertensin arterial ____.. d) Edema ~ 7>" Dislipldemia

~rtenece a(_!ffl~,;;,;;~-~~
----

)5

35) Un paciente sufre un infarto agudo al miocardio, de pared anterior, el que es manejado con tratamiento mdico y trombolisis con estreptoquinasa. Inicialmente se encuentra estable, sin embargo el tercer dia evoluciona con marcada disnea, de rpida instalacin. Al examen fisico se aprecia sudoroso, disneico, con crpitos pulmonares bilaterales y se ausculta un soplo cardiaco, holosistlico intenso, mayor en el pex e irradiado a la axila. El diagnstico ms probable es: a) Rotura de pared libre b) Reinfarto e)Jnfarto de ventriculo derecho Rotura de msculo papilar e) Aneurisma ventricular

---?

La etiologa ms frecuente de la lcera gstrica es: Consumo de AINEs b) Consumo de alcohol _el Hipersecrecjn de gastrinad) Infeccin par Helieeeacter pylori e) Autoinmune 37) Qu caracterstica esperarla encontrar en un nio de 12 meses de edad? a) Desconoce a lo~ extraos )( ~ b) Corre 5'1 e) Canta ,d{Dice frases de 2 a 3 palabras e) Patea una pelota

38) Cul es la dosis mxima de alcohol etlico que puede consumir una persona, sin mayor riesgo de desarrollar una hepatopatra crnica? , ~) gramos al dla las mujeres y 40 gramos al dia los hombres ~~ ~ gramos a a as mujeres ara "lcisho~ '? v._ h e) 100 gramos al di a las mujeres y 200 gramos al dla los om e d) 200 gramos al dla las mujeres y 400 gramos al dla los hombres e) 400 gramos al dia las mujeres y 800 gtamos al dla los hombres

'-1.

39) Un estudio evala la relacin entre la exposicin a alcohol y la aparicin de roscea, para esto seleccionan 1.340 pacientes que consumen alcohol y a 1.590 pacientes que no consumen alcohol y se
siguen en el tiempo, observando el nmero de pacientes que desarrollan roscea. Qu tipo de
e~dioes?

/a1 Caso y control ' b) De prevalencia e) Cohortes d) Ensayo clinico e) Ensayo de campo

40) Un paciente de 63 aos consulta por dolor y debilidad de las extremidades superiores, de
predominio proximal, fatigndose rpidamente al levantar los brazos y realizar actividades como peinarsQ y poner una ampolleta. El examen ffsico no aporta mayor informacin y en sus exmenes presenta elevacin de la VHS hasta 90 mm/hora, con CK normales. El diagnstico ms probable es: Polimiositis b) Lupus erite.matoso sistmico e) Polir-f:aelieYieneur:itis..aguda d) Granulematosis-de-Wgener e) Polimialgia reumtica

41) Qu tipo de demencia se caracteriza por presentar cardas frecuentes e incontinencia urinaria? a) Enfermedad de Alzheimer
b} Demencia por cuerpos de Lewy c)pemencia frontotemporal /d) Hidrocefalia normotensiva e) Enfermedad de Creutzfeld Jacob 42) Los plipos endometriales son una causa relativamente frecuente de: .a)"'Metrorragia b) Hipermenorrea e) Pollmenorrea d) Oligomenorrea e) Dismenorrea

43) Un paciente de 26 aos ha sufrido 2 trombosis venosas profundas en las extremidades inferiores y
un tromboembolismo pulmonar, manej1dos adecuadamente con anticoagulacin. Actualmente se encuentra asintomtico y se suspende ni tratamiento para realizar estudio de trombofilias, en el cual se detecta un dficit de proterna C. La e onducta ms adecuada es: ,ar'Anticoagular a permanencia con cumarl nicos orales - b) Iniciar corticoides e) Iniciar aspirina d) Administrar crioprecipitado de manera P'3ridica e) Observar evolucin
~

~ ~
~

44) Un paciente de 6 aos presenta hipertensin, hematuria y edema generalizado. El diagnstico ms probable es: ;fj Slndrome nefrftico /b) Sndrome nefrtico e) Sindrome hemolitico urmico d) Enfermedad de Berger e} Rin poliquistico

(fffr7\
~

f!%'
~

/'1\.-"1...- ::J ,,.


'~

... (,.)\-

45) Cul de las sig~lentes asociaciones entre patologfa y cuadro clfnico es~orrecta? a) Tiroiditis de Hashimoto . Bocio difuso, indoloro, con elevacin d13 la TSH _./ ..!. b) Adenoma txico- Bcio nodular con TSH elevada y exoftalmo >-:' e) Tiroiditis subaguda - Bocio difuso, de consistencia aumentada)o(lO doloroso, asociado a sintomas de hipotiroidismo _ \ d) Enfermedad de Basedow Graves -Bocio multinodular, doloroso y con frmito~ e) Cncer folicular de tiroides- Bocio nodular con abundantes adenopatias 46) Un lactante de 6 meses de edad presenta fiebre hasta 38,4 C sin otros srntomas. Su examen fsico es normal. La conducta ms adecuada es: a) Hospitalizar, realizar estudio diagnstico completo, incluyendo estudio de liquido cefalorraqufdeo e iniciar antibiticos de amplio espectro, por vfa endovenosa b) Solicitar hemograma, hemocultivo, sedimento de orina y PCR y decidir conducta segn hallazgos e) Solicitar urocultivo y sedimento de orina por puncin suprapbica d) Enviar a domicilio con paracetamol y amoxicilina y{Enviar a domicilio con paracetamol y controlar en 48 horas
11

47) Cul es el diagnstico ms probable ante un paciente con una tetraparesla arreflcti,ca, que present un cuadro diarreico hace 1O dfas? ,, a) Miastenia gravis b)~iastenia de Lambert Eaton ~ ~indrome de Guillain Barr d) Botulismo e) Mononeuritis mltiple 48) Un lactante d&CS~~s presenta palidez, por lo que se solicita h~mograma, que muestra anemia mic~ocitica, con hemoglobina de 9 g/dl. El diagnstico ms probable es: ~Anemia ferropmca b) Anemia por enfermedades crnicas e) Talasemia d) Anemia por dficit de vitamina B12 e) Microesferocitosis 49) Un paciente pos operado de una hemicolectomfa derecha presenta ollguria. Se instala sonda Foley para cuantificar, constatndose una diuresis de 1 O mi/hora. s~~ solicita creatinlna que resulta 2,4 mg/dl y BUN que resulta...50-mgldl-. La fraccin excretada de sodlQ_~; muy baja y los. electrolitos urinarios muestran Na+: SmEq/1, K+: 15mEqll y Cl-: 16mEq/l. El diagnsffco ms probable es: ~) JPfarto renal ~Insuficiencia renal aguda prerrenal e) Insuficiencia renal crnica d) Necrosis tubular aguda e) Insuficiencia renal aguda postrenal 50) Un paciente de ~s consulta por d.olor en el hl~drio derecho, asociado a vmitos alimentarios y fiebre hasta 38,5C. Sus otros signos vitales son normales y presenta Intenso dolor a la palpacin det.hipocondrio1fel:ech9, sin signos peritoneales. 1:1 hemograma demuestra leucocitosis hasta 16.500 x mm3, las pruebas hepticas y la amilasa son normales. El diagnstico ms probable es:
a) Perforaein-1:1e-til~stFica ~olecistitis aguda

e) Pancreatitis aguda d) Coledocolitiasis e) Colangitis aguda 51) Cul de las siguientes patologfas.es compatible con los gases arteriales presentados a continuacin? Pa02: 79 mmHg, PaCo2:54 mm Ha, pH: 7,3 y diferencia alveolo arterial de oxigeno: 3 mm Hg. a Tromboembola pulmonar b ferme a ne oos'r e) Neuma d)-Asma ~ Fibrosis pulmonar

~
52) Una purpera d~consulta por:. fiebre y malestar general, asociado a diagnstico ms probable es: . ~~ ~stos ovulares .
~ndometritis

metrorragi~. El

~ ~

e) Tromboflebitis sptica de las venas plvicas d) Hematoma del canal del parto e) Cuerpo extrao

-~nchQ) que se encuentra bi~ perfundido y hemodinmlcamente estable?

S~mo tratara a un paciente d~ 5G..sfi2,s con una taquicardia, a~60x', regulaiy complejo QRS
-

-------.....

a) Con digitales endovenosos

)1 Con amiodarona endovenosa e) Con adenosina endovenosa d) Con betabloqueo endovenoso e) Con cardioversin elctrica
54) Los tratamientos de eleccin para la amebiasis y giardiasi~ intestinal son respectivamente: a Metronidazo endazol y metron dazol e) Claritro.!!!l9oa-y-elafttf6mieine d) Ciprofloxacino y metronidazol e) Tinidaiory--albend~J

.....-----..

azor

.:._____../

('A

~
~

~
~

55) Un paciente de 60 aos, indigente, alcohlico, sufre un atropello en la va pblica. Qu medida es especialmente importante considerar a la hora de administrar suero glucosado o alimentar a este paciente? a) Administrar insulina b) Administrar vitamina 81 ~dministrar vitamina 812 d) Administrar metano! e) Administrar benzodiacepinas 56) Un paciente de 63 aos, con insuficiencia cardaca, secundaria a un infarto al miocardio, que sufri hace 2 aos. Est en tratamiento con E!!lalapril.-carvediibl, furosemida, aspirina= y atorvastatina, permaneciendo en capacidad funcional 11. Es trafdo, por un cuadro de aum~o~9..m.ar~-.do.~~.~~. ~j_n~..a. asociado a confusin, de 1 horas de evoluci.Qn._AI examen se ob9-t:.V.a__.P-lido, con livideces en las extremidades, en sopor superficial, coru ~150~, irregular, PA(74/32 ~9z FR: 27x' y saturacin arterial de oxigeno d 77%, por pulsometrfa. La auscultacin cardaca demuestra un ritmo irreaular en 3 tiempos, por presencia de un tercer ruido y la auscultacin pulmonar presenta crpitos difusos bilaterales. El monitoreo cardiaco demuestra una taquicardia irregular:, a QRS angosto a 150x'La conducta ms adecuada es: a) Solicitar electrocardiograma de 12 derivaciones y decidir manejo segn hallazgos r:f A b) Administrar amiodarona endovenosa e instalar una bomba de dopamina endovenosa e) Administrar suero fisiolgico y adenosina d) ~dministrar heparina endovenosa y realizar cardioversin farmacolgica con amiodarona ~'Kealizar cardioversin elctrica de inmediato 57) Una mujer de 30 aos, cursando su cuarto embarazo, de .~nas_de gestacin por FUR, con hipermesis gravdica, presenta metronagla y dolor hipogstrico. Al examen ffsico se palpa el tero a la-altfadil pu6s y el tacto v~a!!t~!nornuestra ca_f!J_bio~iViciles. El diagnstico ms probable es: a) Sntomas de aborto - - --------b) Aborto en evolucin e) Embarazo ectpico roto ,...d) Embarazo molar e) Abortcnetenrdo

~
~

~
~

~
~ ~ ~ ~

~
~

~
~
~
~

f!'
~

~
~

~ ~

58) Un paciente de 87 aos sufre un accidente vascular lsqumico, resultando con una hemiplejia que le impide la marcha y con Imposibilidad de hablar. Al quinto df;a de evolucin se observa eritema en la zona sacra. La conducta ms adecuada es: a) Iniciar antibiticos endovenosos b) 1 iciar anticoagulacin con heparina segurar cambios frecuentes de posicin d) Cubrir la zona con apsito e) Resolver quirrgicamente 59) Un recin nacido de 29 semanas de edad gestaclonal presenta dificultad respiratoria progresiva, asociada a desaturacin arterial de oxigeno hasta 80o/o. El diagnstico ms probable es: a) Gardiopatfa congnita b) Taquipnea transitoria e) Hipertensin pulmonar persitente d) Sndrome de aspiracin meconial ~nfermedad de membrana hialina

. 60) Una paciente de !ff~is consulta por astenia, mlal las y artralglas de manos ~hombros. Al
examen fsico se observan 2 lceras orales y se realiz un emograma, en el que estaca trombocitopenia y un anlisis de orina completa en el que se aprecia proteinua +++. El diagnstico ms probable es: a) Dermatomiositis b) Enfermedad de B.ehcet e) Panarteritis nodosa d) j3elviespondiloartropatfa /.-e) Lupus eritematoso sistmico 61) Qu alimentacin indlcarfa a un nio de 4 meses de edad cuya madre ya n~.~~~~--~!ITI~.n~~Lo con leche materna? -~Leche purita al 7 ,5%, ms azcar al 2,5%, ms aceite al1,5% b) Leche purita al7,5%, ms azcar ai2;S%,-ms-ser.eal-a~ e) Leche purita al 7 ,5%, ms azcar al ~07 ms eereal-al-+,5% d) Leche purita al1 0%, ms azcar al 2,5%, ms aceite al 1,5% e) Leche purita al10%, ms azcar al5o/o;-mseerea~% 62) Un pacie,nte de 38 aos es diagnosticado de hipertensin arterial esencial, grado 1. Su examen fsico no presenta alteraciones de importancia y adems su etiectrocardlograma y exmenes generales son normales. Se Inicia dieta hiposdlca y ejercicios, sin embargo persiste con presiones arteriales cercanas a 150/90 mmHg, luego de 3 meses. La conducta ms adecuada es: a) Mantener las indicaciones y controlar en 3 meses b) Aumentar el programa de ejercicios y controlar en 3 meses e) Jpiciar un bloqueador beta-adrenrgico ,.A} Iniciar hidroclorotiazida e) Solicitar ecografia-Doppler de arterias renales. metanefrinas urinarias e fndice de aldosterona sobre actividad de renina plasmtica 63) Un paciente diabtico tipo 2 en tratamiento con metformina 850 m ada 12 horas y glibenclamida 10 mg cada 12 horas presenta gliGmlas de ayuno cercanas a 40 m /d glicemias postprandiales cercanas a 200 mq/dl. Se solicita hemoglobina gllcosilada qu~ resulta , La conducta ms adecuada a) ManteerlratafflieRtab) Aumentar la dosis de gUbetrerail'da e) Aumentar la dosis de metformina dY,niciar-ateFVaStatir.Ja miciar insulina

es:

?J

~
64) Una mujer de 23 aos consulta por leucorrea grumosa y blanquecina, sin mal olor, asociada a prurito vaginal y signos inflamatorios moderados de la pared vaginal. El tratamiento de la patologia d~crita es:
~ ~.luconazol

b) Metronidazol e) Clindamicina d) Corticoides locales e) Doxiciclina


65) Una paciente vio como un perro de Ja calle mat a un gato de una manera muy cruda. Tres meses

~
~

~
~

despus inicia pesadillas y recuerdos vivos del Incidente, lo que le trae mucha angustia. e hecho ha presentado crisis de ansiedad en algunas oportunidades. Por eso evita cualquier contacto con perros o gatos. Refiere sentirse cansada y le cuesta expresar sus emociones. El diagnstico ms probable es ,;;{Trastorno de estrs postraumtico ( b) Trastorno adaptativo e) Depresin psictica d) Trastorno de angustia e} Fobia especifica
66) Una paciente de 28 aos, cursando :~u segundo embarazo, de 35 semanas, consulta por compromiso del estado general, dolor eRig_strico e 1cterlela. Al.examen f1saco se constata FC: 100x'. PA: 150/100 mmHg e ictericia de piel y mucosas. En los exmenes generales se demuestra anemia, trombocitopenia y elevaci~UimtbiA~ las transamlnasas. El diagnstico ms probable es: a) Hepatitis aguda del embarazo ~lgado graso agudo del embarazo ~ ~-'-'PJ Slndrome de HELLP d) Colestasia intraheptica del embarazo e} Coroamnionitis

~
~

'f' .

~
~

~ ~

~
~

~
~ ~
~

67) Un paciente sufre una calda de altur.i, resultando con una fractura de pelvis desplazada. Evoluciona con ortostatismo, taquicardia y tendencia a la hipotensin. Adems de administrar cristaloides endovenosos, la conducta inicial ms adecuada es: a} Transfundir glbulos rojos b) Administrar cido tranexnico ;(Estabilizar la fractura mediante frulas n1~umticas o tutor externo d) Realizar ligadura de las venas plvicas e) Realizar reduccin quirrgica y osteosintesis con placa y tornillos
68) Qu inductor se usa en p_acientes con tendencia a la hipotensin?

a). Tiopental tJ Ketamina ~) Fentariil d) Propofol e) Vecuronio

f!'

<!'
~

~
(!A
~
~

69) Un lactante de 9 meses ya tiene sus 4 Incisivos afuera. Qu medidas de higiene dental debe aconsejar a sus padres? a} Ninguna b) Aseo con un ao hmedo ava o de dientes con pasta dental para nios d) Lavado de dientes con pasta dental convencional e) Enjuague ~~~~l. luego de cada comida 70) Una paciente de 65 aos se realiza una ecogratra abdominal, como parte del estudio de un cuadro de dolor abdominal, detectndose incidentalmente un tumor renal slido, de 5 cm de dimetro. El diagnstico ms probable es: ~ a) Adenoma b) Lipoma ~rcncer renal d) Hidatidosis e) Hemangioma

~
(!"

~ ~

!")
71) Cul es la causa ms frecuente de constipacin en la edad peditrica? a) Bajo consumo de fibra ~aJos hbitos de defecacin e) enfermedad de Hirsprung d) Fibrosis qustica e) Giardiasis 72) Una paciente de 43 aos consulta por astenia. El examen fisic~ no aporta mayor informacin, por lo que decide tomar exmenes generales, que demuestran un hemograma normal, pruebas de funcin renal normales y pruebas hepticas con elevacin de las f~tasas alcaliAas basta 350 1un, hili~Nbina de 2,3 mg/dl, de predominio directo y normalidad de las transe~minasas. El diagnstico de sospecha
~:

tlltl)
~

~
~

~
~
~

a) Hepatitis autoinmune b) Hepatitis viral

----

p Cirrosis biliar primaria


e) Hepatocarcinoma

te.:osaote pmaria

~
~

73) Respecto al s[ndrome de transfusin feto-fetal es FALSO cue: a) pe no ser tratado tjene una !etalidad cercana al 25% b) Se dia-gnostica mediante ecograffa --e) Se caracteriza por oligohldramnios en un gemelo y polihidramnios en el otro ,d}"Se presenta en los embarazos monocoriales-biamniticos e) Se trata amnioscpicamente. mediante coagulacin selectiva de! los vasos comunicantes 74) Un hombre de 67 aos consulta por fatigabilidad y astenia. Al examen fsico destaca palidez de piel y mucosas, por lo que se solicita un hemograma que muestra hematocrito: 28%, hemoglobina: 9,5 g/dl, VCM: 72ft, HCM: 30 pg, blancos: 5.700 x mm3,1infocltos: 40%, neutrfilos: 70%, monocitos: 8%, eosinfilos: 2% y plaquetas: 370.000. El diagnstico ms probable es: mia ferropnica ) mla de enfermedades crnicas e) Anemia hemolltica d) Mielodisplasia e) Mieloma 75) Un nio de 9 aos miente y roba a sus padres con frecuencia, se mete en peleas seguido, es cruel con los animales y presenta una actitud irrespetuosa e indiferente hacia sus padres y profesores. El diagnstico ms probable es: a) Depresin infantil b~lrastomo por ansiedad de separacin rq Trastorno de conducta d) Trastorno oposicionista desafiante e) Sfndrome de Asperger 76) Qu institucin es responsable de administrar los dineros asignados. a la salud pblica? a) Servicio de salud b) Municipalidad e) FONASA ~t1:1te-de satrrd publica e) Central de abastecimiento 77) Un paciente hospitalizado por un tromboembolismo pulmonar, con buena evolucin, presenta sodio plasmtico de 131 mEq/1. No tiene sfntomas neurolglcos y su examen fisico no aporta informacin adicional. Usted sospecha un s[ndrome de secrecin inadecuada de hormona antidiurtica. La conducta ms adecuada es: a) Administrar vasopresina b) Administrar una tiazida e) Instalar un catter venoso centrar e iniciar suero hipertnico por esa vla d_Y.\dministrar suero glucosado ~) mdicar restriccin de la ingesta de agua libre

,._,
~
~

~
~

~
~

~
~

..,
~
~

..,
~

~
~

..,
~

...,
~
~

~
~

"
78) El tratamiento de eleccin de la uretritis gonoccica es: a)i51arttromieina, pero suele tratarse adems con tetraciclinas para cubrir Chlamydia b) ~indamlclua, pero suele tratarse adems con tetraciclinas para cubrir Chlamydia c):Penicttina, pero suele tratarse adems c:on tetraciclinas para cubrir Chfamydia _.el) Ceftriaxona, pero suele tratarse ademn con tetraciclinas para cubrir Chlamydia ~Gentamicina, pero suele tratarse adem{ts con tetraciclinas para cubrir Chfamydia

~ (!'

~ ~

~
~

~
~

79} Una mujer de 35 aos consulta por dolor abdomial bajo, ciclico, que se presenta cada 30 dias y dura 3 dias. El diagnstico ms probabl!e es: a} Sindrome de intestino irritable b) Cistitis intersticial e) lcera gastroduodenal d~roceso inflamatorio plvico t> Dismenorrea 80) Un nio de 5 aos, con sndrome de dficit atencional hiperactivo, es trafdo por su madre, ya que presenta una voz ronca. Refiere que grilta mucho y frecuentemente queda disfnico. El diagnstico ms probable es: a) Edema de Reinke b) Parlisis del nervio faringeo recurrente e) Laringomafacia .EJYNdulos vocales e) Papilomas laringeos
81) Cul de las siguientes recomenda e iones NO daria a un paciente obeso? a) Realizar deporte aerbico b) Consumir pocas grasas e) Reducir fa ingesta de hidratos de carbono ,.,.et(Consumir menos de 1000 caloras diaric1s hasta normalizar su peso e) Realizarse un perfillipidico y glicemia basal 82) Un paciente de 3 aos presenta fiebl'e, decaimiento y tos intensa. Evoluciona luego con dificultad respiratoria y estridor inspiratorio. La auscultacin pulmonar es normal. El agente causal ms probable es: a)Yirus respiratorio sincicial .a'}Virus parainfluenza e) Mycoplasma d) Neumococo e) Enterovirus

~ ~
('A

~
~

(%!'
~

~
~

~ ~
~ ~
~

~
~
~
(FA

83) Un paciente de 50 aos consulta por disnea de 4 meses de evolucin, progresiva, asociada a tos seca. Fuma cerca de 5 cigarrillos diarios;, desde. los 20 aos de edad. Al examen fsico se observa hipocratismo digital y se auscultan camoos pulmonares pequeos, con abundantes crpitos bilaterales. La auscultacin cardfaca es normal. El diagnstico ms probable es: a) EPOC b) Asma e) Cncer pulmonar ~ibrosis pulmonar e) Tuberculosis pulmonar
84} Cul de las siguientes es una medida de p~ln secundari~? ;{)Tornar aspirina para prevenir un infarto '1- ~ "' lb) Tornar aspirina lueg.o...dellaber teoido-tm1nfarto ~. . -....... , 0 e) Realizar kinesioterapia a un nio con parlisis cerebral ~ ~ d) Vacunar Cb.n'lra-l~nfu:teAra-a-paeiefltes-cle-riesgo Controlar la presirrarterlal-a-pa~ieAt~.,:~s

-t\

~
~
~
~ ~

e}

1J

~
~

~ ~

~electasia e) Neumotrax

85) Un paciente se realiza una gastrectomra parcial, debido a un sfndrome pilrico secundario a una lcera gstrica cicatrizada. Al segundo dfa posquirrgico presenta disnea de inicio sbito, asociado a tos con expectoracin hemoptoica. Al examen fsico presenta FC: 92x', T 0 : 37,8C, PA: 12ono mmHg, FR: 16x' y se ausculta el murmullo pulmonar, con algunas crepitaciones bilaterales, mayores a derecha. El diagnstico ms probable es: a) Neumonla
-~---=o"-'-mboembollsmo

e} BronquEte~ISh!

t--------

pulmonar

86) Qu manifestacin clin ucir un accidente vascular enceflico isqumico, por oclusin de la arteria -ce.lll_I:Jfilposterior izquierda

a) Hemipafe&taWfaoiobfafltJial de1ecla y henlihipoestesia derecha


b) Paraparesia y parlisis oculomotora derecha ,.4Ftemianopsia homnima derecha y Ra~.:aLderechad) Hemihipoestesia Ctefech1fYl>arliis-oCUfomotora izquierda e) HemiparesiaJzquie:cla y '=Ompmmis9 de concjencia 87) El factor pronstico ms importante en el cncer de maml es: a) Tamao del tumor .4compromiso de ganglios linfticos e) Presencia de receptores hormonales d) Invasin maligna vascular e) Grado de diferenciacin histolgica
88) Un paciente de 40 aos consulta por dolor ocular Intenso, que aumenta considerablemente con la luz, lo que le dificultad la apertura, asociado a epifora y sin allteraciones de la agudeza visual. Al examen ffsico se aprecia ojo rojo periquerjlico y blefarospasi(Tlo. El diagnstico ms probable es: Glaucoma agudo ):)) Queratitis aguda Uveltis d) Escleritis
e)Cenj~

t9. . e)

89) Respecto a la violencia intrafamiliar es VERDADERO que: a) Son raros los casos de violencia psicolgica .-5 . , b) Es muy infrecuente en los estratos socioecC?nmicos altos...., / crl=l nivel de violencia suele aumentar progresivamente a lo largo del tiempo /cl) Son raros los casos que no se denuncian ""' e) Los agresores caracteristicamente tienen una alta autoestimax 90) Un paciente obeso, sin hbitos txicos, con hepatitis cr~Jlica, de-causa.desconoci~ estudio serolgico para virus hepatotrpicos, presentando ~ntigeng de superficie pa~sitiV.i anticuerpos anti-VHBs negativos, anticuerpos lgM anti-core VHB neaatjyos, antlcuerposg~ __ negativos, anticuerpos lgG anti VHA positivos, anticuerpos anti-VHC negativos. El diagnstico ms probable es: a}.t!epatitis A 15) Hepatitis 8 e) Hepatitis C d) Esteatohepatltis no alcohlica _ r t\ ~ e) Hepatltlsit\-st>l:>r-e-he~ '-.l

\\(?

~
Prueba 180 preguntas nmero 2 Sexto ao medicina PRIMERA PARTE Importante: Esta prueba ser corregida por el lector automtico. por lo que debe preocuparse por contestar adecuadamente la hoja de respuesta. Adems. aquel que sea sororendido copiando reprobar de inmediato.
1) Una p.a.~te de .19 aos presenta un cuadro de 4nimo baio,canhedonja y falta de motivacin, desde

1!'

~ ~

~
hacELmeses), momento en que obtuvo un meno~ :;~~e en la PSU de lo que esperaba. Desde entonces ha aum...entado 6 Kg de peso y dueiie; 1 bpras l i:ffa, cuando antes sola dormir 7 horas. No presenta Ideacin suicida. E:l diagnstico ms probable es: a) Depresin

~
~

~
'l

~
f!A

b> eistiFAia
e) Gidoth 11ief'

~ ~ ~
~

d) Trastorno adaptativo e) H.iQ_otiroidismo

,\\

~
~

2) Un nio de 2jlmeses de edad presenta fiebre e irritabilidad. En el examen fisico se aprecia ~a.con timpno derecho aboml2ad'"()"Veritematoso. La conducta ms adecuada es: a) Derivar a especialista b} Solicitar impedanciometria e) !!Jiciar un macrlido . ..d-}lniciar un AINE y amoxicilina e) Iniciar un AINE 3) Un paciente de~s, fumador e tp.er!enso, inicia dolor precordial, opresivo, intenso, asociado a disnea y sudoracin. Al e:xamen fsico est taguicrdico a 105x', con pres1n arterial de~mmHg, seausculta un ritmo regular en 3 tonQ.U por presencia de un cuarto ruiqo. Se toma electrocardiograma que demuestra el~vacin del segmento ST en V1, V2. VTy V4, coninversin de la onda..J en todas las derivadas precordiales. Cul de las siguientes alternativa le parece ms correcta, para el manejo de _s!) este paciente? __ ayAdministrar aspirina, oxigeno y nitroglic:erina e iniciar el protocolo de trombolisls endovenosa ' b) Administrar aspirina, morfina, oxigeno~~ nitroglicerina y anticoagular con heparina endovenosa, ajustando la dosis por TIPA e) Iniciar AINE6-0f.ales-eA-dgsis..alta~:twlaJ:.Gc:u~-eseGBFEiiegrafia d) Solicitar ecocardiografla trallSesef~t-de 01 gencla e} Administrar aspiriAa, oxigeno y nitroglieeFi~lieitar-test-de-esftrerzo. 4) Una paciente de 29 seman'as de gestacin presenta salida de abundante lquido cJam por los genitales, confirmdose una ReQ_por medio del test de cristalizacin. No s~an modificaciones cervicales .Y el registro basal no estresante es tranquilizador. L-._~pnducta ms..a.decuada..EUi: .- < ....a) Administrar corticoides y antibiticos ara madre y mantener una conducta expectante ..... ~ '-'._: b) Administrar corticoides y tocolfticos ... .,~. e) Administrar corticoides antibiticos y tocoliticos e interrumpir el el embarazo en 48 horas d) lndm:ifet parto con mlsuprostol e) Realizar-Gesr.ea. 5) La enfermedad de Basedow Graves EIS la causa ms frecuente de hipertiroldismo. En su cuadro clnico presenta todo lo siguiente, EXCI:PTO: ..a}Macroglosia b) Retraccin palpebral__.. e) Exoftalmo- d) Temblor e} Intolerancia al calor

~
~

(!'
~

,s \

~
~

~
~ ~

~
~

[;_..\. )

(!'
~

~ ~
~ ~

~ ~
~

~
~

~
~
~
~

La rinitis hormonal es caracterstica de: Hipotiroidismo Hipertiroidismo Slndrome de Cushing d~enopausia ,t) Embarazo 7) Un paciente presenta un a~?ma persistente leve, con tos en relacin al ejercicio y a veces al exponerse al polvo. pero en general sus ~.futomas son poco frecuentes. Su examen fsico es normal, sin embargo el test de metacolina demostr hiperreactividad bronquial. El tratamiento de eleccin es: a) Salbutamol inhalado, segn horario b) Salbutamol inhalado, segn sfntomas e) Corticoides inhalados, segn slntomas . , ACorticoides inhalados, segn horario y salbutamol inhalado, segn sintomas e) Coticoides inhalados, segn sintomas y salbutamol inhalado, s1~gn horario 8) El mejor indicador de la importancia de una enfermedad inFecciosa de corta duracin es: a) Mortalidad b)j...etalidad ,e:flneidencia d) Prevalencia e) Tasa de ataque fiebre ocasional dsde hace 2 meses. En el ltimo tiempo la diarrea se ha -~nguinolenta. Al examen fsico se aprecia ~~ con un abdomen blando, depr~~jblf!, sensible de manera .difus.o,_con ruidos hidroareos presentes y sin signos-de irritacin peritoneal. El diagnstico ms probable es: -a) Enfermedad celiaca b) Diarrea viral e) j)olitis bacteriana A Colitis ulcerosa e) Parasitosis intestinal
1O) Cul es la conducta ms adecuada frente a un lactante diagnosticado de cuyo testculo se encuentra en el conducto inguinal? - a) Observar hastaflos 4aos de VIda b) Solicitar a!fafetoportefna y gol=ladotrofina corinica de manera peridica 9} 1 niciar testosterorn:rsobctltQel /d) Realizar orquiectomia inmediatamente e Realizar descenso testicular y orguidopexia dyrante el segundo ao de vida
9) Un paciente de 25 afips presenta.diauea-aswosarpuJ.o-.!t-temwn..o, asociadas a dolor abd.omlnal~

6) a) b) e)

""
~

~
~
~

~ ~
~

,._,
~
~
~

~ ~

~
~
~

~~

~
~

~
~ ~
~
~

11) Una purpera de 2 das consulta por aumento de volumen y dolor en relacin a la zona axilar_ anterior. Se observa una tumoracin de 5 cm de dimetro, dolorosa a la palpacin con escaso eritema de la piel qe la recubre. El diagnstico ms probable es: a) Mastitis abscedada b) Adenitis supurada e) Hidrosadenitis fi) Glndula mamaria accesoria e) Mastitis linfangitica 12) Un paci~nte de 30 ans.consulta por sncopeS-de-esfueri!e. Al examen fsico se pal~a un ~ulso fUDplio, bjfsjco y se ausculta un soplo cardiaco eyeGt.iv.o, que aumenta con la majOb de v:lsaha El choque de la punta se palpa en la lnea medio-clavicular, en eL"u.rtQ.Q$..P.liD_o intercostal. La auscultacin pulmonar es ngr:.,mal. El diagnstico ms probable es: a) Estenosis artica b) Insuficiencia artica e) 91ncopes vasovagales -d}Miocardiopatla hipertrfica e) Miocardiopatfa dilatada

~
~

~
~

~
~
~

~
~

~
~

~
13) Un lactante dQ:-2-meses, alimen~ exclusivamente con leche materna, pre~_!~.5!~po~!~!~

~
y ele color amarillo clro. La conducta ms adecuada es: a) !melar lactulosa b) Selieitar-coprocult!WS "':} ~ ~ li. ~ t ~ J 11) .O , e) Agr-egarfrmolaictea-y-&~eRc:ler-la~aGtat=~cia..matema \\' v- f "' ~~eilc' de sudn
~bservar ':J
14) Un paciente der~, sin antecedentes de importancia presenta anemia con hematncrita de ;m%, hemoglobina de 10,2 g/dl y VCOO el~.~ Se solicita perfil de fierro 9!!!~.e.str.alru:r.mnia bajs: c=7~' CQJl-<traflSfermrdjsrrifijl([a y=ferr-it~a olevada. El diagnstico ms probable es: a) Anemia por enfermedades crn~ ~nem1a ferropel'ficcr~ ~ e) Talasemia d) Anemia hemoltica e) Anemia por dficit de vitamina 812
15) Una paciente de,23-aes, presenta un at-:aso menstrual de 7 semanas, por lo que se realiza una prueba de embarazo que resulta positiva. Se realiza ecograf[~ transvaginal que no visualiza gestacin ~traute. ri.na. La conducta ms adecuada es:
~~~ Rea!i~~I IJua'lla..eGGQr.afia-en-48-horas_ _

~~as, de .consistencia blanda

r'

~
~

~ ~

e
~

~
~

(1;
(!"

b) -Soiicitar..ubunidad beta-HCG plasmti(:i,) e) Realizar legrado d) Iniciar metotrexato e) Resolver quirrgicamente 16) Un paciente ~IH positivo, sin_!rller.a,oia, con niveles bajos de linfocitos CD4 presenta cefalea y d!ficultad para mver la extremidad infE!rior izquierda. En una instancia posterior sufre una convulsh?_n.. tnicoclnica. Al examen ffsico se cons;tata paresia de la extremidad inferior izquierda y en menor grado de la extremidad superior izquierda. El diagnstico ms probable es: a) Linfoma cerebral b) Meningitis por critococo e) ~eucomalacia multifocal progresiva ,.d) Toxoplasmosis cerebral e) Encefalitis por citomegalovirus

~
~

~ ~

~
~

('A

~
~

~
~

17) Un paciente de ~s es em~~llmente inestable e impdslvo. Abusa del alcohol y consume drogas de manera ocasional. Adems presenta relaciones familiares y amorosas inestables e intensas, ya qu se molesta fcilmente y reacciona de manera agresiva. Tiene clca~de...c.o.r.tes..en..sus.brazos y fue hospitalizado en una oportunidad por intento de suicidio por ingesta de medicamentos. El diagnstico ms probable es: a) Depresin psictica b) Trastorno de adaptacin e) Trastorno bipolar ..}Yfrastomo de personalidad e) Depresin atpica
18) Un paciente fumador de 37 paquetes~o, consulta por disnea de medianos esfuerzos, asociado a tos pcasional. corieiij)eeloracin. Al exHmen fsico se aprecia aumento del dimetro torcico, espiracin ptQ!Qngrula y se ausculta un murrouUo pulmonat=-disrnJnuido g{Qb!_m...Jmte, concampos pulmonares grandes y algunas sibilancias bilaterales. Qu examen es el ms adecuado para confirmar la sospecha diagnstica de e~;te paciente? a) Radiografa de trax b) TAC de trax e) Biopsia pulmonar d) Test de metacolina ,e)Espirometra

~ (f'
~
~

~
~

~
~
~

~ ~

~ ~
~

~
19) Una paciente, multf ara de cursando un e"!barazo de 40 semanas inicia dinjmlca uterina persistente de 4 e por minuto y al tacto vaginal se constata...borramlento cervical completo y . . dilataciGR-de+cm; con membranas ntegras y feto en presentacin ceflica. Sus ltimos 2 partos fueron mediante cesFea. La conducta ms adecuada es: a) Enviar a domicilio e indicar reconsulta en 6 horas ms b) Hospitalizar en preparto y dejar evolucin espontnea e) Realizar rotura de membranas ealizar cesrea e) Iniciar aceleracin occitcica 20))..a presencia de cjUndros hemticos en el sedimento de orina es caracterstica de: )lfGiomerulonefritis -. b) Nefropatia diabtrca e) Pielonefritis.aguda d) Urolitiasis e) Necrosis tubular aguda 21) Un paciente sufre una fractura de fmur, la que es manejada de manera quirrgica. Evoluciona con dificultad respiratoria y compromiso de conciencia. Al examen fsico est en sopor profundo, co~. desaturacin arterial deoxgeiiafcactyseot,servan p~teqyJas. generalizadas.en el tronc.o ..y.Jas extremidades. El diagnstico ms probable es: a) Tromboembolismo pulmonar b) Distrofia simptico refleja e) Prpura trombocitopnico trombtico d) l3eaccin transfusional aguda ?)'t:mbolia grasa __.----~------~

~
~

~
~

~
~

~
~

~ ~

~ ~
~

~
~

23! Cul de las siguientes alternativas


~~ ~isuria

N~ compatible C~l)_(nuresis primad~


' ..
lrt_

L--

.c-R...

b) Predominio nocturno e) Seis aos de edad d) Antecedentes familiares e) Osmolaridad urinaria normal

\~

~
~

~
~

23) Un paciente de:.ZJUios presenta temblor de ambas extremidades superiores, maY.or a L-.d.er.e1Cha. Al examen fisico se aprecia tem lor postura! de las e des s perjores y ligero temblor de la cabeza. La marcha es normal, ~1 gual que los reflejos osteotendneos. El rJlig~Q .m:u:unbab~: a) Enfermedad de Parkinson b) Sindrome cerebeloso e) Corea de Huntington , d) Temblor esencial . e) Esclerosis lateral ariliotrfica 24) Una mujer de 30 aos, asintomtica, se realiza una ecogrda transvaginal como parte de un che~ ecolgico que demuestra untumor anexial derecho de 3 cm de dimetro y aspecto srfdoaustico Refiere tener ciclos regulares y ~u ultima men-struacin comenz hace 20 das. El diagnstico ms probable es: a) Cncer de ovario b) Teratoma maligno e) Teratoma quistico ..d(Cuerpo lteo e) Torsin ovrica 25) Un paciente de 30 aos consulta por astenia y debilidad de varias semanas de evohacln. Al examen fsico se aprecian mculas hiperpigmentadas en la Cilm.lt.Jtn la mucosa oral. Se realizan exmenes generales, enios que destaca una l!gera hiperkalemia. El diagnstico ms probable es: a) Enfermedad de Cushing ~)Insuficiencia suprarrenal primaria e) Hipopituitarismo d) Hipotiroidismo e) Hiperprolactinemia

~
~
~

~
~

~
~
~
~

~
~
~

~
~

~
~

.,
~

~
38) Un paciente de con expectora..cln.a.b.YJJ_Q_nte, especialmente en las maanas, de tipo mucosa, pero que en algunas ocasiones se exacerba, volvindose mucopurulenta, respondiendo al uso de antibiticos automedicados .. Se realiza una radiograffa de trax que demuestra algunas imgenes de engrosamiento br<?~9~~1, que podrian corresponder a br_Qo.qyietaslas difusas. La conducta ms adecuada es: ----- -- - ~a) Indicar antibiticos orales de amplio espectro de manera peridica b) Iniciar broncodilatadores inhalados, asociados a corticoides orales c).glicitar tomografia axial computada de~ trax, con cortes finos ..~fRealizar broncoscopfa e) Resolver quirrgicamente 39) Cul de las siguientes pacientes puede utilizar anti~Q!lge~tivos orales sin mayor contraindicacin? ----------a) Mujer con metrorragia en estudio -< b) Mujer con sfndrome antifosfolfpidos '< '>< e) Mujer con marcados antecedentes familiares de cncer de mama d) Mujer con hepatitis B activa .-('eyMujer con sfndrome de ovario poliqustico 40) Un paciente hospitalizado.por un ac:cid~llt.Eula$cular enceflico reciente, con serias dificultades para tragar, inicia cuadro de tos con expectoracin y fiebre b.a..sta 38,7C, con saturacin de oxigeno de 92% a Fi02 ambiental. Al examen fsico se aU~IDJitan.cr.epitaciGRes y mltiples estertore~J~OJ~ ..Q-.!!~ derecha, por lo que se solicita radiografa de trax qu~_demuestr.a-una.condensairi:e.el.lbulo inferior derecho. El tratamiento antibitico ms adecuado es: a) Vancomicina ...... ...... -----b) Cefotaximo ms claritromicina e) Cefazolina ms clindamicina ;J(cettriaxona ms metronidazol e) Amoxicilina ms cido clavulnico 41) Un paciente pr~~ntadisminuciruilf!~~LPllidad deJ.as..manos y los JliU, especialmente la sensibilidad vibratoria, la de temperatura y la de dolor. El diagnstico ms probable es: a) Mononeuritis mltiple
~Polineuropatia

"
~

~os presenta~s

~
~ ~

~
~

f'
~

f:'
~
~

~
~

~ ~

~
~

~ ~
~
~
~

e) Esclerosis lateral amiotrfica d) Sfndrome de Guillain Barr e) Sindrome piramidal 42) Un paciente de 22 aos, previament~ sano, pres(tota..s.bJtame_nt~_doJ.OLtorcjco izquierdo, con tope inspiratorio_y dis_IJ!!L_alc.aminar_2_c;.Ui!Was. Al examen fsico se aprecia una disminucin del mI!!J_~~J~Jmonar Izquierdo, con h~~~dJl.l~LP-~I..Sln. La conducta mas adecuada es: '~ . a) Solicitar radiograffa de trax b) Solicitar angio-TAC de trax / e) ~olicitar dmero O plasmtico . ~-s .~ d) Puncionar el segundo espacio intercostal en la linea media clavicular ~" ~ e) Resolver quirrgicamente de inmediato

r--

~ ~ ~
~ ~ ~
~

43) Un paciente de 30..afiru; tiene la s~nsacin de ser invadido por grmenes al tocar_c.ualq.uie.r..tip_Q_qe_ superficie, razn por la que 1.~~~- ~r:_e~_!.!entemente sus manos y usa_!!~~ol g_eLrn.uchas.Ye.ce.s..al dfa, lo que le ha significado problemas dermatolgicos y 18-~I:J~~.-~.~.~us ~-r~. Sabe que estas ideas son ridculas y que provienen de su propia mente, pero no sabe cmo controlarlas. Por esto su autoestima ha bajado considerablemente e incluso !ha pensado en suicidarse. Cul es el diagnstico ms probable? a} Trastorno por hipocondra b) Depresin psictica e) .J.rastorno delirante crnico Jl)rastorno obsesivo compulsivo e) Trastorno de personalidad

~
~
~

~
~

~
~
~
44) Una mujer de 48 aos consulta por cu_adros de minutos d1a duracin d!.~r en la cara y el J?!.ch~ que inician de manera brusca y que son ms frecuentes e mti~nsos en las noches. Qu tratamiento es ------- - -- - -- --- -- .J el m"s adecuado para el cuadro descrito? a) Antidepresivos b) Levotiroxina e) Cortisol _}i}'strgenos e) Benzodiaceplnas 45) Una paciente de ~aos consulta por, dolor abdominal de )larjos meses de evolucin, asociado a distensin abdominal y constipacin. El dolor suele aumentar con algunos alimentos uede con la evacuacin de gases o de deposiciooes. Refiere quee"aigu1iis.oportnldades presenta mucosidad en las heces. El examen fsico no ~~Q.tta. mayor..infarmUili)n. El diagn"stico ms probable es: a) Colitis ulcerosa b) Giardiasis _~ndrome de intestino irritable ~ Malabsorcin intestinal e) Cncer de colon 46) Un paciente de 70 aos desde hace algn tiempo sufr~nlor precordial, al caminatl_cuac!rils. el que cede luego de 3 minutos de reposo. Adems ha presentado 2 sfncopes relacionados con el esfuerzo y se cansa fcilmente. Al e en trsico se palpa un pulsO-per.ifrJcculapoccuuptitud.~ se ectivo 111 El diagnstico mls probable es: ausculta un so 1 a) Miocardiopatla dilata a b) Miocardiopatia hipertrfica e) Estenosis mitra! dYEstenosis artica 7""'"e) Insuficiencia mitra! 47) Un nia de]...!,.os pre~enta dolor testicular derecho intenso, de lni~io sbito_._ asociado a.YJnitos. Al examen ffsico se aprecia1iUinto de volumen testicular derecho, asociadas a enrojecimiento. La conducta ms adecuada es: ._L..~ ./ a) Indicar reposo y analgsicos )-,+.t. /V b) Iniciar antibiticos y analgsicos /'v- - e) Solicitar urocultivo ~.r 1 v d) Solicitar ecograffa-doppler testicular ,e7Realizar exploracin quirrgica

~
~
~

~
~

~
~ ~
~
~ ~
~

~
~
~

48) Un paciente presenta ~m clico renll, el que es manejado...con..AlNEs endovenosos, con buena respuesta. Se solicita pielo-TAC que muestra un clculo dul~n el urter derecho, el que es eliminado espontneamente 3 dfu _despus, correspondi~.D~l--- .l:m.~-~'-!l9. ~~- xalato Ae...c.alcio. Cul es la conducta ms adecuada para evitar.recurrenciasJ_,. . a) Disminuir la ingesta de calcio b) Disminuir.la ingesta..de.oxalato e) Aumentar la ingesta de calcio d) Aumentar la ingesta de oxalato (Aumentar la in gesta de agua 49) Un nio sonrie.,_fjja la t:!lirada y es ~ap~-~~ levan~!J~.~~beza en posici.!!_P-rona. Su edad ----aproximada es: a) 1 mes br3 meses _./e) 5 meses d) 7 meses e) 9 meses

"

~
~
~
~

..,
~

~
~

~
~

~
~

~
26) Una mujer se realiza un Pa~J.a,lu que es informado como satisfactorio,. con presencia de clulas con displasia escamosa de alto..gm.do. Se deriva a colposcopia que muestra una zona acetoblanca, la que se bio~ siendo informada como una neoplasia intraepitelial grado 3 (NIE-3). La cordcta ms adecuada es: a) Observar evolucin b) Solicitar PCR de virus papiloma human1) e identificar el serotipo e) Bealizar crioterapia .Et}Realizar cono cervical ._-<-....l~ e) Realizar histerectomta radical ._;, s._-.s, .. ~ \ <.J-1 A ~-l:J\:::, 27) Un paciente de 27 aos ~onsnlta poF-ardor epigstrico, que asci~nde por el trax y. ~Lcu~UQ~ asociado a pequeas regurgitaciones cidas de alimentos, especialmente despus de alguna~ comidas. El examen fsico no aporta m1yor informacin. La conducta ms adecuada es: a) Indicar e icaGinde-H:-pyltm--r ciar un inhlbidor de la bomba de protone"?) cLSolicitar ma a arsolicitar endoscopfa digestiva alta e~ Solicitar radiografa baritada de esfago y estomago presenta disnea 28) Un paciente h!J;mrtenso, de larga data, en tratamiento irregular progresiva de varios meses de evolucin, que actualmente es de esfuerzos moderados. Adems presenta disoea paroxrstica nocturmt y ort9pnea de 2 almohadas. Al examen fsico tiene presin arterial (60111~mmHg, FC: 75x', ligero ~!dema de extremidades lnferlore~, y des.Qiazamiento del cffi)q~e de la plJllta. La auscultacin cardaca es normal, y la auscultacin P'-'Jmonar slo demuestra escasos crpitos en ambas'bases. La r21drograf1a de torax demuestra cardiomegalla moderada y el eleCtrocardiograma slo demuestra signos de hipertrofia del ventrrculo izquierdo. El ecocardiograma es compatible con una cardiopatfa hipe1~a, con . cardiomegalia moderada, sin hipertensin pulmonar y con normalidad de la fraccin de acortamiento. El tratamiento farmacolgico, ms adecuado para este paciente, es: a) Aspirina, atorvastatioa y furosemida , C. b) Furosemida, esplronol~ttma;-aseAGel:lrrJarol y aspil ina e) Amiodarona, digoxina, fursemida y diltia;~em d) Hidralazina, isosorbide, hidroclorotiazida y amiodarona 7)-nalapril, carvedilol y furosemida
,..;

lt'

~
~

"'
~ ~

'}

~
(!!A

~
~. x::: '.=-0

con~-~~

~tr
~

~ fA

~
~

~
~ ~
~

~
~
([~'

29) Un paciente de 47 aos presenta dolor abdominal tipo clico, asociado a ymitos en varias oportunidades y distensin abdominarp.rogresiva. Al examen fsico presenta abdomen distendido_. doloroso sin signos de irritacin peritor~y con ruidos hldroareos aumentados y metlicos. El diagnstico ms probable es: a) Embolia mesentrica J>)"bstruccin intestinal e) Colitis isqumica d) Diverticulitis aguda e) Colitis ulcerosa 30) La fluoxetina y otros antidepreslvos tnhibidores de la recaptura de serotonlna son el tratamiento farmacolgico de eleccin de la depresin en los adultos mayores, dado su mayor perfil de seguridad. Sin embargo no estn libres de efectos adversos. Los ms frecuentes entre stos son:

~ ~
~
~

HipotensiR-arter~al..y~~eso

~
~

Nuseas, sensacin de sequedad bucal y disfuncin sexual ) Constipacin, somnolencia y poliaquiuria d) Hipertensin arterial y nuseas e) Slntomas extra piramidales

(\.--S

~
~ ~
~

~
~

31) Una paciente de 41 aos consulta por dolores articulares..J:Ie las manos, y de. una rodilla de varias semanas de evolucin, que han ido en aumento, asociado a ~ificultad para mover las manos al despertar, que va cediendo a lo largo de la maana. Al examen ffsico se aprecia aumento de volymen, e.titema. edema y dolor de las muecas y de varias articulaciones metacarpofalnficas e interfalngicas distales. El diagnstico ms probable es: ,a?'Artritis reumatoide b) Lupus e) Artrosis d) Gota e) Artritis reactiva
32) Una mujer de 35 aos, cursando un embarazo de 29 sema~s Inicia contracciones uterina persistentes, a razn de 2 cada..10.Jili.nutm; y salida de lquido claro por genitales. El registro fetal es tranquilizador y el tacto vaginal demuestra borramiento cervical comgt~ y dilatacin de 4 cm, con fejs en ceflica, en espinas -3 y membranas rotas. La condu~jta MENQSJndicada es: ~) Administrar tocolfticos b) Administrar antibiticos.. c) Administrar corticoides .... d) Realizar monitoreo fetal continuo "' e) Realizar amniocentesis .

33) Una mujer de ~os, o~, consulta por aparicin de mltiples mculas..bipopigroentadas en la zona media del pecho y de la espalda, que en algunas ocasiornes presenta escaso prurito. El .. diagnstico ms probable es: :) Pitiriasis versicolor b) Pitiriasis alba e) Pitiriasis rosada d) Vitlligo e) Psoriaris gutatta 34) Qu sistema de manejo de la va area es ms adecuado para protegerla de aspirar contenido a) Ventilacin no invasiva a presin positiva b) Mscara larfngea .xflntubacin orotraqueal d) Cnula mayo e) Aspiracin del contenido gstrico por sondanasogtrica
J~strico?

35) Una mujer de 35 aos presenta abundante metrorragia luogo del parto, el que fue por via vagi~ que prese11~como antecedente, la presencia de polihidrmmnios durante la fas~'ff!L.d.e.tmbr.alQ.. Al examen fsico se palpa el tero ':!!stendido, con el fo!l!:!~- ~~~icado en el punto med_~~- ~~~~~--'! 9mb.ligo_ y la apfisis xifoides. El diagOstico ms probable es: '"------------- .. - ..... l~ercia uterina V:,) Laceracin del canal vaginal . e) Hematoma del canal del parto d) Acretismo placentario e) Rotura uterina 36) Una mujer, que trabaja en una empresa quimica, sufre entrada de cido clorhfdrico en el ojo derecho. La conducta ms adecuada es: a) Instalar sello ocular con ungOento antibitico b) Aplicar un lcali en el ojo comprometido e) Be alizar aseo con 100 mi de suero fisiolgico /J) Lavar con abundante agua durante al menos 20 minutos e) Tomar pH lagrimal y decidir conducta segn resultado 37) Son sintomas de la endometriosis excepto: a) Dolor al defecar -------b) Infertilidad ~/ e) Dolor durante las relaciones sexuales _...-,d~.-s'"angrado durante las relaciones sexuales .... e) Algia plvica

~
50) Un paciente de 76 aos sufre ~.da-a niv.el, resultando con Intenso dolor a la movilidad de la extremidad Inferior derecha y co 'imP-OSibilidad para caminar. Al exare"fisfCo'.Ciestaci-diCIaextremidad con ligefa-aodcClon.. co,-rotaci'on ~.delm.uslo. El diagnstico ms probable es: cadera -==----a F ac uxacin de cadera e) Fractura lumbar con compromiso de laB ralees L3 y L4 d) Lesin del nervio femoral e) Lesin del nervio citico

~
~ ~ ~

y.

~ (f"
~
~ ~

~
~

d~J___ao.s.presenta p!rdida progresiva de la audicin a lo largo de 5 afio,_~ln_v.r.tiga.. Se solicita una adi.o.O'le.tr[a_que..muesb:a..unahipoacusiasensodoneuralbilateral, liJJj_trjj:a, con mayor compromiso de los tonos al~~-~el__~..!(l~_~!!"~-~~-fr~-uencias. El diagnstico ms probable es: a) Otoesclerosis b) Otitis media crnica c}-Presbiacusia ....-d) Enfermedad de Meniere e) Trauma acstico

51) Un hombre

~ ~

~ ~
~

,/

52) Un paciente de 69 aos, fl.llllil~Or imp.ortante, presenta desde hace algunos meses, tos con e~pectoracin, en o.casiane.s he_moptoica, asoci~dQ_.- di_sne_a..pr.o.gresiva y ruiteiilaJin otrossintomas. Al examen cardiopulmonar no se detectan alteraciones. El di@SO.stico-ms..probable..es: ?)~ncer pulmonar b) Der.rame pleur~uc) Bronquiectasias d) Tuber.curosiS"" e) EPOC

~ ~

~
~ (!"
~

53) Una mujer cursando embarazo d(s semanaSJS:'esenta te~t de tolerancia a la glucosa, CO!! 75 gramos, resultando con glicemia basal d?-100 y glicemia ~~~~-sobrecarga de 170 9lg/dl., La conducta ms adecuada es: - -~ bservar evolucin ndicar dieta y ejercicios e) Iniciar metformina d) Iniciar glibenclamida e) Iniciar insulina

------._

mg

~
~
~

54) Un nio de 8 meses presenta tos, dmcultad respiratoria y cianosis. Al examen fislco se aprecia taquipne moderada y se ausculta_n.m.lt.lples sibilancias bilaterales. El diagnstico ms probable es: a~iliigms gf:lSfr-tletlva -b)-Neurnorrfa-c~ttptca

S{> O ,

~
~
~

_..,e)"Bronquitis / d) Bronquiolitis e) Asma


55) Un paciente d~os, sin ai"!~~~~!~Oif!.s..d~lmportancia y_ asintomtico, acude a control con urlogo, qule.n. realiza un tacto rectal, p;:.Jpa_l}.~o .~m..n.QduLo._de_tcm_en_eUbulo-P~_cter~~~ de consistencia dura. Se solicita antigeno prosttico especrfico (APE), que resulta 1,2 ng/011 (VN: menor a 4 ng/ml). La conducta masaaecuada-s: ----- --------- ----- --- --- ----. - - - .. --

~
~ ~
~
~
~

a) Observar evolucin b) Realizar nuevo tacto rectal en 6 meses e) SoJicltar-antlene-presttico-Jibre .d)'Realizar biopsias prostticas transrectales

" e) Realizar reseccin transuretral

~
~
~

~ ~
~

56) Un fumador im ortante, h!J;?ertenso y dlsllpldmico, en tratamiento con e alapril, as irina presenta o or en la pantonilla derecha, que le obliga a detenerse luego e caminar ms de 100 metros. No presenta dolor de retiOSOY erexamen fislco slo destaca disminucin de los pulsos pedios y tibiales posteriores, myor a la derecha, sin otras alteraciones. La conducta ms adecuada es: - a) Susp~de(el eigarriUcn~-indican~j"erciCiosb) Suspender el cigarrillo e iniaiar anticoagulacin -&)..Suspender el cigarrillo y solicitar angiograffa femoral d) Suspender el cigarrillo y solicitar PVR e) Suspender el cigarrillo y realizar bypass arterial

paciente"d~,

57) Un paciente di!t_btico tipo-2, en tratamientCUdlD...Qiib.enclarnkl&con mal control metablico presenta prdida sbita de liUdsiR-del-.ojojzquierdo, persistiendn -.QJQ_C?_C?.~-~~~~~~-~~-.l!~-~-P.e9.~.e~~- --' zona perifrica del campo visual temporal inferior. El diagnstico ms probable es: . a) Desprendimiento de retina ;.b)~dema macular / c}..JrambosSaera-vepa central de la retina

,d)~emorragia.v-ltr-ea-'

e)"

etir.iQp.ata.aiabtiea-pr-elifeFativa-

58) Un nio

:r

desviaciones estndar. El diagnstico nutricional es: a) Desnutricin b) .Eutrofia Sobre peso d) Obesidad e) Obesidad mrbida

de4""'"~~tiene-l_eEL.t2~:Vdesviaciones estnda{~~~~-~~~2\tesviaciones estndar, ITE +0,9

59) Qu trastorno de personalidad se asocia .mayormente a !os del_!!!_c.uentes?l a) Esquizoide b) Limitrofe e) Narcisista
~fAntisocial

e) Paranoide

en a) Etiologa coronaria - b) Grado de cardiomegalia ,_...cfFraccin de eyeccin ventricular / d) Presin de fin de distole e) El grado de hipertensin pulmonar secundaria

60)

ElffiGt~:.;~~' l~ficiencia card;=~~' es:


-./--~.---......,\

61) Un paciente sufre un accidente...laboJ:al, resultando con unatfrilcturuxpuema del dedo indjce_ deJrcho. La conducta ms adecuada es: . ---. ..a} Realizar aseo con abundante suero fisiolgico, administrar analgsicos y antibiticos endovenosos y /realizar vacunacin antitetnica segn esquema __ b) Realizar aseo-Con-abunda-nre suero slolOglco;affiintstra-analljslcos_y.antibiticos endovenosos y reElii'Zar
inm~ciqn.coR..yeso

e) ""R:ealizar aseo con abundante suero fisiolgico, administrar analgsicos endovenosos y cubrir con apsito. estril d) Administrar analgsicos endovenosos, realizar curacin de la herida, cubrir con apsito estril y derivar a un centro especializado e) Administrar analgsicos y-aAtibiticas..e_Qc;!Qve~!?.~Q-~J.l!lY@.!.l.~ herida y de_~~L~-~!ro.especializado-de inmediato -- --- - -

"'
62) En un estudio se escogen a ~1cientes diagnosticados deJupus ~~stmic~, de los cuales 200 son seleccionados para reQll?ir prednisona..ms azatioprir:-a, mientras que los otros 300 son seleccionados para recibir prednisona ms leflunomide. A qu tipo de estudio corresponde? a) Caso y control b) Cohortes e) Transversal A-Ensayo clnico e) Ensayo de campo 63) Un paciente de 56 ao.. diagnostil:ado de dao heptico crnico por consumo de alcohol, con hbito alcohlico suspendido hace 3 ~1os conSffii- por aumento del perimetro abdoiTal. Afexamen ffsico se aprecia en buenas condiciom~ Qrientado, con abdomen distendido y ascitis moderada. Se realiza paracentesis diagnstica, que descarta presencia d peritoiUS6act:eriana. La conductams adecuada es: - - - - - - - - --- a) Iniciar lactulosa b) Realizar paracentesis evacuadora ~rriciar rgimen hiposdico y diurticos d) lniciat:..antibitises-eAEievenosOs e) Administrar albmina endovenosa 64) Un paciente de..4!!.os, ~~diabtico tiP-O 2 e1J..1ratamiento con glibenclamida..y-metfor.minar presenta un cuadro de sudoracin, temb.klr-y-lue~..Q. de consciencia. El diagnstico ms probable es: ~ a) Cetoacidosis diabtica b) Accidente vascular enceflico e) Sndrome hiperglicmico hiperosmolar d) Sincope vasovagal y-upoglicemia 65) Una paciente de .J!,o~ consulta pofdisuria.-'y leucorrea abumf.snt~. A la especulocopia se observa leuGGFr-ea-ver-Eie-ab~.mdante y_n1arcados s!gnos inflamatorios de la pared vaginal, con algunas petequias. El agente causal ms probalble es: a) CalTI::IidaalbiG8Rs. b) 6seherichla coli e) Gardner:ella .vaginalis.d}'Trichomopa vaginalis /e) Clamydia trachomatis 66) Un paciente d"~ presenta unt cuadro d~ flbre, artralgias y edema de las extremidades infeiores. Se solicitan exmen~~.don~c~ destacan BUN: 51_!!!9[dl, creatinina: 3,5 mg/dl, hematuria dismrfica, proteinuria +++,N..fS: 50 mtn/h, blanconooo x.mm3, he11J.atocrito~ 32%, Hb: 11 g/dl, VCM: 89ft, HCM: 32 pg, ANA(-) y AN.QA~u.clear. El diagnstico ms probable es: a) Lupus .sistmico b) Clioglobulinemia e) _rteritis de clulas gigantes ~) Granulomatosis de Wegener\::./ .,e) Poliangeltis microscpica
.
~--~

"'

fA

"
f'A
~ ~

(A

f'
~
~ ~
~

fA

"'

~
~

~
~

"
~
~
~

-\

J l. t.\ t e "-':r-:>' '( ~ \.

~ ~
~ ~
~

~
~

67) Un nio de-5"~5ds presenta fiebre alta y cefalea. Al examen fsico se aprecia muy decaido, con signos menrng~qs, por lo que se:e/uq~_uncin lumbar que demuestra salida de un l~uido turb~. con-protei~s 80 mg/dl, glucosa~mgt~l, clulas: 890 x mm3,_polimodonucleares:11~, mononucleares:_13.%. No se visualizan hact~ria~ e~ tinci~n de <?ram. El diagnstico ms probable es pfMeningitis bacteriana aguda <' b) Meningitis viral e) Encefalitis herptica d) Meningitis Tuberculosa e) Infeccin parameningea

f!A
~

,..
~

68) Una mujer de 35 aos presenta desde hace varios aos crL?is de pnico recurrentes, inesperadas, asociadas a anSidad anticipatoria v....D.d:ctas_evitati.v.u, por miedo a morir durante una nueva crisis. Su examen fsico y exmenes generales son nor111ales. El tratamiento farmacolgico ms importante para la patologia descrita es: 9}-"Antidepresivos /b) Antipsicticos e) Estabilizadores del nimo d) Benzodiacepinas e) Ninguno 69) Un
mq~J!..da, que au~~nta paulatinamente, hasta ~~ce~e my_yJrltenso y ~aJa ..f.nsailaca

paciente~iios presenta. un cuadro de dolor abdQininal epigstrjco, deJ.o.t~sidl~ ..

derecha. Al examen ffsico se palpa-abdmnen blando ...depresible. dol6rso dlfusg; mayor en la fosa . lliacaderecha, con signos de Blumberg-y w:esistencia muscular en dicha zona. Los ruidos hidroareos estan presentes. La conducta ms adecuada es: . a) Iniciar antibiticos de...amplio.. espectror ar.talgsiCGs y rgimen liquido b) Iniciar antibiticos de amplio espeatre; . analgsicosyrgimerrce,ro e) Solicitar ecograffa abdominal d) Solicitar TAC de abdomen y pelvis ~~Resolver de manera quirrgica de inmediato 70) Una_grimigesta de 19 aos de-edad, cursando embarazo d.e 31 semanas prese~~ ~-~ema de extremidades inferiores y presin.arteriaLde1:5o"i9o mmHg en varias tomas seriadas, reiliia'das duro1@.6o~s. El resto del examen ffsico no aporta mayor informacin. Que m~dida es ms til para definir e diagnstico? a) Solicitar hemograma con frotls b) Solicitar clearence de creatinina .q}:Solicitar proteinuria de 24 horas .-: d) Solicitar ecografa doppler de arterias uterinas e) Solicitar ecografa doppler de arterias umbilicales 71) La causa ms frecuente de ipsuficJ.encja renal crnica en los pacientes adultos es:
~1 Diabetes mellitus

,/ b) Glomerulonefritis e) Pielonefritis d) Hipertensin arterial e) Postrenal 72) La capacidad de obtener resultados favorables a menor costo se denomina: a) Eficacia .b Costo-beneficio ~e) Costo-efectividad d) Costo-utilidad e) Efectividad

presenta y dolor en el Al 73) Una paciente de examen fsico se palpa r 1O cm bajo el reborde costal. Se solicita hemograma que m':JestrJ:u~m ocrito _40o hemoglobina:...~3.,2-glJrplaquetas:.,440.000 por mm3, blancos: ~-~-'!00 por mm3, llnf.9.c s: 1 %, monocitos 4%, segm~ntado.s:..6..1!/q, .I?.~.~.Uifrm.~s ..1.~%, mielocitos: 4%, promielof?!!9.s: 2% El diagnstico ms probable es: -.... a) Mielodisplasia -- b) J,.eucemia mieloide aguda ~)"'Leucemia mieloide crnica d) Mieloptisis e) Mielofibrosis

4~s

~t;~~-.-;:~~ ocasiol1!~

~.

('A
74) La conducta ms adecuada frente el un lactante con diarrea aguda, sin deshidratacin, de 3 das de evolucin, es:
a) Administrar suero fisiolgico b) Iniciar amioiticosyliquiCios-rehitftat~intesporvra orar e) Iniciar antidiarreicos y iquidos rehidratantes por via oral d) Iniciar corticoides, antiespasmdicos y lquidos rehidratantes por vi a oral ~niciar lquidos rehidratantes por va oral

f!t"

~
~

('t

75) Cul de las siguientes patologras puede confundirse frecuentemente con un trastorno de pnicoi a) 1-linnn:~ ...mi.t..b) Ce.taacidesis
~~.JJ.I,.,.I;
IQ'i) . - - - - - -- ......... ~ .. --~

e) l:f.i_RQtiroidi$Dle

("JJ\

d) E;nfermedad de Cushing ~)-1rastorno de angustia generalizada

(P\

(f?A
~

f'A
(f1t\

t~-g~~~f~~=s

76) Un nio ~e 7 aos presenta aument.o.J;!e volumen facial y edema de extremida~ asociado a orinas espumosas y gm-as de grasa en la orina. El examen ffsico no aporta mayor informacin y la creatini~mia,::!MJlL ANA y sedj_rr~~n!.~ .~t~-~rirt.!.~~.!! ..!:IP~.!~!_s, sin emb~rgo ~~ p~~~~~-~~r.i~ ~~- .?4 hc:>r-~ resulta.:1,.3..glni2..'EI tratamiento ~-~J~ p.atQipg{a.d~~-~rita es:

----

. . _.

('A
(\

e) Antihipertensivos d) lnmunosupresores e) Observacin

(91'1

77) Se realiza un ensay_Q...clni.<;() que compara un nuevo frmaco antihipertensivo contra e_IJ~lf;!Rril. Este nuevo fr!!'aco muestra !!D ri~o relatilvo de efectos adversosdes-;cori'"rfmW,.:Va.lif(;~ c,o.,fianza de ~..6% d~. Cul frmaco recomendara?
a) El nuevo frmaco porque es ms efica2: b) El nuevo frmaco porque tienen menos efectos adversos e) El enalapril porque es ms eficaz j1 El enalapril porque tiene menos efecto~; adversos e) Cualquiera de los dos frmacos, ya que!_ no muestran diferencias significativas

~
~

~
~

78) El examen ms adecuado para proseguir el estudio de un paciente con historia cHnica de angina~ crnica, examen fsi.c..Q..U.~~!rniirtlioQraiaf[Qfr'i'i'al;es: -- ---. - ........ -- - - --

~
~

~
~
~

'al Hoiter d-arritmias /61'"est de esfuerzo e) Troponinas plasmticas d) Ecocardiograma e) Coronariografia


79) Un nio de-a,aos presenta sntollJ.nS de malab.snr..~LQ...tt!ntestinal. Se solicita test de Sudn que confirma esteatorrea. La biopsia de mucosa duodenal, obtenida mediante endoscopa digestiva alta, mues~!ft-!!"~!!~-~~..!!1~ Ye!'C?.sidade.! !nte~~!Jales ~ -~-n SU_!._!~~~- de laf?or~~r~.o d~~~~-~. .~~.~~-~~: - fE!X~9~enica y posJtlvtdad de los ant~u~~E_~-~-~~t~tr~~r:ag_~~~a_l'!!~~a. Eftratam1ento de la patoJogla descrita es: -------a) Antibiticos __.)JrDieta ...- e) Corticoides d) Suplementacin enzimtica e) lnmunosupresores

(P'
(}"'
('!'
~

(1!'1

~
~

~
~
~

80) Un paciente.Q..e 73 aos consulta por t~mblorde las extremidades ~~P-~.!~.r~s, que se acenta al t9.m.~rJ.t.Q~bjeto o adoptar alguna postura co-his.ianos~-Aaemasse-observa un pequeo temblor de la cabeza y del mentn. Camina normalmente y no presenta signos focales ni cerebelosos en el examen ffsico. El diagnstico ms probable es:
a) Envejecimiento normal b) Enfermedad de Parkinson e) Corea de Huntington _;.11-Temblor esencial ./ e) oemencia frontotemporal

~
~
~

~
~

81) El tratamiento de eleccin para la erradicacin del Helicobacter pylori consiste en: a) Omeprazol, levofloxacino y rifampicina por 6 meses b) Omeprazol, clindamicina y ampicilina por 14 das e) Omeprazol, bismuto y metronidazol, por 7 das d) Omeprazol, tetraciclina y ciprofloxacino ,efbmeprazol, amoxicilina y claritromicina por 7 das
82) Durante un trabajo de parto una paciente muestra un registro cardiaco fetal con variabllldml disminuida, presencia de desaceleraciones variables y frec"iffll!la cardlacacercaia1ool'iidos por minuto. La conducta mas adecuada es: --- --------- ---- --------- --- - -

a) Realizar un perfil biofisico b) Administrar oxigeno e indicar reposo en decbito lateral izquierdo e) Observar evolucin espontnea, bajo monitorizacin d) ,L}dfmilislcar...tOGGUti~REievenosos-1~1 Realizar un tacto vaginal, para determinar la via ms expedita de interrupcin del embarazo 83) Un paciente de 8-af\os-consulta por odinofagia, fiebre altcUUtparjcin de..un.exantema.papular..... eritematoso. En el examen fsico se apr~ci ~ngua sabur.ra4u;.e...observanJas.amfgdalas-aumentadas de \amaa,_con exu~~d_o_gd~, adh~rf!.ote: Adems se palpan adenopatfas.cervicales y se observan pete.quias line!.l!~_!.nJ~-fQ~_ .~Mb.i.tsles. El diagnstico ms probable es: a) Sarampin b) Rubeola .:;_) t:scarlatina d) Mononucleosis e) Eritema infeccioso

84) Un paciente diabtico tipo 2 en tratamiento con metfori!Jina-e-insulina se realiza exmenes de control que demuestran presen-cia d~:micr:oalbumlnuria..d.ZO..mg/di,::Sin alteraciones en el clearence de la creatinina. Sus exmenes anteriores presentaban valorc~s similares. La conducta ms adecuada es: a) Aumentar la dosis de insulina b) Agregar un diurtico A1niciar un iECA d) Iniciar cido acetilsalicilico e) Mantener tratamiento y controlar con nuevos exmenesen 3 meses
85) Una paciente {!e ~~S>se ..aprecia.enflaquecido y de una falla menor a la esperada para su ~~ad. Al revisar su curva de creci~!~D~Q -~-E!.-P.!~P!C!. que ~E. ~elocidad de crecimiento fy_~_rumnal.hasta cumplir. el.ao..de-edad, momenui.n ..que_esta..~L~J!Iml. de crecimie[l!lQ_Q.i!!linuy. ..marcadamente. La madre refiere que presenta c;Usurea .frecuente..y.en .sus.exmeoes d~!$.~_'?a anemia ferropnica. El diagnstico ms probable es: -- ------ --------- ---- ... a).SindrQme deT1:1rner b5 RetrasD..constitucional-del crecimientcr e) Fibrosis quistica d) lnmunodeficiencia humoral .e)"Enfermedad celfaca
.'"

86) Qu parmetro es ms adecuado para calcular la slgnificancia estadfstica al comparar los ___P-orcentajes de pacientes que responden a distintos tratamientos, en 2 grupos distintos? a) t de Student .. .... ------~ _,..6f'Chi cuadrado e) Anlisis de varianza d) Test z normal e) r de Pearson

87) Un nio de 6 aos es trado por su madre, ya que presenta epistaxis recurrente, desde hace 2 dfas. Al examen fsico se aprecian petequial; generalizadas, sin otras alteraciones. El diagnstico ms probable es: a) Hemofilia b) Prpura trombocitopnico trombtico e) Prpura trombocitopnico Inmune 9,)-EhfermedaQ de Van Willebrand 'e) Meningococcemia 88) Cul de las siguientes lesiones es ms suger~qte de u nr-lanol!)a? ... a) Mujer de 60 aos con una lesin oscuri3, solevantCfi:i con multiples plos gruesos, ubicada en la cara b) Hombre de 64 a~s, con una lesin hiperpigmentada, de lento crecimiento en el pecho, de superficie rugosa, indolora ' e) Hombre de 65 aos, con una lesin hiperpigmentada en la mano derecha, de color homogno y bordes __ regulares : d) Hombre de 73 aos con lesin solevantada, dm..Qorde perlado y telangiectasias adyacentes, ubicadas en la cara ,..efMujer de 55 aos con les!$:m oscura, de color heterogneo, que ha duplicado su tamao en 2 aos! alcanzando 1 cm de di~efro, ubicada en la pierna izquierda ' 89) Una paciente de 30 aos, asi~!Qro1:i~a, presentarcomohalla:zgo.er:l-una ecograffa transvaginal, la presencia de un mioma uterino'transmural de 4 cm de dimetro. La conducta ms adecuada es: a) Observar evolucin --- - - - _oflniciar anticonceptivos e) Indicar anlogos de GnRH d) Realizar tumorectomia histeroscpopica (resectoscopia) e) Realizar histerectomia 90) Un recin nacido de.B..h.Q!_-~- Q.Q..'lida\, cuyo grupo sang_yL1J.~SLe..$.._~:-.BJ!.~9sitivo, presenta ictericia. Su madre no haba prese.[l~~~ -~-~~~~:Z:~~ .P.r~Y.~.~-~-.Y-~-l!._9.~-~~.1!9!1~~eQ..Q.:B..I!~.9tlV.9 El diagnstico ms probable es: a) Ictericia fisiolgica b) Hemlisis por incompatibilidad Rh ,~~emlisis por incompatibilidad de grupc~ clsico d) Atresia biliar primaria e) Dficit enzimtico heptico

~
~

~
~

'~ *"
~

~
~ ~
~

~
~
~

~
~ ~
~ ~
~

~
~

~ ~

~
~ ~ ~

~ ~ ~

Jff'

TITULO 1) Una paciente de 64 aftas, con antect!dente de hipertensin en tratamiento con enalapril y estenosis mltral, secundarla a enfermedad reumtica, presenta desde hace 4 meses palpitaciones, que en algunas ocasiones se hacen muy Intensas y se asocian a disnea. El en examen frslco destaca pulso irregular, confirmndose mediante electrocardiograf[a, la presencia de fibrilacin auricular. Las siguientes alternativas representan mE1dldas vlidas para el tratamiento de esta paciente, EXCEPTO: a) Acenocumarol oral -JJ t:e~_r. o.:,,:-, u~ b) Amiodarona oral e) Carvedilol oral ~Adenosina endovenosa e) Digoxina oral

~
~

cm tratamiento con hiposdlca, presenta presiones j . 2) Un paciente de 46aaos, hlpertensolos ltimos 3 meses.dietafuma ni padece otras patologlas. El arteriales cercanas 150/90, durante No
examen fsico es nonnal. La conducta ms adecuada es: a) Controlar en 3 meses, sin necesidad dH frmacos ni exmenes b) Solicitar un Holter de presin arterial e) Solicitar ecodoppler renal, fndice de aldosterona/actividad de renina y metanefrinas urinarias ~iciar un diurtico tiazfdico o "!..Ec,. e) Iniciar cido. acetilsalicflico /3) Un paciente de 69 at\os, con anteced,ente de insuficiencia cardiaca crnica, .secundaria a una cardlopatra hlpertenslva, de larga data, en tratamiento con enalapril, esplronolactona, hidroclorotlazlda, aspirina y atorvastatina, consulta por aumento de su slntomatologla y progresin de su disnea, que actualmente aparece ante mfnlmos esfuerzos. El examen ffsico demuestra FC: 87x', regular, PA: 130/80, crpltaclones pulmonares escasas blbasales, y un RR2T sin soplos. La conducta ms adecuada para mejo~ar el pronstico en este paciente es: a) Cambiar el enalapril por'.un inhibidor del receptor de angiotensina 2 b} Iniciar digoxina e) Iniciar propanolol d) Iniciar furosemida ,~nielar carvedilol

~
~
~

~
~
~

~
~
~ (!"
~

~ ~
("

/4) Un paciente de 74 af\os ha presentado sincopes relacionados con la actividad flslca. Adems presenta angina de pecho al caminar 2 ~Cuadras. En el examen fislco de~taca un pulso regular de poca anylitud y un soplo mesoslstlico IIINt, con click de apertura. El diagnstico ms probable es: ..a}Estenosis artica b) Estenosis mitra! e) Insuficiencia artica d} Insuficiencia mitrar e) Miocardlopatla hipertrfica obstructiva

~
~

'f.

~
~

5) En el electrocardiograma las extrasfstoles auriculares se caracterizan por presentar: a) Complejo QRS ancho y presencia de pausa compensatoria ~omplejo QRS angosto y ausencia de pausa compensatoria e) Complejo QRS ancho y ausencia de pausa compensatoria d) Alargamiento del segmento QT y alteraciones de la repolarizacin e) Acortamiento del segmento QT sin altemciones de la re polarizacin 6) Paciente de 65 aos, cursando el cuarto da luego de un Infarto agudo al miocardio, evoluciona sbitamente con disnea y deterioro hemodinmlco. Al examen destaca ritmo regular en 2 tonos, con un ;;opio holosistlico, asociado a frmito. El diagnstico ms probable es: _a) Rotura de la pared libre ventricular b) Infarto de ventrfculo derecho ~otura de msculo papilar d) Diseccin artica e) Infarto del nodo aurlculoventricular

~ ~

~
~
~

~
~

~
~
~

( 1"

_... /

7) Un paciente de 65 aos, sin antecedentes de Importancia, Inicia srncopes, asociados a sudoracin y desorientacin. Al examen fsico presenta FC: 29 lpm, PA: 70/30 mm/Hg, frialdad y palidez de las extremidades, RR2T, sin soplos. El monitoreo electrocardiogr~fico es compatible con un bloqueo aurfculoventricular de tercer grado. La conducta Inicial ms ad~cuada es:

~
~

a) Solicitar electrocardiograma de 12 derivaciones. b) Administrar suero fisiolgico por vra endov~n0sa e) Administrar epinefrina 1 mg por vra endovenosa ~stalar marcapasos externo e) Realizar cardioversin elctrica

~
~ ~

8) Paciente hipertenso, de 67 aos, con dolor torcico intenso, de inicio sbito. Al examen

fsico FC: 110x', regular, PA:230/128, asimetr[a en los pulsos radiales y a la auscultacin cardiaca destaca un soplo diastlico. La conducta inicial ms adecuada a seguir, adems de tomar un ECG es:
a) Realizar coronariograffa urgente . b) Administrar oxigeno, .morfina, nitroglicerina y aspirina inme1diatamente ~ealizar ecocardiograma transesofgico urgente d) Realizar pericardiocentsis e) Solicitar radiograffa de trax
Q

~
~ ~

~
~
~ ~
~

j.. 9) Escoja la asociacin correcta entre la cardiopata congnita :v su mal)ifestacln clfnlca


a) Comunicacin lnterventricular- Soplo sistodlastlico en maquinaria b) Comunicacin lnterauricular- Soplo holosistlico, asociado a infE~cciones respiratorias recurrentes e) Ductus arterioso persistente - Soplo sltlico eyectivo, asociado a desdoblamiento fijo del segundo ruido itf"Ventrlculo nico - Cianosis que no responde a oxigeno en altas concentraciones e) Coartacin artica- Soplo diastlico, asociado a asimetrla de los pulsos pedios j..
1 O) Una paciente de 58 aos, hlpertensa, en tratamiento con enalaprll, presenta un cuadro de 60 minutos de evolucin, de dolor retroesternal9/10, opresivo, irradiado al hombro izquierdo. El examen fisico muestra FC: 85x', PA: 140/100, ritmo cardiaco regular, en 2 tonos, sin soplos y examen pulmonar normal. El electrocardiograma muestra Inversin de la onda T Etn las derivaciones DI, V5 y V6. La conducta ms adecuada es:

~
~

~
~

a) Iniciar aspirina, oxrgeno, nitroglicerina y solicitar test de esfuerzo ~nielar aspirina, oxrgeno, nitroglicerina y solicitar troponinas plasmticas e) Solicitar enzimas cardiacas y realizar ecocardiograma transesofgico d) Realizar coronariogratra de inmediato Q. e) Administrar trombolfticos endovenosos

~
~

/1.1) La cardlopatfa congnita ms frecuente es:


a) Coartacin artica b) Ductus arterioso persistente e) Tronco arterioso d) Comunicacin interauricular ~omunicacin interventricular

' 12) Un paciente de 18 aos presenta dolor precordial, de Instalacin progresiva, que aumenta con la
inspiracin. El examen fislco no aporta mayor Informacin. Se solicita electrocardiograma que demuestra supradesnlvel del segmento ST en todas las derivac:lones precordiales. El tratamiento de la patologfa descrita es: ~ ':.\)~ -' ;t_' e

a) Estreptokinasa

c.o-~'""' :::>

~IN Es

e) Corticoides d) Pericardiocentesis e) Quirrgico

~
.13) Cual de los siguientes factores se nsocla a menor mortalidad hospitalaria en casos de Infarto del miocardio? a) Sexo femenino b) Presencia de diabetes mellitus e) Localizacin en la pared anterior d) Taquicardia sinusal ~ad menor de 45 aos
14) Un paciente de 67 aos, cursando su segundo dia de hospitalizacin por un accidente vascular enceflico hemorrglco, presenta un cuadro clfnlco compatible con una TVP de la extremidad inferior Izquierda, la que se confirma mediante ecografa, con compromiso de la vena poplitea. La conducta ms adecuada es:

~ (i'
/

fJ;

f!'

(J'

~ ~
~
~
/

a) Iniciar aspirina b) Anticoagular con heparina e) Iniciar trombolisis con activador del plasmingeno ~nstalar filtro de vena cava inferior e) Resolver quirrgicamente de inmediato
15) Un paciente de 67 aftos, diabtico, en tratamiento con metformlna y glibenclamlda, mal controlado, presenta episodio actual, caracterizado por sudoracin y palidez. El examen flslco no aporta mayor informacin. Se solicita hemoglucotest, que resulta 233. La conducta ms adecuada es:

~
~
~

a) Enviar a domicilio, aumentando la dosis de hipoglicemiantes orales b) Iniciar insulinoterapia y agregar una estal.ina al tratamiento e) Solicitar un test de tolerancia a la glucos;:, oral, prolongado ~olicitar electrocardiograma e) Solicitar un ecocardiogarama

~ ~
~
~

/16) Un paciente de 40 af\os, fumador de {i cigarrillos al dla, sin otras patologlas de importancia,
presenta disnea de esfuerzos, progresivcn, de 3 meses de evolucin, asociada a ortopnea y edema de extremidades inferiores. Actualmente est en capacidad funcional Al examen fsico presenta FC: 90x' regular, PA: 110/70, RR2T sin soplos, desplazamiento importante del choque de la punta, murmullo pulmonar simtrico, con crpitos escasos blbasales, edema de extremidades inferiores y reflujo hepatoyugular. La radlograffa de Mrax demuestra cardiomegalla, hllios pulmonares prominentes, con redistribucin sangufnc~a hacia los pices. El diagnstico ms probable es:

m.

~
~

'!'
~ ~

a) Cardiopatfa hipertensiva b) Taponamiento pericrdico "-..ef'Miocardiopatra dilatada 10 d) Cardiopatla cronaria e) Pericarditis constrictiva

~
~

"'

/.17) Cul de las siguientes opciones NO t:orresponde a un soplo funcional o inocente? a) Su intensidad es IIINI b) Presenta una tonalidad musical e) No tiene frmito d) Aumenta con el ejercicio .#Es diastlico
/ 18) Un paciente de 70 aftos, cardipata coronario, en CFIII, evoluciona con tos y expectoracin mucopurulenta, agregndose disnea de rEtposo y rpido deterioro del estado general, con sincopes y desorientacin. Al examen ffsico se aprecia plido, con extremidades frias y pulsos dbiles. Se constata FC: 120x', PA: 66/31 mmHg, con RR3T y MP+ con crpltos bilaterales, mayor en lavase derecha. El monltoreo electrocardlogrflctl demuestra taquicardia sin usa l. Se Instala catter de SwanGanz, obtenindose PVC: 21 mmHg (VN: 0-8), PAP: 45/22 mmHg (VN: menor a 35) y PCP: 33 mmHg (VN: menor a 18). El diagnstico ms probable es:

($'

("A
~

~
~

~ock cardiognico

~
~

b) Shock sptico e) Shock hipovolmico d) Shock anafilctico e) Shock medular

'fV C.. t::t-1 \-o-

_,

.....l"bc.dk:> h\t30vo\~~a-
V'.:li

~
~

""'

~
~

,._,
~

~
19) Un paciente de 67 hospitalizado por shock sptico, presenta prdida de conciencia, constatndose ausencia de respiracin y pulso. Se Inician ventilaciones y masaje cardiaco y se Instala monitor electrocardiogrfico. Luego de 5 ciclos contina sin pulso y el monitor demuestra asistolia. La conducta ms adecuada es: a) Realizar desfibrilacin elctrica y continuar el masaje cardiaco ~(
~

l1l)
~

b) Realizar lntobacin orotraqueal. inlciatventilacin a presin positiva. con oxfgeno al100% y continuar masaje cardiaco ~dministrar adrenalina 1mg por vfa endovenosa y continuar con f!l masaje cardiaco d) Instalar marcapasos externo y suspender el masaje cardiaco e) Realizar 5 ciclos ms de masaje cardiaco y reevaluar

~
~
~

20) Un paciente de 61 aos presenta dolor de la extremidad Inferior derecha, de Inicio sbito, asociado a parestesis y dificultad progresiva para mover los ortejos de esa extremidad. Al examen ffslco se aprecia palidez de la pierna derecha, con clara disminucin del pulso pedlo. La conducta ms adecuada es:

a) ~Piicltar Ecograffa Doppler de extremidades inferiores .M'Anticoagular con heparina y realizar anglografl femoral de urgencia e) Solicitar pletismogratra de volumen de extremidades inferiores d) Instalar filtro de vena cava inferior e) Administrar trombolftlcos endovenosos

~
~ ~

~
~

~ ~
~

A)
~
~

~
~

~
~

")
~

~
~
~
~

.d\
~
~

~
~
~

~
~
~

~
~

~
~

PRUEBA REPASO 2 Cardiologa+ Res,piratorio 1) Un paciente de 67 aos, dia~tjco. en tratamie rmina y glibenclamida, mal contr presenta episodio actual, caracterizado po doracin e saco no aporta mayor informacin. Se solicita hemoglucotest, que resulta 3 La conducta ms adecuada es: a) Enviar a domicilio, aumentando la dosi:s de hipoglicem1antes orales ~ b) Iniciar lnsulinoterapia y agregar una estatina al tratamientQQ. ~ e) Solicitar un test de tolerancia a la glucosa oral, prolongado../' -\JI(SoJicitar electrocardiograma e) Siicitar un ecocardlogarama

~
~

2 Un paciente con antecedente de E.E!2..C tabguico, ~o, de 1 O aos de evolucin, consulta por uadro de astenia y mayor..Qi&Aea de la habitual, asociada a edema de extremidadeuo1Eu:i.D.r.e_$_.y__ _ hepatomegalia. Actualmente no fuma )' sigue tratamiento con broncodilatadores y corticoides fn~os. El examen fsico no apGrta mayor informacin. El diagnstico ms probable es: a) Cardiopatla coronaria b) Fibrosis pulmonar .. ;tf~?r:pulmonar e) ~ipertensin pulmonar {La hiperkalemia con frecuencia, se manifiesta electrocardiogrficamente como: a) Alargamiento del segmento QT angosto /: ,, as T picudas d) lnfradesnivel del segmento ST e) Ondas T invertidas
c).~cer

~ ~
~
~

~
~

~
('A
~
~ ~
~

4) conducta ms adecuada ante un paciente diagnosticado de neumonra ATS 1, que no responde al tr tamiento con amoxicilina oral, tueg() de 72 horas, es: ) Mantener el tratamiento con amoxicilina y reevaluar en 24 horas ~?~ciar tratamiento con amoxicilina + cido clavulnico ~~';:?spltalizar y solicitar radiografa de trax d) Solicitar cultivo de expectoracin con antibiograma e) Iniciar tratamiento con moxifloxacino o.ral
pcr fiebre, compromiso del con expectoraciQn dis11ea. Al examen ff~;ico presenta FC: 11.0x', PA: FR:342Q saturacin de oxgeno ~ El examen pulmonar demuestra crpitos localizados en la mitad superior del hemicampo pulmonar izquierdo, asociadoa a matidez y aumento de transmisin de la voz. Se solicita radiograffa de trax que demuestra condensacin del lbulo superior Izquierdo. El tratamiento ., antibitico ms adecuado es: ~ a) Amoxicilina oral _~ .. (' b) Amo~icilina + cido e avulnico oral -\<" ~t~ ~ ..:::,) e na endovenosal td -fr\ ~ ti) "Ceftriaxona + entromtcJna endovenosas r --, e) Claritromicina oral
.mycopu?de~ta-y

5) Un paciente de

~consulta

esta~ner~
f56tU!O,

~ (!$"

~
~

e;/

;.,- _.. . -(

~
~
~

~
~
~

~
~

6) Un paciente de 32 aos, sin antecedentes de importancia presenta un sincope, relacionado con el ejercicio. El examen ffsico no aporta mayor informacin. Se solicita un electrocardiograma que demuestra signos ..de hipertrofia del ventriculo izquierdo, asociado$ a ondas T invertidas gigantes en, las derivaciones precordiales. El diagnstico ms probable es: a) Estenosis artica 'y~ b) Miocardiopatla dilatada ~\ ~ f e) ~jocardjopata hipertrfic_a - _ ~OJ"> ....:,d) lodrome ge Brugada .~ ~ ..< 1 e) i~plasia a~mogenlcc del ventrculo derecho '('

,r

,.V ~Q-~

~J-

t>C\ll

~ ~ f!'A
~

~
~

7) de 24 aftos, consulta por dolor torcico de 3 dlas de evolucin, al que se le ha agregado .fn:a de esfuerzo y dolor en hipocondrio derecho. Al examen se observa cansado, taquicrdico, con ndencia a la hipotensin. La yugulares se encuentran ingurgitadas, con un colapso transitorio arcado, durante el ciclo del pulso venoso. El examen pulmonar es normal ~ se palpa el hfgado por 3 m bajo el reborde costal, siendo moderadamente doloroso. El diagnstico ms probable es:
.

~paciente

,._,
4)
~

~
~

) Endocarditis aguda "b)ltqlnboembolismo pulmonar ~'J,ponamiento pericrdico d) 1 farto agudo al miocardio e) iocarditis aguda

~
~

~
~

) Un paciente de.1Z,.a\os consulta por dolor torcico de inicie, sbito, con tope inspirato-:lo, asociado a disnea. Al examen fisico se observa en buenas condiciones, disnelco, con frecuencia cardaca y presin arterial normales. Se constata disminucin del murmUlllo pulmonar en el hemlcampo pulmonar derecho, asociado a hipersonnrldad a l@ percysin. No se auscultan otros ruidos. La causa ms probable del cuadro descrito es: a) EmboBa pulmonar b) Pericarditis ~ _2~rrame pleural ...~otura de bulas pulmonares e) NeumJtrax a tensin
1

~ ~
~

,., ,.,
~

9) Un pilclente de 64 aos, diabtico e hipertenso, tiene antec,;)dentes de cardlopatra coronarla, con una ci'ruga previa de revaseularizac n miocrdica hace 2 aos. Est en tratamiento con hi "cemiantes , 1 enalap 1 o 1az1 a, a orv na y asplri1@, Consulta por un cuadro de olor precordial izquierdo. intenso. e carcter opresivo, nadlado al hombro Izquierdo, de 20 utos de dura9in y que cede espontneamente 1 hora antes de la consulta. Actualmente se uentra asintomatico y el e!!mn ffsleo resalta normal, al igual que el electrocardiograma, La ducta ms adecuada es: os~it~lizar, admini~trar oxigeno, aspirin~ . h.~parina y antianginosq.~ y solicitaU!nzimas cardiacas dmtmstrar analgstces-eRdm:tenosns y-~=eahzar test de esft:t~meeHatarmmte e) Solicitar-ecQgr~amiRa-Y-GeeiEfiHRSflejo Segn hatrc~es d) Solicitar radiografla de trax y decidir conducta segn hallazgon e) Enviar a domicilio e indicar control ambulatorio con ecocardiograma

~ ~

~
~

~
~

"i'

10) La presencia de un complejo QRS ancho, de morfologia rSR' en las derivacione- es caracterstico de: _.._ --. a) Bloqueo aueuleventricol ~ '":fj ,J,_,. _ ~~ ~ ~
15 Bloqueo completo d a aqueo comp e o e rama Izquierda d) lrlemibloqueo iz erdo anterior ., l "' e) Hemibloqueo tzqui osterior-. ' ........... - - __ ..!-___;:::.....;.__.1.::;--....-..--.
1 -

-t

'\

~"

o-u

,-,
~
~

11) Un paciente coh antecede~te de .tabaquismo y baja de pet;o, se realiza radiografa de trax que demuestra la presencia de 3 ndulos pulmonares radiopacosy redondos. de bordes irregulares. El mayor de ellos d~, ubicado en el lbulo superior deregh-.Y..kui...otros dos de 1,2 y~ ubicados en el lbulo inferior derecho. El diagnstico ms probable es: a) Carcinoma pulmonar de clulas pequeas / b) Carcinoma pulmonar de clulas no pequeas.../ e) Complejo primario tuberculoso d) ,Metstasis de carcing,ma e) Quistes hidatdicos
12) Cul de las siguientes opciones de Informes espiromtricos es ms sugerente d~_~;eoc? 4ia)--VE aey_%-del-tel=i~1 56% del terico VEF1/CYE:..A:Wc.~in-mejorla--con-salbutaiol b) CVF: 62% del terico; VEF1: 60% del terico; VEF1/CVF:Bao/ct:Ofi lllejotfa con salbttanro~ e) CVF: 105% del terico: VEF1.:. 96.%.del te~ VEF1/CVF~ 79%;>.sln mejorla con salbutamol

~
~

~
~
~ ~
~

d) CVF: 11 O% del terico; VEF1: 60% del terico; VEF1/CV~~~~ ;~~:F(~ ~:~~==1 K. e) CVF: 51% del terico; VEF1: 51% del terico; VEF1/CVF: &Mi;~ 1 a on s a \

seA

~
~

~v~~\~~\~~-~~.~ f14
~~~ ~ J J- C/Vlt~c., tt--J; M
,vw;,ck . \
1

~~

pv~.-,

""

~'l .. -ofc;::J_.

D .

~ ~

~
~
~

~ ~ (A
~

~
~
~

~ e)JJI1W.dllcier~Q5m~EL-

13) Una paciente de 67 aos consylta pru:.dienea. Al examen fisico destaca matidez y dismjnncin del murmullo ulmonar en la base pulmonar izquierda. Se realiza radiograffa de trax que demuestra un de ame pleural izquaer ue ocupa el torcio inferior del hemitrax. Se realiza toracocentesis que demuestra protefnas:~/dl; LDH: 30 Ul; 12 clulas por mi, de predomino mononnclear, lactato: 2mg/dl; pH:.U Proteicas plasmtica8:_r,8; 5H plasmtica: 120 Ul Cul de las siguientes etlologfas ~~es mas probable? . . 1l ~,.a._~ a) Empiema X b) Derrame paraneumnico simple v ~v-- e) Tubjrculosis d) C ncer pulmonar 1 ) Un paciente o~o, de .l_ailos, con antecedentes de ~-~li~i~d~e~mi;a~-~~;,~~;~~~ uadro de v"Qmitos y dolor ,abdominal intuRSG, diagnostlcndos 'l elevacin de la lipasa. Evoluciona con dil~.l.ll~d..res iratoria esa ur e n arterial hasta 63%, con altshnos requerimientos de oxigeno para lograr saturacin sobre 85%; Al examen pulmonar se constatan crpitos intensos, difusos y bilaterales. El diagnstico ms probable es: a) Neumonfa ~ b) Tromboembolismo pulmonar ~ e) Edema pulmonar_cardiognico

~
~

~
~

pe}Derrame pleural

~~~

~ ~

~
~
.

~
~

~
~

~
~

~ ~

~ ('"'
~
~

O~

16) Una paciente, con antecedente de insuficiencia rena~rnlca-GR-Remedilisis, loiQ~~~n cua~r9. de malestar eneral co ebre h sta 38 !Sudoracin. de 3 semanas de evolycin. Al examen ffslco es e ral, con leve taquicardia y presin arterial normal. Se observan algunas petequias conjuntivales y en los artejos, el examen pulmonar es normal y se ausculta un soplo diE~sllico IINI en el foco mitrar. En sus exmenes de sangre destaca ~~nla leve, elevaq!.n.-~ los J:~armetrouo.fl.i\m.a.torios, calda moderada en la funcin renal y elevacin discreta de las transaminasas y la bilirrubina. El diagnstico ms probable es: a) Pericarditis aguda iocardiopata dilatada ndo. ca~ditis bacteriana subaguda eucem1a e) Infeccin por VIH 1]i) Una paciente de 23 aos GQnsulta por tos, con abundante expectoracin. mayor en las maanas._ e algunos aos de evolucin. En ocasiones ha presen~~o .~l_(pectoracin hemoptolca. No fu~}! no presetaOffos sntomas. El examen fsico es normal, al igual que la radiografra de trax. El diagnstico ms probable es: ) Tuberculosis b) Cncer pulmonar

~
~

~Bronquiectasias

Vd}Bronquitis crnica e) Absceso pulmonar

~
~
~

~
~

~
~

~
~
/

~
~ ~

,-,
~
~

a Realizar 3 nebuliz tones con salbutamol cada 29 min~y enviar a domicilio con dosis mayores de orticoides orales ) Administrar oxgeno, broncodilatadores...nPulizados cada 20 minutos por 3 ocasiones, dexametasona endovenosa y enviar a domicilio con dosis?fftayores de corticoides orales e) Administrar oxigeno, broncodila'-deres nebulizados, corticoides endovenosos y reevaluar en una hora, y si presenta buena respuesta; enviar~omlcilio con dosis mayores de corticoides orales Di.Hospitalizar, administrar oxigeno, broncodilatadores nebulizados, corticoides endovenosos y reevaluar .... pertdicamente ~ e) Realizar irytubacin orotraqueal y conectar a ventilador mecnico

19) Un p,aciente con antecedente dttasma severo, con varias crisis asmticas en el ltimo mes, razn por la cual est actualmente en tratamiento con corticoides orales. CQDJWLta en urgencia ~dro de di$hea sibilante asociado a cianosis. Al exameo.fisiCj)_p.r_e.~nta J=c-:12x', eA:110nO, li'~ ci 6sis perioral ~~ Se ausc. ul~n ~.ibi.lanci~s inspiratorias Y. esplratmias, difusas. La saturacin e. po p ometra e 87%. La conducta ms adecuada es:

~
~

~
~
~

~
~

~ ~

--u~~
~(b
,;}-"
\-.9-l

-.

~
~

~q-:> r- ""

-,

~
~

...}>v!

~
~
~
Cardiologla

"j__, -

A~

t< -

IZl 1 b'S

:
~re

~ ~ ~
~

1) Paciente 58 aos, DM2, HTA, cursa con dolor retroesternal de 45 minutos de duracin, Intenso. El ECG muestra supradesntvel del ST en DI, V5 y VS. La medida ms Importante, de entre las que se enumeran, es: ~ Ariticoagulacln -~Terapia de reperfusin inmediata V e) Manejo estricto de glicemla y"presln arterial. con frmacos endovenosos d) Solicitar enzimas cardiacas de manera urgente e) Solicitar ecocardiografla de urgencia 2) Un paciente de ~s, sin antecedentes de importancia, presenta PA:rf60i11Q)en varias tomas, en dfas diferentes. En sus exmenes ~aemograma normal, creatln'tn~mgfdl, BUN: 12 mg/dl, sedimento de orina normal,~:,a mE fl Na: 140 mEq/1, Cl: 103 mEq/1 y normalidad del fn~llce de aldosterona/activldad de rentna plasm ca. 1diagnstico de sospecha debe ser: a) Hipertensin esencial b) Hipertensin secundaria a nefropatla mdica ~ro~ e)\,. 2- o('~~ / 'l..\1 e) Hiperaldosteronlsmo primario ':X '- ~A ~ .O Hipertensin renovascular e} Feocromocitoma

e
~
~

~
~
~

y..._

~
~

~
~
~
~

('A

3) Un paciente :ntecedentes de vlvula artica bicspide, endocarditis Infecciosa aguda por S. aureus, que evoluciona con rpido deterioro de la funcin cardraca, constatndose mediante ecograffa una rotura de los velos valvulares, con una Insuficiencia artica severa. Se decide reemplazo valvular quirrgico. Cul. de las siguientes opciones representa la mejor alternativa de manejo? . 1 u~ . _ a) Instalar prtesis biolgica, quedando con anticoagulacin a permanencia, sin necesidad de profilaxis ~~t ' -<,u ~.{;t.1 antibitica para nuevas endocarditis -~ti) Instalar prtesis biolgica. indicando antibiticos como profilaxis, ante procedimientos invasivos, sin ,;. .......~'-' P ~ necesidad de anticoagular @ Instalar prtesis mecnica, Indicando antibiticos como profilaxis. ante procedimi~ntos invasivos, y anticoagulacin a permanencia d} Instalar prtesis mecnica, indicando ;antibiticos como profilaxis, ante procedimientos invasivos, sin necesidad de anticoagular e) Instalar prtesis mecnica, quedando con anticoagulacin a permanencia .. sin necesidad de profilaxis antibitica para nuevas endocarditis

d~con

~resenta

~ ~
~

~ ~ (!i'

P /e-~)( 4) Cul de las siguientes manifestaciones clinicas en;ME-oosTprobable de ser hallada en un paciente con miocardiopata hipertrfica? -~ ~ ... \~ ..J!.~""' - - a) Pulso de amplitud aumentada )- c.o.-.~c..L f"'= ... c..: f4<~ - b) Ondas T invertidas y de gran amplitucl en las derivaciones precordiales del ECG \ @Desplazamiento importante del choque de la punta~ ,~ J~ ~ ~ ~le"-\ et.O..O... d) Soplo sistlico que aumenta con la m13niobra de Valsalva e) Sincopes relacionados con el esfuerz:> ..,. 5) Un nio de 7 meses ha presentado 5 infecciones respiratorias bajas desde el nacimiento. Se fatiga rpidamente durante la succin, por lo que no ha aumenta.do correctamente de peso. Al examen fislco se ausculta un soplo holosistllco intenso y algunas crepitaciones escasas en ambas bases pulmonares. No presenta cianosis. EICJiaQnstlco ms probable es: a) Coartacin artica c.- ....J. b) Ductus arterioso persistente e) Comunicacin interauricular -~Comunicacin interventricular e) Tetralogfa de Fallot 6) La taquicardia paroxfstica supraventricular se manifiesta electrocardiogrficamente como: a) Taquicardia regular, con frecuencia c3rcana a los 150 lpm, con ondas de serrucho en las derivaciones inferiores (011. 0111 y AVF) -Taquicardia regular. con frecuencia cercana a los 190 lpm y complejos QRS angostos/ e) Taquicardia regular, con frecuencia cercana a los 180 lpm y complejos QRS anchos d) Taquicardia irregular. con frecuencia cercana a los 150 lpm y ausencia de onda P 1: . . e) Taquicardia irregular, con frecuencia cercana a los 180 lpm y complejos QRS anchos y angostos alternados
,.
.... :

~
~
~

~ ~
~
~
~

~
(P'
('lt't

'.
(. '

e
: (.

'r

~ ..D

._!1

e
~

,.,
~
~
)\ 7) Una paciente de 68 aos, diabtica e hlpertensa 1 con antecodente de clrugia de revascularlzacln rt'liocrdica, hace 3 aos, en tratamiento con enalapril, aspirln!, carvedllol y atorvastatina, presenta un cuadro de SO minutos de evolucin, de dolor retroesternallntlmso, opresivo, asociado a sudoracin y disnea. El examen ffslco muestra FC: 110x' regular, PA: 140/11)0, RR3TSS, con presencia de R4 y examen pulmonar normal. El electrocardiograma muestra lnfradesnlvel del segmento ST en las derivacl.nes DI, V5 La conducta adecada es: ~J . ~CA - a) Iniciar aspirina, oxigeno y nitroglicerina y controlar con troponinas plamticas, decidiendo manejo segn 1 resultado ...,.. ~ ~ @Administrar oxigeno, aspirina y nitroglicerina y realizar coronarlllgrafla de Inmediato e) Iniciar anticoagulacin con heparina d) Realizar desflbrilaci6n elctrica e} Realizar trombolisls endovenosa de Inmediato

yvs.

ms

~t ~ S"~
~

~
~

8) La hlperkalemla severa es capaz de desencadenar arrltmla1g severas, que Incluyen bloqueo


aurfculoventrlcular y taquicardia ventricular. Qu alteracion1~s electrocardlogrficas son caracteristicas en la hiperkalemla? @ Ondas T picudas y ensanchamiento del complejo QRS b) Ondas T invertidas y alargamiento del segmento' QT e} Ondas P bifsicas y acortamiento del segmento QT d} Disociacin auriculoventricul~r y desviacin del eje a derecha e} Ondas P Invertidas y desviacin del eje a Izquierda

~
~

~
~

'\'IR.""'"' "f.-

9) Un paciente presenta disnea de esfuerzos, rtopnea y dlsn1ea paroxistlca nocturna. Al examen ffslco se observa desplazamiento del choque de la punta cardiaca al quinto espacio Intercostal, en la linea axilar anterior, se ausculta un soplo diastlico IIINI en todo el precordio y se palpa el pulso arterial y.erifrico muy amplio {pulso Celler). El diagnostico ms probable es: ( a} Estenosis artica b} Estenosis mitral -""~le ~~~ ~ (P -+ c...c:xd.:.:.-- ~.)..:Q.. ~Insuficiencia artica d) Insuficiencia mitral ~ _e) Miocardiopatfa hipertrfica obstructiva 1 O) Una paciente de 55 aos, presenta cuadro de fiebre, CEG y artralgtas de 7 dfas de evolucin, sin sintomas digestivos, urinarios ni respiratorios. Al examen SEI aprecia febril, decafda, plida y se ausculta un soplo ~iastllco IIINI, con examen pulmonar normal. El diagnstico ms probable y la conducta ms adecuada son respectivamente: a) Fiebre reumtica aguda e iniciar corticoides endovenosos - b) Fiebre reumtica aguda y administrar penicilina y aspirina en altas dosis Endocarditis bacteriana subaguda y solicitar hemograma. hemocultivos y ecocardiografia d) Endocarditis bacteriana subaguda y solicitar coronarlografla y ventriculograffa e) Endocarditis bacteriana subaguda e Iniciar ceftriaxona endovenosa

~
~

11) Paciente de 60 aos, fumador, HTA. Consulta porque hace 4 ~es inicia dolor torcico opresivo, que aparece cuando el paciente camina 150 metros, o cuando sube 6 peldaos de la escalera. Cede con el reposo, durando cerca de 5 minutos. El ECG slo muestra alteraciones inespec(ficas de la repolarizacin. La mejor conducta a seguir, junto con controlar los factores de riesgo cardiovascular e iniciar aspirina, es: a) Indicar nitroglicerina sublingual en los episodios de dolor b) Solicitar angiografia coronaria a la brevedad ~Solicitar Test de esfuerzo a la brevedad d) Solicitar un nuevo ECG e) Evaluar respuesta a cambios de estilo de vida y aspirina, en 1 mes 12) Un paciente inicia un cuadro caracterizado por vmitos..~ lipotimias y prurito generalizado, mientras coma mariscos en un restaurante. Al examen fsico presenta FC: 120x'. PA: 74/36 mmHg y se observan placas pseudopapulares generalizadas. La conducta ms adecuada es: a) Administrar corticoides endovenosos y reponer la volemia con coloides @ Administrar adrenalina intramuscular y suero fisiolgico endovenoso - - .t o.- f 1 0 e) Administrar suero fisiolgico y antibiticos endovenosos '~' ' :!> c:a-- P d) Administrar un litro de suero fisiolgico y reevaluar peridicamente " e} Administrar sales de rehidratacin oral y reevaluar peridicamente.<

~
~
-~

~
~

~
~

b...

~
~

~
~
-~

~
~

~
~

~ ~

f'
~
~

~ ~
~

13) Un paciente de 57 aos, sedentario, inicia bruscamente dolor torcico Intenso, que aumenta con la inspiracin y se asocia a disnea. Al exCimen fislco slo se observa edema marcado de la extremidad inferior Izquierda, con dolor a la palpac:ln muscular de la pantorrilla, sin otros hallazgos patolgicos. El examen que con mayor probabllldacl entregar el diagnstico es: a) Angiografla femoral b) Dfmero D ~Anglo-TAC de trax d) Electrocardiograma e) Radlograffa de trax

~
~

?( -Jt...._

'f....

~ ~ ~ ~
~
(~!:

14) Un hombre de 76 aos presenta varios smcop.e.s en las ltimas horas. Al examen fsico est soporoso, hlpoperfundldo, tiene una frecuencia cardiaca de 341atidos por minutos y una presin arterial de 86/48 mmHg. Se ausculta un primer ruido cardaco de Intensidad variable. El diagnstico ms proba6le es: <W>Btoqueo AV completo S k>c:...k.. c.o.-..~ o~ V\.~ .,.. b) Enfermedad del nodo sinusal ~--4~~ Q... e) Sincope vasovagal d) Sfndrome coronario agudo e) Fibrilacin auricular ,.:. 15) Cul de los siguientes medicame111tos Incide MENOS en el pronstico de una insuficiencia cardiaca crnica, debida a cardiopatia coronarla? ~ ..-;::::::. ~ Furosemida ~ . . ~..:~-L.\- <.,.p., -..: .k t" .. t=:L- v=..... b) Aspirina N o \ t,.c.~ ..k.. <) e) Enalapril v d) Atorvastatina e) Carvedilol./ 16) Un paciente de 66 aos, hipertens<J, presenta doior-retroesternallntenso, irradiado a dorso. En el examen fsico presenta FC: 127x', PA: 200/120 mmHg, Sat02: 94%, pulsos radiales asimtricos, MP(+), sin ruidos agregados, RR2T con sople' de regurgitacin artica. El diagnstico ms probable es: a) Infarto de ventriculo derecho b) Endocarditis aguda .+,: p..o -A. . e) Tromboembolismo pulmonar d) Infarto miocrdico con rotura de msculo papilar ~ Diseccin artica 17} Un paciente sufre un paro cardiomspiratorio e.n la sala de urgencias, donde usted trabaja. Se inician ventilaciones y masaje cardaco y se Instala monitor electrocardiogrfico. Luego de 5 ciclos contina sin pulso y el monitor demunstra actividad elctrica, con complejos QRS a una frecuencia de 30 por mnuto. La conducta ms adecuada es: A:)... ~e~~ ~~"" \<>-' a) Realizar desfibrilacin elctrica y continuar el masaje cardiaco b) Realizar intubacin orotraqueal, iniciar- ventilacin a presin positiva, con oxigeno al100% y continuar masaje cardiaco . '(5) Administrar adrenalina 1mg por vi a endovenosa y continuar con el masaje cardiaco d) Instalar marcapasos externo y suspender el masaje cardiaco e) Realizar 5 ciclos ms de masaje cardiaco y reevaluar 18) Mujer, 60 aos, con estenosis mitral severa, inicia bruscamente disnea y palpitaciones. Al examen FC:140x-, PA:BS/35, MP(+}, con crpit()S intensos y difusos, Rl2T con soplo diastlico IV/VI, mayor en pex. La conducta inicial ms adecuada es: a) Administrar amiodarona endovenosa :m bolo --{ ~....; Ul"' d:a.. b) Entregar oxigeno a Fi02 elevadas y soporte ventilatorio con Bipap e) Reponer fluidos con suero fisiolgico {-t b. J " -'\ ...... .lo..~ . @Cardioversin elctrica inmediata e) Administrar furosemida endovenosa, ;3sociada a drogas vasoactivas ~j . A c. . . ., A,

~ ~ ~
~

~
~

~
~
~
~~

1/7-o

~ (A ~
~

~ ~ ~ ~
~
~

~ (!A

~
~

~
~

~
19) Una paciente de 46 aos presenta un cuadro de astenia y edema de EEII, progresivo, de 3 meses de duracin. Al examen destaca hepatomegalia, edema blando de extremidades Inferiores e Ingurgitacin yugular, con colapso "x" prominente en el pulso venoso. El examen pulmonar es normal. El diagnstico ms probable es: .:t. c._ ~c.,~c... a) Pericarditis aguda "" ( ~~ ! ) - b) Mlocardiopatla restrictiva ( ~.,. c. ~ ) e) Miocardiopatra .Qilatada ~ jv"'- ~-.\,R. c..!~ 1 ~ =..1t> 1-4Z.A. ~- ..\-'..., 'a) Cardiopatla hipertensiva o ;l)Pecarditis constrictiva bS ' -c::c.c COA c1...! 0 \.Y ~~ ~.a...:~ :Lc.~..._:::o-. 20) Un paciente fumador, dlsllpldmlco e hlpertenso, en tratamiento con enalap~ll y atorvastatina, con buen control de su presin arterial y de sus niveles llpfdlcos, presenta dolor en la pantorrilla derecha, que aparece al caminar cerca de 150 metros, limitndole la marcha y cede con el reposo. El examen ffsico no aporta mayor Informacin. La conducta ms adecuada es: a) Realizar anglograffa de EEII b) Solicitar ecograffa doppler de EEII >< ~ Iniciar ejercicios. suspender el cigarrillo e Iniciar aspirina d) Suspender el cigarrillo e iniciar antlcoagulacln para alcanzar INR entre 2 y 3 ..,... e) Resolver quirrgicamente ....
j '"'-"'

~
~

~
~

~
~ ~
~

~
~
~

~~~~~ ,.:"""'~~-L--..4
f\..c-.A. ~ ~ ~d:
(.o

~
~

~
~

~
~
~
~

~
~

~
~
~

,_,
~
~

~ ~

-,

/'

(~ ~~~ ~~~
t.s
rl\.~
~

EBA 3 Preguntas Cardiologa n un adolescente de 17 aos la presencia de un soplo sistlico de eyeccin asociado a _ aysaoblamiento fijo del segundo ruido permite sospechar el diagnstico de: ~unicacin interauricular b} ductus art~rioso persistente K e) comuni in interventricular d) vlvul artica bicspide e) coa acin artica 2) n Chile, las principales causas de illauflclencia cardiaca son: Cardiopata hipertensiva y valvulopatias /111~rdiopatia coronaria y miocardiopatia dilatada ~rdiopata coronaria y sarctinpatla bipe1tensiva d} cardiopata hipertensiva y miocardiopaua dilatada e> Cardiopatia coronarla y valvulopatlas

~'::!

ce~ ~Y(
~((V ti_\)~~-t:=:::.\~~

(9~

\S

)) ~ ~\\

3) Paciente de 30 aos, sin antecedenteu cardiovasculares de Importancia. Consulta por disnea de esfuerzos, ortpnea y DPN de instauracin progresiva...s, desde hace,_meses. La RxTx muestra cardiome alia i nos de con sti ulmonar. Usted solicita ecocardiografa que muestra dpatacin de todas las cavidades, con F, :50~ Las medidas farmacolgicas ms importantes para este paciente (4?A
<~rticos, IECAs y aspirina

V ::_}

son:' --a) ~rina, estatinas y betablequear:~tes

\ f'-J -f -..

~ ~

t
..

0 ~~
YO

-.

(!;
~

~
~

Betablo uean s d' se lEC d) Esp1ronolactona, tECAs y aspirina e) Amiodarona, betabloqueantes y diurticos

L
p.../\') ~

l_

\.. c(/-\-

1..

.J ~ 1"'-

-l-Lf\.
.

~
~

.,

~
~
~

. X

u'er 53 con disnea de esfuerz()s progresiva, de 2 aos de evolucin, actualmente CFIII. ~ e x ectoracion osad en algunau ocasiones. Al exame~:12ono, FC:102x', RI2T, soplo tthastlico)l,_.......,. rpitos 1 ~~s. El EGG, slo demuestra ~Cxfru El diagnstico ms probable es: a) Cardiopatia hipertensiva. b) TEP-crnico e) 'Jatada --..,d Estenosis mitra! >-> f .../ ~ ..0 .r U.. ~ . ~ e 1C1enc1a ao 1ca rl f~ \..~ i o-J~ l CA r cl..A l--ILJ)) o.JL..A-..o 5) Paciente hipertenso, de~ cico intenso, de inicio sbito. Al examen ffsico FC: ~, tft9.Y.lar, PA:230/12&~imetrfa en los pulsos radia e y a la auscultacin _cardiaca destaca un soplo diastlico. La conducta fnicial ms~ adecuada a segur, adems de tomar un ECG es: ealizar coronariografia urgent~ tninis.trar oxigeno, morfina, nitroglicerina y aspirina inmedi~tamente c...10 fA-O ealiz r ecocardio rama transesofgico urgente _ d) Rea 1 ar pericard1ocen es1s e) So 'citar radiografa de trax 6)

~~~~ r . 6-J.-L

'UJ ~

.l.... ~"- ..

~
~
(i'l

_ j

'\::1--

~
~

~
~

~ ~ ~
~

;<

ujer de 45 aos, hipertensa, consul~t por palpitaciones de 3 das de evolucin. Ingresa en buenas ndiciones, PA: 1.60/100, FC:102x'. El ECG muestra fibrilacin auricular. La conducta inicial ms --adecuada es: a) Realizar cardioversin farmacolgica con amiodarona Y . b) ~ealizar cardioversin elctrica~ ~ciar anticoagulacin inmediata con heparina .- d) Iniciar betabloqueo a permanencia, como control de frecuencia e) Indicar digoxina a permanencia,<"

fl'
~
~

7) Paciente de 59 aos, HTA, DM2, fumadlor, refiere dolor en pantorrilla derecha al caminar 2 cuadras, que le impide seguir caminando y que cede con el reposo. La conducta ms adecuada ante este cuadro clnico es: ~ Ejercicios Qiarjns y control mdjco de sus factores de riesgo cardiovasc !lar b) Solicitar angiografia femoral p~ra ubicar sitio de obstruccin e intentar angioplastia e) Iniciar anticoagulacin oral con cumarinicos d) Derivar a cirujano vascular para ciruga d'~ reperfusin de EEII ~icitar ecografa-doppler de EEII

t"'
~
~

aos con palpitaciones muy Intensas de inicio y trmino brusco, de arias oportunidades en los ltimos aos. Ingresa a urgencias por cuadro de palpitaciones Intensas ue esta vez no ha cedido. Al examen: FC:180x', PA:110no. El ECG muestra taquicardia regulara ORS angosto. La primera medida a seguir es: a) Administrar ad~nosina endovenosa b) AdministrarJ.rhiodarona endovenosa ~) _!3ealizar ardioversin elctrica / ~aliz maniobras vagales ._/ e) Admi 1strar propafenona o flecainida oral
e 9) cardiopatfa congnita ms frecuente es: Comunicacin interaurlcular \ _ omunicacin interventricular ()\~ e) Ductus arterioso persistente ...\ d) Derenaje venoso anmalo e) Tetralogia de FStllot

~a:~O

mlnuW~I~~ ~~~
\.Y
~

~~

~~

(( '\ _ ~

10) ....79 aos, consulta por tinitus y mareos. Al examen: RR2T, aumept6 de la intensidad de~2, auscultacin pulmonar nol"!"al, sin signos ~es en el examen rlgico. El frmaco de eleccin, para el.~anejo de esta situacin, es: ' edipino sublingual ptopril oral e) Nitroglicerina sublingual d) Nitroprusiato endovenoso e) Enalapril oral 11) Mujer, on estenosis mitral severa, inicia bruscamente disnea y palpitaciones. Al examen FC:140x', PA 5/35 P(+), con crpltos intensos y difusos, RI2T con soplo diastlico IVNI, mayor en ._::, pex. La conduc inicial ms adecuada es: . a) Administrar amlodarona endovenosa en bolo ~tregar oxigeno a Fi02 elevadas y soporte ventilatorio con Bipa p ~~ poner fluidos con suero fisiolgico ~ ~)Ca~rsi~el9tricalnmsQjata~~U;~ e~ t~PN \. e) Adm1mstrarlrosemida endovenosa, asociada a drogas vasoactivas_n . l t...l-,_".

Paci~ted;

c~falea,

~!Qu3o, F~:99x'.

so=-

.y-nr
~\T~v

th0-U.
J_

f'\ ,

-7 {V'e-~
~
J

12) Pa i~nte 58 aos, DM2, HTA, cursa con dolor retroesternal de 45 minutos de duracin, intenso. El EC uestra supradesnlvel del ST en DI, V5 y V&. La medida ms Importante, de entre las que se e meran, es: ) Anticoagulacin era ia de re diata e) ManeJo estricto de 'glicemla y presin arterial, con frmacos endovenosos d) Solicitar enzimas c~rdiacas de manera urgente e) Solicitar ecocardiografia de urgencia

~~(..t(Jr(.)V'-1

\.~ 1

L> c.-o~~

13) Paciente diabtico de 66 aos, evoluciona con herida en maleolo externo, de 3 cms de dimetro, sin tendencia a la curacin, dolorosa, con bordes irregulares y escaso eritema circundante. Presenta adems disminucin de la sensibilidad de ambos pies. El diagnstico ms probable es: , . / ~- 91cera por presin A 7 b) Ulcera venosa ~) lcera a , rial J) lcer eurognica

eet1n-bi~

Paciente de 15 af\os, cursando con herida penetrante torcica por arma blanca, evoluciona con ipotensin marcada e ingurgitacin yugular. Al examen MP(+) simtrico. La conducta inmediata a seguir es: a) Realizar ecocardiograma b) Reposicin de fluidos por via perifrica~ealizar pericardiocentsis- dfi~ealizar radiografia de trax e) Solicitar electrocardiograma

e
15) Paciente de 60 aos, fumador, HTl\. Consulta porque hace 4 meses inicia dolor torcico opresivo, que aparece cuando el paciente camina 150 metros, o cuando sube 6 peldaos de la escalera. Cede con el reposo, durando cerca de 5 minutos. El ECG slo muestra alteraciones inespecficas de la repolarizacin. La mejor conducta a SE!guir, junto con controlar los factores de riesgo cardiovascular e i_~!_~iar aspirina, es: -./' ~dicar nitroglicerina sublingual en Jos t3pisodios de dolor b) Solicitar angi'ofafa comReFia a la bre,TedaEJ.. ~J.}-{ D ~ \.. . a ad .A~..ro--->VJ~) o) Solicitar un nuevo ECG e) Evaluar respuesta a cambios de estilo de vida y aspirina, en 1 mes

~ ~ ~

:-o ?

JJ'---

'-/ -f

16)

~ ~
(!A

~ ~ ~
~

iro~olactona, atenolol, aspirina y eutatinas. Evoluciona con obnubilacin, asociada a bradicardia 40x' e hipotensin. El ECG muestra complejos QRS anchos y ondas T picudas en todas las lvaciones. En sus exmenes destaca Na+:147, K+:8,0, Cl-:111. La primera medida a seguir es: rdioversin elctrica urgente ----:A rar luconato de calcio endove:noso e) Iniciar insulina e a, asociada a betaagonistas inhalados d) Indicar marcapasos externo e) Realizar dilisis de urgencia
/

R~Jente DM2, IRC, en hemodilisie;, portador de ICC, de origen coronario, usando enaJapril,

~
~

Ante la sospecha clnica de TVP de extremidades inferiores, el medida diagnstica de eleccin es: lebografia de EEII '~ngioTAC de EEll ffl_COgrafia doppler de EEII d) Pletismografia de volumen e) Angiografia de EEII 18) Paciente hipertenso, de 70 aos. Ha, sufrido 3 sincopes, relacionados al esfuerzo. En el ltimo tiempo se agrega angina de pecho al clmlnar 1 cuadra. Al examen: soplo sistlico ~yectivo WNI. El diagnstico ms probable es: a) Cardiopata coronaria b) Cardiopata hipertensiva e) Mi cardiopatia hipertrfica no obstructiva d iocardiopata dilatada con insuficiencia mitral secundaria ~~te!J.OSis. artica 19} Paciente cursando con IAM con infr.~desnivel del segmento ST en 011, 0111 y aVF. Evoluciona con hipotensin e ingurgitacin yugular. Al examen FC:92x', PA:82/48, RR2T sin soplos, sin signologa ~ / pulmonar. La conducta ms adecuada es: X a) Realizar pericardiocentesis b) Indicar trombolisis de urgencia ~dministrar cristaloides y dopamina por vi a endovenosa d) Solicitar .ecocardiograma e) Solicitar derivaciones precordiales derec:has 20) aciente de 55 aos, hipertenso y dl.abtico, sufre infarto miocrdico no transmural, hace 15 dias. recupera bien, con buena capacidad funcional y fraccin de eyeccin normal, en la ecocardiografa. e realiza un perfillipdico que muestra LDL:120, HDL:45, TG:250. La conducta ms adecuada para el ~ manejo de sus lpidos es: ~dicar dieta y ejercicios ~~ciar at01vastatina "\(1, e) Iniciar gemfibrozilo ~\) d) Iniciar una asociacin entre un fibrato y una estatina e) Su perfil liidico es normal, por lo que n9 HS necesario indicar ninguna medida terapetica

~
~

~
~

~
~ ~
~

~
~

~ ~
~

'v

~
(fP'e
~

~
fl"'
(i"t

f':

(!"

~.
CARDIOLOGIA
1.6

~ ~
~

Cul de los siguientes tratamientos seria recomendable en un paciente de 60 aos con insuficiencia artica severa'!

a) Slo bloqueadores de enzima de conversin


~ecambio valvular artico

e) PIasta con baln d) Pl~ia quirrgica e) Betabloqueadores.

~ ~

30 Cul de las siguientes patologas se asocia a bajo riesgo de embolia sistmica'!


a) Fibrilacin auricular b) Estenosis mitra! e) Infarto anterior y apical

e
~
~ ~ ~
~

d) Miocardiopatia dilatada
~suficiencia artica reumtica

------------------ - - - - - - - - - -

~
~

32 Mujer de 32 aos, con antecedentes de soplo en la infancia, 1-efiere presentar disnea de medianos esfuerzos desde hace un mes, cunsult-.1 pur edema d~ ~xtremidades inf~iiores m la ltima semana. Examen fsico: pulso ir1eguJar de 14;~ por min y soplo hoJosistlico deJa punta grado 4. El diagnstico ms p1obable es: a) Fibrilacin auricular crnica awicular paroxstica e) E>..1rasistola ventricular frecuente d) Fib1ilaLin ventriculm e) Flutter auricular
------------------------~----------

~~btilacin

~
~

~
~

~
~

~
~

49 Hombre de 70 aos, portador de insuiticiencia tenal crnica y enfermedad coronaria estable. Est en un programa de dilisis peritoneal ctnic:a y recibe atenoloJ, furosemida e isosorhide por va oral. Ingresa para reparar una hernia inguinal. La tarde anterior a la ciruga presenta paro cardaco. El ECG muestra asistolia en tres derivadon~. RKi~ m~aje cardia~o y ventilacin por 20 min; tres dosis de adlenalina 1 mg IV, y dos dosis de atropina 1 mg rv. No hay cambios al ECG. Cul debiera ser la conducta siguiente"!
a) Aplicar tres desfibrilaciones elctricaH de .200 W/seg.

~
~

b) Administrar amiodarona 150 mg IV en goteo en 1Omin. e) Administrar adrenalina en dosis alta 5-15 mg IV. pr-suspender los esfuerzos de resucitacidn cardiopulmonar. e) Realizar pericardiocentesis para drenar pericarditis urmica.
-------------~~----

e
~
~

51 Hombre de 54 aos que consulta en Servicio de Urgencia por dolor torcico de dos ho1as de duracin. Ingresa a la Unidad Coronaria donde :1e demuestra un infarto de miocardio en cara nferolateral (CPK 900 U/1 para un normal hasta de 250 U/1), sin ondas Q. El ECG evolutivo demuestra inversin de onda T. Se mantiene bemodinmitamente ~stable, sin insuficiencia cardaca, durante toda la hospitalizacin. Una semana ms tarde, se practica un test de esfuerzo y el paciente no presenta angina ni nuevos cambios al ECG. Cul de las siguientes drogas es la ms til para seguir tratando a este paciente'!
~rAcido

acetilsalicilico

~
~

b) Beta bloqueadores

~
~

~
~

~
e f lnhibidores de la enzima de conversin d) Anticoagulantes orales r;J Isosorbide oral

~
~

~ ~
~

320

Un paciente de 50 aos, hipertenso, inicia cuadro de dolor tordco con intensidad mxima inmediata, hTadiado al dorso y a ambos brazos. Al examen hay un soplo de insuficiencia artica. La plimera posibilidad diagnstica es:
a) infmto miocrdico ~seccin artica e) embolia pulmonar d) neumotrax espontneo e) espasmo esotagico difuso

~
~ ~

4
~

321 El masaje del seno carotdeo disminuye abruptamente la frecue11cia cardaca en las arritmias causadas por:
a) taquicardia sinusal. b) fibrilacin awicular paroxstica ~--taquicardia supraventricular por reentrada. d) taquicardia ventricular. e) taquicardia nodal

~
~ ~
~

~
~

322 En un paciente con hipertensin arterial esencial grado I, la p1i1:nera etapa de tratamiento consiste en:
a) restdccin de actividad fisica b) betabloqueadores. e) diurticos tiazdicos. ~gimen hiposdico. e) ansiolticos.

~
~

~
~
~

323 Hombre joven con una helida penetrante en l~ c~ra anterior del trax 1 cm medial al pezn izqui~rdo, ingresa agitado al Servido de Urgencia. Su presin a1-terial es de 70/50, prestnta disttnsin de las venas del cuello y ruidos respiratorios normales bilateralmente. La conducta inmediata a seguir es:
a) solicitar radiografia de trax b) solicitar electrocardiograma e) realizar drenaje pleural ~eal.izar pericardiocentesis e) realizar intubacin traqueal

~
~

~
~
~

~
~
Al)

324 Cul es la conducta ms apl"opiada a seguh en un hombl"e de 70 aos, aslntomtico y en buen estado, con un aneurisma de la aorta abdominal de 8 cm, descubierto por ultrasonografa"?
a) Realizar tratamiento hipotensor b) Realizar arteriografia translumbar ante cualquier signo de complicacin ,fl-Derivar a ciruga vascular, con vistas a una inteiVencin quirrgica. d) Control semestral con ultrasonido y derivar en caso de aumento de volumen.

~
~

..-,
~
~

.-,

e
b) Captopril e) Candcsartan
~ametildopa

~ ~

fA
~
~

f:'
~

e) Hidralazina
1701 En un paciente con infarto agudo del miocardio de la pared Ierior, aparece bradicardia de 40/min e hipotensin tras la administracin de nitroglicerina Cul de los siguientes frmacos debe indicarse de inmediato por via endovenosa?
a) Digoxina
~opina

~ (!;

e) Isoproterenol

e
~
~
~

d) Lidocaina e) Verapamilo
-------------~----------------------

1703 Cul de los siguientes frmacos se asocia a fibrosis pulmonar?

fA
~

a) Quinidina b) Sotalol e) Propranolol d) Propafenona

~odarona

~---------

"'
(!'
~

1705 Cul de los siguientes medicamentos e;t contraindicado en el infarto del miocardio reciente?
~ifedipino

~ ~

b) Enalapril e) Lidocnina

d) Hepa.rina e) Tiazidas
1706

~ ~
~
~ (A

-----------------

---------~-~---

Un paciente ingresa con una FC de 150/:min, regular. que con masaje carotdeo desciende bruscamente a 75/min, regular, retomando la frecuencia inicial a los pocos segundos. Cul es el diagnstico ms probable? AifFlutter awicular
/"b) Fibrilacin auricular e) Taquicardia supraventricular por reenfrada en el nodo aurculoventricular d) Taquicardia supraven1ricular por reentlada en el haz de Kcnt o accesorio e) Taquicardia ventricular
1707

""
~

~
~

'!'

El aumento de intensidad en la auscultac.in de un soplo cardiaco con la maniobra de Valsalva es propio de:
a) la estenosis artica. 41-bjfa miocardiopatia hipertrfica obstructiva. e) el prolapso ,,alvular mitral. d) la insu.ficienr..ia tricuspidea. e) un soplo inocente.
1710

e
~

""

Qu pacientes con prtesis valvular mitra! mecnica tienen indicat.-in de tratamiento anticoagulnnte oral permanente, con derivados cumarinicos? a) Aquellos con fibrilacin auricular t.-rnica b) Aquellos con antecedentes de cuadros emblicos

e
~

~
~

~
~

~...(o'dos. a+n cuando tengan ritmo sinusal


d) Aquellos que presentan crecinento aw'icular e) Los mayores de 70 aos
---------------~--------"""--~-

~
~

1717 Paciente de 3 7 aos. hipe1ienso de diagnstico reciente. Actualmente con PA de 140/1 00 mmHg. Exmenes de laboratorio: o1ina con pH 5.2 y sin albuminw'i~ electrolitos plasmtic:os Na 140 mEq/1, K 4,.0 mEq/1 y CliOO mEq/1, bicarbonato 25 mEq/1, glicemia 90 mg/dl, creatininemia 0.8 mg/dl, calcemia 9.0 mgldl y uricem.ia 5.0 mgldl. Cul es el diagnstico ms probable?

~
~

,..,
~
~

a) Sndrome de Cushing b) -1Jj.osteronismo primario ~pcrtensin esencial d) Nefritis crnica e) Estenosis arterial renal bilateral
1782 Cual de los siguientes fnnacos reduce la mortalidad en los enfermos con insuficiencia cardiaca

..,
~
~
~
cong~1iva?

~alapril
b) Digital e) Furosemida d) Procainamida e) Acido acetilsalicilico
1785 Cul es la causa ms fiecuente de fibrilacin auricular crnica en un paciente mayor de 70 aos? a) Hipertiroidismo b) Hipokalemia ~.-ardiopata hipmtensiva d) Cardiopata valvular e) Idioptica 2006 Hombre de 55 aos presenta dolor retroesternal intenso irradiado al dorso, de 4 horas de evolucin. Al examen se encuentra asimetria de pulsos en extremidades superiores. Electroc;ardiograma normal. Cul es el diagnstico ms probable? A-Diseccin artica b) Infarto agudo al miocardio sin supradcsnivel ST e) Neumotrax a tensin d) Embolia pulmonar e) Pericarditis aguda

"'
~
~
~

~
~

~
~

~
~
~

~
~

----------

2023 Paciente con crisis de taquicardia paroxstica supraventricular sin compromiso hcmodinnco, en que se sospecha un sndrome de WolffParkinson Whitc. Cul es el tratamiento de primera linea?

~
~
~

a) Lanatsido e ~erapamilo e) Amiodarona d) Propnnolol e) Cardioversin elctrica 2025 Mujer de 32 aos, activa, con antecedente reciente de hemoptisis, pn~senta henp!lresia facio braquio t.'Tllral izquimda. Al examen se detecta arritmia completa.

~
~
~

~
~

..,

~.

~
b) El pacicnt~ es diabtico iru;'Ulina dependiente.

f'

,..

e) La CPK-MB no est alterada a las 3 horas de iniciado el dolor precordial. d) El paciente presenta una hipertensin arterial de 165/95 mmHg. ~~ electrocardiograma revela infradesnivel de ST de 2~5 mm.
---~----------------

('
938 En un padente hospitalizado por infarto miocrdico, la aparicin de un tercer ruido sugiel'e:

~ ~

~ ~ ~

a) ruptura del septum interamicular. b) embolia pulmonar asociada. 4fnsuficiencia cardiaca inminente. d) edema pulmonar agudo. e) reinfarto.

r:
~

942 Cul de las siguientes situaciones requiere indicacin de atropina en un paro cardiorrespiratorio?

~
~1

t!"
~ ~

a) Taquicardia ventricular sin pulso. vagal. e) Fibrilacin ventricular por infarto agudo al miocardio. . d) Pisociacin electromecnica con bradicardia extrema. e) Falta de respuesta al masaje extetnCt.
~sistola por reaccin

~
~

1680 El principal determinante del pronsti<;o a largo plazo de un paciente diabtico e lpertenso que present hace

un mes un infarto del miocardio de pared anteroseptal es:


a) los hallazgos de la corona.riografia.
.-~1 ~1ado de la funcin ventricular.

e
~

e) el grado de control de sus factores d~ tiesgo.

~ ~
~ ~ ~

d) la presencia de angina. e) la presencia de eA1rasistola ventricular.


1683

El uso de cul de los siguientes medicamentos se asocia a una mayor sobrevida en pacientes con insuficiencia cardiaca congestiva?
a) Digitlicos b) Dimticos

~
~

e
'!!'
~

"--q..Beta bloqueadores d) Bloqueadores del calcio e) Antiarritmicos


-------~--

~ ~

1684 Un hombre de 63 aos consulta en urge:Clcia por presentar desde hace 55 minutos palpitaciones sostenidas~ rpidas e irregulares. Cul es el diagnstico ms probable?
u) Taquicardia paroxstica supraventricular J>rFibrilacin auricular paroxstica e) Extrasistolia ventricular polimod'a d) Torsades de Pointes (torsin de las puntas) e) Bloqueo auriculo-ventricular avanzado

e
(!!'

1685 Cual de los siguientes tnctores se asociEL a menor mortalidad hospitalaria en casos de infw.to del miocardio'7

e
~

~ ~

e_

a) Sexo femenino b) Presencia de diabetes mellitus ~) Localizacin en la pared anterior d) Taquicardia sinusal A1fdad menor de 45 aos
----~------

-- -----------------

-----------------~r~---~---

1687
El diagnstico de angina inestable se fundamenta en:
~~historia clnica

b) las anormalidades del examen fisico. e) los niveles de enzimas cardiacas. d) las alteraciones elecb:ocardiogrficas. e) los hallazgos de la coronariografia.

--- --------Cul es el antibitico de eleccin para la profilaxis de endocarditis infecciosa en un paciente que va a ser
1688
sometido a un procedimiento dental? a) Gentamicina b) Azitromicina e) Clindamicina ~oxicilina e) V ancomicina

1690
Mujer de 45 aos con antecedentes de disnea de esfuerzo, palpitacion~s y hemoptisis> que en forma brusca presenta accidente vascular cerebral. Cul es la patologa de base ms probable? a) Hipertensin pulmonar .b}::l!'stenosis mitra! e) Estenosis artica d) Insuficiencia mitral e) Comunicacin Interventricular

1693
Cul es el diagnstico ms probable en un paciente con compromisCJ del estado general, hipotensin, pulso paradjico y yugulares distendidas? a) Tromboembolismo pulmonar b) Pericarditis aguda e) Derrame pcricardico _,Qy'aponamiento cardaco e) Tetraloga de Fallot

1694 La elevacin mxima de las enzimas cardacas antes de las 12 horas post trombolisis en el infittto del miocardio,
es sugerente de:
a) infarto extenso. et"ectiva. e) infdrto complicado. d) pericarditis asociada. e) trombolisis fallida

~ombolisis

1700
Cul de los siguientes tiinnacos es el de primera eleccin en la embarazada con hipertensin arterial crnicn?
a) Nit"edipino

~ ~

~ ~ ~ ~ ~ ~ ~
~

e) Controlar clnicamente y derivar slo en caso de complicaci6n.

358 Un hombre de 53 aos consulta por cefalea, tinitus y mareos de pocas horas de evolucin. Seala no tener antecedentes de hipertensin arterial Se constata presin arterlal.de 220/130 mm.Hg con pulso de 80/min, regular. El resto del examen fsico es normaL De los siguientes tratamientos, t,tul es el ms adecuado?

~ ~ ~
~

a) Nitroglicerina endovenosa b) Nifedipino sublingual e) Nifedipino oral d) Furosemida endovenosa ~aptopril oral

895.

------J----

~ ~
~ (!A
~

El tratamiento de eleccin para una j)aciente de 28 aos con lnsutlclencla mitaal grave, sintomtica, de etiologa reumtica es: ' a) enalapril. b) diurticos y digitlicos. e) valvuloplastia mitral percutanea con baln. ~gia valwlar mitral. e) terapia an.ticoagulante.

~
~

897
Cul de los siguientes frmacos es el ms eficaz para aumentar la sob1evida de los pacientes con insuficiencia cardaca severa?

~
~

~ ~
.~

,....~SIOqueadores Beta. ...b) Digitlico~ e) Diurticos.-... d) Bloqueadores del calcio. e) Acido acetilsalicico
-------'---~----

~
~

898

~ ~

Hombre de 63 aos con antecedentes de soplo sistlico detectado hace 8 aos, consulta por haber presentado un sncope durante ejercido intenso. Cul de las sigUientes es la causa mn probable del sncope?
u) Origen vaso-vaga! b) Bloqueo awiulo-ventriculor (;) Inforto del ~oeardio -d) Insuficienciamitral ~te.osis artica

.~ ~

e
~

899
Mujer de 25 aos consulta por presentar. desde hace 2 aos crisis de palpitaciones rpidas,~ minutos de duracin~ sin factores desencade~nte1i. ~~ re'"~~e otros sntoin~s y J~va una vida no~aL . ..E~ ~ain:s~ico prob_able es: . . ;_'..: .~---- .. . - ._c. . _. . , .

e
(!'A

:~
:~

mas

.,

.....

....

",

.,

.,
.,

''"

~
~

~
~

~
,#aquicardia paroxstica supraven.tricular

~
~
~ ~
~

900
:tn un adolescente de 17 aos la presencia de un soplo sistlico de eyeccin asociado a desdoblamiento fijo del segundo ruido permite hacer el diagnstico de: - .

~omunictl()jn interawicular
b) duetos art~oso persistente

~
~
~ ~

e) comuriica.Qin intetVentricwar
d) vlvula a~ca bit.-spide e) coartacin artica

901
Qu examen debe solicitarse en plimer luga1 a un homb1e de Sfi: aos ret1oesternal de 45 minutos de duracin? a) Radiogmtia de trax b) Gases en sangre arterial e) Tomografia axial computada de trax d) Niveles d~ troponina .fi"Electrocar;diograma
qu~

~
consulta por dolo

~
~

~
~

906
Cul de la,. slgulentes combinaciones de medicamentos, puede producb hlperkalemia de elevancla clnica en pcientes con insuficiencia cardaca?
a) Digo~ enalapril b) Hidroclorotiazida, losartn e) Nifedipino, enalanil

~
~
~

~
~

d) Fwosemida, espironolactona
.,.erLosmtn, espironolactona

~
~

911 Cul es el mejor examen para confirmar el diagnstico de embolia pulmonar?

a) Presin parcial de oxigeno en sangre arterial b) Radiografia de trax e) Cintigrafia pulmonar por perfusin e inhalacin d) Electrocardiograma ~.Artetiografia pu1monar

,.,
--------

~
~

~
~
~

921 La identificacin de los extrasstoles ventriculares en el electrocardiograma se basa en que:


~el

-----..-----

QRS es ancho b) las ondas P tienen diferente morfologa que los complejos sinusalt~s. e) la duracin del segmento PR es variable. d) no tienen pausa compensadora. e) la onda U es prominente

,
~

~
~
~

922 Cul de las siguientes situaciones contraindica la trombolisis en un infarto del miocardio? a) El dolor prec~dial tiene ~de 1 h9ra de ev~lucin.

,.,
~

.., ..,

..

:::.

. .

~ ~

1) Un paciente de 67 afios, diabtico, en tratamiento con metfonniria y glibenclamid, mal controlado, presenta episodio actual, caracterizado por sudoracin y paffdez. Ef examen fsico no aporta mayor informaci~. Se solicita hemogluco1test, que resulta 233. La conducta ms adecuada es:

f!'

~ ~

a) Enviar a domicilio, aumentando la dosis 'de hlpogllcemfantes orales b) Iniciar insulinoterapia y agregar una estatlna al tratamiento ..-'i),Solicitar un test de tole~al'lcla a la glucosa orar; prolongado Lo)-Solicitar electrocardiograma "EfrSolicitar un ecocardiogarama

2} Un paciente con ant~cedente de f:POC tabiquleo, severo, de 10 a~os de eVG{UCin. ecnsulta por
cuadro de astenia y mayor disnea da la h~bitual, asociada a edema de extremidades infeores y hepatomegalia. Actualmente no fuma y sigue tratamiento con broncodllatadores y corticoldes Inhalados. El examen fsico no aporta mayor tnformact6n. El diagnstico ms probable es:

~ ~ ~

t.

a) Cardiopatra coronaria < b) Fibrosis pulmonar ,1 fijCncer ~ or pulmonar

~ ~ ~

\.

l4ipertensl6n pulmonar

~
~

-1 La hlperkalemta con fretuencla, .iWargamiento del segmento QT 'O) QRS angosto ji..e) Ondas T picudas . d} lnfradesnivel del segmento ST e) Ondas T invertidas

se rnan1flesta electrocardlogrffcametit como:


_

4) La conducta ms ;~decuadaante un paciente dlagnosticaao de neumonfa AiS 1, que no responde al


tratamiento con amoxlcilina oral, luego de 72 horas, es:
~Mantener el tratamiento con amoxfcillna y reevaluar en 24 horas ~~Y.niclar tratamiento con amoxicilina + cido clavufnico ~. crHospitalzar y solicitar radtograffa de trax

~
~

d) Solfcitar cultivo de expectoracin con antibfograma e) Iniciar tratamiento con moxlftoxac!no oral
5) Un paciente de 45 allo'S, consulta lor fiebre, com~l't1lS01fe estado gertetl,..tQ!.~n expectoracin mucop~pJenta y disnea. Al exa~en :flslco presenta(FC: 1105.r.PA: 156/1D'!(FR:34x', ~aturacin de oxfgencf,1J8%.~l examen pulmonar demuestra crpJtolrlom:tlizados en la mititL-s~rior del hemtcafup~Jmonar Izquierdo, asociadoa a matidez y aumento de transmisin de la voz. Se sollc'ita radlografla de trax que demuestra condensacin del lbulo superior Izquierdo. El tratamiento antibitico ms adecuado es: a) Amoxlcllina oral

~
~

~
~

~ ~
~

. .:' ~Amoxicilina + cido clavulnico oral 1 f e) eftrlaxona endovenosa

(Ceftriaxona + eritromicina e) Claritromiclna oral

endoven~.as

"'

~ ~
J

6} Un paciente de 32 afiO's, 'Sin ant(!e~del'ffes tle lmportaneit prsenta un srnc(1e; relacionado con el ejercicio. El examen fisico no aporta mayor Informacin. Se solicita un electrocardiograma que demuestra signos de hipertrofia del ventriculo Izquierdo, asociados a ondas T invertidas gigantes en las derivaciones precordiales. El diagnstico ms probable es: a} Estenosis artica

r' (..CbrJiiocardiopatfa hipertrfica l{ "--) Slndrome de Brugada


~

f ..b)..{'Jliocardiopatta dilatada

~ ~

e) Displasia arritmognfca dei ventrrcuro derecho

"
~ ~

""

~'

~!
~-

~{
~~ ...
_ '

~-

~
:..-:.:..

~
~

~
7) Un paclente de 24 aftas, consulta por dolor torcico de 3 dias da evolucin, al que se le ha agregado .disnea de esfuerzo y dolor ~fl hipocondrio derecho. Al examen se observa cansado, taqulcrdlco, con tendencia a la hipotensin. La yugulares se encuentran ingurgitadas, con un colapso transitorio marcado, durante el ciclo d~t pulso venoso. El examen pulmonar es normal y se palpa el hfgado por 3 cm bajo el reborde costal, siendo moderadamente doloroso. El diagnstico ms probable es: .l Endocarditis aguda b) l'romboembolismo pulmonar i;;t)'Taponamiento pericrdico d) Infarto agudo al miocardio e) Miocarditis aguda

"'
~
~ ~

<

~
... ~

8) Un paciente de 17 aos consulta por dolor toracico i:ie 'inicio sbito, con tope nspfratoriC), asociado a disnea. Al examen flslco se observa en buenas condiciones, dlsnelco. con frecuencia canfraca y
presin arterial nonnales. Se constata disminucin del murmullo pulmonar el hemicampo pulmonar derecho, asclado a hipersonorldad a la percusin. No se auscultan otros ruidos. La causa ms probable del cuadro descr!to es: a) Embolia pulmonar . ,Jrserlcardltls ~~errame pleural t'/~ d}ftotura de bulas pulmonares -e} Neumotrax a tensin ,-;
9) Un paciente de 64 aos, diabtico e hiperienso, tiene anteceden'tes de cardlopathi coronarla, con

en

,
~
~

~
~

~
~

una clrugfa previa de revascufarlzacin miocrdica hace 2 aftos. F.st en tratamiento con hipogllcemlantes orales, enalaprll, hidroclorotlazida, atorvastatfna y aspirina. Consulta por un cuadro de dolor precordial Izquierdo, Intenso, de carcter opresivo, Irradiado al hombro Izquierdo, de 20 minutos de duracin y que cede espontneamente 1 hora antes de la consulta. Actualmente se encuentra aslntomtlcb y el examen ffslco,resulta normal, al Igual que el electrocardiograma. La /;... ~~~ducta ms adecuada es: . r:t="';l t .! a).Jf'ospitalizar, administrar oxrgeno, aspirina, heparina y antlanglnosos y soficitar enzimas cardfacas ;ti) Administrar analgsicos endovenosos y reafJZar test de esfuerzo inmediatamente \~ 1. e) Solicitar ecografla con dobutamtna y decidir manejo segn hallazgcs V .:r~ollcltar radiografla de trax y decidir conducta segn hallazgos ~- ~nviar a domicilio e indicar controlambulatorlo con ecocardlogramn

,: 1

~
~

~
~

10) La presencia de un complejo QRS ancho, de morfologfa rSR' en las derivaciones V1 y V2, es caracterrstlco de: a} Bloqueo aurfculoventricular lnfrahisiano X,b) Bloqueo completo de rama derecha ~~Bloqueo comp. teto de rama izquierda emlbloqueo Izquierdo anterior -e} emibloqueo izquierdo posterior

~ ~
~
~
~

"d>-

11) Un paciente con

antecedent~ de tabaquismo' y baja de peso, se realiza radlogratra de trax que

j
\ ~

demuestra la presencia de 3 ndulos pulmonares radiopacos y redondos, de bordes irregulares. El mayor de 9llos de 3 cm, ubicado en el lbulo superior derecho y los otros dos de 1.2 y 1,5 cm, ubicados en el lbulo Inferior derecho. El diagnstico ms probable es: a) Carcinoma pulmonar de clulas pequeftas b) Carcinoma pulmonar de clulas no pequenas e) Complejo primario tuberculoso lf(l>:}1etstasis de carcinoma ~Quistes hldatldlcos Cul de las siguientes opciones de lnformes espiromtiicos es mssugerrita de EPoc? VF: 85% del terico; VEF.1: 56% del terico; VEF1/CVF: 42% .. sb1 mejorfa con salbutamol . VF: 62% del terico; VEF1: 60% del terico; VEF1/CVF: 83%, con mejorla con salbutamol ';--' e) CVF: 105% del terico; VEF1: 96% del terico; VEF1/CVF: 79%, ein mejorla con salbutamol d) CVF: 110% del terico; VEF1: 60% del terico; VEF1/CVF: 58%, (:On mejorfa con salbutamol e) CVF: 51% del terico; VEF1: 51% del terico; VEF1/CVF: 80%, sin me}orra con salbutamol

~ ~
~

; 1 .,{

~
~
~
~

:v.

~
~

.., : ~~
~..,

('

fA

~
13) Una paciente de G7 aos consulta por alsnea. Al examen fislco destaca inatldez y disminucin del murmullo pulmonar en la base pulmonar Izquierda. Se realiza radlograffa de trax que demuestra un derrame pleurallzq~Jerdo que ocupaUI tercio Inferior del hemitrax. Se realiza toracocentesfs que demuestra protetnas: 2,2-.g/dl; LDH: .30}UI; 12 clulas por mi, de predomino mononuclear, lactato: ...2mgldJ; pH: 7,4. Prot~a~ plasmt~ca~[: 7,8; LDH plasmtica: 120 Ul CU\\Ide~lgulentes etlologfas es ms probable?

l .i (A,,

" "
~
~
~

a) Empiema

-;.>,

b} Derrame pa:-aneumico simple


tAC).:Juberculosis ~ncer pulmonar j('~e) Insuficiencia cardiaca 14) Un paciente obeso, de 57 aflos, con antecedentes de dlsllpldemfa, dlatites y lelitlasis, Inicia cuadro de vmitos y dolor abdominal Intenso, diagnosticndose de pancreatltls aguda, con Importante elevacin de la llpasa. Evoluciona con dificultad respiratoria importante y desaturacin arterial hasta . 63%, con altfslmos requerimientos de oxigeno para lograr saturacin sobre 85%. Al examen pulmonar se constatar:t crpitos intensos, dlfusc's y bilaterales. El diagnstico ms probable es: a) Neumonfa . b) Tromboembollsmo pulmonar . .. ~dama pulmonar cardiognico ~~lstrs respiratorio f\':N@errame pleural 15) Un paciente presenta disnea de esftieri:os, ortopliea y aisnea parxfstica hotrna. Afexamen . ffslco se observa desplazamiento delt::hoque de la punta cardfaca al quinto espacio intercostal, en la trnea axilar anterior, se ausculta un soplo diastlico 111M en todo el precordlo y sa palpa el pulso arterial perifrico muy amplio (pulso C:eller). El diagnstico ms probable es: a) Estenosis artica . b) Estenosis mltraJ X e) Insuficiencia artic _,.~suficiencia mitral olocardlopatla hipertrfica obstructiva 16) Una paciente, con antecedente de instiflciehcia renal crnica en hemotillisis, inlcia un cuadro de malestar general, con fiebre hasta 38~t; y sudoracin, de 3 semanas de evolucin. Al examen frslco est febril, con leve taquicardia y presin arterial nonnal. Se observan algunas petequias conjuntfvales y en los ortejos, el examen pulmonar ns normal y se ausculta un soplo diastlico 11M en el foco mltral. En sus exmenes de sangre destaca Qnemla leve, elevacin de los parmetros Inflamatorios, calda moderada en la funcin renal y elevacin discreta de las transamlnasas y la bllirrublna. El diagnstico ms probable es: 1 a) Pericarditis aguda : ~Miocardlopatra dilatada e) ndocardltis bacteriana subagCia ' eucemla e) Infeccin por VIH 17) Una paciente de 23 aos consulta por tos, con abundante expctoracln, mayor en las maanas, de algunos aos de evolucin. En ocasiones ha presentado expectoracin hemoptolca. No fuma y no presenta otros sintomas. El examen fi!S(co es normal, al tguat que la radlograffa de trax. El diagnstico ms probable es: a) Tuberculosis b) Cncer pulmonar :~e) Bronquiectasias l~""Sronqultis crnica eJ Absceso pulmonar

~ ~ ~
~

. :_. . .,_ j
.1

L.

1 V

~
~
(?~"

"
f!!'
~
~ ~

~ ~
~

" e' "


~

~.

f'
t'A
~ ~
~

~
~

('A
~

~
.: ....

~
~
~
18) Una mujer de 45 aos, sin antecedentes da lmportancta, aslntomtlca se realiza un extrasrstoles, con QRS ancho y pausa compensatoria. la c9nducta ms adecuada es: .... :"~:_ 9) Tranquilizar a la paciente y explicarle que su condicin no fe traer consecuencias negativas J'D~olicitar Holter de arritmias ':} Softcitar ecocardiograffa d) Iniciar amlodarona e) l:~i::iar digoxina 19) Un paciente con antece"dete "de a6ma seVfu, o vanas cdsis asmtlcs en el ltimo mes, ra%6n por la cual est actualmente en tratamiento con corticoides orilles. Consulta en urgencia por cuadro de disnea sibilante, asociado a cianosis. Al examen ffslco presenta FC:120x', PA:110170, FR:43x', cianosis perioraJ y tJraje. Se auscultan sfbRancias insplratorlas y espiratorias, difusas. La saturacin por pulsometrra es 87%. La conducta ms adecuada es: -a} Realizar 3 nebulizaciones con salbutamol cada 20 minutos y enviar a domicilio con dosis mayores de corticoides orales . b) Administrar oxrgeno, broncodilatadores nebulizados cada 20 minutos por 3 ocasiones, de>:ametasona e ovenosa y enviar a domicilio con dosis mayores de corticoldes males . . . ;. . e) ministrar oxrgeno, broncodilatadores nebulizados, corticoides ~ndovenosos y reevaluar en una hora, y si santa buena respuesta; enviar a domicilio con dosis mayores de corUcoldes orales X d} Hospitalizar, administrar oxigeno, broncodilatadores nebulizados, corticoides endovenosos y reevaluar peridicamente e) Realizar Intubacin orotraqueal y conectar a ventilador mecnicc
20) Un paciente con antecedente de fibrilacin aucutar en tratamiento en digoxiri, atenoJ)l y

~
~
~

electrocardiograma, como parte de un chequeo de _salud, objetivndose un ritmo slnusal, con alg!Jnas

~
~

~
~

-~ :~
-~

~
~

acenocumarol, es trafdo a urgencias por cuadro astenia y luego compromiso de conciencia de 2d


minutos de duracin. Al examen ffslco est en sopor profundo, con frecuencia cardfaca regular, de 33 lpm y presin arterial de 66/32 mmHg. Las extremidades se ap1seclan plidas y frfas. La conducta ms adecuada es: l()).dmlnlstrar suero fisiolgico endovenoso rpido . . '""'"'BfReallzar reposicin de volumen con cstaloldes y Instalar bomb21 de noreplnefrfna \{e) Instalar marcapasos eXterno .. . . d) Administrar adrenalina 1 mg ev ) Iniciar masaje cardfaco

~
~

~ ~
~

~
~

~
~
~
~

~
~

~
~

~
~
~

..,
~

~~
j~
t~
~

("

f'
~
~
~
RESPIRATORIO 1} Un paciente de 68 aos,...sin antecedntes de Importancia, consulta por cuadro de tos, con expectoracin mucopurulenta, asociado a fiebre hasta 39C y malestar. Al examen se aprec1a en buenas condiciones generales, con FC: 90x', PA: 110/60, FR: 18 rpm, to: 38C, Sat02: 95% a Fi02 ambiental. La auscultacin pulmonarmestra crpitos en la base aerecha. Se solicitaradi'graffa de trax que muestra una zona de condens;acin en el lbulo inferior derecho. El tratamiento de eleccin es: a). Amoxicilina b) Claritromicina e) Ceftriaxona ~-Amoxfcllina + cido clavulnico e) Cllndamiclna 2) Un paciente de 61 aos, con antecedEmte de_5EQC, en tratamiento con corticoides y broncodllatadores Inhalados presenta aumento de su disnea basal, asociado a_t.e.bte y expectora.cin abundante. Al examen se aprecia orientado, taquipneico, con FR: 27x', FC;_91x', PA:..:!..Pl!QJ! mmHg, Sat02: BOcr!Jl Fi02 ambiental. Se ausculta disminucin del murmullo pulmonar, sibilanclas intensas y estertores bilaterales. La conducta ms adecuada es: a) Administrar salbutamol y corticoides inhalados, oxigeno a bajas dosis e Iniciar antibiticos de amplio espectro b) Realizar nebulizacin con salbutamol, administrando oxigeno a bajas dosis, sin necesidad de corticoides ni de antibiticos e) Administrar oxigeno en alta concentracin, realizar nebullzacin con corticoides e iniciar antibiticos orales ~ministrar oxigeno al24%, realizar nebufizacin con salbutamol, administrar corticoides sistmicos e iniciar antibiticos de amplio espectro endovenosos e) Administrar oxigeno a 2 litros por minuto, realizar nebulizacin con cortlcoides y broncodilatadores de corta accin e iniciar antibiticos endovenosos 3) Un paciente de 30 aos, no fumador, ~~on antecedente de neumonfa por adenovirus, hace varios afios, presenta ~la, con abundantE! e~P-e~~oracin, dEt!!g.uno.s.anos.de ..eviUcJ6n. La expectoracin.....~.s..mav.Qr en las maanas y aumenta con el decbito. En algunas ocasiones ha presentado tfemopt!Js~ El examen pulmonar no aporta mayor Informacin. Qu examen es el mejor para hacer efCiraQstlco? 11-. a) Radiograffa simple de trax
~ACdetrax

~ ~

~
~

~ ~

~
~ ~ ~
~

~
~

~
~

~ ~
~

~ ~
~ ~
~

e) Ffbrobroncoscopfa d) Biopsia pulmonar e) Baciloscopfas 4) Un paciente de 30 aos presenta tos con el ejercicio y en varias ocasiones tambin presenta..d.ls.ru!.a,_ asociada a respiracin sibllante.j'resenta adems tos nocturna frecuente y suele hacer crisis obstruc.tivas durantTias infecciones virales. En el examen ffslcosapfeCia en buenas condiciones y destaca slo una leve prolongacin del tiempo de espiracin. Se solicita una esplrometrfa que confirma la presencia de asma bronquial. El tratamiento ms adecuado para este paciente es: a) Salbutamol en caso de sfntomas b) Salmeterol en caso de slntomas y corticeoides inhalados segn horario ~butamol en caso de slntomas y corticoides inhalados, con salmeterol, segn horario d) Budesonida en caso de sintomas y salbutamol segn horario e) Salbutamol y salmeterol en caso de sfntomas y corticoides orales segn horario 5) Un paciente de 72 anos. fumador, presenta tos con expectoracin mucosa, desde hace 4 meses, asociado a disnea de esfuerzos. En su eJ~amen fts1co dest, uflas en yiQj;!e7 reloj. El examen cardiopulmonar es normal. El diagnstico ms probable e -- - .Eg_' a) Derrame pleural ~cer bronquial e) Bronquiectasias d) Tuberculosis pulmonar e) Enfermedad pulmonar obstructiva crnicct

~ ~
~
~

1
(

~ ~ ~

~
~

~
~
~ ~

""

. 6) Una paciente de 52 aos. fumadort [)resenta disnea de esfuerzos progresiva. de 6 meses de ~ evolucin. Al examen ffslco se auscultan crpitos bilaterales. La radiograffa de trax m~estra un patrn en panal de abejas en ambas bases. El diagnstico ms Importante es: a) EPOC btFibrosis pulmonar e) Edema pulmonar d) Neumoconiosis e) Neumonfa por mycoplasma 7) Un paciente de 20 af\os Ingresa por una neumonra. complicada con un derrame pleural paraneumnico. Al examen ffslco est en buenas condiclile'S. eupnelco, con signos vitales normales. El examen pUlmonar es compatible con u~'"{rame pleural pequ;o Se solicitan gases de sangre arterial con los siguientes valores: Pa02: ~mHg, PaC02:'417 m Hg, HC03: 25 mEq/1, pH: 7,39. El . diagn~tico ms probable es: --a) Insuficiencia respiratoria global severa b) Insuficiencia respiratoria parcial, con alcalosis respiratoria e) Acidosis respiratoria, con alcalosis metablica d) .Insuficiencia respiratoria grave, con acidosis metablica y alcalosis respiratoria ~ases venosos 8) El efecto adverso ms frecuente del tratamiento antituberculoso es: a) Alergia dlfAiteraciones hepticas . e) Anemia d) Dao renal e) Alteraciones visuales 9) Los nios con fibrosis qufstica suelen contraer neumonfas causadas por: a) Mycoplasma pneumoniae b) Legionella peumonlae ~treptococcus pneumonlae d) Peudomona aureglnosa e) Streptococcus pyogenes 10) Un recin nacido de 30 semanas de edad gestacionallniclst un cuadro de dificultad respiratoria grave a fos pocos mlhutos de vltla, con taquipnea marcada, y cafda Importante de la saturacin arterial, que requiere altas concentraciones de oxigeno para alcanzar el 90%. El dl~gnstico ms probable es: a) Taquipnea transitoria b) Cardiop.atfa congnita e) Circulacin fetal persistente d) Sindrome aspirativo meconial ~fennedad de membrana hialina

~
~
~

~
~

~ ~

~
~

1
/

~
~ ~
~

~
~

~
~
~

~
~ ~ ~
~

~ ~
~

11) Un paciente consulta por un cuadro de tos. con expectoracin, fiebre y dolor ti~o puntada de costado en el hemitrax izquierdo. La radlograffa de trax demuestra un derrame p eural moderado, asociado a una zona de relleno alveolar en el lbulo inferior lzculerdo. Se realiza puncin del liquido que demuestra clulas 740 por mm3, con 86% de pollmorfonucleares, LDH: 220 Ul/1, proternas: 4,4 mg/dl, ADA: 28 Ul/1, lacfafo: 3,3 mg/dl y pH: 7,28. La conducta ms adecuada es: .._ . a) Solictfar btopslas pleurales por puncin percutnea b) Instalar tubo pleural e Iniciar antlbltlcos endovenosos e) Administrar antibiticos lntrapleurales d~ !.~iar tratamiento antituberculoso &1fliciar antibiticos endovenosos y evacuar el derrame pleural por toracocentesis.

~
12) Un paciente de 45 aos sufre un a<:cldente de trnsito, mientras conduela su vehfculo, resultando
con un ~pe en el trax c;ontra el mubrio. Evoluciona con dificultad respiratoria. Al examen se aprecia taqulpnelco, bien perfundldo, con presin arterial y frecuencia cardiaca normales. La a~sculfacl6n cardaacaes'nonnal y el examen pulmonar slo demuestra er pitos esca.sos ~~~y~~s. El daagnstlco ms probable es: #ontusln pulmonar b) Hemotrax e) Neumotrax d) Trax volante e) Crisis de angustia

~ ~

1
r

~ ~ ~
~ ~

13) Son causa de distrs respiratorio tc,das las patologias nombradas a continuacin, excepto:

a) Sepsis severa de origen urinario .r b) Frmacos e) Pancreatitis aguda . / ~farto extenso de ventriculo izquierdo e) Quemaduras extensas /

(B'

14) Un adolescente de ..13 aos presentu cuadro d~ tos, rinorrea, odinofagia de 12 das de evolucin.

~
~

En las ltimas 48 horas se produce un aumento de la tos y se agregan malestar general y disnea de esfuerzos. Al examen fsico destaca fiebre de 38C y examen pulmonar con crpitos blbasales, mayores a derecha y slbilancias escasas. El resto del examen no aporta mayor informacin. Se solicita radiograffa de trax que confirma un patrn alveolointerstlclal bilateral, mayor en la base derecha. El tratamiento ms adecuado es:
~laritromicina

~ ~
~

b) Clindamlclna e) Amoxicillna d) Amoxlcllina + cido clavulnico e) Cefalosporina de tercera generacin

15) El tratamiento de eleccin para el absceso pulmonar es:


a) Cirugla b) Amoxicilina +cido clavulnico oral e) Drenaje por puncin transtorcica y metronldazol oral d) Ceftriaxona endovenosa ~ndamicina endovenosa y drenaje postura!

~ ~

~ ~ ~
~
~ ~
~

16) Cul de los siguientes pacientes con enfermedad pulmonar obstructlva crnica tiene indicacin de oxfgeno domiciliario a permanencia, para aumentar la sobrevida? a) Paciente de 67 aos, consfntomas muy marcados, con disnea al camin-t3.m~tros .. __ _ ~aciente con EPOC estable que presentn gases arteriales con Pa02: 51 mmH~aC02: :40 mmJ:lg::> e) Paciente que contina fumando ---- d) Paciente con EPOC estable que presentt poliglobulia, Pa02: 65 mmHg y PaC02: 38 mmHg 'Y oDt e) Paciente con EPOC estable, con VEF1 correspondiente a143% del valor terico esperado
17) Un paciente de 67 aos, asintomtico, fumador, se realiza.una radlograffa de trax, como parte de un conjunto de exmenes de chequeo, exigidos en su trabajo. Se Identifica un nodulo pulmonar denso, de 3 centfmetros en el lbulo supEJrlor izquierdo, esplculado. No cuenta con radiograffas previas. La conducta ms adecuada es: a) Solicitar nueva radiografla de trax en 6 meses b) Solicitar nueva radiografa de trax en un mes e) Solicitar TAC de trax, con cortes finos ~ealizar biopsia transbronquial, por fibrobroncoscopla e) Realizar lobectomla superior izquierda 18) .El diagnstico del sfndrome bronquial obstructivo del lactante se diagnostica mediante: ~.cHnfca solamente b} La cllnica + radiograffa de trax e) La cllnica + espirometria d) La clnica+ test de metacolina e) La cllnica + test cutneo para alrgenos respiratorios

~
~

~
~
~

~
~
~

~
~

~
~

,
~
~

~
19) Un paciente d~os presenta un cuadro de malestar general, r!nQ.rrea~osa y odlnofagla, de~ dias de evolucin,- al que se le ha agregado tos, con escasa expectoracin mucosa. Presenta Signos vitales normales, faringe conge~~iva, sin exudado y examen cardlop.ulmo'formai:-EI agente ms probable es: ": a) Micoplasma pneumoniae
~inovlrus

~
~ ~ ~ ~
~

1
j

e) Virus influenza d) Streptococcus pneumolae e) Streptococcus pyogenes

20) _Cul de las siguientes esplrometrras es compatible con el diagnstico -;t:IYBasal CVF:105%, VEF1:71%, VEF1/GVF:61%; Post-salbutamol CVF:110%, b) Basal CVF:91%, VEF1:66%, VEF1/CVF:54%; Post-salbutamol CVF:109%, e) Basal CVF:50%, VEF1:66%, V.EF1/CV~s%; Pqst-salbutamc,l CVF:51%, d) Basal CVF:88%, VEF1 :88%, VEF1~:80%; Post-salbutamol CVF:89%, e) Basal CVF:70%, VEF1 :74%, VEF1/GVF:77%; Post-salbutamol CVF:70%,

e EPOC? VE : ~. VEF1/CVF:60% VEF1~94%, VEF1/CVF:74% VEF1:65%, VEF1/CVF:84% VEF1 :95%, VEF1/CVF:85% VEF1 :74%, VEF1/CVF:76%

~ ~
~
~

~
~ ~
~

~
~
~
~

~
.~

~
~

~
~
~

~
~
~
~

~
~

~
~

~
~

~ !'

~ ~ ~
'

~ \

os1 a de mover la ex rem1 a 1psnaterai, ausencia de sensibilidad termalgsica ipsilateral y usencla de sensibilidad tctil y vibratoria contralateral d) Imposibilidad de mover la extremidad c-ontralateral, ausencia de sensibilidad termalgsica ipsilateral y ausencia de sensibilidad tctil y vibratoria ipsilateral e) Imposibilidad de mover la extremidad ipsilateral, ausencia de sensibilidad termalgsica contralateral y ausencia de sensibilidad tctil y vibratoria contra lateral

'A

"'- J ~ 2) La trombofilla con nlta ms f \J":--/> a a orV deL


'

~~:

JI- 1 \..lC-"_o--

,~ ,
- -

"'

c1t de antitro~na 111 \ e) Dficit de proteina C d) Dficit de p~lna S e) Hiperho cisteinemia

(!!A

~ ~

~ fA

3) Un ~ ciente de 78 aos presenta un (:uadro caracterizado por hemiparesia y hemihipoestesja . Al examen ffsico se observa paciente dere as, a~~das a desorientacin de 2 horas de evo en asgo~upneico, con presin a ena 180/ 1 mmHg, FC: 1~0 lpm y con las alteraciones d scritas previamente. El hemoglucoteut resulta 200 mg/dl. La conducta inicial ms adecuada es: ) Anticoa_gar con heparina . . ~ <Jl b) Admi,prstrar captoprU oral ~ r--r ~ e) SoPitar electrocardiograma ~~ d) dministrar aspirina olicitar TAC de cerebro
-~

f!!'
~ ~
~

4) Cul de los sigul_.entes medicamentcs NO es til como tratamiento profilctico de jaqueca? _,.- :----b) Flunarizina/", e) Propanol9l d) Acido -~afproleo e) Azit'ptilina causa ms fre9-':'ente de muerte por cncer, en la ncer de vesl .ura biliar cer de e ncer de ulmn d) Cncer prstata e) Cnc de origen hematolgico
poblaci~n

,..
~

masculina chilena, es:


~

~ ~ ~
~
~

~ ~
~
~

6) U paciente de 61 af\os presenta temblor de la mano derecha, que se presenta en reposo, con fre uencia de 3 hrtz--ros y que desaparece al tomar objetos o realizar otros movamaentos. Camina con sos cortos y cl~r disminucin del braceo. El diagnstico ms probable es: a) Temblor esc~nial b) indrome cerebeloso / eP n d) Escler sis lateral amiotrfica e) Slndr: me piramidal 7) U paciente de 50 af\os se realiza una ecografa abdominal como parte del estudio de un cuadro de do r abdominal, visualizndose un Y!m_!lr renal izquierdo, slido, de 5 cm de dimetro mayor. La e nduct~ ms adecuada es: Controlar con n~~~::~~:=:c~ ~ 1 S 11 e sus b) Solicitar resonanCia clear ~olicitrTAC de abd d) Solicitar 1ops1a con aguja gruesa e) Realizar nefrectomla radical

'!'
~.
~

~ (A

'.

~sis~ 4s;) Crisis~

8) Un paciente de 52 aos, con antecedente de hipertensin y diabetes, sufre un episodio caracterizado por movimientos involuntarios de la extremidad inferior derecha, s~guldo luego de chupeteo y lengyJft-incOherente, que dura cerca de 15 minutos. Luego se recupera completamente, recordando slo hasta el momento de los movimientos de la ma~o. 1;1 diagnstico ms probable es: a) Accidente isqumico transitorio ~ ~ ~

~}OQ-e

d) Crisis generalizada e) Crisis parcial, secundariamente generalizada

~"'"o

r1 \ ----:1\.

-+

c.. -Ulr--.~L('

A,

9) Un hombre de 66 aos, consulta por astenia. El examen fsico demuestra palidez moderada y se pal adeno atas eneralizadas. El hemograma pn~senta hematocrito: 27%. Hb: 8.8J. blancos: 36.000, linfocito : :~J neutrfilos: 7%, monocitos: 1%, basfilos: 0%, eosinfilos: 0%, ,_P_lj!quetas: 85.000. El diagn1JStieo ms probable es:
~ _pri=j!Jfilm8"\

,A'".~-):} Leucemia lil)ftica crnica

.. /

b)'[eucemia mielpide crnica d) Mieloma / e) M~non leosis infecciosa

e, l.(.,~~~o-.....
~

~u

~CA

.k;' 10) U paciente de 65 aos, sin patologias ni hbitos de lmportancla1&res~ta cuadro de astenia, por '-.. ~~o e se realiza hemograma que demuestra Hcto:..30%, Hb,:_1jlgld, V_M O HCM: 35, blancos: 4.100, u"-{ p quetas: 100.000. Se sospecha dficit de vitamina 812; por lo que se inician aportes intramusculares, ~ saciados a folatos endovenosos, sin lograr respuesta luego de varias semanas de tratamiento. El ~~ diagnstico de sospecha es: ~ a) Aplasia medular . . \'O ,1\ "'~ Jf\ i. V\.. 1 G"e 1 ~-.A._; CA. ~ie!odjsp(aeia- . . } U\.1\ lf\ 1\ V l .3 \-- e) Leucemia mieloide crnica ~J..A.Cj ---e, 1-0 ~ -+-.. c._e,, ~ d) Anemia de enfermedades crnicas ? ~t.. v"O\c..N _o A / lJLI'~ eJ.-.\ CA o (.A:. . .., e) ficit de fierro 11 11 ~. l _-'-'Vl . J ,. r" l v_., ~t..r 'f. . l~~ -~ ro (-_...v u ,....... . . .7 M ~'\ el-\ ) Una adolescent~ de 17 aos de edad presenta desde la Infancia, tendencia a la epistaxis y U-.-<.''A/\.....A. ~~ ingivorragla. Tamtlin presenta petequias y equimosis recurre!ntes. Sus reglas son abundantes y t'~- ~ 1 suele sangrar ms de lo normal luego. de procedimientos dentales. El examen ffsico no aporta ms C>\../ . .,. informacin. El examen que con mayor probabilidad se encontlar alterado es: ~ a) Recuento de plaquetas . b) Tiempo de protrombina ~ e) empo de tromboplastina parcial activada ( ~ 'em o de san la --7 1\ R ~CA -r> ~l!Y"" , e) ibrinogenemia c....t \)Qr-. \1\/Ll"-L.t~. ~

>v-h "

v--

. .J

lf

1-

cr

<

12) Cul de las sig~ientes asociaciones entre la estructura anatmica y sfntoma que ~parece al d_'\~arse, es correcta? Wf-asclculo longitudinal medial - imposibilidad de mirar hacia lateral . b) Asta anterior de la mdula - hipoestesia ~ ~)Ve ix cerebeloso- sta mo multidireccion d) Ga glio estrellado - midriasis e) rvio facial- ptosis"'

x;

1 ) Un paciente de 55 aos sufre un episodio caracterizado por desviacin de la comisura labial erecha y dificultad para mover la mano ipsilateral, de 2 horas de duracin y que se recupera completamente. Al momento del examen se encuentra asintomtica y su examen fisico es normal. La
conducta ms adecuada es: a) Enviar a domicilio iniciando aspirina y una estatina b) Realizar trombolisis endovenosa e) Realizar angiografla carotdea y decidir manejo segn hallazgos d) Hospitalizar, anticoagular con heparina y solicitar ecodoppler carotfdeo ~pltalizar. iniciar medidas de Aeuroproteccin, solicitar una neuroimagen y realizar estudio de fuente em tea . .

!'
14) Un paciente de 65 aiios presenta ast1~nia y palidez de. varias semanas de evolicln. Al examen ffsico se observa plido, delgado, con Vllrias adenopatras pequeas, no dolorosas, en cuello, axilas e Ingles. No presenta otras alteraciones. Se reallia exmenes de sangre e~ que destaca un .hemograma, con hematocrito de 26%, hamoglobina de 8,7 g/dl, blancos~linfocltos: 90Cfo, neutrfilos: 9%, segmentados: 9%, baciliformes: 1%, promielocitos: 0%, blastos: 0%, monocitos: O%, plaquetas: 76.000. El diagnstico ms probable es: oma de Hodking d) Linfoma no Hodking de alto grado e) Mielofibrosis

liAftisa e

linftica a~u=

oC:a>

15) Cul de los siguientes hallazgos es ms orientador a una alteracin de la hemostasia primaria? .a) Alargamien~o del TIPA 4 1)t ~qufmosi,. -=, 1\...e l. vo . \-")J!\titece ntes familiares de hemorragias
~

.l!)~

'\.Q,

e) He artrosis La disminucin de la sensibilidad de la zona distal de las extremidades, especialmente del omponente vibratorio y termalgsico, e:s caracterstica de: a) ononeuritis mltiple in e) Poliradculopatfa desmielinizante d) Dao de la vfa espinotalmica de la mdula e) Dao de los cordones posteriores de la mdula j7) El factor pronstico ms importante m un sarcoma de partes blandas es: a) Tamao del tumor b) Tipo hist9lgico . e) Presencia de adenopatas . / '~Ion microvascular -->Je) Gr do de diferenciacin histolgica 18 n paciente de 34 aos presenta accidente de trnsito, con golpe en la cabeza. Inicialmente e oluciona bien, sin embargo 30 minutos despus evoluciona con cefalea intensa y luego compromiso e conciencia de instalacin relativamente rpida, hasta caer en Glasgow 6. Al examen fisico destaca anisocpria, con midriasis derecha. La explicacin ms probable es: a} Hemorragia subaracnoidea, con compromiso del tercer nervio craneal b) Hematoma intraparenquimatoso, con hemiacin del cfngulo - . e) Hematoma extradural, con herniacin de las amfgdalas cerebelosas -~ ~ \\..J.-'{' r--.-A.~ -~~matoma epidyal, con herniacin del uncus temporal,....,.~c:-0 ~ ~ Ltr-~'~" e) Hematoma subdural, con compresin del troncoencfalo

~
~
~
~

,-J-_

~
~
~

~ ~
~
~\

\L (-'

-\- ..:.. -'

~
~ (P'

~
~

19) La anemia por hemorragias agudas s1~ diferencia de la anemia por hemorragias crnicas en que la primera: a) No se beneficia de tranfusiones de glbuk>s rojos ~~lene un indice reticulocitario elev r .~ '1 esenta aumen o e la transferrina plasmtica t..~ v-:;~ .S. \. d) Es mejor tolerada e) Presenta esquistocitos en el frotis sanguineo

~ ~

~ ~
~
~

20).Una mujer de 67 aos, con antecedente de fibrilacin auricular, presenta hemianopsia homnima i~~erda de inicio sbito, con alta congruencia. Cul es el sitio ms probable de la embolia? ~~ ~eria vertebral izquierda b) Arteria basilar ~ e) Arteria cerebral media izquierda ~) Arteria cerebral ..~.e a cerebral anterior derecha

v_"_\-._~_l-Ro \)__f. .~_u ~> __ . __

tc:t-)u

~.-5.~

~ ~

Jl1 V fe/?;

U rc_.>'--C'-....

(//' V Cl l-.

~
~

~ ~
Prueba 24 Preguntas del EMN ao 2007
~
~

_/

1) Al decir que los fumadores tienen un riesgo relativo de 5 pa1-a cncer pulmonar respecto a los no fumadores, esto quiere decir que: a) Los fumadores tienen un 5% ms de riesgo de tener cncer pulmonar, comparado con los no fumadores b) ..Un 5% de los fumadores desarrollar cncer pulmonar ;!Los fumadores tienen 5 veces ms riesgo de tener cncer pulmonar, comparado con los no fumadores d) La probabilidad de que los fumadores tengan ms riesgo de cncer pulmonar, comparado con los no fumadores, es de 5% e) Un 5% de los cnceres pulmonares se explican por el hbito de fumar 2) Un hombre de 40 anos presenta, desde hace alg(m tiempo, Etlucinaciones, retraimiento social y descuido por presentacin personal. Recuerda. que ha tenido ~~ episodios similares. Al TAC se aprecia atrofia de la corteza frontal. El diagnstico ms probable es: a) Trastorno delirante . b~elirium d) Demencia frontotemporal e) Demencia por cuerpos de .Lewy 3) Un recin nacido de 4 das de vida, de grupo sangufneo A, R:h (-),cuya madre es Rh(+), presenta ictericia, la que se sigue con bilirrubinemia total, llegando al sp.timo da a 10mg/dl. Se alimenta exclusivamente con leche materna. El diagnstico ms probable es: a) Ictericia por leche materna b) Atresia biliar primaria Ictericia fisiolgica d) Incompatibilidad Rh e) Incompatibilidad por grupo clsico 4) Lactante de 9 meses presenta hepatomegalia, palpndose e!l hfgado por 2 cm bajo el reborde costal y percutindose una proyeccin heptica de 5 cm. La conducta ms.aproplada es: a) Solicitar pruebas hepticas . b Solicitar hemograma Solicitar biopsia de mdula sea fd~olicitar biopsia heptica . rrJ Citar al prximo control del nio sano 5) Un hombre es diagnosticado de epilepsia, secundaria a un tumor en el lbulo temporal. Sus crisis se caracterizarn por ser: a) Mioclnicas b) Sensitivo-visuales c)jre sitivo-tctiles ~J ~ icomotoras el otoras puras 6) Un lactante de 2 aos tiene antecedentes de haber nacido prematuro, a las 33 semanas de gestacin y de haber pesado 2.500 gramos al nacer. Sus controles previos son normales. Es trafdo por su madre porque desde hace un mes presenta desviacin de la mirada. Al examen fsico se aprecia estrabismo convergente y leucocorla del ojo Izquierdo. El diagnstico mu probable es: a) jletinopata del prematuro . )fRetinoblastoma e) Atresia del conducto lagrimal d) Glaucoma congnito e) Ambliopa

"
~
~

~ ~
~

~
~

ll{ ~squizofrenia

~
~
~

~
~
~
~

~
~
~
~

~
~

...,
~
~
~

~ ~
~

~
~

~
~
~

~
~

e:

!i'

fl

7)/n hombre de 25 aos presenta desde hace algunos meses lesiones eritematosas, no dolorosas, on caspa fina, entre cejas y en el cuero cabelludo. Cul es el diagnstico ms probable? . ) Pitiriasis rosada ) Dermatitis de contacto

c,~soriasis ~ eermatitis seborreica

~ ~

e) lia capitis

Ji) Una enfermera sufre accidentalmente li.ln pinchazo en l dedo fndice izquierdo, con una aguja que /i:fue utilizada para puncionar a un paciente con alto riesgo de contraer VIH, pero que no se ha realizado ' lo estudios confirmatorios. La conductat ms apropiada es: , / Iniciar terapia combinada b) Tomar anticuerpos al paciente y a la enfermera y esperar los resultados e) Tranquilizar a la enfermera, sin necesidad de mayor estudio d) Iniciar AZr e) Solicitar Elisa para VIH y carga viral al paciente y esperar los resultados
9) t}na paciente de 47 aos consulta por metrorragia. Sus PAP han resultados normales, habindose r~lizado el ltimo hace un ao. Al examcm se observa una lesin solevantada en cuello uterino. La conducta ms apropiada es: /a) Solicitar PAP inmediatamente b)..,.s61icitar inmunoflueorescencia pata virus papiloma humano ~ Derivpr a colposcopia y biopsia de la lesin . d) Re9hzar cono cervical e) ~Ocitar PAP en un ao Una purpera de 40 dias, consulta por dolor en mama izquierda y fiebre, con temperaturas de hasta .7C. El examen muestra la mama con~1estionada, dura, con eritema difuso, dolorosa a la palpacin, on zona fluctuante periareolar y pezn con fisura profunda. Cul es la conducta ms adecuada? Suspender la lactancia materna e iniciar <:loxacilina oral b) Puncionar e iniciar flucloxacilina e) Hospitalizar, iniciar vancomiclna oral y su:spender la lactancia materna . ospitalizar e iniciar cloxacilina endovenCisa spitalizar y realizar drenaje en pabelln bajo cobertura antibitica con cefazolina Un paciente de 34 aos se fractura de' hmero en una calda de altura. Cul de los siguientes azgos sugiere una lesin del nervio radial? ~O ~ ...'f'o.. _.\ .~ .~ ,....,-<. 6--i-.Q~ a) Imposibilidad de abducir el brazo-~ c-...x... ... ~. r ~ JU r (p-J(.., b)rfposibilldad de flectar los dedos~ "-"'-d..J-<-~ Imposibilidad de extender la mueca~ r.' ~~ . . d) Anestesia de los pulpejos de los tres primeros dedos ~ ~ e) Anest~si de los pulpejos del cuarto yquinto dedos .~ vv~\ ~-"

~ ~
~

~
~
~

~
~ ~
~

) A
A
/

~
~

J
.

JL;o

rv--e

~
~
~

Vt"!! ~mitos

12J-.?Cul de las siguientes patologas puede producir hipovolemia con alcalosis metablica? ..J.P.Iarrea 1

"'

cJ-e.,~\'j\( ~ r
,., r\.'-.' ,.,
v.. v

~
~ ~

e) Intoxicacin con benzodiacepinas d) lngesta excesiva de lcteos e) Hiperventilacin -~ ~f.:, J "'~ ,~ ~~

~'V

-< ,.
.... ""'

"'-'f'P
1.

-,

~~'-tt

~\~

._

'

~
~
~

~
~

13) Una mujer de 60 aos, con antecedente de hipertensin arterial crnica, consulta por dolor retroesternal, intenso, irradiado al dorso. Se solicita ECG que se informa con alteraciones inespecfficas de la repolarizacin. Las tro,poninas resultan negartlvas negativas y la radiogratra de trax Jiemuestra ensanchamiento mediastnico. La conducta ms adecuada es: .. d L) alizar coronariografa ~ 0 \. ~C:-~ CNO ( ~ enielar aspirina, oxigeno, morfina y nitroglicerina __ _[ olicitar test de esfuerzo e~ ~~._.,_ dl)tliciar anticoagulacin con heparina y realizar monitoreo electrocardiogrfico continuo (_ ~"L.c. l!) Solicitar ecocardiografia transesofgica d~ inmediato ~~ ~ f\..J-..,~'-'l.~-t

\.-4

-N

L~~c

r!"'
~
~

14) Un paciente, diagnosticado de hipotiroidismo, est en tratamiento con levotiroxina 100ug/dia desde hace 6 meses. No presenta srntomas. Se solicita.TSH, que resulta 0,01 mU/L y T41ibre, que resulta 2.,5.g/dl. La conducta ms adecuada es: a) Solicitar T3 libre . -:n-:-: ~ .. s<llicitar TBG \.J) ~ ~\.. \J_.\JO 1"-A ~4..: Solicitar anticuerpos anti-TPO d) ~mentar la dosis de levotiroxina _.er[)isminulr la dosis de levotlroxlna 15) Un paciente de 48 aos de edad, cuyo padre fue operado a los 55 aos por cncer de colon. ualmente est asintomtico y el examen ffsico no presenta alteracio~~~ Cul es la conducta ms cuada? .~ _ : ./ Solicitar test de sangre oculta en deposiciones ~(.D~ ~ ~ antigeno carcinoembrionario d..A ....31::.\ / f' fflOiicitar colonoscopa ,_ poC.."-' ~ d) Solicitar hemograma y VHS ~~ fcJ.Jv-.~ ~~ ~O"- . . e) Observar evolucin y estudiar slo ante la aparicin de sfntomas . '-"e ( ~.. ~ fi~w ~

'

A
J

/_~olicitar

t_
~'

L--~~-P\.A

O~ ~

16) Un ~fi(ujer de 60 aos, presenta hematuria al final de la mlccln, asociada a disuria y poliaquiuria. El e men ffsico no aporta mayor informacin. La conducta ms adecuada es: ~ 4 olicitar sedimento de orina y urocultivo . .r ," cA. .,;J\-e r}-e ~ .t.t_~.\...o r--.r--e, ~ ) Solicitar piel oTAC )tv-<)....,.~ ~ "? e) Solicitar ecografa renal y vesical 0v c-.cJ-..o .~ "-< t. ~ d) Solicitar cistoscopa ----&~ e) Solicitar pielografia de eliminacin endovenosa .t. ........ ~rc.ur-,
~ -~e ~--.-

Lc,t ~1\.:?

17) Un paciente de 24 aos sufre una herida por arma blanca on el brazo derecho, durante una pelea. Al examen presenta importante hemorragia. Adems de instalar una via venosa para la.administracin de c~istaloides, la conducta inicial ms adecuada es: ~,solicitar hemograma A)) Realizar sutura de la herida e) Administrar cefazolina endovenosa . d)j(ealizar exploracin digital de la herida, con anestesia local ~!) Realizar compresin local

""'

18) Un paciente de 67 aos consulta por fatigabilidad y disnea1 de esfuerzos. Al examen se apres;i~ plidQ, palpndose una leve esplenomegalia. Se solicita hemograma q._ae:-mu.estra hematocrito: ~}," hemoglobina: 8;1JJ!ll, plaquetas: 84.000_por mm3, glbulos blancos: ,25,30n_por mm3, linfocitos.;_10 Yg. neutrfilos segmentados: 22%, baciliformes: 1%, monocltos: 6%, eosinfilos: 1%. El diagnstico ms -.... . / proba~le ~s: ~. ~ a) IJnfoma ~~ ~-- j>)"'leucemia linftica aguda~ ,.......,.../.....,_~ ~. ~~~ ~\~~ -~) Leucemia linftica crnica d) Anell)ia de enfermedades crnicas e) M9nnucleosis infecciosa

fl

/.

9) Para diferenciar a un lquido articular inflamatorio, cul de las siguientes alternativas entrega nformacin ms til? _ 'p.-l~~....o a) Hemartrosis ..j, ~lA; ~.;A.....u- vv ~c)...A ~ "" b) Leucocitos de 2.000 por mm3 e) 9Dmplemento bajo en relacin al plasr:na
~Filancia

e) D_olr articular

...- 2o) Un nio de 13 aos presenta IPT en percentil 50, IPE en percentil 70, ITE en percentil 80. El
di'-gnstico nutricional es:

,.a1 Eutrofia
b) Riesgo de desnutricin e) Desnutricin d) Sobrepeso e) Obesidad

(:.

~
!...1

..

.:.

~
V

Preguntas Cardiologfa 1) En un adolescente de 17 aos la presencia de un soplo sistlico de eyeecfn asocfado a desdoblamiento fijQ del segundo ruido, permite sospechar el diagnstico de: .:'j}}comunlcacln lnterauricular b) ductus arteoso persistente e) comuntcacln lnterventricular d) vlvula artica bicspide e coartacin artica
~ardiopatfa

~
'

~ ~ ~ ~ ~

.filEn ChUe, las prfn'cfpales causas dEt.lnsuflclenciac:aroraca sn: hipertenslva y valvulopaUas . b) Cardiopatfa coronaria y mfocardiopatra dilatada ~ e) Cardiopatla coronaria y cardlopatfa hipertensiva d) Cardiopatfa hipertensiva y miocardiopatra dilatada e) Cardiopatfa coronaria y valvulopatras 3) Paciente de 30 aos, sin antecederites cardiovasculares 'de importncia. Consult por disnea de esfuerzos~ ortopnea y DPN de tnstauracin progresivas, desde hace 2 meses. la RxTx muestra cardiomegalia y signos de congestin pulmonar. Usted solicita ecocardiograffa que muestra dilatacin de todas las cavidades, con FE: 50%. Las medidas farmacolgicas ms Importantes para este paciente son: . a) Aspirina, estatinas y betabloqueantes. ...bl..Diurticos, IECAs y aspirina ~'Betabloqueantes, diurticos e IECAs d) Espironolactona, IECAs y aspirina e} Amiodarona, betabloqueantes y diurticos 4) Mujer 53 aos, con disnea de esfUE!JZOS progresiva, de 2 anos de evolucion, actualmente CFlli. Refiere expectoracin rosada en algunas ocasiones. Al examen, PA:120170, FC:102x", Rl2T, soplo diastlico 111M. Crpitos blbasales. El ECG, slo demuestra ACxFA. El diagnstico ms probable es: a) Cardiopatfa hlpertensiva. . b) TEP crnico Miocardiopatfa dilatada d tenosls mitra! e Insuficiencia artica 5) Paciente hipertenso, de 67 aiios, ccm dolor "torlco littenso. de inicio sblto. Al examn fsico FC: 110x', regular, PA:230/128, asimetrfa e:n los pulsos radiales y a la auscultacin cardiaca destaca un soplo diastlico. La conducta Inicial ms adecuada a seguir, adems de tomar un ECG es: _AReanzar coronariografa urgente r~dministrar oxrgeno, moina, nitroglicerina y aspirina inmediatamente dJ.:::.( t.t/> !:3S:i: (Q... x e) Realizar ecocardiograma transesofgico urgente d) Realizar perfcardlocentsfs e) Solicitar radiografa de trax 6) Mujer de 45 aos, hipertens, consulta por palpitaciones de 3 das de evolucin. lngres en buenas condiciones, PA: 160/100, FC:102x'. EJ ECG muestra fibrilacin aurfcufar. La conducta Inicial ms adecuada es: ~ealizar cardloversin farmac:olglci3 l:dn amfodarona .,,... b} Realizar cardloversin elctrica X e) Iniciar anticoaguladn Inmediata con hepana d) Iniciar betabloqueo a permanencia, como control de frecuencia e) lfldlcar digoxina a permanencia 7) Paciente de 59 anos, HTA, OM2, furtrador, refiere dolorenpantorrfila derticha al carrtinar'2 :ufras. que le Impide seguir caminando y que cede eon el rposo. La conducta ms adecuada ante este cuadro clfnlco es: V.a) Ejercicios diarios y control mdico de ~.us factores de riesgo cardlovascular b) Solicitar anglografia femoral para LJbicar sitio de obstruccin e Intentar angioplastia e) Iniciar anticoagulacin oral con cumarftllcos JQ Derivar a cirujano vascular para cirugl de re perfusin de EEII (!Solicitar ecograffa-doppler de EEII

e
~ ~

~
~
~

1~

~ ~
~
~

~ ~
~

~
~

~
~ ~

~ ~
(4'

.
3' anos en palpitaciones muy intensas de Inicio y trmino brusco, de 'mh'lil~os de duracion en varias oportunidades en los Ctltlmos aos. Ingresa a urgencias por cuadro de pa1pltac1on~ intensas que esta vez no ha cedido. Al examen: FC:180x', PA:110170. El ECG muestra taquicardia regular a QRS osto. La primera medida a seguir es: dministrar adenoslna endovenosa Administrar amlodarona endovenosa . e) Realizar cardioversin elctrica )td) Realizar maniobras vagales e) Administrar propafenona o flecainida oral
8) MuJer

:~

!~

~,_

.~

La cardiopata congnita ms frecu~nte l$: omunicac!n interauricular ~ omunlcacin interventricular e) Ductus arterioso persistente d) Derena}e venoso anmalo e) Tetralogfa de Fallot
10) Paciente de 79 aftas, consutta por cefaiea. tinitus'y mareos. Al xamen; PA:210130, FC':99X'. RR2T, aumento de la Intensidad de R2, auscultacin pulmonar normal, sin signos focales en el examen neurolgico. El fnnaco de eleccin, para el manejo de esta situacin, es: ' Nifedlplno sublingual &Captopril oral e) Nitrogllcerina sublingual d) Nitroprusiato endovenoso e) Enalapril oral

.~

~
~

,:~

/ J!l. v

,;

11} Mujer, 60 afios, en ~tenosls mttral severa,lnll~ bi's~tmentedisnea y ~pitaciohes. Al examen FC:140x'. PA:85l35, MP(+), con crpltos intensos y difusos, H12T con soplo diastlico IVNl, mayor en
ms adecuada es: endovenosa en bolo .. b) Entregar oxigeno a Fi02 elevadas y soporte vemilatorio con Eiipap . e) Reponer fluidos con suero fisiolgico ~el) Cardioversln elctrica Inmediata e) Administrar furosemida endovenosa, aso-ciada a drogas vaso~:~ctivs 12) Paciente 58 aftos, DM2, HTA, cursa con dolor retroesternal de 45 ii)ltnltos d~ duracin, rifn~o~ E\ ECG muestra supradesnlvel del ST en DI, VS y V6. La medida ms importante, de entre las que se enumeran, es: _ . J!} Anticoagulacin . ~EDTerapia de reperfu'siii iiini~dlata CT Manejo estricto de glicemia y presin arterial, con farmacos endvenosos d) Solicitar enzimas cardiacas de manera urgente e) Solicitar ecocardiografia de urgencia
~f:!X. La conducta inicial ~dministrar amloclarona

.: A'!)
:;~
:.t

;~

~: ~
~

t./

..~~
~
~
~
~

13) Paclente diabtico de 66 anos, avolciona cdn herida e11 maleoloexterno, de 3 crlis d dimetro, sin tendencia a la curacin, dolorosa, con bordes Irregulares y escaso eritema circundante. Presenta . adems disminucin de la sensibllldad de ambos pies. El diagnstico ms probable es~ a) 91cera por presin . b) Ulcera venosa /@lcera arterial a) lcera neurogniea e} lcera secundaa a infeccin polimicrobiana 14) Paciente de 15 aos, cursando con herida pertetranle torcica por 'rma blanca. evoiuciona con hipotensin marcada e ingurgitacin yugular. Al examen MP(-t) simtrico. La conducta inmediata a seguir es: a) Realizar cocardiograma . ; ~.Reposicin de fluidos por vra p&'rifrJca v 't.e ealizar pericardiocentesls Realizar radiografia de trax e) Solicitar electrocardiograma

~
.. ~

~
~
~
~

~~
~

~~

"
:,i

~ ~ ~

ff!'

E:!'

~ ~

15) Paciente cie 60 os, fumador, HrA. Cnsulta porque hace 4 meses inicia dolor torcico opresivo, que aparece cuando el paciente camina 150 metros, o cuando sube 6 peldaos de la escalera. Cede con el reposo, durando cerca de 5 minutos. El ECG slo muestra a1teraciones inespeclficas de la repolarizactn .. La mejor conducta a seguir, junto con controlar los factores de riesgo cardiovasculare injciar aspirina, es: f dicar nitroglicerina sublingual en los epfsodios de dolor licitar anglograffa coronaria a la brevedad ){ licitar Test de esfuerzo a la brevedad d)Solicltar un nuevo ECG e) Evaluar respuesta a cambios de estilo de vida y aspirina, eh 1 tnes

~ ~
~

16} Paciente DM2, JRC, en hemodllfuis, portador de ICC, de origen coronrfo, usando enalaprif, espironolactona, atenolol, aspirina y estatlnas. Evoluciona con obnubilacin, asociada a bradicartli de 40x' e hipotensin. El ECG muestra complejos QRS anchos y ondas T picudas en todas las derivaciones. En sus exflmenes destaca Na+;147, K+:8,0, Cl-:111. la primera medida a seguir es:
a} Cardloversln elctrica urgente
~b} Administrar gluconato de calcio endcvenoso

e) Iniciar insufinoterapia endovenosa, a:;ociada a betaagonistas "frihalados d) Indicar marcapasos externo


e} Realizar dilisis de urgencia

~ ~ ~ ~ ~ (!A

11) Ante la sospecha clinica de TVP de a} Aebograffa de EEII 1 AngioTAC de EEII ~ V (e}~cografla doppler de I:EII .

exttemidade~ inferlores,

el medida dfagnsUca de eleccin es:

d) Plelismograffa de volumen , ~.e) Angiograffa de EEII


18) Paciente hlpertenso, de 1 afios. ~-ia sufrldo 3 sfncopes, relacionados aJ esfuerzo. En el itJmo tiempo se agrega angina de pecho al caminar 1 cuadra. Al examen: soplo slstUco eyectlvo 111M. El
diagnstico ms probable es: a} CardiopaUa coronaria b} Cardiopatla hlpertensiva e) Miocardiopatra hipertrfica no obstruttiva 1 ...9l,.Miocardlopatra dilatada con insuficiencia niltral 5eclilidaria

~ ~
~
~

v C~tenosis artica

19} Paciente cursando c-on IAM con infrad~nivel del segmento ST en 011, 0111 y aVF: Evoluciona con
hipotensin e Ingurgitacin yugular. 1\l examen FC:92x', PA:82/48, RR2T sin soplos, sin signologra pulmonar. La conducta ms adecuad es: @Realizar perlcardiocentesis b) lndicartrombofisis de urgencia e) Administrar cristaloides y dopamliia pl)rvla ~ndov-enosa d) Solicitar ecocardiograma ')(e) Solicitar derivaciones precordiales derechas

~ ~
~ ~

~
(!'A

~
~:

~:
~:

20) Paciente de 55 aftas, hfpertenso y dlabtlclJ, sufre Infarto mlocrdlco notransmural, hace 15 das. Se recupera bien, con buena capacidad funcional y fraccin de eyeccin normal, en la ecocardfografa. Se realiza un perfil Upfdlco que muestta LDL:120, HOL:45, TG:250. La conducta ms adecuada para el manejo de sus llpidos es: @ndicar dieta y ejercicios Y" 'Xb) Iniciar atorvastatina e) Iniciar gemfibrozilo . d) Iniciar una asociaCin entre un fibrato y una estatina e) Su perfillidico es normal, por lo que no es necesario inoicl:l"r ninguna medida terapetica

~:
~
~

~
~

t.... , . 1

~
~

.f

'

{LO

/J'-8-

~
~

-----~
Prueba 12 INFEC.TOLOG(A segunda parte

,.,
~

rll\ 7

~~~~

1) Un paciente con VIH, que no sigue tratamiento antirretroviral, consulta por diarrea. Su ltimo .ta recuento de CD4 fue hace 2 aftos y en ese momento era de 21 Oclulas por mm3. Refiere que est e '1 diarrea desde hace 3 meses, que sus deposiciones son aci:iOsas, sin elementos patolgicos. El agc ~ etiolgico ms probable de su cuadro es:

,--\l) Criotosporidium

t) Oxoplasma

d) Mycobacterium avium complex e) Virus de la inmunodeficiencia humana 2) Un paciente portador de una leucemia llof#itiCa aguda, consulta por fiebre y malestar gene[al. Al examen f[sico slo presenta paliaez. Se solicita hemograma que muestra un hematocrito: 27%, blancos: 20.000, con 2% de neutrfilos, 66% de linfocitos y 32% de blastos, plaquetas: 52.000. La conducta ms adecuada es: a) Solicitar estudio para VIH y realizar exmenes de laboratorio en busca de un foco infeccioso b) Iniciar quimioterapia y vancomicina y realizar exmenes de laboratorio en busca de un foco infeccioso e) Iniciar cefazolina + metronidazol y realizar exmenes de laboratorio en busca de un foco infeccioso - d) Iniciar ceftriaxona + amplcllina y realizar exmenes de laboratorio en busca de un foco infeccioso ~) Iniciar ceftazidina + gentamicina y realizar ex~menes de laboratorio en busca de un foco infeccioso

~ ~

~
~

~
~

~ ~

P
c.

fe?

r9

3) Un paciente de 35 aftos presenta disuria Intensa y secrecin uretral. Refiere haber tenido relacio1~ s uales sin proteccin. Se solicita un Gram de secrecin uretral, que demuestra la presencia de occeas Gran negativas intracelulares, por lo que se le administra ceftriaxona 250 mg por va ...., antrainuscular. Dossemana despus reconsulta por persistir con los mismos sfntomas. La conduct ms adecuada es: ~ a) Solicitar urocultivo y sedimento de orina b) Iniciar corticoides e) Administrar ceftriaxona 1 g por una vez, por via intramuscular d) Administrar ciprofloxacino 500 mg por una vez, por vla oral ~nielar doxiciclina por 1O dlas, por va oral .

4) Cul de las siguientes asociaciones entre microorganismo y patologfa es FALSA? .-->Jl) Staphilococcus aureus - Celulitis orbitaria / - lb) Streptococcus pyogenes -:- Escarlatina v e) Streptococcus pyogenes - Erisipela / ,.....d) Staphilococcus aureus - Antrax cutneo e) Staphilococcus aureus- Sindrome de piel escaldada ../

~ 5) Un paciente de 4 aos presenta una cuadro compatible con una meningitis aguda, por lo que se .._ realiza una puncin lumbar, que da salida a U uido do aumento de 'J celular.idad-de predomiulo polimeFfonuclear. La tincin de Gram demuestra bacilos Gram nega avo~ El agente ms probable es: a) Neumococo ~ b) Listeria ..-e) Meningococo

--- -'> Leptospira

~~ 1

6) Cul de las siguientes es una Indicacin quirrgica en la endocarditis bacteriana? ,.....a) Endocarditis sobre estenosis mitral previa . b) l.nfecci6n por e.stafil.ococo dorado tneticllino resistente e) Presencia de vegetaciones . ~ d Insuficiencia cardiaca progresiva _...e) Endocarditis so re ca~ 10pat a congnita

~
7) Un paciente de 50 aos, con antecedente de EPOC tabqulco, bien controlado, con uso de broncodilatadores y hbito tabquico suspendido hace 5 aftos, presenta un cuadro de tos con expectoracin mucopurulenta, asociada a fiebre y dolor tipo puntada de costado izquierdo. No presenta mayor disnea de la habitual. Al examen ffsico est en buenas condiciones generales, con signos vitales normales y satura 94% a Fi02 ambiental. Tiene un MP(+), con espiracin prolongada y ' \ / se ausculta algunas sibllancias difusas y crepitaciones localizadas en la base pulmonar derecha. Se ~ solicita radiografia de trax que demuestra una zona de condensacin en el lbulo inferior izquierdo, sin derrame pleural. La conducta ms adecuada es: a) Enviar a domicilio con amoxicillna oral .~Enviar a domicilio con amnxicllloa..n::s cida daw dnie&-Oral ~Hospitalizar con ceftriaxona endov,mosa d) Hositalizar con ceftriaxona ms claritromicina e) Sor "tar cultivo de expectoracin y decidir tratamiento antibitico segn resultado n nio de 5 aos presenta una placa alopcica de 4 cm en el cuero cabelludo. Al examen se servan pelos quebrados en la pluca y ligera descamacin. El tratamiento ms adecuado para la atologfa descrita es: a) Terbinafina tpica b) Fluconazol oral ~ Griseofulvina oral d) Permetrina oral e) Permetrina tpica 8}

~ ~

~ ~
~
~

~
~

~
~

~ ~
~ ~

~ ~ ~
~ ~
~

Las lesiones caracterfsticas de la escabiosis son: ) Placas eritematosas en tiro al blanco b) Ppulas y vestculas en tronco y zona distal de las extremidades e) Placas con borde arciforme y tendencia a Ja curacin central csQ.: Veslculas perladas y surcos e)--p udoppulas y escoriaciones Un paciente de 5 aos presenta un cuadro de 3 dias de evolucin de fiebre, decaimiento y dinofagia. Al examen fisico se observa faringe eritematosa, con exudado amigdalina adherente. Se palpan varias adenopaUas cervicales y presenta esplenomegalia. El diagnstico ms probable es: a) Infeccin por VIH ~Mononucleosis infecciosa e) Amigdalitis pultcea d) Linfoma e) Infeccin por adenovirus

~ ~
~

J
"

~ ~

12) Cul de las s1gu1entes alteraciones es ms especifica para una sinusitis aguda? a) Radiografa de senos paranasales, con velamiento de un seno maxilar b),.Obstruccin nasal 1(Descarga posterior .~iMcin de salida de pus desde el meato medio e) ensacin de presin fac1ai

~
~

~
~
~

) Un nio de 8 aos presenta male1;tar general y odinofagla. Al examen ffslco se observan vesrculas n el paladar blando, algunas ulceradas. Presenta adems vesfculas en las palmas de las manos. El agente causal es: a) Virus varicela zster CbK_Enterovirus e) Parvovirus 819 d) Virus herpes 8 e) Virus herpes 6

('!'
~
(9iA

--Prueba 11 lNFECTOLOG(A 1) Un paciente VIH, sin tratamiento antirretroviral, presenta cefalect, asociado a paresia .de la extremidad Inferior derecha. Presenta una convulsin tnjco-clnica en una oportunidad. El examen flslco no aporta mayor Informacin. El examen que con mayor probabilidad dar el diagnstico es: ~ Tincin de Gram, cultivo y citoqufmico de LCR / . r b) Tinta china y citoqulmlco de LCR ~ ~~ '4 .~ .. . ~)UT~A~C~de~~~' . d Electroencefalograma e) Serologla para toxoplasma

~
~
~
~ ~
~

r/-'.

..f/1

~
~

2) Cul de las siguientes medidas le parece m~ adecuada para la prevencin del contagio de enfermedades de transmisin sexual, en una mujer sexualmente activa, sin pareja estable? a) Uso de anticonceptivos orales . b) Uso de antibiticos profilcticos e} Vacunacin contra el virus papiloma ~Uso de preservativo durante las relaciones sexuales ../ e) Instalacin de un dispositivo intrauterino . Un paciente de 20 aftos, cursando con una osteomielitis aguda, evoluciona con fiebre, gran caimiento, eritrodermia y marcada hipotensin, asociada a taquicardia, que no responde a la ministracln de suero fisiolgico. Los hemocultivos resultan negativos. El diagnstico ms bable es: a) Shock sptico b) Fasceltis necrotizante e) Endocarditis aguda d) Infeccin conjunta de Estafilococo areo y Estreptococo pyogenes ~Shock txico estafiloccico ~ ~t ~"'-U- ~ - Un paciente de 35 afios sufre accidente de trnsito, resultando con un traumatismo encfalo . . craneano abierto. Se somete a neurocirugfa, con buenos resultadlo, sin embargo a los 4 dfas, evoluciona con fiebre, cefalea y signos men(ngeos, por lo que se realiza puncin lumbar y se solicita estudio de liquido cefalorraqufdeo, que es compatible con una m~enlngltis bacteriana aguda, sin poder visualizarse bacterias. El tratamiento antibitico empfrico ms adecuado es: a) Ceftriaxona + penicilina , b) Ampiclllna + cefotaximo ~ \. G o .~.._"l..kn e) Cefazolina + metronidazol -, ~ ~ ~ ~~ eftazidima + vancomlclna Gentamicina + amplcllina

~
~

""' ~
~
~

) J

~
~

~ ~

~
~

,r-

~
~
~

5) Un nlf'io de 6 afios presenta un cuadro de 2 dfas de evolucin de odlnofagia y fiebre hasta ~8,5C. Evoluciona luego con un exantema papular, generalizado. Al examen flsico se.observa faringe . eritematosa, con algunas petequias y exudado amigdalina bilatell'al, grisceo. Adems presenta una lengua saburral, c-on demarcacin de las papilas y un exantema papular, eritematoso, con algunas petequias lineales en los pliegues cubitales. El agente causal ms probable es: ~) Staphilococcus aureus ~ Streptococcus pyogenes ~ (....0.1\.\.AM~ e) Parvovirus 819 d) Vi s herpes 6 e) E terovirus

~
~

~
. .. )

'-4

~
~
~

6 Una paciente de 60 aos, sin antecedentes de importancia, hospitalizada por una fractura de atada guirrglcamente, evoluciona al 4to dfa con fiebre alta que luego se le agrega disnea. Al examen se aprecia febril y ecafda, presenta F~ 1Q,Ox', PA: ,0/40 mmHg, MP+ con ese~~. RR2T con un soplo diastlic~ intenso. Los pulsos se palpan saltones. la conctucta ms adecuada es: ~~ollcitar hemograma, PCR, hemocultivos y ecocardiografia _ . ~ bjSolicitar hemograma, PCR y radiografia de trax ~ e) Iniciar ceftriaxona endoveosa , ,._.. d) Solicitar cultivo de la punta de los catteres intravasculares que tenga . _ ~ 1J L,.1V e) Iniciar vancomicina ms imipenem por vla endovenosa ~-

('

!:
7) n paciente cursando su 4 dfa post accidente vascular enceflico, presenta fiebre y tos con e pectoracin. Se realiza una radiografia de trax que muestra una condensacin de tipo neumnica n el lbulo inferior izquierdo. Qu tratamiento es ms adecuado? a) Cefazolina + clindamicina b) Amoxicilina e} Levofloxacino d) Ceftrlaxona + claritromicina -e) Cefotaximo + metronidazof 8) La onicomicosis se debe tratar preferentemente con:

~ ~ ~ ~

?...::, a) Terbinafina oral


b) Griseofulvina oral e) Nistatina tpica o d) Clotrimazol tpico e) Anfotericina 8 endovenosa 9) Cuando un paciente en quien se sospecha infeccin por VIH, presenta un test de ELISA positivo, ste debe confirmarse con un Western 131ot. Esto se debe a que: ) El Western Blot permite detectar aquellc,s pacientes que estaban en el periodo de ventana ) El Western Blot mide directamente las protefnas virales, mientras que el EUSA slo mide anticuerpos ) El ELIS~resenta fa:s pnsitjyos. esp~tcialmente por ~=eassiA oFuz:aela a Gtrgs ujns, como el HTLV-1 y W"Stem Blot los descarta d)EI ELISA es menos sensible que el Wes.tern Blot e) El estern Blot es menos sensible que el ELISA

~ ~
~

~
~ ~
~.

:J

~ ~

~ ~
~

1 O Una nia de 5 aos es trafda por su madre, ya que ella piensa que tiene piojos, porque se rasca la beza muy frecuentemente, desde hace un par de semanas. La conducta ms adecuada es: a} Indicar lindano tpico .. b) Indicar permetrina tpica al1 o/o e) Indicar corte de cabello d) Indicar antihistaminicos y permetrina oml ..) e)_B.evisar el cabello y cuero cabelludo, en busca de liendres y tesioaes,_ 11) Cul es la conducta ms adecuada ante un paciente de 4 af'ios que nunca se ha vacunado? a} Administrar la vacuna BCG ahora, las vncunas DPT, Hib, VHB y polio oral en un mes y la vacuna trivfrica en 2 meses ms . ---==*'>Administrar la vacuna BCG tytrjca DPI, Hib VHB y polio oral. todas juntas /e} Aaminist.,.- las vacunas trivica y BCG ahora y las vacunas DPT, Hib, VHB y polio oral en un mes ms d) Admini rar la vacuna DPT solamente e} Admi 1strar la vacuna DT y trivfrlca 12) n nio de 5 aos presenta un cuad1ro de decaimiento, fiebre escasa, asociado a eritema de ambas m jillas. A los 3 das presenta un exante,ma macular eritematoso, que compromete el tronco y las onas proximales de las extremidades, non un patrn reticular. El diagnstico ms probable es: a) Rubeola ) Exantema inespecffico por enterovirus ~} Quinta enferme d - ,.-ti) xantema s 1o e} Sarampi 13) mo se contrae la hidatidosis? a} R r consumo de came de cerdo o cordero, contaminada b or contacto con deposiciones humanas ) Por contacto con deposiciones de perro d) Por consumo de berros contaminados e) Por consumo de pescado contaminado

~.. ) ( "
(!A
~ (.';

~ ~

(ll'

"'

~
(F:'o

~
~ ~ ~
~

~ ~ ~ (!'
~

~
~ ~
14) Una paciente de 38 aos, cursando con una pielonefritis por Escherichla coli multisensible inicia tratamiento antibitico, con adecuada cobertura, sin embargo 5 das despus perman~ce febril, con disuria y dolor lumbar. La conducta ms adecuada es: a) Cambiar el tratamiento antibitico por uno de mayor coberturH ..../ "" b) Solicitar una tomografia axial computada ~ e) Mantener el tratamiento antibitico y reevaluar en 2 das. _::>.:_> Solicitar ecografia _ e) Solicitar nuevo urocultivo y decidir segn antibiograma 15))-4'e.ucorrea por cndida se caracteriza por: ~ .~ rt.eucorrea blanquecina, asociada a disuria intensa ( . ) Leucorrea amarilla verdosa, asociada a vulva en fresa ) Leucorrea blanquecina de mal olor, con slo escasos signos inflamatorios de la pared vaginal - d) osa asociada a rurito vulvar e) Leuco a blanca, grumosa, asociada a cervicitis

...,
~
~

~
~

~
~

~
~

/ a 1agente causal ms frecuente es el Staphilococcus aureus ) Generalmente se manifiesta como una monoartritis:aguda e) El liquido articular presenta una disminucin de su filancia< d) La puncin articular es muy importante para el diagnstico ./ ..... e) El drenaje quirrgico se reserva para aquellos casos que no msponden al tratamiento antibitico endovenoso 17) Una mujer de 33 aos, inmunodeprimida, presenta variae lesiones solevantadas, de menos de 5 mm de dimetro, con una umbilicacin central, en tronco, cabeza y extremidades, no dolorosas, ni pruriginosas. El resto del examen fsico slo demuestra candidiasis oral. El diagnstico ms probable es: ( ~ a) Condiloma b) Cand!diasis cutnea f ,. _ ...; ~ c. l (_(}.._ - e) Prrigo '-J1"'J ~dl Molusco contagioso ~ e) Angiomatosis basilar (Bartone//a hense/ae)

16) especto a la artritis s~ptica es FALSO que:

~
~
~ ~
~
~

~
~

yofJ

~
~
~

18) na paciente. de 40 aos presenta lesiones vulvares rugc,sas, polipofdeas, indoloras, e ecialmente en los labios mayores y menores. El agente causal ms probable es: ) Virus Herpex simplex 2 b) Virus papiloma humano, serotipos 11 y 6 e) Pox virus d) Ureaplasma urealiticum e) Chlamydia trachomatis

19) Un paciente de 6 aos inicia diarrea, asociada a fiebre, dlllor abdominal tipo clico y decaimiento. Al segun~~ ~ra 1~ diarrea se vuelve mucos~nguinolenta. Al examen fsico est decafdo, febril, lo.... 1 Q 11 ~ d!.!bid[ c1n m compromiso hemodinm1co. La conducta ms adecuada es: - ~ '-...JI.f e;-" a) H pitalizar, administrar suero fisiolgico e.iniciar ceftriaxona fmdovenosa "("" '1'1'-../'vf"t.. (..e-n-. ~ b ospitalizar e iniciar ciprofloxacino endovenoso :1f' n./ \V" ..1 ~ Enviar a domicilio con hidratacin oral, antiespasmdicos y antidiarreicos -'f" \,/\--Q d) Dviar a domicilio con hidratacin oral y mtrimnxazol jodjcando antiespasmdicos en caso de dolor ~ ,., ) Enviar a domicilio con rgimen hldrico y solicitar hemograma, hemocultivos y test de deteccin de rotavirus ~ ~ en depo tciones ,

20) 1tratamiento de eleccin para la sifilis primaria, en los pacientes alrgicos a la penicilina, es: a Ciprofloxacino ) Tetraciclina A'Macrlidos d) Clindamicina e) Sales de mercurio

"
b) L?pt~spir?sis e) Hrdatrdos1s tsJ(!Jistom tosis e) Triqui sis Un paciente de 56 aos, proveniente de Temuco, presenta ictericia, dolor abdominal y fiebre. Al men fsico se observa ictrico, con el hfgado palpable 3 cms bajo el reborde. c.ostal, sensible. En el mograma destaca una eosinofllia de 25%. El diagnstico ms probable es: epatitis A ,r

f' "

~ ~

o.r- -'-O

~JJJ. r l(>t.~{_o t-+L~t..4.l>\.

~ ~

ul de las siguientes opciones e:s un tratamiento vlido para una pielonefrltis aguda? a) etronidazol b Amoxicilina ) Macrodantina
~Cefazolina

d!!'

re} Clindamicina
16) Una mujer de 26 aos, cursando su segundo embarazo, de 29 semanas, presenta fiebre hasta 39C, asociada a leucorrea. Al examen fsico presenta to: 38,7, FC: 110x', PA: 120/80. Destaca sensibilidad a la palpacin uterina y a la especuloscopia se aprecia que la leucorrea proviene del OCE. El ~egtstro fetal muestra FCF: 170x', stn otros signos de SFA. La conducta ms adecuada es: ~Administrar corticoides y antibiticos del amplio espectro e interrumpir el embarazo en 48 horas b) Iniciar antibiticos de amplio espectro y mantener una conducta expectante e) Realizar una amniocentesis de inmediato --=)~) iniciar antibiticos de amplio espectro, por vla endovenosa e interrumpir el embarazo por una va expedita e) Realizar cesarea de urgen era, de lhmecuato ,u"''O~ 17) U paciente presenta elevacin per.3istente de las transaminasas. Se solicitan serologas virales que emuestran: lgM VHA (-), lgG VHA {+), Ags YHB (-), Acs VHB_j}, lgM anticore VHB (-), lgG anticore VH (+),anticuerpos totales anti VHC (.;~o), lgM VCA (-), lgG VCA{-j, lgM CMV (-), lgG CMV (-).La e ologfa ms probable de su hepati.tis os: ) Hepas-Ab) Hef*l{itis-A)1"1r e) Hepatitis A, 8 y C d) Hepatitis B y C ~Hepatitis ~ 18) Un paciente de 35 aos presenta aumento de volumen y dolor de ~a rodilla Izquierda y al examen fsico se constata eritema, aumento del calor local y derrame articular. Se realiza artrocentesis que da salida a un lquido articular turbio, de fltancla disminuida, con 90.000 clulas por mm3, 92% de polimorfonucleares. En la tincin de Gram se visualizan cocceas Gram negativas intracelulares. La conducta ms adecuada es: ----. ~nielar ceftriaxona endovenosa y realizar drenaje quirrgico ~ l .a ~ b) Administrar cefazolina intraarticular ~ <.. e) Iniciar vancomieina-endovenosa y realizar drenaje quirrgico b ~C./.;..1 d) Iniciar azitromicina oral y drenar el derrame, mediante artrocentesis repetidas e) Iniciar cloxacilina endovenosa y realizar drenaje quirrgico

~ (!'
~

~ ~

~ ~
~
~

~ ~
~
~
~

r.J..; .J-.

1-(_.t-..

,--u-

~ ~

19) Cul de las siguientes vacunas pue,de usarse durante el embarazo? a) Varicela
~ubeota

e) DPT

e
~
~ ~
~

d) Polio oral ~_!!1jneymnc6ci~

e
~

20) Un paciente de 56 aos, obeso, presenta fiebre y dolor en la extremidad inferior derecha. Al examen se aprecia una placa eritematosa de 15 cm en la pierna derecha, ligeramente sensible a la palpacin, de bordes difciles de delimitu. La conducta ms adecuada es: a) Enviar a domicilio con ciprofloxacino mB clindamicina por va oral b) Solicitar cultivo de la lesin ~Hospitalizar e iniciar cloxacilina endovenosa <Vd) Hospitalizar e iniciar ceftriaxona endovenosa .-1, e) Enviar a domicilio con eritromicina oral " \. r

JL9
'>(

\\

S))!.\

-~~ '<

1t / \.~

~ ~

""S .

-y

'/~

~ ~

~.
Prueba 9 N . ROLOGIA 1 Un paciente de 15 a~ presenta Rarestesias y debilidad de las ~uatro extre volucin, que f:lan ido en aumento. Al examen fsico se aprecia 1lbolici n de lo osteotendJneos e hipotonfa. El diagnsUco ms probable es: ~a) Polirradiculone uopatla elesfnielinizan~{Sindrome de G dlliar:a Barr) bj"Coll)presin medular e) Mi~stenia gravis d) Jnlarto !acunar de troncoencfalo nfarto cerebeloso . Existen numerosos medicamentos que sirven para prevenir la ocurrencia de ataques de migraa. o de estos medicamentos es: a) lndometacina

--..

~ ~

~ ~

~:.LQpanaJOJ -

~ ~i

e-J
~

. e) 'Ergotamina d) VerapamiJo e) Tramado! 3) Un paciente de 5 aos presenta cefah~a y fiebre. Al examen ffsiqo se aprecia en buenas condiciones, ~- con signos vitales normales, sin sig'Osneurolgicos focales, perQ presenta signs menngeos esbozados, razn por la cual decide realizar una puncin lumbar con el siguiente anlisis de lquido cefalorraqudeo: Clulas: 250 x mm3, con ~~ de mo11onucleares, protefnas: 50 mg/ml (VN: 15-30), glucosa: 70 mg/dl (VN: mayor a 40). La tincin de Gram no muestra bacterias. La conducta ms adecuada es: c.... \. . / : a) Iniciar aciclovir endovenoso ~- ~ ~ lt*lnlciar medicamentos antituberculosos y corticoides e.v. r. .J \ f e) lniciar:-~eftrlmu:ma~r:teamjcjna por vla..er.~dov.eRgsarasaeiadasa-dexametasona d) Solicitar TAC de cerebro sin contraste e) E . miento sintomtico, indicando reconsulta ante signos de alarma

~~
~ ~
~
~

~.t ~,

'f

-\-1

u-J

~ ~

~
~
~

~~~!tr::=~=~:;g~os vitales, admloisfrandn ~l, de roo~de ~alntener ~ r~s~3 e) Vjar la hipertermia con medidas fisicas y farmacolgicas ... .ffh ~ (' ~\ :r d~ ~tlicar rgimen cero ..../ " \
e)/~dministrar oxigeno

n paciente de 62 aos, hipertenso de larga data, mal controlado, presenta desde hace 3 horas, un uadro de Instalacin rpida delieiiptejf" y anesteSia derechas. Al examen ffslco est desorientado, con di..fiE:!Ltad para hablar y con ausenctia de respuesta sensitiva y mot~!- en..tl hemicuerpo derecho. Se solicita una tomograffa computa~a d! cerebro~~~:~=:~ ~~~ma!ggl_ilntracerebral izquierdo de 3 cm, Cul de las siguientes indrcac:rones es __ 5NQS_~nnBe_ab]ill a) Controlar frecuentemente con hemoglucotest y agregar insulina de modo de mantener la glicemia dentro de r~t=~gos normales ~

1 lj~ (P

por naricera, a 2litros por minuto

~ ~ ~

~ ~

e
~
~

Un paciente consulta por diplopfa. Al examen fsico presenta e~trabismo convergente y p.tQ!_ erecha. Es capaz de mirar normalmente~ a la derecha, sin embargo al mirar a la izquierda, el ojo quierdo lo hace normalmente, mientras que el ojo derecho se queda en la lfnea media. Lo mismo sucede al mirar hacia arriba y hacia abajo. El nervio comprometido es: . ~ T~cer nervio craneal derecho b) Cuarto nervro craneal izquierdo e) Sexto nervio cr~neal derecho d) Tercer nervio craneal izquierdo e) Sexto nervio cranealizquierdo

~
~

(!'
~

paciente consulta en el servicio de urgencias, por dolor periocular muy Intenso, asociado a e ifora y miosis lpsilateral. El dolor suele presentarse diariamente durante la tarde, desde h e 2 dfas, duran o a gunos minutos. La conducta ms adecuada es: Iniciar amitriptilina ) Solicitar resonancia magntica cerebral e) Admjnistrar AINEs ms corticoldes endovenoso d) Administrar sumatriptn subcutneo o e) S icitar ra ogra a e rax, en busca de un tumor pulmonar ; . 7 Un paciente presenta disminucin de la sensibilidad de las zonas distales de las extremidades, special ente de la vibratoria. No presenta alteraciones motora!S. El diagnstico ms probable es: a) Mon euritis !Illtiple __ ,_ C).... ,. ... \ h'f"~_:' ~-e 0 :l?)-P \2--rv~ y ~ _.., e)-~ drome de arteria espinal posterior~ t14fl ,..Q .LA~~ { v-\. hA ~ d)~~ndrome de arteria espinal anterior . e} Slndrome de Guitlian Barre

) J
~
h

""

r qv--o

. ..

Mujer de 33 aos, consulta por astenia y diplopfa, de 2 semanas de evolucin, fluctuantes. Al xamen se observa ptosls izquierda y ligero estrabismo divergente, que ella refiere no haber tenido previamente. El diagnstico ms probable es: . a) Sindrom~ de Guillan Barr b) Mia!itenia gravjs_. e) Tumoy'hipofisiario d) En~falopatla de Wernicke e) Si 16rome de Claude Bernard Horner 9) n nio de 1 afto presenta un cuadro de odhiofagia y fiebre alta, hasta 39,5C, presentando una . e nvulsin tnico clnica de 2 minutos de duracin, autolimltada. Al examen fsico se encuentra en uenas condiciones generales, con pulso y presin arterial normales, con examen neurolglco y cardiopulmonar slp~lteraciones, destacando slo eritema y adama farfngeo. La conducta ms ,/ _ : adecuada es: a) Enviar~ d-~mkilio con ~ b) Solicita~g~~~~-- e) So~ici r estudio de liquido cefalorraquldeo, PCR y hemograma . d) Ad 1nistrar anticonvulsivantes endovenosos y solicitar resonancia magntica nuclear de cerebro ) 1 ciar cido valproico a permanencia Un paciente de 70 aos, hospitalizado por una neumonia aclqulrlda en la comunidad, ATS 111, senta un cuadro de agitacin psicomatnra, CQR alucinaclones.yisyales e Ideas paranoldes, refiriendo que las enfermeras lo quieren matar. Al examen fsico no presenta signos focales y se aprecia inatento y desorientado. El diagnstico ms probable e!s: a) Enfermedad de Alzhelmer b) AVE isqumico e) Sindrome confusional a.9_u~~ d) Esquizofrenia e) Epilepsia del lbulo temporal

->

r--'r 11) La compresin de la cauda equina se manifiesta clnicamente como: .. / ~ a) Prdida del control de esflnteres,lmpotencia sexual y anestesia en silla de montar ~ ~ b) Afectacin otora asimtrica de las extre des inferiores sin com romlso sensitivo _, e) Afectacin sens1 1va e as cuatro extremidad~s. so_n compromiso motor .~d) C~romiso a~trico de las ralees nerviosas, q~e lnervan las extremidades inferiores, con afectacin sensitivavi0fi5i1i _...., . e) Compromiso stmtrico de extremidades inferiores, con afectacin motora y de la sensibilidad termalgsica, respetando la sensibilidad tctil, vibratoria y proploceptlva ~ . ~ ~- ~

-~

G\A-

. ~ 0\.-

\])rr\-t.A: o.....

.!)-)

r"

~ ~

e e
~

J
1

iagnstico ms probable es: } CrisJ tnico clnica } EP.i(epsia mioclnica e) isis parcial simple - d} risis arci < e} Crisis de ausencia .

f~:racJn, seguidos de dolor epigstrico y desconexin del medio, que dura cerca de 1 minut. El

-e:

1
pa9lente de 30 aos presenta movimientos Involuntarios de la mano derecha, de 20 segundos

~ ~ ~ ~ ~

/13).Cul de los siguientes cuadros clinilcos es ms sugerente de una hemorragia subaracnofdea por rotura de un aneurisma? . a) Cefalea opresiva occipital, con dolor a la palpacin de los msculos cervicales posteriores - b) Cefale.a de inicio sbito, con intensidad1.Qt10 aseeieda a 11mi\gs __.. e) Cefalea intensa, pulstil, frontal, asociad: a fotopsias d) Cefalea de rpida instalacin, asociada a hemiparesia derecha y abolicin de los reflejos osteotendineos derechos e) Cefalea progresiva, mayor con el decbito, asociado a vmitos y edema de papila 14) Un paciente de 67 aos presenta alteraciones de la memoria reciente, asociada torpeza motora y

. alucioaci.o.Des yjsuaJ.es, de cerca de un ao de evolucin, que han ido en aumento. Se inicia


risperidona en bajas dosis, evolucionando con marcado empeoramiento de sus sintomas. El diagnstico ms probable es: a) Demencia frontotemporal ~)De ~nci~a~~~~~~awY : e) Hidroc~falia normotensiva d) Dem~rcia senil
e) D~Jitium

~
~

~
~

~
~

~
~

~ ~

15)'Un paciente presenta dificultad para ~~aminar y vrtigo de inicio sbito. Al examen ffsico se aprecia . ~smetrfa de las extremidades, marcha atxica, con aumento de la base de sustentacin y nistagmo )lilateral, multidireccional. El diagnstico ms probable es: ~) Accidente vascular isqumico de troncoencfalo b) Accidente vascular enceflico de arteria c:erebrat media derecha _ e) Accidente vascular enceflico de arterias cerebrales posteriores .. d) Hemorragia creblosa e) Parlisis vestibular 16) Un paciente presenta hemiparesia de1recha de instalacin brusca, que luego de un dfa comienza a

. . mejorar, hasta ser casi imperceptible al quinto dfa de evolucin. El diagnstico ms probable es: .X.. a) Accidente isqumico transitorio. ~
. b) Parlisis de Todd e) Esclerosis mltiple X _ ~ ::_ 'id) lnf~r ~ ~-- ~e)~ isqumiCo, con transformacin hemCJrrgica /

~
~
~

e e
~
~

lgunos movimientos. Al examen fsico se observa atrofia de las masas musculares de las . xtremidades inferiores, mayor a derechat, con hiperreflexia bilateral, asimtrica y fasciculaciones. No . presenta alteraciones sensitivas. El diag111stico ms probable es: 1 a) Sfndrome de cauda equina ... .-r... . . _ r"' _ , ,, . ~ l ....._, r -:-b} Esclerosis laleFel amietrtiCa ~ .D '~ 0--~ ..t_.~ ~V'- ~ C e) Sindrome piramidal 1 (\. r-... _ d) STndrome extrapiramidal ~ .-~ ~ ~ e) Miastenia gravis

1y)~n paciente de 40 aos presenta debilidad muscular, dificultad para caminar y para coordinar
rJ .

-Al

e
~

,.,
~

~
~) Un paciente de 50 aos presenta cefalea progresiva, desde hace un par de semanas, que suele ser

/'mayor en la maf\ana. A esto se le ha agregado hlpoestesla en la extremidad superior Izquierda y present una convulsin tnico clnica en una oportunidad. El diagnstico ms probable es: a) Eneefalitis-herptica . __ :J ~ b) Pseudotumorcerebral 4 ~ ~ 1 .~~-, -L~ p--r...,...-- r 1"e) Accidente vascular cqn compromiso de los ,ncleos de. la base (1 n 'r.i\. at.dl Tumor cerebral -)) t J ~~o. ~~ _vv- ,~- e) Esclerosis mltiple
...;'1

~
."JJ

fL '"'~..

-,

-(._i.Jc_

~.

,_,
~

~
~ ~
~

ta, posterior izquierda. Suele de.sencadenarse al tocar la zona. Usted sospecha una neuralgia de
.

{un paciente de 33 aos presenta dolor tipo neurlgico, recurrente e Intenso, en la zona cervical

rnold. El tratamiento ms adecuado es: Tramado! b) Ketoprofen9 e) Gst9f'mzpjna d) Diaze~ m e) Clo romazina

~
~
~

20) n paciente de 61 aos presenta temblor de la mano Izquierda, principalmente de reposo, como si e tara monedas y que desaparece al tomar objetos o al realizar otrol movimientos con esa tremldad. Camina con pasos cortos y presenta disminucin ele la expresin facial, del parpadeo y del braceo. El diagnstico ms probable es: a) Temblor escencial b) Slndrome cerebeloso 1" e) Enferm~e~druad;uiiLEiUIS.IJ!l.S.Qnd) ... rosis lateral amlotrfica e) Slndrome piramidal

~
~
~ ~

~
~

..,
4t)

~
~
~

~
~

~
~

~
~

~
~

""'
~

~ ~
~
~

..,

~ ~
Prueba 7 INFECTOLOG[A, segunda parte

~
~

~ ~
~ ~

/) 1) Un paciente de 34 aos presenta algQ[(a en la cavidad bucal y refiere haber bajado ~j{g d~-~-;~n los ltimos 3 meses. E~ examen ffslco no aporta mayor informacin. La conducta ms adecuada es: ~~::~:!:~ ~:~ ~:~: CWJ'9s de hisopado bucal e iniciar fluconazol oral
e) Solicitar carga viral rc.a=r:..:::a~..__""' d) Solicitar Westemblot para VIH e) So 1c1 ar recuen o e 1n oc1tos CD4

e!'
~
~
\

~ (!C'

3) Cul de los siguientes antibiticos puede ser ocupado para tratar infecciones por Staphilococcus au~ slble a la meticilina, con meJor respuesta? efaz e) Metronidazol d) Amoxicilina e) Azotromicina

e
~

~J;~ 1 ~~~ fo,u'f.Q.

e
~

e
~

e'
~

4) Un pacient.e portador de una leucema linftica aguda, consulta-por fiebre y malestar g~&t';~l examen ~J.:~~o presenta palidez. SEt solicita hemograma que muestra un hematocrito 0 o-~ blancos~! con 3% de neutrfilos, 66% de linfocitos y 31% de blastos, plaquetas: 52.000. La conducta mas adecuada es: a) Solicitar estudio para VIH y realizar exmenes de laboratorio en busca de un foco infeccioso b) Iniciar quimioterapia y vancomicina y realizar exmenes de laboratorio en busca de un foco infeccioso e) Iniciar cefazolina + metronidazol y realizar exmenes de laboratorio en busca de un foco infeccioso ) d) Iniciar ceftriaxona + ampicilina y realizar exmenes de laboratorio en busca de un feGo.JI.:eJCClJ:lSCL_ ($/Iniciar ceftazidina + ~entamicina y reali; un fl ~ palpacin de las masas musculares, sin otros hallazgos de importancia. Se solicita hemograma que demuestra eosinofilia importante. El diagnstico ms probable es: a) Larva migrante visceral b) Distomatosis ~Hidatidosis

?~z;-~

7.)- ()

p 5) Un paciente consulta por mialgias ge,nerallzadas y fiebre. Al examen fsico presenta dolor a la

~ ~
~

&rT!!gyinasis}
e) Cistecercosis

~
(!!" ~

f ~
:

~ ~
~

~ ~ ~
~

6) Una paciente est hospitalizada hace 1O horas por una neumonfa del lbulo inferior izquierdo, adquirida en la comunidad,~. con fiebre hasta 41,5C y e~asns requerimientos de oxigeno. Su presin arterial es 1~0180 rnmHg y su pulso es de 100 lpm. Se solicitan tres hemocultivos y se inicia tratamiento con ceftrlaxona, de los que uno de ellos resulta positivo para Staphllococcus anren5, mientras que los otros 2 resultan negatlivos. La conducta ms adecuada es: a) Iniciar vancomicina y supender la ceftrietxona b) Iniciar cefazolina y suspender la ceftria:xona e) Agregar vancomicina al tratamiento con ceftriaxona ~Agregar cloxacilina al tratamiento COft-S(!ttt:'i8*001a---------- - - - - - - .. (~'Mantener la ceftriaxon~, ya que muy probablemente se trata de contaminacin de la muestr

e
~

e
~

~
~

)(
.1

/'"

e::

---v~}~-7 ~

(. "")

7) Un paciente de 66 aos diabtico, mal controlado, presenta aumento de volumen y dolor facial de rpida instalacin asociado a escasa secrecin nasal. Al e~amen se observa '-rjtema y edema facial, con un rea necrtica negruzca en relacin a la prbminencia malar Izquierda. El agente etiolgico ms ~bable es: ../ ~ (j)'Pseudomona aureginosa ~ b) Neumococo e) Staphilococcus aureus .:-CQ ~coccus Qyogenes_ 8) La distomatosis se contrae debido a: a) Relaciones sexuales sin proteccin b) Picaduras de insecto ./ ~gesta de mal cocidos " ~:~gest s silvestres e) lngesta de. a 1~entos con am a) Ciprofloxacino b) Ceftriaxona ;;),Penicilina ~DgxiC!CITi) e) Sulfas

"
~

~
~

~
~

~
~-

~
~-; :

~ 9) El tratamiento de eleccin de la uretritis por Chlamydia~trac:homatls es:

,.,
~
~
~

1O) Un paciente cursando una infeccin, inicia tratamiento antibitico endovenoso. Durante la administracin del frmaco presenta aparicin de lesiones ertematosas, solevantadas en la cara, cuello, hombros, y zona proximal de las extremidades superiores, con prurito. El antibitico que con mayor probabilidad produjo este cuadro es: a) Gentamicina b) Cloxacilina e) Ampicilina

~
~
~

@8=~~!~~~
C
11) Un paciente cursa con fiebre, compromiso del estado general y aparicin de un soplo djast61ico de intensidad mqderado. En el examen fisico presenta adems algunas peteQuias subconjuntivales y en los pulpejos. Los mejores exmenes para el diagnstico son: a) PCR y hemograma 1& Angio C hemocultivos . t9f Hemocultivos ecocardiografia transesof ica d) Ecocardiografia trans or c1ca y emograma e) Ventriculografia y ecocardiografia transesofgica

~
~

~
~
~

12) Las meningoencefalitis virales suelen ser cuadros benignos y autolimitados. El agente etiolgico ms frecuente es: .~) Virus herpes simplex Virus he s zster (9E rovirus l) V1rus 1n uenza e) Adenovirus

""
~

JU

~
~

~
~
~ ~

~
~

~
~

~ ~
~

~~

f''

A~GlQxacilin - ?.,/1-t-srtacon!?pjarfa :~ del tracto urinario febril? ~)Qu tratamiento NO ~ '1"a}rJt~ccln ~


(!IPGentamicina ~ e) Cefazolina

~stCefotaximo

iprofloxacino
-'?

J.-11 ~

....
1--t,..

""it/ ;JA

&(Ir

,_.r-

(?_/'1

f'

~<

V progresiva de la agudeza visual bilateral, de 7 das de evolucin, que actualmente slo le permite
distinguir siluetas. ltimo recuento de CD4 se lorealiz hace 7 meses y en ese entonces era de 72 clular CD4+ por mm3. La conducta ms adecuada es: a} Solicitar RMN de cerebro ~!irzar fondo de Gjol .. / e) Iniciar aciclovir d) Realizar estudio de liquido cefalorraqu Ideo e) Iniciar corticoides endovenosos

Q 15) Un paciente VIH positivo, con tratamiento antirretroviral intermitente, presenta disminucin

Su

~ ~

--JYb>
~

~
~ ~ ~ ~

16) Un paciente de 6 aos presenta dolor y aumento de volumen cervical. Al examen ffsico se aprecia -D una adenopatfa cervical anterior izquiEt,t:'da de 3cm, dolorosa, con escaso eritema en la piel que la J..- recubre. Destaca adems caries y enfe!rmedad periodontal generalizadas en la cavidad bucal. La
}-.)-'
11 r- 0 t"'4

conducta ms adecuada es: a) Administrar una doisis de penjdlina be~~ i~~~~ular b) Iniciar tratamiento con amoxjcilina ms : el C ~ar via or8il ~ - fA Hospitalizar e Iniciar una cefalosporina de tercera generacin por vfa endovenosa . /.J))' Solicitar hemograma, hemocultivos e luM para Bartonella hensellae e) Tomar cultivo por puncin con aguja fina .

--r6

~ ~
~

'!'
~ ~

C.~,la neurocisticercosis se caracteriza por todo lo siguiente EXCEPTO: s causada por Taenia solium ) Suele ser asintomtlca : / -~ ~4-;'J.c...Gc;r:;. (...~-, e) Se contrae por consumir carne de cerdo mal cocid~ - > ~~fA.~ .:::.-,,_~e~,~~-~(~ ~) Los pacientes con neurocisticerco~. no transmiten la enfermedad a otras persona " e) Es una causa rara de epilepsia ,..

_..?

/' _,.

/'

_.,

~
~

18) Qu relacin entre antibi9tiee de elee&i~cJ~ia es correcta? Afl)Zenicilina- Estreptococo betahel]lOitif~ ~~ 1 liS) lmipenem- Estaftlococo dorado .. ~ ~ e} Vancomicina.- Acinetobacter baumanii ~ d} Azitromicina - Pseudomona aureginos~ tA-pe, ...., e) Me~zol- Enteroc@ -~ t?~Pl
1 1 ..

"
fA
~

.d 19) Qu antibiticos son s efectivou para tratar Infecciones causadas por anaerobios? tronidaz / Amoxicilina y levoftoxacino 0 '?e: -c....a c)Doxiclinaypenicilina DA:~ o ~if~ d) Cefazolina y gentamlcina cr _ve) Fluctoxacilina y tinidazol ;v-0 ~

'

~
~
~

~
~

~
~
~

e
f'A
~ ~

~
~
~.

':...(V

~-J

(:'
~

r'
~
~

PRUEBA 7 Preguntas lnfectologia 1) ciente de 75 aos, diabtico, con bu(Jn control metablico, consulta por cuadro de tres dias de us1o Al. e lucin, de_!o!.t. con expectoracin y flebre hasta 38.zlj). En el lti o da se agrega xamen fsico se apreca"paciente decafdo, desorientado, Glasgow 4 PA: , 119 ', FR:28x', t 0 :38,9. En la auscultacin pulmonar se aprecian crpitos en base erecha.8eConstata radiogrffa de trax con condensacin en lbulo inferior derecho, sin derrame pleural: Los gases arteriales demuestran P~, PC0134Bic;!!, pH:7,35, a Fi02 ~f!ibiental. El tratamiento antibitico de eleccin b) Cl~uitromicina e) ftriaxona ms claritromicina ~ e naxona so a e) e az1 mams vancomicina

~ ~

:i~moxicilina ms cido clavulnico


e

-;r~-;:. )
-t- 'Q
.

rJ'.

~,~<-') ~~~"'-

('J

~ t...1.il-

u-}-~ .

~
~

~
~

(A

~
~

2 Un paciente de 56 aos consulta por intenso dolor en rodilla derecha. Al examen se aprecia rodilla erecha eritematosa. caliente y aumentada de volumen. Se realiza artrocentesis, con salida de Uquido urblo. poco fjlante. El estudio cltolgico demuestra ms de 100.000 clulas, con 90% de polimorfonucleares. La microscopfa demuestra cdstales escasos de pjrofosfato de calcio y la tincin de Gram demuestra ~occeas gran E?s1fivas en racim} El tratamiento de eleccin es: .J ~ ~ ~ a) Drenaje quin'lrgicn,_sociado a Van_Qmic' endovenosa ~1--,.......J \' r ico, asociado a Cloxacilina endovenosa S ()./'J r se) Cefazolina endovenosa, e d) Drenaje quirrgico y tratamiento antibitic:o intraarticular e) Cefadroxilo por via oral

i'. (_,...()

~ ,.~ ~

~
~

3) Paciente sin vacunaciones previas, es trafda por su madre por presentar convulsiones fiebre. Al exa~aprecia paC'iite en regulares condiciones e il hemodinmicamente Se realiza _ estable, con signos menfngeos positivos. arlisis del puncin lumbar que da salida a Uquido turbio, con 500 chllas, o , protefnas :: ~r aumentadas y glucosa de un(~ rese!_faa . .- sa atica. La tincin 'de Gram y tinta china ~ resulta~egativas. El diagnstico ms probable es: ~ C.. a) Meni gitis bacteriana aguda o~ --l - \ u ~Me1 n oenc . . ' al (enterqvirus) . ~ 1 . .~ e M 1ingitis tuberculosa ' ' d) mng 1a e) anencefalitis esclerosante subaguda

d~',

~
~
~'.

~
~

~
~
~
~

5) n paciente de 49 aos, consulta por ictericia y compromiso del estado general. Se constata h erbilirru.binem!!.,de predominio directo y transaminasas elevadas. Se solicita serologfa de distintos irus causantes de hepatitis, con el siguiente resultado: HBsAg(-), HBsAc(+), HBcAc(+), lg total antiVHC(-), lgM VHA(-), lgG VHA(+), lgM GMV(+), lg<a, CMV(-), lgM VCA(+), lgG VCA(-). Cul es, entre las siguientes, la causa ms probable de la hepatitis?: - ;; a) Infeccin conjunta por VHA y VHB b Infeccin co nta or 8 \JE ~hfecci con'unta por VEB y CM d) Infeccin aisla a p e) Infeccin conjunta por VEB y VHA

h .

~0.

~
~
~

~ ~

,.,
~
~

~ ~
~
~

~
~
~

,
~

~
~
.~

~ ~
~
~

~
~
~

~
~
~

~
~

2) Paciente-Y!!:J, en tratamiento antirretroviral irregular. Su ltimo recuento de linfocitos CD4(+) fue de 102, hace un ao. Actualmente cu..Sando un cuadro de deterioro del estado general, asociado a li'irit!Paresla braq iot~YFal derRsba y a convulsl@ tnico clnica en 2 acasloaes. Se realiza RMN de . cerebro que muestra una les cortical frontal izquierda de 1 cm de dimetro, con captacin de contraste en anillo, asociada a 3 lesiones similares. m s pequeftas en zona parietal Izquierda. El diagnstico ms probable es:
a) Leucomalacia multifocal rogresiva ~oxop as os e . ~c)lJremngoencefalitls criptoccica d) Linfoma cerebral primario e) Encefalitis por mycobacterias

~
~

~
~

~1-<l .

,.,
~
~

~ ~
~

~
~

~
~

"
~-

~~licitar puncin lumbar con estudio citoqufmfco de LCR y PCR para herpes simplex y decidir manejo
segn resultados
~ ~ !njcjar acicloyjr endovenoso inmediatamt!nte

13) Paciente de 21 aos, es trafdo por sus familiares por presentar un cuadro de 2 dias de evolucin de fiebre alta, asociado a compromiso del ostado general.y cefalea Intensa, al que luego se agrega con~cta paranolde y confusin, seguido de convulsin tonlcoclnlca disminucin cuantitativa. de conciencia. El examen fisico no aport rn yor m ormact n, excepto por du osa g ez e nuca. Se realiza TAC de cerebro que muestra leJ?!ones temporales bilaterales. La conducta ms adecuada, adems de aplicar medidas de soporte, es: a) Iniciar cefalosporinas de tercera generac~n, asociadas a vancomfcina

~
~

y solicitar estudio de LCR, que incluya PCR para virus herpes

r;
~
~
~
~
~

simplex . d) Administrar corticoides endovenosos, en altas dosis, asociados a antibiticos de amplio espectro e) Iniciar antic.?_agulacin inmediata con heparina, asociando ceftriaxona y gan~iclovir ~

14) Pa~ie fede 45 aos, con neumonfa tratada con amoxicillna desde hace 5 dfas. Evoluciona con diarre e solicitan toxinas de clostridium difficile en heces, que resultan positivas. La conducta ms apro ada es:
a) uspender amoxicilina + clavulnico
:.2"~~iciar vancomicina ora
~f"HJiciar metro ida

d) Iniciar cli amicina oral e) Iniciar ftriaxona endovenosa

e
~

~
~ ~
~

b) Eritem nfeccloso e) Sfnd~ e pie-mano-boca d.hSifili secundaria ~Va cea

15) A olescente de 15 aos, consulta perr fiebre, mialgias y exantema muy pruriglnosos. Al examen se ap la paciente febril, en buenas condi::iones, con exantema papulo-pustular que compromete ca za, tronco >l extremidades. Se obsetvan costras en cuero cabelludo y escasas vesrculas en troco. diagnsticJ{ms probable es: ) lmptig~loso

fA
~
~
~ ~
~

16} aciente de 12 af\os, consulta por fiebre. odlnofagia intensa y compromiso del estado general. Al e men fsico destaca faringe muy eritermosa, con exudado amlgdallno blanco, adherente y mltiples Clenopatias cervicales senslhlm:. Se lni(:ia amoxicilina oral, evolucionando con exantema macular eritematoso en tronco y extremidades. El diagnstico ms probable es:
a),.Escarl
~qnonucleosis

infecciosa e) L ucemia d) rimoinfeccin por citomegalovirus e) erpangina 1 ) Hombre de 40 aftos, mapuche, vive e1 la novena regin. Consulta por cuadro de 2 dfas de volucin de fiebre y mialgias generalizadas. muy intensas, en especial al mover los ojos. No ha presentado otros sfntomas y el examen ffsico no aporta mayor informacin. Se realiza hemograma que demuestra H atocrito:38%, Plaquetas:~~30.000, Blancos:12.100, linfocitos:22%, Neutrfilos:45%, Eosinfilo :27 ... Basfilos:1%, Monocitcs:5%. El diagnstico ms probable es:
a) Hidatidosis / b) Larva l])i~nte visceral e) Distorratosis

~ ~

~ ~
~
'"

~
~

~l"\l:lcanas1s

-~ji~o~js

18) El examen de eleccin ante la sospecha de fiebre tifoidea en un paciente con 10 dfas de sfntomas

~ ~
~ ~
~

~ ~
~

19) P VIH (+),con bajo recuento de linfocitos CD4, por diarrea acuosa, sin sem nas ,de evolucin, asociada a baja de peso de 3 kg. El diagnstico mas probable es: iarre r n tos oridium b} nfe n por lsospora e 1 In{! ccin por Mycobacterium avium complex ti) labsorcin intestinal por VIH e) iarrea por frmacos antirretrovirales

fe~te

cons~lta

sang~e, de 6

O) Un paciente diagnosticado de leucemia mieloide aguda, recibe quimioterapia, evolucionando al quinto dia, conJ!.ucopenia, con recyento de neutrfllos menor a 500 cels/mm3. Presenta fiebre hasta 38,7C, sin otros sfntomas. El examen frslc9 y los exmenes radiolgicos, de orina y de sangre, no aportan mayor Informacin. La conducta ms adecuada es: a) Iniciar ceftriaxona ms vancomic" a ~iciar eftazidrna sa na e) Controlar dianamente exmenes de sangre y orina y decidir segn resultados d) Administrar cotrimoxazol en una dosis diaria, por 3 veces a la semana, hasta que ceda la leucopenia e) Iniciar amoxicilina ms ciprofloxacino

("

f'
Pregunta recuperativa lnfectologfa Contstela en la CASILLA No 30 Un paciente esquizofrnico, en tratamiento con clozaplna, presenta f&eb~~ y decaim~to. El examen fsico no aporta mayor informacin. Se solicita un hemograma que muestra hematocrrto: 37%, plaquetas: 37.000, blancos: 2.900, linfocitos: 80o/o,~~monocitos: 13%. La conducta ms adecuada, adems de suspender la clozaplna, es: a) Solicitar e ara VIH y realizar exmenes de laboratorio en busca de un foco infeccioso -<Ji) Iniciar eftazidina gentamicina y reali2:ar exmene,s de laboratorio en busca de un foco infeccioso e) Iniciar ce azo 1na + metromdazol y reah2.ar exmenes de laboratorio en busca de un foco infeccioso d) lnlcictr.ceftriaxona + ampicilina y realizar exme laboratorio en busca de un foco infeccioso e) Iniciar elprOfloxacmo y vancom1c1na y realizar exmenes de laboratono en
2.

f'

" "'
f!'
~

Prueba 8 REUMATOLOG(A

~
~

i ) Esclerodermia /

1l El fenmeno d.e Raynaud es caracterlstico de la siguiente patologia:


b) Dermatomiositis e) Sindrome de Sjorgren d) Artritis reumatoide e) Artritis psoritica

(A
~
~

~
~

v 2) Una paciente de 40 aos presenta dolor en las manos desde hace 3 meses. Refiere marcada rigidez articular, que dura cerca de una hora. Al examen fisico se aprecia artritis de ambas muecas y de

f"

" r
~
~ f'

varias articulaciones metacarpofaliglcas e interfalnglcas proximales, de manera bilateral. El tratamiento de eleccin de la patologla descrita es: '" a) ParacetamoJ (~ J .~ ~ b) Prednisona . ;-:-. ~~\. e) Ciclofosfamjda + corticoid~ -'P' ~- . . . fJ\11(1:;- . d) Alo s AINEs @)) etotrexato
&

~
~ ~
~

3) Un nif\o de 7 aos presenta cojera de :3 semanas de evolucin, con mfnlmo dolor. Al examen fsico se aprecia dolor a la abduccin mxima de la cadera Izquierda, asociado a leve atrofia del cudriceps izquierdo. El diagnstico ms probable es: a) Artritis sptica b) Sinovitis transitoria S),Epifisiolisis de la cabeza femoral (S)IE~fermec:laeJ ele l?er:tl:lese) Sindrome de Osgood Schlater
4) Un paciente de 65 aos presenta astenia y debilidad muscular, especialmente al levantar los brazos y en la musculatura proximal de las extremidades inferiores. Tambin refiere que presenta cefalea frecuente y dolor al masticar por mucho tiempo, lo que lo obliga a comer 1C:mto. En sus exmenes destaca anemia normocfcita normocrmlca, con VRS: 97 mm/h. El diagnOstico ms probable es: a) Lupus !:l_ Polimiositis (!ti Al:te.ritis de c!Yms gigantes d) Poliangefbs microscpica e) Panarteriris nodosa

~ (!A
~

p.
~ ~

~Giomerulonefritis /

5) Cul de las siguiente~es una manifestacin extraarticular de la artritis reumatoide?

L/

-------

"
~

b) Miocarditis / ....;:e) Mononeuritis mltiple d) Epiescleritis e) Derrame pleural /

"'

(A
~

(A
~

"'

6) Una paciente de 26 aos presenta astenia, febrcula ocasional y baja de peso. En su examen fisico slo se asculta un soplo carotdeo derecho y una clara aslmetria de los pulsos radiales. El diagnstico ms probable es: . . a) Pelviespndiloartropatla con compromiso arterial b) Arteritis de clulas gigantes e) Enfennedad de Wegener @ Arteritis de Takayasu _,..- e) Lupus eritematoso sistmico 7) Cul de las siguientes asociaciones entre marcador inmunolgico y patolograes correcta? a) Hipocomplementemla- Artritis reumatoide .,.. b) Factor reumatoldeo- Es on ullosante _)Ct e) Antic op asma de neutrfilo en patrn citoplasmtlco - mlcropoliangeltis (PAM) d) Anticuerpos anticentrmero - Dermatomiositis ~Anticuerpos anti-rlbonucleoprotelna - Enfennedad mixta del tejido conectivo 8) La causa m~uente de codo doloroso es: (J Epicondilitis ,. '6) Esguince . e) Artrosis d) Atrapamiento cubital e) Tendinitis tricipital

't 9) Un paciente de 68 aftas, diab~o e hlp~. bien controlado, presenta dolor en la cadera .,. ' izquierda, desde hace algunos meses,-roque le dificulta la marcha~ Al examen fislco presenta dolor a la flexin y abduccin extremas de la cadera. Se solicita radlogr-afia de cadera que demuestra disminucin del espacio articular y un ostoflto acetabular. La conducta MS.adecuada es: ~ Solicitar anticuerpos antinucleares y factor reumatoideo @.Iniciar paracetamol / e) Iniciar metotrexato '>c!tlniciar prednisona "> 41"niciar diclofenaco

>-

1O) El test de Schlrmer positivo es caracterfstico de: a) Enfennedad de Behcet b) Sarcoidosis e) Sindrome de CREST d) Dermatomiositis

~Sindrui~

SJOrgren

p
ti lZI

11) Una paciente de 45 aftas presenta dolor de larga data en el !hombro derecho, que aumenta con la actividad y que en los ltimos 1O dfas se ha hecho mucho ms Intenso, al punto que le Impide la movilizacin. Al examen fsico est en buenas condiciones, con signos vitales normales y presenta intenso dolor tanto a la movilizacin activa, como a la movilizacin pasiva del hombro, sin signos jnflamatorios. El diagnsticq ms probable es: ~ Bursitis Tendinitis del manguito de rotador e) Artrosis - ,. ,1 / d~ -::> /tAtM-~~ ~-~~ e) Tendinitis blclpital

---

~ 12) Y.na paciente resenta artralgias, con febr~c~~asional. Sie solicita hemograma que demuestra
hematocrito 0 VCM: 87, HCM: 32, blancos.~,.., lnfocltos: 25%, neutrfilos: 68%, monocitos: 7%, plaquetas. 53. O. 1diagnstico ms probable es: a) Arteritis d emporal )<' Granulomatosis de Wegener rioglobulinemla upus eritematoso sistmico _ Poliangeltis microscpica
)<

~
13) Cul de las siguientes zonas son MENOS frecuentemente afectadas por artrosis?

~
~M~-'

~ ~

'b) 1\rticulaciones tnterfalngicas proximales


e) Articulaciones interfalngicas distales ~t d) Rodillas ~ e) Columna vertebral i i

~ ~

~ ~ ~

14) Una paciente de ~os, con antectadente de colon Irritable, presenta fatigabilidad y dolores generalizados. En su examen fsico pre:senta dolor a la palpacin de los msculos del cuello, dorso, cara interna de las rodillas y cara lateral de los codos. El diagnstico ms probable es: Polimlositis ~ b Fibromial la olimialgia reumtica d) Hipotoroidismo e) lupus

~
~

(!'
~ ~ ~
~

15) Cul de las siguientes asociaciones~, entre marcado7inm nolgico y patologa, es INCORRECTA: .~) Anticuerpos anti-nuclolo - Esclerodem1ia b) Anticuerpos anti-DNA de una hebra- Lupus por .drogas e) Anticuerpos anti-Ro - Sndrome de SjOmren ~ d) Antlcueroos...anti oiteplas::a de ne rtr:filo--ARemis-Ge-la1eiporal e) Anticuerpos anti-proteina P- Psicosis lllpica /
16) Un hombre de 67 aos presenta dolor intenso y aumento de volumen delc"dO-i~q~i~~e 1 dfa de evolucin. Al examen fsico se apreca aumento de volumen, con derrame articular, ~a y aumento de la temperatura local. La conducta ms adecuada es: @ Realiz r artrocentesis solicitar io. de liquido articular / b) Iniciar AIN y co e icina e) Solicitar hemograma y hemocultivos d) Solicitar radiograffa de codo izquierdo e) Solicitar cintigrafla sea
17) Cul de las siguientes alteraciones NO suele aparecer en los pacientes con artritis reumatoide? a) Mano en rfaga ~ __., ..._...,.. b) Dedos en cuello de cisne ~ ~- Dedos en sastre S-t ~) Ndttles-.del:feberc.!Jl x e) Ndulos reumatorcfeos S:.,:

~
~

"' ~
~

~
~

18) Qu tratamiento debe indicar ante un paciente de 57 aos que cursa actualmente con un ataque de gota, confirmado con el estudio pertinente? a) Alopurinol
~AtN&s--

~
~
~

C} Antibiticos y AINEs
d) Alopurinol y AINEs
e)Aiopuri~

~
~
~

19) Un paciente de 32 aos presenta una rinosinusitis crnica de dif[cil manejo, asociado a epistaxis frecuente. En el ltimo tiempo se agrega compromiso del estado general, fiebre, hematuria y oliguria. El diagnstico de sospecha es: a) Prpura de Sch6enlein-Henoch f!l)G~ matosls de We ener CJ Vasculitis e urg-Strauss "' d) lupus sistmico eritematoso e) Panarteritis nodosa

~
~

~
~ ~
~

(_

20) Una paciente de 20 aos presenta malestar general, calda de cabello y lesiones faciales eritematosas. Al exam_en fislco se aprecia eritema en los pmulos y en el dorso de la nariz y algunas lceras en el paladar duro y en las mejillas. El diagnstico ms probable es:
a) Dermatomiositis
~Vas.culitis de vaso peque'\o (Y.~upus

a) Enfermedad de Behcet e) Esclerodermia

~
Ensayo 1 sexto ao SEG~N A PARTE 1) U de 5 aos presenta lesiones redicivantes, de tipo papul,r_y_;, .l!e!'_e microveslculares en las me las, fosas cubitales y fosas popliteas. El diagnstico ms probable es: a), rticaria "Dermatitis at9pica ) Dermatitis p contacto alrgica d) Prrigo insectario e) Sarmy' 2) u116aciente sufre una cada de altura, cayendo sobre los talones y evolucionando con importante dol ' en la zona de impacto y en la zona lumbar. Se realizan radiografias de calcneo que demuestran fr tura del calcneo derecho, la que es tratada con analgesia y es inmovilizada con una bota abierta yeso, a la espera de cirugfa. Sin embargo el paciente persiste con dolor lumbar. La conducta ms decuada es: ~olicitar radiografas de columna lumbar b} Solicitar resonancia magntica lumbar e) Indicar kinesioterapia d} Indicar uso de faja lumbar e) Indicar reposo absoluto .---. 3) L.a presencia de hipoalgesia de extremidades inferiores, con hiporreflexfa ..b~:o~~p~:~ibucin distal de las~~~o extremidades sugiere el diagnstico fle: .: e) jUlfermedad de motoneurona. ~5 mononeuropatfa mltiple.
e)~.

e"'

(!~"

~io

,...

(fiA

f!!"'

r~
(9\
(lf!IA

(la\

pamslaleve,~oestesia\

....

"

w~~.

119\

~
(!fA

~
~

f"h
(9\
~

4) l de los siguientes antecedente IEt ~ebe. hacer sospechar .un de testino irritable? ~ a) epsiciones con mucus b fernancia entre constipacin y diarrea ./.. istensin abdominal "d~lza de peso A Anemia ferropnica

diagnstic~. un sndro'!!!
--- ...... - ----

~
~

5) U nio de 5 aos, obeso, presenta pcliurfa y baja dp pesg. Se solicita glicemia de ayuno que re Ita 250 mg/dl. La conducta ms adec:uada es: a nicf~r dieta y ejercicios y controlar en 3 rneses . tiar dieta, ejercicios, solioRar-hemeglebina-glicosilada,-controlarctm resultado Iniciar metfermiRa-y-controlarerr3 mese~; d) Iniciar insulina lenta en 2 dosis, una matinal y otra nocturna /e(Hospitali~~r para inicio de insulinoterapia en esquema intensificado
6) Un p~i,nte de~os, con antecedente de insuficiencia cardfaca crnica, secundaria a una cardioR ta hipertensiva, de larga data, en tratamiento COillUJ!llil.PriJ,.espironolactona, hidro oroi~~z~ aspirina y atorvastatin~, consulta por aumento de su sintomatologa y progresin de su d" nea, que actualmente aparece ante' mfnlmos esfuerzos. El examen fsico demuestra FC: 87x', ar, PA: 130/80, crpitaciones pulmonares escasas bibasale!!,_y un RR2T sin soplos. La conducta s adecuada para mejorar el pronstico en este paciente es: ---~ } Cambiar el enalapril por un inhibidor del receptor de angiotensina 2 b) Iniciar digoxina e) Iniciar propanolol d) Iniciar furosemida -~iciar caiVedilol

fl"
~
~

(!$\

tOI
('1/h

te\

,.,
~

te\
(C\
~

J.
a

~ ./"'" . O) E.~hile, la asociacin de bocio e hipotiroidismo corresponde con mayor frecuencia a: areneiad~yOcfo. 1teracitsn-e-nztmat1ca congnita. . ~~f'-1~ , , , . ('..Q~ roiditis cr!lica. --\--.. H-~d ~ vv-..
'

rO (

&J.

) GAeer-de broldes.

e) hi otiroidismo secundario.

los siguientes casos es ms sugerente de una ii.rQtlJitls subaguda de Quervein? mbre de 35 aos con bocio difuso, de consistencia aumentada y slntomas de hipotiroidismo ujer de 59 aos-een-tir~. pero conslstenchi-dora-;de-maera-dffusa ) Mujer con bocio difuso. exeftalmo-;lli1Xe"tmm1:fpreti15li:il-y-sfntol'ml~1nrh~liDl'lo d) H~bre de 55 aos. con bocio nodular, asociado a pliadenopat!~. cervicales , ~) 1\lujer d.e 30 aos con dolor cervical y otafgia. asociado a. ~.oc~.~~ la palpacin

21

~~de

~2 'es la: ms til para evaluar ..respuesta al ~tam.iento ..en.Mn..P..~.9..l~nte. ~IJ. \lru~taque.agudo de\ l.ni.~dio a .. . ... - --:--..-----.. .
volucin de la frecuencia cardiaca oxir1}.tria de pulso e) mopificacin de las sibilancias d) o servacin del uso de los msculos accesorios . .. e) eterminacin del flujo espiratorio mximo {PEF). ~ 3) El factor pronstico ms importante en el cncer de laringe es: a) El tamao del tumor b) El tipo histolgico '. ~a presencia de adenopatias cervicales comprometidas /d) El grado de d1stonla y estridor ----.. -. e) La persistencia del hbito tabquico 24) Up hombre de 50 aos presenta P.pulas y pstulas en la c~ra, con una base eritematosa con a~gnas telangiectasias. Adems presenta chalaziones con cierta frecuencia. El diagnstico ms r ablees: Acn . ) Lupus eritematoso sitmico ) Dermatitis atpica ) Dermatitis d~ contacto fototalrgica /Roscea

J J

25) Paciente de 25 aos, consulta por haber notado aumento de volumen testicular derecho. La 6rafa testicular muestra tumQT..d.uspecto slido en testiculo derecho. La conducta inicial ms cuada es: ~ . Control ecogrfico en 6 meses Realizar biopsia por puncin del tumor Realizar tumorectomia y biopsia ) Splicitar marcadores tumorales (AFP, HGC y LDH) y decidir cirugla segn resultados ~)"Realizar orquiectomla derecha, por vla Inguinal, independietememte del resultado de los marcadores tu orales.

6) La et~ologa de las d~!,@C~.l.!:!~cio_!!~S 1ru:d1as, en el registro estresante, es: ) Reacc1n vagal. producto de compresin de la cabeza fetal b) yompresin del cordn umbilical p)'Hipoxia fetal ,,... d) Flujo r verso en distole de las arterias umbilicales e) Au nto del retomo venoso fetal, producto de la compresin do la placenta 27} aciente con mieloma mltiple, evoluciona con poliuria, deshidratacin y compromiso de co ciencia. En la analtica sangunea destaca Calcemia:16mEqll. La primera medida a tomar es: a) dministrar magnesio endovenoso ---------..--.., Administrar suero fisiolgico e) Administrar bifosfonatos endovenosos d) Administrar furosemida endovenosa e) Administrar corticoides endovenosos '

14) Un paciente de 50 aos, con antecedente de una ne:fm_c1~ derecha por un trauma renal ha_c..e.. ao!J, se realiza una ecografia como parte del estudio de un dolor abdominal, de caractersticas fu~ ionales, detectndose un tumor renal izquierdo slido, de 2 cm de dimetro. El examen fsico no ' ap 11a mayor informacin. se realiza TAC con contraste que muestra la lesin compatible con un t ~_r de clulas renales, sin adenopatras. La conducta ms adecuada es: Realizar nefrectomia radical Realizar nefrectomfa parcial Solicitar alfafetoproteina y Ca 19.9 d) Iniciar radioterapia e) Solicitar resonancia magntica nuclear 15) Hb.m bre de 65 aos, portador de ;.pjlse~ulmonar tabquico. Ingresa con historia dtDm~.!!@. ~ aun<ento de. su e~ct~~acin habitual) fiebre. Es traido en ambulancia donde reci be 02, 6 lt 1m in. rante.. el trayecto, presenta c~pr_omi s.o..de..concieRGia,. c~usa ms probable de esta alteracic)n de conciencia es: t,.hijJoxia ggtve ... . -.... _
hJJ?erap~

e) neuiOtorax. d) shock sptico. e) meningoencefalitis 16) .Un hombre de ~-af.ios consulta por d~suria de esfuerzos, con disml.oucin.del..calibr-e miccional,. ni ~turia y goteo terminal. El examen fisico ~r informacin. El diagnstico ms probable

A
)

Cncer de prstata Estenosis uretral ' Hiperplasia benigna de la prstata / d) Infeccin del tracto urinario e) Uretritis por clamydia

Cul de las siguientes patologas e.s ms frecuente ermatomiositis ........____./ . ..# spondilitis anquilosante ;:;?' ' e) Artritis reumatoide d) Artrosis e) L~pus eritematoso sistmico

e,rth;:~~:~ue en mujeres?
.

1B( A qu tipo de estudio corresponde el siguiente enunciado?: S~ seleccionan 300 pacientes diagnosticados de artritis reumatoide y se asignan aleatoriamente a dos rup.os de 150 pacientes cada uno. El primer grupo es tratado con infllximab y el segundo es tratado ~n metotrexato. Se comparan las tasas de remisin clnica y los efectos adversos. j Casos y controles ) ~ohortes ,.,c}Ensayo clnico controlado d) Ensayo de campo e) Transversal

CFIII.,kvoluciona con 19) Un paciente de 70 aos, cardipata cor.QD.ario, muc)"purulenta, agregndose cTISeade1:ep'o_::o y rpdo deterioro del e;tado general, CO.lJ..lO~..Q.R~s;_y_ de~ _!ientacil).~ examen fsico se aprecia plido, con extremidades fn_ y puls.os .Q.~ as .blles. Se co stata F<:;.(12~, PA: !36/31 mmHg, con RR3T y MP+ con crpitos bila~s, mayor en lavase d recha. El monitoreo electrocardiogrfiGo demuestra taquicardia sinusal. Se instala catter de Swananz, obtenindose PVC: 21 mmHg (VN: 0-8), PAP: 45/22 mmHg (VN: menor a 35) y PCP: 33 minHg , , (~N: menor a. 1 8~. ~1 diagnstico ms probable es: / ,.a Shock card1ogemco 1 b) Shock sptico e) Shock hipovolmico d) Shock anafilctico e) Shock medular

ei~

tQ.~.~-exp~J;:Jgr.acin.

..

~
~
~

20}~-'hile, la asociacin de bocio e hlpotiroldismo corresponde con mayor ffecuencia a:


reneiadeyodo. teractotce-Zi'iatlca congnita. oiditis cr~ica. . . .

...

rO

oaReer-de liro1tles.
.21 a

~ ~f""'-1-.S:J \ ' ~~ . H~" ~


.

~
~

J..J

e) hi otiro.i~dismo secundario. mbre de 35 aos con bocio difuso, de consistencia aumentada y slntomas de hipotiroidismo ujer de 59 aos-een-tirotae511~ tamano nomnn. pero conslsttmenrdara-;-demarael1f"Citfusa ) Mujer con bocio difuso, exeftalma;-~"thmllfPFetiblalystnttmra"S1te"h1p"tmlrnKiim\o d) H~bre de 55 aos, con bocio nodular, asociado a poliadenopati.~. cervicales ... . ~) Mujer ~e 30 ~os con dolor cervical y otalgia, asociado a.ifoc~.~~.~ la palpacin

~"~t de los siguientes casos es ms sugerente de una ii.mi!Utis subaguda de Quervein?


.

~
~

~
~

22 El .m~dio ms til para evaluar respuesta al !ritaoiiento ..en.u.n~R.2.1~nte ~Wllr'Jl.ataque.agudo.de\ a ~s la: ... _. ---...--..- votucin de la frecuencia cardiaca oxim.tria de pulso e) mo;:tificacin de las sibilancias d) oljservacin del uso de los msculos accesorios. .. e) eterminacin del flujo espiratorio mximo (PEF).

..._
Q

~
~
~
~

3) El factor pronstico ms Importante en el cncer de laringe es: a) El tamao del tumor b) El tipo histolgico '. ~a presencia de adenopatfas cervicale~ comprometidas /d) El grado de d1sfonla y estrldqr ~ ... e) La persistencia del hbito tabquico 24) Uf' hombre de 50 aos presenta P.pul~s y pstulas en la C_!ra, con una base eritematosa con alt!~nas telarigiectasias. Adems presenta chalaziones con cierta frecuencia. El diagnstico msrr,_~able es:

N~

eritematoso sitmico ) Dermatitis atpica ) Dermatitis de ~ontacto fototalrgica jfRoscea

>~Lupus

J
'

25) Paciente de 25 aos, consulta por haber notado aumento de volumen testicular derecho. La ecqgrafa testicular muestra tumQr..d.~to slido en testculo derecho. La conducta inicial ms .cuada es: -. Control ecogrfico en 6 meses Realizar biopsia por puncin del tumor Realizar tumorectomfa y biopsia J Piicitar marcadores tumorales (AFP, HGC y LDH) y decidir cirugia segn resultados ~YRe~lizar orquiectomla derecha, por vi a inguinal, independietemelnte del resultado de los marcadores tu orales.

~
~
~

6) etiologa de las d!...~-~racion!!-!rd.as, en el registro estresante, es: ) Reaccin vagal, producto de compresin de la cabeza fetal . b) 9ompresin del cordn umbilical . .o)'Hipoxia fetal ~/ d) Flujo r verso en distole de las arterias umbilicales e) Au nto del retorno venoso fetal, producto de la compr~sin dn la placenta 27) aciente con mieloma mltiple, evoluciona con poliuria. deshidratacin y compromiso de co ciencia. En la analitica sanguinea de~t~~a Calc~(ni~:16mt;q/l. La primera medida a tomar es: a} dministrar magnesio endovenoso ----------. ,. Administrar suero fisiolgico e) Administrar bifosfonatos endovenosos d) Administrar furosemida endovenosa e) Admini~trcu corticoides endpveno~os

La

~
~
~

~
~
~

..-,
~

.,
~

r
/
1

~
28} \\

~ ~

~ ~

~ ~

y
7

wa. Al examen se aprecia dolor a la palp.a.ci.D abtfp:H...:~I, especialmenteen la fosa ill.ru:_a... ~rda, as.ociad.o_a resistencia muscular. Qu medi~.~~omarra frente a esta paciente? # )) d1car rgimen hv1ano )JJ ~ olicitar TAC de abdomen y pelvis /e) Iniciar antibiticos de amplio espectro ~eallzar.:.<;:olonoscopia de urgencia e) Adminlstrafan"SigeSfcos---- ~--

6il~ paciente d~;os consulta por flebre y dolor en el hemiabdomen inferi.or, mayor a la

:.

~ (!!'

~ !1A

29) Un paciente de 37 aos ha presentad;'>4 crisis de pnido en el ltimo mes, sin poder Identificar una causa aiJl. Refiere estar muy asustado (!n lo que le- pasa, ya que teme que pueda sufrir un infarto en algu a 'las prximas crisis. Debido a esto ha limitado sus salidas de la casa y evita lugares como el m r y las calles muy transitadas, por miedo a que presente una crisis y nadie lo ayude. El nstico ms probable es: ) Trastorno generahtada /)#.'Trastorno defans1e a /.e) .EG~ia-seeial d) Tr~rno de el?~co e) Trastorno por somatizacin

de aASi:gag

30) Un paciente con antecedentes de un traumatismo de uretra presenta uretrorragia y retencin de orina con globo vesical. Cul es la conducta inmediata? -a) Solicitar un~se~agnosuca ~Realizar .una cistostomia suprapbi~ ~ndicar cateterismo uretral intermitente ,. d) Solicitar ecotomografia pelviana e) lnsta~,.-soda Foley uretroves1a1 31) t)I. paciente de 63 aos se real~monectomia del lbulo medio derecho como parte del t (amiento de un adenocarcinoma bronquial. A qu tipo de herida corresponde esta ciruga? erlda limpia ~~da limpia contamin~.}! ~ df Henda contaminada . . d) Herida suela e) Herida infectada 32) Cul de las s s opciones corresponde a una alternativa vlida para la alimentacin lctea de u lactante 8 meses que no recibe pecho materno? e purita al7,5 o, ms azcaJ:..al-25o/cr,-ms-aeelte-al1 54_ che purita al7,5%, ms azcar al2,5%, ms cereal al3% leche purita al7,5%, ms azcar al5%, ms cereal al1,5% d) leche pu~ita al 10%, ms azcar al 2,5%, ms aceite al 1,5% e) leche purita al10%, ms azcar al5%, ms cereal al3%
33} LJl'iitencin primaria de salud, en Chiloe, se financia princip~lmente a travs de:

~
~ ~

~ ~ ~ ~ ~
~

~ ~

~
~

. 17% del sueldo de los trabajadores 1aporte municipal El aporte de los empleadores e) El ~ago de los pacientes
.

r aporte per cpita

f!*'
~
~
~
~

ul de las siguiente~' una indicacin de hemodilisis de urgencia? (l.J iponatremia grave, que no responde al tratamiento mdicov ) Hiperkalemia grave, que no responde al tn:~tamiento mdic~ e) Pericarditis urmica. ._./" ~ d) Edema pulmonar que no responde al tratamiento mdico e) Acidosis .grave, qu~ .no .responde al tr~t~~i~nJp .mdicov . . . ..... ......... k..
~

.. ..

(G\

'!"
~
~

~)

~
~

(---- --.. -.......

35) Una mujer, c~rsando embarazo dL39 semaoas?consulta en urgencias _por contracciones uterinas persistentes. S constata dinmlc;a uterinadp 4 en 10 minutos, con feto en ceflica, cuello uterino com leta nte borrado, dilatacjn cendcal de 5 cm y membranas fntegras. El monitoreo fetal resulta tr qull dor. La conducta ms adecuada es: ~ ____,... E ar a domicilio e indicar reconsulta en algunas horas ospitalizar en preparto y dejar evolucin espontnea

~
~

~
~

e) IAieief-eeeleFasiR eeeitelca
d) Realizar rotura artificial de membranas e) Re~lizar cesrea 36) Una mujer de 30 aos consulta por r:nastodinia especialmente intensa durante los das previos a menstruacin y que cede con sta. Al examen fis1co se constatan mamas dolorosas a la palpacin, de consi ten9iaindurada, con varios ndulos pequeos, bilaterales. El diagnstico ms probable es~ a) E~~mdad de Paget b) ficer de mama multicntrico astopatla fibroqulstica d) Mastitis linfangitica e) Hiperprolactinemia

~
~
~

la

~
~

""' ~
~
~

~
~

~
~

. .)

.t8tn Un nio de 6 meses de edad cae de la cama, golpendose la cabeza. La madre acude muy ( procupada y refiere que inicialmente el lactante llor por variios minutos, pero luego se tranquiliz. Al examen fislco se observa en buenas condiciones, con t~tuscylar normal, reflej9s conservados y ausencia de si,gnos focales. Slo se observa un pequeo hematoma en la zona occipital. La conducta s adecuada es: Enviar a domicilio, e_xplicando los signos de alarma para reconsultar ~ Solicitar radiografia. de crneo ~) Solicitar TAC de cerebrQ d) liciar AlNEs y derivar para atencin ambulatoria con neurlogo e) Iniciar racetamol y fenobarbital, controlando en 24 horas .
ul de las siguientes alternativas spone una n mona adquirida en la comunidad? -- . eumonla por Klebsiella BLI;E aFi de 100. . e) Frecuencia respiratoria de 29 respiraciones por minuto . d) De rame pleural asociado e) mP.romiso de los lbulos inferior y medio derechos

~
~

~
~

~
~

39

mY~;~~~n un paciente cursando .con una

~
~
~

~
~
~

1primer parmetro que se altera en un paciente en shock hipovolmico es: el estado de conciencia. lb)Ja presin ~rterial. 6) el pulso. . d) la p ffujln tisular. e) la ur.sis 41 nio d~s prese.nta un comportamiento agresivo y desconsiderado con los dems. Vive e su abuela a quien le roba en algunas ocasiones. Ha participado en actos de vandalismo, estruyendo vidrios y faroles, pelea con frecuencia con otros menores y habra cnsumido alcohol en varias oportunidades. El diagnstico ms probable es: ATrastorno de condpctab) Trastorno oposicionista desafiante e) Trastorno de personalidad limltrofe d) Depresin infantil e) Dependencia a alcohol

~
~

~
~
~

------. ~ --- .

~ ~
~

,
~

'

~
De los siguientes antibiticos Cul indi~aria en una infeccin urinaria con urocultivo pendiente en a ntuJer embarazada? } cefazolina
b~

~ !"'

~ ~

e} gentamisiAa d) eiprofloxacinoe) cloramfenicol 43) Un hombre de 45 aos es mordido por el gato de su vecina, mientras le ayudaba a atraparlo, para llevarlo a control con su veterinario. Al examen fsico se observan varjos rasguos en los anteb.r.azos y 4 lesiones puntiformes en el dorso de la mano y los dedos derechos sin signos de infeccin. La conducta ms adecuada es: . . . A Realizar aseo y curacin de .la herida, administrar la vacuna antitetnica, indicar amoxicilina ms cido ~vulnico y observar al animal por 1 O das b)~ealizar aseo y curacin de la herida, administrar la vaC!ma antirrbica e indicaf-CloxaGiUna J ,..efRealizar aseo y curacin de la herida, adrninistrarJa vacuna antitetnica y observar al animaf por 10 das, sin necesidad de antibiticos d) Realizar aseo y curacin de la herida, administra la Vacuna anbrrablca e ltidica &iiLvxieiliRa mscido clavulnico e) Realizar aseo y curacin de lc:rhefiea, administrar las vacrula&-aRtiteUmir.a y antirrbica e indicar antibiticosy,.e amplio espectro 44) ac nte de 33 aos consulta por cefalea. de 1 ao de evolucin, de carcter opresivo,de lo acin temporo-occipital, irradiada al cuello y que suele ser ms intensa en la~s. El examen fi "co slo demuestra leve sensi~ilidad a la palpacin de los msculos cervicales posteriores. El iagnstico ms probable es: a) Neuralgia de Arnold b) Migraa con aura e) Migraa sin aura .MCefalea tensional t;;) Fibromlafgia. 45) U p~iente de 65 aos, fumador de.2.D cigarrillos al dfa, ha presentado 4 episodios de neumona en 1 ;llltimossme~es, ~ue han respondido bien al tratamiento antibitico, tdos ellos localizados en el 1 bio inferior izquierdQ. El examen ffsco no aporta mayor Informacin. 1;1 diagnstico ms p il'abte es: .. . Bronquitis.. crAica ) EPOC } Cncer bronquial d} Bronquiecta&ies e} ytamsis:.pulmenar ,

<!"

~ ~ ~
~

~
~
~ ~

~ ~

e
~
~:

~
Alil\ \

~
~

Anticuerpos antinucleares Anticuerpos anti-Ro y anti.:La Anticuerpos anti DNA de una hebra d) Factor.reumatodeo e) Co ,~(emento C3 y C4

6li~:l de las siguientes exmenes permite predecir el grado de actividad lpica?


.

fik'
~ ~
~

($A
~

47) de las siguientes es ms frecuente en el recin re n nacido de trmino? StTdfeFRe de aspiraGiR e niecotilo xardiopatias congnitas ipocalcemia .fleliglob d.ia e).-.Cefa!chematea 11a Y

-~t~1l

coo~diclones

nacc~-;~~d que ~n el

~
(f!.\

~
f!!!"
~

~
~

M":)
/

,r)/:.~
/11

48) Calcule.. el Odds Ratio del consumo de cigarrillo para el desarrollo de cncer de vejiga, con la siguien~~informaci6n (Valor aproximado): . // Cncer de Ve a 100 50 150 Sanos

100 300 400

~~~;AL 350
550
--

~~{?1 ~ 1 ~
1

.J/

5"'

a) 0,17 b) 2,00

o o \

-----

i: ~~o.
lS
O

~ .pA?J
r-.o

e) 2,67
d) 3,50

------~ ~

6,0 /
49) %ul de los siguientes hallazgos indica una mayor gravedad de la Infeccin menlngoccica? a eucocitosis mayor a 15.000/mm3 ~sencia de leucocitosis en el liquido cfalo raquideo. ;:r:.~~pura de menos de 12 hrs. de evolucin. d) Velocidad de sedimentacin 'globular aumentada. e) Presencia de artralgias o artritis.
50 'Una l~ctante de nueve meses que asiste a una guardera se presenta ego fiebre e irritabilidad y se tr cciol) de la orQia izguie.rda. Se observa la membrana timpnica Izquierda abombada y eritematosa,

?!

m ~vimiento con el otoscopio neumtico. El resto del examen fisico es normal y la nia no tiene t edentes de otitis. Cul es el esquema antibitico ms apropiado? oxicilina 40 mglkg/dia durante 1Odas moxicilina 80 a 100 mglkg/dia durante 1Odas ) oxicilina 1 cido clavulnico, 40 mg/kg/dia, durante 1 O dias d) Amoxicilina 1cido clavulnico. 80 mg/kg/dia, durante 1O di as e) Azitromicina, 1o mglkg el dfa 1 y 5 mg/kg del da 2 al 5
Paciente VIH, en tratamiento antirretroviral.irregulf!r. Su ltimo recuento de linfocitos CD4(+) fue de hace un _ao. Actualmente cursando un cuadro de det~_r!Qro del estado general, asociado a emiparesja''braquiocrural derecha y a convulsin tnico clnjca en 2 ocasiones. Se realiza RMN de cerebro c(ue muestra una lesil:l subcortical frontal izquierda de 1 Clll de dimetro, con captacin de cont ste e , asociada a 3 lesiones similares, ms pequeflas en zona parietal izquierda. El di nstico ms probable es: Leucomalacia multifocal progresiva .. Wfoxoplasmosis cerebral /c)Meningoencefalitis criptoccica d) Linfoma cerebral primario e) EncefalitiS por mycobacterias

lgico se suele elevar en los pacientes con este tipo de tumor? Ca 19.9 b) Ca 125 .. ) tigeno caminoembrionario Alfafetoprlena } 'rriti,:' decarboxilada

~ 1-'I~~atocarcinoma se asocia a cirrosis heptic~ de distintas etiolog(as. Qu marcador

a diabetes pregestacional se diferencia de la diabetes ge~stacional, ya que la primera presenta ayor riesgo de: a}yolihidramnios ~ Malformaciones congnitas e} Macrosomia fetal d} Poliglobulia neonatal e) Hipoglicemia neonatal

,./

5 ) t;t~fndrome clnico caracterizado por microc.efalia. restriccin se"era del crecimiento /onalas cardiacas, est relacionado t:on el abuso materno de: anfetaminas tabaco benzodiazepinas d) marihuana A alcohol

intraut~rino y
+

..
.

.. .
.

55) Un paciente de 44 aos sufre trauma ocular derecho, con golpe de alta energfa. Al examen se observa discoria, enoftalmo y marcaiD:I;111smlnoclon de la agudeza visual. La conducta ms adecuada es: ~ a) lnic\ar an~lgslcos, realizar seUe--ocular-t;on ungOento antibitico y derivar a oftalmlogo de manera urgente b) R ~li2;afaseo ocular con abundaRte-stiero fisiolgico, realizar sello ocular con ungOento antibitico y derivar . al ahilogo de manera urgente e) icar antibiticos endovenosos.y-selifies-enti~s. instalar proteccin ocular no compresiva y derivar al almlogo de manera urgente a) Realizar aseo ocular con abundante suero fisiolgico, instalar proteccin ocular no compresiva y derivar de man~ra urgente al oftalmlogo, bajo antibiticos y analgsicos orales . )4(dministrar analgsicos y antibiticos endovenosos, instalar proteccin ocular no compresiva y derivar de ~anera urgente al oftalmlogo
_,

56) Un~ paciente de 24 aos reprueba el ao acadmico, atrasndose un semestre. Evoluciona con nimo.alfrico e irritabilidad, teniendo cJigunas peleas con sus padres. Adems sale menos con sus ami ;;ts y ha descuidado arcialmente su a ariencia fsica. No presenta cambios en el peso ni en los h (tos de sueno. 1 diagnstico ms probable es: Distimia }-9epresiJ:I.. e) Trastorno adaptativo d) T.J:astOJ:t:IO de aosieea~eneraltZaffif' r e) Hipomanla
57) Un nio d~ 4 aos consume una sus~!ncia.desconocjda, evolucionando con agitacin . psic moto.ra, seQ!Jedad bucal, midriasis, ileo intestinal, taquicardia, sudoracin y confusin. El cuadro cli co _9rrespnde a un sfndrome: . N rtlco piceo uscarfnico ...- d) Antlcolinrgico e) Adrenrgico

'
./

,Aff

58) Una pareja consulta por infertilidad dEt un ao de evolucin, a pesar de mantener relaciones sexuales con ~irta frecuencia y de no usar mtodos anticonceptivos. ~ es yo hgmbre sano y tiene ''" hijo_ de 3 aijos con otra mujer. Ella es una mujer sana, pero refiere -~iciQ_~m.~nstruales irregulares, que llegan cada 2 a 3 meses. El examen fsico de ambos es normal. La causa ms probable de esta rtiJicad es: sculina ndometriosis uboperitoneal Anovulacin .. e) Gentica ./.'

..1

59) 32 aos, antecedente reciente facio braq crural izquierda. Al examen se detecta arritmia completa. Cul es la enfermedad causal ms pr ble? suficiencia artica Embolia pulmonar ) Tuberculosis pulmonar ) Infarto agudo del miocardio ~)~Estenosis mitra!

Muj{~~r,

~,con

~-;rti;Ji) pr~sentctbemJparesia

, ''?

60) Un paciente calda nivel, con imposibilidad para caminar y mucho dolor. Al examen~ preset. a..cortaQJient~, leve abduc.cin y rotacin externa del muslo derecho, con pulsos y sensibilic!d distales...eoiisef,Vados: . e."@. ~ ta ra agrafia d~ caderas que demues fractura de cuello femoral derecho, .desp(azcfc:@ (Ga . La condcta ms adecuada es: a)" Inmovilizar con yesq pelvip~dio yadmiol~trafanalgesia endovenosa . b) Estabilizar, administrar analgsicos~ "solicitar exmenes de sangre y orina y realizar inmovilizacin con yeso pelvipedio . e) Iniciar analgesia y antibiticos endovenosos y luego realizar ciruga de reseccin de la cabeza femoral (9J!ugla de Glrldestone)) .... , .,", . .... , ... . . . .. . . . . . . .. i?!""~stabilizar, ~dninistrar analgsicos, solicitar exmen.es 'sangre!Y 6rinr;~ Y realizafosteosh1tesls con o. ,... ' illo din!J-lico de cadera (DHS) o , . stabi.~jzr, administrar analgsicos, solicitar exmenes de sangre y orina e instalar prtesis parcial de era/

ct,;~oa~o~--~ufre

quedand~

de

~o de d.os y medio aos en rinorrea mucopurulenta por fosa rnanas de evolucin. El diagnstico ms probable es: initis mucopurulenta prolongada . }'!jlPosinusitis crnica derecha . ~- c~erpo extrao de fosa nasal derecha d) sinusitis subaguda derecha e) tumor maligno de fosa nasal derecha

~asal derecha, de mal olor, de tres


.

62) Una muj,r-"c.e 35 aos, consulta po~que presenta problemas para manejar. Cada vez que llega a un semforo;da luz est en verde, debe seguir manejando en linea recta, an cuando necesite doblar en esa ~s ma, de modo que slo puede doblar cuando llega a la esquina y el semforo est en rojo o cuan no hay semforo. Esto le hace perder mucho tiempo y ella sabe que esas ideas son pro eidas por su mente y que son ridiculas, pero siente que si dobla, tendr un accidente, lo que la a stia mucho. El tratamiento de eleccin para la patologia descrita es: stabilizadoreS, del nimo (_' ntidepresivos IRSS e) Benzodia;(pinas ... d) A ipsi tlcos atipicos e) ntiP. cticos ti picos

r\J

Cul de los sigui~ntes signos y sintomas se presenta con mayor frecuencia en un paciente con eloma? ..... Esplenomegal~a, Dolores seos ) Disnea ../ d) Epistax~ e) Sindr me febril 3 mejor conducta apte ll sospecha~llnir:a de E.POC es~ ciar broncodilatadors, asociados a corticoides lnhalatorios . b Iniciar terapia bro..ndilatadora con salbutaml e ipratropio licitar radiogrfa de tr~x e iniciar tratamiento segn hallazgos olicitar esp,irmetrla dicar 'genoterapia domiciliaria, asociando broncodilatadores inhalatorio_s

65) Pa i.'nte con embarazo de trmino en trabajo de parto. T~cto vaginal: cuello borrado 100%, dila R,n de 5 cm y membranas i~Jtegras. Test de tolerancia a las contracciones (-). ~e liza rotura artificial de membranas que da salida a liquido con sangre, seguida de bradicardia "f 1severa. El diagnstico ms probable es: placenta previa. b) desprendimiento de plac~nta normoinserta. . ) tura ut~rina. otura de vasa previa; ) aumatism6 del canal del parto

r'
/,

!a?Qu frmacos son ms eficaces para el tratamiento de los sfntomas en la uropata obstructiva.

Alfabloqueantes Betabloqueantes ) Anticolinrgicos d) Antiahdrognicos e) oGfticos ,

~ ~ ~

~
~

6 luha mujer es diagnosticada de un screening realizado en la primera consulta al stetra. Se encuentra asintomtica. Cul de las siguientes alternativas supone un ME~9,_F!_r:Lesgo. ra el feto, al momento del parto?: .--- ) Ausencia de infecciones oportunistas / ~ b~ecuento de linfocitos helpe~ CD4{+) metyor a 1.000 . ~J_;arga viral indetectable d Tratamiento antirretroviral ,}Cesrea

Vl~-te~

~ ~ ~
~
~

. 68J .Cul de los siguientes es un sntoma frecuente en el delirium tremens?


~f Agitacin psicomotora

b) J)elifie-de-ftfiftac} AaeiA-attditiva

d) . lisia e) 'tstona aguda

~
~
~ ~

9) Un flombre de 40 aos presenta rigidc!Z matinal de una hora y artralgias d.e..ar:tl.c.ula.ciones metacarpofalngicas, muecas y tobillos, desde hace 6 semanas. Al examen ffsico se aprecia aumento de volumen y signos inflamatorios en las articulaciones descritas. El diagnstico ms probable es: Artritis reu atoide ' b} Artritis cristales e} Artro d} LuR eritematoso e) A 1s viral

,ar

~
~

~ ~
~
~

1hiperparatiroidismo primario se presenta clsicamente como: Osteoporosis acelerada, asociada a hipercalcemia e hipercalciuria ) eons\w.acin crnica, asociada a htpercaicemia e hiperfosfemia e) Ast i y debilidad muscular, asQ.Ciado..a.bipo.calcemia e hipeffesfemiad) C tracturas musculares e hiperreflexia, asociado a hipercalcemia e hipocalciuria ~ e) lpercalcemia, hipocalciuria e hipofosfemia

~ ~
~
,..

2) El inicio puberat,er. el varn est marcado por:

f!i'
fiA
~

a) el-estirn pUb~fl b) la pubarqui~/ . -~}el au~~nte"de volumen testicular. d) la.aR (n develloaxitar. e) io.de..ver:-ujer primigesta, cursando embarazo de 41+6 semanas, acude a control obsttrico, sin dinmica erina. El tacto vaginal demuestra un cuello cerrado, duro, posterior, sin borramiento. La conducta s adecuada es: a) Realizar cesrea b) Observar evolucin espontnea e) Realizar amniocentesis d) Realizar test de tolerancia a las contracciones -~)1nducir el parto con misoprostol

~
~

(:"

~ ~
~
~

~---

j e r consulta por disUria y leucorrea amarillenta. A la especuloscoph se aprecia pared nal indemne, con cuello uterino muy Inflamado y con abundante secrecin purulenta. El i gnstico ms probable es: Vaginosis bacteriana e ndidiasis . ricomonjasis onorr

lmc1ar cort1co1des y ant1b1t1cos e 10terrump1r el embarazo al completar la madurez pulmonar Realizar amniocentesis . . A~terrumpir el embarazo por una vla expedita bajo cobertura antibitica endovenosa d) Admini~trar aptibiticos intraamniticos e} Solicitar tE7~t ie tolerancia a las contracciones

-~~,.PC~r de cuello uterino --- ---. _ ) ~a. cond~c~ ms ad~:.u~da.an.te- el-dia~nstico..cl[nk!p_~na ~-rloamnionitiJes:

76) U pciente de 45 aos refiere que despierta con cierto grado de disfona, la que luego mejora con el t p~cu..So del dfa. Adems refiere regurgitacin cida de alimentos, especialmente despus-de e er. En algunas ocasiones presenta un ardor retroesternal que asciende hasta el cuello. El examen r;;ro no muestra alteraciones. La conducta ms adecuada es: 1 ~atpolicitar endoscopa digestiva alta . l_niciar omeprazol e) Solicitar pH:-metria esofgica d) Solicita_ymanometrta esofgica . e} Errad'ar Helicobacter pylori

/A

77. especto a los nios con epilepsia es verdader~que: abitualmente se manifiesta como crisis parciales recuentemente pr~_stan antecedentes familiares v . e} Suelen acompa~r5e de retraso mental x d) Generalmente presentan espasticidad de las extremidades e) La resona;JP~magntica nuclear suele aportar mucha informacin para el manejo de estos pacientes
o(

78) esp~o al manejo de la hepatitis A, Cul de las sigulen'tes alternativas es FA~SA? a} e 6 solicitarse tiempo de protrombina para determinar la graVl!dad de la hepatitis ~ diagnstico es clnico, ya que no se cuenta con exmenes diagnsticos especificas -< Debe administrarse inmunoglobulina no especifica a los contactos cercanos )(" d} La vacunacin s~censeja en mayores de un ao, que no han sufrido la enfermedad e) El compromi~--de conciencia determina un peor pronstico .._... .
_,./

79) Un r~i'n nacido prematuro, de 3 dfas de evolucin presenta Irritabilidad, vmjtos, distensin. abd inal y hematoguezia en una oportunidad. Al examen ffs;Jco se observ~ irritable, en buenas co Jciones con el abdomel) djstandido, blando. Se solicita r.:.diografia de abdomen simple que muestra neumatosis intestinal, sin aire en la cavidad peritoneal. La conducta ms adecuada es: ) Realizar colonoscopia b) Solicitar TAC de abdomen y pelvis e iniciar antibiticos endovenosos e) Hidfstar par yia oral iniciar antibltisos orales, st1spende1 la lac:tancla !tlateJJaa y alimeRtar eo~la ji}'dministrar hidratacin por via endov.enosa, instalar sonda nasogstrica, suspender la alimentacin oral e iniciar antibiticos de amplio espectro . e} Resolver quirrgicamente a la brevedad 80) Un paciente de 45 aos refiere que muchas veces presenta deposiciones con algunas estrfas de sangre, sin otros sintomas. Cul de los siguientes diagnsticos.es ms probable? a) Fisura anal . bsceso perlanal ) emorra . emorroides externos e}_~olposis intestinal

r
,.~

~ ~
1

81) A ~escolar de 3 aos presenta fiebre de cinco das de evolucin, asociado a_.Qdinofagia in.t.4IDSL~

e ~ e

de miento. Al e~amen destaca tempercatura axilar de 39C, faringe intensamente con estiva con pi ,.<fas blanquecinas en las amigdalas, adenopal[as_granEies y sens s cadenas ugulares, , ema palpebral leve y hepatomegalia mmsible a 3 cm bajo el reborde costal. Cul es el diagnstico ms probable? , ~) Leucemia linfoblstica aguda _f \ ):r)"Mononucleosis infecciosa ;o e) tJ~aRgif:la d) ~atingoamigdalitis estrpfoc6clca ~-e) Faringoamigdalitis por adenovirus
82) Una paciente de 20 aos presenta malestar general, ca[dl-de cabello y resiones.faclales_ eritematosas. Al examen ffsico se aprecin erit~~~.P-m.YIP..Ly: en el dorso de la nariz y algunas

~ ~ ~ ~ ~ ~
~
-

//lceras en el paladar duro y en las mejJllil$. El diagnstico ms probable es: y 0 Dermatomiositis } Vasculitis de vaso pequ~o ) Lupus )'tnfermedad de Behcet e) clerodermia

Virus ) Staphilococcus aureus meticilino resistente e) Enterococcus faecalis d) Bacteroides fragilis ~los idium difficile _

~1 agente etiolgico ms frecuente de la diarrea asociada al uso de antibiticos es:


.

("" . _

~
~

a
a

84) e

un examen de rutina no se palpan, en primera instancia, los testiculos de un lactante. Sin rgo al examinar bien se constata que estn a la salida del conducto inguinal y que es fcil

'!'

cuada es: Observar evoluin ) Solicitar eco~;frafia inguinoescrotal ) Solicitar ylriograma d) Resolyfr quirrgicamente durante el segundo ao de vida e} Res9lve~ quirrgicamente de inmediato

e
~

\, : ~
~

/!!!A

~ ~
~
~

85) n~..-h-aujer de 33 ao;;.--sintomtica, ac.ude a su control ginecolgico anual, palpndose lun tumor a xi 1derecho. El rest del examen ffsico es normal. Se realiza una ecografa transvagina que el]li.estra un tuino/r.:vric~ ~-!~~-~h~,..Q~:~, cm..i_gufstico, de paredes lisas. La conducta ms adecuada es: , - '/.:.! / ,a) Observar evottJcin lb) Solicitar niV}!(es seriados de Ca-125 \_ - e} Realizar. ropsia porpuncin d} Realiz . - umorectomra e} Reali 'ar-enexe'Ctomtallerectm'
86) a forma de cuantificar el impacto de un programa destinado a la remocin de un riesgo en la
J.

t!'
~
~

p . tacin, es evaluando el riesgo: relativo en los expuestos . 11) atribuible en los no expuestos \..../e) de muerte d} relativo en los no expuestos ~buible porcentual poblacional

f!"
~

~ ~
(l!fi',
\.

'a paciente de 24 aos, consulta por dolor abdominal. Al E1xamen se constata paciente subfebril, olor a la Pfllpacin hipogstrica, palpaciR aRexial d9FeCI:l"4f movilizacin cervical. No hay s de irritacin peritoneal. El diagnstico ms probable es: 1A~ dometriosis yer~roceso inflamatorio plvico e) Apendicitis aguda . d) Absceso tuboovrico roto e) Embarazo tubario 88) Un paciente de 65 aos, fumador, hipertenso y diabtico, en tratamiento farmacolgico que cumple irregularmente, acude a un chequeo mdico, en el que se detecta la presencia de un soplo carotldeo derecho. No ha presentado slntomas neurolglcos y el resto del examen fsico no aporta mayor informacin. Se solicita un ecodoppler carotdeo que demuestra una estenosis carotidea ~~cha de 50% y estenosis carotfdea izquierda de 35%. La conducta ms adecuada es: ~J Derivar para endarterectomfa c;rotfdea . b) Solicitar una angiografa carotfdea y decidir manejo segn hallazgos .e} Solicitar ~asonancia magntica nuclear de cerebro y decidir mane,jo segn hallazgos d} Iniciar .anticoagulacin con cumarnicos y controlar sus factores ele riesgo cardiovascular . ~ejprar el manejo de sus factores de riesgo cardiovascular, agregando aspirina ("'~// Yla causa ms frecuente de elevacin persistente de las transaminasas en Chile es: Alcohol Hepatitis 8 .r- ..- Hepatitis C .--.--.sJ.}--steatosis heptica./ /e) Autoinmune /

portador de una trombofilia congnita, presenta un tromboembolismo pulmonar, por 90) Un.pa lo qu s decide anticoagular con heparina y luego con cumarnicos. Qu examen es el ms adp6 do para ajustar la dosis de cumarnicos que debe ocuear? J)' empo de-sangrfa \.._ / / Tiempo de tromboplastinlrj)Braal acbvado . ~ ~)"'Tiempo de protrombina . . d) Tiempo de trombina e} Fibrinogenemia

~e

\\

0- .

"
~
~
~
Ensayo 1 sexto ao PRIMERA PARTE

.\

l
1

d"!-:
f{

- (~

tt

~
~ ~

t'
~ ~

Importante: Esta prueba ser corregida por elleGtor automtico. por lo que debe preocuparse por contestar adecuadamente la hoja de respuesta. Adems. aquel que sea sororendido copiando reprobar de inmediato. 1),/6mbre de~s, consulta por~a, adinamia y nuseg de 4 meses de evolucin, que han ido aumento. Se controlan exmenes de sangre, entre los que destacan creatinina: 1,1 mg/dl, Cl: 96 Eq/1, NaU..130 mEq/1, ~Eq/1. El diagnstico de sospecha es: a) Hiperaldosteronismo primario x b) Diabetes inspida e) Insuficiencia renal inicial ~~in .ome de Cushing 1 ficiencia suprarrenal crnica ~/ 2 n promedio, Cunto mide un nio de(12 mese$ de edad?
~~

'!'
~

.?J

(:;
~ ~
......

,.

.
.

) 50 cm ~.C.~~ cm

., .::;,
L

(;
~

~~~cm
3)

e) 90 cm

Qu vacuna(s) recibe un nio en segundo bsico? .DPTypolio

~
~

a Trivirica

t) ~CG y trivirica
_.dDT e) Triviri~ y polio
/
./

z.

'~' "

"'
~
~
~

e
fA
~ ~

4) UI1P.aciente de 3'5. . ~!}ps presenta aumen~Q de yolumen_Y. dolor..d.~ l:.r~C?.~UJ.~I;~.!!I.!. y al examen fsi . 'Se constata eritema, aumento del c~alor local y derrame articular. Se realiza artrocentesis que da sa da a un lquido articular turbio, de filancla disminuida, con 90.000 clulas por mm3, 92% de )fmorfonucleares~ En la tincin de c:fraiT1 se visualizan coc~as...Gram..nega.fulilS: intracelulares. La t'nducta ms adecuada es: t:l o......n\...:u.. .o J.Y.lniciar ceftaxona endovenosa y realizar drenaje quirrgico ~? ~dministrar cefazolina intraarticular e) Iniciar vancomicina endovenosa y realizar drenaje quirrgico _..,.clflnici~r azitromicina oral y drenar el derrame, mediante artrocentesls repetidas e) lni~i cloxacilina endovenosa y realizar drenaje quirrgico

~~

f!'
~

~ ~
~ ~.

5),. n p~ciente de 78 aos, con antecedente de ,Ef_QC, en control con tu".tmcodilatadores y corlj~gjsles i alados, con hbito tabquico suspendido hace 10 aos, inicia cuadro de disnea mayor a la habitual, socJda a un aumento en la expectoracin, la que se vuelve mucopurulenta. Al examen ffsico se 0 co~.!:itata paciente disneico, orientado, con F.<2_:_~00x', PA: 130no mmHg, ~x', con tiraje, t : 38,2C, S~t02: 8~%. Se observan signos de hiperiinsflaCn pUfrOilTVse auscultan sibilancias y estertores ,d~so .(.Cul ~e las siguientes medid~s est MENO~ indicad?, par~ .el manejo de este paciente?. ~~d 1n1strar ox1geno en alta concentracin, por mascanlla de rec1rculac1on . 6) ministrar corticoides sistmicos./ e ..ealizar nt3bulizacin con salbutamol .,...)_Administrar ceftriaxona endovenosa >' 5SoliciJa'r gases arteriales 6) Respecto al divert(culo de Zenker es verdadero que: a). Rara vez produce sntomas . o) Se produce por un mecanismo de traccin ,< .. ...-gEl tratamiento suele ser la reseccin quirrgica d) Se produce en la unin gastroesofgica e) Se diagnostica mediante endoscopia digestiva alta

""

~ ~
~
~
(lA

,
7) Una mujer de 26 aos, primlge_s.ta, cursando embarazo de ~~flse.mana.s presenta P.~..r.dida..d.eJ(quide claro por genitales, con olt a cloro. En el examen obsttrico -~-se detec.tn..alteraG.~s. El test de cristalizacin muestra formacin de "hojas de helecho" y el peil biof[slco re~ulta 10~1_p. La ~onducta ms adecuada es: --........-............... ........ .

""
~
~

,.,
~
~

*
.

ab))ln

.'

rtoco

Ad corticoides y antibiticos e inducir el parto en 48 horas -'J ...~~lnlstrar cortlcoides y antibitcos, manteniendo una conducta expectante. El tratamiento farmacolgico de elecci.n para el trastorno de pnico es: a) Benzodiazepinas " --~Antldepresivos inhibidores de la recaptura de serotonina e) Antidepresivos tricfclicos d) Anticonvulslvantes e) Antipsictlcos atiplcos
9)

~nistrar

/ ~

'1,~

1 fO
\ .

~
~

~
~
~

n paciente de 30 aos afirma que el rJ.iablQJ.Q quiere matar:.y que posee sus familiare~.cino.s_ ra que lo ataquen. En ocasiones ha htentado meterse;n5.J cuerpo y habla, amenazndolo e nst.Jitndolo. Sus familiares refieren que es muy aptico y tranquilo, pero que d~sde hace algunos f!)ses est con esas ideas, las que cada vez son ms intensas. El diagnstico ms probable es: Al} Psicosis reactiva V b) Trastorno delirante e} Q.elirium

""

~
~

~-squizofrenia

e) Demencia 1,j.Una paciente por un cuadro que inici con tos y ~nofagia, las que stuvieron presentes po.r 1 das, cediendo luego. Sin embargo evoluciona con ensacin de descarga posterior, hi osmia obstruccin nas21l, mayo_r._.l~Q\.Jl.~rda,.~efa.l~a, se.nsaci9n de presin facial y r norrea purulenta. La conducta ms acfec'iiaaaes: .. . . a) Iniciar antihistamlnicos orales y corticoides tpicos s . . _\- -~ b) Solicitar TAC de cavidades paranasales y decidir segn hallazg1os ..s~ .~ /'e) Iniciar amoxicilina ms cido clavulnico, asociada a Al NEs // d) Iniciar cefalroxilo, asociada a Al NEs e) Iniciar amoxicilina oral, asociada a Al NEs 11) Un paciente sufre un paro cardiores iratorio en la sala de urgencias, donde usted trabaja. Se in cian ventilaciones y masa e cardaaco y se instala monitor electrocardlogrfico. Luego de 5 ciclos e ntina sin pulso_y elrrumltor.demuestra a~~~!dad elctrica. con complejos QJ!~-~~~~~~encia de O por minuto. La conducta ms adecuada es: ) Reili?ar desfibrilacin elctrica y continuar el masaje cardiaco ~ } R~lzar intubacin orotraqueal, iniciar ventilacin a presin poBitiva. con oxigeno al100% y continuar m sje cardiaco Administrar adrenalina 1mg por vla endovenosa y continuar con el masaje cardiaco",....... d) Instalar marcapasos externo y suspender el fllasaje cardiaco e) Rea_lizar 5 ciclos ms de masaje cardiaco y. reevaluar 12)'Paclente de 14 aos, consulta por palpitaciones inten~de 30 minutos de duracin. Habla ~fesentado 2 episodios previos, de simflares caractersticas, pero que cedfan espontneam_!.Ote-~ pocos minutos. Al examen se ve paciente en buenas condiclcmes, ansioso, pulso regular. 1851P_~ r in arterial: 122/74 mmHg, eupn.eico. La conducta inicial m~s adecuada es: ----~---ealtzar maniobras vagales . dministrar adenosina 6 mg endovenoso e Realizar cardioversin elctrica .-r (', ' ~ olicitar electrocardiograma"> dministrar amiodarona endovenosa \ ~\1
/

,-,
~

de,.-24-ao~lonsulta

de_~_g.{!td.~-~yoJucln,

~ ~
~
~
~

~
~
~

~
~
~

~
~

~
~ ~
~

~
~

~
~ ~

~ ~
13) Un paciente presenta fasiculaciones en varios grupos musculares, asociados a hiperreflexia. La cmracin hidroelectrolitica ms probabBe es: a) ipocalcemia Hipokalemia c}Hipomagnesemia ~:jd> Hipercalcemia Hiperkalemia ,,. - .. ,_ 14) U!J paciente de ~Qs.os, cursa con diarrea aguda, Se solicitan y~r~litos plasmticos que demuestrar1 N_a~27, ~:3,9, Cl: 88. La conducta inicial.ms adecuada es tmmstrar: Solucin de NaCI al 0,9% Solucin de NaCI al 3% e) Solucin de NaCI al 3% con 2 gramos de KCI d) SoiUGi~lcesada al a%,oon.2 Q@_mos de NaGI y 1 gramO-e-KCI e) oluc;ih de NaCI al 1,5%
.... ,..

~
.

~ ~ ~

_::ya>

~ ~ ~ ~ ~
~

Jf:

e
~

~
~
~
~

)'l. Qu examen debe solicitarse en primer lugar a un hombre de 58 aos que consulta por dolor etroesternal de 45 minutos de duracin'? a) Radiografia de trax b) Gases en sangre arterial e) Tomografia axial computada de trax ~v.eles de troponina . ctrpcardiograma

(:a~miento de eleccin de la depn!sln severa, duran~~-mb~ es:


pramma nlafaxfna

Amjtriptilint

) Liti
1 oxetina

~
~
~

1 Cl de las siguientes asociaciones entre caractersticas clfnlcas_y patologa es INCORRECTA? ).-esinhibicin y conductas desadaptativa~:;- Demencia frintotemporal V"'o) Caldas frecuentes e incontinencia urinaria- Hidrocefalia normotensiva,_....... e) Empeoramiento de los sfntomas con antipsicticos- Demencia por cuerpos de Lewy ._,/. d) Asociacin a hemipaesia - Demencia vaHcular ~Alucinaciones visuales- Demencia senil 18) Un ensayo clnico controlado y aleatc,rizado, compara dos tratamientos antibiticos A y 8, para el tratamiento de las neumonras aspirativas. Para esto aplica cada tratamiento a 3.000 pacientes con este diagnstico y se comparan las tasas de mortalidad y de efectos adversos. Se obtienen los siguientes resultados p Frmaco A Frmaco B Riesgo relativo T~a de mortalidad 0,67% 0,56% 1,2 097 6,67%() . Ta$ de efectos adversos 1,33% 5.0 0.01 ~u frmaco recomendarfa? ' ... ~ __..... ;. El ant!b!t!co A-Porque-tiooe-meAes--efeet<;)-adver-ses b) El antibitiee-A-porque redu(.;e mas la moffiilcld _fEI antil:iitico 8 porque tiene mnos ~fecto adversos 4 "Ci) El antibitico 8 porque reduce ms la mo11alidad e) C~alquiera de Jos antibiticos A o 8, ya que presentan resultados similares .. 19) En la hepatitis aguda, es determinante.! de mayor gravedad:

: \Ir
- { ' \
J'!i\

~~a presencia de encefalopatia heptica

~
.ti\
~

~El acortamiento del tiempo de P.rotrombina AEI alargamiento del tiempo de protrombina d) La elevacin de ras transaminasas por sobre las 1.000 Utn e) La presencia de esplenomegalia

t'f\
~ ~ ~

~
~
20) Cul de los siguientes cuadros cHnlcos cursa con alteracin de conciencia? a) Esquizofrenia hebefrnica ---/'b) Mania e) Esquizofrenia paranoide :eiJ Es uizofrenia simple elirium v

~
~
~

21) Una paciente, cursando embarazo de 30 semanas, iniciar contracciones uterinas persistentes. Al ex . en fsico se palpa f!to en ceflica, se aprecian con~cciones uterinas Intensas, de 3 en 1O retos y se palpa cuello uterino con 50% de borramiento y 1 cm de dilatacin, cqn membranas tegras. El monitoreo fetal es tranquilizador. La conducta ms adecuada es: .~) Dejar evolucin espontnea b) Administrar corticoides y antibiticos ...~dministrar corticoides y tocoliticos d) Administrar corticoides, antibiticos y tocollti.cos e interrumpir el embarazo en 48 horas e) Realizar cesrea de urgencia
22) Para comparar la mortalidad entre dos pafses, es preferible realizar: a) Un estudio randomizado b) Un anlisis de sensibilidad :,e}Un estudio ecolgico d) Un ajuste de tasas

~
~

,.,.
~
~

~
~

~o

23} Un pciente de 24 aos, con enfermedad celiaca sin..un.ad.m~uado..cw:nplimient.o-de--la -dieta-y:;-~v-/ d ~n, en tratamiento al)tidepresivo, que usa de forma irregular, consulta por mareada asteni-.Y-1s11ea de minimos esfuerzos. Al examen fsico se aprecia mu)r plido, taquicrdico, con un pulso i'prio. Se ausculta un soplo sistlico eyectivo, de caracterfsticas funcionales. Se solicitan exayene. . que son compatibles con una anemia por dficit de vitamina Bj2, con una hemoglobina actual ~ 5,8 ,/ g/di. La conducta ms adeCU-.cD..Jla..entr.e.-las.:sigulentes, es: ~- ___ ..... - .. .. -/ a) Enviar a--domicUiOCo-Vfina 812 vla oral en dosis altas ~- b) Enviar a domicilio;~insistiende en..el..mHnplimlento de la dieta . e) !:J.ospitalizar y administrar vitamina 812 por vla parenteral .~;tr.Hospitalizar y transfundir glbulos rojos _ . . e) Hospitalizar e indicar dieta alta en folatos, vitamina 812 y hierro, asegurndose de que no ingiera ms ~ gluten

---~
At
~

24) La cintica de fierro que ms orienta a anemia de enfermedades crnicas. es: .a) Ferremia alta, transferrina baj y ferritina baja b) Ferremia alta. transferrina alta y ferritina alta \....- ~ e) Ferremia baja. transferrlna alta y_ ferritina baja d).Ferremia ba a a fe a-alter i~E;. er mia baja, transferrina alta y ferritina alta

- 1 ...:~

,._,..._.

.l.

u!'?--

\J./

Fu--~ (2;~--

25 . -~=-;~ciente ctabti~~-t~~-2,-;;m en trbtmiento con insulhia NPH matinal y nocturna, ms r, erzos con insulina cristalina en las 3 comid~s principales: d~ almue~J:Jma. Est sintomtico, sin embargo presenta hem~_g!!:!~~~es~-~a~ uno, cer os a 180 m 1 'La hemoglobina , glicosilada es qe S%; E:l examen f(sico no _aporta mayor inforrnaci6.l:ieonducta ms adecuada es: a) lniciar,.metformiAa- . b) lni~cf glibenclamida . i' e) f\u entar la insulina NPH matinal /d) A entar la insulina NPH nocturna e) .mentar la insulina cristalina del desayuno

J..~) En un anciano con meningitis,. la presencia de bacilos Gram positivos en ellfquido cefalorraquideo ' s sugerente de: a) infeccin por Streptococcus pneumoniae . .~}infeccin por Usteria monocytogenes. e) infeccin por Haemophilus influenzae. d) tuberculosis menlngea. e) contaminacin de la muestra.

(!'

~
2 ) Las siguientes patologias se present1n frecuentemente como nipermenorrea, EXCEPTO: ) Mioma submucoso/ V\. .:.,p ..-.b) Plipo endometrial r ~~ 0 "-"-. e) ~nomiosis n..' jYf:'iperp1!sia endometrial e) Utero septado
28) 1mejor examen para la pesquisa pre1coz de la displasia de cadera en un nio de un mes de edad es a .Radiografia ) Ecografia e) Tomografia axial computada d) Resonancia magntica nuclear ~ El examen fislco

~ ~

~
~ ~

t"-

~ ~
~ ("

e
~

29) Hombre de 52 a.os, obeso, con pirosis y djsfona intermitente gn el ltimo ao, presenta exacerbacin de sus sntomas en el ltimo mes. Actualmente con disfon_.m_gerrnanente y pirosis nocturna que lo despierta. Cul es la conducta ms adecuada?_._____ -----~ - --"- ...... ~) Indicar omeprazol por dos semanas y evEtluar respuesta. b) Solicitar pHmetria de 24 hrs. r~~~~'-.- ~olicitar endoscopia digestiva alta~ d} S ncnar rad1ograha de esofago, estmago y duodeno. e) 1 dicar medidas generales y famotidina.

(!!'

e
~
~

'!'
~

9> Paciente de.]!Mos, consulta por cefalea, tinitus y mareos. Al examen: P~1iQ)FC:99x'. RR2T, /aumento de la intensidad de R2, auscultacin pulmonar normal, sin signos focales en et examen ./ neurolgico. El frmaco de eleccin, para el manejo de esta situacin, es: a).)Jifedipino sublingual ):1 ~apto ril oral . e) Nitro licerina sublingual d) Nit prusiato endovenoso e) E alapril oral
Un paciente ingresa por un cuadro de ealpi~ciones de inicio brusco, de ~ ~or;;~ee e~oluc.io . _. regulr en 2 tonos en) auscultacin cardaca. Se solicita un ECG que demuestra una taguicardia a"'tiRS angoftJi.> ~r, con ondas de serrucho en las derivaciones inferiores. El diagnstico ms probable es: /:-a} Fibrilacin auricular / b) Taquicardia sinusal e} Taquicardia ventricular .9f!:1uHer (aleteo} auricular e} ifJquicardia paroxfstica supraventricutar 3
res~nta F_C: 150 lpm, PA: 110/80 mmHg, extrem1dades bien P-Wfyn(.llq~_yo ntm

~
~

~
~

. .

~!
~

~ ~ ~
~

,2) Los pacientes con trastorno obsesivo compulsivo, que no es adecuadamente tratado, suelen complicarse con: a} .Agorafobia ' .M Depresin /e) Esguizofrenia d) T ~torno bipolar e) ~;:~storno de angustia
...
~

~
~ ~
~ ~

,3) La compresin de la cauda equina se manifiesta clinicamente como: a) Pl!diele-tlel conlFel de esfh:ltEwasrimpgtan,;ia-seMI:mt-y-aAestesia ensiH8-'de-rnontar b) Afectacin motora asimtrica de las extremidades inferiores, sin compromiso sensitivo ~ \_, t!\. ) etaeifi.Sensitfva-Efe-las coatto extremtlaG8S 1 son compromiso motor mpromiso asimtrico de las rafees nerv:iosas, que inervanlas extremidades inferiores, con afectacin tiva y motora e} Cpmpromiso si111trieo Ele extFemidatiesirrfer-fet:es, con afectacin motora y de la sensibilidad termalgsica, respetando la sensibilidad tctil, vibratoria y propioceptiva

~
~

.!!:___

34) El hallazgo de un~ pansistlico 3/6 en el5to espacio intercostal izquierdo en el borde esternal en un lactante de un mes, sugiere la presencia de: -: ) omunicacin interauncular de peqUeno shUnt. comunicacin interventricular. _:'-c)-eeAEiueto artertoso permeabie. / d) coartacin de la aorta . ..-~)soplo in9cente n_, .p~ciente presenta disnea de esfuerzos orto fi 1cp 'se observa des lazamiento del e o ue de la u acio intercostal, en la l) axilar anterior, se auscu u astlico IIINI n todo el precordio y sepa pa e ..arterial perifrico muy amplio ( lso Celler) El diagnstico ms probable es: _../ a) Estenosis artica _./~) stenosis mitral 1 Insuficiencia artica ns d) 1 ficiencia mitral . . ~ t.::J U'~) ! h . e) ocardiopatia hipertrfica obstructiva 0 ~4.9,._ ratamiento farmacolgico del persistente le;El es: . eta agonistas de accin corta inhalados, previos a la actividad 11sica o en caso de sintomas, sin necesidad te vsar corticoides ~Corticoides inhalados segn horario y beta agonistas de accin corta segn sintomas ~~ ~orticoides inhafados segn horario y befa agonistas de accin lama segn stntomas . . d) Cortic(3ides inhalados y beta agonistas de accin lar:ga segn horario, ms beta agonistas de accin corta . segn sintomas e) Corticoides.orales en dosis bajas y aeta aonistas Ele acciR coFte-seg;n-sJntomas
' .1

/~ ~-

~a

J r

. A
~
/

37) Mujer de 40 aos, casada, 5 hijos, relata angusti~da que hace una semana vi al diablo y teme ser agredida por l. Horas antes del episodio, se enter de que su marido tenfa una amante. Refiere un episodio similar hace dos aos, al enterarse de la muerte de su madre. Su afecto es modulado, con un relato fluido y sin alteracin del curso formal del pensamiento. Cul es el diagnstico ms probable? a) Esquizofrenia paranoldea. . ~ b).Trastorno depresivo mayor recurrente con. sintomas psicticos. -:e) T-.fastamo..de pniCQ. . . ~r-astor.no bipolar, episodio m~nl_aco. . ...rr~rastorno ~isciativo~ .

es ms til para prevenir la recurrencia de episodios depresivos o manracos en n paciente diagnosticado de trastorno bipolar? , . Fluoxetina . ~cido valproico . e) Benzodiacepinas . d) lnhibidores de la monoamino-oxidasa c(ozapina

if'i~u me~-;~~~~-:

"

'

J!

39) Un paciente de 72 aos, f.wn.ac:klr, presenta tos con expec~n m1:1eesa. desde hace 4 meses, asociado a disnea de esfuerzos. En su examen fsico destaca.n) nas en vidrio de reOl. El examen ' . cardiopulmonar es normal. El diagnstico ms probable es: ,Jle.o:ame pleural ) Cn9er bronquial ) :pnquie.~tas.tas d Jberculosis pulmonar . "Enfermedad pulmonar obstructiv~ crnica

'

40 1tratamiento farmacolgico de ) Antipsicticos tipicos b) Antipsicticos atipicos e) Anticonvulsivantes . d) lnhibidores de la acetilcolinesterasa e) Antidepresivos

ele,~cin, para el,~} s actual~ente:


.,

~
~

-~

e
~

~
~

~
~ ~

41) Un nio de 8 aos presenta ar.tralgias y artritis de manos y pies, asociad~_a ompmmLso delg,stadg general. E.l examen fisico no aRorta mayo~~nnacin. En sus exmenes de laboratorio destaca elevacin .imQ.Qrfante-de-la.~plasm~ y elevacin moderada de la PCR y VHS. El diagnstico .- ms probable es: a) Artritis por cristales b) Hemocromatosis e) Artritis viral ~rtritis reactiva ~) Artritis juvenil idloptica

e
~

e
~
~

42) Paciente de 66 aos, di tico e hi er1enso bien controlado. Refiere visin de cuerpos volantes (miodesopsias), fotopslas Y. lismrnuc n ae la visin perifrica en una zona del ~~po Vl!_Ual. Cul de los siguientes es el diagnstico ms probable? a) Accidente vascular enceflico estabilizado. \" ~egeneracin macular senil. ~ Desprendimiento de ret~. tl) Retinopata hipertensrva. e) Retinopata diabtica. _,., 43) Un nio de 1O aos, diabtico tipo 1, Em tratamiento con in$..Ul.ina..en_e.squema intensivo, presenta c!ro de sudoracin, temblor y actitud t:1gresiva, seguida luego de lnconciencia. El diagnstico ms bable es: Accidente vascular enceflico Cetoacidosis diabtica Coma hiperglicmico hiperosmolar
~ipoglicemia

~
~

e) Encefalitis herptica

~ ~ ~
~

_..rs~nta glicemias cercanas a;jO mJ~ tanto en ayunas, como postprandiales. El examen ffsico es
hormal y su hemoglobina glicos1lada es de 8,5%. La conducta ms adecuada es: a) f\1:tnlener dieta, indicar ejercicios y controllar en 3 meses t>Yfniciar metforrnina / ~e) Iniciar glibenclamida d) Iniciar insulina e) Iniciar una asociacin entre un hipoglicemiante oral e insulina

). Un paciente de 57 aos, o~ y:::f'ltlco tipo 2, en tratamiento con dieta desde hace 3 aos,

~ ~
~

~
~.

~
~ {$'

(_actante de 11 meses con historia de vmitos y diarrea d.{4 dfas de evolucin. Al examen: ~ado, extremTdadesJrfas, llene capilar de mas de 5 segundos y pulsos dbiles. El tratamiento inicial ns apropiado es la administracin de 20 mi/K de: /a) solucin glucosada en bolo. b) ringer lactato en bolo. J>Ysolucin fisiolgica en bolo. / d) plasma fresco. e) solucin glucosalina en bolo. 4

46~aciente de 30 aos, portador de lcera gstrica por uso de AINEs, consulta por exacerbacin de
sntomas dolorosos. Desde hace 3 horas el dolor se hace mucho ms intenso, irradindose a ' rso y se asocia a. hipotensin. Al examun fsico destaca abdomen en tabla. muy doloroso, con lumber (+)y abolicin de los ruidos inteBtinales. Qu examen debe solicitarse como primera proximacin diagnstica? . ) Ecografa abdominal b) TAC de abdomen y pelvis e) Endoscopfa digestiva alta de urgencia BfRadiografra de abdomen de pie /e) Colangioresonancia s

~ ~
~

e
~
-~

~
~ ~

47} Un hombre de 23 aos sufre .torsin de la rodilla derecha mientras jygaba ftbol, evoluciohando con olor que le limita la marcha. Al examen fisico se aprecia Importante dolor al valgo forzado y a la pal acin de la zona medial de la rodilla. No presenta otras alteraciones y las radiograflas no vi u~lizan lesiones seas. El diagnstico ms probable es: .Rotura del ligamento cruzado anterior 1' Lesin del tendn de la pata de ganso ~~$Juince d~l.l!.gamento colateral medial _ d) Lesin del menisco medial e) Fractura avulsiva de la rtula 48} n paciente de~s presenta temblor ceflico y de lauxtremldades superiores, especialmente al omar alg~na postura o al tomar un objeto. El resto del examen neurolgico es normal. El tr ~amiento ms adecuado es: . . Sertralina / ) Prolopa . ~ e) Diazepam _Jij"Propanolol e) Carbamacepina 49} Cul de los siguientes parmetros de a) Albmina srica b) Hematocrito J., "' e} Creatinina d) Velocidad de eritrosedimentacin e) Bfcarbonato --" 50)

laboratorio~~~~~urante un embarazo normal?


'-~__...

/.) .. _./

J j
"

6~-iciones frecuentes, abundantes, de color amarillento, brillantes, en ocasiones con gotas de sa. La conducta ms adecuada es: Solicitar colonoscopla . Sohcttar test de hemorragias ocUltas en 0ej50Siciones Solicitar coprocultivo y parasitolgico dir~cto d} Iniciar dieta sin gluten ~Solicitar test de Sudn de deposiciones .

ui paciente de 7 aos presenta detencin del crecimiento. La madre refiere que presenta

'Paciente de_-1? aos, se rea.liza perfillip[dico, como chequeo mdico, con el siguiente resultado: Lr~ LDL;.210,.Triglicridos:5BO. La conducta teraputica inicial debe ser: Dieta ms ejercicio y control en 3 meses niciar una estatina ..cfJniciar un fibrato / d) lndicC)lr una asociacin entre una estatina y un fibrato e) Iniciar cido nicotinico aladar blando, vula y faringe posterior. Cul es el diagnstico? -stomatitis herptica erpangina e) Fariflgitis viral d) M nucleosis infecciosa e) tl ndidiasis oral

tf~ctante de 1Q meses de edad, febril y cuyo examen farin!~eo muestra mltiples vesiculas y lceras

f'
~ ~
~
54)

~
~

P~ie~te multf ara de os en tra~jo ele parto, cuyo monitoreo muestra una FC fetal basa.l de tyh1, buena varia i ad y prescmcia d! aceleraciones. La ~otura artificial de membranas da salida a

" "'
~

iquido con meconio fluido, amarrlllo verdoO. Cul es la conducta obsttrica a seguir? nterrupcin fumediata del embarazo mediante cesrea -xeanimacin fetal in tero 6on reevaluacin en 20 minutos K e) J3.eanimacin fetal in tero seguida de interrupcin del embarazo~ dfMantener una conducta expectante y pem1itir que contine el trabajo de parto e) Interrupcin inmediata por la vla ms expetdita

~ ~

~ ~
~
~

J
,,.

Mujer de 28 aos ingresa con un cuadro de ctisis de pnico desde hace seis horas. En la historia estacan mareos y lianto fcil seguido de mioclonfas y contracturas musculares. Al examen:.afebril. esin arterial: 11 ONO mm Hg, frecuencia cardaca: 130 por mrnuto, regular; frecuencia respiratoria: ~Cules son los valores det gases en sangre arterial ms probables de encontrar en una uestra to:nada al ingreso? _ ~ a) R_l::l+,-17 ' p602 40. mmHg1 HC03 14,0 ml::/1 y<.r-~ " _ Q__A ~\.\ ~f' . ,<"b) pH 7,62 1 pC02 20 mmHg 1 HC03 20,0 miSil ~ Q( eJ.-~ c)-pH-'6,99 1pC02 26 mtnt=tg 1t=:IC03 6,0 mE1 (.) "t.,~ .. d) ~ f pC02 4o mmAg 1 HC03 24,0 mt:/1 e) pH 7,46 1 pC02 20 mmHg 1 HC03 14,0 ml~l
56) Una mujer de ~s, cursando un er~.l;>arazo de 7 semanas, consulta por m~trorragia y malestar hipogstrico. Al la especuloscopfa se observa salida de sangre por el orificio cervical externo. El tacto vaginal no demuestra cambios cervicales. La conducta ms adecuada es: a) Realizar ecogra ra o JCJtar mve es plasmticos de subunida beta de gonadotrofina corinica e) Realizar legrado uterino d) Administrar progestgenos orales ~nviar a domicilio con reposo e hidratacin . . ..

~~ ~~

" "
~

Cf'

fJ'
~

57) produce he. , / 6rriigias vftreas? ... a 1nguno . Disminucin de la agudeza visual e) Entopsias d) Escotomas e) Dolor.ocular
/

~.srOtomas

habitalmenb~ la retinopata diabtica proUferativa Ce~;.;ha presentado


- - - - - - - - - .- - - - -

El

~umentar el tamao de la.muestra e,s una estrategia utilizada para disminuir el error producido

fA
~
'

~
~
~

azar ariabtes de confusin e) Sesgos de medida d) Sesgos de memoria e) Tipo 2


59) ~chmte de..~ aos atendido en el seiNicio de urgencia por una fractura de pierna derecha. Se in J yeso bota larga y reposo con pierna en alto en su domicilio. El paciente vuelve a consultar a las horas por dolor intenso y progresivo de la pierna afectada. Cul es el diagnstico ms probable? a) Desplazaminto de la fractura ~ b) Tr.ombgsis-venesa-.pr.ofullda v ~~mpresin neurolgica ~ ~.Indrome compartimenta! -./ e) Necrosis muscular

~
~
~

fJ'
~ ~
~

60V Cul_de las siguientes alternativas e~; un factor de riesgo para el desarrollo de Uf! cncer

. /
1

iabetes mellitus

) Infeccin por virus papiloma

~
~
~

e) Uso de anticonceptivos orales d) Uso de dispositivo intrauterino e) Multiparidad

""

61) Un recin nacido de 43 semanas de edad gestacional con antecedente d.e sufrimiento fetal, durante el parto, presen . u1p JI o a los pocos minutos de nacer. Al examen fisico se ap cla dificultad respiratoria y se a cu an e 1 os s1 1a c1as bilaterales. El diagnstico ms p C?,.bble es: ) Taquipnea transitoria / b) Enfermedad de membrana hialina \ ,./' ..o}:Sndrome aspirativo meconial . /0) Neumona neonatal e) Hipe~~nsin pulmonar persistente
,.

62} na mujer consulta por dolor ocular derecho intenso asociado a ojo rojo profundo y midriasis fija. E ~iagnstico ms probable es: Cluster laucoma agudo e) Uveitis agudad) Parlisis del tercer par e) Neuritis ptica

63) Un paciente recibe J,ma pualada en el trax. Se encuentra en buenas condiciones, con signos vitales normales y sin sangrado activo. La herida e! ~e 2 cm de largo en relacin a la parrilla costal
izqui rda y .el examen cardiopulmonar es normal. La conducta ms adecuada es: a) . rdioce ' b) ~la'"flubo pleur~l ~uturar la herida Solicitr radiografia de trax e) ijeal(za explulaeiR-digltal bajo anestesia local . 64) na muj~r consulta por leucorr-ea de mal CllQr, que en la es;peculoscopfa se aprecia de color gr ceo,,.espumosa y cuyo pH es s.s. El agente etiolgico ms probable es: a Candida albicans ) l;s6herichia coli Gardnerella vaginalis ~) Trichomona vaginalis e) Clamydia trachomatis --.

6S) Mujer de 78 aos previamente sana, inicia hac~dos meseltun cuadro de anorexia, insomnio, ni bao . . b" dad se uea de olvidos frecuentes. Refiere baja de peso de 3 kilos. Mini mental 27/ O;- Examen ffsico normal. Hemoglobina 13 g/dl. ~l de los siguientes es el diagnstico ms probable? 'Hipotiria1smo . ;; } Demencia tipo Alzheimer t1Trastorno depresivo d)_IrasteFfle-de-aRsiedad..generalizado
e) Nee~lsia-oetdre-

_...-----.....

de 7 aos presenta del crecimiento) debrr!dad, astenia y cefalea. Al exainen rolgico destaca defecto del campo visual bitemporal, asfimtrico. El diagnstico ms probable es: Enfermedad de Cushing Enfermedad de Addison _.,e) Craneofaringioma ' d) Diabetes insipida e) Hi rprolactinemia 6 Una paciente de 46 aos consulta por oli amenorrea, se"uida de amenorrea de 4 meses de . volucin. U.sted sospecha que la paciente est clima erica. ,~Qu examen de sangre le permite '/determinar sta situacin de mejor manera? a) Estrgenos plasmticos b) Estrgenos urinarios e) Progesterona plasmtica d) J_H plasmtica /FSH plasmtica

J.~nio

~traso

,,

('

~
68) El motivo de consulta ms frecuenb:! de los pacientes con depresin, en la atencin primaria, es:
a) Ideacin suicida .

"
~
~

/6) Crisis de pnico e) Aumento de peso


pAnedoma

"
.

l3) 91ntomas por somatizacin


69) Una mujer de 56 aos, con antecedetntes de lcera pptica, consulta porque se siente triste y de.sesperada. S marido sufri un Infarto cerebral hace un ao y desde entonces ella debe cuidarlo. Dice que est cansada, sin nimo, se slt:mte triste y ya no disfruta de sus actividades. Asegura que su arido se enferm por culpa de ella, que sus malos deseos le causaron ese dao. Adems est muy esimista porque su lcera pptica la matar en cualquier momento y segn ella no existe tratamiento ue pueda salvarla. El diagnstico ms probable es:

~ ~

) Esquizofrenia ) Tr~stor bipolar, con episodio mixto ~presin psictica -j}"'TFastomo..esqui6oafedWQ ...../e) D~'Rf_esin atipi~a-

~
~ ~ ~ ~ ~

A or
'

la de ictericia, acompaada de coluria y acolia, en un lactante de 6 semanas, se debe s probablemente a:

pres~ncia

cteri_..ci~ fisiolgica

~fresia biliar primaria


e) Hepatitis viral

!ndrome de Gilbert

~ipotiroidismo congnito

~ ~

71) Varn de 55 aos, cuya radiogt=a.fia-de-t.l:ax, tomada por un chequeo n:tdico, muestra un ndulo ,Rulrponar liso, bien demarcado, en el s~Jmento lateral del lbulo inferior derecho. Es-nofmador, no tiep'e antecedentes mrbidos. El examen ffsico, el hemograma y el perfil bioqufmico son normales. G.tu!ll.es la conducta ms apropiada?

~
~
~

)!)

) .Programar de inmediato una broncoscopla. racticar una biopsia transtorcica por puncin. Revisar radiografias de trax previas, si las hay. d) Practicar una tomografia axial.computada del trax. e) Solicitar una planigrafia pulmonar.

~
~

72) Un recin nacido de 4 dfas de vida, di3 grupo sanguneo ~~cuya madre es A Rh o "y, presenta ictericia, con bilirrubinemia que al sptimo da alcanza los 1O mg/dl. El diagnstico m J)robable es:
.. ctericia fisiolgica . l:lem!isjs por incnmpatibmEiad de gmpe-dsico e) ~mlisis por incor:npatibilidaEI-Rh d) Ictericia por leche materna e) Atresia biliar primaria

~
~

~
~
~

~ ('A
~

~& .
{

73) Un paciente de 34-at\Qs presenta dolcr pf!_[Y.ical, luego de levantar muebles, que se irradia por la cara lateral del brazo y antebrazo, hasta Etl pulg~r. Al examen destaca disminucin del reflejo bicipital. La raz nerviosa comprometida es: --- ...-....------

d) es e} T1

~
~
~
~

~ ~

~ ~
74) Una paciente de_5.a~~~c!i~btica e hipertensa, con anteced,~nte de cardiopatra hipertensiva,y de fibrilacin auricular' paroxstica, en tratamiento con amiodarona y acenocumarol. Su hipertensin arterial y su diabetes mellitus estn bien controladas. Durante lo!; controles de los ltimos 6 meses se h atenido en ribn~ sinusal, asintomtica. La conducta ms adecuada es: uspender el acenocumarol y mantener la amiodarona uspender el acenocumarol y .la amiodarona . uspender el acenocumarol e iniciar aspirina d) ~ajar la dosis de acenocumarol, para lograr INR <;arcanos a 1,5 / ' ,efMantener la amiodarona y el acenocumarol ~ 75) El princ.ipal riesgo del recin nacido postrmino, es decir aquel que nace de las 42 semanas de ge taci.n; es: s frimiento fetal p(ma del canal del parto e ipoterrnia Poliglobulia e) Taquipnea transitoria 76) Un adolescente de 16 aos, consulta porque ha notado aparicin de una lesin poliposa de rpido crecimiento en el cuero cabelludo, que en ocasiones le duele al peinarse~ Al examen fsico se observa una tum~,racin similar a una coliflor, multilobulada, Irregular, ptetdiculada, de color amarillo, de 3,8 cm. El diagnstico ms probable es: 1noma maligno. vo sebceo de Jadassohn eratosis seborreica ~. ndiloma de cuero cabelludo rcinoma basocelular

,
~
~
~ ~ ~
~

\''-1

~
~
~
~

~
~

~
~

~
~

paclente consulta po. astena, . Al examen 77) fs se palpan ~nopatlas ng.ut erec as, de hasta 3,5 cm no dolorosas. No presenta . ' s alteraciones. El nemogrma es normal. El diagnostico ms prObable es: .1. ) Linfoma ~ b) Leucemia mieloide aguda e) Enfermedad por araazo de gato d) Tube~culosis e) M9nonuecleosis infecciosa

Vh .

~.

~
~

~
~

/ ecotomografia abdominal. b) colecistografla. e) tomografia axial computada. d) colangiografia transparieto-heptica. e) col nglografia endoscpica.

1.. ' El mtodo diagnstico ms til Qara iniciar el estudio de una ctericia obstructiv_a}es:
--~-... - -- ..--- -~

...

~ ~
~

~
~ ~

79 Qu patologa se asocia ms significativamente a la presEtncia de infecciones urinarias a r peticin en la edad peditrica? ' ) Rin en herradura b) Criptorqutdea e) yreterocele d)'Reflujo vesicoureteral /e) Hipospadia

~
~
~

~
~

~ ~
~ ~
~

"'
~
~
~ ~
80) Se realiza un estudio para evaluar la validez del diagnstico cUnico en 1.000 pacientes fallecidos
por infarto agudo al miocardio; se utiliza como estndar de oro los resultados encontrados en autopsia. : ... .' Diagnstico clinico Hallazgo de autopsia :Infarto Sin infarto
:, -: ...

"'

~
~

160 40 200

80 720 800

. i:.i, ,~r~
.

. : . .::

.i :::.-:;/1. oo"

.: . 760

e acuerdo a estos datos, el valor predictivo positivo del examen clfnico fue:

~
~

1 d)'ao%
e) 95%

~S7%

a) 20% b),33%
/1, .

"\

~ ~ \) /.:

'\ lil~
..,...-

\\Qo""

1(0

.fii'):~
,
lb

Y'~

~ ~
~
~
~

81) Un paciente de ,58 aos, f.!;!!!!!5!9r, con untecedente de (jbrosis pulmonar, secundaria al uso de e ec omiso 1 nitrofurantona durante la infancia, presenta un cuadro co9:frecuencia cardiaca 93x, frecuencia respiratoria de~, presin arterial de 1j)8/G6 mmHg y s~~ra 9~ con Fi02 amblen l. La auscultacin pulmonar demuestra crpitos bilaterales, mucho ms ;Jtensos en la base pulmonar izquierda. El examen cardaco es normal. Se solicita radiografa de trax 1 e muestra una condensacin en el lbulo inferior izquierdo y los signos de fibrosis pulmonar en el /resto de los campos pulmonares, similare~; a los que presentaba en radiografa previas. La conducta ms adecuada es: a) Administrar penicilina benz~tina intramuscular y enviar a domicilio b) tniciar amoxicilina oral y controlar de forma ambulatoria Alniciar amoxicilina + cido clavulnico y controlar de forma ambulatoria d) Hospitalizar en sala e iniciar ceftriaxona endovenosa e) Hospitalizar en UCI e iniciar ceftriaxona ms claritromicina

estado general. Al examear se observa orientado, bien Qerfundido, decaldo, febril has

8 5'

"'
~

~~1
~~
~

82) El
~)

~ ~ ~ ~

enfermedad car~iovascular. ~ fb). osteoporosisv C) ' j e-Airf:teimer,. d).~E . p) RGef"de-ovarie.

bene.ficJ~_la_tarapliLd~.reemplazo hormonal ~l'i'"la~j;~~;~-~~~~-;~iaes disminuir el ~sgo de:


.....--

~ (!'
~

3) La otitis media aguda no suele complic:arse. No obstante, cuando lo hace, es con ms frecuencia con una: a) Petrooitis. . b) Mel)ingitis. _....e....}Mastoiditis. ~d) 9 rdera sbita. e), steomielitis.

.
;

~
~

f!'

~)En una primigesta con embarazo de 30 semanas, que presenta presin arterial de 150/120 mmHg, edema (+++) y compromiso de concienc~ el diagnstico ms probable es: a) status epilptico b) ncef~lopatia heptica reeclampsia severa d) e'liJ{ola de li_quido amnitico e) e ~is hipertensiva 1 8 ) ~na gestante de 4:1 semanas inicia trabajo de Rarto. Presenta 4 contraccloRSS-Gad~ dilatacin cervical ~' mem~s rotas y feto en segunc:;lo plano de Hodge, con presentacin de frente, en posicin nasoiliaca izquierda anterfOr. La conducta ms adecuada es: Observar evolucin espontnea b) Realizar prueba de parto con aceleracin Ctccitcica y anestesia epidural e) Girar la cabeza hasta posicin nasopbica y dejar evolucionar d).Kealizar forceps ~) Realizar cesrea

Pc

""
~

~
~
na nia de 3 aos es traida ya que presenta secrecin Vlginal hemopurulenta. El diagnstico ms .. pr able es: ~ . Abuso sexual ~~t~!l9initis inespeclfica ~uer o extrao vaginal d) vaginal 1 e) Tri omoniasis

~
~

J
!.

/!

Can~diasis

~
~
~

at) Cul es la prueba con mejor.valor predlctlvo para el diagnstico de cncer diferenciado da'
~r ides?
Cintigrama tiroideo ) Tomografia axial computarizada de cuello e) Ecotomografia tiroidea dfi?uncin del ndulo y citologla e)ibeterminacin de anticuerpos antitiroldeos 1 ' ) Mujer de 63 aos. 48 kg de peso. A las 3 ho~s post c.Qlet:I tectomra recibe 1O mg de morfina dovenosa. A los 6 minutos la paciente est en spCSfprofundo Ciantica. con pulso firme de 120 por nuto y extrasistoles ventriculares ocasionales:' a freeuenc espiratoria es de 2 a 3 por minuto. conducta a seguir debe ser: Administrar oxigeno por mascarilla (1 O litros/min) y control de gases arteriales ik:administrar naloxona y ventilar con Amb /h) Administrar naloxona y pedir gases arteriales . d Admini~trar oxigeno por mascarilla (1 O litros/min) y lidocafna endovenosa Intubar y pedir gases arteriales 89 El mejor examen para determinar el pronstico de un embarazo gemelar es: E(fografia del primer trimestre b) E;o rafia del segundo trimestre e) Ec rafia del tercer trimestre d)/A niocentesis e opsia corial ) Un lactante de 1 ~ses ha presentado 3 crisis bronquiah!s obstructivas en 4 meses. que han spondido al tratamiento con salbutamol. La primera de ellan fue en relacin a una infeccin por VRS. ctualmente se encuentra en buenas condiciones y el examen flsico es normal. La conducta ms adecuada es: a) Iniciar salbutamol si presenta nuevas crisis. sin necesidad de u:;o de corticoides b) Iniciar salbutamol de forma horaria. aumentando la frecuencia de las inhalaciones si presenta crisis e) Iniciar salmeterol inhalado segn horario y corticoides inhalado~, a demanda ,....arfniciar corticoldes inhalados segn horario y salbutamol a dem~1nda Jt9) Solicitar radiograffa de trax y test del sudor

~ ~
~

~
~
~

~
~ ~

~
~

~
~

~
~
~
~
~

~
~ ~

.co;
~
~

~ ~
~

~
~ ~ ~
~

~ ~ ~

r..

Prueba 15 . r. GINECOLOG(A + OBSTETRICIA 1) So~ causas de hlperm~norrea, EXCEPTO:

~'
;~
1

r- ry '\ ..

'\~ .:.

~) 1

,1

'

..

v.

a) Miomas uterinos ~/ b) itteraciones de la hemostasis primaria (Hiperplasla endometrial Cl) Adenomiosis . . e) tero septado ... . .

!\

2) Una paciente de 24 aos, primigestta, cursando embarazo de M.semanas-consulta por deterioro del estado general y dolor abdominal. Al examen se aprecia adolorida y algo Ictrica. Sus signos vitales son FC: 96x', PA: 142/96 mmHg, FR: 15x' y el monitoreo fetal resulta normal. Se solicitan exmenes que/demuestran elevacin de las transaminasas y bilirrubina, asociados a anemia y trombocitopenia levs. En sus exmenes de orina presenta proteinuria de 0,5 g/24 horas. El diagnstico ms probable
es~

'!'

11'

J /!f

Hgado graso agudo del embarazo ) Colestasia intraheptica ) p.reclamsia moderada Sindrome de HELLP e) Sepsis por Clostridium perfringens

ftl

f&'

f!'A
~
~

3) paciente multfpara, cursando un embarazo de 35 semanas se realiza ecografa obsttrica, que demuestra un feto con peso estimado en percentil 5, para la edad gestaclonal, con relacin dimetros c1flico/abdominal de 1,2. La etiologia ms probable del retardo del crecimiento intrauterino, en esta . paciente, es '-..__../a) C9f1Stitucional

Upa

re.

./c) Genopata

b.Yftsuficiencia placentaria d) Sufrimiento fetal agudo e) ~iabetes gestacional

~ ::.Ecografta del primer trimestre

tflt\
fC\

.,

'\.._/

,.gt mejor examen para determinar el pronstico de un embarazo gemelar es:

) Ecografia del segundo trimestre

e) Ecografia del tercer trimestre


d) Amniocentesis e)..,Biopsia corial

~
~

/s) El

~
~

Respecto a la endometriosis es VERl>ADERO que: a) uso de anticonceptivos orales es un factor de riesgo b) Habitualmente se diagnostica mediante ecografia e) Es causa de oligomenorrea asoci~.al uso de dispositivo intrauterino -~) Corre~~onde a tejido endometrial dentro del miometrio

str.se

(Z'I!\
~

fC'
~

6) Una .paciente primigesta, cursando embarazo de 34 semanas, presenta disminucin de la per'1:pcin de los movimientos fetales, por lo que se realiza un test de tolerancia a las contracciones q~ demuestra la presencia de desaceleraciones tempranas o DIP 1, sin otras alteraciones. No presenta modificaciones cervicales. La conducta ms adecuada es:

('lb.
(C\

\ _ _, a) Administrar corticoides endovenosos _... b) lnterrumplir el embarazo por la vla ms expedita e) lndu~ir el parto con misoprostol d) ~plicitar registro basal no estresante . ,e~ranquilizar a la madre y enviar a domicilio

~
(tiA

/1 ~) El elemento que ms orienta a un algia plvica crnica, de origen ginecolgico es:


~{.a ciclicidad
b) El antecedente de metrorragia e} El dolor al tacto vaginal d) El uso de anticonceptivos e) La asociacin a distensiqn abdominal

fl'!i\
~
fl/11!:-.

(8\

,.
!W.'f

,.,, .

8) Una mujer, multipara de 2, con ambos partos por vfa vaginal, est cursando un tercer embarazo, en la semana 37 + 3 dias. Inicia trabajo de parto, que se extiende por varias horas. Al examen fislco presenta 3 contracciones en 1O minutos y el tacto vaginal muestra dilatacin completa, con la cabeza fetal en el segundo plano de Hodge, en presentacin de frente, naso-iliaca anterior. La conducta ms adecuada es: a) Realizar prueba de parto. con aceleracin oxitcica y rotura artificiial de membranas b) :Girar la cabeza hata posicin naso-sacra e) Girar la cabeza hasta posicin naso-pbica , :11~ealizar forceps "'---/ /,.~ealizar cesrea

~) 1EI sfndrome de ovario poliquistico se caracteriza por tod9 lo siguiente EXCEPTO:


~str~genos disminuidos
\.. / ' e) Progesterona disminuida "-.../. d) l:;lrmona luteinizante aumentada .~)--Acn ''-./' / /.. 10) Identifique la asociacin.correcta entre edad geS.taeional y hallazgo: ---..../ a) 12 semanas -fondo uterino a la altura del ombligo (.) b) 15 semanas - multipara inicia la percepcin de los movimientos fetales ~ semanas- beta-HCG detectable en suero y orina .>( .. semanas - ausencia de saco gestacional en la ecografla trSnS\faginal e) 30 semanas - altura uteri~~e 15 centimetros

).l) Andrgenos elevados

Id) 6

11) Una paciente de 24 aos, consulta por dolor abdominal. Al examen se constata paciente subfebril, con dolot a la palpacin hipogstrica, palpacin anexial derecha y movilizacin cervical. No hay sig11os de irritacin peritoneal. El diagnstico ms probable es: a) ..Endometriosis . ~toceso inflamatorio plvico ~--,.-/e) Apendicitis aguda d) Absceso tuboovrico roto e) Embarazo tubario 12) Una paciente de 40 aos, multpara de 3, cursando un embarazo de 38 semanas, se realiza un registro basal no estresante que muestra una frecuencia cardiaca fetal basal de 130x', con una ariabllldad de cerca de 10x', sin presencia de aceleraciones ni de desaceleraciones durante un tiempo de observacin de 2Q minutos. La conducta ms adecuada es: a} Solicitar una ecografla-Doppler de arterias umbilicales b) R alizar un perfil biofisico Xfender el monitoreo P<?r 20 minutos adicionales ) errumpir el embarazo de inmediato e) Tra,nquilizar a la madre y dar de alta

' -..

A
/

13)'Una paciente presenta amenorrea.de 6 meses de evolucin, que Inici luego de un aborto ~spontneo. Se solicita test de embarazo que resulta negativo, por lo que se decide solicitar varios /exmenes que demuestran: TSH normal, prolactinema normal, prueba de progesterona negativa, ' \ / ecograf(a TV con endometrio.muy delgado, prueba de estrgenos ms progesterona negativa. El \ ../ .. origen ms probable de la amenorrea es: a) Anovulacin , b) Hipotafmico e) Hippflsiario .
d~vrico

Jfjlterino

) Embarazo gemelar con el primer gemelo en presentacin ceflica y el segundo en presentacin podlica

1,4) Cul de las siguientes situaciones tiene opcin de parto vaginal?

"-!b) Mujer en trabajo de parto con feto en ceflica con presentacin de cara, en posicin mentosacra e} I;JB,barazo con herpes genital activo 59.-.;pfimlgesta con embarazo complicado con diabetes gestacional y feto de 3.800 Kg. ~e} Mujer con una cesrea anterior de tipo corporal

~
15} Una mujer de 27 aos, cursando embarazo de 32 semanas se realiza ecografia obsttrica que demuestra presencia de polihldramnios moderado, sin otras alteraciones. Ha controlado /decuadamente su embarazo, sin haber presentado patologfas previas. La conducta ms adecuada es: a) Inducir el parto con misoprostol vaginal b) Administrar corticoides endovenosos u interrumpir el embarazo en 48 horas ~olicitar un test de tolerancia a la gluolsa 6't) Realizar amniocentesis e) Controlar con nueva ecografla a las 3H semanas, sin necesidad de estudio adicional

16) l)lia mujer de 28 aos, purpera hs1ce 2 das, presenta aumento de volumen y dolor intenso en relf'cin a la axila d~recha. Al examen se aprecia dicho aumento de volumen, de 4 cm, muy doloroso a la palpacin, con escaso eritema. El diagnstico ms probable es: Mastitis abscedada ~Gndula mamaria accesoria ) Trombo8"mbolismo pulmonar d) Adel)it(s supurada e) M~slitis linfnagitica

)
1

17) Identifique la asociacin correcta:


,) Hipermenorrea - Ciclos menstruales d1~ 20 dias _/ b) ~etrorragi~ -:.~nstru!.t?!!:~.~~.. ~~y abundants-enGa~tid~~. y dlj.f~~ir} ~.-.-- e) ~musonagta-- -Hemorragla.t~h:arante..los dJas_Q~_Q~u1~6n/;d} Oligomenorrea - Ciclos menstruales do 37 das /'e) Amenorrea- Aus.encia.de-menstruaoin-en-~es-ltimos 60 di~~18) La.~ causas ms frecuentes de metrorragia, en las pacientes en edad frtil y en las pacientes trlenopusicas, son respectivamente:
Anovulacin y cncer de cuello \_.// 6} Miomas y atrofia endometrial uterino e} d) Endometriosis e infecciones e) Disfuncional y cncer de endometrio 1Jisfuncional y atrofia endometrial

(1!A

~
~
~
~

~ ~

~ (!lA
~
'-.

f!!"
~

~ (!"
~

19) Una paciente de 30 aos presenta dismenorrea desde hace 4 aos, la que ha Ido en aumento. Ademf:is presenta dispareunia profunda que se ha vuelto ms intensa en los ltimos meses. Refiere que ll'eva 2 aos intentando concebir, sn embargo no lo ha logrado a pesar de no usar mtodos an~iconceptivos y a mantener relaciones sexuales frecuentes con su marido. Al examen fsico presenta dolor a la movilizacin del cuello uterino, son otras alteraciones. El diagnstico ms probable ~s: . ,./a) Cnc;ef de cuello uterino \ _..: ~eiomiosis ,.)) l;ridometriosis / d)..Miomatosis Proceso inflamatorio plvico

, 20) El peodo frtil se extiende desde:


'-.l a) Un dia antes hasta un dla despus de la ovulacin b} Cinco dias antes hasta cinco dfas despus de la ovulacin ftCinco das antes hasta el dia de la ovulacin ,. d) Diez dia antes hasta tres di as despus die la ovulacin ./ e) Siete dlas antes hasta tres dias despus de la ovulacin

(8\
~

f'li'
('\11.

f'/11.

o
1

00 ()00 000 00 00 00 00 00
()00

NOMBRE:

2 3

00 ()00 000 00 ()00 000


()00 000 ()00 000 ()00 000

4
5

000

RUT:

FECHA:

()00 8 00 <)00 9 00 ()00

o o 000 o 000 o 000 o o


A B C D E A B C D E

o o

CURSO: RESPUESTAS

ll%

A B C O E
1

2
3

00000 00000 5 35 00000 6 ooooo. 367 00000 7 00000 s 00000 a OO.OO 00000 00000 00000 10 ooo~oo 40 00000
33 4
34
38

00000 00000 00000 00000 00000

31 32

00000

ooooo

39

00000 62 00000 63 00000 64 00000 65 00000 66 00000 67 00000 68 00000 00000 70 00000
61
69

l't

"'\

~
~

11

12 13 14 . 15 16 17
18

00000 00()00
OOC>OO OOC>OO

ooc,oo

OOCtOO. OOCtOO

19 20

00()00
00()00

OOC>OO

ooc>oo

00000 42 00000 43 00000 44 00000 45 00000 46 00000 47 00000 48 00000 49 00000


41

71 72

.73
74 75 76

00000

00000 00000
00000

...
At\'1
~
~
~

nooooo
78 79
80

00000 00000
00000

soooooo
51
52

ooooo

00000

~
~
~

00000 00000 23 00()00 53.00000 24 00000 ' 54 00000 25 00()00 55 00000 26 00<)00 56 00000 27 00C)00 57 00000 28 00<)00 58 00000 29 00()00 59 00000
21 22
30

00()00

81
82

83 84
85

00000

00000 00000

~
~
~

86 87
BB

00000 00000 00000

00000

~ ~

00.000

89 90

QQ()QO

60

00000

00000 00000

~
~

,.rlt!!JI
~

~
~

~ ~

r. \,._)
\

i "~(\ :~\ n r'

Prueba 14 OBSTETRICIA 1) Una paciente de 35 aos presenta atraso menstrual de un mes, por lo que se realiza un test de embarazo, que resulta positivo. Se encuentra asintomtica y acude al gineclogo, quien le realiza una ecografra transvaginal, que no visualiza saco gestacional. La conducta ms adecuada es: a) Repetir el test de embarazo b)..-Realizar nueva ecografia en 24 horas . -ASOticitar beta-HCG plasmtica /' d) Administrar metotrexato e) Re~liz;r legrado uterino 2) Una mujer de 19 aos, primigesta, cursando embarazo de 30 semanas, presenta presin arterial de 156/98 mmHg en su control obsttrico. No presenta srntomas y la protelnuria cualitativa resulta+, sobre un mximo de 4+. La conducta ms adecuada es: ~) Iniciar alfa-metildopa b) Solicitar proteinuria cuantitativa, de 24 horas .P(Solicitar hemograma, tiempos de coagulacin, pruebas hepticas, LDH, pruebas de funcin renal y / proteinuria de 24 horas ~ Administrar sulfato de magnesio endov:moso J lndi'1lr semihospitalizacin y controlar la presin arterial frecuentemente, durante las prximas horas
/

-~
\

3) tCul de las siguientes opciones supone un mayor riesgo de parto prematuro? p(Preclamsia /b~_2iabetes gestaclonal . )1 ~arto prematuro previo d) Vaglnosis bacteriana e) Primigesta

4) La primera ecografia obsttrica, que se realiza a mediados del primer trimestre, tiene como objetivo principal: , j a) Determinar el sexo del beb ' \ b) Identificar los embarazos molares y ect6picos / ~--Detectar malformaciones fetales . / d) Evaluar el crecimiento fetal ~Confirmar la edad gestaclonal 5) Una paciente de 34 aos, cursu:wdo embarazo de 8 semanas, consulta por metrorragia de 2 horas de evolucin, con escaso dolor hipogstrico. Sus signos vitales son normales y presenta tero mayor al tamai1.o esperado para su edad gestacic)nal, palpndose a la altura del pubis. La especuloscopia coiJfirma que la sangre proviene de la cavidad endometrial. La conducta ms adecuada es: ~)'Solicita subunidad beta de gonadotrofina corinica b) Realizar legrado uterino , \ \_/ .P.r'Realizar ecografia ttansvaginal )/l) Realizar registro basal no estresante e) lnicia~_.rretotrexato 6) Uo~ mujer de 40 aos, cursando embarazo de 28 semanas, presenta un test de tolerancia a la gh,aosa, con 75 gramos de glucosa, demostrndose glicemia basal de 107 mg/dl y glicemia post )36brecarga de glucosa de 150 mg/dl. So decide indicar dieta y ejercicios. Dos semanas despus acude /a control, constatndose glicemias basales de 90 mg/dl y postprandiales de 110 mg/dl. La conducta . \...../ m~,-adecuada es: ~"Mantener la dieta y ejercicio, controlando peridicamente . -/'b) Solicitar hemoglobina glicosilada y decidir manejo segn resultado e) Iniciar metformina d) Iniciar insulina e) Solicitar nuevo test de tolerancia a la glucosa

7) Una mujer de 32 aos, cursando embarazo de 37 semanas, diagnosticada de preclamsia severa, sin complicaciones, inicia contracciones uterinas intensas y frecL;entes. Se constata dinmica uterina de 3 en 1 o minutos y se palpa un cuello borrado en un 80%, con dilatacin de 3 cm. El resto de su examen fsico no aporta mayor Informacin. El reglstro fetal no demuestra alteraciones. La conducta ms ~cuada, de entre las siguientes es: /Administrar alfa-metildopa oral durante el trabajo de parto -. ~ ~dministrar su!fato de magnesio endovenoso e) Solicitar eC9dOppler umbilical y de arterias uterinas ("- d) 1~1"-GQI:ti~oliticos e) Aoniriistrar hidralaZlnaon:tiJetalol por vi a endovenosa

,., ,.,
~

~ ~

v'

~
~

~
~

8) Un mujer de 30 aos, cursando embarazo de 10 semanas, 1:onsulta por metrorragia, confirmada c9r{ especuloscopfa. La ecograffa transvaginal muestra un feto vivo~ con saco gestacional indemne. El tcto vaginal no presenta modificaciones cervicales. La cqnducta ms adec~ada es: //a) Hospitalizar y administrar tocofticos 1 , b) Hospitalizar y observar hasta que la metrorragia ceda completamente \../ o}-Enviar a domicilio indicando reposo fd) Solicitar niveles plasmticos de subunidad beta e) Realizar legrado uterino 9) Un paciente de 23 aos, multfpara de uno, cursando un nuevo embarazo de 30 semanas inicia dinmica uterina persistente. Al examen fsico se palpa feto en presentacin ceflica, se constatan 4 contrac;iones cada 10 minutos y el tac~o cervical demuestra borramiento de 50%, con dilatacin cervi9l de 2 centfmetros, con membranas integras. El registro fetal resulta tranquilizador. La conducta msadecuada es: '4\aministrar corticoides endovenosos a la madre e iniciar tocolftieos ) Administrar corticoides y antibiticos a la madre e) Administrar tocoliticos y antibiticos a la madre d) Dejar a evolucin espontnea e) Realizar cesrea

A\
~
~
~

~
~

'

1O) Una mujer de 38 aos, multfpara de 2, con ambos nios nacidos por cesrea, cursa su tercer embarazc;>;'con 40 semanas. Consulta por dinmica uterina y luego salida de abundante lquido por genitales. Se constata dinmica uterina de 3 CU en 10 minutos, se palpa cuello completamente bor9to, con 5 cm de dilatacin y feto en presentacin ceflicca, en posicin occiplto ilfaca izquierda \ ~nsversa en espinas -2. La conducta ms adecuada es: .-..._/a) Administrar tocollticos endovenosos b) Iniciar antibiqticos endovenosos e) Dejar a evolcin espontnea d} Realiz~.r-frceps ~ ~e~_l.izr cesrea . . 11)Una mojer de 26 aos, primigesta, cursando embarazo de ~~O semanas presenta prdida de lquido .claro por genitales, con olor a cloro. En el examen ob.$ttrico no se detectan alteraciones. El test de / cristalizacin muestra formacin de u hojas de helecho" y el perfil bloffslco resulta 10/1 O. La conducta ~ ms adecuada es: a) 1 nducir el parto con misoprostol .
~~~~~

e) Administrar corticoides. antibiticos y tocolfticos d0c:Jministrar cortlcoides y antibiticos e Inducir el parto en 48 horas ~ Administrar corticoides y antibitcos, manteni~ndo una conducta expectante.
'
'

12) .Una mujer de 40 aos, cursando su cuarto embarazo, de 1~ semanas, presenta presin arterial c~rcana a 160/100 mmHg en varios controles. El resto del examen ffslco no presenta alteraciones y .~oto presenta proteinuria cualitativa leve (una+). La ecografa obsttrica no presenta alteraciones. El \ /diagnstico ms probable es: \..._/ a) Embarazo molar b) Preclamsia moderada e) !:JjPertensin transitoria del embarazo 9Yflipert~nsin arterial crnica e) Preclamsia leve

" "

""

13) La diabetes gestacional se produce t>or: . ~)AY.mento de la insulina fetal ,,_/brumento de la neoglucognesis (conver:)in de protefnas en glucosa) en la placenta e) Disminucin de la secrecin de insulina pancretica, por efecto de la progesterona ~ Aumento de hormonas de contrarregulac:in e) Aumento del consumo de glucosa por pc:1rte del feto y de la placenta

. . . . . .

(A

14) Una mujer, cursando embarazo de 3E; semanas presenta metrorragia moderada, asociado a dolor abdominal. Al examen s.e aprecia hlperto~nra uterina, sin modificaciones cervicales. La especuloscopla demuestra salida de sangre por el orificio cervical externo. El registro fetal constata variabilidad m~i!Jl, desaceleraciones variables y taquicardia fetal. El diagnstico ms probable es: . ~~fj5esprendimiento prematuro de placenta normoinserta 1 Placenta previa '):/' e) Rotura de vasa previa . d) Rotura uterina e) Aborto en evolucin

_.:hf

~
~

~
~
~
(l!!l

15) Deben recibir profilaxis para Estreptc,coco betahemoltico grupo B, durante el trabajo de parto, todas ~as siguientes embarazadas, EXCEPTO: a) P~migesta de 17 aos, sin antecedentes de importancia, en trabajo de parto prematuro de 35 semanas de edaf gestacional b) Multpara de .aos, en trabajo de parto de 40 semanas, con antecedente de sepsis neonatal en de su parto anterior -v/ e} Multipara de 28 aos, si!).afltecedentes de importancia, cursando embarazo de 32 semanas, con RPO de 24 horas de evolucin '\,./" . .... d) Primigesta con urocultivo positivo para SG~ . ~r-Multipara, cesarizada anterior, cursando t~mbarazo de 38 semanas, complicado con diabetes gestacional y /~ preclamsia moderada

31.

(A

16.) El sntoma principal de la colestasia intraheptica del embarazo es: _,;{fPrUiito \..._/! b) Coluria e) Ictericia d) Dolor e) Naseas

f:'
~
~

O'
\

(/ i
/1y

,....17) Una mujer de 31 aos, multfpara de uno, cursando embarazo de 37 semanas presenta rotura de

~
~ ~
~

membranas, sin iniciar el trabajo de parto. Al examen se palpa feto en presentacin podlica y se con,~tata ausencia de dinmica uterina y de modificaciones cervicales. En la especuloscopra se ') ~precia salida de escaso Hquido por el OC: E. Las pruebas de bienestar fetal son normales. La conducta /ns adecuada es: ....__,.,.,/ ~)~ucir el parto con misoprostol Realizar cesrea e) Administrar corticoides y antibiticos, reali:zar amniocentesis y solicitar estudio de liquido amnitico y de exmene.s generales a la madre. d) Administrar corticoides y antibiticos e inducir el parto en 48 horas e> ..Administrar corticoides y antibiticos, manteniendo una conducta expectante.

~
~

./.18) El principal objetivo del tratamiento dE!I VIH durante el embarazo, a fin de evitar la transmisin al
---- ./ feto, es: a) Mantener el mximo nivel de linfocitos CD4+ b) Evitar infecciones oportunistas e) Mantener niveles adecuados de antirretrovirales durante el parto )iflograr cargas virales mlnimas , / e) Negativizar el ELISA para VIH

~
~
(li!\
~
(11\

19) Una mujer de 35 aos presenta parto vaginal, persistiendo con abundante hemorragia luego del parto y del alumbramiento. Se palpa el fondo uterino entre la .apfisis xifoides y el ombligo. El diagnstico ms probable es: retismo(51acentarioceracin del canal del-parto , / e) Restos ovulares \ d) Rotura uterina ~Inercia uterina / 20) Cul de las siguientes asociaciones es INCORRECTA? a) Huevo anembrionado -Aborto retenido . / b) Modificaciones cervicales- Aborto Inevitable/' _gfistes tecolutelnicos - Mola completa .. Ya) Metrorragia con hipotensin- Embarazo ectpico roto '---" @' Incompetencia cervical- Abortos tempranos a repeticin _./,..
,./

l\

'---

..-

....

r
o
1

~
NOMBRE.:

~ ~

2 3 4 5

~
RUT:

f!'

~ ~
~ ~
~

6 7
8

FECHA:

00 00 00 00 00 00 00

00 000 000 00 000 000 00 000 000


000 000 000 000 000 000 000 000 000 000 000 000 000 000

o o o o o o o o

o o

CURSO: RESPUESTAS A B C D E
. 1

A B C D E
31
32 33

~
~

~
~
~

~
~ ~ (O' ~

00000 2 00000 3 00000 4 00000 5 00000 6 00000 7 00000 8 00000 9 00000 10 00000 00000 00000 13 00000 14 00000 15 00000 16 00000 17 00000 18 00000 19 00000 20 00000
11 12

A B C D E

34
35 36 37 38 39 40

00000 00000 00000 00000 00000 00000 00000 00000 00000 00000 00000 00000 00000

00000 00000 63 00000 64 00000 65 00000 66 00000 67 00000 68 00000 69 00000 70 00000
61 62 71 72 73 74 75 76

41
42
43

~
(iit'

44 45
46 47
48 49

00000 00000

00000 00000 00000 00000 00000

~
~
~
~
(1it'

nooooo
78 80

00000 00000 00000 00000 00000 00000 00000 00000 00000

79

so
51 52 53 54 55 56 57 58 59 60

f!"'
~

f!!"
~
(191

00000 00000 23 00000 24 00000 25 00000 26 00000 27 00000 28 00000 29 00000 30 00000
21 22

r4'l
~

00000 00000 00000 00000 00000 00000 00000 00000 00000 00000

81 . 82 83
84

85 86
87
88

00000 00000 00000 00000 00000 00000 00000 00000 00000


00.000

89 90

~ ..

re't
~

(4\
(11$:\

!'

Prueba 12 PEDIATR(A

1) Un nio de 6 aos no habla y demor rns de lo normal en empezar a caminar. La madre refiere que
no contesta al hablarle ni tampoco responde _al indicarle cosas. Juega con sus juguetes, sin interactuar con otros nios. Al' examen ff!;ico no presenta signos neurolgicos focales. El diagnstico ms probable es: a) Sim!rome de dficit atencional ~idr~me de Asperger

.)<_
'

.~utismo
d) r

rno del lenguaje e) Hipoacusia

2) Un nio de 6 aos presenta edema facial y de extremidades inferiores, asociado a orinas espumosas. Se ve de buen aspecto y sus signos vitales son normales. Se solicita hemograma que resulta normal, sedimento de orina con cilindros hialinos y gotas de grasa, sin hematuria; fndice pr.oteinuria/creatininuria de 3,1 y albuminuria de 2,7 mg/dl. El diagnstico ms probable es: .,..) Glomerulonefritis postinfecciosa 1 / b) Enfermedad de Berger \_/ e) Sindrome hemolitico urmico d) Glomerulopata membranosa pliilerulopatia por cambios mfnimos
3) Un .nio de 30 das de edad es llevado a control de nio sano. En el examen fislco no es posible pa!Pr el testculo derecho. No tiene regiutros de exmenes testiculares previos y el resto del examen fs1co es normal. La conducta ms adecuada es: . "//') Solicitar testosterona y gonadotropinas plamticas "'-../ b) Solicitar cariograma Solicitar ecografia inguinal y testicular / d~ Solicitar tomografia axial computada de abdomen, pelvis y zona inguinal e) Realizar exploracin quirrgica _..

~ /.
'

4) La obesidad infantil es una patologa cuya frecuencia va en aumento, debido principalmente a un mal desarrollo de hbitos alimenticios y de actividad fsica. Cul de los siguientes nios presenta obesidad? -. ;(Nio de 5 meses, alimentado con lactancia materna exclusiva, con ndice peso/edad ndice /~so/talla +2.405 e in dice talla/edad +0,1 OS .--- ..,. -- -(~}lio de 6 aos con fndice peso/edad +1,0 OS, indice peso/tallaC-.:2,1DS:,e lndice talla/edad -1,105 (-e) matte~7-.a~JMe-2i/' ..--- -- d) Nio de .1 ao con fidiee-peseledad-+1-5 B&,-indice-peso/talla 1-,-5-I:>S e.. fndlce-talla/edad +1;0-98 e) Nia de 4 aos con lndice peso/edad +2,(i:P~EiiGe..peso/~alla-C);flQS-e-indice-talla/edad.=f-2,3DS---

<s o_}

5) Un nio de 6 meses de edad cae de la ::ama, golpendose la cabeza. La madre acude muy preocupada y refiere que inicialmente el lactante llor por varios minutos, pero luego se tranquiliz. Al ex8J11en fsico se observa en buenas condiciones, con tomo muscular normal, reflejos conservados y ausencia de signos focales. Slo se observa un pequeo hematoma en la zona occipital. La conducta rTs adecuada es: \ ,~/a) Enviar a domicilio, explicando los signos de alarma para reconsultar \ / bLSolicitar radiografia de crneo \../ :.ct'Solicitar TAC de cerebro /, d) Iniciar AINEs y derivar para atencin ambulatoria con neurlogo e) lniciar.pracetamol y fenobarbital, controlando en 24 horas /

6) U.n/nio de 13 meses ha presentado 4 otitis medias agudas y 3 neumonfas, desde los 9 meses de e~d. El diagnstico de sospecha es: ...~~ibrosis quistica .-;! ~fic!t de inmunoglobulinas -- e) Dfictt de complemento d) Sindrome de inmunodeficiencia adquirida e) lnmunodeficiencia celular

7) Un urocultivo por sondeo ser diagnstico de infeccin urinaria, si presenta unidades _formadoras de cofonias, en una cantidad mayor de:

a)_l'
1;)'100

/cljt.OOO v/~1'> 1o.oq.o

e) 10Q:OOO

?,:) Fractura de antebrazo


/ ..__/"

S}.~{cul de las slg~lentes lesiones es ms caracterfstica de maltrato infantil?


b) Hematoma de la zona frontal e) Fractura suprancondllea de hmero ~ractura de metatarsiano .-ractura de escpula 9) El tratamiento de eleccin de la laringitis aguda obstructiva es: RAalaEJo ~ a) albuta Adrenalina racmlca inhal_da_ e) bl::if8ri51 y ortlcoJdes inhalados d) Adrenalina racmica y cortlcoides inhalados Adrenalina racmica Inhalada y corticoides orales

r'

1O) La coartacin artica NO presenta:

1 ') Soplo sistlico

b) Mayor riesgo de endocarditi~ ..../ e) Hipertensin arterial ,P(.ianosis . e) Dism;ncin de los pulsos pedios
/

.~

~___,

. /J:! V
-

11)tJn nio de 6 aos presenta una convulsin tnico-clnic:a mientras estaba en su colegio, a}!olimltada, de 5 minutos de duracin. Al examen ffsico no presenta alteraciones. La conducta ms jldecuada es: . /~U nielar anticonvulsivantes , ~olicitar electroencefalograma con privacin de sueo - e) SolicitarTAC de cerebro d) Solicitar creatininfosfoquinasas e) Realizar puncin lumbar y solicitar estudio de liquido cefalorraquideo

V
/

b) Trasmisin de proteccin materna, mediante inmunoglobulina A e) cidos grasos esenciales adecuados a los requerimientos del lactante ~f"'portes ptimos de hierro, para los requerimientos del nio e) Fortalecimiento del lazo madre-hijo

/'> Osmolaridad ptima, para una adecuada absorcin

1~)'i.a lactancia materna tiene todos los siguientes beneficios, EXCEPTO:

13) Un nio de 3 aos presenta cojera de 2 dias de evolucin. Al examen se observa en buenas col)diciones, con slo leve dolor a los movimientos extremo:;a de la cadera izquierda. El diagnstico R)s probable es: ,.) Epifisiolisis de la cabeza femoral ~inovitis transitoria \ _~:1-~) Artritis sptica \ J ~ d) Enferm~.dad de Perthes e) Slndrome de Osgood Schlatter
/'

.\_)J~'Virus

!i)).i6'os lactantes menores, la mayorra de ) Neumococo e) Mycoplasma d) Estreptococo beta-hemolftlco grupo 8 e) Estreptococo beta-hemolltico grupo A

l~s neumonfas son causadas por:

~
15) Un nio de 6 meses de edad prese,nta una pielonefritis aguda, la que es tratada de manera exitosa con ceftriaxona endovenosa. Luego e:s estudiado con ecograffa abdominal que resulta normal y con uretrocistografa miccional seriada, que demuestra leve grado de reflujo vesicoureteral. Se decide indicar profilaxis antibitica, en espera de la resolucin espontnea de su p~tologia. Qu antibiticos .ms adecuados para dicha profilatxis? . ,. iprofloxaeino o nitrofurantofna \'/_ itrofurantona o cefadroxilo . e) Cefadroxilo o cotrimoxazo1 d) Cotrimoxazol o eiprofloxaeino e) Ciprofloxaeino o eefadroxilo

~ fA

" "
~ ~
~

(A

fA

16) t,.~{~ravedad de la enfermedad de J<awasaki se evala MS adecuadamente con: lclfnica /Ecocardiograma : Agiografa coronaria \ 'V d) Hemograma e) Biopsia .

"
(!A

(A

17) antibitico de eleccin para la meningitis bacteriana aguda, a germen desconocido, en nio de 6 aos, es: a) Penicilina + metronidazol Ceftriaxona + metronidazol + dexamet.:ssona gf\'ancomieina + eefotaximo + dexametasona ;/d) Ceftriaxona e) Penicilina+ rifampicina + dexametasona

ELt~~miento yn

"
~
~

18) Un nio de 2 aos presenta desde ayer en la noche mucha tos, con expectoracin, sin fiebre ni rinorrea. Al examen fsico se auscultan algunos crptos y slbilancias en la base pulmonar derecha, sin otras alteraciones. El diagnstico rns probable es: a) Bronquiolitis /'- b) Neumonfa e) Laringitis obstructiva ( ~- ra}j~uerpo extrao bronquial . ~lndrome bronquial obstructlvo 19) Un nio d~ 4 aos es diagnosticad~> de tos convulsiva con IFD para Bordetella pertusis, inicindose tratamiento con eritromicina. Tiene un hermano de 8 aos, que est asintomtico y tiene todas sus. vacunas al dia. La conducta ms adecuada para con el hermano es: ~Indicar control en caso de sfntomas /..,.. b)Solicitar IFD para Bordetella pertusis e) Solicitar hemograma y radiografia de trax d})n?i;~!..~~~ ~0..~~~ de vacuna DPT acelular lm1ar entromtema _,

"
~ ~
~
~
. 1

J.

>'""-

~
~

;e>
.1'

" - - - - - - - : _ , . - - - - - - ... - -

--~

~
~ ~

"'
~
~

20) UJl'lio de 5 aos, sin antecedente.s de importancia, presenta prpura petequial y algunas eq~ifnosis en las extremidades inferiores, de 48 horas de evolucin, sin otros sintomas. La madre r9fiere que habria estado resfriado hacre 1O dias. Al examen trsico est en buenas condiciones, /'.2Jlstatndose slo las alteraciones ya descritas. La conducta ms adecuada es: /_.:J Solicitar hemograma, con recuento de plaquetas \ _/ .,.. b) Solicitar TP, TIPA y tiempo de sangria ve) Iniciar corticoides orales d) Derivar para esplenectomia e} Solicitar biopsia de mdula sea

~
~

~
~

o
1

NOMBRE:

2 3
4

5 . RUT:

6 7
8

FECHA:

00 00 00 00 00 00 00 00 00 00

()00 ()00 ()00 ()00 ()00 ()00 ()00 ()00 <)00 <)00

000 000 000 000 000 000 000 000 000 000

o o o o o o o

o o o
E

4
~

~
~
~

CURSO:

1
. RESPUESTAS A 1 2 3 4 5 6 7 8 9 10 B

O E
31 32 33 34 35 36 57 38 39 40

00000 00000 00000 00000 00000 00000 00000 00000

ooo~oo

ooooo

00000

ooooo

61 62 63 64 65 66 67 68 69 70

ooooo

.-,
~ ~

00000 00000 00000 00000 00000 00000 00000

ooooo

00000

00000 00000 00000 00000 00000 00000 00000

ooooo

~
~

-~
~

11 12 13 14 15 16 17 18 19 20

00000 00000 00000 OOC>OO OOC>OO OOC>OO

ooc,oo

00000 000100 00000

00000 00000 43 00000 44 00000 45 00000 46 00000 47 00000 48 00000 49 00000 so 00000
41 42

00000 00000 73 00000 74 00000 75 00000 76 00000 n 00000 78 00000 79 00000 80 00000
71 72 81 82 83 84 85 86 87 88 89 90

~ ~

....,
~
~

~
~

OOC>OO 00()00 23 00()00 24 00()00 25 00()00 26 00()00 27 00()00 28 00()00 29 00()00 30 00()00
22

21

00000 00000 53 00000 54 00000 55 00000 56 00000 57 00000 58 00000 59 00000 60 00000
51 52

00000 00000 00000 00000 00000 00000 00000 00.000 00000 00000

..,
~
~
~

A)
~

~
~

~ ~
~

(f:'
~
~
Prueba 13 PEDJATR[A 11

.6 I./ )
1

~ ~

~
~
~
~

1) Un nio de 7 aos presenta un cuadro de astenia, fiebre Intermitente y sudoracin de 3 semanas de duracin, que ha tratado con paracetamol. Presenta adems dolor en el hombro derecho y el tobillo ;'izquierdo que limita los movimientos do dichas articulaciones. Al examen fislco destaca palidez y algunas petequias en abdomen y extremidades inferiores. La auscultacin cardiopulmonar es normal. El diagnstico ms probable es: a) Prpura trombocitopnico agudo b) Artritis sptica e) f?ndocarditis infecciosa subaguda rPft:eucemia aguda e) Mononucleosis infecciosa

~ ~
~
~

~ ~
~ ~ ~
(iA

2) Un lactante de 6 meses es trafdo por un cuadro de 2 das de evolucin de fiebre hasta 38,1C, asociado a tos, a lo que hace algunas horas se le agreg dificultad respiratoria. Al examen fsico se obsery con taquipnea leve, asociada a, aleteo nasal, sin uso de musculatura accesoria ni cianosis. La auscitacin pulmonar demuestra escasas sibilancias difusas y la saturacin de oxigeno por pul,ometria es 97%. Se encuentra afeb~ril. La conducta ms adecuada es: ~dminstrar salbutamol inhalado y si responde bien, enviar a domicilio con inhalaciones de salbutamol e dicaciones de control en caso de aparicin de signos de alarma Administrar oxigeno y bfOncodilatadore:s nebulizados y si responde bien, enviar a domicilio con salbutamol en aerosol y prednisona oralp~ 5 dfas, con indicaciones de reconsultar en caso de empeoramiento e) Administrar ac;trenalinfi~9riica por nebulizacin y dexametasona oral y si responde bien enviar a domicilio con indicaciones de control'im caso de aparicin de signos de alarma d) Administrar broncodlata~ofes nebulizados y una dosis de corticoides orales, y si responde bien, enviar a domicilio con salbutamoi~~ inhalador, asociado a amoxicilina oral e) Ho~pitalizar, administrar oxigeno, bronc:odilatadores inhalados, corticoides endovenosos y solicitar radiografia de trax

3)/~n segundo bsico se administra la vacuna DT, en lugar de la vacuna DPT. Esto se debe
ndamentalmente a: La baja incidencia de tos convulsiva en nios mayores L~ alta inmunidad alcanzad.a con las dosis anteriores de la vacuna DPT, contra la tos convulsiva ~}{'a alta tasa de efectos adversos de la vacuna DPT en nios mayores /d) La poca efectividad de la vacuna DPT E!n nios mayores e) La administracin conjunta de la vacuna trivfrica, que hace innecesaria la vacuna DPT 4) Una nia de 7 aos, extranjera, cons1Jita por un cuadro de 4 dfas de evolucin, que comenz con mal,estar general, artralgias, ligera odinofagia y fiebre ha~ta 38,2C, a lo que se agreg un exantema eritematoso generalizado, macular, de c:olor rosado. En el examen fsico destacan adems varias Cien_ opatfas cervicales posteriores. El diagnstico ms probable es: S,!3rampin 'Rubeola ,.../e) Eritema infeccioso d) Exantema sbito e) Es~rlatina
/

~
~

~ ~
~ ('A
~
.

~ ..
~

~
~
~
~

\;

5) Un nio de 3 meses presenta un sopllo sistlico eyectivo IIINI, asociado a un desdoblamiento fijo del seAundo ruido. Usted debe sospechar: ~1 G,omunica~in interventricular . b){;6municacin interauricular Vc) Ductus. rterioso persistente d) Coartcin artica e) Tr~posicin de grandes vasos
1

6) tul de las siguientes cardiopatas congnitas se acompaa de cianosis? Drenaje anmalo de las venas pulmonares ) Comunicacin interventricular ~}Ventriculo nico .. d) Ductus arterioso persistente e) Coartacin artica

~
~ ~
7) Cul de las siguientes alternativas representa mejor E~l cuadro clfnico de la

glomerulonefritis postestreptoccic~?/ a) Hematuria dismrfica, proteinuriqJ~y~ edema e hipertensin arterial, asociadas a elevacin transitoria del complemento, que inicia 7 a 14_dfas despus ele una amigdalitis pultcea b) Hematuria aislada, con compleme~qrm.al, durante una infeccin estreptoccica e) Proteinuria, lipiduria, orin~spumosas y edema, sin hipertensin y con complemento normal, q~ inicia 1O das despus de una amigdalitis pultcea .-:2.J'+-iematuria, edema e hipertensin arterial, asociada a hipocomplementemia, que inicia 1O dfas fespus de una infeccin estreptoccica . ~e) Proteinuria, edema e hipertensin arterial, asociada a hipc,complementemia, durante una infeccin estreptoccica

~
~

~
~

~ ~

~
~

'-......_ f

8) Un lactante de 6 meses presenta fiebre hasta 38,7C, sin otrc,s srntomas y con examen fsico normal. La conducta ms adecuada es: ~ a) Iniciar antipirticos, solicitar hemograma y sedimento de orina y decidir conducta segn resultados b) Iniciar cefadroxilo oral y antipirticos, sin necesidad de exmene~s ~iG.iar antipirticos y controlar en 48 horas o en caso de empeoramiento, sin necesidad de mayor estudio ~~~spitalizar, iniciar antibiticos endovenosos y solicitar hemograma, VHS, sedimento de orina, urocultivo, rad1ogr~fa de trax y puncin lumbar, decidiendo manejo segn resultados e) lnipiar antipirticos y claritromicina, controt.ando en 48 horas o en caso de empeoramiento Un nio de 3 meses presenta una circunferencia craneana e11 percentil 95 para su edad. La talla y peso estn en percentil 80 para su edad. Al examen fsico se olbserva en buenas condiciones generales, con signos vitales normales y fontanelas a nivel. El examen neurolgico es normal, al igual que el desarrollo psicomotor. El diagnstico ms probable es: a) Hidrocefalia b) Craneosinostosis e) Menirgitis viral d) X"ORCH Macrocefalia constitucional

~
~
~

~
~

~
~

~
~ ~

7
/

1O). La adrenarquia se caracteriza por: a) Aparicin de vello pbico b.) A.u mento de la velocidad de crecimiento e) lryicio de la fertilidad / d) lor axilar adulto ... /) Pigmentacin cutnea 11) Qu hallazgos espera usted encontrar en las pruebas hepticas de un nio con hepatitis A? a) Elevacin discreta de las transaminasas e hiperbilirrubinemla de predominio indirecto p(Elevacin marcada de las transaminasas e hiperbilirrubinemia dB predominio indir~cto e) Elevacin discreta-de.J.~ lrans-_minasas .~. hlperbilirrubinemia .de .predominio...directo t) Elevacin marcada de taslrasarmiiasase hiperbilirrubinemia dB predominio directo e) Elevacin marcada de las transaminasas sin,hiperbilirrubinemia

~
~

~
~

~
~
~
~

12) Un latante de 6 meses de edad, 7.200 grs. de peso y que r3cibi durante 3 meses lactancia
materJl, es enviado desde su Sala-Cuna al servicio de urgenca por un cuadro diarreico de 24 horas de 7volucin, con fiebre baja y presencia de 4 vmitos en las ltimas 12 horas. Al examen fsico usted lo/ncuentra con mucosas algo secas, consciente, irritable, sediento. La madre se ve preocupada, eceptiva y cooperadora. Cul es la conducta que se debe ini:ciar? . a) HoSf)it'alizaHtJ-nmo-;paratehidrataein intravenos' b) Indicar dieta hdricacon variados liquidas (aguas de hierba, Coca-Cola, etc) por 8 horas, seguida de realimentacin a concentracione~ graduales de leche (que dure 4 ;:,5 dlas) y control cada 2 a 3 das p) 'Administracin de sales de rehi.dratacin oral segn esquema preciso, durante 4 a 6 horas, seguida de realimentacin relativamente rpida y control peridico del paciente~ d) Administracin de un antibitico de amplio espectro y sales de mhidratacin oral, con seguimiento frecuente del paciente e) Dieta hldrica con variados liquidas (aguas de hierba, Coca-Cola. etc) por 8 horas, seguida de . realimentacin a concentraciones graduales de leche (que dure 4 l 5 das) y control cada 2 a 3 das, adems de administracin de frmacos opioides que reduzcan lamotilidad 1n~estinal (loperamida, por ejemplo)

~
4t)

J;

~
~

~
~
~

~
~

,.,
~

~ ~
~ ~
13) En IQ.s nios, la complicacin ms fr1acuente de la varicela es: a) Cecebelitis postvaricela b)~umonfa varicelatosa ~) Hepatitis viral 'V~ SC>breinfeccin de las lesiones por bacterias Gram positivas e) Slndrome de Reye

~ ~ ~ ~
~
~

..

~
~

14) Un recin nacido prematuro, de 3 dia.s de evolucin presenta Irritabilidad, vmitos, distensin abdominal y hematoquezia en una oportunidad. Al examen ffsico se observa irritable, en buenas condiciones con el abdomen distendido, blando. Se solicita radiografra de abdomen simple que demuestra neumatosis intestinal, sin ail'l:! en la cavidad peritoneal. La conducta ms adecuada es: a) Realip:ir colonoscopfa b) S~iitar TAC de abdomen y pelvis e iniciar antibiticos endovenosos idratar por vla oral, iniciar antibiticos orales, suspender la lactancia materna y alimentar con frmula lf~dministrar hidratacin por vla endovenosa, instalar sonda nasogstrica, suspender la alimentacin oral e flciar antibiticos de amplio espectro e) Resolver quirrgicamente a la brevedad 15) Los 3 hitos del desarrollo psicomotor, que el lactante logra normalmente al cumplir el ao de edad, son: ____ ....- .. a) Gat'a, sala5-f)rtmeras palat;msy pasa objetosdeuna mano a otra Camina sin apoyo, pasa objetos de una rnano a otra y dice sus primeras palabras -~ Caminar sin apoyo, uso de la pinza madura entre pulgar e fndice, y uso de las primeras palabras d) UsqJialas-pfimeFBs-palabras, continensia. urio.aria diuJoa y.caminar_conapoyo e) Caminar sin apoyo,~formacin de .las primeras .frases.y__co.ntinen.cla.urinaria diurna.

f!it'

~ ~ ~ 1

J{

16) Un nio de 1O aos, diabtico tipo 1, en tratamiento con insulina en esquema intensivo, presenta cuadro de sudoracin, temblor y actitud agresiva, seguida luego de inconciencia. El diagnstico ms prqbble es: ~)Accidente vascular enceflico /b) Cetoacidosis diabtica "--/ c.) ~oma hiperglicmico hiperosmolar ..ctf'Hipoglicemia 7 e) Encefalitis herptica 17) Un recin nacido de 20 dias de vida presenta hematemesis en una oportunidad, sin otros sintomas. Al examen ffsico se observa en buenas condiciones generales, bien peundido, con pulso y presin arteriill normales para la edad. El examen segmentarlo es normal. La conducta ms adecuada es: a)-Solicitar endoscopfa digestiva alta ,6) Iniciar reposicin de volumen con cristaloides por va venosa perifrica gruesa \,,J/ ptSoliitar test de Apt-Downey de la sangre emitida _./l) Solicitar hemograma e) Adm~nistrar omeprazol endovenoso
~

lf!A

J,:m
, \ --

1~-Nio de 2 aos consulta por cuadro dle 2 dfas de evolucin de otalgia y otorrea escasa a derecha. A la otoscopia se observa el tfmpano derecho eritematoso y abombado, con salida de pus escasa. El {ratamiento antibitico de eleccin es: aY;;t(moxicilina Cefadroxilo ~) Amoxic!Jina ms cido clavulnico d) Azitro.niicina e) Ciprfloxacino
/

;....

19)"Cul de lo~ siguientes microorganismos producir infecciones con mayor frecuencia en un nio 6n inmunodeficiencia por alteracin de fia produccin de inmunoglobulinas? a) Candida albicans b) Pseudomona aureginosa . e) Citomegalovirus d} Escherichia coli ..-)18\re ptococcus pneumoniae

~ ~ ~
/

J.

20) ta causa ms frecuente de hipotiroidismo congnito es: . )' Hipopituitarismo . teracic;mes de la conversin perifrica de T4 en T3 isgenesi~ tiroidea ) ficit de yodo e) Tiroiditis crnica

,.,
~ ~
~

'i
~

~
~

~
~
~
l .~, '.

~ ~

~ ~

~
~ ~ ~
~

"'
~

'~" "
~
~
~

"i
~

~
~
~
~

~
~

~
~

~
~
0
1

~.

~
~

NOMBRE:

3
4 5

~
~
~
RUT:

6 7 8

~
~ ~
~

FECHA:

00 00 00 00 00 00 00 00 00

00 000 000
000 000 000 000 000 000 000 000 000

000 000 000 000 000 000 000 000 000

o o o o o
D E

o o o o

o o

CURSO: RESPUESTAS A B C D E A
31 32

e
~
~

B C

A B C D E

~
~

~
~
~

00000 00000 3 00000 4 00000 S 00000 6 00000 7 00000 B 00000 9 00000 10 00000
2

00000 00000 33 00000 34 00000 35 00000 36 00000 37 00000 38 00000 39 00000 40 00000
41 42 43 44 45 46 47 48 49 50

00000 00000 63 00000 64 00000 65 00000 66 00000 67 00000 68 00000 69 00000 70 00000
61 62 71

~
~
~

~
~

fft'
~
~

00000 00000 13 00000 14 00000 15 00000 16 00000 17 00000 18 00000 19 00000 20 00000
12

11

00000 00000 00000 00000 00000 00000 00000 00000 00000 00000 00000 00000 00000 00000 00000 00000 00000 00000 00000 00000

7~
73 74 75 76

n
78 79 80

00000 00000 00000 00000 00000 00000 00000 00000 00000 00000 00000 00000 00000 00000 00000 00000 00000 00.000 00000 00000

(!'
~

fJ'

("'

(!"
~

00000 00000 23 00000 24 00000 25 00000 26 00000 27 00000 28 00000 29 00000 30 00000
21 22

51 52 53 54 55 56 57 58 59 60

81 82 83 84 85 86 87 88 89 90

~
~
~

~
~

~
-

Prueba 11 NEONATOLOGfA 1) Pul de las siguientes patologfas NO es ms frecuente en los recin nacidos prematuros? a) $psis neonatal . b ,.Ictericia . / Hemorragia periventricular\ / . ) E!Jlerocolitis necrotizante ~,/ _pardiopatas congnitas / 2) Cul de las siguientes opciones corresponde a una alternativa vlida para la alimentacin lctea de un latante de 8 meses, que no recibe pecho materno? ~che purita al 7,5%, ms azcar al 2,5%, ms aceite al 1,5% eche purita al 7 ,5%, ms azcar al 2,5%, ms cereal al 3% eche purita al 7 ,5%, ms azcar al 5%, ms cereal al 1 ,5% d) Leche purita al1 0%, ms azcar al 2,5%, ms aceite al 1,5% e) Leche purita al10o/o, ms azcar al5%, ms cereal al3%
,. 3) El recin nacido pequeo para la edad gestacional NO tiene mayor riesgo de: a) Poliglobulia /-:> .. b) Hipoglicemia .(::~Lt' e). Muerte _ .--- __ .. -- -- ... - -. . .. , ermedad de m_emtbrana..hlU~.?... --:.: ~ ~sfixia ne a1 ..

' / /

.~

"

!.f

4) Las .dos primeras causas de muerte neonatal son: a) ~fformaciones congnitas y asfixia neonatal 9> Asfixia y sepsis neonatal . \_//e) Problemas relacionados con la prematurez y asfixia neonatal A1yPfblemas relacionados con la prematurez y malformaciones congnitas e) Sepsis neonatal y muerte sbita

#''

5) Un ~edn nacido de trmino, de 27 dfa1; de vida, alimentado exclusivamente con pecho materno, presenta ictericia hasta las rodillas, asociada a coluria y acolla. El resto del examen fsico no aporta ~yor informacin. El diagnstico ms p.robable es: a) Ictericia fisiolgica b) Hemlisis e) Hipotiroidismo d) Ictericia por lactancia materna t:if Atresia biliar primaria

. 6) t6'ul de las siguientes asociaciones entre hallazgo en un recin nacido y caractersticas clnicas es JNCORRECTA? . ; /a) Bolsa serosangulnea - Sobrepasa las suturas/ \._/ b) Millium sebceo- Compromiso de la nari:c:v _,.. c~ritema txico --: Respeta las palmas y plantas v' _,;J)'t3ranuloma U!llbilical - Salida continua de liquido por el ombligo e) Mancha Sc:Jhiln -Ubicacin en la linea media de la cabeza

../"'

'V5

7) Un repn nacido de trmino, hijo de mndre diabtica, de 4 horas de vida presenta una ,9.-0.nv.ulsin._ Al exarfien trsico se apre~llQJ'I.e.acti~~. El diagnstico ms probable es: . a) ;ricefalopatia hipxico-isqumica ~JCiipoglicemia Hipocalcemia d) Hiponatremia e) Epilepsia

,..,
~ ~
8} Un recin nacido, de 29 semanas de gestacin, presenta rpido deterioro, asociado a compromiso de consciencia. Al examen fsico presenta abombamiento dEl la fontanela anterior y anisocoria. El diagnstico ms probable es: a). Meningitis neonatal .b) arlisis cerebral Encefalopatla hipxico-isqumica Hemo ia eriventric lar 'e) Hipocalcemia neonatal 9) Un recin nacido de 4 das de vida presenta rechazo alimentario y dificultades para regular la temperatura. En el examen flslco se aprecia en buenas condiciones generales, pero con cierta disminucin del tono muscular. La conducta ms adecuada es: a) Asegurar un ambiente temperado y administrar liquidas tibios, hasta recuperar la temperatura normal, sin nec~sidad de mayor estudio b).lhiciar cefadroxilo oral y enviar a domicilio explicando los signc1s de alarma ) Regular la temperatura y solicitar hemograma, hemocultivos, glicemia, calcemia, electrolitos y pruebas de /funcin renal, decidiendo el manejo segn hallazgos~ .. d) Iniciar soporte, solicitar hemograma, hemocultivos, glicemia, Ci3lcemla, electrolitos, pruebas de. funcin renal, gases venosos, radiografla de trax, puncin lumbar y exmenes de orina e iniciar una asociacin de rtaxon~ y vancomlcina endovenosa, lnmediiltamente ... nici~r- soporte, solicitar hemograma, hemoJtivos, glicemia, calcemla, electrolitos, pruebas de funcin . aygases venosos, radiografla de trax, puncin lumbar y exmenes de orina e iniciar una asociacin de /'taximo y ampicilina endovenosa, inmediatamente -..:~~ a) La radiografia de trax caracteristica, muestra un patrn demlleop_Jtn vid_~.~-~merililo . b) Es la causa ms frecuente de sindrome de dificultad respiratoria del recfn nacido pretrmino ...... e) Su manejo inicial requiere de antibiticos de amplio espectro';,. < ) d) Se asocia a neumotrax ms frecuente en Jos recin nacidos por cesrea

~
~

~
~

~
~
~

~
~

..

~
~

J~?~ Respecto a la taquipnea transitoria del recin nacido es verdadero que:

tfts

11} ,Qu situacin NO es fisiolgica en un recin nacido? a) Signo de Babinsky _b)..- Metrorragia \ ... ,.....-e) Cefalohematoma "-.-./ ... d) Hernia umbilical riebre 12) Los nuesos que con mayor frecuencia se lesionas durante el parto son: a) Papefales b)lrntales 9) Fmures
v~etavfculas

. \

.9) Costillas

13) Un recin nacido de 3 dfas de vida, nacido por cesrea y nlimentado con pecho materno exclusivo,
con, an;.eedente de sufrimiento fetal agudo, presenta decaimiento, actitud de dolor, distensin ab~i.nal, vmitos y deposiciones sanguinolentas en una oportunidad. El diagnstico ms probable
!

\ \. _.. "

;fennedad de Hirschsprung nterocolitis necrotizante } stenosis hipertrfica del pfloro d) Invaginacin intestinal e) lleo meconial

:'

~
14} Una mujer embarazada de 28 sem.anas, Rh negativo, se realiza test de Coombs indirecto como parte del control prenatal, el que resulta negativo. La conducta ms adecuada es: a) Realizar test de Coombs directo antes del parto y decidir conducta segn resultados )( b) Realizar nuevos test de Coombs indin~cto mensualmente y decidir conducta segn resultados ( ""'- ~ea !izar ecogra_fas doppler seriadas. de arteria cerebral media fetal y decidir conducta segn hallazgos ~dministrar inmunoglobulina anti-D (Rhogam) a la madre. y continuar controles con test de Coombs indirecto e) Solicitar pruebas de inmunizacin contra grupos attpicos. ya que las mujeres Rh negativo no tienen riesgo de hef!)lisis fetal por incompatibilidad dEl grupo Rh
/

~
h

/'

1~)'La imagen de doble burbuja es caracteristica de: \ /,tEi} Enterocolitis necrotizante \./ b) Enfermedad de Hirschsprung ~~ibrosis qustica ~tresia duodenal e) Estenosis hipertrfica del plloro

J!!-

~ ~ ~
~
~
.

16) Un recin nacido, con ant~cedente de liquido amnitico con meconio espeso, nace ciantico, sin m9Vmientos respiratorios y con marc;:~da hipotonra. Se inicia estimulacin y aporte de oxrgeno, pnnaneciendo en apnea, con FC cercana a 70x'. la conducta ms adecuada es: //a) Realizar desfibrilacin elctrica /~niciar masaje cardiaco 1 "ciar ventilacin a presin positiva niciar masaje cardiaco y ventilacin a presin positiva e} Administrar adrenalina

~ ~
~

e'!'
~
~

17) Un recin nacido de trmino, de 3 dlias de vida, presenta ictericia desde hace 24 horas, que actualmente llega a los muslos. Al nacnr pes 3-420 gramos, se alimenta adecuadamente y actualmente pesa 3.230 gramos. El examen fisico no aporta mayor informacin. La conducta ms ' ) adecuada es: - --- ~~ ~) ~spender la lactancia materna e iniciar frmula .-. ,..-b-}.--solicitar bilirrubinemia -....._,/.. ;?) Iniciar fototerapia ./ d) Realizar exanguineotransfusin e) Solicitar pruebas hepticas. prueba de Coombs directa, hemograma. y hemocultivos 18) Las infecciones connatales (TORCh1) son producidas principalmente por: a} Escherichia coli -ti) l'IH .. ,;ritomegalovirus JO~ ~lamidia e} Mycoplasma

~
~

\'--..///

~ \ ~ \ ...
~
~

19) Un recin nacido de 43 semanas de edad gestacional, con antecedente de sufrimiento fetal, durante el parto, presenta taquipnea y quejido a los pocos minutos de nacer. Al examen fsico se aprecia dificultad respiratoria y se auscultan crpitos y sibilanclas bilaterales. El diagnstico ms /probable es: /' a) Taquipnea transitoria __./ b_)>~nfermedad de membrana hialina ~ ndrome aspirativo meconial / d} Neumonia neonatal ! e) Hipertensin pulmonar persistente

A 1

../

~
~

~ ~
~
~

20t,ul de las siguientes alternativas c:orresponde a una contraindicacin absoluta de la lactancia m:iterna? \ /') Infeccin materna por hepatitis -8 ... -....._......../ b} Mastitis llnfangitica Consumo materno de cocaina 7 d) Prtesis mamari~ de silicona e) Getasdel-pez~n

_,
o
1

NOMBRE:

2
3

4
5

RUT:

6 7
8

FECHA:

9
K

00 ()00 oo <)OO 00 ()00 00 ()00 00 ()00 00 ()00 00 <)00 00 ()00 00 ()00 00 C)QO

000 000 000 000 000 000

o ooo o 000 o 000 o


000

~ !&)

o o o
o o o

,.,

o
ooooo ooooo
A B

CURSO:

1
RESPUESTAS A B C O E
1 2

~
~

A B
31 32 33 34 35 36 7 S 38 39 40

00000 00000 6 00000 7 00000 B 00000 9 00000 10 00000


5 11 12 13 14 15 16

~ 00000

ooooo

ooo~oo

00000 00000 00000 00000 00000 00000 00000 00000 00000 00000 00000

oo.ooo ooooo

D E
61 62 63 64 65 66 67 68 69 70

D E

~
~
~

00000 00000 00000 00000 00000 00000 00000 00000 00000 00000 00000

~
~

~
~

~
~

00000 000,00 000100

41 42 43 44 45 46 47 48 49 50

71

7~
73 74

~
~
~
~ ~ ~

OQQIQQ

00000 00000 17 OOC,OO 18


19 20

OOC,OO 00()00 OOC>OO

00000 '75 00000 00000 76 00000 00000 77 00000 00000 78 00000 00000 79 00000 00000 80 00000 00000 00000 00000 00000 00000 00000 00000 00000 00000 00000 00000 00000 83 00000 84 00000 85 00000 86 00000 87 00000 88 00.000 89 00000 90 00000
81 82

00000

00000

OOC>OO OOC>OO 23 OOC>OO 24 OOC>OO 25 00()00 26 00()00 27 00000 28 00()00 29 00()00 30 .00()00
22

21

~
~

51 52 53 54 55 56 57 58 59 60

~ ~ ~

~
~

~
~
~
1

~ ~

~
~ ~ ~
~
Prueba 16 GINECOLOG(A 1) Una paciente de 34 aos, con antecedente de SOP presenta metrorragia de 4 dias de evolucin, asociada lipotimias. Al examen fisico se c:onstata PA: 110/60 mmHg, FC: 100x', t 0 : 36,7C, palidez y frialdad de extremidades. A la especulosc:opfa se observa sangre saliendo por el OCE y el test de embarazo en orina resulta negativo. La conducta MS adecuada, adems de una adecuada reposicin de la volemia, es: ~oli,pitar beta-HCG plasmtica ~ ~iar anticonceptivos orales en dosis habituales e) Solicitar hemograma y perfil de hierro d) Enviar a domicilio, indicando control en CEISO de recurrencia de Jo sintomas Realizar legrado uterino 2) La etiologa de la endometriosis suele !Ser: ~ Anovulacin Hipoestrogenismo e) Embarazo
~trognica

,
1 >

(A

f'A
~

~~ C!t}
~

'.v)
f!'
.,.

~
f'!"
.~

/'

/,b~truccin al tracto de salida

~.,... . Jevolucin. Usted sospecha que la paciente est climatrica. Qu examen de sangre le permite
determinar esta situacin de mejor manetra? a) Estrgenos plasmticos b) Estrgenos urinarios e) Progesterona plasmtica .

~),Una paciente de 46 aos consulta por c)ligomenorrea, seguida de amenorrea de 4 meses de

\C

'!'

J'~SH plasmtica

~~Hplasmtica

" _., \(

f"

,. 4) Una paciente es diagnosticada de una neoplasia intraepitelial de alto grado (NIE-3). La conducta ms ~decuada es: a) ~'ntrolar con Papanicolaou mensualmente ~ b}'"Controlar con Papanicolaou cada 6 meses ').~,Realizar legrado bipsico Indicar conizacin cervical e) Indicar histerectomla total 5) Una n'a de 4 aos es traida por su madre, ya que presenta metrorragia. Al examen supeicial se cons~t escasa metrorragia, asociada a leucorrea de mal olor. El diagnstico ms probable es: a) Vylvovaginitis .in especifica , b),Metrorragia traumtica ~~enarqui~ ~uerpq. xtrao vaginal ) e) Abu~ sexual

lf!IA '
~

"'
~

fd

'

~
Ita\

,. ~
~

6) .Es contraindicacin del uso de anticonceptivos orales: Nuliparidad "-._/'b) Sndrome de ovario poliqulstico e) Antecedente familiar de cncer de endornetrio ~~nfeceden~!9 familiar de cncer de ovario Hepatitis aguda

?1

?'

~
~
~
~:;-

(!A
~

/ 7) Un\rhujer de 27 aos acude a control de rutina con gineclogo, quien palpa un tumor anexial izq~rdo. Se solicita ecograf(a transvagi nal, que revela un tumor anexial Izquierdo de 4,6 cm, de ~/ ecto quistico. La conducta ms adecuada es: niciar anticonceptivos orales y controlar Em 3 meses ' niciar tamoxifeno " e) Realizar biopsia por puncin d) Realizar tumorectomla, con biopsia intracperatoria e) Realizar anexectomla

~
~

~
~

~
~
~
Es ms frecuente en las mujeres postmenopjusicas El fa9tor de riesgo ms importante e~g..meftiparidad El- prolapso medio suele trata~eol11st~etomla ms fijacin de la cpula vaginal ~d(~cistocele es un tipo de prolapso~nterlr. mientras que el reetocele es un-tipo de prolapso posterior ''C.os sntomas ms frecuentes suelenser el dolor y la metrorraoia
~

8) a) b) e)

Respecto al prolapso genital es FALSO que:

~
~

9) Las dos causas ms frecuentes de infertilidad son: a) Masculina y endometriosis


nqvlacin y endometriosis .. ~sculina y anovulacin ~ Tuboperitoneal y masculina e) Endometriosis y tuboperitoneal

~ ~
~
4:)

b)

10) La preba de progesterona es muy til para identificar la~' metrorragias causadas por:
a) Hi13-ostrogenismo b) ~Iteraciones uterinas cY!'lteraciones hipotalmicas ~-Ciclos anovulatorios e) Malformaciones genitales

~
~

~
~
~
~

11) Una paciente de 34 aos se realiza PAP que es informado como muestra satisfactoria, con
a} ,Repetir PAPen un ao epetir PAP en 6 meses -~ -erivar a patologa cervical. para colposcopia /. d} Realizar cono fro e) Re~~_jzar histerectomia total

presencia de clulas con displasla moderada. La conducta ms adecuada es: /

~
~
~
~

y biopsia

12) Una mujer de 50 aos se realiza una mamografa, que es informada como Birrads 2. La conducta
decuada es:
""--/ alizar nueva mamografa en un ao ealizar nueva mamografia en 6 meses e) Solicitar ecografa mamaria d) Realizar biopsia mamaria con mamtomo e) Solicitar estr~genos y progesterona plasmticas .

~
~

~
~

x/1_3) Los anticonceptivos orales aumentan el riesgo de:


~r:~cer-de-mam.al
.
.

~
~

) cceree...c.uello.-uter~AQ
e) Neop~asia trofoblsti~9 .

b)~C~eer-d.e..erldomettiQ cf. Ghcer de ovario -...

~
~
~

14) J]na paciente consulta por dismenorrea e hipermenorrea de cerca de un ao de evolucin. Al exmen fsico destaca tero aumentado de tamao, como un ~~mbarazo de 1 Osemanas, sin embargo el t6~e embarazo es negativo. El diagnstico de sospecha es: /8) Endometriosis .

~ bJ.Proceso inflamatorio plvico

~Ad~

a) Hiperplasia endometrial
e) Cncer de endometrio

15) Una'paclente de 45 aos, consulta por leucorrea. A la espe1culscopia se aprecia secrecin blanco

. \...7'"

gris)'ea, de mal olor, con mnima inflamacin de la pared vagfinal. El tratamiento ms adecuado de la p~oga descrita es: ~~l fVIetronidazol oral
~) Doxiciclina oral e) Ceftriaxona intramuscular

,.

d) Fluconazol oral e) Nistatina tpica

"
~

~ A

A
;

16) Una mujer de 46 aos, multfpara de 4, doble cesarizada anterior, consulta por metrorragia autolil)1itada, durante el dfa 14 de su ciclo menstrual. Se solicita ecografra transvaginal que demuestra 3 mi9inas transmurales, el mayor de ellos de 3 cm. La conducta ms adecuada es: ~ealizar miomectomia.por via histeroscpica : Soliitar biopsia de endometrio Iniciar anticonceptivos orales ) Realizar histerectomfa total e) Realizar histerectomfa radical ms anexoctomfa bilateral

!!!lt\

17) Una mujer de 60 aos, asintomtica, presenta un tumor anexial derecho, ecogrfico, de 5,8 cm, de a$pecto qustico con algunas increscem:ias. La conducta ms adecuada es: a} Iniciar terapia de reemplazo hormonal con estrgenos solos 1 b) Iniciar terapia de reemplazo hormonal con estrgenos ms progesterona e) Realizar biopsia por puncin con aguja gruesa e~lizar tumorectomla con biopsia intraoperatoria alizar anexectomia con biopsia intraoperatoria 1

~ ('A

J; J!l

(!A.

~
~
~

' /' 8) Respecto al cncer de cuello uterino es verddero que: ~~e asocia a presencia de virus papiloma human~;-.s. e_r.otipos 6 y 11 ~uele estar precedido por lesiones intraepiteliales:--r .:-' e) Su factor pronstico ms importante es el tipo histQgo d) La etapificacin se realiza de manera quirrgica \. e) Suele tener sobrevidas muy corta~ -:., 1
/
.,. \.

(!!A

~ ~
('A

19) Una mujer consulta por disuria y leuc:orrea amarillenta. A la especuloscopfa se aprecia pared vagi~al indemne, con cuello uterino muy inflamado y con abundante secrecin purulenta. El dias;fnstico ms probable es: a)Vaginosis bacteriana /b) Candidiasis \ \ / e~ Tr.icomo~_lasis ,!_J- .Gonorrea / e) Cn<;ir de cuello uterino
/~

~
~

~
~-.

29(Qu tipo de terapia de reemplazo hormonal indicarla a una mujer perimenopuslca, .tfi~terectomizada, con marcados sfntoma.s vasomotores? \J~l.-Estrgenos orales (b) Estrgenos ms progestgenos orales e) Progestgenos orales d) Estrgenos tpicos e) Progestgenos tpicos

~
~

~
(~!A

~
~

(jFI:\

~
~

~
~
(/F!'

I'(UJ

,.,
000

NOMBRE:

2
3

00 000 000
00 000 00 00 00 00 00

00 000 000

4
5

o 00 000 000 o 00 000 000 o


000 000 000 000 000 000 000 000 000 000

o o

~
~
~

RUT:

7
8

o o o
o o o

~
~
~

FECHA:

9
K

~
~

"""'

CURSO:

1
A B C O E
1 2 3 4
5

RESPUESTAS A B C
31 32 33 34
35

O E

A 8

C O E

7 8 9 10

00()00 00()00 00()00 00<)00 00<)00 00<)00 00()00 00<)00 00<)00 00<)00

36 57 38 39 40

00000 61 00000 00000 62 00000 00000 63 00000 00000 ' 64 00000 00000 65 00000 00000 66 00000 00000 67 00000 00000 68 00000 00000 69 00000 00000 70 00000 00000 00000 00000 00000 00000 00000 00000 00000 00000 00000 7 ~ 00000 73 00000 74 00000 75 00000 76 00000 17 00000 78 00000 79 00000
71

~ ~
~
~
~

Al)

~
~

OOC>OO 12 00()00 13 00()00 4 1 00()00 15 00()00 16 00000 17 ' 00()00 18 001000 19 001000 20
oo~ooo
~ooooo ~ooooo

11

41 42
43

44
45
46

~
~

47 48

49

so
51

ooooo ooooo
so

~
~
~ ~

~ ~

00000 81 00000 00000 82 00000 53 00000 83 00000 M00000 54 00000 84 00000 ~ooooo 55 00000 85 00000 ~ooooo 56 00000 . 86 00000 ~ooooo 57 00000 . 87 00000 ~ooooo 58 00000 88 00.000 ~ooooo 59 00000 89 00000 ~ooooo 60 00000 90 00000

nooooo

52

~
~

~ ~

'

~
~

~
~ ~

~
Prueba 9 . NEUROLOGfA

!'A

1) La presentacin clnica ente.-del-sinclrome de Guillian Barr es: a) Pafa s1a esptica, con compromiso respiratorio b) Tetrapares1a flac1da, fluctuante, con matcatfo-cofllPI'Oll\lso tfe los prpados.y_o.culmtores e) .+etr.aparesia-fleiea;-febril, con s1gnos nietngeos-esbezados - -~ etra aresia con hipotonia y arreflexia, con compromiso de los neiVios faciales Tetraparesia progresiva, con au~encia ele compromlso sensitivo

2) Un paciente sufre un trauma lumbar, al caer de altura, presentando incontinencia urinaria. Al


ex9men destaca anes~esia en silla de montar de la zona perineal. La movilidad .,extremidades inferiores son normales.l~l sitio ms probable de la lesin es: /a) Asta anterior de la mdula b) Haz corticoespinal e) Haz espinotalmico d) Cauda equina __ r o n o medular
~)-Angiopatl

y sensibilidad de las

~
~
(itt\

/J

3) La causa ms frecuente de accidentes vasculares de tipo hemorrgico, es: iloidea b) Hipertensin arte ~~~~-~ rte.r:is.. cerebrales d~mbolias cardiacas Traumatismos encefalocraneano

'J
f!it'
~

4) Un paciente de 26 aos refiere cefalea recurrente, muy intensa y molesta, que compromete la zona retroocular, parietal y frontal y es de carcter pulstil. Refiere que los cuadros se hacen ms frecuentes cuando presenta sobrecarga de trabajo y cuando presenta el dolor, ste es exacerbado por los ruidos intensos y por la luz. El diagn1stico ms probable es: agraa /}>) Cefalea tensional s> Neuralgia del trigmino d) Cefalea en racimo e) Hipertensin endocraneana 5) ~aspecto al delirium es FALSO que: a) Suele acompaarse de ideas paranoideH tl) Junto con tratar la causa, de deben administrar antipsicticos Upicos en dosis bajas ~ agitacin se trata con benzodiazepims d) Lo ms caracterstico es la afectacin det la atencin e) El paciente debe usar sus lentes y atigi.fcmos, si Jos usa

~ ~
~
~
~

.\

,..
~
(fflt\

6) U,n ~aciente de 67 aos sufre una cafcla a nivel, al resbalarse en el piso hmedo, resultando con un golpe en la zona occipital, con una pequea herida de 1 cm, que sangra inicialmente, de fonna Gtolim ita da. Al examen ffsico se encuentra en buenas condiciones generales, con signos vitales normales, al igual que el examen neurolgico completo. la conducta ms adecuada es: \ a) / ; Enviar a domicilio con analgsico orales '-b)))olicltar radiografia de crneo ;.!f''SolicitarJAC de cerebro y crneo, sin contraste d) Solicitar.RMN de cerebro e) Hospializar, con analgsicos endovenosos y antibiticos profilcticos

~
~
(F'

sinhibicin y conductas desadaptatlvas- Demencia frintotemporal \ ......... idas frecuentes e incontinencia urinaria -Hidrocefalia normotensiva l..- _... e) Empeoramiento de Jos sfntomas con antipsicticos- Demen~iapor cuerpos de Lewy d)_.,Asociacin a hemipaesia- Demencia va:3cular .. , ___ /Alucinaciones visuales - Demencia senil

~l de las siguientes asociaciones entre caracterlsticas clnicas y pa~iogia es iNCORRECTA?

~
(ff?t\

~
~

,.,
~

~
~

8) Cful de las siguientes alteraciones es MENOS probable dn encontrar en un paciente con esclerosis lat,ral amiotrfica? a).1Fasciculaciones ) Atrofia muscular ) Hiperreflexia /: ~ipoestesla e) Disfagia 9) Un paciente de 67 aos, hipertenso y diabtico presenta de forma sbita una hemiparesia derecha, asociada a afasia, de 30 minutos de duracin. Se solicita TAC de cerebro que descarta hemorragias. Es manejado adecuadamente, con medidas de neuroprotecclln y afortunadamente, los srntomas ini~ian una recuperacin espontnea antes de comenzar el protocolo de trombolisis. Al segundo da ~st en excelentes condiciones. sin dficit neurolgico ni otras alteraciones del examen fisico. La /conducta ms adecuada es: a) Solicitar TAC de cerebro control b) Solicitar electroencefalograma SCilicitar Holter de arritmias, ecocardiograma y eco-doppler carotideo h) Iniciar aspirina e) Iniciar anticoagulacin oral a permanencia

~
~

~
~
~

~ ~
~

-'- /

~
~
~

., '-

10) Un paciente de 45 aos presenta cefalea holocrnea, de Inicio sbito, muy intensa, asociada vmitos. No refiere historia de cefaleas previas y la intensidad del dolor es 10/1 O. El examen fsico, 61uyendo el fondo de ojo es normal. La conducta ms adecUJada es: . Administrar paracetamol oral Administrar oxigeno al1 00% y triptanes subcutneos e) Administrar AINEs endovenosos ~~niciar analgesia endovenosa y solicitar puncin lumbar ?:..s-o~icitar TAC de cerebro y administrar analgsicos endovenosos lozapina Risperidona Carbamacepina ) Ergotamina e) Amitrip!iJina

~ l

A
..

.{Cul de los siguientes medicamentos produce parkinsonismo por drogas con ms frecuencia?

V;!

12) UJlpaciente presenta de forma brusca imposibilidad para ~vacar palabras. No es capaz de nombrar obj~fos, slo puede decir algunas palabras cortas, con muchct dificultad, sin embargo repite con ayor facilidad. Presenta slo mfnima dificultad de la comprensin. El resto del examen fislco no porta mayor informacin. El diagnstico ms probable es: ' -~ Infarto silviano derecho ccidente isqumico frontal izquierdo --lfarto postrolndico izqueirdo d) Isquemia aguda de troncoencfalo e) Infarto cerebeloso

13) Existen tratamientos profilcticos para la migraa, que disminuyen la f~cuencia y la intensidad de los episodios de jaqueca. Todos los medicamentos enumerados a continuacin pueden ser usados /, como tratamiento profilctico, EXCEPTO: ~f~ymatriptn / . / 'b) Propanolol .._/ g,Acido valproico . d) Flunarizina ~ e) A/ptilina .. / 14J'1Jn paciente presenta una hemianopsia homnima derechn de baja congruencia. La lesin se lcalizar probablemente en: ~) El nervio ptico derecho . 'V b) La clntilla ptica derecha e) El lbulo occipital izquierdo d). 6t lbulo occipital derecho .e(La cintilla ptica izquierda
/

~
15) U~_.paclente presenta ptosis derec:ha, asociada a miosis ipsllateral. No presenta alteraciones en la agudza visual, ni en la oculomotilidad. El diagnstico ms probable es: a)..ompresin del tercer nervio craneal derecho )i) Afectacin m. eta.blica del tercer neryio craneal derecho

" "

V~) Afe~-rr~-~t ne.111iofacial.~t~r~cho

~ ~
.

d) ~~~ral9l-_se~~m;\!=lria del trlgmino .. ~~indrome de Claude .sernard Horner

A~ems del ABC, lo primero que die be hacer frente a un paciente que ingresa convulsionando, es:
dministrar lorazepam endovenoso oner una gasa en la boca del paciente, para evitar que se muerda la lengua olicitar una resonancia magntica nuclear d} Administrar fenitoina por sonda nasogstrica e} Administrar diazepam intramuscular
1

~
.

ff$\

17) U,n/paciente de 68 aos presenta debilidad progresiva de la mano Izquierda, de 7 dlas de evolucin. Refire adems cefalea matinal, asociada a vmitos y present una convulsin tnicoclnica en una brtunidad. El diagnstico ms probable es: Accidente vascular enceflico !acunar Hipertensin endocraneana idioptica ) Accidente vascular enceflico de tipo hemorrgico d) Hematoma subdural crnico

(!A
~

r.ilmor cerebral
1.8)

-~r)H Hiperte~aqulcardia y p~~acral"mstest8creTas eXfreml'dads inferiores


e) Hipoten~ ..-taquicardia-y-par.aplejia-alSOGiada-a..ane.stesia.d.e Jas_extr.emid_ad~~jnferiores d) Hipertnsin,.bradlcardla y tetraplejia, asociada a anestesia de las 1.~~r..e.midades ............. e) HipoteRsip_,-taquicat:dia-y-tetr.aplejia-a!3ociada-a-anestesiade las-extremidades inferiores
.
-....., .............. ~-~--~-'
.

~ipotensin, l:[suiiGafdi~plejia, asociada a anestesia de las 4 extremidad~~. ______ ..

L{~eccin medular completa a nivl!l de C5 produce:

~
~
~

(iF1'
~

Vt
l"' 1

~
~

1~ Los agentes etiolgicos ms frecuentes de las meningoencefalitis virales y las meningitis bacterianas agudas, son respectivamente: la) Virus herpes y meningococo b) Virus herpes y neumococo Enterovirus y neumococo ) Enterovirus y meningococo e) Enterovirus y estafilococo areo

~
~

20) Identifique la asociacin correcta entre rarz nyrviosa afectada y cuadro clrnico: a) C5- Afectacin de la sensibilidad del ~~lgar .. <. b) C7 -Afectacin del reflejo biclpital .2( LS - Afectacin del reflejo aquiliano /1...4 -Afectacin del reflejo rotuliano2"' ~ /81 -Afectacin de la sensibilidad de la ,piel sobre el malolo interno

(!li:\

~
~

'

(11$'

~
(f!\

~
~

~
~
~

~
~

o
1

NOMBRE:

2 3

RUT:

7
8

FECHA:

9
K

00 00 00 00 C)QQ

00 00 00

00

00 ()00 000 00 ()00 000


()00

()00 ()00 ()00 ()00 <)00 C>OO

000

000
000

000

000

o 000 o 000 o 000 o o


A B

o o

o o o

CURSO: RESPUESTAS A B e
1
2
3

O E
31 32
33

e O E
61 62

A B

e O E

ooooo
00000 00000 00000 000100
00000

00000 00000

4
5
6

34
35 36 57
38
39

7
8
9

ooc,oo
OOC,OO

,o
11

40

00000 00000 00000 00000 00000 00000

00000 00000 00000 00000

00000 00000 63 00000 64 00000


65

66 67
68

69 70

00000 00000 00000 00000 00000 00000

OOC>OO OOC>OO 13 OOC>OO 14 00()00 15 00()00 16 00()00 17 00()00 18 00()00 19 00()00 20 "00<)00
12

00000 00000 43 00000 44 00000 45 00000 46 00000 4 ~ 00000 48 00000 49 00000 so 00000
41

71

42

7~
73
74

75 76

00000

00000 00000 00000 00000 00000 00000 00000 00000 00000

~ ~ ~
~ ~ ~ ~

n
78 79 80

..,
~

00<)00 22 00()00 23 00()00 24 00()00 25 00(000 26 00000 27 00000 28 00000 29 00000 30 00000
21

00000 52 00000 53 00000 54 00000 55 . 00000 56 00000 57 00000 58 00000 59 00000 60 00000
51

81 82 83 84 85
86

87 88
89

90

00000 00000 00000 00000 00000 00000 00000 00.000 00000 00000

~ ~
~

~
~

~
~

~
~

~ ~

"

~ ~

~)luna paciente un soplo carotideo derecho y febricula ocasional ydebaja de peso.radiales.examen fisico de 26 alios presenta astenia, En su slo se asculta una clara aslmetrfa los pulsos El diagnstico
ms probable es: a) Pelviespndiloartropatia con compromiso arterial b) Arteritis de clulas gigantes e) Enfermedad de Wegener ~rteritis de Takayasu e) Lupus eritematoso sistmico .

*'

7) CuJ.de las siguientes asociacic>nes entre marcador inmunolgico y patologia es correcta? a) Higoomplementemia -Artritis reurnatoide b)Jctol:-r.e~:~matolcle.o- spondilitis.anquilosante.. ., )l_) Anticuerpos anticitoplasma de neutrfilo en patrn citoplasfu,~tico- micropoliangeitis (PAM) \../ ~) Anticuerpos anticentrmero - Dermatomiositis ~ ?yAntic~erpos anti-ribonucleoprotelna - Enfermedad mixta del tejido conectivo

~ ~
~

~~picondilitis fl-J>) Esguince

8) La.. causa ms frecuente de codo doloroso es:

~~)Artrosis

~
~
~

d) Atrapamiento cubital e) Tendinitis tricipital 9) Un p'aciente de 68 aos, diabtico e hipertenso, bien controlado, presenta dolor en la cadera izquirda, desde hace algunos mesE!S, lo que le dificulta la marcha. Al examen fsico presenta dolor a la ..flexin y abduccin extremas de l.a cadera. Se solicita radiografia de cadera que demuestra ,~disminucin del espacio articular y un osteofito acetabular. La conducta MS adecuada es: ~ a) Solicitar anticuerpos antinucleares y factor reumatofdeo glniciar paracetamol e) Iniciar metotrexato d) Iniciar prednisona e) lnicir diclofenaco Enfermedad de Behcet Sarcoidosis e) Sfndrome de CREST d) Dermatomiositis frSJndrome de Sjorgren

~
~
~

"'

{~1 test de Schirmer positivo es Cclracteristico de:

r'

~ ~
~

"'
~

11) Una paciente de 45 aos presenta1 dolor de larga data en el hombro derecho, que aumenta con la actividad y que en los ltimos 10 dias; se ha hecho mucho ms intenso, al punto que le impide la movilizac.in. Al examen fsico est en buenas condiciones, con signos vitales normales y presenta
intenso dolor tanto a la movilizacin activa, como a la movilizacin pasiva del hombro, sin signos inflamatorios. El diagnstico ms probable es: del manguito de rotador ...- c)_,Artrosis - - -........., .Y~).1 Capsulitis adhesiva\ ~ 1enCJtnitis-bicipital..-a)~ursitis ~""!"endinitis

('A
~

~
~
(fA

12) Una paciente presenta artralgias, c:on febricula ocasional. Se solicita hemograma que demuestra
"'matocrito: 35%, VCM: 87, HCM: 32, blancos: 3.900, linfocitos: 25%, neutrfilos: 68%, monocitos: 7%, plaquetas: 53.000. El diagnstico ms probable es: a) Arteritis de la temporal b) Granulomatosis de Wegener e) Crioglobulinemia ;;t)"t.upus eritematoso sistmico e) Poliangeltis microscpica

~
~
~

'v

.'

~
(lA ~

~
~

fegunta recuperativa lnfectologra Contstela en la CASILLA N 30 Un paciente esquizofrnico, en trata~iento con clozapina, presenta. fiebre y decaimiento. El examen fsico no aporta mayor informacin. Se solicita un hemograma que muestra hematocrlto: 37%, plaquetas: 237.000, blancos: 2.900, linfocitos: 80%, Linfocitos: 7%, monocitos: 13%. La conducta ms adecuada, adems de suspender la clozapina, es: a}j)olicitar estudio para VIH y realizar exmenes de laboratorio en busca de un foco infeccioso )f[ !niciar ceftazidina + gentamicina y realizar exmenes de laboratorio en busca de un foco infeccioso ' e) Iniciar cefazollna + metronldazol y realizar exmenes qe laboratorio en busca de un foco infeccioso d) Iniciar ceftriaxona + ampicilina y relizar exmenes de laboratorio en busca de un foco infeccioso e) Iniciar ciprofloxacino y vancomicina y realizar exmenes de laboratorio en busca de un foco infeccioso Prueba 8 REUMATOLOGA

f~nmeno de Raynaud es caracterrstico de la siguiente patologia: sclerodermia " :,t> ermatomiositis '-...._/ e) Slndrome de Sjorgren d) Artritis reumatoide e) Artritis psoritica
2) Una paciente de 40 aos presenta dolor en las manos desde ha,:e 3 meses. Refiere marcada rigidez articular, que dura cerca de una hora. Al examen fisico se aprecia artritis de ambas mufiecas y de varia~ .rticulaciones metacarpofalngicas e lntealngicas proximales, de manera bilateral. El tra~amlento de eleccin de la patolog[a descrita es: a}"Paracetamol ,/b) Prednisona e) Ciclofosfamida + corticoides d) Alopurinol ms AlNEs ~~etotrexato

.V/

3) Un pifio de 7 aos presenta cojera de 3 semanas de evolucin, ::on mnimo dolor. Al examen fisico se apfecia dolor a la abduccin mxima de la cadera izquierda, asociado a leve atrofia del cudriceps iz~ierdo. El diagnstico ms probable es: p) Artritis sptica ./ b) Sinovitis transitoria '-..,./ e) Epifisiolisis de la cabeza femoral ..eff. Enfermedad de Perthes /e) Slndrome de Osgood Schlater 4) Ur,r paciente de 65 aos presenta astenia y debilidad muscular, especialmente al levantar los brazos y ' " la musculatura proximal de las extremidades inferiores. Tamlbln refiere que presenta cefalea frecuente y dolor al masticar por mucho tiempo, lo que lo obliga a comer lento. En sus exmenes estaca anemia normoc[cita normocrmica, con VHS: 97 mm/h. El diagnstico ms probable es: ) Lupus . b) P91imiositis ~}4;rteri~is de clulas gigantes d) Pojingetis microscpica anarteriris nodosa .... -. . Cul de las sigulente"s no J~ una manifestacin extraartlcular de la artritis reumatoide? Glomerulonefritis \.__.J b) Miocarditis e) Mononeuritis mltiple d) Epiescleritis e)' Derrame pleural
./
\

13) Cul de las siguientes zonas son MENOS frecuentemente afectadas por artrosis? a),MU~ -./._ b)""Artteuaciones interfalngicas proximales 1 ' e) Articulaciones interfalngicas distales d) Rodillas /Columna vertebral

~
~

'J.

14) Una paciente de 40 aos, con antecedc:tnte de colon irritable, presenta fatigabilidad y dolores generalizados. En su examen fsico presenta dolor a la palpacin de los msculos del cuello, dorso, 'Jta1nterna de las rodillas y cara lateral da los codos. El diagnstico ms probable es: Polimiositis , ,.blfJ.[rQiniaTQia ) ' e) Polimialgia reumtica d) Hipotoroidismo e) Lupus

~ !\.. (!" 1 '

15) Cul de las siguientes asociaciones, entre marcador inmunolgico y patologa, es INCORRECTA: a) Anticuerpos anti-nuclolo - Esclerodermia )(Anticuerpos anti-DNA de una hebra - Lupus por gr6gas 9_ Anticuerpos anti-Ro - Sindrome de Sjorgmh._// (g)> An~icuemos .anti:citoplasma de neutrfilo - Arteritis de la temporal e) An icuerpos anti-protefna P -Psicosis lpica
r

\ _. \/

16) Un hombre de 67 aos presenta dolor intenso y aumento de volumen del codo izquierdo, de 1 dfa de evplucin. Al examen fsico se aprecia aumento de volumen, con derrame articular, eritema y nto de la temperatura local. La conducta ms adecuada es: ealizar artrocentesis y solicitar estudio do liquido articular ) niciar AINES y colchicina e) Solicitar hemograma y hemocultivos d) Solicitar ra~iografla de codo izquierdo e) Solicit~.r.citigrafia sea

17) Jrul de las siguientes alteraciones NO suele aparecer en los pacientes con artritis reumatoide?
a),.Mano en rfaga . /b') Dedos en cuello de cisne \,.,_,-" e) Dedos en sastre ~dulos de Heberden ./) Ndulos reumatoldeos

18)
1

~,Qu tratamiento debe d~,gota, confirmado con el

indicar ante un paciente de 57 aos que cursa actualmente con un ataque estudio pertimmte?

/ ~1NEs

a) Alopurinol d) Alopurinol y AINEs e) Alopurino, conchicina y AINEs

/j Antibiticos y Al NEs

(IIA

~
('$'

<"'
~

19) ~.h paciente de 32 aos presenta una rinosinusitis crnica de dificil manejo, asociado a epistaxis fre6uente. En el ltimo tiempo se agrega eompromiso del estado general, fiebre, hematuria y oliguria. ,E diagnstico de sospecha es: 1 a) Prpura de Schoenlein-Henoch ,))(3ranulomatosis de Wegener .- e) Vasculitis de Churg-Strauss d) Lupus sistmico eritematoso e) Panarteritis nodosa

<"'
~
~
(9:1..

<""
~

~ ~
~

/
20) .na paciente de 20 aftos presenta malestar general, cardal de cabello y lesiones faciales er.itematosas. Al examen fsico se aprecia eritema en los pmulos y en el dorso de la nariz y algunas .(ilceras en el paladar duro y en las mejillas. El diagnstico m;is probable es: _/ a) Dermatomiositis - A.Vasculitis de vaso pequeo Lupus d) Enfermedad de Behcet e) Esclerodermia

,
~
~

~ ~
~

""

~
~

~
~
~

~
~ ~

~
~

~
~

~
~
~

A)
~
~

~
~

.-,

-,

""' ~

(!"
(A,

o
1

~
~
~
(lA
NOMBRE:

2
3 4
S

~ ~
~
~

RUT:

6 7 8

FECHA:

9
K

00 00 00 00 00 00 00 00 00

00 000 000
000 000 000 000 000 000 000 000 000

000 000 000 000 000 000 000

000

000

o
o o o

o o o o

"'
~

o o
O E

CURSO: RESPUESTAS A B C O E
2

e
~ f!iJ'
(FA
~

~
1

A B C
31 32 33 34 35 36

A B C D E

f!"
(lA

~
(J\

00000 00000 3 00000 4 00000 S 00000 S 00000 7 00000 8 00000 9 00000 10 00000 00000 00000 13 00000 14 00000 15 00000 16 00000 17 00000 18 00000 19 00000 20 00000
11 12

~7
38 39 40

00000 00000 00000 00000 00000 00000 00000 00000 00000 00000

00000 00000 63 00000 64 00000 65 00000 66 00000 67 00000 SB 00000 69 00000 70 00000
61 62

~.
~
~
.

'

~
~

00000 00000 43 00000 44 00000 45 00000 46 00000 47 00000 48 00000 49 00000 so 00000
41 42 51 52 53 54 55 56 57 58 59 60

00000 00000 73 00000 74 00000 75 00000 76 00000 77 00000 78 00000 79 00000 80 00000.
71 72 82

~
('A
(!!P\
~

(lA

f!"

00000 22 00000 23 00000 24 00000 25 00000 26 00000 27 00000 28 00000 29 00000 30 00000
21

00000 00000 00000 00000 00000 00000 00000 00000 00000 00000

00000 00000 83 00000 84 00000 85 00000 86 00000 87 00000 88 00.000 89 00000 90 00000
81

~
~ ~

~
~ _/
-

'7
Prueba 10 HEMATOLOGiA

'

\'~\1 .) \

~l\"-'

'
1) Un paciente de 57 aos, diabtico, hipertenso, con antecedente de una fibrilacin auricular crnica, en tratamiento con acenocumarol y betabloqueo, presenta fatigabilidad y palidez. El examen ffsico no aporta mayor Informacin. Se solicita1n exmenes que demuestran un electrocardiograma en fibrilacin auricular a 64 latidos, por minuto, sin otras alteraciones y un hemograma, con plaquetas y leuc5)citos normales, pero con hematocrito: 24%, Hemoglobina: 8,1 g/dl, VCM: 69 ft, HCM: 24 pg. El frfltis sangufneo presenta anisocitosi:s e hlpocromia. El diagnstico ms probable es: .afAnemia ferropnica , /fb) Anemia de enfermedades crnicas "-"' e) Anemia por dficit de folatos . d) Anemia por hipotiroidismo e) Anemia hemolitica

"

e
~
~

2) Un paoiente de 36 aos presenta aUlmento de volumen inguinal, de 4 meses de evolucin, asociado peso y fiebre recurrente. Al uxamen ffsico se aprecia algo plido y presenta varias a baja a~dpatras inguinales derechas, algunas de ellas confluentes, siendo la mayor de casi 4 centmetros. El azo se palpa de tamao normal. El diagnstico ms probable es: , ? Leucemia linftica aguda \_/ b) Leucemia linftica crnica ~:leucemia mielide aguda /:~ Linfoma _ .. e) Adenop~,tis reactivas 3) Un piri'o de 10 meses, asintomtico, alimentado con leche materna y una comida, presenta anemia co9 tlemoglobina de 9,2 g/di, sin alteracin de las otras series de clulas sanguneas. La conducta ms jidecuada es: '---/a) Solicitar biopsia de mdula sea b) Solicitar perfil de hierro, niveles de vitamina 812 y folatos a.Y'-1niciar tratamiento con hierro oral /d) Solicitar prueba de Coombs e) Solicitar pruebas de screening para talc1semias 4) Un paciente de 16 aos consulta por fiebre, compromiso del estado general de 7 das de evolucin, al que se le agreg dolor farfngeo en la!~ ltimas horas. Al examen se encuentra febril, y se aprecia plido. la faringe est eritematosa, con aumento de volumen amigdalino, mayor a derecha, sin exudado. Se observan varias petequias en el tronco y extremidades y se palpa esplenomegalia ~erada. El diagnstico ms probable es: ~ ~?nonucleosis infecciosa ) Mielodisplasia Qtbeucemia aguda f[ecemra m1eloide crnica e) Linfoma

p'

~
~

~
~

~
~

~ ~ ~

'\/,.
/\

~(
~
~

~
~
('F\

'v

~
~ ~
~

5) Un paciente de 67 aos, con antecedE}nte de una limitacin crnica del flujo areo, producto de una tuberculosis en su juventud, que trata Ct>n broncodilatadores, de manera Irregular. Actualmente presenta. disnea de pequeos esfuerzos .. Acude a control con un hemograma que demuestra hema__tdrito: 60%, hemoglobina: 19,6 g/dl, plaquetas: ~0.000 por mm3, blancos: 7.800 por mm3, con frp{ula diferencial normal. El diagnstic:o ms probabT es: l}Hemograma normal / ~) Anemia por enfermedades crnicas / e) Policitemia vera d)~Trombocitosis esencial rtlJPoliglobulia P.Q_f--hipexemia.
'
~.

:'. ~

.. _.
.')

,) ~

'.

~
~

A
r'

6) Un riio de 7 aos consulta por fiebre, dolores seos y males.tar general. Al examen se observan pet'quias en las extremidades inferiores, sin otros hallazgos. Se solicita hemograma que demuestra: o: 26%, plaquetas: 36.000, blancos: 2.400, con 90% de linfocitos y 1O% de neutrfilos. La conducta s adecuada es: - . Solicitar lgM VGA e lgM CMV Solicitar pruebas de agregacin plaquetaria e) Solicitar tiempo de sangrfa, TP y TTPA d) Solicitar perfil de fierro e niveles plasmticos de vitamina 812 ~'Solicitar biopsia de mdula sea 7) El prpura trombocitopnico inmune se caracteriza por todo lo siguiente EXCEPTO: a) Aus.encia de esplenomegalia b) ~efequias Hemorragias de las mucosas . ~~argamiento del tiempo de sangri~, roliadenopatias . 8) Una paciente cursando una septicemia por E. coli, en contexto de una plelonefrltis complicada, presenta Jlematoquezia en una oportunidad y aparicin de equmosis en las extremidades inferiores. En su~.e-:Xmenes destacan plaquetas de 40.000 por mm3, alarnamiento del TTPA y fibrinogenemla de BOmg/cll. La conducta ms adecuada para su trastorno de la hemostasis es: /nticoagular con heparina endovenosa ) Iniciar aspirina ~ pdministrar plasma fresco congelado L' -~\ d) Administrar crioprecipitado _,.. e) Realizar transfusi_n de plaquetas 9) Un paciente de 67 aos consulta por astenia y palidez. Se scJiicita un cell-dyn que demuestra una aneqlia moderada, normocftica. Se solicita perfil de hierro que constata una ferremia disminuida, transferrina muy aumentada, con bajo fndice de saturacin y ferritina muy disminuida. La conducta ' adecuada es: licitar endoscopfa digestiva alta e iniciar sulfato de fierro por vfa oral b) Diagnosticar anemia por enfermedades crnicas y solicitar exmenes generales, para determinar la causa e) Repetir el perfil de hierro d) Iniciar vitamina 812 por vla parenteral e) Iniciar hierro oral y controlar con un hemograma en 3 meses 10) Un paziente de 14 aos consulta por fiebre y odinofagia de 3 dfas de evolucin. Al examen presen~ un tinte subictrico de la piel, se. palpan varias adenopatias cervicales, y se aprecia _cierto grad9"ile esplenomegalia. Se solicitan un hemograma que demuestra hemtocrito: 46%, plaquetas: 27~~oo x mm3, blancos: 16.700 x mm3, con 78% de linfocitos, algunos de ellos con atiplas, correspondientes a aumento del citoplasma. El diagnstico ms probable es: ) Leucemia linftica crnica ) Leucemia linfoide aguda ) Mielodisplasia ~ononucleosis infecciosa /e) Linfoma de Hodgkin
!!~~:~~'~?inacin de anemia, hipercalcemia, elevacin de la creatinina y dolores seos, debe hacer
a) l:!~f9ma

/ -~--./

'

toma ficit de vitamina 812 ) H pervitaminosis D e) Malaabsorcin intestinal

-~

~ ~
~

12) Un paciente de 20 aflos presenta tendencia a las equimosis y suele tener sangramiento gingival cuando se cepilla los dientes, desde su niez. Refiere adems que cuando se hace heridas, suelen sangrar por varios minutos. Al examen slo se observan algunas petequias. Qu examen estar alter,do con mayor probabilidad? a) Uempo de protrombina 1empo de sangrla empo de tomboplastina parcial activadcl ecuento de plaquetas e) Fibrinogenemia

~ ~

~ ~

e e
(tiA

13) Un paciente con antecedente de un flctor V de Leyden presenta sbitamente disnea y dolor torcico, con tope inspiratorio a derecha. No presenta compromisos hemdinmico ni desaturacin import~mte. Se solicita un angioTAC que es compatible con un tromboembolismo pulmonar derecho. Se ~ide Iniciar anticoagulacin con he>arina no fraccionada por vra endovenosa. Qu examen debe realizar para controlar un adecuado nivel de anticoagulacin? a{Tiempo de protrombina (TP) e INR , t) Tiempo de trombina (TI) "-../ A,;fe'mpo de tromboplastina parcial activada (TIPA) d) Tiempo de sangrfa e) No requiere control con exmenes \ '-/ ' 14) Los linfomas suelen derivarse de las siguientes alteraciones genticas: a) Mutaciones autosmlcas dominantes -;-erMutaciones autosmicas recesivas e) Mutaciones ligadas al sexo !fi.Ir_anslocaciones cromsmicas -........ e) Aneuploidlas . - ~-....._ ..... ... 15) Un hombre de 66 aos, consulta por astenia. El examen ffsico demuestra palidez moderada y se palpan adenopatas pequeas generali~~das. El hemograma presenta hematocrito: 27%; Hb: 8,8; blancos: 36.000 por mm3; linfocit~~~i neutrfilos: 7%; monocitos: 1%; basfilos: O%; eosinfilos: O%; plaquetas: 85.000. El diagnstic.o m:s probable es: a)JJnfoma ) Leucemia mieloide crnica )(Leucemia linftica crnica d) Mieloma e) Mielodisplasia

~
~

~
~

~
~

~
(~!'

~
~

~
~

~
~

~
~

16) Un paciente de 24 aos, con enfermedad celiaca sin un adecuado cumplimiento de la dieta y depresin, en tratamiento antidepresivo, que usa de forma irregular, consulta por marcada astenia y disnea de mnimos esfuerzos. Al examen ffsico se aprecia muy plido, taquicrdico, con un pulso amplio. Se ausculta un soplo sistlico eyoctivo, de caractersticas funcionales. Se solicitan exmenes q4-e/son compatibles con una anemia por dficit de vitamina 812, con una hemoglobina actual de 5,8 g{dl. La conducta ms adecuada, de entre! las siguientes, es: /a) Enviar a domicilio con vitamina 812 va oral en dosis altas ./ b) Enviar a domicilio, insistiendo en el cumplimiento de la dieta \ '-._/ e) Hospitalizar y administrar vitamina 812 por vfa parenteral ~ospitalizar y transfundir glbulos rojos ,~)Hospitalizar e indicar dieta alta en folatos, vitamina 812 y hierro, asegurndose de que no ingiera ms gluten/"
f3J_Hiperbilirrubir-~emiadepraoiinfodifecto

~
(~!"

1~){~ anemia hemoHtica inmune se c~_rae:teriza por:

~ \_ / e) Frotis con dacrlocitos '_/ ~umento de los reticulocitos ~ e) Cuerpos de Howeii-Jolly


~
~

/b) Elevacin de la haptoglobina

~
~

Aj

~
~

~
18) Paciente de 50 aos, consulta por dolor en hipocondrio l~tquierdo y fiebre recurrente. Al examen destaca esplenomegalla importante. El hemograma muestra Hcto: 39%, Hb: 13mg/dl blancos: 43.000, con 95% de neutrfilos, 24% de baclliformes, 5% de promielccitos y 1% de blastos; r.ecuento de plaquetas: 460.000. El diagnstico ms probable es: , 1 ~} Leucemia mieloide crnica -.......;-......... "'~ucemia mieloide aguda / ' /.e} Mononucleosis infecciosa d) Fiebre tifofdea e) Mielodiplasia de las siguientes a~mias no suele ser macrocftica? Por consumo de alcohol V Por hipotiroidismo ~or mielodisplasias ./:~ ~or enfermedades crnicas e) P?r dficit de folatos
~Cul

~
~ ~

~ ~

,_,
~

~ ~
~

20

ul de los siguientes cuadros es ms sugerente de unu hemofilia? aciente de sexo mas~o y 16 aos, que ha sufrido varias h13martrosis de rodillas y otras articulaciones, e manera espontnea b) Paciente de sexo masculino y 14 aos, que presenta epistaxis frecuentes, asociado a aparicin de petequias y equimosis con pequeos golpes. .e} P~ieflt menino de 13 aos, que presenta hematuria y artralgias, asociado a un prpura de l;I&Zia~----extremidades inferiores y e d) Pacie~sexe-feiT)ertifle;-de-lBaf\os, eJI:I&desde su infancia ha prensentado tendencia a presentar equimosis, sangra ms de lo normal en procedimientos dentales y tiene reglas muy abundantes. e) Paciente de sexo masculino, de 5 aos, que presenta un prpura petequial generalizado, de 3 das de evolucin, que empez una semana despus de una infeccin respiratoria alta.

~
~

~
~
~
~

~
~ ~
~

~
~

~
~

~
~

e:
~

~~
~

o
1

NOMBRE:

2 3 4

~ ~
~
~

RUT:

e
FECHA:

6 7

8
9
K

00 00 00 00 00 00 00

00 000 000 o 00 000 000 o 00 000 000 o


000 000 000 000 000 000 000 000 000 000 000 000 000 000

o o o o o o o
O

~ ~

CURSO: RESPUESTAS A B C D E
1
2 3 4 5 6 7

e
~

A
31

B C

E
61 62 63
64

A B C O E

~
~

~
("
~

8
9 10

00000 00000 00000 00000 00000 00000 00000 00000 00000 00000 00000 00000 00000 00000 00000 00000 00000 00000 00000

00000

00000 00000 33 00000 34 00000 35 00000 36 00000 37 00000 38 00000 39 00000 40 00000
32

65 66 67 68 69 70

~
~

00000 00000 00000 00000 00000 00000 00000 00000 00000 00000 00000 00000 00000 00000 00000 00000 00000 00000 00000

11
12 13 14 15 16 17
18

~ ~
(!lA

~
~

19 20

~
~

00000 00000 43 00000 44 00000 45 00000 46 00000 47 00000 48 00000 49 00000 50 00000
41 42 51 52 53
54

71
7

73 74 75 76

nooooo
78 79 80 81 82 83 84 85 86 87 88 89 90

~
~
~

('i;
~
~
~

00000 00000 23 00000 24 00000 25 00000 26 00000 27 00000 28 00000 29 00000 30 00000
21

22

55 56 57 58 59 60

00000

00000 00000 00000 00000 00000 00000 00000

00000 00000

00.000

00000 00000 00000 00000 00000 00000 00000

00000 00000

~
~

~
~-.

~
~

f!'

Prueba 17 TRAUMATOLOG(A y ANESTESIA

~ ~ ~
~

7ptesenta ~n mayor riesgo. de nusoas postoperatorias, todos los siguientes, EXCEPTO: ~) .xexo masculino . /No fumador..._.... . . . Uso de opiceos en la ciwgfa / d) Historia de nuseas en clrugias previas e) Historia de nuseas ante oloreS: desagradables o en carreteras con muchas curvas

~
~

<!'
~

e
~

2) Un paciente de 45 aos sufre cada do altura, cayendo sobre la extremidad superior derecha y resultando con fractura del tercio medio de la difisis humeral. Qu estructura se afectar con mayor procabilidad? ~)'Nervio axilar \ /b) Nervio mediano ~ ~ervio radial Nervio cubital e) Nervio musculocutneo

/a)

3) Por qu mecanismo se produce la mayor parte de las fracturas de cadera?


Caldas a nivel / ) Caldas de .altura . \ _ / e) Rotacir:J forzada d) Fractur espontnea durante la marcha normal e) Accid~ntes de trnsito
o ,/

f!'
~

~ (!A
~

f'A
~

Cul de las siguientes asoci~ciones es incorrecta? strofia simptico refleja - Di~fninucin de los pulsos distales // b) Embolia grasa - Rash pet@Sf'ial e) Embolia grasa - Edema pulmonar no cardiog~!}.ia _/ d) Distrofia simptico refleja - Cambios en la piefy vellos di_tales e) Slndrome compartimenta! - Dolor a la movilizacin distal 5) Cul de las siguientes. histrlas clnicas es MS sugerente de una cefalea postpu"ncin? a) Cef~lea intensa, de ini~o'sbito, luego de una endarterectomia carotdea, que fue realizada bajo anestesia gen~rl, con drogas endov~~-,-~~ b~Cefalea intensa, mayor~ el decbito, asociada a vmitos, que inicia luego de una histerectomia radical, _Jalizada bajo anestesia epi ural ~ ,Pfeefalea intensa, que aumenta al sentarse o al pararse, que inicia a 24 horas despus de una osetosintesis /de tibia, realizada bajo anestesia raquidea d) Cefalea intensa, que aumenta con el deGbito y que est asociada a dificultades para ver y desorientacin, que aparece luego de una cistoscopia, realizada bajo anestesia espinal e) Cefal(jla.intensa, que aumenta al sentarse o al pararse y que inicia algunas horas despus de de un parto vagin~l;'realizado bajo anestesia peridural 6)/i,Cul de las siguientes 'es un signo de mayor gravedad ante un traumatismo craneal? Herida de cuero cabelludo . J~alida de liquido transparente por un oiclo , e) Hematoma de cuero cabelludo en la frente d) Convulsin de 30 segundos inmediatamr3nte despus del golpe e) Desorientacin de 4 minutos de duracin, despus del golpe, seguido de amnesia de esos momentos

~
~

~ ~ ~

~
~

/)

~ ~
~

~ ~ ~
~

~
~

7) Un paciente de 35 aos consulta en el consultorio de Futaleuf {zona rural), donde usted es el ... mdico de turno. Sufri una cada a caballo hace 30 minutos, resultando con una fractura expuesta del antebrazo de 30 minutos de evolucin. No presenta sangramiento importante y sus signos vitales son normales. Cul de las siguientes medidas es MENOS adecuada para el manejo de este paciente? erivacin inmediata a un centro de fll~)'r complej!dad seo con abundante suero fisilgico ntibiticos endovenosos de amplio espectro\.. ' / d) Vacunacin antitetnia. e) AINES esdovenosos .
,.,/

~
~

8) Respecto a la displasia de cadera es FALSO que: a) Es ms frecuente en mujeres y en nios con antecedente de presentacin podlica b) UIJ6 cada 1.000 nios la presentan e) El signo de Barlow se caracteriza por la posibilidad de luxar la cadera del recin nacido, mientras que el de / ni reposiciona la cadera que est luxada _ h radiografia es el examen de eleccin para el diagnstico en el recin nacido '\.//e) Los nios que se diagnostican despus de los 18 meses de edad suelen ser tratados de manera quirrgica
9) Qu alternativas son ms adecuadas para el manejo de la vra area en un paciente operado de urgel')cia por una obstruccin Intestinal? a) JVJscara larfngea e intubacin retrgrada b}IJ)!\Jbacin retrgrada e intubacin en secuencia rpida . ~)1ntubacin en secuencia rpida e intubacin vigil ""'-/ ~) Intubacin vigil e intubacin con gula e) Intubacin con guia y mscara larlngea

,_,
~
~

,.,
~ ~

1 O) Un paciente de 40 aos presenta aumento de volumen en relacin a la rodilla derecha que ha crecido lentamente a los largo del ltimo ao y que presenta dolor en algunas ocasiones. Al examen fsico se aprecia aumento de volumen de consistencia sea. Se realiza una radiografa, que muestra un tumor seo de 12 cm de dimetro, multiloculado, de aspecto irregular, que compromete la arquitectura .de la epfisis distal del fmur. El diagnstico ms probable es: / a) Osteocondroma "'-!' ~ Osteoma osteoide Tumor _de clulas gigantes ~~QI' de Ewing _,-.-u~)osarcoma

~
~
~

~
~

--....,"

/ A
. z"' v:>

~
~

(Qu fractura supone una mayor gravedad? Fractura expuesta de fmur, con herida de 5 cm, debida a calda de altura Fractura expuesta de fmur;-cen-pequea-herida,-debida a .impacto de bala F;actura expuesta de tobillo,de-2.haras dEL~9Jyciqnt.PPf golpe) directo_gFractura expuesta de ~erna, con herida de gran tamao y mlnlrna cobertura de partes blandas, debida a _, accidente de trnsito "_,.. " e) Fr~ctura expuesta de antebrazo, con restos de tierra en la herida, por calda en bicicleta 12) Una nia de 12 aos, asintomtica, presenta un test de Adams que muestra cierto grado de asimetra en el troco y se visualiza una giba costal leve. Se solicitan radiografas de columna que ~y}stran una escoliosis con desviacin de 10%. La conducta ms adecuada es: _...}~hdicar ejercicios posturales y observar evolucin )l.) Solicitar TAC de columna ~) Indicar uso de cors dorsolumbar d) indicar vendaje en ocho e) Re~plucin quirrgica 13)/Los efectos adversos ms frecuentes de los inductores endovenosos clsicos, como el propofol y tiopental, son: . _ipertermia maligna ,.9) Hipotensin e) Nuseas y vmitos d) Respiratorios e) Al~~~-~ias

~
~

~ ~

~
~
~
~

14)1n paciente presenta dolor lumbar de inicio sbito, que inaci al movilizar unos muebles y que se iryadia por la cara anterior del muslo y la cara medial de la pierna, hasta el malolo medial. Al examen tsico destaca disminucin del reflejo rotuliano. La raiz comprometida es: a) L1 \_/ : S1 b)
~~
.

,d) L4 i'e) L5

,.._

~
15) ~specto a la epitroclertis ~erdadero que:

nti(Se caracteriza por dolor a la palpacin de la insercin proximal de los msculos flexores del antebrazo
'V
?e) Su tratamielo Sete-ser-quirrgico d) Frecuentemente.se.asocia.Jt.lndrone de atrapamiento del nervio cubital e) Es..msltecueote-q~;~ela-eplconQjJitis

a) .s.e le llama codo del tenista

~ ~

'Cul es la presentacin ctrnica m~1s habitual de las fracturas de cadera? cottafniento, rotacin Interna y aduccin del muslo, con imposibilidad de caminar ' c6rtamiento, rotacin externa y abduccin del muslo, con imposibilidad de caminar 9 cortamiento, rotacin interna y abducGin del muslo, con imposibilidad de caminar V d ) Posicin pdica, equimosis, crujido articular y dolor intenso durante la marcha e) Posicin impdica, aumento de volumen y dolor intenso durante la marcha

~ ~

e
~
~

~tracurio
-

17) Qu medicamento se usa como relajante muscular en la tcnica de induccin-intubacin en secuencia rpida? a) Etorttidato b) Ketmina

1Rocuronio (~'Succinilcotina

~
~

18) Un paciente sufre un golpe directo sobre la pierna derecha, al ser barrido mientras jugaba ftbol.
Evoluciona con dolor intenso de la rodilla derecha, que le impide caminar. Al examen fisico se aprecia derrame articular y destaca signo de cajn posterior. Las radiograffas no muestran lesiones seas. El ,. diagnstico ms probable es: '-...._// ~}esin del menisco medial /'~ Ro_'.tura del ligamento cruzado anterior -~ Rotura del ligamento cruzado posterior d) Rotura del ligamento colateral medial e) Rotura del ligamento colateral lateral

~
~

e
~
~

e
~ ~

19) Cul de las siguientes fracturas suele producirse secundaria a osteoporosis? a) Fractura de cpula radial _,. b) Fractura de difisis femoral e) Fracturas. costales / ~~~ractura de extremo distal del radio e Fractra de tobillo

/ /.

~
~
~

f!'
~

~ ~
~

20)'Un paciente sufre hiperextensln forzada del dedo indice derecho, evolucionando con dolor, afrmento de volumen y equimosis, que limita los movimientos del dedo, especialmente la flexin de la .../articulacin interfalnglca proximal. Se solicita radiografa anteroposterior y lateral, que no detecta le~_l)es seas. El diagnstico ms probable es: .fEsguince de dedo /"'b) Fractura avulsiva de la base de la falanga media e) Fractura avulsiva del extremo distal de la falange proximal d) Luxacin intealngica proximal e) Dedo en gatillo

f.:"

e
~

~
~

~
(fl'

<!'

o
1

NOMBRE:

3
4

RUT:

6 7 8

FECHA:

00 ()00 00 ()00 00 ()00 00 ()00 00 ()00 00 ()00 00 ()00 00 ()00 00 (000 00 ~ooo

000

000 o 000 o 000 o 000 o


000

o 000 o

,.

o 000 o 000 o
000

o o

""

CURSO: RESPUESTAS A B C D E
1

~
A B C O E
61 62 63
64

A B C O E

~
~
~

2
3
4

s
7

s s
10

00000 31 00000 00000 32 00000 00000 33 00000 00000 34 00000 00000 35 00000 OOCOO 36 00000 00000 57 00000 OOC>OO 38 00000 OOC>OO 39 00000 OOC>OO 40 00000
41

00000 00000 00000 00000 00000 00000 00000 00000 00000 00000 00000 00000 00000 00000

00000

~
~

65 66 67 68 69 70

~
~

~
~

OOC>OO 00()00 13 00()00 14 00()00 15 00()00 16 00()00 17 00()00 18 00()00 19 00<)00 20 00()00
11 12

42
43

44
45 46 47 48 49 50

00000 00000 00000 00000 00000 00000 00000 00000 00000 00000 00000 00000 00000 00000 00000 00000 00000

71 72 73 74 75 76

,.,
~

00000 00000

nooooo
78 79 80 81 82 83

ooooo
00000 00000 00000

~ ~
~

~ ~
~

00000 00000 30 00000


28 29

00()00 22 00000 23 00000 24 00000 25 00000 26 00000 27 00000


21

51 52 53

54
55 56 57
58 59

00000

84

85
86

87 88 89 90

60

00000 00000

00000 00000 00000 00000 00000 00000 00000 00.000

~ ~

~
~

~
~
~

~
~ ~

~
~ ~

\;)

1\QQ(u

1) Una mlijer de 47 aos que tom anovulatorios durante 3 aos, hace 15 aos, no bebedora, desarri a lo largo de los ltimos meses un cuadro de astenia, prurito ocasional y leve ictericia. El estudio muestra bilirrubinemia de 2,7 m,1./dl, AST 72 u.i., ALT 85 u.i., fosfatasa alcalina triple del limite .s~p'erior de la normalidad y gamma glutc1mil transpeptidasa quntupla. La ecografa abdominal es orm. al. Tras confirmar su diagnstico dt! sospecha mediante el pertinente estudio, cul serfa el ratamiento?: AAcido ~odeso?}jQg.!jgt_._ b) Predniso~a. e) Azatiop~ha. d) Papi~(omla transendoscpica. e) In ioicin teraputica y seguimiento. A un paciente de 70 aos, colecistectomizado, con ictericia de 48 horas de evolucin, Bilirrubina otal de 8 mg/dl y Bilirrubina directa de 6 mg/dl, fosfatasa alcalina 620 UIIL, fiebre de 39C y leucocitosis mayor de 20000 con desviac:in izda, se le realiza ecografia abdominal siendo informada como coledocolitiasis. El tratamiento inh::ial debe ser: a) Reppsicin hidroelectrolitica y antibioterapia nicamente, posponiendo cualquier otro proceder a la desapricin de Jos sintomas y signos de la infeccin. b) Re.posicin hidroelectrolitica, antibioterapia y Japarotomia urgente: e) ~~posicin hidroelectrolitica, antibioterapia y corticosteroides. d) Reposicin hidroelectrolitica, antibioterapia y litotricia. Reposicin hidroelectroltica, antibioterapia y esfinterotomfa + drenaje biliar mediante colangiografa etrgrada endoscpica.
3) Indique cual de las siguientes proposiciones le parece CIERTA en relacin con un episodio agudo de gota: a) Se acompaa siempre de ~ceR=Ua:<' b) El tratamiento requiere alopurinol. )("' e) J.;.'as bursas no se inflaman. :ll.' ~1-os diurticos tiazldicos inducen hiperuril;emia y gota. El anlisis de lquido sinovial no aporta d:dos tiles . .a

e ~
~

J4)
~

~
~

4) Hombre de 74 aos con historia de un mes de evolucin con debilidad, fiebre y dolor en regin lumbar. Dos aos antes se le habra implcmtado una vlvula artica biolgica por una estenosis artica calcificada. A la exploracin fsica est algo confuso con T8 de 37'9, TA de 110/80, pulso alterial de 88 1.p.m. y auscultacin cardiaca con soplo sistlico IIINI en foco artico sin componente diastlico. El hemograma muestra 11300 leucocitos ccan 87% de PMN, Hb de 10'1 gr/dl, Hto de 32% y VSG de 55 mm/h. Los hemocultivos fueron positivo:3 para Enterococcus faecalis y la ecocardiografa transesofgica mostr una verruga en la vlvula artica. Tras 14 das de tratamiento con amplicilina y gentamicina el paciente continuaba con !fiebre y el ECG mostr un PR de 0'26 segundos. Qu actitud, de las siguientes, recomendaria?: a) Sustituir la ampicilina por una cefalosporina de 38 generacin. b) Sustituir la gerttamicina por vancomicina. e) Aadir un.-ntihistaminico por sospecha de fiebre medicamentosa. d)Eealiz_p(nicamente seguimiento clnico, considerando que la evolucin del paciente est siendo normal. ..~Realizar una nueva ecocardiograffa transesofgica para descartar infeccin perivalvular.
5)

(!'
~ ~
~

a enfermedad neurolgica (afectacin del sistema nervioso central o perifrico) con mayor cidencia, entre los siguientes, es: f"Herpes zoster ;'h) ACV o ictus .. e) Demencic,' d) Jaqueca: e) Epil/6ia. 6) A ie una mujer de 80 aos con cefalea, hemicraneal derecha incapacitante y claudicacin al icar. El primer estudio complementarlo que hay que solicitar, entre los siguientes, es: mograma, VSG y bioqulmica srica rutinaria. b) Tomografa computada de crneo. e) Resonancia magntica de crneo. d) Rx de la articulacin tmpora-mandibular. e) Ortopantomografia mandibular.

e
~
~

~
(/!'
~

7) /n hombre de 20 aos consulta por dificultad que se agudiza con los esfuerzos flslcos de e5.blodlna. El estudio gasomtrico demuestra hipercapnla y gradiente alveolo rterial de oxigeno normal. Hacia cual de las siguientes patologias orientar su diagnstico?: 1~nfermedad neu omucular. .fb) Asma bronquial. ~ e) Enfermedad intersticial pulmonar. d) Neumonitis por hipersensibilidad. e) Shunt vfiscular-pulmonar.

;~diografla

t'

res~iratorla

, ,
~

,.,
~

~
~

8) Du~ilte una sustitucin veraniega de Pediatrra una madre de~ 41 aos nos consulta que a su hija prim ~gnita de 9 meses cada vez resulta ms dlffcil colocarle E!l paal. El parto fue por cesrea y la ni ha seguido controles rutinarios del recin nacido sano. Al explorarla apreciamos una marcada dificultad para la separacin de los muslos del beb y una cierta resistencia a la movilidad activa de 1 s caderas, asf como aslmetrra de los pliegues inguinales y ghlteos. Nuestra actitud diagnstica eber ser: S!YRadiografia de caderas. RO) Ecograffa de caderas. e) Exploracin neurolgica exhaustiva. d} Pun.cin lumbar y anlisis de lquido cefalorraqudeo. e) .~sonancia magntica y lumbar.

,.,
,_,
~ ~ ~
~

j
,

s(un colega te pide consejo acerca de 3 pacientes, de la misma familia, que llevan varios dia con dolor n hipocondrio derecho, diarrea y fiebre. En los anlisis rutinarios se detecta una eosinofilia del 40% 2.500 eosinfilos/lJL). Qu pregunta te parece ms relevante ~n la anamnesls, de cara a diagnosticar 1patgeno responsable?: . a) lngesta de lcteos sin higienizar. b) lngesta de pastelerfa o comida sin refrigerar. e) Contacto con perros. . ~lngesta de berros. / e) Contacto con gatos. 1 O) El cido holo-trans-reinoico (ATRA) es una teraputica especialmente eficaz en una de las siguientes variedades de leucemia: ?U.eucemia aguda linfoblstica. 1!.[ ~euc~mia aguda promielocltica. e) ~emia mieloide crnica. d) eucemia linftica crnica. Leucemia aguda megacariocltica.

~
~

~
~

~
~

~
~
~
~

11) Ante un cuadro de fotopsia persistente en el ojo derecho localizada a la altura de la punta de la nariz y visin de puntos mviles en forma do lluvia, la localizac:in ms probable del desgarro retiniano
~emporal s~perior. b) Nasal supenor. e) Nasal inferior. d) Temporal inferior. e) En c;ualquier cuadrante.
~
ser~;,

A,. ~~s criterios diagnsticos para el trastorno por estrs postraumtico incluyen todos los siguientes
EXCEPTO: . a) La re-experimentacin del episodio. b) El aumento del grado de alerta. e) La evitacin de estimulas. ~1 estado de nimo deprimido. /e) Re~rici6n de la vida afectiva . .

Koilocitos son clulas patognomnicas de infeccin por: . / a) Virus herpes simple 1. b) Virus herpes simple 2. ...-:cfftapiloma virus humano.
d) Ch/amydia Trachomatis.

..J~~s

e) Gonococia.

~
/

e
~ ~
~

Cuando se administra Sulfato de Ma!gnesio para el tratamiento de la preecampsia-eclampsia y recen signos de sobredosificacin qu antidoto se debe emplear?: Carbonato sdico. ulfato ferroso. e) Nitroprusiato. d) Simpaticqniimticos. ~uco9ato clcico.

/
d

/
15) ciente nuligesta de 23 aos diagnosticada de tumoracin ovrica ecograficamente sospechosa
alignidad. Se informa de la posibilidad de realizar cirugfa conservadora en caso de cumplirse Jertas requisitos indispensables. Cul de los siguientes supuestos desaconsejara dicha actitud quirrgica?: a) Ca. Epitelial infiltrante bien diferenciado. b) Biopsia de EpiP,lon negativa. e) Tumor encaP..slado. d) Valores de. ..a. 125 superiores a 35 U en estudio preoperatorio.

(!!'
~

~
~

~scitis//
\

16) Cy de los siguientes microorganismos no produce sindrome monocucleslco:


~~rus de la inmunodeficiencia humana (VIH). ~ \'r.oxoplasma Gondii.

~ ~

e} Virus de Epstein-Barr.
AW~Jsteria Mon9cytogenes.

e
~

7e) CitomeQ!.1o\iirus. 17) IIJ.dic{~e cual de las siguientes lesiones pigmentarias, constituye con mayor frecuencia un P!Yursor potencial del melanoma cutnoo. . ~1 Nevus congnito. ...\.._,/' ~) Nevus azul. e) Nevus de clulas fusiformes y epitelioidee. d) Halo-nevus. / ,Pfl'Jevus displsico .. ,.,.. ,. . ....... 18) En 1980, 89.538 enfermeras de los EE.UU. entre 34 y 59 aos y sin historia previa de cncer, cumplimenJaron un cuestionario diettico validado previamente y diseado para medir el consumo individual'le grasa total, grasa saturada, cido linoleico y colesterol, asr como otros nutrientes. Las enfermras se clasificaron entonces en cinco grupos de igual tamao segn los niveles de su ingesta de gr~sa. Se compar 1~ ~:paricin de nuevos casos de cncer de mama entre los grupos. El disefio del e tdio es: / Ecolgico o de ~ase poblacional. ) De prevalencja~ e) Ensayo clif)io , _; e) Des~9pivo de morbimortalidad. 1_9) ~(~e quiere realizar un ensayo clnico de diseo cruzado, el estudio NO podr ser al mismo t1erhpo:
,-

~
~

~
~
~

~
~

~
~

~ohortes:'

1#11\

~ \_Ag~~a:~~~acin !!leatOa.
~

~,. ~

~ De grupos p9ralelos . .... / d) De tamaf\oae muestra predeterminado. e) Enma.sarado.

~
-

~vj}Una manifestacin del sndrome de Dumping.

20J.t(~/~iarrea por dficit de lactasa es:


b) Una diarrea de tipo secretor.

~.

~ na manifestacin del sndrome del intestino corto.


Una diarrea de tipo osmtico. e) Una manifestacin del sndrome de sobrec.recimiento bacteriano.

~(_))
Prueba 6 INFECTOLOG[A

,-~~ ~-o.-Y:_Y),

1) Una paciente de 35 aos, consulta por flujo genital, abundante, de mal olor. A la especuloscopia se observa)6ucorrea griscea, de mal olor, con ausencia de signos inflamatorios en las paredes vaginajes. El agente etiolgico ms probable es: a) Tr :homona vaginallis , b andida albicans Bacterias anaerobias d) Neisseria gonorreae e) Escherichia coli

, V

2) Un~i de 5 aos presenta un cuadro de decaimiento, fiebr~ escasa, asociado a eritema de ambas mejill . A los 3 das presenta un exantema macular eritemato!;o, que compromete el tronco y las zonJl proximales de las extremidades, con un patrn reticular. El diagnstico ms probable es: ~,.Rubeola .Al~.. xantema inespecfico por enterovirus ~ Quinta enfermedad tl) Exantema sbito e) Sarampin 3) Un paciente de 70 aos es trado por su familia por un cuadto de decaimiento marcado, asociado a fiebre alta y luego desorientacin. En su examen fsico se aprecia desorientado, en regulares condipfones, con signos meninges positivos. Se realiza una puncin lumbar y se solicita estudio de LC~con 150.000 clulas por mm3, con 92% de polimorfonucleares, proteinas de 95 mg/dl y glucosa de 20 mg/dl. La conducta ms adecuada, adems de ofrecer soporte, es: Realizar tincin de Gram y cultivo de LCR, administrar corticoide's endovenosos y decidir tratamiento antibitico segn hallazgos b) (rlniciar tratamiento empirico con cefazolina, metronidazol y corticoides endovenosos rl nielar tratamiento emprico con ceftriaxona, ampicilina y corticoides endovenosos Id) lniciaJI(ratamiento emprico con cloxacilina, gentamicina y corticoides endovenosos e) ln~ir tratamiento emplrico con cefotaximo y corticoides endovenosos

\/1
/ /

/tW*I~rp~~ma
e) 01ftena
~serva

(n nio de 11 meses presenta irritabilidad y decaimiento, at~ociados. a fiebre hasta 38,2C. Al xamen fsico presenta vesiculas en el paladar blando, varias de ellas ulceradas, con tendencia a onfluir, asociado a algunas adenopatas cervicales anterioren. El diagnstico ms probable es: a) Mononucleosis infecciosa b) Amigdalitis pultcea c)P,imoinf~ccln herptica .

5)

J.l~ciente defarngeo asociado aun cuadro amigdalina blanco adherente. Al examen fsico se 10 aos presenta de odlnofagla y fiebre hasta 39,5'C. eritema exudado adenopat(as
Pre~enta

/cervicales bilaterales y bazo palpable 2 cm bajo el reborde co!;tal. Se solicita hemograma que muestra , // lecocitosis de 17.000, con 75% de linfocitos, algunos de ellos atpicos. La conducta ms adecuada es: '\....../ a) Iniciar ceftriaxona endovenosa b) Iniciar aciclovir oral e) Administrar penicilina benzatina intramuscular ~niciar.A1NEs y solicitar lgM VCA e) soyetfar cultivo faringeo

,/~ Sifilis primaria

~a;~opatas inguinales. El diagnstico ms probable es:


e) Linfogranuloma venreo d) Chancroide e) Condiloma acuminado

6) u'n paciente de 44 aos presenta varias lceras confluentes y dolorosas en el glande, asociadas a

\._./ j}!' Herpes genital

~ ~

..

7) Una paciente de 30 aos presenta un cuadro de tos con expectoracin, asociada a fiebre y disnea, de 3 dfas de evolucin y cuyos sntomas han ido empeorando. Al examen fsico se aprecia bastante taquipneica y se auscultan crpitos en la base pulmonar derecha y en la mitad superior del campo pulmonar izquierdo. Se solicita radiografa de trax que es compatible con una neumonfa multilobar, con compromiso de los lbulos medio e inferior derecho y superior izquierdo. El tratamiento antibitico ms adecuado es: a) Amoxicilina b) Amoxicilina ms cido clavulnico tfCeftnaxon e ricptona + levofloxacino ) G~micina + vancomicina

1tratamiento de eleccin de la onic(>micosis es:


lotrimazol tpico hTerbinafina oral e) Fluconazol oral d) Terbinafina tpica e) Griseofulvina oral 9) Un pij.efente presenta desde hace 6 s1~manas tos persistente, con expectoracin mucopurulenta, asoc!Ptlos a fiebre intermitente y baja d(~ peso de 4 kilogramos. Al examen fisico presenta ere 1taciones y estertores en ambos pces pulmonares. La radiografa de trax muestra reas de fi rosis y retraccin en los pices pulmlnares. La conducta ms adecuada es: ) Iniciar ceftriaxona endovenosa b) Solicitar tincin de Gram y cultivo de lav;3do bronquioloalveolar e) Iniciar cloxacilina endovenosa ~olipitar baciloscoplas y cultivo para myGobacterias e) l~iar clindamicina endovenosa El agente etiolgico ms frecente de la artritis sptica en los niiios es: \ . / ~Estafilococo dorado /b) Estreptococo betahemolitico grupo A e) Estreptococo betahemolitico grupo B d} Gonococo e) Neumococo

~ ~
~
~

~
~

~ (A

~
~

~
~

. 16)

~
~

~
~\

~
~

j
\

~ ~
~

~ ~
~

~ 1

{un paciente de 47 aos, obeso, se realiza una la pierna derecha mientras realizaba e porte. Evoluciona con eritema, dolor y aumento de volumen de la zona. Al examen ffsico est febril, on una placa eritematosa de 10 cm de dimetro, con bordes difciles de definir, asociado a ~inopatras inguinales ipsilaterales. El tratamiento ms adecuado es: ~/t;loxacilina (o) Ceftriaxona e) Amoxicilina ~ Ciind micina e) e ofloxacino 2) Un paciente de 20 aos presenta un cuadro de un da de evolucin de fiebre marcada y decaimiento seguido de actitud agresiva y desorientacin. Finalmente presenta una co.p;-t.Y!~l<?..~ Al _,~amen flsico est febril, soporoso, sin signos focales. La conducta ms adecuada es: - ~"Administrar aciclovir endovenoso y solicitar TACcerebral y estudio de lquido cefalorraqudeo, que incluya PCR para virus herpes b) Realizar puncin lumbar y solicitar citoqumico, gram y cuitivo de lquido cefalorraqudeo, decidiendo tratamiento segn hallazgos e) Iniciar antibiticos de amplio espectro por va endovenosa y solicitar citoqulmico, gram y cultivo de liquido cefalorraqudeo d} Solicitar TAC de cerebro y decidir manejo segn hallazgos e) Iniciar corticoides endovenosos, antibiticCJs de amplio espectro y solicitar resonancia magntica nuclear de cerebro

herida~~

~
~

~
~
~

~
(lA

(!)

/b) Ciprofloxacino + diclofenaco


~) Gentamicina + ibuprofeno
d) Cefotaximo + ketoprofeno e) Metronidazol + paracetamol

ffi~~xicilina + ibuprofeno

13,/cul de los siguientes tratamientos es el ms adecuado pa.ra una amigdalitis pultcea?

14) E)Aratamiento ms adecuado para una infeccin del tracto urinario febril, cuyo urocultlvo est pel)dente es: "Nitrofurantoina btMoxifloxacino ,e Cefradina d) Clindamicina e) Cl~ritromicina

.l examen de eleccin ante la sospecha de fiebre tifoidea, c.on 1O dfas de sfntomas, es:

emocultivo opsia de mdula sea e) Coprocultivo d) lgM e lgG para Salmonella typhi e) El examen fsico

\ x,..

. ...- 16) La sospecha clfnica de hepatitis A, se confirma con:

a) PCR para VHA . , '. . b) Antigeno de superficie y anticuerpos anti-core de tipo lgM ~ticuer.po tales para VHA ( d) lgM U:tU:I-Y"Fitn. . ruebas hepticas

v1

17) Un pciente de 20 aos presenta un cuadro de fiebre y decialmiento, con aparicin de un exantema vesicJ.IIopustular generalizado y pruriginoso, que inici en la cabeza, comprometiendo el cuero ca_!}e11udo. Al examen fsico se observan algunas lesiones como pstulas y otras como costras. La onducta ms adecuada es:

~olicitar lgM para varicela e Iniciar tratamiento sintomtico .?Y!~oiciar aciclovir oral y tratamiento sintomtico

. / e) Indicar tratamiento sintomtico con clorfenamina y paracetamol d) Iniciar aspirina y ceterizina e) Iniciar corticoides orales

vA

18) Un paciente de ~s presenta un cuadro clfnico compatibles con una m~n~ngitis agud~. El nlisis citoqufll)JB9-ttS::COmpatlb.l.~on una meningitis bacteriana agu.da y la ticin de Gram demuestra ((__Cceas gram positivas: El agente ms probable ElS:
) Meningococo ___ ---- .... -/

b) Haemophilus e) Listeria d) ~.s~filococo areo

~:.~eumococo

1S) Ante la sospecha clnica de sffilis secundaria se debe: /a) Solicitar FTA-ABS o MHA TP . ~/ b} Administrar penicilina benzatina intramuscular e) Iniciar !JOB cefalosporina de tercera generacin ~o~itar VDRL o RPR . e) h')itiar un macrlido

,
\
\__/

~6) El tratamiento antibitico de eleccin para un paciente que! presenta una celulitis periorbltaria, sin
/compromiso de la agudeza visual, oculomotilidad ni de los reflejos oculares, es:
a) Cefazolina b) Pipercilina + tazobactam e) Vancomlcina d). Penicilina ~ Ceftriaxona '
~

~
~-

~
~
~ ~

o
1

NOMBRE:

3
4 5

~
~
~ ~

RUT:

S 7 8

FECHA:

00 00 00 00 00 00 00 00 00

00 000 000
000 000 000 000 000 000 000 000 000

000 000. 000 000 000 000 000 000 000

o
o

o o o o o o o o
E

f!';
~ ~ ~

CURSO: RESPUESTAS A
1 2

e D E
31 32 33 34 35 36 37 38 39 40

A B C

A B
61

e O E

~
~

~
~ (A

~
~

00000 00000 3 00000 4 00000 5 00000 S 00000 7 00000 8 00000 9 00000 10 00000 00000 00000 13 00000 14 00000 15 00000 16 00000 17 00000 18 00000 19 00000 20 00000
11 12

00000 00000 00000 00000 00000 00000 00000 00000 00000 00000

s2
63 64 65 SS 67 68 69 70

00000 00000 00000 00000 00000 00000 00000 00000 00000 00000 00000 00000 00000 00000 00000 00000 00000 00000 00000 00000 00000 00000 00000 00000 00000 00000 00000 00.000 00000 00000

~
~ ~

41

~ ~
~
~
~ ~

00000 00000 43 00000 44 00000 45 00000 46 00000 47 00000 48 00000 49 00000 50 00000
42 51

71 72 73 74 75 76 77 78 79

so
81 82 83 84 85 BG 87 88 89 90

~
~
~

~
~

"
~
~

~ (!'
~

00000 00000 23 00000 24 00000 25 00000 26 00000 27 00000 28 '00000 29 00000 30 00000
21 22

00000 52 00000 53 00000 54 00000 55 00000 56 00000 57 00000 58 00000 59 00000 60 00000

~
~
~

2'

~ !'
Prueba 7
INFECTOLOGiA, segunda parte 1) Un paciente de 34 aos presenta.-lgorra.enla..ca'lida.cl.b.~~l y refiere haber bajado ..4.Kg. de peso en los ltimos 3 meses. El examen fsico. no aporta .mayor informacin. La conducta ms adecuada es:
1

a) S~t.cultive-parahongos de hisop.ado bucal e iniciar fluconazol oral Jlr'Solicitar ELISA para VIH) ~)Sohc1tar carga v1ral para -VIH ~Solicitar Westemblot para VIH e) Solicitar recuento de linfocitos CD4
zo~ perianal. El tratamiento de eleccin es:

~
.

2) Uryr{io de 3 aos presenta prurito anal importante. La madre visualiz un pequeo gusanito en la

~
~

)"Prazicuantel ) Metronidazol ) yotrimoxazol ~fMebendazol e) Benzinidazol

e
~

Cul de los siguientes antibiticos puede ser ocupado para tratar infecciones por Staphilococcus eus sensible a la meticilina, con me,jor respuesta?
\ V efazolina eftriaxona e) Metronilazol d) Amoxicilina e) Azotromicina

(!P'
~

~
~

4) Un paciente portador de una leucemia linftica aguda, consulta por fiebre y malestar general. Al en fsico slo presenta palidez. Su solicita hemograma que muestra un hematocrito: 27%, b neos: 20.000, con 2% de neutrfilos, 66% de linfocitos y 32% de blastos, plaquetas: 52.000. La onducta ms adecuada es:

~
~ ~

//a) Solicitar estudio para VIH y realizar exmenes de laboratorio en busca de un foco Infeccioso b) Iniciar quimioterapia y vancomicina y realizar exmenes de laboratorio en busca de un foco infeccioso e) Iniciar cefazolina + metronidazol y reali~:ar exmenes de laboratorio en busca de un foco infeccioso ~~iciar ceftriaxona + ampicilina y realizar exmenes de laboratorio en busca de un foco infeccioso _,niciar ceftazidina + gentamicina y reali2:ar exmenes de laboratorio en busca de un foco infeccioso

,7'

~
~

5Y.Jh paciente consulta por mialgias generalizadas y fiebre. Al examen fsico presenta dolor a la de las masas musculares, sin otros hallazgos de importancia. Se solicita hemograma que demuestra eosinofilia importante. El diagnstico ms probable es:
~~~acin

~
~
~

/' '-../

a) Larva migrante visceral b) Distomatosis e} tJidatidosis ;;tfTriqujnbsis e) C!.stcercosis

f!!'
~
~

(~na paciente est hospitalizada hace 10 horas por una neumona del lbulo inferior izquierdo,

dqui.rida en la comunidad, ATS 111, con fiebre hasta 41,5C y escasos requerimientos de oxigeno. Su presin arterial es 120/80 mmHg y su pulso es de 100 lpm. Se solicitan tres hemocultivos y se inicia ratamiento con ceftriaxona, de los que uno de ellos resulta positivo para Staphilococcus aureus, mientras que los otros 2 resultan negati\fos. La conducta ms adecuada es: a) Iniciar vancomicina y supender la ceftria>:ona
b) Iniciar cefazolina y suspender la ceftriaxona e) Agregar vancomicina al tratamiento con c:eftriaxona d) Agregar cloxacilina al tratamiento con ce1lriaxona

~
~ ~

rantener la ceflriaxona, ya que muy probablemente se trata de contaminacin de la muestra

~
~

~ ~

J:

7) Un paciente de 66 aos diabtico, mal controlado, presenta 21umento de volumen y dolor facial de rpjda instalacin asociado a escasa secrecin nasal. Al examEtn se observa eritema y edema facial, con un rea necrtica negruzca en relacin a la prominencia malar Izquierda. El agente etiolgico ms fobable es: ) Pseudomona aureginosa . b) Neumococo e) Staphilococcus aureus d) Rtreptococcus pyogenes /Mucor

8) Ll;.distomatosis se contrae debido a: a) 'ilacines sexuales sin proteccin b) Picaduras de insecto 1 . 9) lngesta de carnes y lcteos mal cocidos ~ pnge~ta de berros silvestres e)Jsta de alimentos contaminados con facas
~ El tratamiento de eleccin de la uretritis por Chlamydla trachomatls es: a) Ciprofloxacino b) Ceftriaxona e) Penicilina J( Doxiciclina /"e) Sulfas

vA

1O) Un pa6iente cursando una Infeccin, inicia tratamiento antibitico endovenoso. Durante la administracin del frmaco presenta aparicin de lesiones eriftematosas, solevantadas en la cara, cu;11f>, hombros, y zona proximal de las extremidades superiores, con prurito. El antibitico que con mayor probabilidad produjo este cuadro es: Gentamicina b~ Cloxacilina e) Ampicilina d) Clindamicina rvancomicina 11) Un paciente cursa con fiebre, compromiso del estado general y aparicin de un soplo diastlico de intensidad moderado. En el examen ffsico presenta adems aDgunas petequias subconjuntivates y en los pulpejos. Los mejores exmenes para el diagnstico son: a) PCR y hemograma pl_Angio TAC y hemocultivos ~Hemocultivos y ecocardiografia transesofgica ]JjEcocardiografa transtorcica y hemograma /) Ventriculografa y ecocardiografia transesofgica

12) ~as meningoencefalitls virales suelen ser cuadros benignos y autolimitados. El agente etiolgico m~s frecuente es: a) Virus herpes simplex v~b~irus herpes zster Enterovirus d) Virus influenza e) A~~povirus

aj Enfermedad de Chagas //b) rylalaria \./ fifftidatidosis d) Oxiuriasis e) Sama

1 ~r'i.a vmica es una manifestacin de:

~
1~} Qu tratamiento NO aconsejara a una infeccin del tracto urinario febril? )f}Cio:cacilina . b) Giprofloxacino

e"

~~efotaximo
e) Cefazolina

\../~~ Gentamicina

eu'
~

15) Un paciente VIH positivo, con tratamiento antirretroviral intermitente, presenta disminucin progresiva de la agudeza visual bilateraa, de 7 dias de evolucin, que actualmente slo le permite dis~rguir siluetas. Su ltimo recuento de CD4 se lo realiz hace 7 meses y en ese entonces era de 72 ular CD4+ por mm3. La conducta m~; adecuada es: ~licitar RMN de cerebro 9.:1'~ealizar fondo de ojo Iniciar aciclovir d) Realizar estudio de liquido cefalorraquideo e) Iniciar corticoides endovenosos

r'
~ ~
~ ~

16) Un paciente de 6 aos presenta dolor y aumento de volumen cervical. Al examen fisico se aprecia
una adenopata cervical anterior Izquierda de 3 cm, dolorosa, con escaso eritema en la piel que la recubre. Destaca adems caries y enfermedad periodontal generalizadas en la cavidad bucal. La conducta ms adecuada es: a) Administrar una doisis de penicilina benz~tin~Jntramuscular. ~niciar tratamiento con amoxicilinamas~~iaocTav~t6ib por vi a or_;L: :~Hospitalizar e iniciar urtacefalosporint:n:ltdeifcera:geefaci.potvia endovenosa ~Solicitar hemograma, hemocultivos e lgM para Bartonella hensellae /'Tomar cultivo por puncin con aguja fina 17) !- neurocisticercosis se caracteriza por todo lo siguiente EXCEPTO: a)..Es causada por Taenia solium b) Suele ser asintomtica \ / c*Se contrae por consumir carne de cerdo rnal cocida '\.._/ 1J) Los pacientes con neurocisticercos no transmiten la enfermedad a otras persona e) Es una causa rara de epilepsia

~
~ ~ ~

~
~
~

\_./'~ lmipenem - Estafilococo dorado

~enicilina - Estreptococo beta-hemolitico grupo A


e) Vancomicina - Acinetobacter baumanii d) Azitromicina - Pseudomona aureginosa e) ~~tronidazol - Enterococo _

18)6u relacin entre antibitico de elec;cin y

be._cteri~ es correcta?

~
~ ~ ~ ~
\

"---

) A

~
~

Qu antibiticos son ms efectivos para tratar infecciones causadas por anaerobios? Metronidazol y clindamicina b) Amoxicilina y levofloxacino e) Doxiclina y penicilina d) Cefazolina y gentamicina e) Flucloxacilina y tinidazol
20) Un" paciente VIH, mal controlado consulta por disfagia. Presenta algorra en la cavidad oral y

~
~

~
~
~

regtfento de CD4 de 140 clulas x mm3. El diagnstico ms probable es: a)"Sarcoma de Kaposi esofgico , /6) Esofagitis por criptosporidium . / ..9YEsofagitis por cndida .~ /d) Esofagitis por citomegalovirus e) Esofagitis por virus de la inmunodeficiencicl humana

~
~ ~ ~

(A
~

o
1

NOMBRE:

2 3

RUT:

7
8

00 000 000 00 000 000 00 000 000 00 000 000 00 000 000 00 000 000 00 000. 000 00 000 000

o
o o
o o

FECHA:

9 K

o o o 00 000 000 o 00 000 000 o o


RESPUESTAS
A B
~

CURSO:

O E
31
32 33

1
2

3 4

5
6

7
8
9

00()00 00()00 00()00 00()00 00()00 00000 00<)00 00<)00 00<)00

34
35

10

00000 57 00000 38 00000 39 00000 OOC>OO 40 00000


36

00000 00000 00000 00000 00000

00000 ~ 62 00000 63 00000 ~ 64 00000 ~


61
65

66
67
68 69

00()00 00()00 13 00(000 14 00000 15 00000 16 00000 17 00000 18 00000 19 00000 20 00000
11
12

00000 00000 43 00000 44 00000 45 00000 46 00000 47 00000 48 00000 49 00000 so 00000
41
42

00000 ~ 00000 ~ 71 00000 ~ 72 00000 ~ 73 00000 ~ 74 00000 ~ 75 00000


70
76
77
78 79
80

00000 ~ 00000 ~ 00000 00000 ~

00000 ~ 00000 ~ 00000 ~ 00000

00000 ~
~ ~ ~ ~ ~

00000 00000 23 00000 24 00000 25 00000 26 00000 27


22
28 29

21

51 52
53

54
55 56
57 58 59

00000

00,000
oc~ooo

00000

00000 00000 00000 00000 00000 00000

81 82
83

84
85 86
87 BB 89

00000 00000

30

oc,ooo

60

ooooo

00000 00000 00000 00000 00000 00000 00000 00.000

90

00000 ~
~
~

ooooo

~ ~

~
~

r'

~
Gastroenterologa 1) Un paciente de 24 aos presenta un cuadro de dolor abdominal y deposiciones diarreicas de 6 semanas de evolucin, que en alguna:s ocasiones han sido sanguinolentas. Refiere que ha bajado 4 kg de peso desde el inicio de los sfntomcts y presenta sensacin febril ocasional. En esos d(as adems present un absceso perianal que fue drenado quirrgicamente. El diagnstico ms probable es: a) Colitis viral b) Colitis bacteriana e) Colitis por parsitos d) Colitis ulcerosa e} Enfermedad de Crohn 2) Un paciente en tratamiento con ibuprofeno por artrosis, inicia de manera brusca lipotimias, con marcado ortostatimo. En una ocasin :adems presenta deposiciones negras de muy mal olor. Al examen fsico se observa plido y el tGtcto rectal demuestra melena. La primera medida a realizar es: a) Realizar endoscopa digestiva alta b) Administrar omeprazol endovenoso e) Realizar transfusin de glbulos rojos d) Solicitar hemograma e) Administrar cristaloides endovenosos 3)Un paciente de 66 aos presenta un cuadro de constipacin marcada de 3 das de evolucin, sin poder presentar deposiciones, asociado a dolor abdominal. Luego presenta deposiciones diarreicas explosivas en una oportunidad. RefierE! adems historia de palidez y molestias abdominales de 1 mes de evolucin. El examen fisico muestra~ un paciente plido, con abdomen blando, depreslble y el tacto rectal demuestra una ampolla rectal sin deposiciones, sin hallar otras alteraciones. Cul es el diagnstico ms probable? a) Vlvulo de colon sigmoides b) Cncer de colon e) Obstruccin intestinal por bridas d) Slndrome de Ogilvie e) Diverticulitis 4) Un paciente de 30 aos, previamente sano, presenta dolor epigstrico intenso, constante, irradiado al dorso y acompaado de vmitos alimentarios en varias oportunidades. Presenta FC: 97x', t 0 : 36,9C, PA: 120/80 mmHg. El examen abdominctl demuestra dolor a la palpacin epigstrica, ruidos hidroareos presentes y ausencia de shgnos de irritacin peritoneal. La conducta ms adecuada es: a) Administrar analgsicos y solicitar radio9rafia simple de abdomen y pelvis b) Administrar suero fisiolgico, analgsicCis y antibiticos endovenosos e) Administrar analgesia y solicitar ecografa abdominal, hemograma y lipasa plasmtica d) Administrar analgesia y solicitar un TAC de abdomen y pelvis e) Realizar laparotomia exploradora 5) Un paciente cursa con una hepatitis atguda por virus B. Cmo esperarla encontrar los marcadores virales? a) Antgeno VHBs(+), anticuerpos anti-VHBs(-), lgM anticore(+) b) Antigeno VHBs(+), anticuerpos anti-VHEis(+), lgM anticore(-) e) Antgeno VHBs(-), anticuerpos anti-VHB::;(+), lgM anticore(+) d) Antgeno VHBs(+), anticuerpos anti-VHBs(-), lgM anticore(-) e) Antigeno VHBs(-), anticuerpos anti-VHBs(-), lgM anticore(+) 6) Cul es el tratamiento de eleccin pa1ra la ascitis moderada, no complicada, en un paciente cirrtico por consumo de alcohol? a) Dieta b) Espironolactona y furosemida e) Albmina endovenosa d) Paracentesis peridicas e) Instalacin de un TIPS (Shunt portosistrnico)

~ !!'

(A .!"

~ ~
~
~ ~
~-

~
~
~

~ ~ ~
~

~ ~

~
(/!t'

~
~
~

,.

,...,

~
~ ~

~
~

7) Un paciente celaco puede consumir libremente todo lo que se nombra a continuacin, EXCEPTO: a) Ensalada de frutas b) Miel de abeja e) Atn enlatado d) Cereal de maiz e) Pan de centeno 8) Un paciente de 23 aos consulta por epigastralgia de larga data, que aparece especialmente despus de comer. El examen fsico es normal. Se solicita endoscopfa digestiva alta que demuestra una esofagitis distal lineal, compatible con reflujo gastroesofgico. La conducta ms adecuada, de entre las siguientes, es: a) Iniciar omeprazol . b) Realizar tratamiento erradicador para Helicobacter pylori e) Solicitar pH-metria esofgica de 24 horas d) Solicitar manometria esofgica e) Diagnosticar dispepsia funcional y realizar educacin 9) La causa ms frecuente de elevacin persistente de las transaminasas en Chile es: a) Alcohol b) Hepatitis 8 e) Hepatitis C d) Esteatosis heptica e) Autoinmune 10) Cul de las siguientes historias clnicas es ms compatible con el cuadro clinico caracterstico de la cirrosis biliar primaria? a) Mujer de 40 aos, con astenia y prurito, con elevacin marcada de las fosfatasas alc.alinas y discreta de la bilirrubina b) Hombre de 60 aos, con ictericia y presencia de ANCA e) Mujer de 60 aos, con antecedente de vitiligo, con elevacin persistente de transaminasas, sin hbitos txicos y con serologia viral negativa d) Mujer de 30 aos con ictericia fluctuante, de predominio indirecto y elevacin de la GGT y fosfatasas alcalinas, con GOT y GPT normales e) Hombre de 60 aos con icteriacia, coluria y acolia, asociado a baja de peso 11) El sintoma preponderante en la acalasla esofgica es: a) Regurgitacin de alimentos cidos b) Disfagia lgica de instalacin progresiva e) Dolor retroesternal durante la deglucin d) Dificultad para tragar liquidas y slidos, de curso fluctuante e) Sensacin de cuerpo extrao, de localizacin esofgica alta 12) Un paciente de 60 aos, sin antecedentes de importancia, presenta cuadro de vmitos postprandiales abundantes, que suceden algunos minutos despus de ingerir alimentos. Refiere que los sintomas iniciaron hace 4 meses, pero que se han vuelto cada vez ms Intensos, vomitando incluso luego de ingerir pequeas cantidades de alimentos. El diagnstico ms probable es: a) Pancreatitis crnica b) Cncer esofgico e) Cncer gstrico d) Gastritis crnica atrfica e) Diverticulos esofgicos 13) Un paciente cirrtico por consumo de alcohol ingresa por desori~ntacin y fiebre. Al examen se constata ascitis moderada, Ictericia y asterixis. Se decide realizar paracentesis diagnstica, que da salida a Uquido amarillo, transparente, con recuento celular de 490 clulas por mm3, con 90% polimorfonuclear. La conducta ms adecuada es: a) Solicitar evaluacin por cirujano. b) Realizar paracentesis evacuadora, asociada a antibiticos endovenosos. e) Iniciar Cefotaximo, sin evacuar la ascitis d) Administrar lactulosa, a fin de lograr 2 a 3 deposiciones diarias e) Administrar cristaloides endovenosos, asociados a terlipresina

~
14) Un paciente de 30 aos, consulta por un cuadro de algunos aos de evolucin, de dolor abdominal tipo clico, intermitente, asociado a distensin abdominal. Refiere trnsito intestinal lento, con deposiciones duras, cada 4 a 5 das. En el ltimo tiempo lo sfntomas se han vuelto ms frecuentes, lo que lo tiene intranquilo. El examen fsico resulta normal. El diagnstico ms probable es: a) Obstruccin intestinal b) Enfermedad celiaca e) Slndrome de intestino irritable d) Colitis ulcerosa e) Cncer de colon 15) Un nio de 2 aos presenta diarrea acuosa abundante, de 3 das de evolucin, con cerca de 12 deposiciones diarias. Tambin present fiebre hasta 38C durante los primeros das. En las ltimas horas se agrega irritabilidad important:!. Al examen fsico presenta signos vitales normales, llora sin lgrimas y toma lquidos vidamente. La conducta ms adecuada es: a) Enviar a domicilio con rehidratacin orctl segn plan A b) Solicitar coprocultivo y Elisa para rotavirus en heces y reponer volumen por sonda nasoenteral e) Indicar sales de rehidratacin oral, 100 cc/Kg, a tomar en 4 horas y reevaluar d) Indicar sales de rehidratacin, luego de cada deposicin y antibiticos por vra oral e) Administrar bolo de suero fisiolgico endovenoso 16) Un paciente inicia diarrea acuosa y dolor abdominal clico, 2 das despus de completar un tratamiento antibitico con amoxicilina, debido a una neumonra adquirida en la comunidad. Se solicitan toxinas de Clostridium difficilu en heces, las que resultan positivas. La conducta ms adecuada es: a) Solicitar coplrocultivo y decidir manejo Begn resultado b) Iniciar metronldazol oral e) Iniciar clindamicina oral d) Reiniciar la amoxicilina, agregando cido clavulnico e} Iniciar rehidratacin oral, sin necesidad de iniciar antibiticos 17) Un paciente presenta pirosis frecuente, luego de comer, especialmente cuando come antes de ir a dormir. Tambin refiere regurgitacin cida de alimentos, varias veces por semana. El examen fsico es normal. La conducta ms. adecuada t~s; a) Indicar medidas generales antireflujo e iniciar un inhibidor de la bomba de protones b) Solicitar una Ph-metria esofgica e) Prohibir el consumo de los alimentos qua le producen los sintomas y controlar en 4 semanas d) Solicitar endoscopla digestiva alta e) Solicitar manometria esofgica e iniciar bloqueadores H2 18) Un paciente de 60 aos presenta emisin de deposiciones mezcladas con sangre en dos ocasiones en el mismo da, sin embargo las deposiciones se normalizan en los dias siguientes. No ha presentado otros sintomas y el examen fsico no aporta mayor Informacin. La conducta ms adecuada es: a) Solicitar prueba de hemorragias ocultas b) Realizar colonoscopia . e) Solicitar antfgeno carcinoembrionario d) Solicitar ecografa abdominal e) Observar y controlar en un mes 19) El tratamiento de la lcera duodenal no complicada suele ser: a) Omeprazol a permanencia b} Antibiticos e inhibidores de la bomba del protones por 1O di as e) Antiinflamatorios no esteroidales d) lnyectoterapia endoscpica e) Quirrgico 20) El factor pronstico ms importante on el cncer gstrico es: a) El tipo histolgico del tumor b) El tamao del tumor e) La presencia de adenopatas metastsiCciS d) El grado de invasin transmural del tumor e) La presencia de anemia

~ ~

~ ~

~ ~ ~
~

~
~ ~

~
~

r'
~

~
f!A

'-

("A

~
~

~ ('A

f'/Pt
(11\

fi!t'

fP'
~

(f"

o
1

NOMBRE:

2 3

00 00 00 00

4
5

RUT:

7.QQ
8

00

()00 000 o 00 (000 000 o 00 IQQO 000 o

<)00 000 ()00 000 ()00 000

o o

FECHA:

o o 00 000 000 o 00 000 000 o


~ooo

000 000 000

o
A B e D E
61 62 63
64

e u RSO:
RESPUESTAS A B C O E
1 2

A B e
31 32 33

D E

ooc,oo OOC,OO oocoo s OQC)QQ


3

000100

34
35 38

7 OOC>OO
s

OOC>OO OOC>OO OOC>OO

7 '3 38 39 40

s
10

00000

00000 00000 00000 00000 00000 00000 00000 00000 00000 00000 00000 00000 00000 00000 00000 00000 00000 00000 00000 00000 00000 00000 00000 00000 00000 00000 00000 00000 00000

00000 00000 00000 00000 00000 00000 00000 00000 00000 00000 00000 00000 00000 00000 00000 00000 00000

~ ~

00000

~
~

65 66

~
~

67
68 69 70

00000

~
~

~
~
~

00()00 00()00 13 00()00 14 00()00 15 00<)00 16 00<)00 17 00<)00 18 00f)00 19 00000 20 00()00
11 12
21

41
42

71
7

43

73 74

44 45 46 47
48 49 50

~
~

75
76

78 79

nooooo
80 81 82
83

~
~
J

22
23 24 25
26

oo~ooo oo~ooo

51 52 53 54 55 56 57 58
59

27 28

ooooo
00000

00000 00000 00000

84
85 86 87
88

00000 29 00000
30

00000

00000

00000 00000 00000 00000 00000 00000 00000 00.000

~
~
;

~
~

~
~
~

89 90

so

00000

00000

~
~
~
~

Al)

Gastroenterolog(a 1) .Un paciente de 24 aos presenta un cuadro de dolor abdominal y deposiciones diarreicas de 6 semanas de evolucin, que en algunas o':aslones han sido sanguinolentas. Refiere que ha b~g Ce peso desde ell11icio de los s[ntomas y presenta sensacin febril ocasional. En esos dias adems present un absceso perlanal que fue drenado quirrgicamente. El diagnstico ms probable es: a) Colitis viral ~89t~ G{l..()t.J\c.A b) Colitis bacteriana ... _ _ .. __ e) Colitis por parsitos 1\1\~~~ ~~. . C-A. - ~\ d) Colitis ulcerosa /(;)Enfermedad de CrOhn~~-A.-~~(,~ -M-S~. (~U.~' ifJ\. ~~~Mt,O ~ t~\.,;"" 2) Un paciente en tratamienTo con ibuprofeno por artrosis, inicfa de manera brusca lipotimias, con marcado ortostatimo. En una ocasin ade,ms presenta deposiciones negras de muy mal olor. Al examen fsico se observa plido y el tactc' rectal demuestra melena. La primera medida a realizar es: a) Realizar endoscopfa digestiva alta b) Administrar omeprazol endovenoso e) Realizar transfusin de glbulos rojos d) Solicitar hemograma / @Administrar cristaloides endovenosos 3) Un paciente de 66 aos presenta un cuadro de cons~acjo marcada de 3 ~las de evolucin, !WL poder presentar deposiciones, asociado e' dolor abdo01'al. Luego presenta deposiciones diarreicas ~ 6-UJ~ explosivas en una oPftunidad. Refiere adems historJade palidez y molestiasabdominales de 1 mes de evolucin. El examen fislco muestra un..e_aciente plido, con abdomen blando, depresible y el tacto ~ r.e~tal demuestra una ampolla rectal sin dt:!posiciones, sin hallar otras alteraciones. Cual es el ~ 'aiagnstico ms probable? .. ~ Vlvulo de colon sigmoides , \! b 1 , ~ / (b~Cncerde colon:. t.9.~~4~'t'. ~J.)~ ~~}.)r\ICS-~.}..1"\'\Cl~Ne~ l\-~ '~' "'"' ... e) Obstruccin intestinal por bridas '" d) Slndrome de Ogilvie ~ e) Diverticulitis
~

eu\,f\0 ~
~

,,

4) Un paciente d 30 aos, previamente Sctno, presenta dolor epigstrico Intenso, constante, irradiado tJ6.L~~ C al dorso Y..~compaado de vmitos alimentarios en varias oportunidades. Presenta FC: 97x', e: 36,9C, ~~riS\'"~. tta( bO PA: 120/30 mm Hg. El examen abdominal demuestra dolor a la palpacin epigstrica, ruidos
hidroareos presentes y ausencia de signos de irritacin peritoneal. La conducta ms adecuada es: a) Administrar analgsicos y solicitar radiogmfla simple de abdomen y pelvi~ .. b) Administrar suero fisiolgico, analgsicos y antibiticos endovenosos /@Administrar analgesia y solicitar ecografla abdominal, hemograma y lipasa plasmtica. d) Administrar analgesia y solicitar un TAC dH abdomen y pelvis e) Realizar laparotomfa exploradora

~ -.. \:.

f!!A

5) Un paciente cursa con una hepatitis aguda por yirus B. Cmo esperarra encontrar Jos marcadores virales? .. . l\)~~@ ~ --t'~1Antgeno VHBs(+}anticuerpos anti-VHBs(-), lgM anticore(+).. " r"\ JJ(} ~nA \.:l U\ S. l~\'\S. \!?fAntigeno VHBs(+)<"anticuerpos anti-VHBs(+), lgM anticore{-) 1\e fcl.\~ ~ o M ~~W ~~ ~.. e) Antgeno VHBs(-), anticuerpos anti-VHBs{+), lgM anticore{+),.. d) Antgeno VHBs(+)l'anticuerpos anti-VHBs(-), lgM anticore(-) _ e) ~ntgeno VHBs(-), anticuerpos anti-VHBs{), lgM anticore(+)..p ~ O\.OU'Q.

f\a

6) Cul es el tratamiento de eleccin parct la ascitis moderada, no complicada, en un paciente cirrtico por consumo de alcohol? .... ~. a) Dieta '2b) Espironolactona y furosemida ~ Albmina endovenosa.< i::l) Paracentesis peridicas e) Instalacin de un TIPS (Shunt portosistmico)

~h\fte~'9n""OO\
~

q\0

~ ~~A.t\OOk\.\ \tt5l~n'eA ~

7) Un paciente celiaco puede consumir libremente todo lo que se nombra a continuacin, EXCEPTO: a) Ensalada de frutas b) Miel de abeja e) Atn enlatado d) Cereal de malz 1@Pan de centeno,. 8) Un paciente de 23 aos consulta por eplgastralgia de larga data, que aparece especialmente despus de comer. El examen fislco es normal. Se solicita endoscopfa digestiva alta que demuestra una esofagitis distal lineal, compatible con reflujo gastroeSOfQico. La conducta ms adecuada, de entre las siguientes, es: ...,. . e{)\.(aA "1\.-"" ~ \ /@Iniciar omeprazol ~ ~ e-"ru~""' ()j ~ ~~~ "-"t!O\ AJ. UJU k.l c.A -.1 b) Realizar tratamiento erradicador para Helicobacter pylori e) Solicitar pH-metrla esofgica de 24 horas d) Solicitar manometrla esofgica r.r:.. e) Diagnosticar dispepsia funcional y realizar educacil"l-:\)lr\ ~~t: ~~ ~ ~ ~ e. O.~
L '

9) La causa ms frecuente de elevacin persistente de las transamlnasas en Chile es: @) Alcohol -tao~~~ t-~qj:\JT\5 O.~.<!... o) Hepatitis B e) Hepatitis C -Q d) Esteatosis heptica ~~f!. W..~~\\e.'Ol~\\T\~ e) Autoinmune

10) Cul de las siguientes historias clfnicas es ms compatibln con el cuadro clfnlco caracterstico de 'rrosis biliar primaria? -/ ujb~r de 40 aos, con astenia y prurito. con elevacin marcada de las fosfatasas alcalinas y ~!s~-:.ta de la rru 1na b) Hombre de 60 aos. con ictericia y presencia de ANCA e) Mujer de 60 aos, con antecedente de vltlli@o con elevacin pers.istente de transaminasas, sin hbitos txicos y con serologla viral negativa ~\bn ~ ~U ~,u~~ . d) Mujer .de 30 aos con Ictericia fluctuante, de predominio indirecto y elevacin de la GGT y fosfatasas alcalinas, con GOT y GPT normales e) Hombre de 60 aos con icteriacia, coluria y acolla, asociado a baja de peso -b CA~ cal. 11) El srntoma preponderante en la aca las la esofgica es: a) Regurgitacin de alimentos cidos b) Disfagia lgica de instalacin progresiva e) Dolor retroestemal durante la deglucin @'Dificultad para tragar liquides y slidos, de curso fluctuante _ e) Sensacin de cuerpo extrao, de localizacin esofgica alta

12) Un paciente de 60 aos, sin antecedentes de Importancia, presenta cuadro de vmitos egstprandiales abundantes, que suceden algunos minutos despus de Ingerir alimentos. Refiere que los srntomas Iniciaron hace 4 meses, pero que se han vuelto cada vez ms intensos, vomitando incluso luego de lngerar pequeas cantidades de alientos. El diagnstico mas pro6"'able es: ~ P~ncreatltis ?r~nica ~O_ iR,\l;'Us.o ~~\\O -c.A\J~ .. (!!}Cancer esofag1co U\lJ?"A, ,~..,::> ~ ~e) Cncer gstrico ..---.~.~'O tbt-l d) Gastritis crnica atrfica -<ol\~f~~ CAt!Ft{C:A: OU. Al e) Divertfculos esofgicos UM. ~~M Aml

e:
~
~
~ (!'
~

f!'

'l

~ ~ ~
~ ~
~

14) Un paciente d~os, consulta por un cuadro de algunos aos de evolucin, de dolor abdominal tipo clico, intermitente, asociado a..Ql!:ensln abdominal. Refiere trnsito intestinal lento, con deposlCiones duras, cada 4 a 5 dias. En el ltimo tiempo lo sintomas se han vuelto ms frecuentes, lo que1o tiene mtranqullo. El examen fsice> resulta normal. El diagnstico ms probable es: OOlQ~ .... n.l\a.t\ .\:- ~ \t- ~'N~"~() t\ 3q E\Jt\Wt\q, ~CA~~\:) a) Obstruccin intestinal b) Enfermedad celiaca \~~~ - ' 11~11\ /@Sndrome de intestino irritable d) Colitis ulcerosa e) Cncer de colon

~Ol.

~ ~
~
~

15) Un nio de...2 aos presenta diarrea acuosa abundante, de 3 dias de evolucin, con cerca de 12 deposiciones diarias. Tambin present fiebre hasta 38C durante los prrmeros das. En las ltimas horas se agreg 1rritabilida importante. Al examen fis1co presenta signos vitales normalesVJpra sfu ~ (@grima os av La conducta ms adecuada es: ttAr.\W () a) Enviar a dom1c1 10 con rehidratac1on oral segn plan A b) Solicitar coprocultivo y Elisa para rotavirus en heces y reponer volumen por sonda nasoenteral -oc) Indicar sales de rehldratacin oral, 100 cc/Kg, a tomar en 4 horas y reevaluar ~Wlll & '()\,~~ lJ.C~ d) Indicar sales de rehidratacin, luego de c:ada deposicin y antibiticos por vfa oral ~dministrar bolo de suero fisiolgico_ end10venoso 16) Un paciente inicia diarrea acuosa y d-olor abdominal clico, 2 das despys de completar un tratamiento antibitico con amoxicilina, '~ebido a una neumonra adquirida en la comunidad. Se solicitan toxinas de Clostridlum difficile !.h heces, las que resultan positivas. La conducta ms adecuada es: a) Solicitar coplrocultivo y decidir manejo segn resultado Iniciar metronidazol oral .. e) Iniciar clindamicina oral d) Reiniciar la amoxicilina, agregando cido clavulnico e) Iniciar rehidratacin oral, sin necesidad d1~ iniciar antibiticos

/Cih

~ ~ ~ (!A
~ ~

~ ~
~

17) Un paciente presenta pirosis frecuente, luego de comer, especiall'l}ente cuando come antes de ir a dormir. Tambin refiere regurgitacin cida de alimentos, varias veces por semana. El examen fsico \ es normal. La conducta ms adecuada es;; b'l \ j " ~ ~ (.,!) /@Indicar medidas generales antireflujo e iniciar un inhibidor de la bomba de protonesb) Solicitar una Ph-metra esofgica e) Prohibir el consumo de los alimentos que le producen los sntomas y controlar en 4 semanas d) Solicitar endoscopfa digestiva alta e) Solicitar manometria esofgica e iniciar bloqueadores H2

~
~

~
~
~

18) Un paciente de 60 aos presenta emfs;in de deposiciones mezcladas en el mismo dfa, ~ embargo las depOSciones se norma rzan en los das siguientes. No ha presentado otros sntomas y el examen fsico no apm1a mayor informacin. La conducta ms adecuada es: a) Solicitar prueba de hemorragias ocultas ~ Realizar colonoscopa,. e) Solicitar antgeno carcinoembrionario d) Solicitar ecografla abdominal._ e) Observar y controlar en un mes

19) El tratamiento de la lcera duodenal no complicada suele ser: -1> b) Antibiticos e inhibidores de la bomba de protones por 10 das e) Antiinflamatorios no esteroidales d) lnyectoterapia endoscpica e) Quirrgico

@ Omeprazol a permanencia

~
~

~ fiA
~
($\
~

20) El factor pronstico ms importante eu el cncer gstrico es: a) El tipo histolgico del tumor b) El tamao del tumor' _.el La presencia de adenopatfas metastsicas W El grado de invasin transmural del tumor e) La presencia de anemia

~
~

Prueba 4 GASTROENTEROLOGIA 1) Un paciente de 44 aos presenta dolor abdominal epigstrico Intenso, constante, de 24 horas de evolucin, asociado a vmitos alimentarios en varias o ortunidades. Se aprecia muy adolorido, sudoroso, taquicrdico con tendencaa - a ti po ens on. Se observa un rea de ~J~osis er 1. a domen es blando, doloroso a la palpacin, c:on signos de ir:[l@.g__o_peritoneal y isminucin marcada de los ruidos hldroareos. El diagnstico ms probable es: a) E.mbolia mesentrica b) Trombosis de vena mesentrica e) Estallid.o heptico d) PerfQrcin de lcera pptica ~al")reatitis aguda, 2) paciente consulta por dolor epigstrico fluctuante, de carcter urente, que suele ser ms intenso elites perodos de ayunas. El examen fsico es normal. La conducta ms adecuada es: Iniciar tratamiento con omeprazol . rO~ . .Realizar erradicacin de Helicobacter pylori e) Soljcitar pH-metr[a esofgica ~ \)\<:L. ~iicitar endoscopfa digestiva alta e) .. Realizar educacin y controlar en un mes

V~

en otras oportunidades ha pr~sentado cuadros similares. Refi~re que sus deposiciones son abundantes, con gotas de grasa. La conducta ms adecuada .es: a) Solicitar colonoscopfa "lll..solicitar test de Sudn en deposiciones ./ e) irticiar dieta sin gluten d) Solicitar test de 0-xilosa e) Solicitar endoscopla digestiva alta

a) Un paciente consulta por diarrea de 4_emanas de evolucin. asociado a baii..dft_p_ug. Refiere que

4) Un paciente de a2_ios presenta cyadro de diarrea, de 2~ hcoras de evolucin, asociada a dolor abdominal.:![po clico y fiebre hasta ssocJsu esposa tiene sfntomas similares, pero menos intensos. Al examen se aprecia en buenas condiciones generales, con s:ignos vitales normales, hidratado. El abdomen se palpa blando, depresible, con aumento de los ruidos hjdroareos. La conducta ms adecuada es: a) Iniciar sales de rehidratacin oral y antibiticos . ~ b) Indicar ingesta de Jql:lidos-abur.u;JaRtes-y-aRti~l3aSfftdicos-en-e:ase-de-Eielor e) Indicar ingesta abundante de liquidos..e..iRiciar-aRtidiarreisas1 antiespasmdicos y antibiticos ~Jndicar hidratacin oral, antidiarreicos y antiespasmdicos en cctso de dolor ~ :>endicar hidratacin oral y solicitar coprocultivo 5) Una paciente consulta por cuadro de dolor en hipaconddo.derecho intenso, asoc!g{.fj)_S .vmitoSalimentarios en 4 oportunidades. Se encuentra en buenas condiciones, y en su examen ffsico destaca dolor..al RSipar la zona, sin irritacin peritoneal. Se solicita ecografa abdominal que demuestra ve~rcula biliar con paredes edematosas y de grosor aumentado y barro biliar, con visualizacin qe un clcuo de 2 cm, impactado en eJ bacinete. La amilasa y pruebas hepticas son normales. La conducta adecuada es: " Ad~inistrar analgsicos y realizar colecistectomla laparscpica bajo cobertura antibitica, de inmediato b) Iniciar antibiticos endovenosos de amplio espectro y programar colecistectomla para 15 d[as ms e) Administrar suero fisiolgico, antibiticos y antiespasmdicos y solicitar colangiopancreatografia retrgrada endoscpica . d) Iniciar analgsicos endovenosos e indicar rgimen cero e) Iniciar antiespasmdicos y AINEs e indicar rgimen estricto sin grasas 6) Cul de las siguientes medidas aconsejara a un pac~.~~e _c;;Q.n refluJQ_9-.~trQ~~-Q.f!gico? Evitar usar hipnticos - Beber agua antes de dormir x e) Erradicar Helicobacter pylori, si es que lo porta ~ d) Inclinar la cama, de modo que los pies quede ms altos que la cabeza 4 e) Beber moderadas cantidades de alcohol junto con las comidas
)/

\J
.
~

~~ \) .

A . J

~
7) Un paciente de 35 aos presenta dc)lor abdominal y vmitos. En sus exmenes destaca una elevacin importante de la lipasa plumti.ca. La ecografa abdominal y las pruebas hepticas son normales. Est en buenSCondiciones pero adolorido. la conducta ms adecuada es: Indicar rgimen cero e iniciar analgesia e hidratacin endovenosa b} Iniciar AINEs y antibiticos de amplio espectro, con rgimen hidrico -~Indicar rgimen cero y solicitar colagiografia retrgrada d) Solicitar resonancia magntica nuclear de abdomen y pelvis e) Resolver quirrgicamente a} El diverticulo de Zenker se produce por un mecanismo de pulsiri -~ b Los diverticulos dis suelen trat se con reseccin del diverticulo ms esfinterotomla esofgica inferior ){' e diagnostican ediante 1 e presentan con disfagia, regurgitacin no cida y en algunas ocasiones aspiracin e} Los diverticulos del cuerpo esofgico 8Uelen ser asintomticos
~

r'

r'
~
~

~ ("'

8) Respecto a los divertculos esofgic:os es FALSO que:

~ ~ ~

.1

f!!'
~

idos ~ ~ JJ.~ b)Arr 0~ e alletas d) Tortas e) Cerveza

9 Qu alimento puedE\ consumir l~brE!mente un paciente con enfermedad ceUaca?

()L,-..~.
.\})o..~ u:-~~

vu

~
~ ~
/
r "

f!?'
~
~

1O) Una paciente de 24 aos presenta el olor abdominal y diarrea de 4 meses de evolucin. En el ltimo tiempo ha empeorado, siendo muy frecuente y_yolvindose disentrica. Refiere adems sensacin f~il y baja de peso de 5 Kg_, desde qu.3 iniciaron los sntomas. Al examen fsico est plida, con un abdomen blando y depresible, sensible de manera difusa, sin signos de irritacin peritoneal. La conducta ms adecuada es: \ , - ~ <i})solicitar colonoscopia . ~ .. .1~ ...; V"'b) Iniciar ciprofloxacino oral ~e) Iniciar metronidazol oral d) ~iciar corticoides orales Solicitar parasitolgico de deposiciones y coprocultivo

~
~

~
~
~

1) Una paciente de 30 aQ_p.r.e.se.ot-.Y~ cuadro de diarrea de larga data, asociada a distensin y malestar abd_gminal. Las deposiciones !;on .blandas de color normal, cor~.mucosidad, pero sin .~~ng~~ i alimentos sin digerir. Refiere que ha Hubrdo 3 Kg de peso en el ultimo ao. El examen ffsico no ~ porta mayor informacin. El diagnstico ms probable es: . A--t, J..a) Colitis viral vr-\ v b) Colitis bacteriana ~~ ~ e) Enfermedad celiaca d) Colitis ulcerosa wcolopatia funcional
12) El objetivo del tratamiento erradicadllr de Helicobacter pylorl, en los pacientes con lcera pptica, es: a)....,Acelerar la cicatrizacin ~isminuir el riesgo de cncer gstrico e) Disminuir las recurren cas de la lcera d) PreVeir las hemorragias e) Paliar Jos sfntomas

:p

--

.J

~X
~
~

~
~
~ ~

13) Uri paciente de 33 aos inicia cuadro de disfagia, de localizacin~, con sntomas Intermitentes y que se relacionan tanto con la jnges.tade.Jlguidos, como de-slidos. En algunas oportunidades presenta regurgitacin_no cida. El examen fsico es normal. La conducta ms adecuada es: al Realizar endosco)i8digestiva alta . @Solicitar manometria esofgica . r--"- 1 e) Solicitar pH-metria esofgica de 24 horas c._e.._...Lt r'--" ~ ~niciar tratamiento con omeprazol ~ -~Solicitar radiografias baritadas V.

t\

fP'

~
~

.b

.. /

14) Un paciente con cirrosis heptica por consumo de alcoj)QI.,..pr.es~nta....hematemesis, seguida de ortostatismo. Al examen ffsico se aprecia plido, con FC: s;, 1iono) T 0 : 36,4C, abdomen blando depresible, con ascitis clnica moderada. La conducta ms ader~uada es: a) Administrar cristaloides y omeprazol endovenosos eponer la volemia-c"On sueroiis,-o""l-grholc'"o.-Ty.,..,rllllle~ar.:hz;::a::;:r.,tr::;;a:;:;n-;;:sfr;-;u;-;:-strxo:;:;n~a:r.::e::-g:;rl,.,ob~u:-:1'1~os~rojos dministrar coloides endovenosos y solicitar endoscopla digestiVi3 alta Administrar suero fisiolgico y terlipresina por vi a endovenosa y E:ollcitar endoscopta digestiva alta e) Administrar cristaloides y glbulos rojos y solicitar hemograma y endoscopfa digestiva alta

JA!

( .. \

\\..j

previamente sano, constipacin desde hace 3 semanas, con 15) Un paciente deposiciones pequeas que en 'IR par de ocasjones han mostrado estrfas de sangre. No refiere otros sfntomas y el examen ffslco es normal. La conducta ms adecuada es: a) Solicitar test de sangre oculta en deposiciones ~ _ (fiv---().1'b) Solicitar ecografa abdominal ~c)..f-A \ e) Solicitar TAC de abdomen y pelvis ~S ~ d) Solicitar colonoscopia e) Iniciar dieta rica en fibra y controlar en 7 dias

d~s,

pr~senta

Jl-J

16/La complicacin ms frecuente del cncer de colon es:


) Perfo~acin e) Fistulas e) Infeccin paciente deCs, sin antecedentes de importancia, presenta cuadro de vmitos p st 'alas abundant@S, que suceden algunos minutos despus_c!~-~erir alimentos. Refiere que 1 s sfntomas iniciaron hace 4 meses, pero que se han vuelto cadaVez ms intensos, vomitando ncluso luego de ingerir pequeas cantidades de alientos. El diagnstico ms probable es: c. CA~ a) Pancreatitis crnica .._, \ G-....r-"1 J ~ ~_) b) Cnceresofgico ~ r:o~ ~~.Jr.::~,-4-e~ 'j \ 'O~ c\..c.fo, cru~ ~ncer gstrico ~ d) Gastritis ~rnica atrfica - ~ el a; ~rd= .

rHemorragia digestiva

~struccin

17)~n

dl

l,:;

~ ~
~

e) Divertlculosesofgicos

2(" ~~grafla abdominal


-. (r} Colofloscopfa

18) Una mujer de 66 aos presenta un cuadro de dolor abdominal e 'i!ri'IJJa izquierda, de 7 horas de evolucin. Al examen tiene un abdomen blando depresible, doloroso, c~tt sagnos de Blumberg en la zona. Le mejor alternativa para evaluar a esta paciente es: a) Hemograma y PCR e .~.;h <:.. 1

~on;.!nte ~~~\ya

i;; %

.JI ~Q {;f !J61b\L.....,

o\.A,....o~v~
~

"'~

~.,.._ . {.:.. ~

e) E~pforar quirrgicamente
1

A
.

La causa ms frecuente de hemorragia digestiva baja es: Angiodisplasias ----------Cncer de colon . ~nfermedad diverticular.., --d}t>lipos de colon e) Colitis isqumica 20) Un de 45 aos, con antecedente de por un cuadro de dolr abdomif!.!l intenso, tipo clico, asociado avomltos allment~ distensin abdominal. Al rfi'en ffsico aprecia en buenas condiciones generales, con abdomen doloroso dif~..P_y_um~nt ~. inten.s.idad..de los ruidos hidroareos, sin signos peritoneales. El diagnstico ms probable es: 1 Pa"Creatitis aguca--- --- Coledocolitiasis residual tl Colitisisquemtca @'Obstruccin intestinal por bridas ~astroenteritis aguda

p~ente

c'e~~~~~~;~--~~~B~s, consu~

se

~
RESPIRATORIO ) Un paciente d~ 68 aos, sin antecedentes de importancia, consulta por cuadro de tos, con expectoracin mdcopUrulenta, asociado a fiebre hasta 39C y malestar. Al examen se aprecia en buenas condiciones generales, con FQ;..~, PA: 110/60, FR: 18 rpm, t 0 : 38C, Sat02:;..9.5.% a Fi02 ambiental. La auscultacin pulmonar muestra crepites en la base derelia:S"e solicita radiografia de trax que muestra una zona de condensacin en el lbulo inferior derecho. El tratamiento de eleccin
~Amoxicilina

\.e:

e
~

b) Claritromicina e) Ceftriaxona d) Amoxicilina + cido clavulnico e) Clindamicina 2) Un paciente de 61 aos, con antecedente de EPOC, en tratamiento con corticoides y broncodi1atadores inhalados presenta aumento de su disnea basal, asociado a fiebre y expectoracin abuni~I examen se aprecia orientado, taquipneico, con FR~', F~: 91x', PA: 140/100 mmHg, SatO : 80o/~ ~)Fi02 ambiental. Se ausculta disminucin del murmullo pulmonar, sibilancias intensas y este DJ:es-bilaterales. La conducta ms e~decuada es: a) Administrar salbutamol y corticoldes inhalados, oxigeno a bajas dosis e Iniciar antibiticos de amplio espectro b) Realizar nebulizacin con salbutamol, administrando oxigeno a bajas dosis, sin necesidad de corticoides ni de antibiticos e) Administrar oxigeno en alta concentracin, realizar nebulizacin con corticoides e iniciar antibiticos orales ~Administrar oxgeno al 24%, realizar nebulizacin con salbutamol, administrar corticoides sistmicos e imciar antibiticos de amplio espectro endovenosos .)!f. A~ministrar oxgeno a 2 litros por minuto, realizar nebulizacin con corticoides y broncodilatadores de corta accin e iniciar antibiticos endovenosos 3) Un paciente de 30 aos, no fumador, con antecedente de netJmGRia pn~norirus, hace varios aftos presenta tos diaria, con abundante expectoracin, de algunos aos de evo uc1or. La exP, ctoracln es mayor en las maanas :v aumenta con el decbito. En algunas ocasiones ha pr. sentado hemoptisis. El examen pulmcnar no aporta mayor Informacin. Qu examen es el mejor "'- ara hacer el diagnstico? a) Radiografa simple de trax b) TAC de trax ~ibrobroncoscopia d~iopsia pulmonar e) Baciloscoplas 4) Un paciente de 30 aos presenta tos con el ejercicio y en varias ocasiones tambin presenta disnea, asociada a respiracin sibilante. Presentat adems tos nocturna frecuente y suele hacer crisis obstructivas durante las infecciones viralas. En el examen fisico se aprecia en buenas condiciones y .._/ destaca slo una leve prolongacin del tiumpo de espiracin. Se solicita una espirometria que ~ confirma la presencia de asma bronquial. El tratamiento ms adecuado para este paciente es: a) Salbutamol en caso de slntomas ");:12)-'Salmeterol en caso de sintomas y corticoides inhalados segn horario "':'JF1 Salbutamol en caso de sintomas y corticoides inhalados, con salmeterol, segn horario ,--ti) Budesonida en caso de sintomas y salbutamol segn horario e) Salbutamol y salmeterol en caso de sintomas y corticoides orales segn horario

e e e
~ ~
~
~

~
~ ~
~

~ ~ ~
~
~

~
~
~

~
~

~
(IQ

lun paciente de 72 tos con expectoracin mucosa, desde hace 4 meses, sociado a disnea de esfuerzos. En su examen flsico destacan uas en vidrio de reloj. El examen o<..::...:.------ardiopulmonar es normal. El diagnstico ms probable es: a) Derrame pleural ~Cncer bronquial e) Bronquiectasias d) Tuberculosis pulmonar e) Enfennedad pulmonar obstructiva crnica

aos...f!!!!!ador,.pres~>nta

f!FA {C\

~
(!f$h

~
~

1/

J:

Una paciente de 52 aos, fumador, presenta disnea de esfuerzos progresiva, de 6 meses de evolucin. Al examen f(sico se auscultan crpitos bilaterales. La radiografa de trax muestra un patrn en panal de abejas en ambas bases. El diagnstico ms importante es: a) EPOC ~ibrosis pulmonar e) Edema pulmonar d) Neumoconiosis e) Neumonla por mycoplasma 7) Un paciente de 20 aos ingresa por una neumona, complicada con un derrame pleural paraneumnic<;>. Al examen fsico est en buenas condiciones, uupneico, con signos vitales normales. El examen pulmonar es compatible con un derrame pleural pequeo. Se solicitan gases de sangre erial con los siguientes valores: Pa02: 38 mrriHg, Pa~02: 47 mmHg, HC03: 25 mEq/1, pH: 7,39. El agnstico ms probable es: -------Insuficiencia respiratoria global severa Insuficiencia respiratoria parcial, con alcalosis respiratoria e) Acidosis respiratoria, con alcalosis metablica d) Insuficiencia respiratoria grave, con acidosis metablica y alcalosis respiratoria "Gases venosos

V
~

./

8)EI.efecto dverso ms frecuente del tratamiento antituberculoso es: a)Aiergia ~Iteraciones hepticas e) Anemia OtfDao renal e) Alteraciones visuales

9) Los nios con fibrosis qu[stica suelen contraer neumonias c:ausadas por: a) Mycoplasma pneumoniae _ __):1}.1:egioneJI&f)eumanjae (-~Streptococcus pneumoniae .l~d)Peudomona aureginosa e) Streptcoccus pyogenes 1O) Un recin nacido de 30 semanas de edad gestacionallnicia un cuadro de dlficylt.a.d respiratQria grpv~ a .IQ~. p~co_~_ _!llinutos de vida, con taqul~a, y ca~a"lmRortante de la satu_racin artenal, que reqUietEtalt~s::ca_ncentraciones de oxrgeno para alcanzar el90%. El diagnstico ms robable es: ~ ) Taquipnea transitoria ~ / ) Cardiopatfa congnita " ) Circulacin fetal persiste.nte . ..d)...Sfndrome aspirativo meconial /, )e(.~nfermedad de membrana hialina
//

11) Un paciente consulta por un cuadro de tos, con expectoracin, fiebre y dolor tipo,{untada de costado en el hemitrax izquierdo. La radiografa de trax demuestra un derrame pleural moderado, asociado a una zona de relleno alveolar e!J_.e -lpbulo inferior izquierdo. Se realiza puncin del lquido que demuestra clula_!1 4.0-pnr mm3, cor(.86 <:te polimoonucleares, LDH: 2~.111, proternas: 4,4 ... mg/dl, ADA: 28 Ul}l, lactato: 3.~ mg/dl y p . $La conducta ms adecuada&s:-- . a) Solicitar biopsias pleurales por"pUiitln pef utanea / 'Wfnstalar tubo pleural e iniciar antibiticos endovenosos '-'' e) Administrar antibiticos intrapleurales d) Iniciar tratamiento antituberculoso . ~nielar antibiticos endovenosos y evacuar el derrame pleural por toracocentesis

)
1

12?!:) paciente de 45 aos sufre un accidente de trnsito, mientras conducra su vehculo, resultando co un golpe en el trax contra el manubrio. Evoluciona con dificultad respiratoria. Al examen se a9 ecla taquipneico, bien perfundido, con presin artiD]al y frecu!l).c.ia..car.dlac.a_l)ormales. La .J~scultalJtn cardiaca es normal y el examen pulmonar slo demuestra crpitos escasos difusos. El diagnstico ms probable es: :)contusin pulmonar: b) Hemotrax e) Neumotrax d) Trax volante e) Crisis de angustia 13) Son causa de distrs respiratoriQ _todas las patologas nombradas a continuacin, excepto: epsis severa de origen urinario;:::/' -~ nnacos /, ancreatitis, aguda -~ ~ Infarto extenso de ventriculo izquierdo -~ ) Quemaduras extensas .._,/

f!!'

~
~, a"
\.
~

14) Un adolescente de 13 aos presenta <:uadro de tos, rinorrea, odinofagia de 12 dfas de evolucin. En las ltimas 48 horas se produce un aumento de la tos y se agregan malestar general y disnea de 1 esfuerzos. Al examen fsico destaca fiebre de 38C y examen pulmonar con crpitos bibasales, 1 mayores a derecha y sibilanclas escasas. El resto del examen no aporta mayor informacin. Se solicita radiografia de trax que confirma un patrn alveolointersticial bilateral, mayor en la base derecha. El tra~miento ms adecuado es: ~tr")fCiaritromicina ~~UJ--b) Clindamicina tJCT \ e) Amoxicilina d) Amoxicilina + cido clavulnico e) C~flosporina de tercera generacin

1
15) El tratamiento de eleccin para el absceso pulmonar es:

?~Ciruga

~
~

vl
1

~ ~
~ ~

-'-.j/b) Paciepte con EPOC estable que presenta gases arteriales con Pa02: 51 mmHg y PaC02: 40 mmHg e) Paci~nte que contina fumando -~ d) Paciente con EPOC estable que presenta poliglobulia, Pa02: 65 mmHg y PaC02: 38 mmHg e) P~iente con EPOC estable, con VEF1 correspondiente al43% del valor terico esperado
1

16{~cul de los~-~~~~~~t:s,~aclen~e~ ~:~--~nfermedad pulmonar obstructiva crnica tiene indicacin d~ oxfgenq 1 domiciliario a permanencia, pua aumentar la sobrevida? . p) Pacient de 67 aos, con sntomas muy marcados, con disnea al caminar 3 metros

b) Amoxicilina + cido clavulnico oral e) Dren~j por puncin transtorcica y metronidazol oral d) Ceftriaxona endovenosa e) Clirtdamicina endovenosa y drenaje postura!

~
~
~

J>

~ ~

17)1Un paciente de 67 aos, asintomtlco, fumador, se realiza una radiografa de trax, como parte de un/conjunto de exmenes de chequeo, exigidos en su trabajo. Se identifica un ndulo pulmonar dpnso, de 3 centmetros en el lbulo superior izquierdo, espiculado. No cuenta con radiografas previas. La conducta ms adecuada es: Solicitar nueva radiografa de trax en 6 meses b) Solicitar nueva radiografa de trax en un mes 'PlSolicitar TAC de trax, con cortes finos a) .Realizar biopsia transbronquial, por fibrobroncoscopia e) !Realizar lobectomia superior izquierda
r

e
~

(!$'

~ ~
~

1~) El diagnstico del sndrome bronquial obstructivo del lactante se diagnostica mediante: . J:1f La clnica solamente '\._/b) La cllnica + radiografia de trax e) La clnica + espirometra d) La clnica+ test de metacolina e) La clinica +test cutneo para alrgenos respiratorios

~
~

,
~
~

~
19) Un PJfe de 45 aos presenta un cuadro de malestar genetal, rlnorrea acuosa y odlnofagla, de 2 das d~_volucln, al que se le ha agregado tos, con escasa expectoracin mucosa. Presenta signos vitaleP. normales, faringe congestiva, sin exudado y examen cardiopulmonar normal. El agente ms able es: tcoplasma pneumoniae hlnovirus e) Virus influenza d) Streptococcus pneumoiae e) Streptococcus pyogenes 20) Cul de las siguientes esplrometrfas es compatible con el diagnstico de EPOC? a) Basal CVF:105%, VEF1:71%, VEF1/CVF:61%; Post-salbutamol CVF:110%, VEF1:72%, b} Basal CVF:91%, VEF1:66%, VEF1/CVF:54%; Post-salbutamol CVF:109%, VEF1:94%, gJ Basal CVF:SO%, VEF1:66%, VEF1/CVF:86%; Post-salbutamol CVF:51%, VEF1:65%, ~ Basal CVF:88%, VEF1 :88%, VEF1/CVF:80%; Post-salbutamol CVF:89%, VEF1 :95%, e) Basal CVF:70%, VEF1:74%, VEF1/CVF:77%; Post-salbutamol CVF:70%, VEF1:74%, VEF1/CVF:60% VEF1/CVF:74% VEF1/CVF:84% VEF1/CVF:85% VEF1/CVF:76%

,
~

~
~

....,

~
~
~ ~

~
~ ~
~

~
~

.,
~

,.,
~

~
~

-,
~

~
~

~ ~
~
~

RESPIRATORIO
1) U~r"paciente de 68 aos, sin antecednntes de lmportancii!(.. consulta por cua. dro de tos, con ex ctoracin mucopurulenta, asociado a fle"re nasta_~ . .y malestar.~;!men se aprecia en b enas condiciones generales, con FS~', PA: 110/60, FR: 'l.[Jpm, t~ 38C~at02: 95% a Fi02 biental. La auscultacin pulmonar muestra c~itos en la base derec~e solicita radiografa de trax que muestra una zona de conden!;acin en el lbulo mferior derecho. El tratamiento de eleccin es: ,.,a} Amoxicilina b) Claritromicina e) Ceftriaxona ~. Amoxicilina + cido clavulnico r f/) Clindamicina 2) Un paciente de 61 aos, con antecedtmte d~C, en tratamiento con corticoides y.. broncodilatadores inhalados presenta aumento de su disnea basal, asociado a fiebre y expectoracin abundante. Al examen se aprecia orientado, taquipneico, con FR: 27x', FC: 91x', PA: 140/100 mmHg, Sato~(SO% a Fi02 ambiental. Se ausculta disminucin del murmullo pulmonar, slbilancias intensas y esJ~rlores bilaterales. La conducta ms adecuada es: ~.dministrar salbutamol y corticoides inhalados, oxigeno a bajas dosis e iniciar antibiticos de amplio pectro Realizar nebulizacin con salbutamol, administrando oxigeno a bajas dosis, sin necesidad de corticoides ni e antibiticos 'AE!ministrar oxgeno en alta concentraci~realizar nebulizaein-eon-corticoides-einiciarantibitieos-erales T\.d)..Administrar.oxgeQQ_?!~~yo, realizar,nebulizacin con salbutamol, administrar corticoides sistmicos e -?'nciaraAtibfOlcos de amplio esp-ectro-endovenosos
' e} Administraf-6*igeRG-a-2-litr.gs..por-mir:~uto,..reati~:..r.~ebuliz-acin-con-corticoides-ybroncodllatadores\:fe~orta '

~ ~

~ ~ ~
~

~ ~

. e A
~
~ ~

~ ~

aGGin_e_iniciac.ar:ttibitiees-endeveneses-3) Un paciente de 30 aos, no fumador, (:on antecedente de neumona por adenovirus,~hace varios aos, presenta tos diaria, con abundante expectoracin, de algunqs-af\dErde-volucin. La expectoracin es mayor en las mafia nas y aumenta con el decbito. En algunas ocasiones ha prJ!~.~~'?. hemoptisis. El examen pulmc>nar no aporta mayor informacin. Qu examen es el mejor para hacer eT1iagnost1co? .Z:~adiografia simple de trax ,-~TAC de trax )( e) Fibrobroncoscopia d) Biopsia pulmonar e) Baciloscopias
4) Un paciente de 30 aos presenta tos c>n el ejercicio y en varias ocasiones tambin presenta disnea,

~
~

~
~ ~

~ ~
~

~
~
~

,>(

asociada a respiracin sibilante. Presenta adems tos nocturna frecuente y suele hacer crisis obstructivas durante las infecciones virares. En el examen frsico se aprecia en buenas condiciones y destaca slo una leve prolongacin del tiempo de espiracin. Se solicita una espirometra que confirma la presencia de asma bronquial. El tratamiento ms adecuado para este paciente es: a) Salbutamol en caso de sintomas 1 ~-s'atmeterot en caso de sintomas y corticoides inhalados segn horario '-/ ~ Salbutamol en caso de sintomas y corticoiaes ih"llladOS-Cort salrneterol, segn horario d~f:leeSGRida..eJ+.GaSe~e-sh ttOIIISS y saltmtamot-segtm-her-ario . e) Salootamol..;t-Salme\er-el-eR-sase-Eie-simerAas-y-eeFtlselGes-erales-segMar.ar.io...:l 5) Un paciente de 72 aos. fumador, presunta tos con expectoracin mucosa, desde hace 4 meses, asociado a disnea de isfuerzos. En su ex;imen trsico destacan uas eVfefo de reloj. El examen cardiopulmonar es nonnal. El diagnstico ms probable es: a) Derrame pleural __:b). Cncer bronquial __. ) Bronquiectasias ~)uberculosis pulmonar /. r Enfermedad pulmonar obstructiva crnica .._,....
V

e
~
~

><

'-~,

~ ~ ~
) Una paciente de 52 aos, fumador, presenta disnea de esfunrzos progresiva, de 6 meses de evolucin. Al examen fsico se auscultan crtpitos bilaterales. La radiografa de trax muestra un patrn en pa.nal de abejas en ambas bases. 1diagnstico ms Importante es:
~:? ~ibrosis pulmonar~

~
~
~

at~POC

G) Edema pulmonar d) Neumoconiosis e) Neumonfa por mycoplasma 7) Un paciente de 20 aos ingresa por una neumonra, complic::.da con un derrame pleural les. paraneumnico. Al'examen ffsico est en buenas condiciones, eupneico, con signos vj!aleS-Ao~. El examen pulmonar es compatible con un derrame pleural pequef\o. Se solicitan rus d~ng.~:e arterial con los siguientes valores: Pa02: 38 mmHg, PaC02: 4'7 mmHg, HC03: 2 mEqn, pff: 7,39):1 diagnstico ms probable es: . . ~~ a) nsuficiencia respiratoria global severa ~ respiratoria parcial, con alcalosis respiratoria J . e) Acidosis respiratoria, con alcalosis metablica !]-

,._,
~
~
~

~ ~

-~nsuficiencia

""'
~

Gases venosos 8) El efecto adverso ms frecuente del tratamiento antituberculoso es: a) Alergia ~lte~aciones hepticas
c)A~mla

~
~

d)

ano renal Iteraciones visuales

~
~ ~

os nif\os con fibrosis qu[stica suelen contraer neumon[as causadas por: a) ~coplasma pneumoniae .AJ)Legionella peumoniae . < e} ~treptococcus pneumoniae ~eudoniona aureginosa e) Stre lococcus pyogenes 10) n recin nacido de 30 semanas de edad gestacionalinlcla un cuadro de dificultad respiratoria gr ve a los pocos minutos de v1da, con taqUipnea marcada, y calda importante de la saturacin a erial, que requiere altas concentraciones de oxgeno para alcanzar el 90%. El diagnstico ms robable es: a} Taquipnea transitoria b} Cardiopatia congnita e} Circulacin fetal persistente d) Sindrome aspirativo meconial (""e'f Enfermedad de membrana hialina 11) Un paciente consulta por un cuadro de tos, con expectoracin, fiebre y dolor tipo puntada de costado en el hemitrax izquierdo. La radiografa de trax demuestra un derrame pleural moderado, asociado a una zona de relleno alveolar en el lbulo inferior l:zquierdo. Se realiza puncin del trquido que ~~muestra clul"~ 740 por mm3, con 86% de polimoom~s, LDH: 220 Ul/1, proterna~.!_ mg!,dl, ADA: 28 Ul/1, lactato: 3,3 mg/dl y pH: 7,28. La conducta ms adecuada es: olicitar biopsias pleurales por punc16n percutnea ~ 'Qv--_,___.o , nstalar tubo pleural e iniciar antibiticos endovenosos Administrar antibiticos intrapleurales ) nielar tratamiento antituberculoso . .Ainiciar antibiticos endovenosos y evacuar el derrame pleural por toracocentesis

~
~

~ ~

..,
~
~

~
~

~ ~

~
~

~
~

~ ~
~

~
~
~

""
Al)

1 ) Un paciente de 45 aos sufre un accidente de trnsito, mientras conducfa su vehculo, resultando on un golpe en el trax contra.el manubrio. Evoluciona con dificultad respiratoria~ Al examen se aprecia taquipneico, bien perfundido, con p~ln arterial y frecuencia cardiaca normales. La auscultacin cardfa~a es normal y el examen pulmonar slo demuestra crpitos escasos difusos. El dia9Jlstico ms prot)able es: ?.!Contusin pulmonar b) Hemotrax e) Neumotrax d) Trax yolante e) Crisis/de angustia 1 13) on causa de distrs respirato~odas las patologfas nombradas a continuacin, excepto: a epsis severa de origen urinario ) Frmacos .../ e) Pancreatitis aguda ..../"" ~)1farto extenso de ventriculo izquierdo e) Quemaduras extensas._.....-

e
e!'

14) Un adolescente de j_3 aos presenta <:uadro de tos. rjnorrea~o~inofagia d~ 12 das de evolucinJ En las ltimas 48 horas se produce un aumefl!oy! la tos y se agmgan malestar general y disnea_d.~- esfuerzos. Al examen fisico destaca fiebre qe 3SOO y examen pulmonar con crpitos blbpales,mayores a derecha y sibilancias escasas. El del examen no aporta mayor informacin. Se solicita ~~ radiografa de trax que confirma un patrn alveolointersticial bilateraJ1 mayor en la base derecha. El -. tratamiento ms adecuado es: ,, o \/)~-"-Y'.. ~_ ~) Claritromicina ,7 O"J - '\.., ~ -- .r. .16) CI!Jldamicina _c).-A'moxicilina ~ d) Amoxicilina + cido clavulnico e) Cefalosporina de tercera generacin

resto

~
~
JIG\.

~1 ~h~~~tamiento de eleccin para el ~ceso pulmonar es:


moxicilina + cido clavulnico oral renaje por puncin transtorcica y metronidazol oral \ ~~eftriaxona endovenosa -~ vlindamicina endovenosa y drenaje postura! 16) Cul de los siguientes pacientes con enfermedad pulmonar obstructiva crnica tiene indicacin de oxgeno domiciliario a permanencia, p;ua aumentar la sobrevida? a) Paciente de 67 aos, con sfntomas muy marcados, con disnea al caminar 3 metros ...:>b) Paciente con EPOC estable que presenta gases arteriales con Pa02: 51 mmHg y PaC02: 40 mml:lg ../ le) Paciente que contina fumando ,.d)'"Pacie.nte con EPOC estable que presenta poliglobulia, Pa02: 65 mmHg y PaC02: 38 mmHg e) Paciente con EPOC estable, con VEF1 correspondiente al 43% del valor terico esperado 17 n paciente de 67 aos, asintomtico,.f.!:!mador, se realiza una radiogratra de trax, como parte de u-' conjunto de exmenes de chequeo, exigidos en su trabajo. Se identifica un o.dulo pulmonat:.. enso, de 3 centfmetros en el lbulo superior izquierdo, espiculado. No cuenta con radiografras previas. La conducta ms adecuada es: a) Solicitar .nueva radiografta de trax en 6 rr.eses b) Solicitar nueva radiografta de trax en un mes .....e)"'Solic;t~r TAC de trax, con cortes finos d) Re.aiizar biopsia transbronquial, por fibrobmncoscopfa . alizar lobectomla superior izquierda El diagnstico del sndrome bronquial obstructivo del lactante se diagnostica mediante: a clnica solamente b) La clnica + radiografia de trax e) La clinica + espirometria d) La clnica + test de metacolina e) La clnica+ test cutneo para alrgenos respiratorios

' -

~ ~
~

~;>(
~

~
~

~
~

~ ~
~
~

~ ~ ~
__ , ~-

,.,
~

~
~
19 Un paciente de 45 aos presenta un cuadro de malestar general, rlnorrea acuosa y odinofagia, de 2

di s de evolucin, al que se le ha agregado tos, con escasa expectoracin mucosa. Presenta signos v les normales, faringe congestiva, sin exudado y examen Citrdiopulmonar normal. El agente ms ro able es:
icoplasma pneumoniae Rhinovirus 1 Virus influenza d) Streptococcus pneumoiae e) Streptococcus pyogenes

~
~

~ ~
~
~

Cul de las siguientes espirometrfas es compatible con el diagnstico de EPOC?


sal CVF:105%, VEF1:71%, ) sal CVF:91%, VEF1:66%, e) Basal CVF:50%, VEF1:66%, d) Basal CVF:88%, VEF1 :88%, e) Basal CVF:70%, VEF1:74%, VEF1/CVF:61%; VEF1/CVF:54%; VEF1/CVF:86%; VEF1/CVF:80%; VEF1/CVF:77%; Post-salbutamol CVF:110%, Post-salbutamol CVF:109%, Post-salbutamol CVF:51%, Post-salbutamol CVF:89%, Post-salbutamol CVF:70%, VEF1:72%, VEF1:94%, VEF1:65%, VEF1 :95%, VEF1:74%, VEF1/CVF:60% VEF1/CVF:74% VEF1/CVF:84% VEF1/CVF:85% VEF1/CVF:76%

~
~

~
~

~
~
~

~
~
~

~
~

~
~
~
~

~
~

~
~

.-, ...,
~

~ ~
~
~

~
~

41)

~.

RUT

fA
~ ~
~
NOMBRE:

o
1
2 3
4

~ ~ ~ ~ ~ ~ ~ ~ ~

RUT:

6 7
8

FECHA:

00 00 00 00 00 00 00 00 00 00

000 000 000 000 000 000 000 000 000 000

000 000 000 000 000 000 000 000 000 000

o o o o o o o

o o o o
O E

CURSO: RESPUESTAS A
1
2 3 4

B C

O E
31 32 33 34 35 36 37 38 39 40

B C

A
61 62 63 64

B C O E

e
~

~ ~
~ (!A

5
6 7
8 9

~ f'A
~
~

10

00000 00000 00000 00000 00000 00000 00000 00000 00000 00000

00000 00000 00000 00000 00000 00000 00000 00000 00000 00000 00000 00000 00000 00000 00000 00000 00000 00000 00000 00000 00000 00000 00000 00000 00000 00000 00000 00000 00000 00000

65
66 67 68 69 70

00000 00000 00000 00000 00000 00000 00000 00000 00000 00000

~ ~ ~ ~
~

f!!'
(~"'

00000 12 00000 13 00000 . 14 00000 15 00000 16 00000 17 00000 18 00000 19 00000 20 00000
11
21 22 23 24 25 26 27 28

41 42 43

44
45 46 47 48 49 50

00000 00000 73 00000 74 00000 75 00000 76 00000 77 00000 78 00000 79 00000 80 00000
71 72 81 82 83

f!"
~

~ ~
~

~
~

29
30

00000 00000 00000 00000 00000 00000 00000 00000 00000 00000

51 52 53 54 55 56 57 58 59 60

84
85 86 87 88 89 90

00000 00000 00000 00000 00000 00000 00000 00000 00000 00000

~
~

tmli)
~
~
PRUEBA 1 Preguntas gastroenterologla (primera. parte}

"'

~
~

1) Un paciente de 24 aos consulta por dolor epigstrico recurrente, de caracteristlcas urentes. Usted solicita una endoscopia digestiva alta que demuestra una lcera duodenal activa, sin estigmas de sangrado reciente. El t~ de ur-easa resulta positblo. La medida teraputicas de primera lrnea es: a))Jmeprazol en dos dosis diarias, a permanencia. .--wrratamiento de erradicacin de H. pylori por 7 a 1O dias. ) Dieta estricta, evitando grasas, condimentos e irritantes y control endoscpico cada 3 meses. d) lnhibidores H2 diarios, asociados a anticidos, segn sintomas. . e) Prohibir el uso de AINEs y asociar un inhibidor de la bomba de protones en dosis nica diaria.
2) Cuatro horas despus de acudir a un banquete, 25 personas Inician sbitamente un cuadro de nuseas, vmitos y dolores abdominales. Cul de los siguientes agentes es el causante ms probable de estos sintomas? a) Clostridium botulinum. b) Salmonella e) Escherichia coli enterotoxignica 91[ Estafilococo Aureo e) Shigella 3) Cul de los siguientes cereales puede ser tomado libremente por los pacientes con enfermedad celiaca?: a) Cebada. b) Centeno e) Trigo ~Arroz e) Avena 4). u.na paciente de .54 ao~ en tratamiento con levofloxacino por ne~monfa adquirida e~ h~.. '?.o':"l-!_!1~~~~ imela dolor abdomanal y diarrea acuosa. La conducta Inicial m~1s ad cuada es: - a) Suspender ellevofloxacino e iniciar amoxicilina ms Ac. clavulnico b} Agregar metronidazol al tratamiento antibitico .fl(Solicitar toxinas de Clostridium difficile en heces d} Solicita colonoscopfa y manejar segun hallazgos e) Tranquilizar a la paciente y explicarle que es normal presentar diarrea al usar antibiticos de amplio espectro 5) La causa ms frecuente de la pancreatitis aguda es: a) Hipertrigliceridemia b) Transgresin alimentaria ttitiasis biliar d) lngesta alcohlica e) ldioptica 6) Paciente de 67 aos, fumador, refiere disfagia lgica de 3 meses de evolucin, asociado a baja de peso de 7 kg. El axamen..fisico no aporta mayor informacin. El diagnstico ms probable es: a) Acalasia esofgica A b) Divertlculo de Zenker e) Cncer incipiente de esfago d) !i,stenosis esofgica secundaria a RGE yfCncer esofgico avanzado 7) Mujer de 78 aos, hipertensa y portadora de fibrilacin auric:ular crnica, presenta dolor abdominal muy intenso, permanente, de Inicio brusco, asociado a nusec1s. Al examen ffsico se palpa abdomen blando, sin signos de irritacin peritoneal y RHA aumentados. El diagnstico ms probable es: a) Pancreatitis aguda b) Colangitis aguda ~mbolia mesentrica A) Perforacin de lcera pptica e) Absceso heptico

~
~

,_,
~
~

~
~
~

~
~
~
~

,.,

~
~

~
~ ~ ~

~
~

~
~

..,

~
~

,.,

"
~
~
~

8) En un paciente con metstasis pulmonares, pero sin afectacin heptica, por un carcinoma de origen intestinal. Cul es la localizacin ms probable del primario?:
~ecto

~ ~ ~

"'b) Colon sigmoide


e) Colon ascendente d) Intestino delgado e) Estmago 9) El factor pronstico ms importante en el cncer gstrico es: a) El tipo histolgico del tumor b) El tamao del tumor e}. La presencia de adenopatias metastsic;as ,?9"f~l grado de invasin transmural del tumor , e) La presencia de anemia 1O) Paciente de 30 aos, consulta por vmitos y diarrea acuosa de 2 d(as de evolucin, asociado a dolor abdominal y compromiso del estado general. Al examen se aprecia hidratado, con r' axilar: ~;oc, FC:831pm y PA: 130/80. La conducta ms adecuada a seguir es: A:!'Heposicin de liquidas por vfa oral .- b) Hidratacin endovenosa, asociado a ceftriaxona e) Antidiarreicos y antiespasmdicos d) Hidratacin, antibiticos y antidiarreicos orales e) Ciprofloxacino 500 mg cada 12 horas por 7 dlas, asociado a sales de rehidratacin oral, segn plan A. 11) Paciente de 45 aos, con epigastralgla recurrente, especialmente postprandial, que en algunas ocasiones asciende hasta el trax. Se realiza endoscopra alta que demuestra gastritis antral crnica. Refiere regurgitacin ocasional, sin otra!; molestias. La conducta ms adecuada es: a) Solicitar pHmetria de 24 horas y actuar segn resultado j~ !}ealizar tratamiento erradicador de H. pylori /5>Aniciar medidas generales antirreflujo, asociando tratamiento con omeprazol a dosis estndar . tJ') Diagnosticar Dispepsia funcional, educar e iniciar medidas generales e) Solicitar ecografla abdominal y actuar se1~n hallazgos 12) Mujer de 33 aos, con dolor abdominal abdominal, mayor en fosa iliaca izquierda, de aos de evolucin, asociado a distensin abdominal y deposiciones frecuentes, de consistencia disminuida, con mucosidad. El diagnstico ms probable es: a) 13iarrea de origen viral .....b1:Sfndrome de intestino irritable ?' e) Enfermedad celiaca d) Colitis ulcerosa e) Amebiasis intestinal 13) Una paciente de 24 aos presenta desde algn tiempo diarrea con emisin de moco y sangre, y dolor abdominal, sin que existan antecedentes epidemiolgicos de inters. Qu diagnstico serra, entre los siguientes, el ms probable y qu exploracin realizarla para confirmarlo?: a) Diverticulosis y TAC de abdomen y pelvis b) Argiodisplasia del colon y colonoscopla ...P)fnfermedad inflamatoria intestinal y colonoscopia /'d) Divertculo de Meckel y gammagrafia con 99Tc pertecnetato e) Plipo colorectal y rectoscopia 14) Ante una mujer de 60 aos con antecodentes de un cuadro de malnutricin en la infancia, que presenta una diarrea crnica a lo largo de! 4 aos con esteatorrea, anemia ferropnica y ad.~lgazamiento progresivo, la causa ms probable de su diarrea es: afEnfermedad celiaca ... 'b) Enfermedad de Crohn e) Colitis ulcerosa d) Cncer de Colon e) Pancreatitis crnica

f!'!'
~ ~ ~

~
(~!;

~
~ ~
~

~ f!4'
(!A
~

~
~

~ ~
~

~
~
~

~
(!"

~
~
~

~ ~
~

~
~
~

(FA

~
~

~
~
~

~
15) Mujer de 75 aos, hipertensa, con hipercolesterolemia, quet hace 2 aos fue estudiada por cuadro de diarrea, con colonoscopfa, que result nomal. Acude al serilclo de urgencia por cuadro de dolor abdominal de 4 horas de evolucin, mayor en hemiabdomen 12:qulerdo, asociados a sudoracin y emisin de sangre franca por el ano. La exploracin fsica destaca abdomen doloroso, con leve distensin. Cul es el diagnstico ms probable? a) Cncer de colon b) Enfermedad inflamatoria intestinal e) Disentera por Shiguella d) Colitis pseudomembranosa gf':Colitis isqumica 16) Paciente de 30 aos, portador de lcera gstrica por uso de AINEs, consulta por exacerbacin de sus sntomas dolorosos. Desde hace 3 horas el dolor se hace mucho ms intenso, irradindose a dorso y se asocia a hipotensin. Al examen fsico destaca abdomen en tabla, muy doloroso, con Blumber (+) y abolicin de los ruidos intestinales. Qu examc~n debe solicitarse como primera aproximacin diagnstica? a) Ecografla abdominal b).JAC de abdomen y pelvis yendoscopia digestiva alta de urgencia ~ Radiografla de abdomen de pie e) Colangioresonancia 17) Paciente de sexo femenino, de 45 aos, inicia dolor en Hipocondrio derecho, muy intenso, as.oclado a fiebre de un d(a de evolucin. La analftica sangurnea muestra Hcto: 42%, G. Blancos: 15.200, GOT: 55, GPT: 48, 8111: 1,7, amllasa normal. La ecogratla abdominal muestra vescula biliar distendida con paredes engrosadas, barro biliar y clculo de 12 mm de dimetro, Impactado en el b~lnete. El tratamiento de eleccin es: .r ~olecistectomfa laparoscpica de urgencia ,b) Tratamiento antibitico y colecistectomia en fro en 10 das e) Colangiografia retrgrada endoscpica y descomprensin de la vfa biliar d) Colecistectomia abierta a la brevedad e) Reposicin de volumen y antibiticos y evaluacin quirrgica en 3 meses 18) Paciente de 50 aos, con cirrosis heptica por alcohol, presenta episodio de hematemesis y melena, asociado a ortostatismo. Al examen en decbito, destaca FC:1011pm, PA:110/65 mmHg y conjuntivas plidas. La medida teraputica Inicial ms importante es: a) Admistrar omeprazol endovenoso b));ransfusin de 2 unidades de glbulos rojos, Juego de tomar homatocrito ~ndoscopia de urgencia y ligadura de vrices esofgicas 1:1) Terlipresina endovenosa y sonda de sengstaken e) Reposicin de volumen con cristaloides por va perifrica 19) El tratamiento de eleccin para la Infestacin intestinal por Giardla lambia es: a) Albendazol b) Permetrina dMetronidazol Ciprofloxacino / e) Conducta expectante

~
~

~
~
~

~
~

~
~
~

~
~
~

~ ~
~ ~
~

~
~

~
~
~

~
~

/d)

~
~
~

20) Cul de las slgulnetes alternativas corresponde a un tratamiento de primera linea para erradicar H. Pylori? ,afAmoxicilina, claritromicina y omeprazol b) Bismuto coloidal, tetraclclina y metronidazol e) Bismuto coloidal, tetraciclina, metronidazol y omeprazol d) Famotidina, claritromicina y omeprazol e) Omeprazol, bismuto coloidal y amoxicilina

~ ~

RUT

~
~
~
NOMBRE:

~t~ . J~

('!'
~

~
~

FECHA:

1 00 200 3 00 4 00 5 00 6 00 7 00 8 00 9 00
K

00 000 000 o 000 000 o 000 000 o


000 000

~ ~ ~ ~
~

ooo ooo o 000 ooo. o 000 000 o 000 000 o 000 000 o 000 000 o o
RESPUESTAS O E A 8 C O E

cu'Jsf~
A B C
1
A B C O E

~
~

~
~

~ ~

~
(!A

~
~

~ ~ ~

~
~
~

0000 2 0000 3 0000 4 00<900 5 0000 6 0000@) 7 0000 8 0000 9 0000 10 0000 11 00>00 12 0@000 13 0000 14 0000 15 0000 16 000@>0 17 @0000 18 00@00 19 00@00 20 0000 00000 00000 00000 00000 00000 00000 27 00000 28 00000 29 00000 30 00000 21 22 23 24 25 26

31 32 33 34 35 36 37 38 39 40

00000 00000 00000 00000 00000 00000 00000 00000 00000 00000

e!'
~
~

~ ~ ~ ~
~

41 00000 42 00000 43 00000 44 00000 45 00000 46 00000 47 00000 48 00000 49 00000 50 00000 51 00000 52 00000 53 00000 54 00000 55 00000 56 00000 57 00000 58 00000 59 00000 60 00000

61 00000 62 00000 63 00000 64 00000 65 00000 66 00000 67 00000 68 00000 69 00000 70 00000 71 00000 72 00000 73 00000 74 00000 75 00000 76 00000 77 00000 78 00000 79 00000 80 00000
81

82 83 84 85
86

87 88 89 90

00000 00000 00000 00000 00000 00000 00000 00000 00000 00000

e
~

e ~ -e----

f!'

~ ~

~ ~

e!'
~)
~

Prueba 4 CIRU 'A, TRAUMATOLOGA y UROLOGA {Bancos de preguntas) 1) ~Cul de los siguientes frmacos debe suspenderse necesariamente antes de una ciruga electiva? Hipoglicemiantes
~~~tihipertensivos ~{ ':~ticoagulantes

~~
~

'~
~ ~ ~

d) Antidepre ivos e) Anticon lsivantes 2) ;Lo sintomas y signos de la enfermedad arterioesclertica de la bifurcacin de la aorta abdominal (Ss. ome de Leriche) incluyen: c.-1-J ;claudicacin del glteo y pierna lL '('\. 3 .dolor de la extremidad inferior ~ f\-e_( M e eyaculacin retrgrada \.._ _ _Lp _ _ ... r , ,. ~ . . _ o ) gangrena del pie ~~v\/ t/'-() ..J--"-'V" ~ ) rubor el pie

e
~

~ ~
~ ~

e
~

~ ~ ~
~

a
1

jer de 19 aos, en el quinto dia de menstruacin. Desde hace 24 horas est con dolor continuo en 11aca derecha y ha tenido en cuatro oportunidades deposiciones liquidas sin sangre. En el amen hay resistencia muscular en la zona del dolor y esbozo de Blumberg. Examen ginecolgico normal. Temperatura axilar es de 38C y rectal de 39.5C. El recuento de blancos es de 22.000/mm3. L~ ~era posibilidad diagnstica es: g-ap~ndiciti guda. b) colecisf s aguda e) proce o inflamatorio pelviano. d) sa~ onellosis intestinal. 'lonefritis aguda.
fo na ictericia fluctuante orienta al diagnstico de: olangitis esclerosante o~ ~atitis aguda . d) he iitis alcohlica ~) t ores hepticos :) Cul es el diagnstico ms probable frente a un paciente que consulta por una masa perianal pequea, dolorosa, de 2 das de evolucin y sin sangrado rectal? a) Fisura anal . 1 ~~"' yv-e -t clL{ b) Hemorroide interno grado 2 l~-c)yondilomas ~cuminados perianales ~- "- t- 0 ..... ...() t'11} Absceso penanal O rv q - - ' - ' - e) Absceso pelvirrectal 6) Nia de 3 meses, con antecedentes de parto en podlica y madre con displasia de cadera tratada. Al examen fisico destaca limitacin a la abduccin de cadera derecha. al es la conducta ms adecuada? a) Solicitar radiografa de cadera derecha b) Solicitar radiograffa de pelvis antero posterior e) Indicar uso de doble paal d) Indicar uso de correas de Pavlik e) Observarrontrolar en un mes 7) Pacievfe de 22 aos, sufre cada sobre la mano con el codo en extensin. Presenta dolor, leve aume t:'o de volumen e impotencia funcional del codo, con imposibilidad de pronosupinar en forma activ . El agnstico ms probable es una fractura: de epitrclea. ) marginal de capitelum.

~.masio/Coledociana

~ ~ ~ ~

e!'
~ ~

e!'
~ ~ ~
~

e e
~

~Upracondilea ~d) de cpula radial.

'

e) de olcranon.

8 Cul de los siguientes diagnsticos es indispensable descartar en un paciente con fractura de elvis inestable y globo vesical, que no puede orinar espontneamente? a) Rotura vesical b} Trauma renal e) Adenoma de prstata d)..EStenosi retral ") Rotur retral posterior

1O)

potenciales complicaciones de una Enfermedad Diverticular de larga data stula Colo- Vesical ...- t-' JLv ~ r{ "'- .-1-11""-~ c.~ 0-J'..J~ ) Absceso perianal t~ .L.:-~ e) Cncer de sigmoides r-- (r'-' ' d) Incontinencia anal e) Col' i ulcerosa

c-.

En qu pacientes sugeriria usted adelantar los controles do pesquisa de cncer prosttico? ) Antecedentes de prostatitis crnica b) ecedente personal de cncer testicular Antecedentes de padre con cncer de prstata d) Antecedentes de tabaquismo e) Antecedentes de padre y hermano con hiperplasia prosttica 12) U aciente de 65 aos portador de una masa inguinal que ;:mtes se reducfa y desde hace 3 horas no ede reducir, consulta en policlnica. Al examen impresion:a como una hernia Inguinal, i eductible, sensible. La conducta ms apropiada en ese momonto es: a) Solicitar interconsulta con Urlogo b) S ar exmenes de sangre pre operatorios y programar cirugla electiva o ramar ciru ra de urgencia s italizar d) Hospitalizar, iniciar antibiticos yana gesicos y reevaluar en 24 horas e) Hospitalizar y programar cirugla diferida 13) P ciente de 30 aos ingresa con traumatismo craneoenceflico grave, fractura de pelvis estable y co usin pubnonar. Tres dfas despus presenta una trombosis venosa profunda proximal a la rodilla. ya~nducta ms apropiada es: :?rn~~alar un filtro en la vena cava inferior b) indicar compresin neumtica intermitente e) iniciar tratamiento anticoagulante con heparina fraccionada d) indicar vendas de compresin elstica graduada y deambulacin precoz e) Resolver quirrgicamente de inmediato 14) Pacien con fractura de pierna inmovilizada con bota larga de yeso, recibiendo antiinflamatorios no este adales por va oral. Horas despus de la inmovilizacin presenta aumento del dolor y de vol u en de la pierna que no cede al abrir el yeso. Presenta adems dolor a la movilizacin de los d os del pi. Cul es la causa ms probable de este cuadro? ) Trombosis venosa profunda b)jlistrfia simptico-refleja Slndrome compartamental d) Desplazamiento de la fractura e) Dosis y via inadecuada de antiinflamatorios

~ f'

~
~

fA

t!'
~ ~
~

15) Cul de las siguientes conductas es. ms adecuada frente al diagnstico clnico y ecogrfico de tuJ!!Or te ticular? a) Re ar orquectomia por vla inguinal t b) icitar cintigrafia osea Controlar con nueva ecografia en 30 das. d) Realizar biopsia testicular por puncin ~ ~~ . e) Rear ar exploracin quirrgica testicular por va escrotal _
especto al slndrome de humeral: Es un problema infrecuente de consulta Jndica ._.. ~Los pacientes frecuentemente acusan dolor nocturno t) Se produce por atrapamiento del tendn del msculo serrato anterior d) Los sintomas con atpicos cuando hay rotura del tendn involucrado e) Siempre tiene la misma etiologia, independientemente del gupo etreo al que afecta 16

~camien~()

(J

e
~ ~ ~
~

"'

~
~

17) Un hombre de 55 aos sufre una cada desde un 3 piso, se ce comprometido de consciencia, frecue la respiratoria de 37x', con fractura expuesta de antebrazo, con lesin sangrante en zona par" otemporal izquierda, la trquea esbi desviada a la izquierda y el murmullo pulmonar derecho e~t /_ olido. La Primera accin a tomar es: ( _ r. <._) y ~~ a) Poner collar cervical "1 ~-ro ~ u-.. ._..r ' ,...9)" Puncionar segundo espacio intercostal dorecho \\J Q)J e) Solicitar TAC de cerebro d) Tomar set de radiografas de politraumatizado (AP pelvis, AP de trax, Lat columna cervical) e) p er dos vas venosas e iniciar reposicin de volumen.

~
~

~ ~

~ ~

) Paciente de 60 aos, muy fumador, c()n adenopatfa de consistencia ptrea en regin submandibular izquierda. Este hallazgo corresponde con mayor probabilidad a una adenopata secundaria a: a) cncer pulmonar b) cncer de estmago e) cncer diferenciado de tiroides ~) cncer hipofarfngo " e) cnc r de trquea 19) s contraindicacin de la mastectomfa parcial: a eceptores hormonales negativos ) ~u mor m! 1ltifgcal e) ncer ductal invasor d) Tumor e 5 cms e) Cnc r in situ 20} a trada sintomtica de la obstruccin intestinal es a oolor abdominal, vmitos y fiebre ) dolor abdominal, fiebre e imposibilidad de eliminar gases e) vmitos, constipacin y dolor abdominal "d)'QQla.r: abdo_mi~l, distensin abdominal y vll!.i!~ e)'vmitos, dolor a6dos:RiRal y abohcJon de-tos-ruidos hidroareos

~
~ (!A

~
~.

~
~

~ ~

e
~
~
~

~
~

e.
~

Prueba 15 GINECOLOG[A + OBSTETRICIA


1) Son causas de hipermenorrea, EXCEPTO: a) Miomas uterinos b) \Iteraciones de la hemostasis primaria 1-tiperplasia endometrial ~) Adenomiosis e) tero septado

zt

2) Una paciente de 24 aos, primigesta, cUrsando embarazo de 31 semanas consulta por deterioro del estado general y d9J9-t:.~~dominal. Al examen se aprecia adolorida y ~~.9.~J~tdoa. Sus signos vitales son FC: 96x', PA: (142/9.-JllmHg, FR: 15x' y el monitoreo fetal resulta normal. Se solicitan exmenes que demuestran etevaci~-~~~!!~.aminasas._y_biJjrtutlina, a!_C?.c~~dos a ~~!mla.y.tromb.9.citopenia./ leves. En sus exmenes de orma presenta proteinuria de 0,5 u/24 horas. El diagnstico ms probable es: a) Hfgado graso agudo del embarazo b) Colesta~ia intraheptica e) Preclamsia moderada ~hdrome de HELLP .:--..'1:!) Sepsis por Clostridium perfringens 3) Una paciente multipara, cursando un embarazo de 35 semimas se realiza ecograffa obsttrica, que demuestra un feto con peso estimado en percentil 5, para la edad gestacional, con relacin dimetros ceflico/abdominal de 1,2. La etiologra ms probable del retardo del crecimiento intrauterino, en esta p~Iente, es ~ ~onstitucional . (_;\.._y").._/ b} Insuficiencia placentaria S\-""'-'.--(~~\...-[: e) Genopatfa d) Sufrimiento fetal agudo e) Diabetes gestacional 4),ymejor examen para determinar el pronstico de un embarazo gemelar es:
A~Ecografla del primer trimestre
1! ....

b) Ecografia del segundo trimestre e) Ecografia del tercer trimestre d) Amniocentesis e) Biopsia corial

5) Respecto a la endometriosis es VERDADERO que: a) El uso de anticonceptivos orales es un factor de riesgo b) Habitualmente se diagnostica mediante ecografa ~ qausa de oligomenorrea s'asocia al uso de dispositivo intrauterino ./ ~) Corresponde a tejido endometrial dentro del miometrio // 6) Una paciente primlgesta, cursando embarazo de 34 semanas, presenta disminucin de la percepcin de los movimientos fetales, por lo que se realiza un test de tolerancia a las contrac~iones que demuestra la presencia de desaceleraciones tempranas o DIP 1, sin otras alteraciones. No presenta modificaciones cervicales. La conducta ms adecuada es: a) Administrar corticoides endovenosos b) lnterrumplir el embarazo por la vfa ms expedita e) Inducir el parto con misoprostol d) Solicitar registro basal no estresante Jt1 Tranquilizar a la madre y enviar a domicilio
/

~,La

7) El elemento que ms orienta a un algia plvica crnica, de origen ginecolgico es: ciclicidad b)iEJ antecedente de metrorragia e) El dolor al tacto vaginal d) El uso de anticonceptivos e) La asociacin a distensin abdominal

~ ~ ~ ~

~ ~

8) Una mujer, multipara de 2, con ambos partos por vra vaginal, est cursando un tercer embarazo, en la semana 37 + 3 das. Inicia trabajo dt~ parto, que se extiende por varias horas. Al examen fsico presenta 3 contracciones en 1O minutos y el tacto vaginal rhuestra dilatacin comp.leta, con la cabeza fetal en el segundo plano de Hodge, en presentacin--eJe frente, naso-iliaca anterior. La conducta ms adecuada es:
a) Realizar prueba de parto, con aceleracin oxitcica y rotura artificial de membranas b) Girar la cabeza hata posicin naso-sa::ra e) Girar la cabeza hasta posicin naso-pbica 9), ~ealizar forceps (ftf Realizar cesrea
....

9) E} sfndrome de ovario poliqustico se caracteriza por todo lo siguiente EXCEPTO:


a). Andrgenos elevados ,l?)"'Estrgenos disminuidos /;:C') Progesterona disminuida d) Hormona luteinizante aumentada e) Acn

er;
~

1O) Identifique la asociacin

e
~

a) 12 se.f!1anas -.fondcfuterio

entre edad gestacional y hallazgo: ..... ~ ~_15.~eraoa..~_mu1tiparainici~napercepci6n.de los movimientos fetales

corr~.cta

a-ia altura del omQI.igo

~
~

~semanas - beta-HCG detectable en suero y orina . . .9iC6.~~manas - a~~~.~~i.. d.e..$aco gestack:mal en la ecografa transvaginal e) 30 sernana~ ~altura uterina de 15 centlmetros

e
~

e
~

11) Una paciente de 24 aos, consulta por dolor abdominal. Al examen se constata paciente subfebril, con dolor a la palpacl.rt hipogstrica, palpacin anexial derecha y movilizacin cervical. No hay signos de irritacin perJtoneai..EI diagnstico ms probable es: .>fondometriosis /j>r Proceso inflamatorio plvico e) Apendicitis aguda d) Absceso tuboovrico roto e) Embarazo tubario

~
~

12) Una paciente de 40 aos, multpara de 3, cursando un embarazo de 38 semanas, se realiza un registro basal no estresante que muestra una frecuencia cardaca fetal basal de 130x', con una variabilidad de cerca de 1Ox', sin presencia de aceleraciones ni de desaceleraciones durante un tiempo de 9~~ervacin de 20 minutos. La cond1ucta ms adecuada es:
a)'Slicitar una ecografia-Doppter de arterias umbilicales ./ -:6) Realizar un perfil bioflsico e) Extender el monitoreo por 20 minutos adicionales d) Interrumpir el embarazo de inmediato e) Tranquilizar a la madre y dar de alta

~
~1

~
~

e!'
~ ~ ~ ~
~
~

.-.-

13) Una paciente presenta amenorrea dE! 6 meses de evolucin, que inici luego de un aborto espontneo. Se solicita test de embarazo que resulta negativo, por lo que se decide solicitar varios exmenes que demuestran: TSH normal, prolactinema normal, prueba de progesterona negativa, ecografa TV con endometrio muy delgado, prueba de estrgenos ms progesterona negativa. El origen ms probable de la amenorrea es: a) Anovulacin b) Hipotalmico e) Hipofisiario d~-vrico e. ' terina

'

14) Cul de las siguientes situaci31Jes ti:ene opcin de parto vaginal?


a) Embarazo gemelar con el pri.mer.,.~melo en presentacin ceflica y el segundo en presentacin podlica b) Mujer en trabajo de parto con feto @lcef~~.liea con presentacin de cara, en posicin mentosacra e) Embarazo con herpes genital activo'. . dYJ5rimigesta con embarazo complicado con diabetes gestacional y feto de 3.800 f:<g. -=-) Mujer con una cesrea anterior de tipo ccrporal

~
(1"-

e
~

15) Una mujer de 27 aos, cursando embarazo de 32 semanas t3e realiza ecografia obsttrica que
demuestra presencia de polihldramnios moderado, sin otras alteraciones. Ha controlado adecuadamente su embarazo, sin haber presentado patologias previas. La conducta ms adecuada es: a) Inducir el parto con misoprostol vaginal b)~dministrar corticoides endovenosos e interrumpir el embarazo E!n 48 horas c5 solicitar un test de tolerancia a la glucosa d) Realizar amniocentesis e) Controlar con nueva ecografia a las 36 semanas, sin necesidad de estudio adicional

16) Una mujer de 28 aos, purpera hace 2 dias, presenta aumento de volumen y dolor intenso en relacin a la axila derecha. Al examen se aprecia dichQ aumento de volumen, de 4 cm, muy doloroso a
la palpacin, con escaso eritema. El diagnstico ms probable es: a))l1astitis abscedada Glndula mamaria accesoria e) Jromboembolismo pulmonar 9)'Adenitis supurada -'e) Mastitis linfnagltica

/M

.... 17) Identifique la asociacin correcta: a) Hipermenorrea - Ci~menstruales 'de 2o di as b) Metrorrg-.:.:~MestraCio""es muy..ab.und.a.o.t-~...en: cantidad y. duracin q~i~uso'!.Qia.~~~-Qrragia durantelos dias de ovulacin Aj Ohgomenorrf8 - Ciclos menstruales de 37 dfas e) Amenorrea- Ausenci'-1 de menstruacin en los ltimos 60 dias

. ") '-r-"\JL.-Yl ...


18) Las causas ms frecuentes de metrorragia, en las pacientos en edad frtil y en las pacientes pqstmenopusicas, son respectivamente: ,..aJ Disfuncional y atrofia endometrial .,.. .b) Anovulacin y cncer de cuello uterino e) Miomas y atrofia endometrial d) Endometriosis e infecciones e) Disfuncional y cncer de endometrio

19) Una paciente de 30 aos presenta dismenorrea desde hace 4 aos, la que ha ido en aumento.
Adems presenta dispareunia profunda que ~e ha vuelto ms intensa en los ltimos meses. Refiere que lleva 2 aos intentando concebir, sin embargo no lo ha logrado a pesar de no usar mtodos anticonceptivos y a mantener relaciones sexuales frecuentes con su marido. Al examen fisico presenta dolor a la movilizacin del cuello uterino, son otras alteraciones. El diagnstico ms probable es: a) Cncer de cuello uterino .. blAdenomiosis .&) Endometriosis < d) Miomatosis e) Proceso inflamatorio plvico 20) El periodo frtil se extiende desde: a) Un dla antes hasta un dla despus de la ovulacin b) Cinco dlas antes hasta cinco dias despus de la ovulacin ~)tinca dias antes hasta el dia de la ovulacin . d) Diez da antes hasta tres dias despus de la ovulacin e) Siete das antes hasta tres dias despus de la ovulacin

~
~

RUT

o
1
~OMB RE:

~
~

2 3

4
5

RUT:

6 7
8

~ ~
~
FECHA:

9
K

00 00 00 00 00 00 00 00 00 00

000 000 000 000 000 000 000 000 000 000

000 000 000 000 000 000 000 000

000 000

o o
o o
O E

o o o o o o

~
~

C U RS 0: RESPUESTAS A B C
1

e ~ e
~

~ ~

O E
31

A
61

B C

D E

~ ~ ~
~

00000 00000 3 00000 4 00000 5 00000 6 00000 7 00000 8 00000 00000 10 00000
9

00000 00000 33 00000 34 00000 35 00000 36 00000 37 00000 00000 39 00000 40 00000
32
38

62
63 64 65 66 67
SS 69

70

00000 00000 00000 00000 00000 00000 00000 00000 00000 00000

~
~

~
~
~

~
(!!A

00000 00000 13 00000 14 00000 15 00000 16 00000 17 00000 18 00000 19 00000 20 00000
11 12 21 22 23 24 25 26 27

00000 00000 43 00000 44 00000 45 00000 46 00000 47 00000 48 00000 49 00000 so 00000
41 42 51 52

71

72

00000 00000 73 00000 74 00000 75 00000 76 00000 77 00000 78 00000 79 00000 80 00000

~ ~
~ ~ ~
~ ~

28
29 30

00000 00000 00000 00000 00000 00000 00000 00000 00000 00000

00000 81 00000 00000 82 00000 53 00000 . 83 00000 54 00000 84 00000 55 00000 85 00000 56 00000 86 00000 57 00000 87 00000 58 00000 88 00.000 59 00000 89 00000 60 00000 90 00000

~
~

~ ~
~

~ ~ ~ ~

1) Un paciente consulta por doloren hit>ocondrio derecho y epigastrio, de 2 horas de evolucin, intenso y asociado a vmitos alimentarios, en varias ocasiones. Al examen fsico se encuentra adolorido, en buenas condiciones, con PA: 120/80 mmHg, FC: 89x', FR: 18x', t0 : 36,4C. El abdomen es blando depresible, sensible en hipocondrio derecho, con RHA normales, sin signos de irritacin peritoneal. El hemograma, pruebas hepticas y Jipasa son normale~. La conducta ms adecuada es: a) Enviar a domicilio ~~>n antiespsmdicos y dieta liviana b))niciar antibiticos y ~nalgsicos ~nlicitar ecografia abdominal .. :/ cj) So~~reFdt?seepfa:;diges1it7a;;alta . e) Ralii!ar-1aparetGmfa_ .........-

IJ!

e
~

~
~

~
~

2) Un paciente de 65 aos sufre una carda de altura golpendose el trax contra el suelo. Evoluciona con cierto grado de dificultad respiratoria y al examen se constata abolicin del murmullo pulmonar de la mitad inferior del campo pulmonar derecho y matidez en dicha zona. El diagnstico ms probable es: a) Tromboembolismo pulmonar b) Contusin pulmonar e) Neumotrax ~emotrax /e) Derrame pericrdico 3) Un paciente de 23 aos acude a control por aumento de volumen cervical. AL examen fisico se palpa un ndulo de 2,5 cm en la zona sUJpraesternal, de consistencia dura, que asciende con la deglucin. Se solicita TSH que resulta normal y ecografia cervical que confirma un ndulo slido, de ecogenicidad homognea, sin presencia de adenopatas cervicales. La conducta ms adecuada es: a) Realizar seguimiento ecogrfico olicitar anticuerpos anti-TPO ~.Mf.itar biopsia por puncin Cf).;j.alizar tumorectomia y l;>iopsia qQerivar para tiroidectoma total
4) Un paciente de 67 aos sufre prdida de la visin del ojo ~zquierdo hace 3 ~fas, de inicio sbi~o y 1O

~
~

e"
~

~
~
~

e!!'
~
~

minutos de duracin, que se recuper e~spontneamente. No present otros sntomas. Al examen fsico destaca un soplo carotdeo izquie1rdo. La conducta diagnstica ms adecuada es: a) Realizar un ecocardiograma b) Realizar una angiografia carotidea e) Bealizar un angioTAC de trax y cuello AG:fRealizar un Ecodoppler carotdeo Ye) Solicitar un perfillipidico y dmero D ;- 5) Respecto a los hemorroides externos es verdadero que: a) Son causa frecuente de hemorragia de brigen proctolgico .-b) La complicacin ms grave es la fluxin hemorroidal .e) Se producen por infecciones perianales recurrentes.:\,:;."' ,dy Son intensamente dolorosos cuando de trombosa~' e) Su diagnstico requiere de una eeografia doppler~~

~
~

)'\"Y"

(!"

~ ~ ~ ~

6) Un paciente de 33 aos presenta sintcmatologfa de un sndrome de Sjorgren, por lo que se realiza una biopsia quirrgica de glndula salival menor de labio inferior. A qu tipo de herida corresponde este procedimiento? a) Limpia b) ~iiflpia contaminada

o):;~ontaminada

e
~ ~
~

/t)-Sucia e) Infectada

~
~
~

~
~

JI!;

7) Un hombre de 42 aos, trabajador de una empresa forestal, sufre herida de gran tamafto en el muslo derecho, al enterrarse una estaca de pino. Al examen fisico tiene una herida de 8 cm de largo y cerca de 4 cm de profundidad, con abundantes restos vegetales. El paciente habrra recibido un esquema completo de vacunacin antitetnica hace 30 aos y una dosis de refuerzo hace 4 ai\os, debido a un accidente similar, pero con una herida de tamao m1LJcho menor. La conducta ms adecuada es: a) Administrar esquema completo de toxoide tetnico e inmunoglobulina hiperinmune b) Administrar una nueva dosis de toxoide antitetnico y adems inmunoglobulina hiperinmune e) Administrar inmunoglobulina hiperinmune, sin necesidad de nuevas dosis de toxoide d) Administrar una dosis de toxoide tetnico, de refuerzo, sin necesidad de inmunoglobulina e) No administrar vacuna ni inmunoglobulina antitetnica

8) Respecto a las lceras venosas, es cierto que:


a) Se localizan frecuentemente j}n el malolo externo ~ . . b) Suelen ser muy dolorosas""'( e) Su tratamiento incluye antibiticos de amplio espectr=.:: ~e producen por-hi~tteSicmcapilar hiposttica e) Rara vez presentan alteraciones de la piel circundante ..

;y

9) Las causas ms frecuentes de obstruccin de colon son: a) Ohiertfculos y angiodisplasias -~'{Bridas y hernias e) Hernias y vlvulos d) Cncer y diverticulitis e) Bridas y cncer

"'

<

10) Cul de las siguientes opciones NO representa un riesgo mayor de presentar nuseas y vmitos en el postoperatorio? _jifPaciente fumador .... 1:6) Sexo femenino --: e) Uso de morfina durante la ciruga \.... d) Historia de nuseas fciles de desencadenar e) Historia de nuseas en cirugfas anteriores .. 11) Cul de las siguientes afirmaciones respecto a las hernias es falsa? a) La hernia encarcerada es dolorosa e imposible de reducir / b) La hernia crural es ms frecuente en las mujeres, que en los hombres jc) La hernia inguinal indirecta es la ms frecuente en ambos sexos L- / d)j.tf!.hrnia estrangulada presenta compromiso vascular del contenido herniario'( l: hernia crural, contiene un saco herniario que protruye por sobre el ligamento inguinal \.
12) Respecto al cncer gstrico es falso que: a) Es la primera causa de muerte por cncer en la poblacin masculina chilena b) El nico tratamiento con Intencin curativa es la cirugia e) Su etiologia se asocia a la presencia de Helicobacter pylori d) Se diagnostica y etapifica mediante biopsia endoscpica y tomografia computariaza., respectivamente Su factor prons.tico ms importante es la presencia de adenopailas comprometidas

..P)
/'

z>

13) Una paciente presenta dolor abdominal tipo clico, asociadlo a vmitos y distensin abdominal con
bazuqueo y aumento de los ruidos hidroareos. La ecografia abdominal demuestra neumobilia. La sospecha diagnstica es: a) lntususcepcin b) Colecistitis aguda e)pbstruccin intestinal por bridas Ad5 fleo biliar -7 e) Vlvulo Intestinal

~:

"'
~

" "
t!'
~
~

14) Una paciente de 39 ao~ se palpa varios ndulos mamarios de pequeo tamao, algunos de ellos sensibles. Se realiza una mamograffa qua es Informada como Blrrads 2. La conducta ms adecuada es:. a):tfealizar nueva mamografia en un afio "~-b) Realizar nueva mamograffa en 6 meses e) Soncitar ecografia mamaria d) Solicitar biopsia por aguja fina e) Solicitar biopsia con mamtomo
15) Un paciente ingresa por un cuadro de coleclstitis aguda, comprobada por ecograffa y exmenes de sangre. La conducta anestsica ms adecuada, para el manejo de la vra area es: a) Ocupar mscara larlngea . b)Realizar intubacin en secuencia rpida . ,....) Inducir, administrar relajantes musculares, ventilar con amb y realizar Intubacin orotraqueal una vez lograda una buena relajacin d} Instalar una cnula Mayo e) Realizar anestesia supe~al con dosis bajas de gases halogenados, sin necesidad de utilizar dispositivos de manejo de la vla area. '....

l.

~ ~ ~ ~ ~
~

~
~

16) La complicacin ms Importante del aneurisma artico abdominal operado, es: a) Rotura del aneurisma b) Infeccin de la herida operatoria e) Embolias arteriales . d) Infeccin de la prtesis artica e) Infarto agudo al miocardio
17) La conducta inicial, ms adecuada, ante un paciente con dolor abdominal intenso, vmitos y , elevacin marcada de la lipasa, es: 9})dicar rgimen cero, reponer volumen con cristaloides e iniciar analgsicos endovenosos .....b) Solicitar TAC de abdomen y pelvis y si se encuentra alterado, resolver quirrgicamente ' e} Iniciar antibiticos de amplio espectro, antiespasmdicos y AINEs por vfa oral d) Iniciar rgimen liviano, sin grasas y solicit;:tr ecograffa abdominal e) Realizar colangiograffa retrgrada endosc:plca de inmediato 18) Cul de las siguientes opciones NO es una indicacin de clrug(a en un paciente con endocarditis bacteriana? a} Endocarditis sobre vlvula artica bicspcle b} Abscesos perivalvulares e) Embolias spticas a repeticin d) Fracaso del tratamiento mdico ~.-Insuficiencia cardiaca secundaria a rotura valvular
/

~ ~ ~ ~

~
~

~ ~ f'
~
~
~

~
~

e
-~
~ ~ ~
-~

19) U~a P,aciente es operada con reseccin de un lipoma de 5 cm, en el muslo derecho. Durante la cirugra recibi anestesia regional. Inicialmente bien, sin embargo evoluciona con eritema de la herida quirrgica, que se extiende a la piel circundante, con un halo eritematoso de 5 cm y escaso exudado purulento. El antibitico de eleccin para el tratamiento de esta complicacin es: a) Ceftriaxona ; b) Cefazolina e) Gentamicina .. d) Cirpofloxacino e} Azitromicina
20) Un paciente con histoa de epigastralghil~:~rente y recurrente, de larga data, presenta un esguince de tobillo, por lo que inicia analgesia con ketorpofeno oral. El dolor epigstrico se hace muy Intenso y se irradia al dorso. Al examen presenta un abdomen muy doloroso, de forma difusa, con signos de irritacin petoneal marcados y abolicin de los ruidos hidroareos. La conducta ms adecuada es: at~dministrar.omeprazol endovenoso y antit.Jitlcosde amplio espectro, reevaluando"peridicamente );>} Realizar endoscopia digestiva alta de urge:ncia . e} Suspend~r el ketoprofeno e iniciar tramado! y. omeprazol oral J) Administrar suero fisiolgico endovenoso y solicitar radiografia de abdomen y trax ~e pie e} Realizar apendioetomfa de inmediato . _,... : .: . .
~

"'

ff:.

:.
'

'

'

.. ..... ..
. 1 -
. .. 4:

.,
A)
RUT

o
1

00 000 000
00 000 000 00. 000 000 00 ;000 000 00 000 ooo.

NOMBRE:

2
3
4

o o o o
o

4)

~
~
~

RUT:

7
8

FECHA:

9
K

00 00 00 00
00

000 000 000 000


000

000 Q 000 o 000 o 000 o

~
~

~
~
~

000 . o.

CURSO:

~
RESPUESTAS
A
1
2

~
A
61

B C

D E
31
32

A B

e e o e

~
~

s
6

7
B 9

.OOC>OO . 00000.
38

ooc)oo ooc)oo 00()00 ooc>oo OOC>OO ooc>oo

00000

35
36

ooooo ooooo ~ 00000 ooooo 00000 ooooo


33

00000

6~
63

64

65
66

37

f
68

ooooo ooooo 00000 ooooo 00000 ooooo

00000

-,

..,
~

00000

10
t

OOC>OO
QQ()QQ.

39

40

00000 00000 00000 00000 00000

69

70

00000 00000

~
~

.
~

~
11 12 13 14 15 16 17 18 19
20

00()00 00()00 00()00 00()00 00()00

41 42 43 44

71 72 73 74 75

oo<)OO, ooooo
.es
46 47 48 49
SO

00.000 00000 00000

"')

00000 00000 00000. 00000 00000

76
77 78 79
BO

00000 00000

ooooo

~
~
~

00000

00000 00()00

00000

00000

00000 00000 00000 00000

00000 00000

~ ~

21
. 22 23

00<)00 00<)00

51
52 53

00000 00000
00000. 00000 00000 OO.OO 00000 00000 00000

81
82 83

..,
~
~ ~

24 25

26
27

~B
29

00()00 OOC>OO 54 00()00 55 00000 56 00000 S7 00000 . SB

a4
85

as
Bi
BB
89

30

ooooo ooooo
60

00000

59

90

ooooo

00000 00000 00000 00000 00000 00000 00000

., ,
~

~
~

~
Pre ntas Cirugla 1) na mujer de 47 aos, asintomtl<:a, se realiza una mamografia, como parte de un

~ ~
~

~
~

equeo mdico. Es informada come> Blrrad_s,2. La conducta ms adecuada es: Realizar nueva mamograffa en un at\o ~ b) Sol2!"cita ecograffa mamaria -- e) Rear ar nueva mamografla en 6 meses / d) D rvar para biopsia e) olicitar resonancia magntica nuclear ) Un paciente de 63 aflos se realiza neumonectomfa del lbulo medio derecho como parte del tratamiento de un adenocarcinoma bronquial. A qu tipo de herida corresponde esta cirugia? a) Herida limpia ~erida limpia contaminada e) Herida contaminada d) Herida sucia Herida infectada
}

e
~ ~
~
~

~
~

~ ~

Un paciente de 40 aos, camioner1), consulta por intenso dolor anal, de un dia de olucin, de inicio rpido y que aumenta al sentarse y al defecar. Al examen se observa aumento de volu.men azuloso en relacin a la zona perianal posterior, myy doloroso. El diagnstico ms probable es: a) Fisura anal aguda b) ~sceso perianal -~(('JO P]Hpmorroide externo trombosado ,_/d) emorroide interno trombosado e) licoma ) Un paciente de 65 aftos, hipertens1) mal controlado consulta por dolor torcico intenso, irradiado a dorso, de 40 minytos de d~n. Al examen ffslco se constata P(J.: 210/130 mrAg, FC: 120x' y auscultacin cardiaca con ritmo regular en 2 tonos, y con soplo diastlico d~ regurgitacin artica. La auscultacin pulmonar es normal. El diagn~stico ms probable es: a) Aneurisrha artico roto b) Jpfa~lagudo al miocardio .4J?ise cin artica d) Ul ra pptica perforada e) 1 uficiencia cardiaca descompensada por crisis hipertensiva 5 na paciente de 67 aos,

f!'

~
-

~
~

c~e:etomlzada~ consulta por dolor abdominal, ~o clico,

'"

Al examen fsico destaca abdomen dtstendido, doloroso difuso, con ruidos hidroareos a~menta~os. ~e solicita radiografa de abdomen simple de pie, .que..muestrcunlti.p.Jes mveles hldroaQieos. La conducta ms adecuada es: a) SOiicitar TAC de abdomen y pelvis y decidir manejo segn hallazgos b) Iniciar hidratacin y antibiticos endovenosos y observar evolucin por 24 horas e) Derivar para colonoscopla de urgencia d)~iciar hidratlcin oral fraccionada y antibiticos orales ,.e) Rsolver quirrgicamente de inmediato, bajo cobertura antibitica
(

~ ,(lP'
~
~
~.
'

..

'

6) La mejor Imagen para evaluar un auniento de volunien ce una eXtremidad, u~rcoma de partes blandas es: ;:) Ecograffa de partes blandas /b) Cintigraffa \ e) Tomograffa axial computarizada _Q) =nancia magntica nuclear ..., ~ e) ;(agrafias simples seriadas

compatlbl~ con..

\\-D

~ 0 ....~.' _ "" )._P - 1 ~ ,.., ~ ~-,~


~
~

~
~

7) La complicacin ms frecuente de la apendicectomia ef;:


~ a) Piloflebitis

~)Infeccin de la heridaoperator"

c..t.JL~~

1\i\}"t ,.

e ehiscencJa e la en a operatoria .. d) Atelectasia e) Filtracin de contenido intestinal

~
~
~

8) ~aciente de 65 aos cae del tercer piso de un edificio en construccin y es traido al ho ital, donde Ud. est de turno. Llega en la parte poste1ior de una camioneta. 15 minutos d pus del accidente. Viene inconsciente, no res on ~, estfmylos, slo se palpa pulso rotdeo muy dbil, frecuen toria 35 ,, trquea desyjada hacia J.a._ erectia. o se ausculta murmullo pulmonar en hemlt9,rau izquierdo, el que se percute t1m~co, adems se " acturas costales y enfisema subcutneo. Est plido y su o roso, con abdomen distendido y tenso. Tiene una extensa herida en crneo q~e sangra profusamente y una fractura expuesta en antebrazo derecho que tambin sangra activamente. Cul seria su primera accin sobre este paciente?
a) Comprimir herida.s sangrantes mientras le colocan 2 vfas venosas gruesas y le administran . suero fisiolgico a chorro _prPnc!onar hemitrax izquierdo para descomprimir neumotrax a tensin e) Pro!Jder con intubacin orotraqueal d) C9focar collar cervical . e) 11oner una cnula mayo y ventilar con amb

~
~

~
~

Al)
~

~
~

fi

~
~
~

Un lactante de 7 meses de edad es trado por su madrn ya que presenta llanto y actitud de or 40 minutos de duracin, seguidos de eliminacin de deposiciones rojas, tipo rmelada affotillcr.Akxamen fisico se observa lactante adolorido, con flexin de las caderas y llanto al palpar el abdomen. La conducta ms adecuada es:

a) Resoive(quirrgicamente de inmediato . b)J!liiarfristaloides y antibiticos endovenosos y observar evolucin por 24 horas _ jtT Solicjtar ecograffa abdominal . ,.. / W ~ d) Sol"itar TAC de abdomen y pelvis , \1\ v o~ '".........o~ ' 1\ e) alizarcolonoscopfa ~LI'C>~r~

~
~

~&-)~
~

) Un paciente de 33 aos, sin antecedentes de importancia ni historia de cefaleas previas, presenta. cefalea hol~nea muy-intensa, de ir&lo sbito, de 6 horas de evolucin, asociada a vlll!!9s y que no cede con analgsicos. Al e:Kamen fisico est afebril y no hay signos neurolgicos focales, pero se C.Q!lstata rigidez dEt nuca. Se solicita TAC de cerebro sin contraste, el que resulta normal. La actitud ms adec:uada es:
a) Solicitar resonancia magntica nuclear de cerebro b) Enviar a domicilio con analgsicos orales e) Realizar TAC de cerebro con ~entraste ~ealizar puncin lumbar e) Hospitalizar y administrar AINEs y corticoides endovenosos

"'
~
~
~ ~ ~
/J

tJ ....J ~

.CJ-(J'v"

v t"-P

l ~

----~
~

~ ~ ~
~

,.,

~
....

~ ~ . ~

Una mujer de 57 aos, consulta por ndulo tiroideo derecho de 3 cms, duro, que se viliza con la deglucin. No presenta adenopatias palp.ables ni ecogrficas y la citologia aspiracin con aguja fina resulta compatible con carcinoma papilar de tiroides. La nducta ms adecuada es: ~erivar para tiroidectomla total r b) Derivar para hemitiroidectomra derecha ms linfadenectomla cervical ipsilateral -t L.o 1 o \.....0 C.....c.... ~.t._) e) Administrar yodo radiactivo ~ CA~ -<e
d) Iniciar quimioterapia y radioterapia e} Administrar levotiroxina y observar evolucin por un mes

~
~
~

~~ '

_11

/\

~ ~
~

Jb) Una angpgraffa con estenosis de arb~ria femoral superficial mayor a 90%.
~ or isqumico en reposo de la extmmidad.

12} En un paciente con antecedentes de tabaquismo y disllpidemla, debe sospecharse isgemia e ca de una extremidad inferior ante: Claudicacin intermitente a 1s nc1a meor a 20 metros.

a<

e) Ausen ia de pulso femoral y popHteo. d) Un 1 lice tobillolbrazo de 0.6.

~
~

) En un paciente con coleclstitis aguda es recomendable diferir la cirugia si:


a) Han pasado menos d48 horas desde el inicio de Jos sfntomas _,.b)ffan pas.~do ms de 1O dlas desde el inicio de los srntomas -1 c} El pa9inte es mayor de 65 afias d) ExJste sospecha de coledocolitiasis concomitante paciente se encuentra febril

----'\ u"..._:.c._q,

f .l.A-4 ( a-~-

. /

~
~

~
~

En un paciente con cncer de colon sigmoides, operado de urgencia por una truccin intestinal secundaria, la intervencin quirrgica ms adecuada es:
a) Reseccin segmentara y anastomosis colo-colnica ?""ti(Colectomfa izquierda, sutura de mun rectal y colostomra proximal "' e) Hemicolectomfa izquierda y anastomosis leo-transverso d) Colostomfa derivativa sin tocar el tumor re) Reseccin endoluminaldel tumor, con asa .1

~eV\.

~-

~
~

..~

e!'y_~ ~El nodo sinusal se ubica en la aurlcula izquierda

ffi .
A

n cardiologia, el concepto de "dominancia izquierda" significa que:

> (!"

~
~
~
~
~
~
~

) El tabique interventricular est formado mayormente por fibras musculares del ventriculo izquierdo e) La arteria descendente anterior se ori!~ina de la coronaria izquierda La arteria descendente posterior se origina de la arteria circunfleja e) La presin arterial sistmica es mayor que la presin de la arteria pulmonar

16) Cul de las siguientes opciones corresponde a una indicacin de cirugfa en el contexto d~a pancreatitis aguda:
.~Necrosis

~ ~
~

b) Pseudoquiste de 4 cm e) Abscesos d) ColeccloDeS peripancreticas e) Distress.. respiratorio


,/

1!!)/a cbauflsab!l't. s frecuente de fiebre, ~ego de_.~~~.-~~~.~~~~~ ..~~:


~~om

o e 1 1s

---- --

~ ~ ~ f'l"
~

~Atelectasia

\_../""e) Infeccin de la herida operatoria d) !jipertermia maligna ?.)'Neumonra

~
~

_f!!'

~
~
~

n~ujer de 24 af\os, sufre accld~nte de trnsito, golpepdose el trax y evolucionando disnea. Al examen fisico se aprecia paciente taquipneca, adolorida, con abolicin del mullo pulmonar y matidez en el hemitrax derecho. El diagnstico ms probable es:
.

..

~
~ ~
~ ~

) eumotrax a tensin b) Contusin pulmonar .;aYHemotrax d) T rax volante e) eumotrax abierto .

9) Una paciente de 67 aftos consulta por cuadro de dolor abdominal bajo, asociado a constipacin y sensacin febril. Al examen se cpnstatan ttamperatura d~oc, FC: 90 lpm, PA: 140/80 mmHg. Abdomen blando, depresibi~J Sft.nsible.:.en fosa lllca Izquierda, con sign.Q_ de Blumb~rg esbozado. El examen ms a~ecuado para confirmar la sospecha diagnstica ./ . es: fuograf(a de abdomen y pelvis con contraste 1.0 ~ ~{... ~ adiqgraffa de abdomen simple de pie y decbito
olonoscopfa d) Ec;6grafla abdominal e) ~nema barita do

~ ~
~ ~

~ ~
~
~

J,

6) Respecto al divertfculo de Zenker es verdadero que:


f.J

) Rara vez produce slntomas . .. l - / b) Se produce por un mecanismo de tracc1n ~ ~1 tratamiento suele ser la reseccin quirrgica d) Se produce en la unin gastroesofgica e) Se diagnostica mediante endoscopfa digestiva alta

~
~

~
~
~

~
~
~

~ ~
~
~
~

~
~ ~
~

~
~

~
~

Prueba repaso W 6 REUMATOLOGiA +DIABETES Y NUTRICIN 1) Un ho;!Jlbre de 69 aos, sin an~ecedentes mdicos de inters, presenta una ,monoartritis aguda de rodilla a radiografia simple de rodilla muestra calcificacin de ambos meniscos. Cul de las sigu ntes exploraciones complement<trias hay que solicitar en primer lugar para establecer la er loga de la artritis?: Determinacin de cido rico en sangre. b) At1roscopia de rodilla. r ~amen del lquido sinovial con microscopio de luz polarizada. (C1! ~esonancia n;agntica nuclear de la rodilla. . e) Determi~6n de factor reumatoide en sangre.

p ~n
2)

r {cin con las vasculitis sistmicas, seale cul de las siguientes afirmaciones es ~A: oliarteritis nodosa (PAN) clsica, cursa con frecuencia con glomerulonefritis y capilaritis pulmonar. .)(' b a presencia de anticuerpos anti-citoplasma de los neutrfilos con patrn perinuclear es mucho ms ecuente en la poliarteritis microscpica que en la PAN clsica. e) El tratamiento ms eficaz para la granulomatosis de Wegener consiste en la administracin conjunta de ciclofosfamida y glucocorticoides. d) La presencia de asma bronquial grave y eosinofilia perifrica son caracterlsticas de la granulomatosis alrgica de Churg-Strauss. e) Es frecuente la asociacin de manifestaciones propias de varios sndromes de vasulitis en un mismo paciente.
/

3) I;I mbre de 30 aos, que presenta un cuadro clnico d~J.O meses de evolucin consistente en dolor tnbar continuo, que le despierta por la noche, y que se acompaa de rigidez matutiruL..cle Jre_ sJ]_ClliiS e dliracin. Cul sera el tratamtento de primera eleccin?: ) Diazepan oral. ~Dexametasona intramuscular. ..-:'"~ lndometacina oral. d) Metamizol intramuscular. . e) S9les de oro intramuscular.

1 porcentaje de hemoglobina glicosil.ada, si se determina correctamente, trol 9iabtico, aproximadamente, durante: 1 rnes anterior. ~os 3 meses anteriores. ~ L~~ 5 meses anteriores. d) ):os 7 meses anteriores. eY Los 9 meses anteriores.

ofre~e una estimacin del

)IIeva padeciendo artralgias en las articulaciones metacarpofalngicas 5) Un paciente de 15fiO (M.CE),_m..u~cas_y_ obillos cuatro meses. Posteriormente desarrolla artritis en MCF, intei'faJangicas proximales de manos, metatarsofalngic;~ muecas y tobillos, acampanadas der (gidez matutrna efe ms de 3 horas de duracin. Cual es el diagnstico ms probable?: a) Artrosis generalizada. ....... b) Reumatismo poliarticular agudo (fiebre reumtica). ota poliarticular. rtritis reumatoide. nfermedad de W hipple.

6) Paciente dej i8..aoQs remitido a la consulta por objetivarse, en una analtl<::.a rutinaria, una cifra de ~...ri.c..a...d-9-mg/~~ (normal en homor hasta 7 por el mtodo de la uricasa), siendo el resto de la as analtica, incluida funcin renal, normal. lllo refiere antecedentes personales de inters, excepto que es fumador de medio aque.te..de-tabaooal da: No bebe alcohol ni ha padectdo episodios de litiasis r~al, noaolores articulares. La exploracin ffsi ca es normal y laTA de 120/70 mmHg. Qu actitud teraj)iJtica, c;!e las siguientes, es la ms ir1dicada?: A 'lni.CL:l[Jratamiento con alopurinol. b) Iniciar tratamiento con alopurinol y colchicina . .)?iniciar tratamiento con frmacos uricosrio:>s. -~Iniciar tratamiento con frmacos uricosricos y colchicina. -~No realizar tratamiento alguno.

8) Un muchacho de 12 aos acude al hospital con disminucin del apetltp, aumento de la sed, miccin frecu nte y prdida de peso durante las ultimas tres semanas. En las ltimas 24 horas aparece letlj ico. Los anlisis muestran una natremia de 147 mEg/1. y_na potasemia de 5.4 mEq/1, un cloro de 11 mEq/1, un bicarb~de 6 mEg/1, una glucosa d 536 mgldb una urea de 54 mg/dl, una creatinina e 2 mg/dl y un pH ~La cetonuria es positiva. Cual de las siguientes es la medida ms adecuada para comenzar el tratamiento de este paciente?: a) Hidratacin con salino hipotnicOriQ-U.--deinsolina sabcutaneay-blcarbonate;- ~idratacin con salino isotnico y p~rfusin i. v. de insulina. e) Hidratacin con salino isotnico, petfus16n de insulina i.v. y bicarbonam. d) Hidratacin con salino hipotnioo,-psI:JSiA-de-instlli~ieeffieRSto. e) Hidratacin con salino hipotnico y 10U. de insulina subcutnea.

10) Enferma de 36 aos con antecedentes de esplenectomfa por trombocitopenla a los 22.aos y cuatro abortos, el ltimo seguido de un cuadroc:le trombosas 'lenosa profunda. Desde hace unas 8 semanas prSenta un cuadro de artralgias geer811Zadas, astenia, abultamientos ganglionares y fiebre. , .../CUl de las sig~ientes ~ruebas de l.aboratorio cree de mayor utl~.l~~~-~~~.!.~~~~~-~diagnstico?: e"\ !) Factor reumato1de y anticuerpos antmucleares (ANA). (t)) Anticuerpos antinucleares (ANA) y anticardiolipina. ~ P. (- . _ ..A-':'i:_ J.\-_+e) FactOHeumatoide y aRticuer:po eRfer:n:edad de Lyme.. ~ -, ' , ~nticuerpo anti-ADN nativo y anti-Ro (SSA) ..., e) Anticuerp?s-afttieitef)lasFRa del net rtrfiiQ__~NCi\s)~.-nt~RaSt 11) Una paciente de 67 aos presenta astenia y debilidad de hJs extremidades superiores, de predomi~al, asociada a claudicacin mandibular, ce'f!!,ea frecuente y fiebre ocasional. En sus exmenes generales destaca anemia con marcada elevacin de la VHS. El diagnstico ms probable
~
~

/a}~ranulomatosis de Wegener
e) P~angeltis microscpica ~)1\rteritis de clulas gigantes e) Polimiositis

~
~

~) Lupus sistmico

~
~ ~

1 ) C. ul de las siguientes alteraciones es ms probable de encontrar en un liquido articular de un . aciente cursando con una artritis sptica? Predominio linfocitario ~Filancia disminuida e) .Celuladad baja d) ~lucosa aumeRtada e) p4'oteinas disFRittidas

~
~
~

-~
~
.~

"

"

,.

) A
\

ft~eucocitosis e) Ulceras orales

Qu caracterfstic:a.dellupus eritematoso slstmlco1 Deterioro de la funcin renal v -

alteracin~s
v

~ ~

d) Positividad de los anticuerpos antinucleares y de los anticuerpos anti-DNA de doble hebra / e) Disminucin del complemento V 14) U,n paciente con ant~cedente de psc)rfasjs presenta artritis de la rodilla izquierda y del tobillo d Cho, asociado a dactilitis del 2 i3 ortejos Izquierdos. El resto del examen fislco no aporta m'ayor rmacin. El diagnstico ms probable es: rtritis psoritica b) Slndt=eme de Reite e) Esclerodermia d) Dermatomiositis e} Artritis por cristales 15) Cul de las siguientes alteraciones es menos probable de encontrar en un paciente con artritis reumatoide? a) Ndulos reumatoideos ~ Derrame pleural v" efritis / Anemia e) Miocarditis---'. 16) L s'anticuerpos antlhlstonas son caractersticos de: terodermia cosis lpica e) Lupus cutneo subagudo d) Dermatomiositis ~upus por drC?gas 17) ~!-sndrome m~tablico o sfndrome )(,se caracteriza por todo lo siguiente excepto: ~besidad .,// ~L'iesistencia a la insulina ,/.., . ~Hipercolesterolemia v- d) Hipertensin arterial e) Hipertrig1iceridemia

~ ~
(!!'A

1 '\

~
~

~ ~

~
~

fA
~ ~
~

18)_.Cul de las siguientes es una contrc~cln de la insulina?


~nsuficiencia renal ~ e)

~}Hipertensin arterial no controlada ./ Hemoglobina glicosilada mayor a 10% _ ....

~inguna de las anteriores


19) Cul de los siguientes hallazgos NO se observa en el fondo de ojo de la retinopatfa diabtica? sminucin de los cruces arteriovenosos - -------)'vasos de neoformacin -e) Exudados algodonosos - d) Exudados creos ---- e) Microhemorragias _.. / 20) Qu tratamiento est Indicado ante In aparicin de microalbumlnurla persistente en un paciente con diabetes mellitus tipo 2? rrsunna Dieta hipoproteica . Dieta hiperproteica ,diECAs e) Atorvastatina ...

d) Desnutricin calrica

~ (/A
~
(tiA
(!!;:>.

~
~

~
~
~
~

~
~

f!'l'

_r___ .

() Pregunta recuperativa. Marque la alternativa correcta en la casilla nmero 30 30} Escoja la asociacin INCORRECTA entre cuadro clnico y patologa_ c~te-ae dolor abdominal agudo. ~)J)olor abdominal, asocia vmitos icteri colu - coldocolitiasis . \ . olor ab-o 1na e inicio sbil_? muy inten~o. con abdomen blando, depresible y sin irritcin peritoneal- / \ bolia mesentrica .i' .~ e) Dolor epigstrico en pualada, con abdomen en tabla- lcera pptica perforada ~r abdominal intenso, tipo clico, con ab..~n de los,ruidos hidroareoy o.Q;;.!_r:ucci~~n~\/ e) Dolor en fosa iliaca derecha, con signo de 81umberg- apendicitis aguda 1'

"V

?'

,x/J '

Prueba 4 ~FROLOGIA

/?

/ 1 ) El mejor examen para diferenciar una insuficiencia renal aguda, de una insuficiencia renal crnica es: a) Clearence de creatinina b) Relacin 8UN/creatinina plasmtica raccin excretada de sodio cografa renal 1 aratohormona plasmtica

6 ~)Qu hallazgo es MENOS probable de encontrar en un paciente con una glomerulonefrtis aguda
postestFepto.c.cica!? . a) Hipocomplementemia V' b) Hipertensin arterial ....;..v. . e) Elevacin de la creatinina W . ~ dros eritrocitarios ~Lipid .

J '41U

(JI 5 _':> '1:17 .&f""'

~.... ;r 9' ~~ !2..-;/l.ow

C
/

/ 3) Paciente de 67 aos, con IRC secundaria ~Consulta por asteni a y disnea de esfuerzos que ant;5 no tenia. En el exaemen fsico d7;tas:a~~~z d_ piel y mucosas y examen cardiopulmonar e normal, El hemograma demuestra Hc~ :25%~ 1 VCM:82; HCM:32; leucocitos y plaquetas normales. La ;cintic de hierro resulta normal. El traJaiento ms adecuado es: a) Sulfato de hierro oral Q) Transfusiones peridicas de glbulos rojos ~ Eritropoyetina subcutnea a) Vitamina 812 y hierro, por va parenteral e) Vitamina 812 y hierro, por va oral

A
/

8 7 Qu alteracin es. m ~_prq,l;>able de ser pesquisada en un


/
a~,

/ 4) Una paciente presenta un de di..uria, ie . C. Al :xarnen fsico est~n buenas condiciones, con-pu~sin positiva.a derecha. Se solicita sedimento de orin~que demuestra piociituria y bacteriuria y urocultivo que resulta positivo para E. coli ltisensible. E J,-tratamiento de eleccin es : efadroxilo,../ rnpicilina e) Gentamicina d) Nitrofurantoina e) Metronidazol

d~

cu~dro

~bJ;Jy

''Sta.3~

;~nd_rome n~_frti~~mbios mnimo:; n:_!.'_~~IS -~gLQe,a).L a) H1pertens1on '

aciente editrico cursando con un

.(f)t Hiperlipidemia ' \"e)' Hematuria dismrfica '1.: d) Hematuria no dismrfica Y e) Hiperproteinemia /

----

/i~~ueb,as de funcleR-Fefl81, con ~o urinario muy dismmuldp~ El diagnstico ms probable es:


.
~Sndrome hemolltico urmico

/.Una paciente .cursando un

~dfo diarreico Intenso, ~volu!llona con oligurla.y aumento de las


. ..\.. ~-:- ~ \NO\.]fi'-' ~ ~\ v"('-" / .... /"{}'- ,-

nsufici~ncia renal aguda de origen prerrenall..../"

e Necrosis tubular aguda d) Insuficiencia renal aguda tubulolntersticial e) Glomerulonefritis aguda postinfecciosa

~
(

) A
le
XP
/ /

7) Un paciente de 33 aos presentlJlolldlpsia y ~liuria de algunos meses de evolucin. Actualmente orina cerca de 8 litros al"da. Se soliCita gli'Cemia q~e resulta normal. Se realiza test de la sed que . demuestra osmolaridades Plasmticas y urinarias basales disminuidas y1que se normalizan luego de rivacln de agua. El aiagnstico ms probable es: lndrome de secreCin inadecuada de ADH ,., .1 ~ ~ ~~ ) Diabetes inslpida nefrognica )1l' --'";) ~ ~ ~olidipsia primaria -a) Diabetes inslpida central Uso de diurticos

s:..r /~

// . /

) Un paciente afectado de cncer pulmonar, comienza con un cuadro de c~tipacin de z dias de evolucin, al que hace ~~.I?Jiil.le..agr.ega-polifia,y desde h_!g~Jdgunas...ho.r.as.tambl~e agregdeso.rm_t~!;Ln. Al examen fislco se aprecia con c:k!iqklrataci(m moderada, en sopor pro~ alteraciones neurolgicas focales. Los signos vitales son normales, al igual que el hemoglucotest. Qu alteracin electroltica es ms probable ~n.este paciente~? a) Hipematremia Hiperkalemia Hipercalcemia / 1 remia )< etHipocalcemia

9) Un paciente sufre una caida de altura, resultando con fractu;ra expuesta de ambas pi~rnas, con . importante hemorragia. Ingresa .al hospital donde recibe cristaloides,.AINEs y antibiticos endovenosos y luego se educen las fracturas en pabell6n. ev;oluciona con otiguria y elevacion de la creatinina hasta 3,8 mg/dl. Se mantiene con suero fisiolgico, Uogrando presiones arteriales cercanas a 110/70. Dos dfas despus reinicia diuresis normales, sin embargo la creatinina plasmtica se eleva a 4,2 mg/dl. El diagnstico ms prob~ble es: a) Insuficiencia renal prerrenal b) Nefritis por AINEs :/ /m) Necrosis tubular aguda .,/ . \'a) Insuficiencia renal secundaria a rabdomiolisis e) Trombosis de vena renal .)(1 O) Cul de las siguientes opciones es causa de a) Diabetes inslpida ' b Hipertiroidismo L~_ ,/_/1_ ~ _J. Uso de furosemidao-"':> r. \Y ~WA-A ~ tn"d ome de secrecin inadec ada de hormona an idiurtica ndrome nefrtico

~recuente

hipo.natr.em~

ff

Bz~ ~;~

11) Un paciente diabtico tipo 2, en tratami~metfo~_lJ:.cY491ibenclamida se real!.~a lmeR~~ de control que demuest~~nlnemi~UN 8 i9ls}f,. clea.rence de creatin~~_!!!v' proteinuria de 24 horas.140 g y albuminuria dei4 horas: Lmlt.-~us exmenes anteriores presentaban valores sim1 es. La conducta ms adecuada es:- a) Solicitar hemoglobina glicosilada . b) Mantener el tratamiento y controlar en 3 meses e) Iniciar insulinoterapia -t. ~ Solicitar ecografla renal "plniciar enalapril "'>

: '

t'
~

./h varios episodios dEiJlematyria indolora~ El examen ftsico resulta normal. La anaUtlca demuestra

/~ 2) Joven de 15 aos, con cuadro de 2 dfas de evolucin de odinofagia, fiebre leve y rinorrea. Presenta
hemograma, funcin ~~mal y heptica ..normales. El sedimento de orina muestra er~tr9~itos. d.i.$.ro.9rfi~Qs. abundantes. 'Erdiagnstico ms probable es: - - - - - - - a) Enfermedad de cambios mlnimos (nefmsis lipoidea) b) Glomerulopatra membranosa omerulonefritis rpidamente progresiiJa nfermedad de Berger omerulonefritis aguda postestreptoccica

.!41

~ !A

t!
0

!A

Pf-

~ ~
~

13) Una paciente de 19 afios cursando t:on pielo~efritis.aguda.por. . P-r.oteus-mlrabiWs sensible, inicia tratamiento con ciprpfloxacino oral. Al tercer di~ de estar usando el antibitico persiste. febril y muy decafda. La conducta ms adecuada es: a) Cambiar el cipofloxacino por ceftaxona endovenosa @Solicitar nuevo urocultivo .K e) Mantener el tratamiento con clprofloxacino y reevaluar en 48 horas d) Solicitar cintigrafia renal y vesical V(Solicitar ecografla renal 14) las causas ms frecuentes de insuficiencia renal crnica, en nios y adultos, son respectivamente: a) Prerrenal y prerrenal alformaciones nefrourolgicas e hipertensin . lomerulonefritis y diabetes Ir . . - ... / alformaciones nefrourolglcas y diaber's./' ) lomerulonefritis e hipertensin arterial

~
~

Xv

~
~

~ )( ~
fA
~

f'!'
~

15) El sindrome .~emoltico urmico, suetle presentarse luego de infecciorJes intestinales por algunas cepas de Esc~erichia coli ehter~~~C?.rr~.9~~as y algunas otr~en~eroba~~t~~s. ~~~~a~~~~- se presenta con. )k-'~"Yf~ ~~ a) Edema. orinas espumosas y hematuria / . .. .l ~ipertensin arterial. edema y hematuril ~ c~ematuria. hipertensin arterial y peteqUiias o a) ~ petequias y orinas espumosas ' eYliguri~ palidez y edema ll"'V. . ' ~{-\

~x.v
~

~ ("

16) "La insuficiencia renal crnica produc:e alteraciones de la homeostasis del calciofY. elff.!,toro, las e se caracterizjln por: /,1/ ~ ~ Calcio aumentdo. fsforo bajo y paratohormona elev.'da . -e:=:>\Y ) Calcio disminuido. fsforo disminuido y paratohormona elevada +J-: ~ ~~ .{ /' e) Calcio bajo. fsforo aumentado y paratohormona disminuida./ 1/L(j " (..f d) Calcio dismin sfor:o..alto y paratohormona elevada "\./' 1'- T\{ e) Ca~cio aumenta o. fsforo aumentado y paratohormona disminuida ( ..L u,._a. U::,O

/ H :> / 5"4 /
~

tll -)

~X
~

D.. 17) Cul de las siguientes es una indicacJn de heJD..Q.~s de ur.gencia en un 4J insuficiencia renal?

Qacient~~n

~~

r-o

"' ~
($\

~ricarditis urmica ..../

a) Anemia severa

e) Hipocalemia que no responde al tratamiento mdico ~J-fipocalcemia que no responde al tratami,ento m~dico ~. ~ipernatremia severa con compromiso do consciencia

~x
~
~
(!t\
(!lA
~ ~
~

18) Un paciente de 67 aos presenta insuficlenct aos atrs, con clearence de creatlnina actual elevada en varias oportunidades. Cuft_l.e!s el tra ~~'\ee~~hiposdica. hipoc;alrica y ejercicios; ~ A ms diurticos / e) lo ueadores de calcio PfJrRestriccin hidrosalina y diurticos . e)Remodilisis

ndaria a uso de estceptomiclna, varios a presentado cifraS"depreSilifterial n. o...de..elecclo_p.ara..$.U..blP.ertensin arterial?

1/ril"

~
~
~--,._.

____

~.
~

~
19) Un paciente de 35 aos, previamente sano y asintomtlco, presenta en un perfil bioqumico, realizado como chequeo, una calcemla de U,l..mg/1 (VN: 8,5 a 1Ct,5 mg/dl). El examen fsico no aporta mayor informacin. La conducta ms adecu!ida es: . a) Iniciar furosemida olicitar electrocardiograma y pruebas de funcin renal olicitar niveles plasmticos de paratohormona ndicar una dieta baja en calCio y controlar con nueva calcemla en 3 meses e) Indicar una dieta rica en calcio y controlar con nu~va calcemia en 1 mes

~ ~

i
./

~
~
~

20) Un nio de 6 aos inicia un cuadro caracterizado por orlna.s..espumosas 'y edema generalizado. El examen f(sico no aporta mayor informacin. En sus exmenes se constata proteinuria de 2,5 g/24h, sin otras alteraciones urinarias, hlpoalbuminemia d 2....2 g/dl. El BUIN, creatinina y complemento resultan normales y los ANCA y ANA, negativos. La conducta ms adecuada es: ~ 0 a) Administrar ~ ~ <b) Solicitar biopsia renal ~l.-Ir--o ~ e) Iniciar corticoides endovenosos y ciclofosfamida ~iciar IECAS , erniciar corticoides orales

~
~

~
~

~
~
.~

A)
~
~

~
~

~
~

~.

~
PRUE A 4 Preguntas Nefrologla 1) M er de 64 aos, ingresa por cuadro' de hemorragia digestiva por lcera astrlca. Inicialmente pre enta hipotensin y ollguria. En 'sus exmenes destaca Cms.;,My 8\Ht.W). Se administra volumen co suero fisiolgico y se realiza tratamiento endoscplco de la lcera, con buen resultado. Se anti~ne con presin arterial cercana a 110170. A13er dfa relnicla'buenas djuresjs, cercanas a . )~~OO~c/da, sin embargo persiste con ra:3,~ La causa mas probable de su alteracin de la funcin
r~:

~~ipovolemia con compromiso prerrenal ~ecrosis tubular ag!Jda '

e) Nefropata diabtica
d) vl~~.. artica bicspide e) co;racin artica aclente de 67 afios, con IRC secundaria a DM2. Consulta por astenia y disnea de esfuerzos que tes no tenia. En el exaemen fisico det~ palidez de e1el y m..Y..Q.Q!IaS y examen cardioputmonar ormal. El hemograma demuestra H~~; VcM:8_3; HCM:32; leuco~ltos y plquetas normales. La cintica de hierro resulta normal. El tratamiento ms adecuado es: a) Sulfato de hierro oral b~lransfusiones peridicas de giQblilos rojos ~ritropoyetina subcutnea,._/ d) Vitamina 812 y hierro, por vla parenteral e) Vit tlina 812 y hierro, por vfa oral 2) 3 aciente con mieloma mltiple, evoluciona con poliu~a; deshidratacin y compromiso de conciencia. En la ~nalitica sangu(nea destaca Calcemia 16mEqp. La primera medida a tomar es: a) ~dministrar ~gnesio endovenoso ~dministrar.-suero fisiolgico-t

('A

~ ~
~

~
~

<"'
~
~

e) Adm~n~~tr,r hifsstQ.natos.~~-ndo~~nosos n
d) Adma r~ fu.rosemida endovenosa e) Ad nistrar corticoldes endovenosos

~ (!!A
~

~
~

~
~

n escolar de 5 aos es .tra(do por su madre, ya que presenta edema de prpados. Al examen se precia paciente no,rmotensn, EC:100x', con edema facial, escrotal y de extremidades. Los exmenes de orina muestran protelnuria (++++); gctas de grasa(+), sin hematuria. Usted solicita proteinuria de 24 ~JS que muestra proteinuria en rango nefrtlco. La conducta ms adecuada es: ~licitar C3, C4, ANA, creatinina e iniciar tratamiento corticoidal b) Indicar restriccin de sodio y agua e indicar un diurtico e) Realizar electroforesis de protelnas en orina d) Explicar a la madre el caracter benigno del cuadro e indicar control S.O.S. e) Sol,r biopsia renal 5) P ciente diabtico de 45 aos, con control irregular de sus gllcemlas con metformlna. Su presin a rial es norm.at .. En sus exmenes de control destaca creatinina:.Q.JI; BUN:12.i buminuria:1QOmg/24h. La.conducta ms adecuada, adems de mejorar el control de sus glicemias, es: , --.. )

~
~

~
(~!'

-~~

~
~

...... Q!.~Q.J-~n~apriL~~ tra!am~.~nto-:. .orsriS~nderhipo_~lice"iante~:Q"(les e lnlc:iar insulina e) lni if prednisna d) spechar glomerulopatia membranosa y realizar control de protelnurla cada 3 meses Realizar biopsia renal

~
~ (lA ~
~

6) El examen de eleccin para diferenciar entre una insuficiencia renal aguda y una insuficiencia renal crnica es: a) Producto calcio-fsforo b) Hemograma X">< e) Orina completa / '"~ografia -renal ......r e) Proteinuria de 24 horas

~
(!A

~
~ ~
~

.: . .

(Un paciente de 30 aos, ingresa por compromiso de Los gases arteriales. mu. estran los resultados: plj;.7.2ll; pCo_2:12; HCO,:14. diagnstico cido bas!! es: a) Acidosis respiratoria y acidosis metablica ~ --+- ~ ~ ~_[ b) Acidosis metablica aislada Jo+:.<--fJ ~"""' __A~idosis metablica y alcalosis respiratora J.> ~ A - (9:...a ( '- I . . ....ci)"J\cidosis respiratoria y alcalosis metablica """ . (.......C CQ)-f ~ e) Acidosis respiratoria aislada \\!.. \

:!~ulentes

p02:~06;

conr;le~<u;:~1la.

8) Joven de 15 aos, con cuadro de 2 dias de evolucin de odin~la,-flebre levey rinorr~a. Presenta varios episodios de hematutla=iBdolr8. El examen fisico reSmt normal. La anaUtlca demuestra hemograma, funcin renal y heptica normales. El sedimento dc3 orina muestra eritrocitos dlsmrficos' abundantes. El diagnstico mas probable es: /. a) Enfermedad de cambios minimos (nefrosis lipoidea) A b) Glomerulopatia membranosa . e) Glomeru pidamente progresiva . n 1 A ~ }_ ~) nfe de ~C'v~ <..\,~ __ p __}u~ ~o erulonefritis aguda postestreptoccica v-') ~ ~ ~'""'

Lo,

e entre las siguientes alternativas, el mejor examen para determinar el origen prerrenal de una suficiencia renal aguda es: a) El examen fisico b) La relacin entre BUN y ereatinina e) El elearence de ereatinina de 24 horas ~fraccin excretada de sodio e) La ori i completa

1 O) L causa ms frecuente de insuficiencia renal crnica, en Chile, es:


pertensin arterial iabetes mellitus e) Nefropatia tubulointersticial d) Glomerulonefritis e) Pielonefritis a repeticin Preguntas Urologla 11) Paciente fumadora de 65 aos de edad, consulta por~. sin otros slntomas. El seimento de otina muestra ms de 1p0 eritrocitos por campo, slo dismg.rf.ia y la ecograffa reulta nor:mal. El diagnstico ms probable y el ex~ para confirmarl~ son respectivamente: ~~litiasis y radiografia simple de abdomen "'b)JRfeccin 11rjnaria baja y YI=GGl:lltiuo e) opta --Jd) ncer de e) retritis gonoccica oligosintomtica y cultivo de Thayer-Martin 2) Mujer de 60 aos, consulta porque hace 2 semanas inicia escapes de pequeas cantidades de orina cuando tose o se rfe y cuando levanta objetos muy pesados. Ultimamente sto le causa vergenza. La primera aproximacin diagnstica debe ser solicitar: -~mento de orina y urocultivo b) Uretrocistomanometrta e) Ecografia renal y vesical d) Papanicolau e) Estradiol, FSH y LH a)

.X

13) Paciente de 26 aos consulta por compromiso del estado general, asociado a dolor y aparicin de lesiones cutneas en el pene. Refiere tener relaciones sexuaiEts con distintas mujeres, sin proteccin. Al examen se constata paciente febril hasta 38,5C, con adenopatfas inguinales bilaterales, dolorosas. En el glande y cuerpo del pene, se aprecian mltiples lceras confluentes, dolorosas. El diagnstico ms probable es: a) Chancro sifilitico b) Chancroide e) Gonorrea d) Linfogranuloma venreo

~.

~ (>'

"
) Paciente de 20 aos, sufre accidente de .trnsito, evoluciona con anuria de 6 horas de evolucin, a esar de deseos Intensos de orinar. Presin arterial y pulso normales. Al examen se observa salida de sangre fresca por uretra. La medida inicial a seguir es: J ~ ~ rv-C/~ 0 -~ J~~ ) 1 stalar sonda Foley t.J O[c. .[~
olicitar uretrocistografla retrgrada e} A ministrar suero fisiolgico hasta ranudar diuresis d} Solicitar un nuevo ECG e) Evaluar respuesta a cambios de estilo de~ vida y aspirina, en 1 mes

~ ~ ~

~
~

"
~

C(r.S>,f/~"e

C/{

J'c4 v<-

~ ~

fA
~
~
~

~
~

~
(A
~

...?'\b} l:!_ormonoterapia A-e,~-r ~ Q.N ~ ~ fL S~ -/e) QUimtoliapia y"rt:ldioterapia d) Cirugfa radical ms quimioterapia postoperatoria ~atamiento paliativo con analgesia y ma1nejo de complicaciones

17) Paciente de 86 aos, con cncer de prstata, diagnosticada por biopsia y APE mayor de 100 ng/ml, en contexto de dolor lumbar de dificil manejo. La clntigrafa sea es compatible con numerosas metstasis lumbares. El tratamiento m!s adecuado es: a} Prostatectomla radical ms linfadenectornla plvica \ .~ ~ G<:> ~ ~~

f . .J"

18) Pac nte de 25 af\os, consulta por haber notado aumento de volumen testicular derecho. La ecogr a testicular muestra tumor de aspecto slido en testrculo derecho. La conducta Inicial ms ade ada es:
ontrol ecogrfico en 6 meses Realizar biopsia por puncin del tumor e) Realizar tumorectomla y biopsia d} Solicitar marcador~,s-tlmorales (AFP, HGC y LDH) y decidir cirugla segn resultados ~-alizar orquie9tomla derecha, por vfa inguinal, independietemente del resultado de los marcadores tumorales. . .....a)

fA
~

~
(''

fA
~ ~
~
~

19) Nio,.d~ 3 meses, en el examen de rutina no se palpa el testfculo derecho. Al examinarlo es posible palpy-16 en relacin a la salida del conducto inguinal y se puede descender fcilmente hasta el es (oto, sin embargo vuelve al conducto Inguinal al soltarlo. Cul de las siguiente opciones 1'1 resenta mejor la conducta que debe tomarse?
+-41'-J-1-1-anquilizar a la madre e indicarle que esta condicin no tiene ninguna repercusin negativa sobre el actante. b) Solicitar ecografia inguinal y testicular e} Realizar orquidopexia quirrgica al cumplir 18 meses d) Realizar orquidopexia inmediata e) Realizar orquiectomia para evitar el riesgo de cncer de testrculo

1/A
~

e
~
~

~ ~

~
~

~
~

~
) Paciente embarazada de 35 semanas, inicia compromiso del estado general, fiebre hasta~ disuria dolorosa. Al examen fsico se observa paciente en buenas condiciones generales, normotensa, con FC:90x'. puo percusin (+). El sedimento de orina muesti-a leucoclturia y el urocultivo resulta positivo para E. coli multisenslble. _El antibitico de eleccin para el tratamiento de esta condicin, por va oral, es. a) Amoxicilina b) Ciprofloxacino .... _~~ j?entamicina
~efadroxilo

""

~
~

~
~
~
~
~

e) Nitrofurantoina

..,
~
~ ~
~

~
~

,.,
~ ~

~
~

~
~
~

~
~
~ ~

~
~ ~

~
~
~

~
~

~
a 21
UNTAS PGINA EMN PSIQUIATRfA, SALUD PBLICA Y ESPECIALIDADES

~
~

~
~
~
~
) Paciente de 53 aos, lavandera, abandonada por marido alcohlico. Tiene 6 hijos, el mayor de los cuales es alcohlico y los dos menores tienen serias dificultades en el colegio. Desde los 20 aos presenta episodios do vmitos, dolores abdominales y musculares, intolerancia a diversos alimentos, palpitaciones, mareos, dificultad para deglutir, visin borrosa y menstruacin dolorosa. Consulta por prurito anal y dolor rectal, sin causa orgnica demostrada por exmenes. Se aprecia triste y se queja de insomnio de conciliacin. Su personalidad es introvertida, pesimista, ansiosa y demandante de afecto. El diagnstico ms probable es: a) trastorno de ansiedad generalizada b) episodio depresivo mayor, leve, con somatizaciones "):(trastorno de somatizacin. d) tras mo hipocondriaco e) t tomo de personalidad limtrofe a esquizofrenia simple se ~aracterlza por: ausencia de sintomas activos ) predominio de sfntomas primarios e) desintegracin rpida de la personalidad d) predominio de sintomas secundarios e) a encia de defecto esquizofrnico El tratamiento Inicial para un paciente con trastorno de pnico es: a) Diazepam 20 mg cada12 hrs 1 Clomipramina 75 mg cada 8 hrs b) Alprazolam 1 mg cada 8 hrs 1 Fluoxetina 20 mg cada 12 hrs omip mina 150 mg cada noche 1 Clonazepam 2 mg cada 8 hrs pra am 0.5 mg cada 8 hrs 1 Fl~a 1O mg cada maana e) Dia pam 5 mg cada 12 hrs 1 Fluoxetina 1O mg cada 8 hrs 5)

~ ~
~

~
~

~
~

~
~

~
~

~
~
~
~-

wp ,:1

, ll~ 1 ~vr-

seregosi~tnica

diferencia de la ansiedad generalizada, la angustia en la crisis de pnico se caracteriza por:

~ ~~).gf'.JQ
..1
;rv.J:)

c,l.t ('cvJNCfi
e~
- \

~
~

b) ser exces1va e) relacionars..econ events cotidianos ~o teneAilotivo aparente e) inclujr/xpectacin ansiosa


1'

N'-' St

... .

rvo~

L--

...

~
~

~ ~
~ ~ ~

un servicio de urgencia, adems det hospitalizar y realizar monitoreo cardiovascular, cul es la ucta inmediata frente a un paciente con intoxicacin por cocarna? \ .n ., ministrar antagonistas de opiceos ~ ~-- ~~ b} Administrar neurolpticos ~Averiguar consumo de otras sustancias ~b . ofAveriguar consumo de otras sustancias y administrar antagonistas de opiceos. e) Averi9Liar consumo de otras sustancias y administrar neurolpticos.

,.

7)

d~l es el tratamiento de eleccin para un paciente de 35 aos, con crisis de angustia intensa, sin

~
~

ftacin, en el que se ha descartado patologra somtica? Clonazepam 2 mg oral Alprazolam 2 mg oral e) Diazepam 1O mg intramuscular d) Clorpromazina 25 mg intramuscular ~orazepam 2 mg sublingual

~
~

s:

6) En un lactante de 4 meses con una desviacin ocular permanente, ia conducta correcta a seguir

derivar al especialista al detectar la patologia b) hacer un covert-test y controlar cada 3 meses e) esperar a que cumpla 6 meses antes de derivar d) esperar a que cumpla 9 meses antes de derivar e) esperar hasta el ao de edad antes de derivar 17) C. es el diagnstico ms probable en un paciente que presenta aumento de volumen infla atorio en el territorio naso orbital izquierdo? a) acrioadenitis aguda. Meibomitis aguda. .... acriocistitis aguda. d). acriocistitis crnica fistulizada. . e) Carcinoma basocelular infectado.

torrea. \ . ..... rtigo.--? OlJ-;0 " ~ e) Paresia facial. -~ .-:LCJ.D S,.... ~ /d) Nistagmus espontneo_.!~) ~ -~ e) Cefalea intensa. ___.

18) Cul de los siguientes sintomas o signos puede encontrar.se en una otitis media aguda no ~ complicada? tO..(.otV-(_.C!l { ~ ~ r / ~)'fi O C\J ~

~
'-"l

.41!1!

J
J

19) Nio de 5 aos consul.ta por presentar rinorrea mucopurulenta

varios dfas de evolucin. Cul es el diagnstico ms probable? a) Sinusitis de origen dentario. c~uerpo extrao nasal. e) Tumor nasal. d) Fistula de liquido cefalorraquideo. e) Rinosinusitis aguda.

ftid~ por la fosa nasal izquierda, de

20) Un pre-escolar de 3 aos presenta placas eritemato-descamativas pruriginosas en mejillas, pfiegues antecubitales y fosas poplfteas, recidivantes, de dos ;aos de evolucin. ~~ul es el diagnstico ms probable? a) Psoriasis J2) Dermatitis de contacto --Roermitis atpica Ci)bermatitis seborreica e) Sarna

~
8) de 37 aos, previamente san(). Su padre falleci en un accidente hace 2 meses. Desde entoJ(ces presenta insomnio de conciliac:in y despertar precoz, angustia, anhedonia, anorexia, faijgabilidad, alteraciones de la atencin y concentracin, tristeza e ideacin suicida franca, todo lo 6alle impide realizar su trabajo. Cul es el diagnstico ms probable? ) Trastorno de estrs post traumtico b) Trastorno de adaptacin pisodio depresivo pisodio bipolar mixto e) Reae6in a estrs agudo ,_,, /1' '"!.. 1_Qu sucede cuando se enfatiza la prevencin y promocin de la salud? ) Aumentan las secuelas, pero baja la incidencia ) Aumentan las secuelas, pero baja la prev.alencia e) Requiere menor cobertura de la intervencin ~ d~ Disminuye el costo pero aumenta la prevc:tlencia ~ -~ ~ejora el costo/efecto de la intervencin
niv~les. de complejidad creciente bajo el concepto de red permite principalmente: a) tener un claro diagnstico del perfil epidemiolgico b) disear planes de fomento de la salud efectivos e) cumplir con las polticas ministeriales ,->.. 11\ n-. ~A.... tUizar el personal en forma efectiva y en :su mxima capacidad - ' ' ' r , f_...;,e) el uso racional de todos los recursos del sistema

PacJin~e

~ ~ ~

ff'

~X
~
~

1O) La organizacin del sistema de salud en

X-

~ ~

..... /

~
~

~ ~ ~ ~
~ ~
~ ~

11) En la programacin de actividades hospitalarias se utiliza el indicador lndice ocupacional o Porcentaje de Ocupacin , que relaciona: ~ a) Nmero de consultas con total de horas d3 box disponibles ~Olas cama ocupados con nmero de egresos hospitalarios ft) N. ero de cirugas realizadas con total do horas de pabelln disponibles d) oras ocupadas con horas totales contratadas de los funcionarios ~ las cama ocupados con total de dfas carna disponibles 12) La tcnica denominada pareamlento (matching) se utiliza en los estudios epidemiolgicos analiticos para controlar: ~a) errores aleatorios. , ~variables de confusin. e) pr ida de sujetos drante el estudio. d) s go de los observadores. e) precisin de los resultados.
3) Qu tipo de estudio es el ms adecUc1do para estimar la proporcin de individuos que tiene una enfermedad en una poblacin? a) Descriptivo. ::bkDe prevalencia. e) De incidencia. d) Casos y controles. e) Ensayo clinico.

(A
~
~ ~

V
,!'.

14) Cual de las siguientes patologfas tiene peor pronstico:

e
~

a) Queratosis actnica b) Carcinoma basocelular ~) Queratosis seborreica ~evo melanoctico ~ e)~noma espinocelular

e e
~

~ ~

~
Prueba x UROLOG(A
1) Paciente de 57 aos, sin antecedente! S de importancia. En control de salud se pesquisa APE: 5,4 ng/ml. El tacto rectal es compatible con hlperplasla benlngna de la prstata grado 2. No presenta sfntomas urinarios. La conducta ms adecuada es: a) Mantener control de APE y tacto rectal anual, sin tomar ninguna medida adicional b) Realizar nuevo control de APE en 3 meses nielar tamsulosina ealizar biopsia prosttica transrectal Realizar reseccin trasuretral del adenCJma prosttico

~ ~

~ ~
~

f!'
~

e
~
~

2) Un paciente de,..3.Q..afios presenta aumento de volumen testicular, de consistencia aumentada. Al examen ffslco se palpa una h!mor~ en relacin al testlculo derecho, de cerca de .~e..J!imel_!'~. La conducta ms adecuada es: , a) Solicitar ecografa testicular ,. / b) Solicitar alfa-fetoporteina y subunidad beta de gonadotrofina corinica humana e) Realizar biopsia por puncin con aguja !}ruesa d) Realizar tumorectomia e) Realizar orquiectomla radical 3) Un paciente cursando na obstruccin..Jntestinal, recientemente operada presenta oliguria y creatinina e 2,5 mg/dl. Se constata K+ urinario:~mEq/1, Na+ urinario:. 8 mEg/1, Cl- urinario: 1_3 mEq/1, '-/ s!fn sedimento e orina normal. La patologia que meJor explica la ~levacin de la creatinina es: -"' a} Insuficiencia renal aguda prerrenal ,. Necrosis tubular aguda e) Nefritis intersticial d) Pielonefritis aguda . e) !"'suficiencia renal aguda postrrenal ,... n paciente de 45 aos es hospitaliza do para realizacin de una colecistectomfa laparoscpJca lectiva, debida a una colelltiasJs sintomtica. En el postoperatorlo presenta retencin urinaria, por lo que se Instala una sonda Foley. Algunae1 horas despus el paciente presenta importante ~jgr_en rel!lcin a la sonda. Al examen flslco se aprecia el g~nde aumentado de tamao, edematoso y doloroso a la palpacin, con el prepucio retrafdo. El diagnstico ms probable es: a) Balanopostitis aguda b) lnfe9cin urinaria baja .. e) P~rafimosis d) yretritis por sonda /'Trombosis de la vena testicular Un hombre de 58 aos, diabtico tipo 2, en tr~tamiento con glibenclamlda, ms metformin!J desde hace varios aos, presenta edema facial y de extremidades Inferiores, asociado a orinas espumosas. El examen ffsico no aporta mayor Informacin. Se solicitan exmenes que constatan un clearence de creatinina de~ ml/mln, BUN: ~3 mg/dl, proteinuria de 24 ho.ras de 5,1 gramos, sedimento de orina sin alteraciones y albumTnemia de 2,2 g/dl. El diagn6stico ms probable es: a) Nefropatla membranosa b) Glomeruloesclerosis difusa e) Nefrosis lipoidea d) Insuficiencia renal terminal e) Nefritis intersticial

e
~

~ ~ ~
~
~

~ ~ ~
~
~
.

~ ~
~

) J:
~

~
~

~ ~
~
~
/

~ ~

6) Una paciente de 30 aos se realiza una1 ecografra abdominal como parte del estudio de un cuadro crnico de dolor abdominal, con examen ffsico normal, probablemente secundario a una colopatra funcional. En dicha ecografa se identifican 2 quistes renales izquierdos, de 2 y 4 cm cada uno. La conducta ms adecuada, respecto a este hallazgo, es: ................. ~} Emar-a-demicllio,...sit+ReGaSidad de maycu: estudio ni tratamiento 1 e , b} Controlar con ecografas peridicas 1'-' 0 ~e) Solicitar estudio gentico para descartar enfermedad renal poliquistica del adulto . d) Solicitar TAC abdominal e) Resolver quirrgicamente

~ ~

e e

,.f
~
"")
' ' t

7)

fsj~~ ~e aprecia el escroto aumentado de tamao, cQn prueba. de transilumlnacin positiva. El

u./hombre de 68 aos presenta aumento de vglumen lndolrmu:tel testfculo derecho. Al examen


.

~
~

2ragnstico ms probable es:


~) Cncer testicular

b) Hernia lnguinoescrotal e) Varicocele d) Torsin testicular /e) Hidr ele ul es el tratamiento ms adecuado para evitar el hlperparatiroidismo secundario a la uficlencia renal crnica, en un paciente con clearence de creatlnina de 40 ml/min y producto calcio sforo de 45? a) Dieta baja en calcio y baja en fsforo b) Calcitonina -e) Carbpiato de calcio oral d) Diyreticos de asa D'rticos tiazidicos

~
~ ~

~
~

~
~
~

~
~ ~
~

a alternativa que ms orienta a una glomerulonefritis aguda postestreptoccica, en un nio que sa con un sndrome nefrtico, es: . nticuerpos antinucieares b) Elevacin de la creatinina e) Cultivo faringeo positivo para Streptococcus pyogenes . ....., d)~ thplementemia transitoria H'poc . . {.,v~A.F ~ e) Cilin ros hemticos

J:

1 O) n paciente presenta fas~culaciones .en vaos grupos mu!sculares, as'oclados a hiperreflexia. La 1 eracin hidroelectroltica ms probable es: Hipocalcemia }'\.1.0 \~ ft::,. 'f'J->-(L CCJ_,. Hipokalemia . \ \ e) Hipomagnesemia d) Hlpercatcemia e) H;perkalemia / 1.) El examen de mayor rendimiento para confirmar la sospe~ha de urolitiasis es: a) Sedimento de orina b) Radiograffa renal e) Ecografia renal y vesical .....-d) PieloTAC e) Pielografia de eliminacin, con contraste endovenoso

~
~

't.l.

(._.

~
~

~
~
~

~ ~

12) ~~ conducta ms adecuada ante un paciente de 1O aos, ceursando con un sindrome nefrtico
puro, asociado a hipertensin y hematuria es: .... Iniciar con corticoides orales Administrar pulsos entiovenosos de metilprednisolona Iniciar inmunosupresres, asociados a corticoides endovenosos d) Iniciar una asociacin de corticoides, inmunosupresores y antibiticos, con cobertura para estreptococo betahemolitico del grupo A _.)!)Derivar para realizacin de una biopsia renal 13) Cul de los siguientes cuadros clfnicos es ms sugerente de una incontinencia urinaria de esfuerzos? , a) Mujer c;le 70 aos, demenciada, que presenta escapes de grandes cantidades de orina 3 o 4 veces al dia, razn pr la cual debe usar paales b) IVJuj'er de 20 aos con escape c~ntinuo de orina en pequeas <:antidades, lo que la obliga a ocupar \

~
~

~ ~

~
~

~ ~
~

p~(ectores

\_,e) Mujer de 50 aos que en algunas ocasiones siente deseos incCJntrolables de orinar, por lo que no alcanza a
ir al bao, orinndose antes -d) Mujer de 55 aos con escape de pequeas cantidades de orina, al realizar tareas donde requiere hacer mucha fuerza y al toser _ e) Mujer de 60 aos con grandes escapes de orina en algunas oeasic;mes, especialmente cuando se rre

~ ~
~

A)

~
~

... ~

~
14) as causas ms frecuentes de insuficiencia~, en nifios y adultos, son re pectivamente: ..____._,_... Prerrenal y prerrenal b) Malformaciones nefrourolgicas e hipt~rtensin e) Glomef\llonefritis y diabetes ..,el) Malfo / aciones nefrourolgicas y diabetes e) Gl erulonefritis e hipertensin arterial

~ A

!:

~
..

~ ~ ~

Cul de las siguientes no es una Indicacin de hemodillsis de urgencia? ) Hiponatremia grave, que no responde al tratamiento mdico b) Hiperkalemia grave, que no responde al tratamiento mttico ._,... e) Pericarditis urmica .../ d) Edema pulmonar que no responde al tratamiento mdico e) Acidosis grave, que no responde al tratamiento mdico "\../"'

e
~
~

16) El diagnstico de la uretr is onocclca se realiza preferentemente con: a) Cultivo de secrecin uretral en hom resyla cllnica en mujeres ~ Tincin de Gram de secrecin uretral en mujeres y la clnica en hombres t).>Cultivo de secrecin uretral en mujeres y tlncin de Gram de secrecin uretral en hombres , d) Cultivo de secrecin uretral en hombre!; y tincin de Gram de secrecin uretral en mujeres e) L~rblinica en ambos s.exos

ff::
(!'

11 Un hombre de 60 aos consulta por disuria de esfuerzos, con disminucin del calibre miccional,
acturia y goteo tenninal. El examen ffsico no aporta mayor infonnacln. El diagnstico ms probable es: a) Cncer de prstata b) Estenosis uretral . ....-e) Hiperplasia benigna de la prstata d) Infeccin del tracto urinario e) Uretritis por clamydia

~
~

~
~

~ ~ (!'
~
~

( ~~~:':fea~~~~::' primario

18) La causa ms frecuente de h!e_ercalt~mla en pacientes hospitalizados es:

~""' !M>.r p>k.~a(Y.:.

.. , ~-..;.~"'-"en~ t"

~ ~

f!'
~

) A
/

e) Insuficiencia renal aguda prerrenal


lnsufi~incia renal crnica Crer
/

Un paciente de 20 aos presenta presiones arteriales elevadas en varias tomas, realizadas en dias stintos. El examen fsico es normal. Se solicitan hemograma, creatinlna plasmtica, ~UN, sedimento e orina, electrolitos plasmticos, lndice de aldosterona 1 actividad de renina plasmtica y etranefrinas urinarias, Identificndose slo hipokalemia de 2,8 mEq/1, con normalidad de todos los ros parmetros. El diagnstico de sospecha es: a) Hipertensin arterial esencial , b) Hipertensin arterial renovascular e) Hiperaldosteronismo primario d) Sindrome de secrecin inadecuada de ADH e) nfermedad de Berger

fA
~ ~
~
~

1 Una nif\a de 2 aos presenta un cuadrro de 2 dias de evolucin de disuria y orinas de mal olor. El 20)

examen fsico resulta normal. Se solicita urocultivo por sondeo que resulta positivo para E. coli multisensible, con 30.000 unidades form~tdoras de colonias. La conducta ms adecuada es: a) Observar, sin necesidad de mayor estudio a menos que recurran o progresen los slntomas b) Observar evolucin y controlar nuevo uroeultivo en 7 dias p en caso de fiebre e) Iniciar nitrofurantoina oral y controlar con urocultivo en 7 dias d) Iniciar ceftriaxona endovenosa, controlar c:on urocultivos y solicitar cintigrafia renal esttica ,. e) Iniciar cefadroxilo oral, contr~lar con urocullivos, y una vez resuelto el cuadro, solicitar ecografia renvesical y uretrocistografia retrgrada elidoscpica

(A
~-

~ f!'

e .,e____

~
Prueba 21 GINECOLOG(A y OBSTETRICIA, Preguntas 2009

~r ~

'/

~ ~

~ ~

~ ~
~

t.('a multrpara de 38 aos, cursando se cuarto embarazo, de 33 semanas consulta por aparicin de rito pal!!loplantar intenso, mayor en las noches, asociados a Ictericia. El examen f[sico no aporta yor informacin. El diagnstico ms probable es: Sndrome .e(e HELLP . Hgado (aso agudo del embarazo 9~ ~ar s viral fl.'~o~~ tasia intraheptica del embarazo _ . --\,/e) ls munizacin Rh na paciente de 22 aos, cursando embarazo de El registro tresante muestra desaceleraciones :tar as en relacin al80 de las contracciones uterinas. La frecuencia cardica basal deTieto es 13olpi y la variablli a es normal. La cond~cta ms adecuada es: /' ~. .~ealizarph tacto vaginal para determinar la vi a de interrupcin Inmediata ' b) Solicit ( ecodoppler umbilical e) Rear ar perfil biofisico d) Es rar evolucin espontnea e) ministrar corticoides endove1 iOSOS'j'"'flR1ieitiees ~ tocultieos

2)

trmln~

~ ~

~
~

~
~

(f'
~

~ ~

Paciente multfpara de~n tr.abaln de parto, cuyo monitoreo muestra una EC fetal basal de . 130/min, buena variabilidad y pres~a de aceleraciones. La rotura artificial de membranas da salida a un liquido c~m meconio fluido, ~marrillo verdoso_. Cul es la conducta obsttrica a seguir? a) Interrupcin inmediata del embarazo mediante cesrea b) Reaniry{acin fetal in tero con reevaluacin en 20 minutos ~a ifnacin fetal in tero seguida de interrupcin del embarazo ~~ :!~ ener una conducta expectante y permitir que contine el trabajo de parto e) 1 errupcin inmediata por la via ms expedita Paciente de 20 aftos, con menstru~lones .previas irreg llares resenta atraso menstrual de 6 semanas por lo que se realiza test de embarazo, que res~l posltiv Se realiza ecografa transvag,itJal que no nutestra..sgo embrionario intraute.dn.o. Slo se o s rva endometrio enQrosado, sin liquido libre peritoneal. Se controlan niveles d~ gonadotrofina corinica (HCG,) cuyo resultado es 1.000 Ul/dl ._.., ~bnducta ms adecuada es: e d? Solicitar nuevos niveles de HCG y eco~)rafla transvaginal en 2 dfas _,.. 1111 CtJt.>.-t. ~ J.-> /b) Indicar metotrexato f e) Resolver quirrgicamente, mediante laparoscopfa B tt(..,(~ d) Resolver quirrgicamente, mediante laparotomla e) Dejar: ev,eiGcin es~nea, sin necesidad de nuevos controles ni tratamientos

-~
~
~
~

Jbe

~
~
~

~ ~

(!!'
~
~
~-

/ e)'i:u

5) Uciente de presenta de 4 meses de duracin. El examen ffslco es normal, fio que se solicita test de embarazo, que resulta negathzo; prolactma y TSH. gye resultan normales; rueba d~ progesterona, que resulta negativa y prueba de estrgenos + progesterona, que resulta ositlva; F~~ y LH. que resultan elevadas. El diagnstico ms probable es: Al ..-;a 'N-"~C/: A~Q'II:JI8CIOI pl l- Ir'.. AJ(07 ~v-Hipogonadjsmo hjpotalmico ~,r~ (71-q- - 1 ~~~o~ e~ de o!'~en u~erln? ~ ~ QO t19 ,/0-dtl'Qsufi ienca. ovanca pnmana 1'"' l

3hf\~s

~menorrea

. . .

~ ~ ~

na adolescente de 17 aos, sin antecedentes de importancia, es trafda por su madre ya que an no resenta menstruacin. No presenta otros sfntomas y su desarrollo puberal es normal. Al examen ginecolgico se observa himen perforado y el resto del examen flslco no muestra alteraciones. La conducta ms adecuada es: a) Iniciar anticonceptivos orales b) Solicitar ecografla transvaginal , e) Solicitar TSH, hemograma, fndice de andrgenos libres, prolactina, FSH, LH, cariograma, androstenediona y J_;--hidroxiprogesteron- .Ytf}Soli.citar prueba de progesterona e) lniciarpulsos de agonistas de GriRH

~
~

..f!j_ __

~~_matocrito ~ -T- .r~._.o~.~


d) Filtracin g!_9merular e) Fosfatas s alcalinas .

Cul de los siguientes parmetros disminuye normalmen1te durante el embarazo? a) Frecuencia cardlaca b) y_91emia l.. 1.. o : o . . " ./

9) Un paciente multipara de 2, cursando embarazo de 33 semanas fue diagnosticda de diabetes ges cional mediante teSfae tolerancia a la glucosa, a las 28 semanas de gestacin. Se Inici dieta, sin e argo persiste con glicemias postprandaales cercanas a 1!50 mg/dl. La conducta ms adecuada es: Proseguir con dieta y ejercicios por 4 semanas ms y controlar b) Iniciar metformina ' ~~ ~lar glibenclamida ~~!~iar insulina e) Iniciar una asociacin de glibenclamida e Insulina

10) Una pacient! primiges\a, de 30 semanas de gestacin, a~lntomtica, presen~ presin arterial de 150190 en una oc El resto del examen fisico es normal. Se contrQia proteinuria cualitativa que resu a levemente positiva. La conducta ms adecuada es: ~a) Iniciar diurticos, asociados a iECA b) Indicar dieta y ejercicios --""'e car semihos italizacin de 6 horas, controlando la ;~rteriat en yarjas ocasiones ~ / l.niciar alfametildopa y solicitar examenes que determinen la gravedad del cuadro e) Administrar corticoides e interrumpir el embarazo 11) Una mujer es diagnosticada dtt.Y!!! en relacin a un screcmjng realizado en la primera consulta al obstetra. Se encuentra asintomtica. Cul de las siguientes alternativas supone un MENOR riesgo para el feto, al momento del parto?: Y a) Ausencia de infecciones oportunistas b) . elper CD4(+) mayor a 1.000 ~e ar a viral indetectable . d)r ratamiento anttrre rov1ra ..4) Cesrea
12) Una paciente obesa, con antecedente de cono cervical por una NIE 3 y de vrices en las extremidades inferiores ha presentado 21bgrtos espontnecls a las 19 y 17 semanas respectivamente. En ambas oportunidades present escaso dolor y pcas contracciones uterinas. El diagnstico ms ,/probablees:

~ ~SMa

~ ~bortos de origen gentico - d) Insuficiencia ltea ~Incompetencia cervical

~~~ndrome de anticuerpos antifosfolipidos

_il\t_,~,~ ~
-

t.~~r ~ .
~

..-:-r

~ ...l\ ;.. ~~ ~.

.,CJN" "\b

~.l 1 _ \ ~ ~

13

Identifique. la entre la historia cUnica y la patologfa mamaria Mujer de 65 aos, con ndulo mamario irregular, que ha aumentado de tamao y se asocia a "piel de naranjan de la mama comprometida - Cncer de mama infiltrante ..._/"' 'er de 20 aos con ndulo mamarlo de 3 cm, y consistencia gomosa, fcil de movilizar- Fibroadenoma ...../ er de 45 aos con mastalgia premenstrual y rl}amas con ndulos d~uef\o tj-f!f\o, bilaterales. t\r--0-~ -1--o rc;ce:;::""\\.J ~ ;~~ L-lJ es - Carcinoma ductal in situ x d) Purpera con aumento de volumen doloroso de 4 cm e11la axila derecha - Glndula mamaria accesoria,__....e) Purpera con eritema y dolor mamario bilateral- Mastitis linfctngtica

asoclaci~

~
4) El una ecografa del primer trimesttre de un embarazo gemelar se observa el signo lambda. Qu tipo de embarazo gemelar es? __ ~ a) Dicigtico ----{ ~\ _ ..o~-

~ ~

~-

~ !A

b) ~ocigtico ~~~

d) Monocorial e) Monoamnitico

-~ n '~
\~ ..._o.N

v-P~

~ ~

15) Una paciente de 33aos, presenta una historia de dismenorrea crnica, que ha ido en aumento. Se realiza una Ecograffa transvaginal que demuestra un lllm~r ovrico derecho de 3,5 cm, gufstico, de contenido en vidrio esmerilado. Se solicita Ca-125 que resulta moderadamente aumentado. La ~ conducta masadecuada es: a) Realizarertexectoia bilateJal y biopsias perito11eales ~~ ,.b) Iniciar anticonceptivos orales y realizar nueva ecografa en 3 meses .r e) Iniciar anlogos de GnRH a permanencia d) Realizar puncin percutnea ~~lizar t\J=ReFesteFRa la~aFeso~ioa -

16) a mujer de 30 aos, cursando un embarazo de 7 semanas, consulta por metrorragia y malestar h" ogstrico. Al la especuloscop[a se observa salida de sangre por el orificio cervical externo. El tacto /'\ \ _?~nal no demuestra cambios cervicales. La conducta ms adecuada es: ~_:_i:-R; ' ~ .'\ ~~ealizar ecografia transvaginal ~~o~ CJ. o :b) Solicitar niveles plasmticos de subunidad beta de gonadotrofina corinica ~ L. . ~./~ (... ... __ e) Realizar legr~~o uterino -l- ~\_g..._' _ ~~~- e A~tinistrar-5rogestgenos orales -q_e '-t"'- ~.g ( ~ ~ r,._nvia[ domicilio con reposo e hidratadn -_ ~- ~\o'-0
~

-. , ..
~

--

(!!A
-

~
~ ~
~

1l na gestante de 41 semanas inicia trabajo de parto. Presen~ 4 cor:.tr:aseioRes eada 19 RtiRYtos, on dilatacin cervical de 7 cm, membranas rotas y feto en seg(ndo plao de Hodge, con presentacin de frente, en posicin nasoiliaca izquierda anterior. Lconducta ms adecuada es: a) Observar evolucin espontnea ~ b) Realizar prueba de parto con aceleracin occitcica y anestesia epidural e) Girar la cabeza hasta posicin nasopbica y dejar evolucionar d) Realizar forceps ,P) Realizar cesrea
18) Una mujer con promiscuidad sexual present.a UIJ~~-:~5'-C"i~-crdolorosa, indurada, en relacin al labio mayor der.echor:c"y~ no s;mgra ni presenta exudado. El diagnstico ms ,../' probable es: -. ~ a) Gonorrea J, o

~
~

~nulomavenreo

~~\. ~

~~~~~ide e) Donovanosis

~
~
(4'

~
~
~

~
~
~
~

de 16 aos, cuya menarquia fue oligomenorrea y polimenorrea,. 20) na si otro compromiso sistmico. Cul et; la conducta mas adecuada? ) Realizar un estudio endocrino de su funcionamiento reproductivo b) Estudiar alteraciones genticas de su desarrollo gonadal ) T ar con anticonceptivos para regularizar su ciclo servar la evolucin espontnea e) Realizar ecograffa con seguimiento folicular

adol~scente

~'@resenta

(!$'

r'.

~
PRUEBA 11, PEDIATR[A + GINECOLOGiA +OBSTETRICIA 1) Cul de las siguientes patologas suele presentarse como metrorragia? a) Hiperplasia endometrial b) Miomatosis uterina e) Endometriosis d) Plipo endometrial e) Adenomiosis
2) Una mujer est en trabajo de parto, doasde h~ce algunas horas. Presenta 2 contracciones cada diez minutos, y el tacto vagina demuestra cuello uterino completamente borrado, con dilatacin de 10 cm y se palpa la cabeza fetal en presentacin de vrtice, en espinas -2. Se administra la anestesia epidural, se inicia aceleracin con. oxitocina, logrando 4 contracciones por minuto y se realiza rotura artificial de membranas, dando salida a liquido amn ltico claro. Se reevala luego de 2 horas sin presentar cambios. La conducta ms adecuada es: a) Dejar evolucin espontnea b) Realizar forceps e) Suspender el goteo oxitsico d) Aumentar el goteo oxitsico e) Realizar cesrea

e!'

~ ~

~ ~
~

tf.'
~ ~

~
~

~
~

~
~

3) Cul de los siguientes es un signo de asfixia neonatal? a) DIP 1 b) pH de sangre de cordn de 7,2 e) Poliglobulia d) Signo de Babinsky e) Meconio espeso en el lquido amnitico 4) Una paciente de 55 aos, con sintoma.s climatricos, acude a control ginecolgico, detectndose una pequea lesin solevantada, de 1 cm en la zona posterior del cuello uterino. Su ltimo PAP se lo realiz hace 6 meses y fye informado como normal. La conducta ms adecuada es: a) Solicitar nuevo PAPen 6 meses b) Solicitar nuevo PAP inmediatamente e) Derivar a colposcopia y biopsia d) Realizar conizacin cervical e) realizar histerectomla total 5) Un lactante puede levantar la cabeza y mirar a su alrededor, sin embargo an no es capaz de sentarse. Qu edad tiene? a) 1 a 2 meses b) 3 a 4 meses e) 5 a 6 meses d) 7 a 8 meses e) 9 a 10 meses 6) Una paciente de 28 aos, obesa, cursando embarazo de 8 semanas, se realiza glicemia de ayuno que resulta en 108 mg/dl. No presenta sfntomas y el examen fsico no aporta mayor informacin. La conducta ms adecuada es: a) Mantener los controles rutinarios del embarazo, sin necesidad de estudios ni tratamientos adicionales b) Solicitar nueva glicemia de ayuno e) Solicitar test de tolerancia a la glucosa oral d) Iniciar dieta y ejercicios y controlar con nueva glicemia de ayunos en 2 semanas e) Iniciar insulina 7) Un lactante de 7 meses, previamente sano, presenta rinorrea, tos y luego dificultad para respirar. Al examen fsico se aprecia taquipneico, con retraccin intercostal y se ausculta mltiples slbilancias inspiratorias y espiratorias. El agente causal ms probable es: a) Neumococo b) Estreptococo betahemolitico grupo B o} Virus respiratorio sincicial d) Virus parainfluenza e) Virus influenza

~ ~

~
~
~
~

.!!!'
~
.~

~ ~ ~

~
~
~

~
~
~
~

~
~

(IP'

~,

_____

8) Una mujer acude a control obsttrico, con un embarazo avanzado, que no ha sido controlado previamente por mdicos y que no se ha realizado exmenes de sangre ni ecografas. Recuerda el da de su ltima menstruacin, que fue hace 32 semanas y 5 dfas y refiere que previamente sus reglas eran regulares. Tambin refiere que hace 13 semanas y 3 dlas Inici la percepcin de los movimientos fetales. La mejor forma de determinar la edad gestacional en e!;ta paciente es: a) Fecha de la ltima regla b) Ecografa obsttrica, con fetometra e) Fecha de inicio de la percepcin de los movimientos fetales. d) Altura utena e) Niveles plasmticos de gonadotrofina conica humana 9) Un nio de 6 aos consulta por malestar general de 3 semanas de evolucin, asociado a fiebre intermitente y cefalea, que en el ltimo tiempo se ha hecho diaria y que responde slo parcialmente al uso de paracetamol. Al examen fsico se aprecia plido y se palpa esplenomegalia moderada. No se encuentran otras alteraciones. El diagnstico ms probable es: a) Mononucleosis infecciosa b) Linfoma e) Leucemia aguda d) Meningitis viral e) Tumor del sistema nervioso central 1O) Una paciente de 33 aos, primigesta, cursando embarazo ele 35 semanas, sufre rotura de me~branas, la que se confirma con la posltivldad del test de cristalizacin. Al examen fsico se palpa feto en presentacin ceflica, no hay dinmica uterina ni modi"caciones cervicales y las pruebas de monitorizacin fetal son normales. La conducta ms adecuadct es: a) Administrar antibiticos y corticoides y mantener una conducta e~xpectante b) Administrar corticoides endovenosos y antibiticos e interrumpir el embarazo en 48 horas con misoprostol e) Realizar amniocentesis y decidir manejo segn hallazgos d) Inducir el parto con misoprostol e) Realizar cesrea 11) Una mujer de 20 aos consulta por leuc.orrea y prurito v~l-var. Al examen se observa leucorrea blanca grumosa, con signos inflamatorios en la pared vaginal. El.agente causal ms probable es: a) Gandida albicans b) Trichomona Vaginalis e) Gardnerella vaginalis d) Neisseria gonorreae e) Treponema pallidum 12) La causa ms frecuente de anemia en los nios es: a) Ferropnica b) Talasemias e) De enfermedades crnicas d) Autoinmune e) Por dficit de folatos 13) Cul de los siguientes hallazgos es considerado como n:>rmal en un recin nacido? a) Salida continua de liquido por el ombligo b) Bolsa serosangulnea e) Petequias d) Leucocoria e) Hipotonla 14) El feto grande para la edad gestacional se define como aquel que: a) Pesa ms de 4.000 g b) Pesa ms de 4.500 g e) Presenta un peso fetal mayor al percentil 90 para su talla d) Presenta un peso fetal mayor al percentil 90 para su edad gestacional e) Presenta una relacin permetro abdominal 1 permetro ceflico mayor a 1

~ ~
15) Un lactante menor presenta infeccicnes respiratorias recurrentes, que han respondido adecuadamente el tratamiento antibitico. Al examen fsico destaca auscultacin pulmonar con escasos crpitos bibasales y auscultacin cardiaca con un soplo continuo. El diagnstico ms probable es: a) Coartacin artica b) Trasposicin de grandes vasos e) Ductus arterioso persistente d) Comunicacin interauricular . e) Comunicacin intetventricular 16) Una mujer de 34 aos consulta por dismenorrea marcada, que ha aumentado en los ltimos meses. Se solicita una ecografa que demuestrct un tumor ovrico izquierdo de 4 cm de dimetro, quistico, con contenido en vidrio esmerilado. El diagnstico ms probable es: a} Quiste folicular b} Cncer de ovario e) Cuerpo lteo d} Endometrioma e} Teratoma 17) Un nio de 6 aos, presenta desde hace unos meses, diarrea y baja de peso, asociado a piel seca, glositis y cafda del cabello. Refiere que presenta diarrea acuosa, con gotas de grasa, sin presencia de sangre en las deposiciones. La etiologfa ms probable es: a} Enfermedad inflamatoria intestinal b} Giardasis e) Mala absorcin intestinal d) Invaginacin intestinal e) Estenosis pilrica hipertrfica 18) Un mujer de 35 aos, diagnosticada de hipertensin arterial crnica, en tratamiento con hidroclorotiazlda, con adecuados controles, presenta atraso menstrual, por lo que se realiza un test de embarazo que resulta positivo. La conducta ms adecuada para el tratamiento de su hipertensin arterial es: a) Suspender la hidroclorotiazida e iniciar alfametil-dopa b) Mantener la hidroclorotiazida, sin necesidad de otros medicamentos e) Suspender la hidrocloritiazida e iniciar enalapril d) Suspender la hidroclorotazida e iniciar sulfato de magnesio e) Solicitar proteinuria de 24 horas para decidir el hlpotensor ms adecuado 19) Cul de las siguientes coondiciones es ms frecuente en el recin nacido pretrmino que en el recin nacido de trmino? a) Sfndrome de aspiracin de meconio b) Cardiopatas congnitas e) Hipocalcemia d) Poliglobulia e) Cefalohematoma 20) Una paciente cursando un embarazo de 34 semanas, presenta un retraso del crecimiento intrauterino, con feto creciendo en percentil 5. Qu afirmacin es FALSA? a) En caso de no haber complicaciones debo interrumpirse el embarazo a las 37 semanas b) Debe solicitarse ecografa-Doppler de artmias uterinas cada 1 o 2 semanas e) La presencia de oligohidramnios es un criterio de interrupcin del embarazo d) Deben realizarse 3 veces a la semana un perfil biofisico o bien un registro basal no estresante e) Debe indicarse reposo en decbito lateral izquierdo 21) Una paciente presenta ciclos menstrUi3les de 24 dfas de duracin y en cada menstruacin presenta sangrado muy escaso, menor a 20 ce y de slo 2 das de duracin. Se puede afirmar que presenta: a) Oligomenorrea b) Polimenorrea e) Hipomenorrea d) Polimenorrea e hipomenorrea e) Oligomenorrea e hipermenorrea

~ (A ~

~
~

~
~

~
~
~

~
~

~ ~ ~
~

f'
~ ~
~

~
~

~
~

~
~

~
~

('A
~

~
~

22) Un paciente de 6 aos presenta fiebre, cefalea y signos menngeos, por lo que se solicita una puncin lumbar que da salida a lfquido purulento, con 100.000 leucocitos por mm3, de marcado predominio polimorfonuclear, glucosa baja y protenas elevadas. La tincin de Gram muestra cocceas gram negativas. El agente causal ms probable es: a) Meningococo b} Neumococo e) Haemophilus influenza d) Staphilococo e) Listeria 23) La causa ms frecuente de constipacin en la edad pedltrlca es: a) Enfermedad de Hirschsprung b) Polipos juveniles e) Enfermedades inflamatorias intestinales d) Malos hbitos e) Giardasis 24) Un recin nacido de t(mino, de 5 das de vida, presenta ictericia, desde hace 2 das~ que actualmente llega hasta los muslos. Al nacer pes 3.600 g y actualmente pesa 3.650 g, alimentndose exclusivamente con leche materna. No presenta otros sntomas y el examen fsico n aporta mayor informacin. El diagnstico ms probable es: a) Ictericia fisiolgica b) Ictericia hemoltica por incompatibilidad de grupo clsico e) Ictericia hemolftica por incompatibilidad de grupo Rh d) Ictericia por leche materna e) Hipotiroidismo congnito 25) Una paciente presenta amenorrea secundaria a una falla ovrica prematura de origen autoinmune. La prueba de progesterona y la prueba de estrgenos ms progesterona resultarn respectivamente: a) Positiva y positiva b) Positiva y negativa e) Negativa y positiva d) Negativa y negativa e) No podrn ser realizadas 26) Un recin nacido de 36 semanas de gestacin, presenta dificultad respiratoria luego de nacer. Se inicia oxigeno y se realiza radiografa de trax que muestra hilios pulmonares prominentes, aumento de la trama vascular pulmonar y leve derrame pleural bilateral. El diagnstico ms probable es: a) Enfermedad de membrana hialina b) Cardiopatia congnita e) Hipertensin pulmonar persistente d) Sindrome de aspiracin meconial e) Taquipnea transitoria 27) Una paciente de 34 aos, cursando embarazo de 11 semanas inicia metrorragia, asociada a dolor hipogstrico. Se realiza ecografa transvaginal que demuestra saco gestacional indemne, con visualizacin de latidos cardiofetales. El tacto vaginal muestra1 cuello completamente borrado, con dilatacin de 3 cm. La conducta ms adecuada es: a) Enviar a domicilio indicando reposo absoluto b) Realizar ce relaje de emergencia e) Administrar progestgenos y reevaluar en 48 horas d) Hospitalizar, administrar analgsicos y tocoliticos e) Realizar legrado 28) Qu examen NO pediria al inicio del estudio de una pareja que consulta por infertilidad de 18 meses de evolucin? a} Ecografa transvaginal b} Espermiograma e) Seguimiento foliular d) Test postcoital e} Histerosalpingografla

'

~
29) El punto de reparo de la presentacic)n de Bregma es: a) La fontanela posterior b) La fontanela anterior e) La frente d) La nariz e) El mentn

*'

30) Qu medicamento es de eleccin para la profilaxis de infecciones del tracto urinario, en un nio con reflujo vesicoureteralleve? a) Nitrofurantoina b} Eritromicina e) Ciprofloxacino d} Gentamicina e) Amoxicilina + cido clavulnico
31) Un paciente de 6 aos presenta edema facial y de extremidades Inferiores. Al examen fisico presenta signos vitales normales para su edad. Se solicita estudio de orina completa que demuestra cilindros creos, sin hemtauria ni leucot~ituria, con proteinuria de 24 horas de 2,5 g. La creatinina plasmtica y los electrolitos plasmtico~:; son normales. El diagnstico ms probable es: a) Sfndrome hemolitico urmico b) Enfermedad de Berger e) Glomerulonefritis aguda postestreptoccica d) Rin poliquistico e) Enfermedad por cambios minimos

~
~
~

~
~

~
~

(*"
~

32) Desde qu edad deben iniciar la in~esta de alimentos slidos los nios alimentados con lactancia materna exclusiva? a) 3 meses b) 6 meses e) 9 meses d) 10 meses e) 12 meses
33) En cul de las siguientes pacientes aconsejara el uso de anticonceptivos por vfa transdrmica en lugar de via oral? a) Mujer con cncer de endometrio b) Mujer con antecedente familiar de cncer de mama e) Mujer con lupus eritematosos sistmico d) Mujer con hipertrigliceridemia e) Mujer purpera, dando lactancia

e!"
(!"
~

~
~

34) La enfermedad de Kawasaki suele complicarse con:


a) Meningitis asptica b) Trombosis venosa e) Artritis deformante d) Uvetis e) Aneurismas coronarios
35) La endometriosis es una causa de dismenorrea a) La clnica b} Ecograffa transvaginal e) Biopsia de endometrio d} Laparoscopa e) Ca-125

(*'
~
~

y de infertilidad. Suele diagnosticarse mediante:

~
~

~
f"J!!

t'

36) Qu opcin caracteriza mejor a un niilo de 1O meses de edad? a) Fija la mirada y presenta sonrisa social b) Se sienta por si mismo, vocaliza (ag) y es. capaz de tomar objetos y pasarlos de una mano a la otra e) Se para por si mismo, dice cerca de 1O palabras y se muestra indiferente a la presencia de extraos d) Se para por s mismo, sin caminar, entiende el concepto de "no" y llora frente a los extraos e) Camina sin apoyo y dice cerca de 10 palabras

t-'
~

~ t""
(R\

37) Una mujer de 18 aos, primigesta, cursando 31 semanas d embarazo, consulta por malestar general y cefalea progresiva. Al examen se aprecia taquicrdica e hipertensa, con marcados edemas en ambas extremidades inferiores. El diagnstico ms probable es: a) Hipertensin transitoria del embarazo b) Sindrome de HELLP e) Preclamsia d) Embarazo molar e) lsoinmunizacin Rh 38) La conducta ms adecuad ante un lactante cursando una diarrea aguda por rotavirus, con deshidratacin severa (mayor al10% del peso corporal), es: a) Hidratar con suero fisiolgico por vla endovenosa b) Indicar sales de rehidratacin oral y reevaluar en 6 horas e) Indicar sales de rehidratacin oral luego de cada deposicin diarr;ica, segn Plan A d) Iniciar antibiticos y sales de rehidratacin oral e) Iniciar sales de rehidratacin oral e indicar vacunacin contra el r:>tavirus 39) Los corticoides prenatales han aumentado considerablemente la sobrevida de los recin nacidos prematuros. Reducen la incidencia de todo lo citado a continuacin, EXCEPTO: a) Sepsis neonatal . b) Enferm.edad de membrana hialina e) Hemorragia periventricular d) Muerte neonatal precoz e) Enterocolitis necrotizante 40) La primera causa de insuficiencia renal aguda, en la edad peditrica, es: a) Malformaciones nefrourolgicas b) Glomrulonefritis e) Sindrome hemolitico urmico d) Prerrenal e) Necrosis tubular aguda 41) La etiologa de las desaceleraciones tard[as, en el registro estresante, es: a) Reaccin vagal, producto de compresin de la cabeza fetal b) Compresin del cordn umbilical e) Hipoxia fetal d) Flujo reverso en distole de las arterias umbilicales e) Aumento del retorno venoso fetal, producto de la compresin de la placenta 42) Segn el programa nacional de vacunaciones, al cumplir un ao, los nios deben recibir: a) Las vacunas DPT y polio oral b) La vacuna trivirica e) Las vacunas trivirica, DPT y polio oral d) Las vacunas DPT, polio oral, Hib y VHB e) La vacuna DT 43) Una mujer de 34 aos, cursando embarazo de 35 semanas presenta poliaquiuria, sin disuria, que la hace ir al bao cerca de 15 veces al dfa. Se solicita sedimento de orina que resulta normal y urocultivo que resulta negativo. La conducta ms adecuada es: a) Iniciar cefadroxilo en una dosis diaria b) Solicitar serologa contra mycoplasma e) Solicitar urodinamia d) Disminuir el consumo de agua e) Tranquilizar a la paciente, envindola explicando la normalidad de su condicin 44) Cul de las siguientes mujeres tiene mayor riesgo de presentar cncer de endometrio? a) Mujer con promiscuidad sexual y antecedente de procesos inflamatorios plvicos tratados b) Mujer obesa, con diabetes mellitus 2 e hipertensin arterial e) Mujer postmenopusica, con uso de terapia de reemplazo hormonal por 1O aos d) Mujer con endometriosis e) Mujer con miomatosis uterina sintomtica

'

~
45) Durante la primera semana de vida los recin nacidos pueden presentar una baja de peso de mximo: a) 500 gramos b) 100 gramos e) 5% del peso de nacimiento d) 1O% del peso de nacimiento e} 20% del peso de nacimiento 46) Los nio con enuresis primaria, habitualmente presentan: a) Constipacin b) Alteracin en la continencia diurna e) Antecedentes familiares de enuresis d) Disuria e) Osmolaridad urinaria disminuida 47) Son causas de oligohidramnios, EXCEPTO: a) Insuficiencia placentaria b} Rotura prematura de membranas e) Diabetes mellitus gestacional d) Preclamsia e) Malformaciones renales 48) El primer signo de desarrollo puberaU en las nias es: a) Vello pbico b) Menarquia e) Botn mamario d) Adrenarquia ": e) Aceleracin del crecimiento 49) Una mujer de 20 aos, cursando embarazo de 29 semanas, presenta fiebre persistente, hasta 38,2C, sin presentar otros sintomas ni otras alteraciones en el examen fisico. Se realiza estudio rutinario sin poder identificar un foco infoccioso, por lo que se realiza amniocentesls que demuestra 31 leucocitos por mm3, glucosa de 25 mtJ/dl. A la tincin de Gram no se visualizan bacterias y el cultivo de lfquido amnitico resulta negativo. La conducta ms adecuada es: a) Descartar el foco uterino y continuar el estudio de la fiebre b) Iniciar antibiticos de amplio espectro y mantener una conducta expectante e) Administrar antibiticos y corticoides e interrumpir el embarazo en 48 horas d) Interrumpir el embarazo bajo cobertura antibitica e) Realizar cesrea de urgencia 50) Un nio llora y hace berrinches 'cuando es llevado al jardrn infantil, quedando muy ansioso y llorando por largo rato luego que su madre se va, siendo dificil de consolar. La madre suele irse muy ansiosa y preocupada del estado de su hiJo. El diagnstico ms probable es: a) Trastorno oposicionista desafiante b) Trastorno de conducta e) Depresin infantil d) Trastorno por ansiedad de separacin e) Trastorno por dficit atencional 51) Los anticonceptivos orales con microdosis de estrgenos se caracterizan por: a) Presentar mayor incidencia de goteo {spoUing) que los de macrodosis b) Presentar mayor riesgo de cncer de mama que los de macrodosis e} Presentar mayor riesgo de trombosis veno!;a profunda que los de macrodosis d) Tener una mayor efectividad en la prevencin de embarazo que los de macrodosis e) Ser inyectables 52) El tratamiento de eleccin de una larinnitis obstructiva aguda moderada es: a) Salbutamol inhalado b) Salbutamol y corticoides inhalados e) Adrenalina racmica inhalada y corticoides inhalados d) Adrenalina racmica inhalada y corticoides orales e) Corticoides inhalados y corticoides orales

~ ~

~ ~

f!'A
~
~

~ ~

f!l'o
~

e
~
~

~
~

~
~
~

~
~

~ (iiA
~

~ ~
~
~
~

'"

~
~

~
~
53) Un recin nacido de 4 das de vida, nacido prematuro alas 33 semanas, presenta desde algunas hora dificultad para regular la temperatura corporal y debilidad para alimentarse. Al examen presenta tono muscular disminuido y debilidad del refljo de succin. El diagnstico ms probable es: a) Parlisis cerebral b) Hipoglicemia e) Hipocalcemia d) Sepsis e) Sndrome de Guillian Barr 54) Una paciente en edad frtil, sexualmente activa, presenta desde hace una semana dolor hipogstrico, asociado a dispareunia profunda. Al tacto vaginal se constata dolor a la movilizacin cervical y a la palpacin anexial. El diagnstico ms probable e:s: a) Cncer de cuello uterino b) Neoplasia intraepltelial e) Cncer de endometrio d) Procesos inflamatorio plvico e) Endometriosis 55) La cardiopat[a congnita ms frecuente es: a) Comunicacin interauricular b) Foramen oval e) Comunicacin interventricutar d) Dueto arterioso persistente e) Coartacin artica 56) Mujer de 35 aos, cesarizada anterior, cursando embarazo de 30 semanas, con polihidramnios, de causa no precisada, presenta salida de abundante lquido ciare por los genitales. No presenta dinmica uterina y la especuloscopfa demuestra salida de lfquido transparente por el orificio cervical externo. Se palpa el feto en podlica y no hay modificaciones c.ervicales. El registro basal no estresante es reactivo y el perfil biofsico es 10/10. La conduct'l ms adecuada es: a) Administrar antibiticos y corticoides y mantener una conducta expectante b) Administrar corticoides endovenosos y antibiticos e interrumpir el embarazo en 48 horas e) Administrar corticoides, antibiticos y tocollticos por via endovenosa d) Inducir el parto con misoprostol e) Realizar cesrea 57) Respecto a la hepatitis A, es verdadero que: a) No existe vacuna para su prevencin b) Se transmite por via vertical e) El diagnstico de certeza se realiza con la deteccin del antlgeno de superficie d) El tiempo de protrombina es un muy buen indicador de la severidad de la enfermedad e) Suele presentarse con ictericia y elevacin de las transaminasan cercana al doble de los valores normales 58) Una mujer de 35 aos se realiza una mamografa, que es informada como Birrads O. La conducta ms adecuada es: a} Repetir la mamografia en un ao b} Repetir la mamografla en 6 meses e} Realizar ecografla mamaria d) Solicitar T AC mamario e) Derivar para biopsia por puncin 59) Un nio de 6 aos presenta fiebre hasta 39 y decaimiento, asociado a la aparicin de mltiples ulceraciones en la cavidad oral, comprometiendo el paladar blando, paladar duro, lengua, labios y encas. Adems se palpan varias adenopatias cervicales sensibles. El diagnstico ms probable es: a) Herpangina b) Mononuecleosis infecciosa e) Primoinfeccin herptica d) Escarlatina e} Faringoamlgdalitis por anaerobios

A'\
~

""'
~

~
~

~
~
~

~
41)

~
~

~
~
~

~
60) Una mujer se realiza un Papanicolaou, que es informado como muestra NO satisfactoria. La conducta ms adecuada es: a) Controlar PAP anualmente b) Solicitar estudio de virus papiloma humano e) Repetir PAP de inmediato d} Derivar a colposcopia e} Derivar a conizacin

"

"

~ ~ ~

61) En promedio, Cunto mide un nio de 12 meses de edad?


a) 37 cm b) 50 cm e) 62 cm d) 75 cm e) 90 cm

(A

~ ~
~

62) Una paciente cursando embarazo de 36 semanas de gestacin, consulta por disminucin de la percepcin de los movimientos fetales.!Se solicita registro basal no estresante que es informado como no reactivo, luego de 40 minutos de monitorizacin. La conducta ms adecuada es: a) Enviar a domicilio b) Extender el registro por 60 minutos ms e) Solicitar ecografa-Doppler de arterias umbilicales d) Realizar test de tolerancia a las contracciones e) Interrumpir el embarazo por la vla ms expedita 63) Qu alternativa es adecuada para alimentar a un lactante de 3 meses de edad, cuya madre ya no puede continuar amamantndolo? a) Leche purita al 7 ,5%, ms azcar al 2,5%, ms aceite al 1,5% b) Leche purita al7,5%, ms azcar al2,5%, ms cereal al3% e) Leche purita al 7 ,5%, ms azcar al 5%, ms cereal al1,5% d) Leche purita al1 0%, ms azcar al 2,5%, ms aceite al1,5% e) Leche purita al10%, ms azcar al5%, rns cereal al3% 64) Un nio de 7 aos presenta fiebre, decaimiento y un exantema polimorfo, generalizado, con ppulas, vesculas, pstulas y costras en1 la cabeza, tronco y extremidades. El agente causal ms probable es: a) Streptococcus b) Enterovirus e) Virus herpes 6 d) Parvovirus 819 e) Virus varicela zoster 65) Una paciente de 34 aos, cursando su tercer embarazo, de 30 semanas, inicia metrorragia, asociado a dolor abdominal intenso. Al eJ<amen fsico se palpa el tero doloroso e hipertnico. El monitoreo fetal demuestra variabilidad di:;minuida, sin otras alteraciones. La conducta ms ade-cuada es: a) Administrar corticoides y mantener una ccnducta expectante b) Administrar corticoides e inducir el parto en 48 horas, con misoprostol e) Realizar prueba de trabajo de parto d) Solicitar un perfil biofisico y decidir conduc:ta segn hallazgos e) Interrumpir el embarazo por la vla ms expedita
66) Cul de las siguientes es una Indicacin de hospitalizacin en un nio que sufre un golpe en la cabeza? a) Hematoma del cuero cabelludo de 4 cm b) Amnesia del momento del golpe e) Prdida de consciencia de segundos de duracin, inmediatamente despus del golpe d) Nuseas e) Sospecha de maltrato infantil

~ ~
~

~ ~
~

r'
~

,..
~

~
~
~
~

~ ~

~ ~
~

~
~

~
~

~
~

67) Los miomas que con mayor frequencla presentan sintomas !;on: a) lntramurales b) Subserosos e) Pediculados d) Submucosos e) Capsulados 68) Una mujer de.24 aos, con antecedentes de promiscuidad sexual, inicial dolor abdominal, asociado a metrorragia. Su ltima menstruacin fue hace 2 meses. Al exa.men ffsico se constata FC: 11 ox, PA: 90/50 mmHg, FR: 16x', con dolor a la palpacin hipogstrica, sin signos de irritacin peritoneal. Se realiza test de embarazo que resulta positivo y se realiza una ecograffa transvaginal que no visualiza saco gestacional. Se administran analgsicos y cristaloides. La conducta ms adecuada es: a) Solicitar beta HCG plasmtica y administrar cristaloides endovenosos b) Controlar en 48.horas con nueva ecografa transvaginal e) 1 niciar metotrexato d) Realizar legrado uterino e) Resolver quirrgicamente de Inmediato
.

69) Un recin nacido de 5 dfas de vida presenta varias pstulas pequeas, con base eritematosa en la cara, el tronco.y las extremidades. Se aprecia de buen aspecto y el resto del examen fsico es normal. El diagnstico ms probable es: a) Eritema txico b) Varicela neonatal e) Herpes neonatal d) Slfilis congl'lita e) Exantema sbito 70) Los principales beneficios de la terapia de reemplazo hormonal son: a) Prevencin del envejecimiento prematuro y control de Jos sintomas vasomotores climatricos b) Control de Jos sintmas vasomotores climatricos y prevencin de la osteoporosis e) Prevencin del envejecimiento prematuro y de la osteoporosls d) Control de los slntomas vasomotores, prevencin de eventos cardiovasculares y prevencin de la osteoporosis e) Control de los sntomas vasomotores, prevencin del envejecimiento prematuro y prevencin de eventos cardiovasculares

e!'
~
~
~
NOMBRE:

o
1

00 000 000 00

2
3

00 000 000

~
~
RUT:

~ ~
~
FECHA:

00 00 5 00 6 00 7 00 8 00 9 00
4
K

o o 000 000 o 000 000 o 000 000 o


000 000 000 000 000 000 000 000 000 000

o o o

o o
o

~
~

CURSO: RESPUESTAS A B C D E A B C
31 32 33 34 35 36 7 !3 38 39 40

~
~

D E
61
62 63

A 8

C D E

~
~

(fA
~

~
~
~

00000 00000 3 00000 4 00000 5 00000 6 00000 7 00000 8 00000 9 00000 10 00000
1

00000 00000 00000 00000 00000 00000 00000 00000 00000 00000 00000 00000 00000 00000 00000 00000 00000 00000 00000 00000

64
65 66 67 68 69 70

00000 00000 00000 00000 00000 00000 00000 00000 00000 00000 00000 00000 00000 00000 00000 00000 00000 00000 00000

~
(11!1

~ ~ ~
~

~
~1'

00000 00000 13 00000 14 00000 15 00000 16 00000 17 00000 18 00000 19 00000 20 00000
11
12

41

71 72

42 43

73
74 75 76

44
45 46 47

48 49

78 79

nooooo
80

50

~
~
~

(S'

tfA
($'

(fA
(lA
(fA
('$'

00000 00000 23 00000 24 00000 25 00000 26 00000 27 00000 28 00000 29 00000 30 00000
21
22

00000 00000 53 00000 54 00000 55 00000 56 00000 57 00000


51

81 82 83 84

52

85
8

87 88 89 90

58 59 60

00000

00000 00000 00000 00000 00000 00000 00000 00000 00000


00.000

00000 00000

~
~

~
1. 2. 3. 4. S. 6. 7. 8.

LA CAUSA MS FRECUENTE
REUMATOLOGIA~
Monoartritis aguda en nios Monoartritis aguda en adultos mayores Patologa articular Panarteritis nodosa Vasculitis sistmica primaria Dolor lumbar Dolor en hombro Dolor articular de manos

~
Dr. Guillermo Guevara Aliaga

~ ('!'

~
~ ~
~

LA CAUSA MS FRECUENTE
9. Artritis sptica 10. Artritis sptica en adolesce'ntes y adultos j,enes 11. Osteomielitis 12. Lumbago inflamatorio 13. Prpura en pediatrfa 14. Prpura no trombocltopnico 15. Muerte en AR 16. Muerte en LES

LA CAUSA .MS FRECUENTE


17. 18. 19. 20. 21. 22. 23. 24. Muerte en Esclerodermia limitada Muerte en esclerodermia difusa Manifestacin extrarticular en AR Artritis por cristales Raynaud intenso y recurrente Poliartritis crnica de manos Osteoporosis Osteoporosis en hombres (viejos)

~
~

~
~
~
~

~
~
~

(!'A

~
~
~

~
~ (!'A

1. Lumbago inflamatorio 2. Monoartritis ag~.;~da 3. Seguimiento de P~rpura de Sch-Hen 4. Sospecha de tendinitis o entesitis S. Sospecha de Wegener 6. Sospecha de panarteritis nodosa 7. Dg Arteritis de la temporal 8. Etapificar compromiso renal en lupus

EXAMEN(es) MS IMPORTANTE

EXAMEN(es) MS
IMPORTANTE
9. 10. 11; 12. 13. 14. 15. 16.
Marcador sensible LES Marcador espedfico LES (2) Marcador lupus por drogas Marcador enfermedad mixta del TC Marcador sensible Sjorgren Marcador espedfico de Sjrgren (2) Marcador CREST Marcador esclerodermia difusa

_r _____

"'
EXAMEN(es) MS IMPORTANTE
17. Marcador psicosis lpica 18. Marcador actividad lpica (2) 19. Marcador AR

~)

~ ~

: EXAMEN(es) MS IMPORTANTE
23. 24. 25. 26. 27. 28. 29. 30. Sd. Tnel carpiaa:-ao Sospecha de ostepporosis Sospecha de Takayasu Nii1o que claudica (primer elcamen) F'rimer ataque de gota [lg fibromialgia [lar pronstico de Kawasaki [lg Poliange(tis microscpica

"'
~

20. 21. 22. 23. 24.

Marcador arteritis de la temporal Marcador de Polimiositis/dermatomiositis Sospecha clnica de artrosis Marcador de Artritis reumatoide juvenil Evaluar pronstico de E. Kawasaki

,
~
~

~
~ ~

~
~

TRATAMIENTO
1. 2. 3. 4. S.

TRATAMIENTO
. 9. 10. 11. . 12. 13. 14. 15. 16. l:spondilitis anquil~sante liipertensln pulmonar l:nfermedad re~~tica aguda Pr~venlr nueva E. reumtica 1\rteritis de la temporal Polimialgia reumtica Epicondilitis Sd. Manguito Retador

~
~

Artrosis Artritis Reumatoide Lupus sistmico Sd de Sjorgren Artritis sptica 6. Prevenir ataque gotoso 7. Prevenir ataque de condrocalcinosis 8. Artritis aguda por cristal~s

~
~

~ ~
~

~
~

~ ~
~

~
~ ~
~

TRATAMIENTO
17. Capsulitis adhesia de hombro

TRATAMIENTO
25. 26. 27. 28. 29. 30. 31. 32. IOsteoporosis :Sd. Antifosfolfpidos :Sd. Antifosfolpidos en el embarazo .#\rtritis viral :Sd. Tnel carpiano rakayasu AR que no responde a DMARs LES sev~ro (Ej anemia hemoltica o . Glomerulonefritis lpica severa)

18. Wegener con compromiso nasosinusal 19. Wegener con otro compromiso

~
~

20. Poliangeitis microscpica 21. Prpura de Sch-Hen 22. Prpura trombocitopnico idioptico en nios 23. Prpura trobocitopnico en adultos .. 24. Fibromialgia

TRATAMIENTO
33. 34. 35. 36. 37. 38. 39. 40. 41. 42. Gota ms litiasis por c. rico PAM Polimiositis Fibromialgia E. Still Paget sintomtica Paget asintomtica Sd. Kawasaki Hombro congelado P1T

CASOS CLNICOS
1. Paciente de 70 aos, con dolor lumbar progresivo

de 4 meses de evolucin, mayor con los movimientos. En las ltimas semanas se agrega paresia a la flexin plantar del pie derecho. 2. Un paciente de 34 aos, consulta por dolor en el cuello, dorso y extremidad superior derecha, que apareci luego de levantar y mover algunos muebles. Al examen presenta disminucin del reflejo blclpital e hipoestesia en el pulgar derecho. Qu rafz nerviosa es la afectada? 3. Mujer, 30 aos, con 4 abortos espontneos. Actualmente con cuadro sugerente de TVP.

~ ~
~
~

CASOS CLNICOS
4. Mujer 30 aos, con utralgias, adenopatfas cervicales, fiebre ocasional y ndulos eritematosos y dolorosos en la subdermis de las EEII. RxTx patrn inflltrativo retlculonodular. S. Sensacin de boca seca y arenilla ocular. Tes1c de Schirmer: hlpolacrtmia. Se agrega artritis no erosiva de manos 6. Mujer de 17 aos con eritema violeta en pr>ados superiores y eritema sobre articulaciones IFP y
MCF

CASOS CLNICOS
7. Mujer de 23 aos, con debilidad, especialmente de las masas musculares proximales, asociados a fiebre ocasional y astenia. Presenta adems eritema de la piel de los hombros y elevacin de lasCK. S. Paciente con fenmeno de Raynaud intenso de algunos aos de evolucin, asociado a sensacin de tirantez en cara en los ltimos meses. Al examen: esclerosis de piel de dedos. 9. Paciente con antecedente de esclerodermia sistmica difusa inicia dificultad para tragar y disnea. Al examen se auscultan crpitos gruesos y se observan uas en vidrio de reloj.

~
~

~
~

~
~ ~
~.

~
~

~
~
~

CASOS CLNICOS
10. Mujer con artralgias y oligoartritis no erosiva ele
1~.

CASOS CLNICOS
Duea de casa de 30 aos, consulta por dolor en el codo de 10 dfas de evolucin, que en los ltimos dfas se ha hecho muy intenso al limpiar los muebles y vidrios. Al examen dolor a la palpacin de la Insercin de los extensores del antebrazo. 14. Hombre de 42 aos con dolor de larga data hombro derecho, que se ha intensificado lo que le impide la movilizacin. Al examen fisico sin signos inflamatorios y dolor a la movilizacin activa y pasiva del hombro.

~
~
~

~
~

manos y rodillas, asociado a astenia. No presenta alteraciones renales ni de otro rgano. Se solicita perfil ENA con positivldad de anticuerpos antirrtbonucleoprotefna. 11. Paciente hipotirordea, consulta por parestesias en pulgar, lndice y dedo medio derecho, mayor en las noches, por lo que sacude la mano, para "despertarla". 12. Paciente con parestesias y debilidad a la flexin del4to y Sto dedos derechos

~
~

~
~

CASOS CLNICOS
15. Mujer de 32 aos, con antecedente de colon irritable, consulta por astenia y dolores generalizados. Al examen sin artritis. Dolor a la palpacin de msculos cervicales, trapecios, supraespinosos, cara Interna de rodillas y extensores de la mueca. 16. Nio de 3 aos que empieza a cojear. Al examen, dolor a la manipulacin de la cadera izquierda. Se ve de buen aspecto, sin fiebre ni signos inflamatorios evide~tes. 17. Nio de 8 aos que cojea desde hace S dfas, no refiere dolor. Al examen dolor en los movimientos de la cadera Izquierda.

CASOS CLNICOS
18. Ni ro de S aos, con fiebre de 6 d(as de evolucin, ascciada a edema de manos, artritis de mueca, adt!nopat(a cervical, labios rojos y partidos y exuntema maculopapular generalizado, mayor en tronco. 19. Niflo de 8 aos, consulta por fiebre diaria, asociado a compromiso del estado general y emmtema maculopapular que aparece en tronco y zonas proximales de las extremidades mientras dura la fiebre y luego desaparece. Al examen flslco resulta normal, excepto por artritis del tobillo derecho y de 2 articulaciones MTF.

CASOS CLNICOS
20. Adolescente obeso de 14 aos, consula por dolor en la Ingle Izquierda y claudicacin al caminar. 21. Nio de 3 aos presenta claudicacin de la marcha y dect;~lmiento. Al examen se ~precia febril hasta 39,5C, con llanto a la movilizacin de la cadera izquierda 22. Hombre de 24 aos, con dolor lumbar, mayor de reposo, con rigidez matinal de 30 minutos. 23. Paciente que comienza con artritis de rodilla derecha y 2 MCF izquierda, asociada cojuntivitis y ardor al orinar.

CASOS CLNICOS
24. Mujer con artritis de tobl.llo derecho y algunas MTF. Presenta adems dedos en salchicha y se observan l~siones ungeales tipo piquetes de
p~~Jaro

25. Hombre de 27 aos con lumbago de caracter(sticas inflamatorias, asociado a uveftis, eritema nodoso y marcada dificultad para eJI.tender el cuello y flectar la zona lumbar, de modo que apenas alcanza a tocarse las rodillas eJI:tendidas

CASOS CLNICOS
26. Paciente de 30 aos, con rinosinusitis crnica de difcil tratamiento, con epistaxis frecuente. Evoluciona con HTAy oliguria. En los exmenes destaca hematuria dismrfica y ANCA(+), en patrn "e". 27. Hombre de 60 aos, que en relacin a cuadro respiratorio alto de 1 d(a de evolucin, Inicia HTA. y hematuria dlsmrfica, con elevacin de la creatinina.

28. Hombre de 55 aos consulta por astenia. Refiere que se "le cansan los brazos" al realizar actividades como peinarse o poner una ampolleta. Tambin refiere mialgias difusas ocasionales. En la anaUtica slo destaca VHS:86 29. Hombre de 60 aos, con mialgias generalizadas, astenia y baja de peso de 1 mes de evolucin, asociado a fiebre ocasional, cefalea y claudicacin mandibular. En una oportunidad perdi visin de ojo derecho por 10 minutos. 30. Mujer de 19 aos, consulta por astenia y ad.inamia, con febrfcula ocasional. Al examen destaca marcada disminucin del pulso radial dlerecho.

CASOS CLNICOS

CASOS CLNICOS.
31. Paciente VHC (+}, refiere hematuria, artralgias y lesiones cutneas en los dedos y en las on!jas. 32. Adolescente de 15 aos, consulta por cuadro de 5 das de artralgias de manos y pies, asociados a diarrea disentrica y exantema eritematoso, palpable en glteos y muslos. 33. Hombre de 28 aos, consulta por cuadro de S ellas de evolucin de malestar general, fiebre, oligurla y parestesias migratorias de extremidades. En el examen fisico se aprecian lesiones purprit:as en EEII. En la analitica destaca crea:3,4 Hcto:27%, plaq:25.000, ANA(-) y ANCA(-)

CASOS CLNICOS
34. Mujer 42 aos, consulta por artralgias de 1 mes de evolucin, asociado a rigidez matinal de 1 hora de duracin y edema de manos. 35. Mujer de SO aos consulta por artralgias de manos de 3 meses de evolucin. Al examen: artritis de muecas, varias MCF y IFP de ambas manos. 36. Mujer de 17 aos presenta artritis de articulaciones IFD,IFP, MCF y MTF de todas las extremidades, desde hace 5 das, asociado a fiebre, malest~r general y rash macular eritematoso.

~ ~

"

"
~

~ ('A

~
~

~
~

~
~ ~ ~

~
~ ~
~
~

37. Mujer de 61 aos, campesina, sin controles mdicos, con.historia de artralgias de larga data y dificultad para usar las manos. Al examen presenta dedos en rfaga, atrofia de los m!;culos lnterseos, dedos en cuello de cisne y en dedo de sastre. 38. Una paciente de 67 aos consulta por dolor de la rodilla izquierda de 3 semanas de evolucin, que aparece especialmente durante la m~rcha. Al examen presenta signos de derrame articular moderado, sin eritema ni aumento de la temperatura local. La movilida~ es completa.. pero dolorosa, especialmente en la flexin extrema y se palpa crepitacin articular.

CASOS CLNICOS

CASOS CLNICOS
39. Mujer con historia de artritis de muecas, recidivante. La Rx muestra lesiones en sacabocado 40. Paciente de 22 ailos, con artritis simtrica de manos. En sus exmenes destaca trobocltopenia y llnfopenla. 41. Mujer con artralgias, historia de abortos recurrentes e insuficiencia renal de reciente diagnstico. 42. Mujer de 18 aos, consulta por fiebre ocasional, artralgtas, CEG y baja de peso. Refiere adems cafda de cabello. El examen fsico no aporta mayor Informacin. La anaUtica destaca anemia

~ ~ ~

CASOS CLNICOS
43. Mujer de 28 aos, consulta por astenia y mial~las. Al examen: eritema de mejillas y dorso de la nariz, que respeta el labio superior, 3 lceras orales Indoloras. 44. Mujer de 18 aos co~ lesiones recidivantes en los hombros, eritematosas, indoloras, consistentes en placas de 1 a 2 cm, cofluentes y de curso sinuoso, que aparecen especialmente luego de exposicin al sol y que al desaparecer dejan pequeas cicatrices hipopigmentadas

CASOS CLNICOS
45. Mujer de 57 aos, consulta por artralgias de manos de 1 aiio de evolucin, que se ha hecho ms Intensa. Al examen aumento de volumen, doloroso, de consistencia dura en IFD y en IFP de ambas manos. 46. Una paciente con antecedentes de AR en tratamiento con metotrexato y corticoides presenta de manera brusca dolor para la movilizacin de la cadera izquierda, que le impide la marcha. Se solicita radiografa que muestra deformacin y descalcificacin de la cabeza femoral.

~ ~
~

(!!"
~

~
~
~ ~

~.

~
~
~

CASOS CLNICOS
47. Hombre obeso, HTA de 40 aos, que luego de fiesta, despirta con intenso dolor en relacin a 1ra articulacin MTF derecha. Al examen: artritis muy dolorosa y eritematosa. 48. Hombre de 65 aos presenta intenso dolor en la rodilla derecha, asociado a aumento de volumen y eritema. Al examen derrame articular y eritema. Se realiza artrocentesis con liquido con 30.000 cel, cristales abundantes con birrefringef!cia de elongacin positiva 49. Hombre de 54 aos, asintomtico, presenta cido rico de 9,2 mgfdl.

CASOS CLNICOS
SO. Hombre de 23 aos presenta artritis de rodilla . derecha. Se realiza puncin articular con . visualizacin de cocceas gram negativas

~ ~

~
~

~
~

,.,
~
~

~
~
11

1. 2. 3. 4. S. 6.

Virus artritognicos (3): Factores pronsticos en AR: Factores pronsticos en LES: Manifestaciones Extraarticulares AR: Manifestaciones Extraartic en EAA AR y corticoides: 1. Articulacin ms afectada,en Artrosis 8. En gota 9. En condrocalcinosls 10. En Sinovitis transitoria 11. En Artritis s pica 12. En psoriasis

TIPS~'

~
~

~
~

~
~ ~

~
~

~
~

~
~
~
~

~
~

~
~

~ ~
~
~

;
.
//

~\
Pregunjs Salud Pblica

:'

!' !'

1) A/qu tipo de estudio corresponde el siguiente enunciado?: "Se selecciona un grupo de aclentes, afectos de hepatocarc:inoma y otro grupo de 100 pacientes, de edad similar, se comparan la tasas de e):posicin previa a alcohol"

sy

aso control } studio de prevalencia e} Cohortes d) Ensayo clfnico e) Ecolgica

.;:. ~ O lt-\ ' r\ e..

~-

~
~
~
~:
~--

2)

a}ieatorizar ~Jt~:ealizar anlisis estratificado Aumentar el tamaFio de la muestra d) Enmascarar {doble ciego) e) Aparear (matching)

La~~r estrategia para dismlnu Ir el error aleatorio es:


-:::--.

1' ,..._,-....4--rc.... ...

,.

3) Calcule ~alor r.ict~


a) O133 50
1

Sensibilida~_:_ ~,8;_ sp~ciflcid~JftAf~;J'r~l~nc~de enfermeda


.

posi~e una prueba diagnstica e


'~~ -6~--:e;. . . 1
:

1..9-~.~~
...

f!'
~

-~ d) o 80
e) 0190

lOEt')

1),~

~ \~ ~V!
/V

i""; .
-T -

'

P.:l}t". ~

'

'o .;

r-5-L._.'B-.-'t <80 1 ' ( -- <--~-.--~.~:r.;.-:. ~./ 7 . -:- =t--p. - .


\-~ ~:fj_
y

los siguientes dato.s.:.. ~.


~
~

i -..

.( .

... .

.--,.

.. \

. { o.o

o,__

f ~_:~:~l1::...
~ _7-

Re-. ;( ..

. .

::t.

'f '

~
~
~

~
~
~
~

4) A qu tipo de estudio corresponde el siguiente enficiado?: "Se selecciona una muestra"-- de 5000 pacientes y se les miden los niveles de colesterol y triglicridos, determinando el ..../ nmero de pacientes con hipercolesterolemia e hipertrigliceridemia" ~ ~ransversal "'--~ . ~ Ensayo de campo \>~.sc.0L e) Ecolgico d) Cohortes e) lnt~riencin comunitaria 5) ~~seleccionan 300 pacientes con hipercolesterolemia y se aleatorizan a 3 grupos de 100 tientes cada uno. El primer grupo recibe atorvastatlna, el segundo grupo recibe emfibrozilo y el tercero recibe una a1sociacin entre gemfibrozilo y atorvastatina. Se amparan los niveles de colesterol luego de 3 meses de tratamiento. La mejor prueba para determinar la significancia estadistica es: a) La "t" de Student b) Chi cuaqrado e} La nr" }:le Pearson d) Pru 6a exacta de Fisher A lisis de varianza

\Oc
---

~
~
~

"' "'
f1"
~

Ji>

6 El exceso de riesgo de enfermar, en los pacientes sometidos a un factor de riesgo, se enomina:


a) Odds ratio b) Likelihood ratio e) !)iesgo relativo J'i)Riesgo atribuible e) Riesgo atribuible poblacional

~
~
~

(/!'
~

fA
~

~ ~

~ ~6itterctana ~.;-rcL~r-~
d) Proteccin de salud e) Promoci~p de salud

.,

/~revenc!n secundaria _ ~..........\ _.J

7} Un paciente sufre de claudicacin Intermitente. Se le indican ejercicios diarios. Esta indicacin corresponde a una medida de: a) Prevencin primaria ,~~ ~--<o ~ O~r~.
-=>

"'
~
~

'f.

~
~

~ ~ ~

8) La de insumos y la implementacin de equipos, por parte de los distintos se91:icios de salud, est determinada de la siguiente manera:
JYComprar el equipamiento ms moderno disponible J.....afompr.arlos de acuerdo a los precios y garantlas que ofrecen los proveedores e) Renovarlos anualmente para evitar prdida de garantfas y obsolescencia d) E ~rnalizar todos los servicios que requieran de equipamiento e) omprar al mismo proveedor. garantizando asf compatibilidad tcnica

Y.~~ra

~
~

~
~

~a central nacional de abastecimiento


hYI instituto de salud pblica,
t)Ta secretaria regtonal ministerial d) La superintendencia de isapres e) El instituto mdico legal

) La institucin encargada de fiscalizar el actuar de las fatrmacias es:


__.

~ ~

e_ t. t-S..A ~..M T

~
~

~ ~
~

1O) El indtce de ocupacin de un hospital est dado por la relacin entre:


a) La duracin promedio de las hospitalizaciones y el total de dlas-cama disponibles (pacientes ingresados y el total de camas . s pacientes dados de alta y el total de camas (el os dlas-cama ocupados y el total de dlas-cama disponibles e) La duracin promedio de las hospitalizaciones y el total de dlas-carr:':a disponibles Reumatologla

~
~
~
~

11) Una mujer de 57 aos, consulta por artralgias de man(os, de larga evolucin. Al examen fsico se ~precia aumento de volumen de consistencia dura en relacin a las articulaciones inteatngicas proximales y distales. El diagnstico ms probable es:
a) Apitis.reumatoide ~_:rtritis gotosa ~Artrosis . d) Esponsmartropatla seronegativa e) Es_~.~erodermia sistmica limitada

~
~

~
~

~(La manifestacin exJraartLcular ms frecuente de la artritis re~matodea es:


( /a) Escleritis-epiescleritis \..._/ b) Mononeuritis mltiple A1\ldulos reumatoldeos d) Afectacin pleura-pulmonar e) Afectacin cardlaco-pericrdica

~
~
~
~

1~Cul de las siguientes exmenes permite predecir e!l grado de actividad lpica?

) Anticuerpos antinucleares
b) Anticuerpos anti-Ro y anti-La e) Anticuerpos anti DNA de una hebra .-d) Facto'f'rematoldea- ------.... ~~ComplementQ. Q_3__ y_C4_.

~
~

~
~
~

~ fl!' f'!;
~
~

14) La.,ptesentacin clnica ms habitual de la .sranulomatosls de Wegeoer es:


a) Astenia fiebre, compromiso de coronarias y arterias mesentricas ~..)"Giomuronefritis ~pidamente progre!3iva y ANCA positivo en patrn perinuclear ~e) Asma refractario a tratamiento y con posterioridad afectacin renal ~inosinusitis de dificil manejo, asociada a glomerulonefritis rpidamente progresiva y afectacin pulmonar e) Insuficiencia renal, hematuria, accidentes vasculares enceflicos a repeticin y prpura palpable de extremidades inferiores

".

~ ~

e
~
~

15) La ca_9s ms frecuente de codo doloroso es:


a) Artr~sls b) T tdinitis bicipital picondilitis lateral y medial ) ursitis e} Entesitis tricJpital

~ ~ ~
~
~ ~

16) Un paciente ~.e 56 aos, con anb~cedente de gota, inicia dolor Intenso en la rodilla derecha, asociado a aumento de volumen y eritema. Al examen ffsico se aprecia paciente febril hasta 38,4C, levemente hipertenso, con aumento de volumen, eritema, aumento del calor local y signos de derrame artic:ular, en relacin a la rodilla izquierda. La conducta ms adecuada es:
a) Iniciar AINEs, colchicina y alopurinol b) Iniciar Al NEs y colchicina, sin alopurinol e) Iniciar Al NEs y realizar infiltracin loc~al con corticoides __cijjficiar AINEs y solicitar estudio de Uquido sinovial -r- 6} Iniciar cefazolina y AINEs endovenosos y derivar a traumatologra, para aseo quirrgico inmediato

"'
~

f'

(!'

(A
~

~
~
~

17) Una ~ujer de 43 aos, presenta ;utralgias de manos, asociados a rigidez matinal de 1 hora, ~e varios aos de evolucin. Al examen fsico se observa deformacin en cuello de clsl)e ..y en botn de camisa de vario! dedos y desviacin hacia cubital de los dedos de bas manos, asociado a artritis de mufiecas y de algunas articulaciones etacarpofalngicas e intealnglcas proximales. El diagnstico ms probable es:

~
~
~

Artrosis avanzada b) Artritis lpica no erosiva e} Artritis gotosa tofcea #rtritis reumatordea deformante e) Esclerodermia

18) Una mujer de 24 aos, consulta por artralgias de manos y fiebre ocasional. El examen fisico no aporta mayor informacin }' los exmenes de laboratorio arrojan linfopenia y proteinuria. El diagnstico ms probable es:
a) Arteritis de la temporal
.~upus eritematoso sistmico

fA
~
~

~
~
~

e) Leucemia aguda d) Artritis viral e) Vasculitis de vaso pequeo

19) La dermatomiositis es una enferrnedad autoinmune, que ha sido asociada a la presencia de un cncer concomitante. Los marcadores inmunolgicos ms caracteristicos de esta patologra son:
a) Anticuerpos anti centrmero b) Anticuerpos anti nuclolo ~;~'fAnticuerpos anti Jo-1 d) Anticuerpos anti Ro y anti La e) Anticuerpos anti ribonucleoproterna

f!'
~ ~
~

~
~

...
J
',

r.'

..

,
.......... ~ .

~
~
~

20) .Un paciente de aos, consulta por dolor artc~lar . REtfiere dolor y aumento de : volumen recidivante y 11Jigratorio de grandes y pequeas artic~laciones. Al examen fsico se aprecia artritis de la rodilla y co~<> derechos, y de la rodilla, muecY tobillo izquierdo. Tambin se observa.dactilitis de 3 dedos y 2.ortejos, asoch1dos a p.its (picotazos) ung.ueales. ; El di~gnstico ms probable es:. ~) Artritis psoritica 'b) Espondilitis anquilosant~ e) Enfermedad de Still (Artritis juvenil idioptica) d) Artriti~ reumatoide e) Lupus.eritem.atoso sistmico

S5

~
~

~
~ ~

~
~

~
~
~

~
~

... .
,

~
~

~
~ ~
~

\
1

~
~

..J

i1
\

~
~
..~
:

~
~
~

.~

...

~ ~

~ ~

~
~

~
~

..,!

You might also like